You are on page 1of 528

DUYÊN HẢI 2023

TRƯỜNG THPT CHUYÊN KỲ THI CHỌN HỌC SINH GIỎI KHU VỰC DUYÊN HẢI
TUYÊN QUANG VÀ ĐỒNG BẰNG BẮC BỘ LẦN THỨ XIV, NĂM 2023

ĐỀ THI ĐỀ XUẤT
Môn: Hóa học – Lớp 11
Thời gian: 180 phút (Không kể thời gian giao đề)
Đề gồm 8 câu, trong 04 trang
Câu 1 (2,5 điểm) Tốc độ phản ứng.
1.1. Cơ chế phản ứng I3– + 2N3– → 3I– + 3N2(k) trong dung môi CS2 được đề nghị như sau:

Biết S2CN3– và (S2CN3)2 là tiểu phân trung gian rất hoạt động. Xác định phương trình tốc độ của
phản ứng và bậc của phản ứng (nếu có).
1.2. Phẩm màu xanh Brilliant Blue FCF (ký hiệu là E133)
được sử dụng nhiều trong công nghiệp thực phẩm. Trong
dung dịch nước, E133 bị oxi hóa bởi nước Javen theo
phản ứng: E133 + ClO– → Sản phẩm không màu
Động học của phản ứng này được nghiên cứu bằng cách
theo dõi biến thiên nồng độ E133 theo thời gian (bằng
phương pháp phân tích quang học). Kết quả cho thấy phản
ứng có bậc động học.
Thí nghiệm 1: Trộn 25,0 ml dung dịch E133 có nồng độ C1 = 4,545.10–6 M với 1,0 ml dung dịch
NaClO nồng độ C2 = 1,360.10–2 M. Theo dõi nồng độ E133 theo thời gian ở 298K như sau:
t (phút) 2,5 5,0 7,5 10,0
CE133 (.10–6 M) 2,222 1,129 0,575 0,292
a. Xác định bậc của phản ứng.
b. Tính hằng số tốc độ của phản ứng và thời gian bán phản ứng trong điều kiện thí nghiệm.
Thí nghiệm 2: trộn 25,0 ml dung dịch E133 có nồng độ C3 = 5,200.10–6 M với 1,0 ml dung dịch
NaClO có nồng độ C4 = 8,500.10–3 M. Theo dõi nồng độ E133 theo thời gian ở 298K như sau:
t (phút) 4,1 8,2
CE133 (.10–6 M) 2,50 1,25
c. Chỉ ra rằng trong điều kiện thí nghiệm 2, bậc của phản ứng không thay đổi so với thí nghiệm 1
và tính hằng số tốc độ của phản ứng trong điều kiện này.
d. Từ kết quả thu được ở hai thí nghiệm trên cho biết ngoài E133, tốc độ phản ứng còn phụ thuộc
vào nồng độ của chất nào khác? Xác định bậc riêng phần của chất đó. Tính hằng số tốc độ của
phản ứng nghiên cứu ở 298K nếu các chất phản ứng được lấy theo đúng hệ số tỉ lượng trên
phương trình hóa học.
Câu 2 (2,5 điểm) Cân bằng và phản ứng trong dung dịch. Pin điện – Điện phân.
Acqui Ni–Cd được mô tả bởi sơ đồ: (–) Cd | Cd(OH)2 | K+,OH– | Ni2O3 | Ni(OH)2 | Ni (+)
ở đây Ni chỉ đóng vai trò dẫn điện còn Cd tham gia vào phản ứng điện cực.
a. Chỉ ra anot, catot trong quá trình nạp điện cho acqui và trong quá trình acqui phóng điện. Hãy
viết phương trình hóa học của các phản ứng xảy ra tại các điện cực và phản ứng tổng quát trong
quá trình acqui phóng điện và trong quá trình nạp điện cho acqui.
b. Acqui Ni–Cd còn được coi là acqui kiềm, hãy giải thích tên gọi này.
c. Người ta nạp acqui trong 10h với hiệu điện thế U = 1,8V và cường độ dòng điện I = 0,5A.
i) Xác định lượng điện đã nạp cho acqui (Q1).
ii) Xác định khối lượng Ni2O3 và Cd đã chuyển hóa trong quá trình nạp.
iii) Tính năng lượng mà acqui đã nhận được (W1). Lượng năng lượng này được lưu trữ dưới
dạng điện năng hay hóa năng.
d. Người ta cho acqui phóng điện qua một điện trở R = 105 Ω, khi đó cường độ dòng điện đo
được là I = 12,0 mA; hiệu điện thế giữa hai điện cực của acqui là E = 1,30 V.
i) Xác định điện trở trong của acqui (r).
ii) Trong quá trình phóng điện, lượng Ni2O3 đã giảm 14,65 g. Tính lượng điện mà acqui đã
cung cấp (Q2) và năng lượng tương ứng (W2).
e. Tính hiệu suất điện: ρ1 = Q1/Q2 và hiệu suất năng lượng ρ2 = W1/W2 . Cho biết: Ni = 58,7
g/mol; Cd = 112,4 g/mol; O = 16 g/mol; H = 1 g/mol.
Câu 3 (2,5 điểm) Nhiệt động học và cân bằng hóa học.
PbCO3 và ZnO thường được sử dụng làm bột tạo màu trắng. H2S trong không khí có thể làm hư
hại các bột màu này do các phản ứng sau:
PbCO3 (r) + H2S (k) ⎯→ PbS (r) + CO2 (k) + H2O (h) (1)
ZnO (r) + H2S (k) ⎯→ ZnS (r) + H2O (h) (2)
a) Tính hằng số cân bằng của các phản ứng (1) và (2).
b) Cần khống chế nồng độ tối đa của H2S trong không khí bằng bao nhiêu g/m3 để các bột màu
nói trên không bị hư hại?
c) Trong 2 chất màu nói trên, chất nào ưu thế hơn khi môi trường có H2S, tại sao?
d) Bằng cách xử lí với dung dịch H2O2, có thể làm trắng lại các mảng bị đổi màu do sự hình
thành PbS. Viết phương trình của phản ứng xảy ra trong cách xử lí này.
e) Hãy chứng tỏ rằng, về mặt nhiệt động học, oxi của không khí có thể thay thế H2O2 trong
phương pháp xử lí trên. Để tính toán có thể sử dụng các dữ kiện và bảng sau: T= 298K; áp suất khí
quyển p = 1,000 atm; % thể tích của các khí và hơi trong không khí: N2 77,90; O2 20,70; CO2 0,026;
H2O (h) 0,40; các khí khác: 1,03.
PbCO3(r) H2S(k) PbS(r) ZnO(r) ZnS(r) CO2(k) H2O(h) PbSO4(r) H2O2(l)
ΔfG°298
– 626,0 – 33,0 – 92,6 – 318,0 – 184,8 – 394,2 – 228,5 – 811,5 120,4
kJ/mol
Màu trắng đen trắng trắng trắng
Câu 4 (2,5 điểm) Hóa nguyên tố (Kim loại, phi kim nhóm IVA, VA). Phức chất.
4.1. Kim loại đồng đã được sử dụng để tạo công cụ lao động từ 8000 năm trước công nguyên.
Đến ngày nay đồng và các hợp chất, đặc biệt phức chất của đồng đã được nghiên cứu ứng dụng
trong rất nhiều lĩnh vực. Trong các phức chất, ion Cu(II) thường tạo số phối trí 4 hoặc 6.
a. Sử dụng thuyết VB, mô tả các liên kết hình thành trong phức [CuCl4]2– và [Cu(CN)4]2–. Biết
[CuCl4]2– có cấu trúc tử diện, [Cu(CN)4]2– là phức vuông phẳng.
b. Dung dịch CuCl2 đặc có màu xanh lá cây, khi pha loãng, màu sắc chuyển dần sang xanh nước
biển. Hãy sử dụng thuyết trường tinh thể giải thích hiện tượng trên.
c. Phức chất [Cu(ImH)4(NO3)2] là phức chất bát diện của Cu2+ với 4
phối tử imidazole và 2 phối từ nitrato (NO3–) có cấu trúc như sau: Biết
độ dài liên kết Cu–NImidazole = 2,00 Å, Cu–ONO3 = 2,55 Å. Giải thích sự
khác nhau của độ dài các liên kết trên.
4.2. Photpho sunfua có thể được tạo thành bằng cách đun nóng photpho trắng (P4) với lưu
huỳnh (S8). Khi phản ứng này được thực hiện ở nhiệt độ thấp, một loạt các sản phẩm từ P4S3 tới
P4S10 được tạo thành.
a. Trên thực tế, P4S4 tồn tại hai dạng đồng phân khác nhau trong đó cấu trúc
của một đồng phân được biểu diễn như trên. Đồng phân thứ hai có tâm đối
xứng với các liên kết P–S và P–P. Đề nghị một cấu trúc cho đồng phân thứ
hai của P4S4.
b. Cho các phản ứng theo sơ đồ sau:
(1) P4(trắng) + Cl2 → X (2) X + Cl2(trong CCl4) → Y
(3) Y + BCl3(trong CCl4) → Z (4) Z + H2O(dư, nóng) → ........
Xác định công thức các chất X, Y, Z và hoàn thành các phản ứng trong sơ đồ đó. Vẽ cấu trúc
phân tử của phân tử Z, biết rằng Z là hợp chất ion.
Câu 5 (2,5 điểm) Đại cương hữu cơ.
5.1. Xác định nguyên tử Nitrogen (N) có tính base mạnh nhất trong các chất từ A1 đến A4 và
viết quá trình nhận một proton (H+) của các chất từ A1 đên A6.

A1 A2 A3 A4 A5 A6
5.2. Cho phản ứng sau:

Cho biết chất nào trong số các đồng phân sản phẩm A9 và A10 tạo thành, sản phẩm nào là sản
phảm chính trong các trường hợp sau:
a. Sử dụng MeI là nguồn Me+.
b. Sử dụng Me3O+ BF4– là nguồn Me+.
Câu 6 (2,5 điểm) Sơ đồ tổng hợp hữu cơ. Cơ chế phản ứng hóa hữu cơ.
6.1. Đề nghị cơ chế cho quá trình chuyển hóa sau:

6.2. Quy trình tổng hợp một loại ceratopicanol như sau:
Câu 7 (2,5 điểm) Xác định cấu trúc các chất hữu cơ
7.1. Valdecoxib là một loại thuốc chống viêm. Để tổng hợp Valdecoxib, người ta cho
benzaldehyde phản ứng với KCN trong dung môi EtOH/H2O thu được G1. Cho G1 phản ứng với
N2 H4 ở nhiệt độ cao, sau đó oxi hóa sản phẩm thì thu được G2. Phản ứng của G2 với
hydroxylamin tạo thành oxim G3. Xử lý G3 bằng 2 đương lượng BuLi, sau đó cho phản ứng với
anhydrit axetic thu được G4. Tuy nhiên G4 không bền và dễ chuyển thành hợp chất G5. Phổ 1H–
NMR của G5 cho biết có 1 proton linh động và trong phân tử có chứ liên kết dạng O–C–O. Đem
G5 phản ứng với acid chlorosulfonic đầu tiên thu được G6 (C16H13NO), sau đó G6 tiếp tục bị
sunfonyl hóa ở vị trí para thì thu được hai sản phẩm G7 và
G8, trong đó G7 là sản phẩn chính.Xử lí G7 vs NH3 trong
điều kiện thích hợp thu được Valdecoxib. Còn nếu cho
Valdecoxib phản ứng với Ac2O/Py sẽ thu được Parecoxib. Xác
định cấu tạo các chất từ G1 đến G8 và cấu tạo Valdecoxib.
7.2. Đề nghị sơ đồ tổng hợp Tipifarnib từ các chất hữu cơ nhỏ cho trước dưới đây và các chất
xúc tác cần thiết tự chọn, điều kiện phản ứng đầy đủ.

Câu 8 (2,5 điểm) Hóa học các hợp chất thiên nhiên
8.1. Đường D–Galactose là đồng phân cấu hình ở vị trí C4 của D–Glucose. Khử hóa D–
galactose thu được hợp chất không quang hoạt A, nếu khử D–Glucose thì sản phẩm thu được lại
quang hoạt B. D–Galactose tác dụng với PhN2H3 dư thu được sản phẩm C chứa 2 liên kết
hydrogen nội phân tử. Đóng vòng furan 5 cạnh D–Galactose với MeOH/H+ tạo thành D.
a. Vẽ cấu trúc Fisher của D–Galactose, A, B, C và dạng Haworth của D.
b. Vẽ cấu trúc dạng liên kết hydrogen nội phân tử của C.
c. Cho D–Galactose tác dụng với (CH2OH)2/H+ tạo thành E, sau đó E phản ứng tiếp với 2 phân
tử acetone thu được F. Chuyển nhóm OH trong F thành Br rồi thủy phân với NaOH thu được G.
Loại bỏ các nhóm bảo vệ trong G thu được D–Talose. Vẽ cấu trúc Fisher của D–Talose, E, F, G.
d. Chitin là loại polysacharide phổ biến thứ 2 trong tự nhiên. Cấu trúc polymer của Chitin có
monome là các D–Glucose liên kết lại với nhau bằng liên kết 2,6–glycosidic, sau đó thay nhóm
OH ở C2 bằng nhóm (AcNH–). Vẽ đoạn mạch cấu trúc chứa 3 monomer của Chitin.
8.2. Ba amino acid Arg và Gln và Glu có cấu trúc như hình dưới.

a. Xác định nguyên tử N có tính base mạnh nhất trong hai aminoacid Arginine (Arg), Glutamine
(Gln) và nhóm COOH có tính acid mạnh nhất trong Glutamic acid (Glu).
b. Vẽ dạng tồn tại chính của Glutamic acid (Glu) trong dung dịch đệm có pH = 9.
––––––––––––––––––––––––––
GV ra đề đề xuất: Phạm Quốc Long – THPT Chuyên Tuyên Quang
SĐT: 0382428843
TRƯỜNG THPT CHUYÊN KỲ THI CHỌN HỌC SINH GIỎI KHU VỰC DUYÊN HẢI
TUYÊN QUANG VÀ ĐỒNG BẰNG BẮC BỘ LẦN THỨ XIV, NĂM 2023

ĐÁP ÁN ĐỀ XUẤT
Môn: Hóa học – Lớp 11
Thời gian: 180 phút (Không kể thời gian giao đề)
Đề gồm 8 câu, trong 04 trang
Câu 1 (2,5 điểm) Tốc độ phản ứng.
1.1. Cơ chế phản ứng I3– + 2N3– → 3I– + 3N2(k) trong dung môi CS2 được đề nghị như sau:

Biết S2CN3– và (S2CN3)2 là tiểu phân trung gian rất hoạt động. Xác định phương trình tốc độ của
phản ứng và bậc của phản ứng (nếu có).
1.2. Phẩm màu xanh Brilliant Blue FCF (ký hiệu là E133)
được sử dụng nhiều trong công nghiệp thực phẩm. Trong
dung dịch nước, E133 bị oxi hóa bởi nước Javen theo
phản ứng: E133 + ClO– → Sản phẩm không màu
Động học của phản ứng này được nghiên cứu bằng cách
theo dõi biến thiên nồng độ E133 theo thời gian (bằng
phương pháp phân tích quang học). Kết quả cho thấy phản
ứng có bậc động học.
Thí nghiệm 1: Trộn 25,0 ml dung dịch E133 có nồng độ C1 = 4,545.10–6 M với 1,0 ml dung dịch
NaClO nồng độ C2 = 1,360.10–2 M. Theo dõi nồng độ E133 theo thời gian ở 298K như sau:
t (phút) 2,5 5,0 7,5 10,0
–6
CE133 (.10 M) 2,222 1,129 0,575 0,292
a. Xác định bậc của phản ứng.
b. Tính hằng số tốc độ của phản ứng và thời gian bán phản ứng trong điều kiện thí nghiệm.
Thí nghiệm 2: trộn 25,0 ml dung dịch E133 có nồng độ C3 = 5,200.10–6 M với 1,0 ml dung dịch
NaClO có nồng độ C4 = 8,500.10–3 M. Theo dõi nồng độ E133 theo thời gian ở 298K như sau:
t (phút) 4,1 8,2
CE133 (.10–6 M) 2,50 1,25
c. Chỉ ra rằng trong điều kiện thí nghiệm 2, bậc của phản ứng không thay đổi so với thí nghiệm 1
và tính hằng số tốc độ của phản ứng trong điều kiện này.
d. Từ kết quả thu được ở hai thí nghiệm trên cho biết ngoài E133, tốc độ phản ứng còn phụ thuộc
vào nồng độ của chất nào khác? Xác định bậc riêng phần của chất đó. Tính hằng số tốc độ của
phản ứng nghiên cứu ở 298K nếu các chất phản ứng được lấy theo đúng hệ số tỉ lượng trên
phương trình hóa học.
Câu Hướng dẫn chấm Điểm
– – –
1.1 Phản ứng I3 + 2N3 → 3I + 3N2(k)
𝑑[𝐼3−] 𝑑[𝑁3−] 𝑑[𝐼−] 𝑑[𝑁2]
v=− =− =+ =+
𝑑𝑡 2𝑑𝑡 3𝑑𝑡 3𝑑𝑡
𝑑[𝑁2] – 2
v= = k4.[(S2CN3)2].[N3 ] (1)
3𝑑𝑡
Biết S2CN3– và (S2CN3)2 là tiểu phân trung gian hoạt động. Vậy áp dụng nguyên 1,0
lí nồng độ ổn định cho hai tiểu phân trung gian này, ta được biểu thức: điểm
𝑑[(S2CN3)2]
= k2.[S2CN3–]2.[I3–] – k4.[(S2CN3)2].[N3–]2 = 0
𝑑𝑡
=> [S2CN3–]2 = k4.[(S2CN3)2].[N3–]2/( k2.[I3–])
𝑑[S2CN3−]
= k1.[N3–].[CS2] – (k2+k3).[S2CN3–]2.[I3–] + 2.k4.[(S2CN3)2].[N3–]2 = 0
𝑑𝑡
=> k1.[N3–].[CS2] – k3.[S2CN3–]2.[I3–] + k4.[(S2CN3)2].[N3–]2 = 0
=> k1.[CS2] – k3.k4.[(S2CN3)2].[N3–]/k2 + k4.[(S2CN3)2].[N3–] = 0
𝑘1.𝑘2
=> [(S2CN3)2] = .[CS2].[N3–]–1 thay vào (1) ta được:
𝑘4(𝑘3−𝑘2)
𝑑[𝑁2] 𝑘1.𝑘2
v= = k4.[(S2CN3)2].[N3–]2 = .[CS2].[N3–]
3𝑑𝑡 𝑘3−𝑘2
𝑘1.𝑘2
CS2 là dung môi vì vậy [CS2] không đổi, đặt kpư = .[CS2]
𝑘3−𝑘2

Phương trình tốc độ của phản ứng là: v = kpư.[N3 ]
Bậc của phản ứng là bậc 1.
1.2 a. Tính lại nồng độ tại t = 0 phút.
CE133 (t=0min) = 4,37.10–6 M, CClO– (t=0min) = 5,23.10–4 M
Biểu thức tốc độ: v = k.[ClO–]x.[E133]y, ta thấy CE133 < CClO–
Đặt k’ = k.[ClO–]x, lúc này biểu thức tốc độ là: v = k’.[E133]y
Giả sử rằng y = 1, ta có bảng giá trị k như sau đối với phản ứng của E133 theo
bậc 1:
t (phút) 2,5 5,0 7,5 10,0
k’ (min–1) 0,2705 0,2707 0,2704 0,2706
Thấy rằng giá trị k’1 ~ k’2 ~ k’3 ~ k’4 => vậy giả sử đúng, y = 1, bậc riêng phần 0,5đ
của E133 là bậc 1. 0,25đ
–1
b. Ta có: k’ = 0,2706 min và t1/2 = 2,56 min
c. Tính lại nồng độ tại t = 0 phút.
C’E133 (t=0min) = 5,00.10–6 M, C’ClO– (t=0min) = 3,27.10–4 M
Biểu thức tốc độ: v = k.[ClO–]x.[E133]y, ta thấy C’E133 < C’ClO–
Đặt k’’ = k.[ClO–]xtn2, lúc này biểu thức tốc độ là: v = k’’.[E133]ytn2
Giả sử rằng y = 1, ta có bảng giá trị k như sau đối với phản ứng của E133 theo
bậc 1:
t (phút) 4,1 8,2
–1
k’’ (min ) 0,1691 0,1691 0,5đ
Thấy rằng giá trị k’’1 ~ k’’2 => vậy giả sử đúng, y = 1, bậc riêng phần của E133
là bậc 1 và không đổi trong thí nghiệm 2.
d. Tốc độ phản ứng còn phụ thuộc vào nồng độ của ClO–.
Với k’ = k.[ClO–]x = 0,2706 min–1 và k’’ = k.[ClO–]xtn2 = 0,1691 min–1
=> k’/k’’ = ([ClO–]/[ClO–]tn2)x = (5,23/3,27)x = 0,2706/0,1691 => x ~ 1 0,25đ

Bậc riêng phần của ClO là bậc 1.
=> k = 517,26 min–1
Câu 2 (2,5 điểm) Cân bằng và phản ứng trong dung dịch. Pin điện – Điện phân.
Acqui Ni–Cd được mô tả bởi sơ đồ: (–) Cd | Cd(OH)2 | K+,OH– | Ni2O3 | Ni(OH)2 | Ni (+)
ở đây Ni chỉ đóng vai trò dẫn điện còn Cd tham gia vào phản ứng điện cực.
a. Chỉ ra anot, catot trong quá trình nạp điện cho acqui và trong quá trình acqui phóng điện. Hãy
viết phương trình hóa học của các phản ứng xảy ra tại các điện cực và phản ứng tổng quát trong
quá trình acqui phóng điện và trong quá trình nạp điện cho acqui.
b. Acqui Ni–Cd còn được coi là acqui kiềm, hãy giải thích tên gọi này.
c. Người ta nạp acqui trong 10h với hiệu điện thế U = 1,8V và cường độ dòng điện I = 0,5A.
i) Xác định lượng điện đã nạp cho acqui (Q1).
ii) Xác định khối lượng Ni2O3 và Cd đã chuyển hóa trong quá trình nạp.
iii) Tính năng lượng mà acqui đã nhận được (W1). Lượng năng lượng này được lưu trữ dưới
dạng điện năng hay hóa năng.
d. Người ta cho acqui phóng điện qua một điện trở R = 105 Ω, khi đó cường độ dòng điện đo
được là I = 12,0 mA; hiệu điện thế giữa hai điện cực của acqui là E = 1,30 V.
i) Xác định điện trở trong của acqui (r).
ii) Trong quá trình phóng điện, lượng Ni2O3 đã giảm 14,65 g. Tính lượng điện mà acqui đã
cung cấp (Q2) và năng lượng tương ứng (W2).
e. Tính hiệu suất điện: ρ1 = Q1/Q2 và hiệu suất năng lượng ρ2 = W1/W2 . Cho biết: Ni = 58,7
g/mol; Cd = 112,4 g/mol; O = 16 g/mol; H = 1 g/mol.
Câu Hướng dẫn chấm Điểm
2 a. Khi phóng điện: Anot: Cd/Cd(OH)2, KOH
Phản ứng điện cực: Cd + 2OH– → Cd(OH)2 + 2e
Catot: Ni2O3/Ni(OH)2,KOH
Phản ứng điện cực: Ni2O3 + 3H2O + 2e → 2Ni(OH)2 + 2OH–
Phản ứng tổng quát khi phóng điện: Cd + Ni2O3 + 3H2O → 2Ni(OH)2 + Cd(OH)2
(0,25điểm)
Catot: Cd/Cd(OH)2, KOH
Phản ứng điện cực: Cd(OH)2 + 2e → Cd + 2OH–
Anot: Ni2O3/Ni(OH)2,KOH
Phản ứng điện cực: 2Ni(OH)2 + 2OH– → Ni2O3 + 3H2O + 2e
Phản ứng tổng quát khi nạp điện: 2Ni(OH)2 + Cd(OH)2 → Cd + Ni2O3 + 3H2O
(0,25điểm)
b. Acqui Ni–Cd còn được coi là acqui kiềm vì loại này sử dụng chất điện phân
kiềm là KOH hoặc NaOH thay vì chất điện phân là NH4Cl có tính axit hoặc ZnCl2.
(0,25điểm) 2,5
c.i) Lượng điện đã nạp cho acqui (Q1) điểm
Q1 = It = 0,5.10.3600 = 18000 (C). (0,25điểm)
ii) Ta có: ne = It/F = 18000/96485 = 0,1866 mol
nNi2O3 = nCd = 0,187/2 = 0,0933 mol => mNi2O3 = 15,428 gam
và mCd = 10,485 gam. (0,25điểm)
iii) Lượng năng lượng này được lưu trữ dưới dạng hóa năng.
W1 = UIt = 1,8.0,5.10.3600 = 32400 (J) (0,25điểm)
d.i) Điện trở trong của acqui (r):
Epin = Imạch.(R + r) => r = Epin/ Imạch – R = 1,30/0.012 – 105 = 3,33 (Ω)
(0,25điểm)
ii) nNi2O3 = 0,0886 mol
Q2 = nNi2O3.n.F = 0,0886.2.96485 = 17097 (C)
W2 = UIt = EIt = 1,3.17097 = 22226 (J) (0,25điểm)
e. Ta có: Hiệu suất điện: ρ1 = Q1/Q2 = 1,053 (0,25điểm)
Hiệu suất năng lượng ρ2 = W1/W2 = 1,458 (0,25điểm)
Câu 3 (2,5 điểm) PbCO3 và ZnO thường được sử dụng làm bột tạo màu trắng. H2S trong không
khí có thể làm hư hại các bột màu này do các phản ứng sau:
PbCO3 (r) + H2S (k) ⎯→ PbS (r) + CO2 (k) + H2O (h) (1)
ZnO (r) + H2S (k) ⎯→ ZnS (r) + H2O (h) (2)
a) Tính hằng số cân bằng của các phản ứng (1) và (2).
b) Cần khống chế nồng độ tối đa của H2S trong không khí bằng bao nhiêu g/m3 để các bột màu
nói trên không bị hư hại?
c) Trong 2 chất màu nói trên, chất nào ưu thế hơn khi môi trường có H2S, tại sao?
d) Bằng cách xử lí với dung dịch H2O2, có thể làm trắng lại các mảng bị đổi màu do sự hình
thành PbS. Viết phương trình của phản ứng xảy ra trong cách xử lí này.
e) Hãy chứng tỏ rằng, về mặt nhiệt động học, oxi của không khí có thể thay thế H2O2 trong
phương pháp xử lí trên. Để tính toán có thể sử dụng các dữ kiện và bảng sau: T= 298K; áp suất khí
quyển p = 1,000 atm; % thể tích của các khí và hơi trong không khí: N2 77,90; O2 20,70; CO2 0,026;
H2O (h) 0,40; các khí khác: 1,03.
PbCO3(r) H2S(k) PbS(r) ZnO(r) ZnS(r) CO2(k) H2O(h) PbSO4(r) H2O2(l)
ΔfG°298
– 626,0 – 33,0 – 92,6 – 318,0 – 184,8 – 394,2 – 228,5 – 811,5 120,4
kJ/mol
Màu trắng đen trắng trắng trắng
Câu Hướng dẫn chấm Điểm
3 a. Đối với phản ứng (1)
ΔG°(1) = (–92.6 – 394.2 – 228.5 + 626.0 + 33.0) kJ/mol = –56,3 kJ/mol
K(1) = e– ΔG°(1)/RT = e56300/8,314.298 = 7,4.109. 0,5
– Đối với phản ứng (2) điểm
ΔG°(2)=(–184.8 –228.5 + 318.0 + 33.0) kJ/mol = – 62,3 kJ/mol
K(2) = e– ΔG°(2)/RT = e62300/8,314.298 = 8,3.1010
b. Đối với phản ứng (1)
2.6  10−4  4 10−3
ΔG(1)= –RTlnK(1)+ RT.ln
pH 2 S
Điều kiện để (1) ưu thế theo chiều thuận:
2.6  10−4  4 10−3
ΔG(1) =–RTlnK(1) + RT.ln <0 (a)
pH 2 S
2.6  10−4  4 10−3
→ pH2S > 9
= 1,4.10–16 bar (b)
7, 4.10
Để bảo vệ được mầu trắng PbCO3 thì nồng độ H2S được phép trong không khí tối
đa là:
34.(1,4.10–16.1000 L)/(0,082 L.bar.mol–1.K–1.298K) = 1,9.10–13 g/m3 0,75
– Đối với phản ứng (2) điểm
4 10−3
ΔG(2) = – RTlnK(2) + RT.ln
pH 2 S
Điều kiện để (2) ưu thế theo chiều thuận:
4 10−3
ΔG(2) = – RTlnK(2) + RT.ln <0 (c)
pH 2 S
4 10−3
→ pH2S > = 4,8.10–14 bar
8,3.1010
Để bảo vệ được mầu trắng ZnO thì nồng độ H2S được phép trong không khí tối đa
là:
34.(4,8.10–14.1000 L)/(0,082 L.bar.mol–1.K–1.298K) = 6,7.10–11 g/m3
c. ZnO ưu thế hơn vì:
– Phản ứng (1) tự diễn biến ở những nồng độ H2S nhỏ hơn. 0,5
– Sản phẩm của (1) là PbS có mầu đen còn sản phẩm của (2) là ZnS vẫn còn là điểm
mầu trắng.
PbS + 4H2O2 ⎯→ PbSO4 + 4H2O (3) 0,25
điểm
PbS + 2 O2 ⎯→ PbSO4 (4)
ΔG° = –811.5 kJ/mol + 92.6 kJ/mol = – 718.9 kJ/mol
1 0,5
ΔG = – 718.9 kJ/mol + RT.ln 2
= – 711,1 kJ/mol
0.207 điểm
Phản ứng (4) có thể tự diển ra trong không khí ở nhiệt độ 298 K. Oxi của không
khí có thể tái tạo màu trắng bằng cách oxi hóa PbS ⎯→ PbSO4.
Câu 4 (2,5 điểm)
4.1. Kim loại đồng đã được sử dụng để tạo công cụ lao động từ 8000 năm trước công nguyên.
Đến ngày nay đồng và các hợp chất, đặc biệt phức chất của đồng đã được nghiên cứu ứng dụng
trong rất nhiều lĩnh vực. Trong các phức chất, ion Cu(II) thường tạo số phối trí 4 hoặc 6.
a. Sử dụng thuyết VB, mô tả các liên kết hình thành trong phức [CuCl4]2– và [Cu(CN)4]2–. Biết
[CuCl4]2– có cấu trúc tử diện, [Cu(CN)4]2– là phức vuông phẳng.
b. Dung dịch CuCl2 đặc có màu xanh lá cây, khi pha loãng, màu sắc chuyển dần sang xanh nước
biển. Hãy sử dụng thuyết trường tinh thể giải thích hiện tượng trên.
c. Phức chất [Cu(ImH)4(NO3)2] là phức chất bát diện của Cu2+ với 4
phối tử imidazole và 2 phối từ nitrato (NO3–) có cấu trúc như sau: Biết
độ dài liên kết Cu–NImidazole = 2,00 Å, Cu–ONO3 = 2,55 Å. Giải thích sự
khác nhau của độ dài các liên kết trên.
4.2. Photpho sunfua có thể được tạo thành bằng cách đun nóng photpho trắng (P4) với lưu
huỳnh (S8). Khi phản ứng này được thực hiện ở nhiệt độ thấp, một loạt các sản phẩm từ P4S3 tới
P4S10 được tạo thành.
a. Trên thực tế, P4S4 tồn tại hai dạng đồng phân khác nhau trong đó cấu trúc
của một đồng phân được biểu diễn như trên. Đồng phân thứ hai có tâm đối
xứng với các liên kết P–S và P–P. Đề nghị một cấu trúc cho đồng phân thứ
hai của P4S4.
b. Cho các phản ứng theo sơ đồ sau:
(1) P4(trắng) + Cl2 → X (2) X + Cl2(trong CCl4) → Y
(3) Y + BCl3(trong CCl4) → Z (4) Z + H2O(dư, nóng) → ........
Xác định công thức các chất X, Y, Z và hoàn thành các phản ứng trong sơ đồ đó. Vẽ cấu trúc
phân tử của phân tử Z, biết rằng Z là hợp chất ion.
Câu Hướng dẫn chấm Điểm
10 1
4.1 a. 29Cu: [Ar] 3d 4s
Cu2+: [Ar] 3d9
+ Với phức [CuCl4]2–: Ion Cl– là phối tử trường yếu, không có sự dồn e độc thân
trong trường hợp này, Cu2+ lai hóa sp3, phức có dạng hình học tứ diện, thuận từ.

(0,25điểm)

+ Với phức [Cu(CN)4] : Ion CN là phối tử trường mạnh, không có sự dồn e độc
2–

thân trong trường hợp này, phức có dạng hình học vuông phẳng do có xảy ra hiện
tượng kéo dài trục z (hiện tượng Jan–Teller), suy biến cấu trúc thành dạng vuông
phẳng. Lúc này, Cu2+ lai hóa sp2d, phức thuận từ.

1,5
điểm

Vuông phẳng
(0,25điểm)
b. + Khi ở dạng dung dịch đặc, nồng độ Cl– lớn, phức chủ yếu tạo thành ở dạng
phức clorua là chính, khi pha loãng, nồng độ H2O vượt trội, phức aquơ lúc này là
phức chính.
[CuCl4]2– + xH2O [Cu(H2O)4–xClx](2–x) + (4–x)Cl–
+ Bước sóng của màu xanh lá cây dài hơn của màu xanh dương. Cl– là phối tử
trường yếu hơn so với H2O, trường tách của phối tử H2O lớn hơn Cl–, tức là ∆EH2O
> ∆ECl– => λH2O < λCl–. Vì vậy khi pha loãng dung dịch muối CuCl2 đặc sẽ chuyển
dần từ màu xanh lá cây sang màu xanh dương. (0,5điểm)
c. d(Cu–NImidazole) = 2,00 Å < d(Cu–ONO3) = 2,55 Å do những nguyên nhân sau:
+ r(NImidazole) < r(ONO3)
+ Có hiện tượng tạo thêm liên kết πp(hệ thơm)→4d của Cu. (0,25điểm)
4.2 a. Cấu trúc của P4S4 đồng phân. 0,25 điểm

b. Mỗi chất đúng được 0,125 điểm. Mỗi phương trình đúng được 0,125 điểm.
X là PCl3. Y là PCl5. Z là [PCl4]+[BCl4]–
o
1) P4 (trắng) + 6Cl2 ⎯⎯⎯⎯ t thöôø
ng
→ 4PCl3
1,25
(2) PCl3 + Cl2(dư) ⎯⎯⎯ CCl 4
→ PCl5 điểm
(3) PCl5 + BCl3 ⎯⎯⎯ CCl 4
→ [PCl4]+[BCl4]–
to
(4) [PCl4]+[BCl4]– + 7H2O ⎯⎯ → H3PO4 + H3BO3 + 8HCl
Cấu trúc của Z: 0,125 điểm

Câu 5 (2,5 điểm)


5.1. Xác định nguyên tử Nitrogen (N) có tính base mạnh nhất trong các chất từ A1 đến A4 và
viết quá trình nhận một proton (H+) của các chất từ A1 đên A6.

A1 A2 A3 A4 A5 A6
5.2. Cho phản ứng sau:

Cho biết chất nào trong số các đồng phân sản phẩm A9 và A10 tạo thành, sản phẩm nào là sản
phảm chính trong các trường hợp sau:
a. Sử dụng MeI là nguồn Me+.
b. Sử dụng Me3O+ BF4– là nguồn Me+.
Câu Hướng dẫn chấm Điểm
5.1 Mỗi chất viết đúng được 0,25 điểm.

1,5
điểm
Qúa trình nhận proton (H+) của A5 và A6:

(0,25 điểm)

(0,25 điểm)
5.2 a. Sử dụng MeI là nguồn Me . Sản phẩm chính là A9: (0,25 điểm)
+

Do trong trường hợp methyl iodide, điện tích dương trên nguyên tử carbon là rất
nhỏ (các giá trị độ âm điện trên carbon, hydrogen và iodide là rất gần nhau), chúng
ta có thể kết luận rằng nó là một tác nhân electrophile “mềm”.
Vì vậy, nó sẽ phản ứng với nguyên tử carbon của enolate ion, tạo thành sản phẩm 1,0
kiểu A9. (0,25 điểm) điểm

b. Sử dụng Me3O BF4 là nguồn Me . Sản phẩm chính là A10: (0,25 điểm)
+ +

Trong trường hợp trimethyloxonium ion thì nguyên tử carbon dương điện đáng kể.
Đây là electrophile “cứng”. Nó phản ứng với oxygen tạo thành sản phẩm kiểu A10.
(0,25 điểm)
Câu 6 (2,5 điểm)
6.1. Đề nghị cơ chế cho quá trình chuyển hóa sau:

6.2. Quy trình tổng hợp một loại ceratopicanol như sau:

Vẽ (không cần giải thích) công thức cấu tạo của các chất từ E1 đến E8 trong sơ đồ.
Câu Hướng dẫn chấm Điểm
6.1

0,5
điểm

6.2 Xác định đúng mỗi chất được 0,125 điểm. 2,0
điểm
Câu 7 (2,5 điểm)
7.1. Valdecoxib là một loại thuốc chống viêm. Để tổng hợp Valdecoxib, người ta cho
benzaldehyde phản ứng với KCN trong dung môi EtOH/H2O thu được G1. Cho G1 phản ứng với
N2 H4 ở nhiệt độ cao, sau đó oxi hóa sản phẩm thì thu được G2. Phản ứng của G2 với
hydroxylamin tạo thành oxim G3. Xử lý G3 bằng 2 đương lượng BuLi, sau đó cho phản ứng với
anhydrit axetic thu được G4. Tuy nhiên G4 không bền và dễ chuyển thành hợp chất G5. Phổ 1H–
NMR của G5 cho biết có 1 proton linh động và trong phân tử có chứ liên kết dạng O–C–O. Đem
G5 phản ứng với acid chlorosulfonic đầu tiên thu được G6 (C16H13NO), sau đó G6 tiếp tục bị
sunfonyl hóa ở vị trí para thì thu được hai sản phẩm G7 và
G8, trong đó G7 là sản phẩn chính.Xử lí G7 vs NH3 trong
điều kiện thích hợp thu được Valdecoxib. Còn nếu cho
Valdecoxib phản ứng với Ac2O/Py sẽ thu được Parecoxib. Xác
định cấu tạo các chất từ G1 đến G8 và cấu tạo Valdecoxib.
7.2. Đề nghị sơ đồ tổng hợp Tipifarnib từ các chất hữu cơ nhỏ cho trước dưới đây và các chất
xúc tác cần thiết tự chọn, điều kiện phản ứng đầy đủ.

Câu Hướng dẫn chấm Điểm


7.1 Vẽ đúng mỗi sản phẩm được 0,25 điểm.

2,0
điểm
7.2

0,5
điểm

Câu 8 (2,5 điểm)


8.1. Đường D–Galactose là đồng phân cấu hình ở vị trí C4 của D–Glucose. Khử hóa D–
galactose thu được hợp chất không quang hoạt A, nếu khử D–Glucose thì sản phẩm thu được lại
quang hoạt B. D–Galactose tác dụng với PhN2H3 dư thu được sản phẩm C chứa 2 liên kết
hydrogen nội phân tử. Đóng vòng furan 5 cạnh D–Galactose với MeOH/H+ tạo thành D.
a. Vẽ cấu trúc Fisher của D–Galactose, A, B, C và dạng Haworth của D.
b. Vẽ cấu trúc dạng liên kết hydrogen nội phân tử của C.
c. Cho D–Galactose tác dụng với (CH2OH)2/H+ tạo thành E, sau đó E phản ứng tiếp với 2 phân
tử acetone thu được F. Chuyển nhóm OH trong F thành Br rồi thủy phân với NaOH thu được G.
Loại bỏ các nhóm bảo vệ trong G thu được D–Talose. Vẽ cấu trúc Fisher của D–Talose, E, F, G.
d. Chitin là loại polysacharide phổ biến thứ 2 trong tự nhiên. Cấu trúc polymer của Chitin có
monome là các D–Glucose liên kết lại với nhau bằng liên kết 2,6–glycosidic, sau đó thay nhóm
OH ở C2 bằng nhóm (AcNH–). Vẽ đoạn mạch cấu trúc chứa 3 monomer của Chitin.
8.2. Hai amino acid Arg và Gln và Glu có cấu trúc như hình dưới.

a. Xác định nguyên tử N có tính base mạnh nhất trong hai aminoacid Arginine (Arg), Glutamine
(Gln) và nhóm COOH có tính acid mạnh nhất trong Glutamic acid (Glu).
b. Vẽ dạng tồn tại chính của Glutamic acid (Glu) trong dung dịch đệm có pH = 9.
Câu Hướng dẫn chấm Điểm
8.1 a. Xác định đúng mỗi chất được 0,125 điểm.

1,5
điểm

b. 0,125 điểm.
c. Xác định đúng mỗi chất được 0,125 điểm.

d. Cấu trúc của Chitin 0,25


điểm.
8.2

a.
Ở Arg thì N lai hóa sp2 có tính base mạnh nhất cho có hai nhóm NH2 và NHR liên
hợp vào làm tăng mật độ electron trên nguyên tử N. (0,25 điểm)

Ở Gln thì N ở gần nhóm COOH có tính base mạnh hơn do cặp electron trên
nguyên tử chỉ chịu ảnh hưởng của các hiệu ứng +I và –I từ các nguyên tử xung
quanh trong khi nhóm NH2 amide chịu ảnh hưởng liên hợp vào nhóm CO làm 1,0
giảm tính base. (0,25 điểm) điểm

Nhóm COOH gần nhóm NH2 chịu ảnh hưởng –I của nhóm NH2 nên tính acid tăng
trong khi nhóm COOH cuối mạch nhận hiệu ứng +I từ gốc RH no nên tính acid
giảm. (0,25 điểm)
b. Ở pH = 9, chủ yếu là pH phân li của nhóm amoni, do đó, dạng tồn tại chính sẽ là
cả dạng acid và base liên hợp của nhóm amoni. (0,25 điểm)

––––––––––––––––Hết–––––––––––––––
HƯỚNG DẪN CHẤM

Hướng dẫn Điểm


Câu Cho phản ứng: 2N2O5(k) → 4NO2(k) + O2(k)
1 a. Biểu thức của định luật tốc độ phản ứng cho phản ứng trên và tính hằng số tốc độ phản
ứng: v = k.[N2O5]x
Dựa vào số liệu cho suy ra x = 1 hay v = k.[N2O5]
0,5đ
Tính k của các thí nghiệm suy ra k trung bình k = 2,28.10-3 (phút-1)
b. Thời gian cần để nồng độ N2O5 giảm từ 0,150M xuống còn 0,050M.
Áp dụng biểu thức của động học bậc nhất:
[N 2O5 ]0 0,150
kt = ln = ln = 2, 28.10−3.t t = 481 phút 0,5đ
[N 2O5 ] 0, 050
c. Tốc độ đầu của phản ứng khi nồng độ N2O5 bằng 0,150M là 2,37.10-3, mol.l-1.phút-1 tại
400C. Năng lượng hoạt hoá của phản ứng.
Tại 400C có k2 = 2,37.10-3 : 0,150 = 1,58.10-2 (phút-1)
Áp dụng phương trình Arrhenus:
k  E 1 1  1,58.10−2  E  1 1 
ln  2  = a  -  . Thay các số liệu: ln  −3 
= a  - 
 k1  R  T1 T2   2, 28.10  8,314  298 313 
Ea = 1,00.105 (J/mol) 0,5đ
d[N 2 O5 ]
d. Thiết lập biểu thức của tốc độ phản ứng .
dt
k
N2O5 ⎯⎯
1
→ NO2 + NO3
k'
NO2 + NO3 ⎯⎯
1
→ N2O5
k
NO2 + NO3 ⎯⎯→
2
NO2 + NO + O2
k
NO + N2O5 ⎯⎯
3
→ 3NO2
Áp dụng nguyên lí nồng độ ổn định đối với NO3 và NO:
d[NO 3 ]
= k1.[N2O5] - k1' .[NO2].[NO3] – k2.[NO2].[NO3] = 0 (1)
dt
d[NO]
= k2.[NO2].[NO3] – k3.[NO].[N2O5] = 0 (2)
dt
d[N 2 O5 ] 0,5đ
= - (k1.[N2O5] + k3.[NO].[N2O5] ) + k1' .[NO2].[NO3]
dt
Từ (1) và (2) suy ra: k1.[N2O5] = ( k1' + k2).[NO2].[NO3]
k3.[NO].[N2O5] = k2.[NO2].[NO3]
k2 k
= 3 [NO]
k + k2 k1
'
1

k1k2 k3 0,5đ
[NO] = [NO2].[NO3] = .[NO].[N2O5]
k3 (k1' + k2 ) k2
d[N 2 O5 ] k
= - k1.[N2O5] - k3.[NO].[N2O5] + k1' . 3 .[NO].[N2O5]
dt k2
k2 k1' 0,5đ
= k1.[N2O5].( -1 - + )
k1' + k 2 k1' + k2
Câu ⎯⎯
→ AgOH + H + (1) K1 = 10−11,7
Ag + + H 2O ⎯

2
2.1 ⎯⎯
→ PbOH + + H + (2) K 2 = 10−7,8
Pb2+ + H 2O ⎯

Do K2 >>K1 nên cân bằng (2) quyết định pH của hệ
⎯⎯
→ PbOH + + H + K 2 = 10−7,8
Pb 2+ + H 2O ⎯

0,1
0,1 − x x x
x2
 = 10−7,8  x = 10−4,4  pH = 4, 4 0,25đ
0,1 − x
Câu Khi thêm KI và HNO3 vào dung dịch A:
2 CAg+ = 0, 025M;CPb2+ = 0, 05M;CI− = 0,125M;CH+ = 0,1M
2.2
a Ag + + I − → AgI 
0, 025 0,125
− 0,1
Pb 2+ + 2I − → PbI 2 
0, 05 0,1
− − 0,25đ
Có đồng thời hai kết tủa AgI và PbI2
⎯⎯
→ Ag + + I− K s1 = 10−16
AgI ⎯
⎯ (3)
⎯⎯
→ Pb2+ + 2I − K s2 = 10−7,86
PbI2 ⎯
⎯ (4)
K s1  Ks2 ,  cân bằng (4) là chủ yếu.
Sự tạo phức hidroxo của Pb2+ là không đáng kể vì có H+ dư.
⎯⎯
→ PbOH + + H + K 2 = 10−7,8
Pb2+ + H2O ⎯

[PbOH+ ] 10−7,8
 2+
= −1 = 10−6,8  [PbOH + ]  [Pb2+ ] 0,25đ
[Pb ] 10
⎯⎯
→ Pb2+ + 2I − K s2 = 10−7,86
PbI2 ⎯

x 2x
(2x) 2 .x = 10−7,86  x = 1,51.10−3 M  [I − ] = 2x = 2,302.10−3 M
Ks1 10−16
[Ag + ] = −
= −3
= 3,31.10−14 M 0,25đ
[I ] 3,02.10
Thế của điện cực Ag khi nhúng vào dung dịch X
E1 = E 0Ag+ /Ag + 0, 0592 log[Ag + ] = 0, 799 + 0, 0592 log(3,31.10−14 ) = 0, 001V
Dung dịch Y:
⎯⎯
→ AgSCN  1012
Ag + + SCN − ⎯

0, 01 0, 04
− 0, 03
⎯⎯
→ Ag + + SCN −
AgSCN ⎯
⎯ 10−12
0, 03
x 0, 03 + x
x(x + 0,03) = 10−12  [Ag + ] = x = 3,33.10−11 M 0,25đ
Thế của điện cực Ag nhúng trong dung dịch Y:
E 2 = 0, 799 + 0, 0592 log[Ag + ]=0,799+0,0592log(3,33.10−11 ) = 0,179 V 0,25đ
Vì E2 > E1  Sơ đồ pin: (−)Ag, AgI I − SCN − AgSCN, Ag( +) 0,25đ
Các phản ứng:
b Tại catot: AgSCN + e ⎯⎯
→ Ag + SCN-
Tại anot: Ag + I- ⎯⎯
→ AgI + e 0,25đ
Phản ứng tổng quát: AgSCN + I ⎯⎯
→ Ag + AgI + SCN
- -

Sức điện động của pin ở 25oC:


c E = E2 –E1 = 0,179 – 0,001= 0,178V 0,25đ
Hằng số cân bằng của phản ứng tổng quát trong pin:
K s(AgSCN) 10−12
K= = −16 = 104
K s(AgI) 10 0,25đ
Thêm FeCl3 vào dung dịch X sẽ xảy ra phản ứng: Fe3+ + SCN − → FeSCN 2+
d  Nồng độ ion SCN- giảm, do đó, nồng độ ion Ag+ tăng, E2 tăng dẫn đến E tăng. 0,25đ
Câu Khí N2, là khí lí tưởng  CV = 5/2R; CP = 7/2R;  = 7/5 = 1,4.
3.1 nRT1 2,5.0, 082.250
V1 = =  34,167 L 0,25đ
P1 1, 5
Quá trình đoạn nhiệt bất thuận nghịch  Q = 0
A = U = nCV ( T2 – T1 ) = 2,5.5 / 2.8.314.(300 − 250) = 2598,125 J
H = nCP ( T2 – T1 ) = 2,5.7 / 2.8.314.(300 − 250) = 3637,375 J 0,25đ
Mặt khác: A = −P2 (V2 − V1 ) = −nRT2 + P2 V1
A + nRT2 2598,125 + 2,5.8,314.300
 P2 = = = 258542, 6 N / m 2  2,552 atm
V1 34,167.10−3
0,25đ
nRT2 2,5.0, 082.300
 V2 = =  24,1 L
P2 2,552
V2 T 24,1 5 300
S = nR ln + nCV ln 2 = 2,5.8,314.ln + 2,5. .8,314.ln  2, 219 J / K 0,25đ
V1 T1 34,167 2 250
Câu 0,54
Số mol glucozơ: n = = 0, 003 mol
3.2 180
a
2, 20.1
Số mol O2 ban đầu: n O2, bd =  0, 09 mol
0, 082.298
0,25đ
C6H12O6(r) + 6O2(k) ⎯⎯
→ 6CO2(k) + 6H2O(l) (1)
 Số mol O2 phản ứng: n O2, pu = 6.0, 003 = 0, 018 mol

 Số mol các chất sau phản ứng cháy:


n CO2 = 6.0, 003 = 0, 018 mol ; n H2O = 6.0, 003 = 0, 018 mol
n O2 = 0, 09 − 0, 018 = 0, 072 mol
Vì nhiệt lượng kế đẳng tích và cách nhiệt tuyệt đối nên ta có sơ đồ:
b Q=0
C6 H12O6 (r) + 6O 2 (k) ⎯⎯⎯⎯→ 6CO 2 (k) + 6H 2O(l) 299, 4K
+  O2 (k)du , NLK  + O2 (k) du , NLK  0,25đ
Q1 Q2
6CO2 (k) + 6H 2O(l) 298K
+ O2 (k)du , NLK 

 Q = Q1 + Q 2 = 0

(
 Q1 = − n CO2 .Cv,CO2 + n H2O(l) .CV,H2O(l) + n O2,du .CV,O2 (k) + Cnlk .(299, 4 − 298) )
Vì các khí là khí lí tưởng nên:
Q1 = − 0,018.(36, 4 − 8,314) + 0,018.74,5 + 0,072.(29, 4 − 8,314) + 5996,7.1, 4
0,25đ
Q1  −8400, 091 J
Q1 −8400, 091
 Udc,298 = = = − 2800030,33J / mol  −2800, 0 kJ / mol
n 0, 003 0,25đ
Vì phản ứng (1) có nkhí = 0 và áp suất không ảnh hưởng đến H nên
 Hodc,298 = Udc,298 = −2800, 0 kJ / mol
0,25đ
Vậy nhiệt đốt cháy C6H12O6 ở 298 K, 1atm bằng –2800,0 kJ/mol.
Từ (1) có:
c H o(1) = H dc,298
o
= 6 ( H oht,298 ) + 6 ( H oht,298 ) − ( H oht,298 )
CO2 (k ) H 2 O(l) C6 H12 O6 (r)

 ( H oht,298 ) = 6 ( H oht,298 ) + 6 ( H oht,298 ) − H dc,298


o
C6 H12 O6 (r) CO2 (k ) H 2 O(l)

( H o
)
ht,298 C H O (r)
6 12 6
= 6.(−393,5) + 6.(−285, 2) + 2800, 0 = −1272, 2 kJ / mol

Vậy nhiệt hình thành chuẩn của C6H12O6 ở 298 K bằng – 1272,2 kJ/mol. 0,25đ
Câu n H+ 0,1 2
4.1 D phản ứng hết với HCl tạo ra CO2 và = = nên D phải là muối cacbonat vì
n CO2 0, 05 1
2H+ + CO32- CO2 + H2O 0,25đ
ở nhiệt độ nóng chảy, D không bị phân hủy  D là muối cacbonat của kim loại kiềm  D
có công thức M2CO3.
Mặt khác 0,1 mol chất C phản ứng với CO2 dư tạo thành hợp chất D và 2,4 gam B  C
phải là peroxit hoặc superoxit, B là O2 0,25đ
Đặt công thức của C là AxOy.
Lượng O2 có trong 0,1 mol C =16 .0,05 + 2,4 = 3,2 (g).
3, 2.100
Vậy khối lượng C là = 7,1 (g)  MC = 71 (g/mol)
45, 07
7,1 − 3, 2 3, 2 3,9
x:y= : = : 0, 2 và x.MA+ 16y = 71
MA 16 M A
0,25đ
 MA= 39 và y =2; x =1.Vậy A là K, B là O2; C là KO2; D là K2CO3
PTHH : K + O2 KO2; 4KO2 + 2CO2 2K2CO3 + 3O2
K2CO3 + 2 HCl 2 KCl + CO2 + H2O
0,25đ
Gọi công thức của X là CxHyNzOt
Câu  x : y : z : t = 1:4:2:1
4.2 Vì phân tử khối của X là 60 và Y là đồng phân của X
 công thức phân tử của X và Y là CH4N2O 0,25đ
Y tác dụng với dung dịch NaOH tạo ra NH3 nên Y là NH4OCN
X được sử dụng làm phân bón nên X là (NH2)2CO
0,25đ
PTHH: NH4OCN + NaOH ⎯⎯ → NH3 + NaOCN + H2O
o
t

Các PTHH:
Câu Si + Mg ⎯ ⎯→ Mg2Si
to
(1)
4.3
(2) Mg2Si + H2O → Mg(OH)2 + SiH4
⎯⎯→
o
t
(3) SiH4 + 2O2 SiO2 + 2H2O
SiO2 + 2C ⎯ ⎯→
o
t
(4) Si + 2CO 1,0đ
Si + 2F2 ⎯⎯→
o
t
(5) SiF4
Si + O2 ⎯⎯→ SiO2
o
t
(6)
(7) Si + 2NaOH + H2O → Na2SiO3 + 2H2
(8) SiO2 + 4HF → SiF4 + 2H2O
⎯⎯→
o
t
(9) SiO2 + 2NaOH Na2SiO3 + H2O
(10) Na2SiO3 + CO2 + H2O ⎯⎯→ H2SiO3 + Na2CO3
Câu Tính axit của B > C
5.1a Giái thích: nhóm NO2 có hiệu ứng – I và – C. Ở vị trí para hiệu ứng – C của NO2 ảnh hưởng 0,25đ
hút electron trực tiếp đến nhóm OH còn ở vị trí meta nhóm NO2 chỉ có hiệu ứng với vòng
benzen mà không ảnh hưởng trực tiếp đến nhóm OH.
Câu Nhiệt độ sôi và tính tan trong nước của A < B.
5.1b Giái thích: Vì A có liên kết hiđro nội phân tử còn B có liên kết hiđro liên phân tử
O
H

0,25đ
O
N

O
NH2
COOH
H
Câu HS
HS
H
5.2 COOH
NH2

0,5đ
(S) - penicillamin (R) - penicillamin

Câu Đặt X: CxHy


5.3 88,235.126
MX = 136  x = = 10
12.100 0,25đ
y = 136 – 12. 10 = 16
 Công thức phân tử của X là C10H16
10.2 + 2 - 16
ΔX = =3
2
X tác dụng Br2 theo tỉ lệ mol 1:2  X có 2 liên kết  và 1 vòng
X không tác dụng với dung dịch AgNO3/NH3  X không có liên kết ba đầu mạch. 0,25đ
O

O3 O
X (C10H16) CH2 = O +

O
 Công thức cấu tạo của X có thể là:
0,75đ

 

X2 X3
X1

X1, X2 và X3 đều có 1 trung tâm cacbon bất đối và 2 liên kết pi (đều không có đồng phân 0,25đ
hình học) nên X luôn có hai đồng phân lập thể.
O
Br NC COOH C2H5OOC
COOC2H5
Câu
6.1
0,25đ
OMe OMe OMe OMe
A B C E
C2H5OOC CHO C2H5OOC COOC2H5

0,25đ

OMe OMe
F G
Chất A có độ bất bão hòa bằng 1
Câu Chất A có phản ứng iodofom  A có nhóm CH3-CH(OH)- hoặc CH3CO-
6.2 Chất A không cộng H2  A không có liên kết đôi C=C và không có vòng 3, 4 cạnh.
Nếu ozon phân khử hoặc oxi hóa hỗn hợp X thì sản phẩm thấy xuất hiện xiclopentanon 
A có vòng 5 cạnh.
A có thể là:

Chất A tách nước thu được 3 sản phẩm B, C và D


0,25đ

Nên CTCT của A là


Ba chất B, C và D lần lượt là:
0,25đ
O

Câu
7.1 O

O
t0 O
Diels - Alder
O
A
O O
O
B
HOOC O
COOH hv 1.0đ
1. Na2CO3
Pb(OAc)4
Basketen 2. HCl

C
D
Xác định cấu trúc của X(C10H18O):  = 2
Câu - X không làm mất mầu dung dịch nước brom và dung dịch thuốc tím loãng chứng tỏ trong
7.2 X không có nối đôi hay nối ba. 0,25
- X không tác dụng với hiđro trên chất xúc tác niken chứng tỏ trong X không có nhóm chức
cacbonyl.
- X tác dụng với axit clohiđric đậm đặc sinh ra 1-clo-4-(1-clo-1-metyletyl)-1-
metylxiclohexan, trong X có vòng no và có liên kết ete.
=> Suy ra cấu trúc của X:

0,25

2. 2.
a. a. B (CH H20O ) - H-2O
H2O
A (CH10HO) O)
B i)
(C 10 1020O2) 2 A (C10 18 18

Suy ra Y là một điol có bộ khung cacbon như X


OH
+
H
H2O XA
0,25
OH
BY

Gọi tên Y: 1-hiđroxi-4-(-1-hiđroxi-1-metyletyl)-1-metylxiclohexan


ii) 0,25

(+)
OH

OH
OH OH OH H
+

H
+ O
OH
(+)

Cis-B
Cis-Y +
H
0,25
A
X

(+)

OH
OH OH
OH OH + O
H
OH
(+)

Trans-B
Trans-Y H
+
XA
0,25
Câu 1. Phản ứng thủy phân: C20H27O11N + 2H2O →2C6H12O6 + C6H5CHO + HCN
8 glucose benzaldehyde hydrocyanic acid 1.0đ
2. Cấu trúc của A: C6H5 - CH(CN) - O - C12H21O10
Lk glicoside hợp phần đường chứa 2 mảnh glucose 1.5đ
HỘI CÁC TRƯỜNG CHUYÊN ĐỀ THI MÔN HÓA HỌC LỚP 11
VÙNG DUYÊN HẢI VÀ ĐỒNG BẰNG BẮC BỘ Thời gian làm bài 180 phút
TRƯỜNG THPT CHUYÊN TRẦN PHÚ
ĐỀ + HDC THI ĐỀ XUẤT Người ra đề : Nguyễn Thị Thanh Thúy
(Gồm có 12 trang, 8 câu) Đt : 0983.481.700

Câu 1. (2,5 điểm) Tốc độ phản ứng.


1.1. Phản ứng ion – phân tử tổng hợp khí amoniac trong “khí đục” giữa các vì sao xảy ra ở
nhiệt độ cực kì thấp. Sơ đồ phản ứng như sau:
+H2 +H2 +H2 +H2 +e
N→ NH+ → NH2+ → NH3+ → NH4+ → NH3
a.Nguồn gốc của năng lượng hoạt hóa trong phản ứng hóa học là gì?
b. Tốc độ của nhiều phản ứng ion – phân tử gần như không phụ thuộc vào nhiệt độ. Nhận xét
về năng lượng hoạt hóa của các phản ứng đó. Tại sao chúng có thể xảy ra được ở vùng giữa các
vì sao, nơi có nhiệt độ rất thấp?
1.2. Sự phân hủy NH3 thành N2 và H2 được tiến hành trên bề mặt volfram (W) có năng lượng
hoạt hóa Ea = 163 kJ.mol-1; khi không có mặt xúc tác, Ea = 335 kJ.mol-1.
a. Phản ứng trên bề mặt W ở 298 K nhanh hơn xấp xỉ bao nhiêu lần so với phản ứng không có
xúc tác?
b. Tốc độ phản ứng phân hủy NH3 trên bề mặt W tuân theo quy luật động học có dạng:
[NH3 ]
𝑣=𝑘 ;
[H2 ]
Trong đó: k là hằng số tốc độ phản ứng; [NH3] và [H2] là nồng độ của NH3 và H2.
Vì sao tốc độ phản ứng lại tỉ lệ nghịch với nồng độ của H2?
Hướng dẫn chấm:
Câu Nội dung Điểm

1.1. Các phản ứng hóa học bao gồm những quá trình phá vỡ và hình thành các liên 1,0
kết. Năng lượng hoạt hóa gắn liền với năng lượng phá vỡ các liên kết ban đầu
và tạo ra do sự sắp xếp để hình thành cấu tạo phân tử các chất trong phản ứng.
Đối với các phản ứng ion – phân tử sự thay đổi này là không đáng kể.
Hằng số tốc độ (k) phụ thuộc vào nhiệt độ theo phương trình Arrhenius:
k(T) = A.exp(- Ea/RT)
Ở đây A là thừa số trước mũ; Ea là năng lượng hoạt hóa; R là hằng số khí; T
là nhiệt độ tuyệt đối.
Mỗi phản ứng hóa học có một năng lượng hoạt hóa xác định, phụ thuộc vào
bản chất của các chất tham gia phản ứng và như nói ở trên là phụ thuộc vào sự
thay đổi về cấu trúc phân tử của các chất trong phản ứng. Đối với các phản ứng
ion – phân tử, sự thay đổi này là không đáng kể nên tốc độ phản ứng gần
như không phụ thuộc nhiệt độ và do đó năng lượng hoạt hóa bằng không.
Nhiệt độ ở vùng giữa các vì sao cực kỳ thấp, chỉ những phản ứng có năng
lượng hoạt hóa rất thấp (ví dụ như phản ứng tổng hợp amoniac nói trên) mới xảy
ra với tốc độ đáng kể.

Trang 1 / 12
1.2. 1,0

Vì nồng độ hydro ([H2]) nằm ở mẫu số trong biểu thức động học, điều đó cho 0,5
biết sự có mặt của H2 làm giảm tốc độ phản ứng. Để phản ứng phân hủy NH3 có
thể xảy ra, các phân tử NH3 phải được hấp thụ trên bề mặt chất xúc tác. Nếu H2
được hấp thụ trên bề mặt chất xúc tác sẽ dẫn tới ít vị trí còn trống hơn cho các
phân tử NH3 được hấp thụ. Hệ quả là tốc độ phản ứng giảm.

Câu 2. (2,5 điểm) Cân bằng và phản ứng trong dung dịch. Pin điện - Điện phân.
Methyl đỏ (NaMR) là chất chỉ thị acid-base phổ biến, được ứng
dụng trong các phép trắc quang xác định pH.
Phương pháp sau đây được sử dụng để xác định pKa của methyl
đỏ:
Bước 1: Tạo dung dịch mẫu. Methyl đỏ (M = 291,9)
Lấy 10 ml dung dịch NaMR chuẩn, thêm vào 25 mL dung dịch
sodium acetate 0,04 M, một lượng acetic acid 0,02 M và pha thành
100 ml dung dịch. pH của dung dịch tạo thành là 4,80.
Bước 2: Đo độ hấp thụ quang.
Lấy một lượng mẫu vào cuvette dài 1 cm, rồi đo độ hấp thụ quang tại hai bước sóng: 425 nm
và 524 nm, thu được các kết quả lần lượt là 0,280 và 0,852.
Dưới đây là phổ hấp thụ và hệ số hấp thụ mol của các
dạng acid và base của methyl đỏ.
ε524(HMR) = 44640 M-1cm-1
ε425(HMR) = 3860 M-1cm-1
ε524(MR-) = 740 M-1cm-1
ε425(MR-) = 18530 M-1cm-1
(непротонована форма = dạng không-proton hóa
[base]; протонована форма = dạng proton hóa [acid])
2.1. Đề xuất cấu tạo dạng base và acid của methyl đỏ.
2.2. Xác định màu sắc của dung dịch methyl đỏ.
2.3. Từ dữ kiện trên, hãy tính pKa của chỉ thị methyl đỏ.
2.4. Tính thể tích acetic acid 0,02 M đã thêm vào ban đầu. Biết pKa(CH3COOH) = 4,8.
2.5. Tính khối lượng (theo mg) NaMR có trong 10 ml dung dịch chuẩn ban đầu.
Hướng dẫn chấm:
Câu/ Nội dung Điểm
Ý
Trang 2 / 12
2.1. Đề xuất cấu tạo dạng base và acid của methyl đỏ. 0,5

2.2. Xác định màu sắc của dung dịch methyl đỏ. 0,5
Ở pH = 1: màu đỏ
Ở pH = 8: màu vàng (cam)
2.3. Từ dữ kiện trên, hãy tính pKa của chỉ thị methyl đỏ. 0,5
Độ hấp thụ tổng của hai dạng là:
A = A(MR-) + A(HMR) = εi(MR-)·l·[MR-] + εi(HMR)·l·[HMR]
Ứng với mỗi bước sóng, ta có:
0.852 = 740[MR-] + 44640 [HMR] 0.280 = 18530 [MR-] + 3860 [HMR]
Giải hệ phương trình:
[MR-] = 1,12·10-5 М, [HMR] = 1,89·10-5
Ka = ([MR-]·[H+])/[HMR] = (1,12·10-5) · (1.585·10-5)/(1,89·10-5)
= 9,367·10-6
pKa = 5,03 hoặc: pKa = pH - log ([MR-]/[HMR]) = 4,80 - (-0.228) = 5,03
2.4. Tính thể tích acetic acid 0,02 M đã thêm vào ban đầu. 0,5
Biết Ka(CH3COOH) = 4,8.
Đặt số mol acetic acid là x. Do tỉ lệ nồng độ mol của dạng proton hóa và
không-proton hóa chỉ phụ thuộc vào số mol chất này, nên:

2.5. Tính khối lượng (theo mg) NaMR có trong 10 ml dung dịch chuẩn ban 0,5
đầu.
Từ dữ kiện trắc quang: tổng nồng độ là 3,01·10-5 M. Do vậy, nồng độ ban
đầu là 3,01·10-4 M. Số mol chất là 3,01·10-6 mol, tương đương với 0,877
mg.

Câu 3. (2,5 điểm) Nhiệt động học và cân bằng hóa học.
Tìm kiếm, sử dụng và lưu trữ năng lượng là một trong những vấn đề quan trọng trong thế giới
ngày nay. Năng lượng có thể được lưu trữ theo phương pháp hoá học bởi các chu trình sulfur-
iodine. Chu trình này là một phương pháp sản xuất nhiên liệu hydrogen có hiệu quả hơn điện phân
nước.
(1). I2 (g) + SO2 (g) + 2H2O ⇌ H2SO4 (g) + 2HI (g)

Trang 3 / 12
(2). 2H2SO4 (g) ⇌ 2SO2 (g) + 2H2O (g) + O2 (g)
(3). 2HI (g) ⇌ H2 (g) + I2 (g)
3.1. Sử dụng dữ kiện và các phương trình cho ở cuối bài để trả lời các câu hỏi sau về phản ứng
(3):
a. Tính hằng số cân bằng ở 298 K (K298).
b. Tính hằng số cân bằng ở 723 K (K723). Giả sử ΔrH, ΔrS không phụ thuộc vào nhiệt độ.
3.2. Các điều kiện đã được chọn để (3) phản ứng cân bằng ở trên chuyển dịch theo chiều từ trái
sang phải. Giả sử rằng sản phẩm của phản ứng (1) tham gia hết vào các phản ứng (2), (3). Viết
phương trình phản ứng tổng của chu trình sulfur-iodine.
3.3. Biến thiên enthalpy chuẩn của phản ứng ở 298 K đối với phản ứng (2) là +439 kJ.mol-1.
Sử dụng giá trị ΔfH (298 K) với H2O(g) trong bảng dưới để tính biến thiên enthalpy chuẩn của
phản ứng (1) ở 298 K.
3.4. Tính năng lượng được lưu trữ với trong một vòng chu trình sulfur-iodine đối với mỗi mol
sulfur nguyên tử ở 298 K.
Dữ kiện: R = 8,314 J.K-1.mol-1
HI(g) H2(g) I2(g) H2O(g)
ΔfHo (298 K) / kJ mol-1 26.5 62.4 -242
So (298 K) / J K-1 mol-1 207 131 261 189

Hướng dẫn chấm:


Câu Nội dung Điểm

3.1. 1,
0

3.2. 0,
5
3.3. 0,
5

3.4. 0,
5

Câu 4. (2,5 điểm) Hóa nguyên tố (Kim loại, phi kim nhóm IVA, VA). Phức chất.

Trang 4 / 12
Titanium dioxide thường được dùng làm trắng và tạo độ sáng cho thuốc đánh răng, sơn, nhựa,
kính râm, và thậm chí là trong thực phẩm dưới dạng phụ gia E171 trong đồ ngọt, kẹo cao su, nước
sốt màu trắng và bột đường làm bánh (E171 có chứa các hạt nano titanium dioxide và thường
được sử dụng trong nhiều loại sản phẩm tiêu dùng, bao gồm cả thuốc và mỹ phẩm. Ngày
8/10/2021, các nước thành viên Liên minh châu Âu (EU) đã nhất trí cấm việc sử dụng chất phụ
gia E171 trong thực phẩm, sau khi Cơ quan An toàn thực phẩm châu Âu (EFSA) nghi ngờ về tính
an toàn của chất vốn được sử rộng rãi này.)
Mặc dù titanium dioxide tồn tại trong tự nhiên dưới dạng khoáng rutile, nhưng dạng này không
đủ tinh khiết để sử dụng làm sắc tố. Thay vào đó, nó được sản xuất nhân tạo, hoặc là từ rutile thô,
hoặc từ một khoáng chất khác có hàm lượng lớn hơn là ilmenite (FeTiO3).
Một quy trình công nghiệp để sản xuất TiO2 đi qua titanium(IV) chloride, TiCl4, thường được
biết đến với cái tên “Tickle”.
4.1. Xác định số oxid hoá của titanium trong ilmenite, FeTiO3.
4.2. Trong “quá trình chloride”, titanium(IV) chloride được tạo thành từ quặng titanium và
carbon. Giả sử rằng sản phẩm tạo thành là các chloride kim loại và carbon monoxide, hãy viết
phương trình hóa học của phản ứng:
a. Giữa rutile (TiO2), chlorine và carbon.
b. Giữa ilmenite (TeTiO3), chlorine và carbon. Biết rằng trong quá trình này, sắt của ilmenite
bị oxid hoá.
4.3. Trong giai đoạn cuối của quy trình sản xuất, TiCl4 phản ứng với khí oxygen, tạo thành
TiO2, tạo thành khí chlorine theo phương trình sau:
TiCl4(g) + O2(g) → TiO2(s) + 2Cl2(g) H = -175 kJ.mol-1
Tính biến thiên enthalpy tạo thành chuẩn của TiCl4(g) và năng lượng phân li liên kết của TiCl4.
Biết:
Biến thiên enthalpy tạo thành chuẩn, ∆fH° của TiO2(s) là: -939 kJ.mol–1
Biến thiên enthalpy nguyên tử hóa, ∆atH° của Ti(s) là: 473 kJ.mol–1
Năng lượng liên kết của Cl2(g) là: 242 kJ.mol–1
Hướng dẫn chấm:
Câu/ Nội dung Điểm
Ý
4.1. Số oxi hóa của Ti trong FeTiO3: +3 hoặc +4 0,5
4.2. TiO2(s) + 2Cl2(g) + 2C(s) → TiCl4(l) + 2CO(g) 1,0
2FeTiO3(s) + 7Cl2(g)+ 6C(s) →2 FeCl3(s)+ 2TiCl4(l)+ 6CO(g)
4.3. - Tính biến thiên enthalpy tạo thành chuẩn của TiCl4(g): 1,0

∆fHo(TiCl4(g)) = ∆fHo(TiO2(s)) – ∆Ho


= –939 – (–175) kJ.mol-1 = –764 kJ.mol-1

Trang 5 / 12
- Tính năng lượng phân li liên kết của TiCl4:

4 x b.s.TiCl4(g) = ∆atHo(Ti(s)) + 2 · b.s.Cl2(g) – ∆fHo(TiCl4(g))


= 473 + 2 x 242 – (–764) kJ.mol-1 = 1721 kJ.mol-1 + 2. b.s.(Cl2(g))
Năng lượng liên kết trung bình trong TiCl4 là: 430 kJ.mol-1

Câu 5. (2,5 điểm) Đại cương hữu cơ.


Tiến hành phân tích nguyên tố 0,5000 gam một hợp chất X ở áp suất 1,0135 bar và nhiệt độ 25
o
C, có 683,9 cm3 CO2 và 0,323825 gam H2O đã được tạo thành. Hợp chất X có khối lượng mol
khoảng 250 gam/mol. Dưới đây là phổ hồng ngoại, 1H NMR và dữ liệu phân tích phổ 13C NMR
của X.

Phổ hồng ngoại

Phổ 1H NMR
Trang 6 / 12
Dữ liệu phân tích phổ 13C NMR: 13.6; 35.7; 57.8; 59.5; 125.3; 127.6; 128.7; 141.1; 174.5 ppm
5.1. Xác định công thức phân tử của hợp chất
5.2. Vẽ công thức cấu tạo của hợp chất chưa biết. Gán các tín hiệu trong phổ 1H NMR với các
proton trong hợp chất bằng các kí hiệu chữ cái a, b, c, …
Hướng dẫn chấm:
Câu/ Nội dung Điểm
Ý
5.1. 1,5

5.2. 1,0

Câu 6. (2,5 điểm) Sơ đồ tổng hợp hữu cơ. C ơ chế phản ứng hóa hữu cơ.
6.1. Phản ứng giữa aminoethanol và acetic anhydride xảy ra theo sơ đồ sau:

Khi tiến hành phản ứng trong K2CO3 thu được một sản phẩm không vòng B. A cũng có thể
chuyển thành B dưới tác dụng của K2CO3
a. Xác định B.
b. Cho biết vai trò của HCl trong sự tạo thành A và K2CO3 đối với sự tạo thành B.
c. Vì sao A chuyển hóa thành B dưới tác dụng của K2CO3?
6.2. C được tổng hợp bằng phản ứng dưới đây. Tuy nhiên kết quả lại thu được một sản phẩm
đồng phân. Xác định cấu trúc của D, E.

Trang 7 / 12
Hướng dẫn chấm:
Câu/ Nội dung Điểm
Ý
6.1. a. Xác định B. 0,5

b. Vai trò của HCl trong sự tạo thành A và K2CO3 đối với sự tạo thành B: 0,5
Trong sự tạo thành A: HCl proton hóa amine khiến nguyên tử N mất tính
nucleophile. Khi dùng base thì N vốn có tính nucleophile mạnh hơn O sẽ tấn
công thu được B.
c. Vì sao A chuyển hóa thành B dưới tác dụng của K2CO3? 0,5
Khi chuyển A thành B dưới tác dụng K2CO3 thì đầu tiên sẽ tạo thành nhóm
amine tự do từ muối amonium của A, nhóm amine tự do sẽ tấn công vào ester
để tạo trung gian vòng 5 cạnh và cắt đứt liên kết C-O, tạo thành A.
6.2. Mỗi cấu tạo đúng: 0,5 điểm 1,0

Câu 7. (2,5 điểm) Xác định cấu trúc các chất hữu cơ (mô tả sơ đồ tổng hợp bằng lời
dẫn).
Phát hiện gần đây về việc nhiễm trùng virus Zika khi mang thai được cho
là có liên quan tới dị tật đầu nhỏ ở trẻ sơ sinh đã buộc Tổ chức Y tế thế giới
(WHO) ra tuyên bố Tình trạng Khẩn cấp Y tế Công cộng Quốc tế PHEIC
(Public Health Emergency of International Concern) vào ngày 01 tháng
02 năm 2016.
Virus Zika đi vào cơ thể người khi họ bị muỗi Aedes nhiễm bệnh đốt. Hiện nay (tính tới tháng
02/2016) không có một loại vaccine hoặc thuốc điều trị nào có thể chống lại virus Zika, do đó
việc giảm số lượng muỗi và khả năng bị chúng đốt là những việc làm quan trọng để giảm tỉ lệ
nhiễm trùng.
Sử dụng thuốc trừ sâu để xử lí các quần thể muỗi là cách làm hiệu quả để giảm các bệnh dịch
lây truyền từ chúng. Một trong các họ thuốc trừ sâu thường dùng để xử lí quần thể muỗi là
pyrethroid, bởi chúng có tác động tới môi trường thấp hơn các loại thuốc cơ phosphate và cũng ít
độc hại với cơ thể người hơn.
Trang 8 / 12
Mặc dù virus Zika chỉ mới được phát hiện gần đây, nhưng từ hàng trăm năm
trước các hợp chất hoá học đã được dùng để chống lại các căn bệnh truyền nhiễm
do muỗi. Sốt rét cũng là một loại bệnh truyền nhiễm gây ra do muỗi, đa số các hoá
chất dùng để điều trị sốt rét có nguồn gốc quinoline. Một trong số đó, primaquine,
là loại thuốc duy nhất điều trị được cả sốt rét tái phát và cấp tính.
Trong quy trình tổng hợp primaquine, tất cả các nguyên tử carbon trong sản phẩm đều có nguồn
gốc từ các tác nhân U và V.

Nhóm NH2 trong tác nhân U là một tác nhân nucleophile, còn tác nhân V là tác nhân
electrophile. Tuy nhiên, để tổng hợp primaquine bởi phản ứng thế nucleophile giữa các tác nhân
này rất khó khăn do có nhiều sản phẩm có thể được tạo thành. Từ 1 phân tử tác nhân V, và phản
ứng với U chỉ sử dụng duy nhất nguyên tử nitrogen trong nhóm NH2 làm tâm nucleophile, có thể
tạo ra tới 4 sản phẩm: 2 trong số đó là đồng phân cấu tạo, 2 chất còn lại có công thức phân tử
khác.
7.1. Xác định công thức cấu tạo của 4 sản phẩm đó.
7.2. Thực tế, chỉ có 1 trong 4 sản phẩm được xác định ở 7.1 là hợp chất sử dụng được cho tổng
hợp primaquine và cùng chỉ chiếm một lượng nhỏ trong hỗn hợp sản phẩm của phản ứng giữa U
và V, do vậy quy trình tổng hợp primaquine phải tiến hành theo 7 giai đoạn như sau:

Xác định công thức cấu tạo của các anion W-, hợp chất X, Y và sản phẩm phụ Z.

Hướng dẫn chấm:


Câu/Ý Nội dung Điểm
7.1. Mỗi cấu tạo đúng: 0,25 điểm 1,0

Trang 9 / 12
Mỗi cấu tạo đúng của W-, X, Y: 0,25 điểm; 1,5
Cấu tạo đúng của Z: 0,5 điểm;

Câu 8. (2,5 điểm) Hóa học các hợp chất thiên nhiên (Cacbohidrat và các hợp chất hữu
cơ chứa nito đơn giản)
Các amino acid thiên nhiên, glutamine (Gln) và glutamic acid (Glu) chỉ khác nhau bởi
nhóm amide và carboxylic ở cuối mạch. Tuy nhiên, chỉ một khác biệt nhỏ như vậy cũng
gây ra những khác biệt lớn đến tính chất hóa học.

8.1. Viết các cân bằng trong dung dịch nước của mỗi amino acid.
8.2. Xác định công thức tính hằng số cân bằng của các cân bằng trong ý 8.1. Tính điểm
đẳng điện IP (là giá trị pH mà tại đó điện tích dương bằng điện tích âm) của mỗi amino
acid. Biết rằng:
pKa pK1 pK2 pK3
Trang 10 / 12
Amino acid
Glutamine 2,17 9,13 -
Glutamic acid 2,19 9,67 4,25
8.3. Hoàn thành sơ đồ chuyển hóa sau:

Biết rằng:
- [H] = khử hóa; [O] = oxid hóa; t = đun
nóng
- C cũng là một amino acid có trong tự
nhiên;
- D, E, F, J có hệ 3 vòng ngưng tụ;
- D và J là các đồng phân cấu tạo.

Hướng dẫn chấm:


Câu/ Nội dung Điểm
Ý
8.1. 0,5

8.2. 1,0

Trang 11 / 12
8.3. 1,0

Trang 12 / 12
HỘI CÁC TRƯỜNG CHUYÊN ĐỀ THI MÔN HÓA HỌC
VÙNG DUYÊN HẢI VÀ ĐỒNG BẰNG BẮC KHỐI 11 - NĂM 2023
BỘ Thời gian: 180 phút
TRƯỜNG THPT CHUYÊN LÊ THÁNH (Đề thi có 09 trang, gồm 8 câu)
TÔNG
QUẢNG NAM
ĐỀ THI ĐỀ XUẤT
Câu 1 (2,5 điểm): Tốc độ phản ứng
Các ester bị thủy phân trong môi trường nước, và quá trình này được xúc tác bởi cả acid
và base. Bảng dưới đây đưa ra sự phụ thuộc của nồng độ methyl acetate trong nước với
thời gian. Dung dịch ban đầu được điều chế bằng cách trộn khoảng 20ml ester (khối lượng
riêng d = 0,933 kg/L, 980ml nước và một lượng xúc tác sulfuric acid (pH~2, được duy trì
không đổi, bỏ qua thể tích acid).
Thời gian 10 150 400 720 1010
(phút)
[MeCOOMe] 0,269 0,246 0,211 0,173 0,144
(M)
a) Xác định bậc phản ứng theo ester và tính hằng số tốc độ biểu kiến (kobs).
Khi thay thế methyl acetate bởi methyl butylrate (25ml ester, d = 0,9 kg/L, 975ml nước
và acid) thì có sự thay đổi như sau: Trong 14,5 giờ đầu tiên, nồng độ ester trong nước
được xác định thấy là 0,157 M và không thay đổi, rồi sau đó giảm.
b) Giải thích ngắn gọn hiện tượng này.
c) Tính hằng số tốc độ kobs của phản ứng thủy phân methyl butylrate, cũng như nồng
độ của nó sau 1 ngày kể từ khi bắt đầu thí nghiệm.
Khi thủy phân methyl acetate có mặt base (dư 50%) thì thu được các dữ kiện sau:
Thời gian 0 0,5 1 1,75
(phút)
[MeCOOMe] 0,500 0,338 0,246 0,165
(M)
r (M/s) 7,39.10-3 3,91.10-3 2,4.10-3 1,35.10-3

Trang 1/20
d) Xác định bậc phản ứng của ester trong trường hợp này và tính hằng số tốc độ của phản
ứng.
HDC:
a) 0,5 điểm
Phản ứng có bậc là 1 theo ester.
kobs = 6,27.10-4 phút-1
b) 0,5 điểm
Do độ tan hạn chế của methyl butylrate, lượng ester lấy ban đầu không được trộn
lẫn hoàn toàn với nước, phase hữu cơ vẫn còn. Khi phản ứng thủy phân diễn ra,
nồng độ cân bằng của ester vẫn được duy trì trong pha nước.
c) 0,5 điểm
25.0,9
d[ester] △Cester -0,157.0,975
r=- =- = 102
= 7,96 . 10-5 (M.phút-1) = kobs[ester]
dt △t 0,975.14,5.60

7,96.10-5
⇒kobs = = 5,1.10-4 phút-1
0,157

Khi nồng độ nhỏ hơn độ tan, quan sát được động học bậc một như bình thường:
C24h = C15h . exp(-kt) = 0,157 . exp(-5,1.10-4.9,5.60) = 0,117 M
d) 1,0 điểm
R1COOR2 + OH- → R1COOR- + R2OH
[OH]- = 0,25 + [ester]
r = k’[ester]a[OH-]b
r0 0,5 0,75
ln = a. ln + b. ln
r0,5 0,338 0,588
r0 0,5 0,75
ln = a. ln + b. ln
r1 0,246 0,496
Giải hệ phương trình: a=1, b=1
Bậc phản ứng của ester bằng 1.
Ta tính hằng số tốc độ k’ như sau:

=
[ ][ ]
Lấy giá trị trung bình, k’ = 1,97.10-2 M-1phút-1
Câu 2 (2,5 điểm): Cân bằng và phản ứng trong dung dịch. Pin điện – Điện phân.

Trang 2/20
2.1. Ở nhiệt độ 25oC, độ tan của Fe2(SO4)3 là khoảng 245 gam/1,00 L nước, có nghĩa là
nồng độ khoảng 0,6 M. Một sinh viên dự kiến pha 100,0 mL dung dịch Fe2(SO4)3 nồng
độ 0,100 M. Tuy nhiên khi pha xong thì trong dung dịch thu được (gọi là dung dịch A)
xuất hiện vẩn đục màu nâu.
a) Bằng tính toán, hãy giải thích hiện tượng trên.
b) Tính pH của dung dịch A.
c) Để làm tan vẩn đục, sinh viên này thêm từ từ từng giọt dung dịch H2SO4 đặc (nồng độ
98%, khối lượng riêng D = 1,84 g/mL) vào 100,0 mL dung dịch A kể trên (vừa thêm
dung dịch, vừa khuấy). Tính số giọt dung dịch H2SO4 đặc tối thiểu mà sinh viên này cần
thêm vào dung dịch A để dung dịch trở nên trong suốt. Coi thể tích mỗi giọt là như nhau
và đều là 0,030 mL.
Cho biết: pKa(HSO4–) = 1,99; pKs(Fe(OH)3) = 37,00; pKw = 14,00.
Fe3+ + H2O ⇌ FeOH2+ *β = 10–2,17
HDC: Câu a) 0,5 điểm Câu b) 0,5 điểm Câu c) 0,5 điểm

b)
Viết các quá trình và cân bằng.
h = [FeOH2+] + 3(CFe3+ - [Fe3+] – [FeOH2+]) – [HSO4-]
= 3CFe3+ - 3[Fe3+] - 2[FeOH2+] - [HSO4-]

⇒ pH = 2,00

Trang 3/20
2.2. Thêm 10,00 mL dung dịch Fe2+ 0,10 M vào 30,00 mL dung dịch gồm MnO4− 0,020
M và H+ 0,72 M được dung dịch B.
a) Viết và cân bằng phản ứng oxi hoá khử. Tính hằng số cân bằng của phản ứng.
b) Tính thế của điện cực Pt nhúng trong dung dịch B.
c) Viết sơ đồ pin điện khi ghép điện cực Pt nhúng trong dung dịch B với điện cực Cu
nhúng trong dung dịch C thu được khi trộn 10 mL dung dịch Cu+ 0,02 M với 10 mL
dung dịch NH3 2,0M. Tính sức điện động của pin.
Cho biết: Eo(MnO4−, H+/Mn2+) = 1,510 V; Eo(Fe3+/Fe2+) = 0,771 V; Eo(Cu+/Cu) = 0,521
V;
Phức Cu+ − NH3 có: lgk1 = 5,93; lgk2 = 4,93; pKa(NH4+) = 9,24.
HDC: Câu a) 0,25 điểm Câu b)0,25 điểm Câu c) 0,5 điểm

Trang 4/20
Câu 3 (2,5 điểm): Nhiệt động học và cân bằng hóa học.
3.1. Một nhà hóa học đã thực hiện các phép đo áp suất hơi nước ở các nhiệt độ khác nhau
và các dữ kiện được cho trong giản đồ dưới đây. Trong một bình dung tích 100 L, có một
chất khí bền được bão hòa hơi nước ở nhiệt độ 40oC và áp suất tổng bằng 1000 mmHg.
Xác định khối lượng nước được ngưng tụ trong điều kiện đẳng áp nếu nhiệt độ của hỗn
hợp khí giảm đi 15oC.

HDC: 1 điểm
PH2 O(40o C) =7366 Pa từ phương trình hồi quy
PH2 O(25o C) =2995 Pa (từ phương trình hồi quy)
PH2 O
n(H2O) = . ntổng
Ptổng

ntổng = PV/RT = 5,13 mol


⇒ △n(H2O) = 0,17 mol ⇒ m(H2O) = 3,06 g

3.2. Ở nhiệt độ cao PCl5 bị phân li theo phản ứng: PCl5(k) ⇌ PCl3(k)+ Cl2(k)
Người ta thực hiện các thí nghiệm sau:
a) Thí nghiệm 1: Cho 83,2956 gam PCl5 vào bình dung tích V ở nhiệt độ T. Sau khi đạt
tới cân bằng đo được P1 = 2,700 atm; hỗn hợp khí trong bình có tỉ khối so với hidro
bằng 68,862. Tính độ phân li α1 và KP1.
b) Thí nghiệm 2: Được tiến hành với các điều kiện nhiệt độ và lượng chất như thí
nghiệm 1 nhưng trong bình phản ứng có dung tích V’ = 2V. Tính áp suất P2 của bình
phản ứng tại trạng thái cân bằng.
c) Thí nghiệm 3: Thêm tiếp vào hệ phản ứng ở thí nghiệm 1 một lượng Ar có thể tích
bằng V (ở T, P1), giữ nhiệt độ và dung tích của bình phản ứng không đổi. Tính áp suất

Trang 5/20
P3 của bình phản ứng khi cân bằng mới được thiết lập.
d) Thí nghiệm 4: Thêm tiếp vào hệ phản ứng ở thí nghiệm 1 một lượng Cl2 có thể tích
bằng V (ở T, P1), giữ nhiệt độ và dung tích của bình phản ứng không đổi. Tính áp suất
P4 của bình phản ứng khi cân bằng mới được thiết lập.
e) Thí nghiệm 5: Giữ nguyên lượng PCl5 và dung tích bình như ở thí nghiệm 1 nhưng hạ
nhiệt độ của bình đến T’ = 0,9T thì đo được áp suất cân bằng là P5 = 1,944 atm. Tính KP5
và độ phân li α5 của PCl5, từ đó cho biết phản ứng thuận là tỏa nhiệt hay thu nhiệt.
Cho biết: Cl = 35,453; P = 30,974; H = 1,008; Các khí đều là khí lí tưởng.
HDC:
a) 0,5 điểm

PCl5 → PCl3 + Cl2


Ban đầu 0,4
CB (1-α1).0,4 0,4.α1 0,4.α1
ntổng = (1+α1).0,4
83,2956
= ⇒ = 0,5
68,862.2
α21
KP1 = . P = 0,9 (atm)
(1-α1 )(1+α1 ) 1
b) 0,25 điểm
PCl5 → PCl3 + Cl2
Ban đầu 0,4
CB 0,4-x x x
ntổng = 0,4 + x
x 2 x 2
( ) ( ) (0,4+x) . 0,082 . T
0,4+x 0,4+x
KP1 = . P2 = . = 0,9 (atm) (I)
0,4-x 0,4-x 2V
( ) ( )
0,4+x 0,4+x
T P1 2,7
Ở TN1: = = = 54,878 (II)
V nR 0,4(1+0,5).0,082

Từ (I) và (II) ⇒ x=0,2472 ⇒P2=1,4562 (atm) ~ 1,5 (atm)

c) 0,25 điểm

Trang 6/20
Pi.V = ni.R.T ⇒ Khi thêm Ar, V= const ⇒ Pi = const
⇒ P3 = P1 + PAr = 2P1 = 5,4 (atm)
d) 0,25 điểm Thêm Cl2 ⇒ Cân bằng dịch chuyển
Tại thời điểm vừa thêm Cl2:
PCl2 = 3,6 atm; PPCl3 = 0,9 atm; PPCl5 = 0,9 atm
PCl3 + Cl2 PCl5

Ban đầu 0,9 3,6 0,9
CB 0,9-y 3,6-y 0,9+y
(0,9 − )(3,6 − )
KP1 = = 0,9 (atm)
(0,9+y)
⇒ y = 0,4955 ⇒ P4 = 4,9 (atm)
e) 0,25 điểm
′ ′ 1,944 0,9 . 2,7
T'=0,9T ⇔ = 0,9 ⇔ = 0,9 ⇔ =
′ ′ ′ 0,4(1 + 0,5)
⇒ n’ = 0,48 (mol) = (1+α5).0,4 ⇒ α5 = 0,2
α25
KP5 = . P = 0,081 (atm)
(1-α5 )(1+α5 ) 5

Câu 4 (2,5 điểm): Hóa nguyên tố (Kim loại, phi kim nhóm IVA, VA). Phức chất.
4.1. Hợp chất A, tạo thành từ các nguyên tố X và Y, là sản phẩm hóa học quan trọng có
thể được sử dụng để làm chất bôi trơn, thuốc trừ sâu và những hóa chất tương tự. A có
thể được điều chế bởi phản ứng của FeP2 – là sản phẩm phụ của quá trình điều chế X –
với pyrite. Trong phản ứng này, cũng thu được một hợp chất B lưỡng nguyên tố, B hòa
tan trong sulfuric acid loãng tạo thành khí C, còn phản ứng với sulfuric acid đặc thì giải
phóng sulfur dioxide. Chất C tạo kết tủa với rất nhiều ion kim loại. Chất A tinh khiết có
màu vàng, bền nhiệt, nhưng trong không khí ẩm dễ bị phân hủy, gây ra mùi trứng thối.
A bị thủy phân trong ethanol, thu được hợp chất diester D (phân tử có 1 nguyên tử X ở
trung tâm) và giải phóng khí C. Phản ứng giữa D với Cl2 dùng để chế tạo E (là nguyên
liệu cho thuốc trừ sâu), đồng thời giải phóng acid khí F và đơn chất Y (tỉ lệ mol sản
phẩm 1:1:1). Trộn lẫn rồi đun nóng hỗn hợp phosphorus pentoxide thu được hai hợp
Trang 7/20
chất G1 và G2 có cấu trúc đói xứng.
a) Viết công thức các chất A, C-F, G1, G2.
b) Viết phương trình điều chế A từ phản ứng giữa pyrite với FeP2.
c) Viết phương trình phản ứng của B với sulfuric acid đặc.
HDC:
a) 0,5 điểm A: P4S10; C: H2S; D: S=P(OCH2CH3)2(SH); E: S=P(OCH2CH3)2Cl; F:
HCl;
G1: P4O6S4; G2: P4S6O4
b) 0,25 điểm 2FeP2 + 12FeS2 → 14FeS + P4S10
c) 0,25 điểm 2FeS + 10H2SO4 (đặc) → Fe2(SO4)3 + 9SO2 + 10H2O
4.2. Trong một ống nghiệm thạch anh, cho iodine phản ứng với các mảnh đơn chất kim
loại chuyển tiếp M ở 280oC để tạo thành chất rắn X màu nâu đỏ trên bề mặt kim loại.
Đun nóng X trong bình chân không ở 300oC để phân hủy, tạo thành chất rắn Y màu đỏ
và chất khí màu tím, quá trình đi kèm với sự giảm khối lượng 29.4%. Y được chuyển
hóa thành chất màu đỏ thẫm Z khi tiếp xúc với không khí khô (khối lượng tăng 5.25%
và Z có nhiều hơn Y 1 nguyên tử oxygen). Cho sulfate của M phản ứng hoàn toàn với
phối tử hữu cơ hai càng 2,2'-bipyrimidine (kí hiệu là bipy, khối lượng phân tử 156.2),
sau khi phản ứng diễn ra hoàn toàn thì thêm một lượng vừa đủ KI trong methanol vào
bình phản ứng để nhận được các tinh thể màu xanh lục A. A phản ứng với Mg tạo thành
chất màu xanh lam B, chất này bị khử tiếp bởi LiAlH4 trong THF, tạo thành chất rắn
màu đỏ C. Nếu cho B phản ứng với đơn chất iodine thì trước tiên tạo thành chất rắn
màu đỏ khác là D, sau đó D phản ứng tiếp với đơn chất iodine và chuyển trở lại thành
tinh thể màu xanh lục A. Kết quả phân tích nguyên tố cho thấy rằng A có thể được xem
là phức của Y với kết hợp bipy. So với Y, A có khối lượng lớn hơn 157% và không
chứa các phân tử dung môi. Các phép đo độ cảm từ cho thấy moment từ hiệu dụng của
A là 3,75 B, còn C nghịch từ.
a) Xác định công thức của X, Y, Z.
b) Xác định công thức của A.
c) Biểu diễn cấu tạo của B, C, D (có thể biểu diễn các phối tử ở dạng viết tắt).
d) Trong các nghiên cứu về thuốc chống ung thư, phức chất M, biểu diễn trong hình

Trang 8/20
dưới, thu hút nhiều sự chú ý. Hãy viết số oxid hóa và cấu hình electron hóa trị của M
trong phức chất.

Phức chất của M.


HDC:
a)0,5 điểm
Đặt công thức X là MIx, Y là MIy, thì Z là MOIy.
16
Khối lượng đương lượng mol của Y là Mr (Y) = = 304,8. Sau khi thử các giá trị
0,0525

hóa trị (y) thì y = 2, tương ứng Mr (M) = 51 là phù hợp nhất. Vậy M là vanadium (V), Y
là VI2, Z là VOI2.
Sau khi thử các giá trị x thì trường hợp x = 3 phù hợp, tương ứng với X là VI3.
b)0,25 điểm
A có moment từ là 3,75 B, tương ứng với số electron độc thân là 3, tức là ion V2+. Đặt
công thức A là V(bipy)xI2, ta có:
M(VI2 ) 1
= ⇒x≈3
M(V(bipy)x I2 ) 2,57
Công thức của A là V(bipy)3I2.
c) 0,5 điểm
B: V(bipy)3
C: Li[V(bipy)3] (hoặc Li[V(bipy)3]·4THF)
D: [V(bipy)3]I
d) 0,25 điểm Số oxid hóa: +4; cấu hình d1.
Câu 5 (2,5 điểm): Đại cương hữu cơ.
5.1. Cho các chất có công thức sau:

Trang 9/20
a. Vẽ cân băng giữa 2 đồng phân cấu dạng ghê, xác định cấu dạng bền hơn và gần kí hiệu
R, S cho từng đồng phân lập thể của A1. Biết: tương tác của 2 nhóm methyl in đậm quyết
định đến độ bền của cấu dạng
b. Cho giá trị pKa của acid liên hợp với A2 và A3 lần lượt là 5,1 và 12,1. Giải thích tính
base cao bất thường của A3 so với A2.
c. Cho giá trị pKa (ứng với nhóm -NH-) của A4, A5 và A6 trong dung môi DMSO (xếp
ngẫu nhiên) là 25,4; 20,7 và 26,6. Gán giá trị pKa cho từng chất A4, A5 và A6.
5.1 a, Vẽ cân bằng giữa 2 đồng phân cấu dạng ghế, xác định cấu dạng bền
hơn và gần kí hiệu R, S cho từng đồng phân lập thể của A1. Biết: tương
tác của 2 nhóm methyl in đậm quyết định đến độ bền của cấu dạng

0,25
đ

b. Cho giá trị pKa của acid liên hợp với A2 và A3 lần lượt là 5,1 và 12,1.
Giải thích tính base cao bất thường của A3 so với A2.
Trong A3 có thể năng cao do hiệu ứng lập thể của các nhóm methyl khiến
cho các cặp e tự do trên nitrogen khó liên hợp vào vòng thơm và 2 cặp
electron tự do trên 2 nguyên tử nitrogen đây nhau vì ở gần nhau. Quá trình 0,5 đ
proton hóa A3 tạo ra acid liên hợp HA3+ đã làm bền hoá hệ do một cặp
electron tự do trên một nitrogen liên kết với proton và cặp electron tự do
của nguyên tử nitrogen còn lại tạo liên kết hydrogen …N-HN.... Cấu trúc
“2 càng” giữ proton này rất bền nên acid liên hợp HA3+ được bển hoá,
dẫn đến tính base của A3 mạnh hơn A2 (acid liên hợp HA2+ không có cấu
trúc giữ proton 2 càng” và A2 it bị tương tác không gian mạnh của các 0,25
nhóm methyl nên anh hưởng không nhiều đến sự liên hợp của cặp - tự do đ
vào vòng thơm) rất nhiều.

Trang 10/20
0,5 đ
c. Cho giá trị pKa (ứng với nhóm -NH-) của A4, A5 và A6 trong dung
môi DMSO (xếp ngẫu nhiên) là 25,4; 20,7 và 26,6. Gán giá trị pKa cho
từng chất A4, A5 và A6.

5.2 Ở cấu dạng A có sự siêu liên hợp của σC-N với σ*C-N, ở cấu dạng B có sự
siêu liên hợp của cặp electron không liên kết của nguyên tử nitơ nN với σ*C-
N. 0,25
- Sự siêu liên hợp σC-N → σ*C-N ở cấu dạng A yếu hơn sự siêu liên hợp nN đ
→ σ*C-N ở cấu dạng B (xem giản đồ năng lượng bên phải hình).
- Tương tác 1,3-diaxial giữa nhóm t-Bu trục với hai cặp electron không liên 0,25
kết trên N là không đáng kể. đ

- Ngoài ra, cấu dạng B bền hơn vì có momen lưỡng cực nhỏ hơn vì momen
lưỡng một cặp e-n ở hướng liên kết biên ngược hướng với hai cặp e-n ở
hướng liên kết trục.

0,25
đ

- Do đó, cấu dạng B bền hơn cấu dạng A.


0,25
đ

Trang 11/20
5.2. Như đã biết, nhóm t-Bu rất cồng kềnh nên thường chiếm liên kết biên (equatiorial,
viết tắt là e). Tuy nhiên, nghiên cứu cho thấy ở 1,3,5-tri-tert-butylhexahydro-1,3,5-
triazine, cấu dạng A – với cả ba nhóm t-Bu biên – lại kém bền hơn so với cấu dạng B –
có một nhóm t-Bu chiếm liên kết trục (axial, viết tắt là a). Ở điều kiện thường, cấu dạng
B chiếm tới 85%. Giải thích sự bất thường này.

Câu 6 (2,5 điểm): Sơ đồ tổng hợp. Cơ chế phản ứng hợp chất hữu cơ
Strychnine là một alkaloid kết tinh có vị đắng,
rất độc, được dùng làm thuốc trừ sâu, đặc biệt
là là để tiêu diệt các động vật có xương sống
nhỏ như chim và động vật gặm nhấm. Hóa
chất này được thu thập tự nhiên từ cây
Strychnos nux-vomica. Trong hóa học hữu cơ,
đối quang tinh khiết của strychnine được sử
dụng để phân giải các phân tử hữu cơ. Do tính
phức tạp, hiệu ứng sinh lí và nhu cầu cao, các nhà tổng hợp hữu cơ đã tìm kiếm nhiều
hướng khác nhau để tổng hợp toàn phần hợp chất này trong nhiều năm. Trong bài này,
bạn sẽ được biết đến một trong những con đường tổng hợp strychnine ngắn nhất

Trang 12/20
1) Vẽ cấu trúc hợp chất B. Trong phản ứng từ A thành B, tác nhân DCC được chuyển
thành hợp chất khác. Vẽ cấu trúc của nó.
2) Viết cơ chế tạo thành hợp chất D. Trong phản ứng này, vai trò của BF3 là để tạo càng
với nhóm carbonyl của aldehyde không no, nhưng bạn không cần biểu diễn nó trong cơ
chế. Nhớ rằng trong quá trình này có xảy ra chuyển vị [1,5] hydride. 2 nguyên tử carbon
được đánh số trong cấu trúc C và D chỉ ra vị trí các nguyên tử trong cả 2 cấu trúc (trước
và sau phản ứng).
3) Sau giai đoạn đầu tiên của phản ứng tách methyl của amine (từ D thành E), hợp chất
trung gian D1 được tạo thành, sau khi xử lí với dung dịch nước Na2CO3 tạo thành hợp
chất E. Đề xuất cơ chế phù hợp của chuyển hóa từ D thành D1. Xác định D1. Nhớ rằng
chloromethane là sản phẩm phụ của giai đoạn này.
4) Vẽ cấu trúc của H và I, chỉ rõ hóa lập thể.
5) Phản ứng H thành I được gọi là Horner-Wadsworth-Emmons. Vẽ tất cả các cấu trúc
cộng hưởng có thể có của hợp chất trung gian có tính nucleophile chính, được tạo ra tức
thì (in situ - trực tiếp trong hỗn hợp phản ứng) sẽ phản ứng với hợp chất H.
6) Trong phản ứng từ H thành I, thực tế có haisản phẩm được tạo thành. Một trong số
chúng là sản phẩm mục tiêu I, còn lại là sản phẩm phụ I*. Vẽ cấu trúc I*.

Trang 13/20
7) Vẽ cấu trúc của hợp chất về mặt lí thuyết sẽ thu được từ I nếu chúng ta sử dụng LiAlH4
thay vì hệ khử hóa DIBAL-H/NaBH4.
HDC:
1. ( 0,25 đ)

2. ( 0,5 đ)

3. ( 0,5 đ)

4. ( 0,5 đ)

5. ( 0,25 đ)

6. ( 0,25 đ)

Trang 14/20
7. ( 0,25 đ)

Câu 7 (2,5 điểm): Xác định cấu trúc các chất hữu cơ (mô tả sơ đồ tổng hợp bằng lời
dẫn)
7.1. Hợp chất hữu cơ A dưới đây dùng hai bước chuyển hoá tạo thành lần lượt sản phẩm
B và C. Hợp chất B (C10H15NO2) có cấu trúc 3 vòng, B cân bằng thuận nghịch với B’
giống như một hemiacetal. Khi đem B xử lý với axit TFA ở 90oC trong dung môi DCM
tạo ra hợp chất spiro 2 vòng C, một lactam vòng 7 cạnh.

Tìm cấu tạo của B, B’ và C. Giải thích cơ chế từng bước trong chuyển hoá.
7.2. Hợp chất Spiro[3.3]heptan-1-on đem xử lý với TsNHNH2 thu được hợp chất U có bộ
khung cacbon không thay đổi. Đem U tác dụng với NaH cho hiệu suất 100% tạo thành
muối V. Nhiệt phân muối V sinh ra cấu tử trung gian kém bền W, cấu tử này lần lượt
chuyển hoá thành các hidrocacbon từ X1  X6 khác nhau có cùng công thức phân tử
C7H10. Trong đó, X3 và X4 cùng xảy ra phản ứng mở vòng tương tự như xiclobuten dưới
tác dụng của nhiệt độ, sinh ra các sản phẩm X5 và X6 tương ứng.
Viết công thức cấu tạo của Spiro[3.3]heptan-1-on và các chất U, V, W, X1, X2, X3, X4,
X5, X6. Biết rằng X2 và X3 đều chứa anken 4 nhóm thế.

Trang 15/20
7.1 Tìm đúng công thức B, B’ và C, mỗi công thức đúng 0,30 điểm 1,5
(0,30x3)

Cơ chế tạo thành B (0,30 điểm)

Cơ chế tạo thành C (0,30 điểm)

7.2 Mỗi công thức đúng được 0,1 điểm (0,1x10 = 1 ) 1,0

Trang 16/20
Câu 8 (2,5 điểm): Hóa học các hợp chất thiên nhiên (Cacbohidrat và các hợp chất hữu
cơ chứa nito đơn giản)
Trong hoá học của carbohydrate, nhiều phương pháp đã được phát triển để thiết lập cấu
trúc của các oligosaccharide, chủ yếu là kích thước vòng của các gốc monosaccharide
và các vị trí mà qua đó chúng gắn kết với nhau. Một phương pháp là methyl hoá hoàn
toàn rồi thuỷ phân tất cả các liên kết glycosidic.
a) Monosaccharide phải ở vị trí nào trong oligosaccharide nếu kết quả của sự methyl
hoá hoàn toàn và sau đó thuỷ phân thì tạo thành dẫn xuất methyl của monosaccharide
đó, trong đó:
A. không có nhóm alcohol nào không bị methyl hoá.
B. một nhóm alcohol không bị methyl hoá.
C. hai nhóm alcohol không bị methyl hoá.
Trisaccharide X chứa lần lượt các đơn vị A, B, C nối với nhau (từ trái sang phải). Methyl
hoá hoàn toàn X và sau đó thuỷ phân tạo thành 2,3,4,6-tetra-O-methyl-D-glucose; 3,4,6-
tri-O-methyl-D-glucose; và 1,2,3,6-tetra-O-methyl-D-galactose. Dữ kiện nhận được
không cho phép kết luận chắc chắn về cấu trúc của X.

Trang 17/20
b) Vẽ tất cả các cấu trúc có thể có của X tương ứng với nhóm sản phẩm được mô tả ở
trên. Chú ý rằng tất cả các đơn vị monosaccharide trong X được nối với nhau bởi các liên
kết β-glycosidic.
Để xác định rõ cấu trúc, X được tham gia vào phản ứng khử hoá trước, sau đó methyl hoá
hoàn toàn rồi thủy phân. Kết quả thu được là 2,3,4,6-tetra-O-methyl-D-glucose; 3,4,6-tri-
O-methyl-D-glucose và 1,2,3,5,6-penta- O-methyl-D-dulcite (dulcite là sản phẩm khử
hoá galactose, còn gọi là galactitol).
c) Xác định cấu trúc của 1,2,3,5,6-penta-O-methyl-D-dulcite và X.
Trong đường tiêu hoá của đa số các động vật và con người, hầu như không có enzyme α-
galactosidase – có khả năng phân cắt các gốc α-galactosyl đầu mạch từ các
oligosaccharides – khiến cho quá trình tiêu hoá các hợp chất như oligosaccharide Y trở
nên phức tạp. Sau khi xử lí α-galactosidase với Y, hai sản phẩm E và F (theo tỉ lệ mol
2:1) dễ hấp thụ vào cơ thể. F không phản ứng với thuốc thử Tollens, và khi đun nóng vừa
phải trong môi trường acid yếu thì tạo thành hỗn hợp đẳng mol các sản phẩm G và H,
một trong số chúng là đơn vị chính của polysaccharide tạo thành thành tế bào trong các
loài thực vật, và chuyển hoá giữa G và H có thể được thực hiện qua sự tạo thành các trung
gian osazone và ozonide. Oxi hoá E bởi nitric acid đặc tạo thành dicarboxylic acid không
quang hoạt. E thuộc nhóm các threo.
d) Viết phương trình tạo thành G và H từ F.
e) Vẽ cấu trúc E và Y.
HDC:
a) 0,5 điểm
A. ở đầu không-khử.
B. bên trong chuỗi hoặc ở đầu khử.
C. ở vị trí nhánh.
b)0,5 điểm

Trang 18/20
c)0,5 điểm

1,2,3,5,6-penta-O-
methyl-D-dulcite

d)0,5 điểm

e)0,5 điểm

Trang 19/20
---HẾT---
Người ra đề: Nguyễn Thị Trang 0935267729

Trang 20/20
SỞ GD VÀ ĐT QUẢNG TRỊ ĐỀ THI ĐỀ NGHỊ DUYÊN HẢI BẮC BỘ NĂM 2023
TRƯỜNG THPT CHUYÊN LÊ QUÝ ĐÔN MÔN: HOÁ HỌC LỚP 11
Thời gian làm bài: 180 phút
----------------------------

Câu 1 (2,5 điểm) Tốc độ phản ứng.


1. Sulfuryl chloride (SO2Cl2) phân hủy pha khí tạo thành sulfur dioxide (SO2) và chlorine (Cl2). Các dữ
kiện sau nhận được khi đun nóng một mẫu chứa 5,00.10-2 mol sulfuryl chloride tới 600 K  1 K trong bình
chứa 5,00.10-1 L.
Thời gian (h) 0,00 2,00 4,00 8,00 16,00
Áp suất (atm) 4,93 6,34 7,33 8,56 9,52
Tốc độ được xác định theo -d[SO2Cl2]/dt.
a) Xác định giá trị hằng số tốc độ của phản ứng phân hủy sulfuryl chloride ở 600 K.
b) Tính chu kì bán hủy của phản ứng.
c) Tính áp suất trong bình sau 0,500 giờ và 12,0 giờ.
d) Tính phần sulfuryl chloride còn lại sau 20,0 giờ.
2. Trong bối cảnh tác hại xấu của sự biến đổi khí hậu ngày càng thể hiện rõ trên phạm vi toàn cầu, hóa học
khí quyển ngày càng có tầm quan trọng to lớn. Một trong những phản ứng được quan tâm nhiều của lĩnh
vực này là sự phân hủy nitrogen oxide (NO) trong pha khí
2NO(k) → N2(k) + O2(k)
Phản ứng này có cơ chế được thừa nhận rộng rãi là
2NO N2O + O
O + NO O2 + N
N+ NO N2 + O
2O + M  O2 + M

k4

k 4

Hãy lập biểu thức định luật tốc độ phân hủy NO theo các giả định: nồng độ N đạt giá trị dừng, tốc độ giai
đoạn phát triển mạch vượt trội so với tốc độ giai đoạn khơi mào (khởi đầu) và cân bằng giữa oxygen
nguyên tử và oxygen phân tử được thiết lập.
Hướng dẫn chấm:
Ý Hướng dẫn giải Điểm
1 a) Áp suất tỉ lệ thuận với nồng độ, do đó có thể sử dụng dữ kiện để giải bài toán do:
d[SO2Cl2]/dt tỉ lệ thuận với –dP(SO2Cl2)/dt.
Gọi P0 là áp suất riêng phần ban đầu của SO2Cl2; x là áp suất riêng phần của SO2 ở thời
điểm t.
SO2Cl2 (g)  SO2 (g) + Cl2 (g)
Bđ: P0
t: P0-x x x
t=0: P tổng = P0 = 4,93 atm.
t: P tổng = P0 + x = x + 4,93
Thời gian (h): 0,00 2,00 4,00 8,00 16,00
P tổng (atm): 4,93 6,34 7,33 8,56 9,52 0,25

1
PSO2 (atm): 0 1,41 2,4 3,63 4,59
a) Giả sử phản ứng là bậc 1 thì hằng số tốc độ của phản ứng có dạng: 0,5
1 P
k  ln 0 (1)
t P0  x
t = 2h  k2 ≈ 0,168 h-1
t= 4h  k2 ≈ 0,168 h-1
t = 8h  k3 ≈ 0,167 h-1
t=16h  k4 ≈ 0,167 h-1
ta thấy các giá trị k xấp xỉ nhau nên phản ứng (1) là phản ứng bậc 1.
(k1  k 2  k3  k 4 )
k 0,168h1
4
b) t1/2 = ln2/k = 0,6931/0,168 = 4,13 h. 0,25
c) sau 0,500 giờ: 0,5
ln PSO2Cl2 = -kt + lnP0 = -0,168.0,5 + ln4,93 = 1,511
 PSO2Cl2 = 4,53 atm.
PCl2 = PSO2 = 4,93 – 4,53 = 0,4 atm.
P tổng = 4,53 + 0,4 + 0,4 = 5,33 atm.
Sau 12 h:
ln PSO2Cl2 = -kt + lnP0 = -0,168.12 + ln4,93 = -0,42  PSO2Cl2 = 0,66 atm.
PCl2 = PSO2 = 4,93 – 0,66 = 4,27 atm.
P tổng = 0,66 + 4,27 + 4,27 = 9,2 atm.
d) ln(PSO2Cl2/P0) = -0,168.20 = -3,36 0,25
 PSO2Cl2/P0 = 3,47.10-2 = 3,47%.
2. Theo định luật tốc độ, ta có:

(1)
Nồng độ đạt trạng thái dừng:

(2)
Thế (2) vào (1) ta có: 0,25

(3)
Theo điều kiện tốc độ giai đoạn phát triển mạch vượt trội so với tốc độ giai đoạn khơi
mào:

k2[NO].[O] >> 2k1[NO]2 nên (4)


Theo điều kiện cân bằng giữa oxi nguyên tử và oxi phân tử được thiết lập:

K4[O]2 = k-4[O2] → [O] = (5) 0,25


Thế (5) vào (4) ta có

(6)

2
Kí hiệu: k = -2k2 (7) 0,25

Phương trình động học được viết lại là


Câu 2 (2,5 điểm) Cân bằng và phản ứng trong dung dịch. Pin điện - Điện phân.
1. Cho hai pin điện hóa có sơ đồ:
Pin 1: Pt, H2 (1atm)/HCl (10-3M)/Hg2Cl2, Hg
Pin 2: Pt, H2 (1atm)/NaOH (10-3M), NaCl(10-3M)/Hg2Cl2, Hg
Suất điện động của các pin tương ứng là E1 và E2. Biết EHg
o
Cl / Hg  0, 2682 (V). 2 2

a) Viết phương trình hóa học các nửa phản ứng xảy ra trên các điện cực và phản ứng tổng cộng xảy ra
khi các pin làm việc.
b) Tính E1 và thiết lập mối liên hệ giữa E2 và Kw ở 25oC.
c) Nối hai điện cực hiđro của hai pin với nhau để tạo thành một pin kép. Ở 25oC, suất điện động của pin
này là 0,4726 V. Xác định Kw ở nhiệt độ này.
2. Trong dung dịch chứa ion Zn2+ và ion C2O42- có thể có các cân bằng sau:
Zn2+ + C2O42- ⇌ ZnC2O4* β1 = 104,85
Zn2+ + 2C2O42- ⇌ Zn(C2O4)22- β2 = 107,55
Zn2+ + C2O42- ⇌ ZnC2O4 (r) Ks-1 = 107,56
Trong đó ZnC2O4* là dạng phức tan trong nước còn ZnC2O4 (r) là kết tủa.
a) Bằng tính toán hãy cho biết có thể kết tủa được hoàn toàn ion Zn2+ từ dung dịch Zn2+0,10 M bằng
dung dịch C2O42- hay không? Nếu được, cho biết nồng độ C2O42-cần sử dụng.
b) Tính pH của dung dịch nước để độ tan của ZnC2O4 là 0,01 M.
Cho biết: H2C2O4 có pKa1 = 1,25; pKa2 = 4,27; H2S có pKa1 = 7,02; pKa2 = 12,90; pKs(ZnS) = 23,8.
Hướng dẫn chấm
Ý Nội dung Điểm
1. a. Pin 1:
Tại anot: H2  2H+ + 2e 0,125
Tại catot: Hg2Cl2 + 2e  2Hg + 2Cl -

Tổng cộng: H2 + Hg2Cl2  2Hg + 2Cl- + 2H+ 0,125


Pin 2:
Tại anot: H2 + 2OH-  2H2O + 2e 0,125
Tại catot: Hg2Cl2 + 2e  2Hg + 2Cl -
-
Tổng cộng: H2 + Hg2Cl2 + 2OH 2Hg + 2Cl- + 2H2O 0,125
b.
0,059 [ H  ]2 [Cl  ]2 0,059 [ H  ]2 [Cl  ]2
E1  E1o  lg  EHg
o
Cl / Hg  E2 H
o

/ H2
 lg
2 pH 2
2 2
2 pH 2

10 .103 3 0,25
E1  0, 2682  0,059lg  0,6222 (V)
1

0,059 [Cl  ]2 0,059 [Cl  ]2 0,25


E2  E2o  lg  E o
 E o
 lg
pH .[OH  ]2 pH .[OH  ]2
Hg Cl / Hg H O/ H
2 2
2 2
2 2 2

6
0,059 10
E2  0, 2682  0,059lg K w  lg 6  0, 2682  0,059lg K w (V)
2 10 .1

3
Ý Nội dung Điểm
c. Suất điện động của pin kép:
 Kw 
 3 
E  E o  0,059lg  3   E o  0,059.lg106.K w
10
10
 0,4726  0,059.6  0,059lg K w
 K w  1014,01 0,25
2+
2 Gọi S là tổng các dạng tồn tại của Zn trong dung dịch:

0,5
Do vậy không thể kết tủa hoàn toàn được ion Zn2+ ra khỏi dung dịch bằng C2O42-.
Để độ tan là 0,01 M:

0,25

Giải phương trình, thu được: 0,25


[C2O42-] = 3,42.10-6M hoặc [C2O42-] = 8,23.10-3 M.
Xét trường hợp: [C2O42-] = 3,42.10-6M  [Zn2+] = 8,05.10-3M
C(C2O42-) = [ZnC2O4*] + 2[Zn(C2O4)22-] + [C2O42-] + [HC2O4-] + [H2C2O4] = 0,01M

Giải ra: pH = 1,22.


Xét trường hợp: [C2O42-] = 8,23.10-3 M  [Zn2+] = 3,45.10-6M.
Một cách tương tự ta thấy phương trình vô nghiệm.
Vậy để độ tan của ZnC2O4 trong dung dịch là 0,01 M thì cần duy trì pH = 1,22. 0,25
Câu 3 (2,5 điểm) Nhiệt động học và cân bằng hóa học.
1. Cho bảng số liệu sau: CH4 (k) C(gr) + 2H2 (k) H 0298,15  74,85 kJ.mol1 (3)
CH4 (k) C(gr) H2 (k)
0 -1 1 186,19 5,69 130,59
S298,15 (J.K .mol )
C0p,298,15 (J.K -1.mol1 ) 35,71 8,64 28,84
(a) Tính Kp của phản ứng (3) ở 250C.
(b) Xác định H0T và Kp ở 7270C, coi C 0p không phụ thuộc vào nhiệt độ.
Cho T(K) = t0(C) + 273,15.
2. Cho một ít methanol vào một chai có thể tích 500ml đầy không khí, P = 1,100.10 5 Pa, t = 25oC. Lắc
mạnh chai cho đến khi không khí bão hòa rượu thì có chút lượng rượu dư ra. Hàn chai lại, bật tia lửa điện
rồi hạ nhanh nhiệt độ xuống 25oC. Tính nhiệt tỏa ra của quá trình.
Biết: không khí chứa 20% oxi theo thể tích; áp suất của hơi nước bão hòa ở 298K bằng 4,58 torr.

4
H s0, 298K (kJ/mol) của các chất: CH3OHl ;CH3OH k ; H 2Ol ; H 2Ok ;CO 2k lần lượt bằng: -238,7; -200,7; -285,8;
-241,5; -393,5.
0
S 298K (J/Kmol) của CH3OHl ;CH3OH h lần lượt bằng: 126,8; 239,7.
Hướng dẫn chấm:
ý Nội dung Điểm
1(a) S298,15  5,69 + 130,59.2 - 186,19 = 80,68 (J.K-1.mol-1);
0 0,5

→ G 0298,15  H 0298,15 - T S0298,15


= 74,85.103 – 298,15.80,68 = 50,8.103 (J.mol-1)
50,8.103
→ G 0298,15  - RTlnKp → Kp = exp( ) = 1,26.10-9
8,314.298,15
1(b) C0298  8,64 + 28,84.2 - 35,71 = 30,61 (J.K-1.mol-1)
Vì C0p,298,15 các chất không thay đổi theo nhiệt độ nên
T
H  H   C0p dT
0 0
T 298,15
298,15
0,25
= 74,85.103 + 30,61(T-298,15) = 65,723.103 + 30,61.T
→ H1000,15
0
 65,723.103 + 30,61.1000,15 = 96,34.103 (J.mol-1)
 ln K H0T
T T
1 65,723.103 + 30,61.T 0,25
(
T P
)0
RT 2
→ 
298
d ln K  
R 298 T2
dT

→ KP(1000) = 13,116.
2 a. nkhông khí = 500.10-6.1,1.105/(8,314.298) = 0,0222 (mol)
Xét phản ứng : CH3OH (l) CH3OH (k) (1)
∆H(1) = -200,7 + 238,7 = 38 (kJ/mol);
∆S(1) = 239,7 -126,8 = 112,9 (J/mol) 0,25
3
∆G(1) = 38.10 – 298.112,9 = 4355,8 (J/mol)
 Kp(1) = 0,17237 atm = P(CH3OH k) 0,25
 n (CH3OH)k = 0,17237.500.10 /(0,082.298) = 3,52704.10 (mol)
-3 -3

Vậy nkhông khí còn lại = 0,0222 - 3,52704.10-3 = 0,01867 (mol)


n(O2)còn lại = 0,01867.20% = 3,734.10-3 (mol) 0,25
Khi bật tia lửa điện rồi đưa nhanh về 298K, ở trong chai diễn ra phản ứng:
CH3OH(k) + 3/2O2(k) → CO2(k) + 2H2O
-3 -3
Bđ 3,52704.10 3,734.10
-3
Pư 2,48933.10 3,734.10-3 2,48933.10-3 4,97867.10-3
Cb 1,03771.10-3 0 2,48933.10-3 4,97867.10-3
- Nếu nước ở thể khí ở 250C
P(H2O) = (4,97867.10-3.0,082.298)/(500.10-3) = 0,24332 (atm) = 184,9213 (torr) > P hơi 0,25
nước bão hòa nên H2O ở thể lỏng.
∆H0 = -285,8.2 – 393,5 -3/2.0 + 200,7 = -764,4 (kJ/mol)
Vì quá trình đẳng tích nên:
∆U0 = -764,4 –(-3/2).8,314.298.10-3 = -760,6836 (kJ/mol) 0,5
-3
Nhiệt tỏa ra = -760,6836.2,48933.10 = -1,894 (kJ).

5
Câu 4 (2,5 điểm) Hóa nguyên tố (Kim loại, phi kim nhóm IVA, VA). Phức chất.
Nguyên tố X (có nhiều dạng thù hình) có một anion chứa oxygen đóng vai trò quan trọng trong ô nhiễm
nước. Độ âm điện của nó nhỏ hơn oxygen. Nó co thể tạo hợp chất với halogen. Ngoài hai oxide đơn phân
tử còn có những oxide cao phân tử. Bên cạnh đó, X cũng có vai trò rất quan trọng trong sinh hóa. Các
orbital p của nó chỉ có một electron.
1) Đó là nguyên tố nào? Viết cấu hình của nó.
X có thể tạo được với hydrogen nhiều hợp chất cộng hóa trị có công thức chung là XaHb; dãy hợp chất này
tương tự như dãy đồng đẳng của alkane.
2) Viết công thức cấu tạo 4 chất đầu của dãy.
Một trong số các hợp chất trên có 3 đồng phân lập thể (tương tự tactric acid). Vẽ các cấu trúc lập thể này.
3) Xác định hợp chất này.
Nguyên tố X tạo được những acid có chứa oxygen (các oxoacid) với công thức chung là H3XOn
(n=2,3 và 4).
4) Viết công thức cấu tạo của 3 acid này. Đánh dấu (dấu hoa thị hoặc mũi tên) các nguyên tử H và ghi số
oxi hóa của X trong các hợp chất này.
Một hợp chất dị vòng của X, với cấu trúc phẳng do J.Liebig và F.Wohler tổng hợp từ năm 1834 được tạo
thành từ NH4Cl với pentachloride của X; sản phẩm phụ của phản ứng này là một khí dễ tan trong nước và
phản ứng như một acid mạnh.
5) a) Viết phương trình phản ứng.
b) Viết công thức cấu tạo của hợp chất (NXCl2)3.
Hợp chất vô cơ vừa nêu ở trên có tính chất khác thường khi bị đun nóng: nó sôi ở 2560C khi bị đun nóng
nhanh. Nếu đun nóng chậm nó bắt đầu nóng chảy ở 2500C; làm nguội nhanh chất lỏng này thì ta được một
chất tương tự cao su.
6) Giải thích tính chất đặc biệt này.
HƯỚNG DẪN CHẤM:
Ý Hướng dẫn giải Điểm
2 3
1) Phosphorus. Cấu hình [Ne]3s 3p 0,25
2) Công thức cấu tạo của 4 chất đầu tiên: 0,5

3) Chất 4: 0,5
PH2 PH2 PH2
H : : H H :

: H H : H :
PH2 PH2 PH2

4) Công thức cấu tạo của các chất: 0,5

5) a) 3NH4Cl + 3PCl5  (NPCl2)3 + 12HCl 0,5


6
b) Công thức cấu tạo:

6) Đun nóng mạnh  Chất nóng chảy không bị gãy vòng. 0,5
Đun nóng chậm  Vòng bị bẻ gãy thành các polime có hệ liên hợp pi.

Câu 5 (2,5 điểm) Đại cương hữu cơ.


1. Cho các hợp chất sau:

Giải thích các vấn đề sau


a) Hợp chất A tồn tại chủ yếu dạng enol B.
b) Hợp chất C là hợp chất thơm.
c) D làm mất Br- tạo thành carbocation dễ hơn dẫn xuất E.
d) So sánh tính axit của F và G.
2. Mentol là một terpene có trong tinh dầu bạc hà. Công thức cấu tạo của mentol như sau:

a) Hãy cho biết mentol có bao nhiêu đồng phân lập thể.
b) Viết 1 đồng phân lập thể của mentol và viết 2 cấu dạng ghế của đồng phân đó.
Hướng dẫn chấm:
Ý Hướng dẫn giải Điểm
1 a) B tạo cộng hưởng thành hệ thơm nên B ưu thế hơn A 0,25

b) Cặp e trên N cho vào hệ thống giúp cho mỗi vòng đều có 6eπ thỏa mãn Huckel 0,25

c) Cation sinh ra từ D có được sự ổn định từ nhiều hệ thơm hơn so với E(cũng có thể giải 0,5
thích cation D có nhiều cộng hưởng nên bền hơn)

7
d) Anion cyclopentadienyl có năm cấu trúc cộng hưởng tương đương nhau 0,5

Còn anion indenyl tuy có nhiều cộng hưởng hơn nhưng những cấu trúc khiến cho hệ thơm
benzene bị phá vỡ làm giảm đi tính bền của hệ khiến cho nó trở nên ít ưu thế hơn

Thế nên dù có nhiều cộng hưởng hơn, nhưng sự thiếu ổn định trong các cộng hưởng (phá vỡ
hệ thơm vòng benzene) khiến cho anion indenyl kém bền hơn cyclopentadienyl, nên
xiclopentadiene có tính axit cao hơn indene G.
2 a) Mentol có 8 đồng phân lập thể (do có 3 nguyên tử carbon bất đối, phân tử không đối xứng). 0,25
b) Cấu trúc của 1 đồng phân:

0,75

Câu 6 (2,5 điểm) Sơ đồ tổng hợp hữu cơ. Cơ chế phản ứng hóa hữu cơ.
1. Demethoxy lycoraminone là một alkaloid được sử dụng trong điều trị bệnh alzheimer và các bệnh rối
loạn trí nhớ. Hợp chất này được tổng hợp theo sơ đồ phản ứng sau:

8
Biết rằng: E7 có nhóm C=O liên hợp vòng thơm, E11 có chứa đoạn mạch -CO-NH-CO-.

Bỏ qua yếu tố lập thể, vẽ công thức cấu tạo các chất từ E1 đến E14.
2. Viết cơ chế của các phản ứng dưới đây:
a)

b)

Hướng dẫn chấm:


Ý Hướng dẫn giải Điểm
1 0,125/1
chất
= 1,75

9
2a 0,5

2b 0,25

Câu 7 (2,5 điểm) Xác định cấu trúc các chất hữu cơ (mô tả sơ đồ tổng hợp bằng lời dẫn)
Hai lactone thơm A, B có công thức phân tử [C10H10O4] đều tan trong dung dịch NaOH loãng nhưng
không tan trong dung dịch NaHCO3. Cả A và B cho phản ứng màu với dung dịch FeCl3. Khi cho A phản
ứng CH3I/K2CO3 tạo ra chất C[C11H12O4]. Biết C chứa ba nhóm methyl không giống nhau, trong đó có
một nhóm methyl liên kết vòng thơm. Xử lý C với BCl3 để tách loại một nhóm methyl tạo ra D là một
đồng phân mới của A. D có một nhóm hiđroxi tạo liên kết hiđro nội phân tử.
Cho chất E (2-metyl-1,3-dihidroxibenzene) phản ứng MeI/K2CO3 tạo F[C9H12O2], F được khử bằng
Li/NH3 lỏng có mặt 2 -methylpropan-2-ol cho một đien đối xứng và không liên hợp G. Tiếp tục cho G
phản ứng KNH2/NH3 lỏng tạo ra một sản phẩm H. Thực hiện phản ứng ozon phân H rồi xử lý tiếp thu
được nhiều sản phẩm trong đó có metyl 2-oxopropanoate. Thực hiện phản ứng giữa H với đimethylbut-2-
yndioate, đun nóng tạo K[C15H20O6], tiếp tục đun nóng K để loại eten tạo được một ester thơm L. Thủy
phân L trong môi trường base rồi axid hóa dung dịch tạo thành M[C11H12O6], đun nóng M trong chân
không tạo ra N[C11H10O5]. Khử N bằng NaBH4 trong DMF(N,N-dimethylformamide) tạo C và một lactone
P đồng phân, P cũng có thể thu được nhờ methyl hóa B. Hãy biện luận xác định cấu tạo các chất từ A đến
P.
Hướng dẫn chấm:

Ý Nội dung Điểm


10
-A, B [C10H10O4] có π+v=6 là một lactone vòng thơm, tan trong NaOH, không tác dụng
(2 đ) NaHCO3, phản ứng màu FeCl3 nên có -OH phenol
-A phản ứng MeI/K2CO3 tạo C có C11H12O4, vậy C đã thêm một nhóm metyl nên -OH 0,25
phenol chuyển thành -OMe. C có ba nhóm metyl không giống nhau, có một nhóm liên
kết trực tiếp vòng benzene.
Phản ứng ozone phân H thu nhiều sản phẩm trong đó có methyl-2-oxo-propanoate nên H

0,25

Xét phản ứng E:


Các
chất từ
E đến
N:
0,125
x8=1
điểm

C loại một nhóm methyl tạo D, D có một nhóm hidroxi tạo liên kết hidro nội phân tử
nên C có nhóm carbonyl cạnh nhóm -OMe. Do đó C là (II), P là (I), từ đó xác định cấu
tạo B, D

0,5

Methyl hóa B được P nên có 2 cấu tạo B thỏa mãn là B1,B2

A là đồng phân của D nên A khác D ở vị trí nhóm hidroxi và methoxi, cấu tạo phù hợp
A: 0,5

11
Xác định được C, P: 0,5
Xác định đúng A và B: 0,5
Câu 8 (2,5 điểm) Hóa học các hợp chất thiên nhiên (Carbohydrate và các hợp chất hữu cơ chứa nito đơn
giản)
1) Pentapeptide X (C22H29N5O9) chứa 5 aminoacid tự nhiên khác nhau ký hiệu từ A1 đến A5. Xác định các
aminoacid và trình tự của chúng trong X từ các dữ kiện sau:
- Xử lý X bằng chymotrypsin tạo thành một dipeptide và một tripeptide.
- Phương pháp Edman thu được một dẫn xuất không quang hoạt và tetrapeptide X1. Tiếp tục phân tích X1
bằng enzyme carboxypeptidase thu được amino acid A2 và X2. Phân tích phổ khối lượng cho thấy A2 = A1
+ 14.
- Xử lý X2 với dung dịch nước của imidazole rồi phân tích sản phẩm tạo thành bằng enzyme
aminopeptidase thu được aminoacid A3, sản phẩm còn lại khi phản ứng với NBS/H2O tạo hợp chất Y
(C9H7Br2NO3) là một –lacton có cấu trúc dạng quinoid và aminoacid A4. Muối sodium của A4 là một chất
tạo ngọt thông dụng trong công nghiệp thực phẩm.
- Aminoacid A3 được tổng hợp theo sơ đồ sau.
O 1.Hg(OAc)2, MeOH
+
2. KBr 1. aq NaOH/H 1.HBr
H2C
OMe C3H7NO 3
C5H9BrO3 C4H9NO 3
2. NH 3 2. NH 3
Lập luận xác định cấu trúc của X
2) Một monosaccharide A là một xylose. Tên gọi này bắt nguồn từ 1 từ trong tiếng Hy Lạp, nghĩa là gỗ,
ám chỉ đường này chỉ xuất hiện trong vỏ cây gỗ. Một dẫn xuất của nó là xilytol (đường khử) được tìm thấy
trong cây Bạch Dương được các nhà khoa học quan tâm do có thể sử dụng thay thế đường cho bệnh nhân
tiểu đường.

a) Hãy xác định cấu trúc các chất chưa biết, biểu diễn bằng công thức Fischer. Biết F có tên là idose, có
tên IUPAC là (2S, 3R, 4S, 5R)-2,3,4,5,6-pentahydroxyhexanal.
b) Vẽ các đồng phân của F dưới dạng vòng pyranose và gọi tên chúng. Mối quan hệ giữa các đồng phân
này là gì?
c) Xác định số lượng các disaccharide thu được từ các đồng phân của F thu được từ câu b, biết chúng đều
không cho phản ứng dương tính với thuốc thử Fehling. Hãy thể hiện chúng bằng công thức Haworth.
Hướng dẫn chấm:
Ý Hướng dẫn giải Điểm
1 Do sản phẩm sinh ra khi phân tích Edman không quang hoạt, suy ra aminoacid đầu N
chỉ có thể là Glycine (Gly), từ đó aminoacid đầu C A2 sẽ là Ala (hơn Gly 14 đơn vị).
Từ chuỗi tổng hợp có thể dễ dàng xác định A3 chính là Ser:

12
O 1.Hg(OAc)2, MeOH
O O HO O
2. KBr +
1. aq NaOH/H
H2C 1.HBr
OMe
MeO OMe MeO OMe OH
Br
2. NH 3
NH2 2. NH 3 0,25
NH2

Serin
Muối sodium của A4 được dùng nhiều trong thực phẩm thì A4 chính là Glu (sodium
glutamate là chất tạo ngọt). Dẫn xuất bromine của A3 có dạng quinoid chứng tỏ A3 có
vòng thơm, do dẫn xuất này có 9C nên A3 sẽ là tyrosine (Tyr). Cấu trúc của dẫn xuất
này như sau:
Br NH2
0,25
O
O O
Br
Khi phân cắt X bằng chymotrypsin (cắt sau aminoacid thơm) tạo thành 1 tripeptide
và 1 dipeptide thì aminoacid thứ 3 là Tyr.
Aminopeptidase là enzyme xác định đầu N, như vậy trong tripeptide X2 aminoacid 0,25
đầu N là Ser, như vậy aminoacid đứng sau Tyr phải là Glu. Tức trật tự sắp xếp các
aminoacid trong X sẽ là Gly-Ser-Tyr-Glu-Ala
Dựa vào công thức phân tử có thể thấy sẽ tồn tại một liên kết ester trong X. Liên kết
này chỉ bị cắt bằng imidazole/H2O. Tức sẽ tồn tại một liên kết ester giữa nhóm OH
của Ser và 1 nhóm COOH tự do của Glu. Vậy cấu trúc của X sẽ là:
Gly Tyr Ala
Ser Glu 0,25
O
O
2 0,75

Mỗi công thức đúng được 0.125 đ* 6 chất =0.75 điểm


b. 0,75

Mỗi cấu tạo và tên gọi: 0.125 điểm * 2 = 0.25 điểm


13
2 chất là đồng phân anomer/epimer của nhau (0.125 điểm)
c. Có 3 đồng phân disaccharide phù hợp với yêu cầu (0.125 điểm)

Mỗi cấu tạo chính xác được (0.125 điểm*3 = 0.375 điểm).

Giáo viên ra đề: Nguyễn Thị Hạnh


SĐT: 0915326238
Email: hanhhoalqd@gmail.com

14
HỘI CÁC TRƯỜNG CHUYÊN VÙNG HDC ĐỀ THI MÔN HÓA HỌC – KHỐI 11
DUYÊN HẢI VÀ ĐỒNG BẰNG BẮC BỘ NĂM 2023
TRƯỜNG THPT CHUYÊN LÀO CAI Thời gian làm bài: 180 phút
(Đề thi gồm có 17 trang, gồm 8 câu)
ĐỀ THI ĐỀ XUẤT

Câu 1. (2,5 điểm) Tốc độ phản ứng


N2O5 nóng chảy ở 30oC, sôi ở 47oC, bắt đầu thăng hoa ở nhiệt độ khoảng 25oC. Ở pha khí, N2O5 phân
hủy theo phương trình phản ứng: 2N2O5(k) → 4NO2(k) + O2(k) (a)
Các kết quả nghiên cứu động học cho thấy rằng hằng số tốc độ của phản ứng (a) có thể tính bằng công
thức:
103,137 kJ / mol

13
k = 4,1.10 . e RT
s-1
1.1. Hãy cho biết giá trị của thừa số tần suất (A), năng lượng hoạt động hóa (Ea) và viết biểu thức của
định luật tốc độ của phản ứng (a).
1.2. Tính hệ số góc của đường thẳng logk = f(T-1) (T là nhiệt độ tuyệt đối) cho phản ứng (a). Ở nhiệt độ
nào ta có tốc độ phản ứng v = [N2O5].s-1?
d [N 2 O5 ]
1.3. Tính giá trị của đạo hàm khi tiến hành phản ứng (a) trong bình phản ứng kín có thể tích
dt
12,0 dm3, tại thời điểm trong bình có 0,0453 mol N2O5 và áp suất riêng phần của N2O5 bằng 0,1 atm. Tại
thời điểm đó, tính trung bình trong 1s (giây) có bao nhiêu phân tử N2O5 bị phân hủy trong 1 đơn vị thể
tích của bình phản ứng (L)?
1.4. Người ta đề nghị cơ chế dưới đây cho phản ứng (a):
k1
N2O5(k) k −1
NO2(k)+ NO3(k) (b)

NO2(k)+ NO3(k) ⎯⎯
k2
→ NO(k)+ O2(k)+ NO2(k) (c)

NO(k)+ NO3(k) ⎯⎯
k3
→ 2 NO2(k) (d)
Cơ chế này được cho là hợp lí, vì nếu nhân đôi phương trình (b) rồi cộng với (c) và (d) thì sẽ thu được
phương trình (a) và từ cơ chế này có thể rút ra định luật tốc độ thực nghiệm. Hãy tìm liên hệ giữa hằng số
tốc độ k trong định luật tốc độ của phản ứng (a) với hằng số tốc độ của các giai đoạn sơ cấp trung gian
(k1, k-1, k2 ...) và cho biết trong trường hợp giả định nào thì phản ứng (b) có thể đạt được cân bằng tạm
thời và khi đó giai đoạn nào là giai đoạn quyết định tốc độ chung của phản ứng. Cho hằng số khí R =
8,314 J.K-1.mol-1= 0,082 L.atm.K-1.mol-1; số Avogadro NA = 6,02.1023.
Ý ĐÁP ÁN ĐIỂM

1 - A= 4,1.1013 s-1; Ea = 103,137 kJ/mol. 0,25


- Hằng số tốc độ có thứ nguyên của (thời gian)-1 nên phản ứng là bậc 1 và định luật 0,25
tốc độ là v = k[N2O5]

2 lnk = ln4,1.1013 – 103137.R-1.T-1

103137
Logk = log4,1 + 13 - T −1 = 13,6 -5387. T-1
8,314x2,303
0,25
Hệ số góc của đường thằng logk = f(T-1) bằng - 5387 K.
0,25
Khi k =1s , logk = 0 và v = 1s [N2O5] →13,6 -5387. T = 0 → T = 396 K.
-1 -1 -1

3 1 d [N 2 O5 ] d [N 2 O5 ]
v= − = k [N2O5] → = -2 k [N2O5] (1)
2 dt dt

Giả định các khí là lí tưởng:

PN2O5 .V 0,1.12
PN2O5.V = nN2O5.RT → T = = K = 323 K (2)
n N2O5 .R 0, 0453.0, 082

103137

13
k (T=323 K) = 4,1.10 . e 8,314x323
= 8,58.10-4 s-1 (3); [N2O5] = 0,0453/12 (mol.L-1) 0,25
-3
= 3,78.10 M (4)

d [N 2 O5 ]
Thay (3), (4) vào (2) ta có : = -2. 8,58.10-4 s-1. 3,78.10-3 M = - 6,48.10-6
dt
M.s-1.

Số phân tử N2O5 phân hủy trong 1 giây, trong 1 L là: 6,48.10-6x6,02.1023 = 3,90.1018 0,25
phân tử /s

4 N2O5(k)
k1
NO2(k)+ NO3(k)
k −1

NO2(k)+ NO3(k) ⎯⎯
k2
→ NO(k)+ O2(k)+ NO2(k)

NO(k)+ NO3(k) ⎯⎯
k3
→ 2 NO2(k)

d [O 2 ]
v= k2[NO2][ NO3] (1)
dt 0,125

0,125
d [NO3 ]
= 2k1[N2O5] –2 k-1[NO2][ NO3]- k2 [NO2][ NO3] – k3[NO][ NO3] = 0 (2)
dt
0,125
d [NO]
= k2[NO2][ NO3] – k3[NO][ NO3] = 0 (3) 0,125
dt

Lấy (2) – (3) → 2k1[N2O5] – 2k-1[NO2][ NO3] - 2 k2[NO2][ NO3] = 0 (4)


0,125
k1[N 2O5 ]
→ [ NO3] = (5)
[NO2 ] ( k -1 + k2 )
0,125
k 2 [NO2 ]k1[N 2O5 ] k 2 .k1
Thay (5) vào (1) → v = = [N O ]
[NO2 ] ( k -1 + k2 ) ( k -1 + k2 ) 2 5

k 2 .k1 k .k 0,25
Đặt k = (5). Khi k-1>> k2 → k = 2 1 = Kcb1. k2, phản ứng (b) đạt được
( k -1 + k2 ) k -1

cân bằng tạm thời, còn phản ứng (c) là giai đoạn chậm quyết định tốc độ chung của
phản ứng.

Câu 2. (2,5 điểm) Cân bằng trong dung dịch điện li

2.1. Dung dịch A chứa H2C2O4 (0,05M); HCl (0,1M), NH3 (0,1M)

a. Tính pH của dung dịch A?

b. Trộn 1ml dung dịch A với 1 ml dung dịch chứa CaCl2 (0,05M) và HCl (0,01M).

Có kết tủa CaC2O4 tách ra không? Nếu có, hãy tính độ tan của CaC2O4.

Cho pKa: NH4+ (9,24); H2C2O4 (1,25; 4,27); pKs: CaC2O4 (8,75)

2.2. Pin chì-axit hay còn gọi là ắc quy chì gồm anot là tấm lưới chì phủ kín bởi chì xốp, catot là tấm lưới
chì phủ kín bởi PbO2 xốp. Cả 2 điện cực được nhúng trong dung dịch điện li H2SO4.
a. Hãy viết các quá trình xảy ra trên từng điện cực, phản ứng trong pin khi pin chì-axit phóng điện và biểu
diễn sơ đồ của pin.
Cho biết: E0 của Pb2+/Pb là -0,126V; của PbO2/Pb2+ = 1,455V; HSO4- có pKa = 2,00; PbSO4 có pKS = 7,66;
ở 250C.
b. Tính E0 của PbSO4/Pb; PbO2/PbSO4 và tính điện áp V của ắc quy chì, nếu CH2SO4 ≈ 1,8M.

Ý ĐÁP ÁN ĐIỂM
Phản ứng xảy ra:
NH3 + H+ ⇌ NH4+ Ka-1 = 109,24>>
0,1 0,1
- - 0,1
TPGH: NH4+ (0,1); H2C2O4 (0,05) 0,25
Các cân bằng:
2.1 H2C2O4 ⇌ H+ + HC2O4- Ka1 = 10-1,25 (1)
a HC2O4- ⇌ H+ + C2O42- Ka2 = 10-4,27 (2)
NH4+ ⇌ NH3 + H+ Ka = 10-9,24 (3)
So sánh: Ka1 >> Ka2 >> Ka  cân bằng (1) là chủ yếu
H2C2O4 ⇌ H+ + HC2O4- Ka1 = 10-1,25 (1)
[] 0,05 – x x x
x2 0,125
 = 10-1,25  x = 0,0319  pH =1,50
0, 05 − x
Trộn 1ml dung dịch A với 1 ml dung dịch chứa CaCl2 (0,05M) và HCl (0,01M)
Sau khi trộn tính lại nồng độ:
C NH + = 0,05M CCa 2+ = 0,025M
4

C H 2C2O4 = 0,025M CH + = 0,005M

Tính CC O2− để xét điều kiện kết tủa?


2 4

H2C2O4 ⇌ H+ + HC2O4- Ka1 = 10-1,25


(1)
0,025 0,005
b
0,025 – x 0,005+x x
x.(0, 005 + x)
 = 10-1,25  x = 0,0178
0, 025 − x
HC2O4- ⇌ H+ + C2O42- Ka2 = 10-4,27
(2)
0,0178 0,0228
0,0178-y 0,0228+y y 0,125
y.(0, 0228 + y )
 = 10-4,27  y = 4,175.10-5
0, 0178 − y
Xét CCa 2+ . CC O2− > Ks  xuất hiện CaC2O4 0,125
2 4

Phản ứng:
Ca2+ + H2C2O4 ⇌ CaC2O4 + 2H+ K=103,23>>
0,025 0,025 0,005
- - 0,055
TPGH: CaC2O4, H+ (0,055), NH4+ (0,05M) 0,125
Tính SCaC2O4 ?

CaC2O4 ⇌ Ca2+ + C2O42- Ks1 = 10-8,75 (4)


S S
Các quá trình phụ:
Ca2+ + H2O ⇌ CaOH+ + H+ * (CaOH + ) = 10-12,6 (5)

C2O42- + H+ ⇌ HC2O4- Ka2-1 = 104,27 (6)


HC2O4- + H+ ⇌ H2C2O4 Ka1-1 = 101,25 (7)
Nhận xét: do môi trường axit (H+ 0,055M) nên cân bằng tạo phức hiđroxo của Ca2+
có thể bỏ qua.
Ta có: S = [Ca2+]
0,125
S = [C2O42-] + [HC2O4-] + [H2C2O4]
= [C2O42− ]. (1 + K a−21.h + K a−11.K a−21.h 2 )

S
Vậy [C2O42− ]= −1
1 + K .h + K a−11.K a−21.h 2
a2

S2
 Ks=[Ca2+].[C2O42-]=
1 + K a−21.h + K a−11.K a−21.h 2

Thay h = 0,055  S = 1,9.10-3 (M) 0,125

2.2 2.1,455
a. Catot: PbO2 + 4 H+ + 2e  Pb2+ + 2 H2O 10 0,0592
0,125
-
HSO4  + H SO42- +
10 -2

2+ 2-
Pb + SO4  PbSO4 107,66
Phản ứng trên catot: PbO2 + HSO4- + 3H+ + 2e  PbSO4 + 2 H2O K1 (*) 0,125
−2.( −0,126)
Anot: Pb  Pb2+ + 2e 10 0,0592
HSO4- SO42-+ H+10-2
Pb2+ + SO42  PbSO4 107,66 0,125
- +
Phản ứng trên anot: Pb + HSO4  PbSO4 + H + 2e K2 (**)
Phản ứng xảy ra trong pin khi pin hoạt động:
0,125
PbO2 + Pb + 2 HSO4- + 2 H+  2 PbSO4 + 2 H2O (***)
Sơ đồ pin: (a) Pb│PbSO4, H+, HSO4-│PbO2 (Pb)(c)
2.E0PbO2 /PbSO4 2.1,455
10 0,0592 .10-2. 107,66 → E0PbO /PbSO = 1,62 (V)
0,0592
b. Từ (*): 10 = K1 =
2 4

−2.E0PbSO4 /Pb −2.( −0,126) 0,25


Từ (**): 10 0,0592 = K2 = 10 0,0592 .10-2. 107,66 → E0PbSO4 /Pb = - 0,29
(V)
Ta có: V = E(c) – E(a) = E PbO - E PbSO 0,25
2 /PbSO4 4 /Pb

0,0592 0,0592 [H + ]
V = E 0PbO + .log([HSO-4 ].[H + ]3 ) - E 0PbSO /Pb − .log
2 /PbSO4 2 4 2 [HSO-4 ]
0,0592
V = E 0PbO /PbSO - E 0PbSO /Pb + .log([HSO-4 ]2.[H + ]2 ) 0,25
2 4 4 2

Trong đó [HSO-4 ], [H + ] được tính theo cân bằng:


- 2-
HSO4  H+ + SO4 Ka = 10-2
0,25
[ ] 1,8 – x 1,8 + x x

4 ] = x = 9,89.10 (M) → [H ] = 1,81 (M); [ HSO 4 ] = 1,79 (M)


[ SO2- -3 + -

0,0592 2 2
V = 1,62 + 0,29 + log{(1,79) .(1,81) } = 1,94 (V)
2

Câu 3. (2,5 điểm) Nhiệt động học và cân bằng hóa học.
3.1. Xét các phản ứng:
⎯⎯
→ 2CO
C( r ) + CO2 (k) ⎯
⎯ (1)

⎯⎯
→ 2CO(k) + O2 (k)
2CO2 (k) ⎯
⎯ (2)
Trạng thái cân bằng của phản ứng (1) được xác định bởi các dữ kiện sau:
Nhiệt độ (0C ) Áp suất toàn phần (atm) % nCO trong hỗn hợp
800 2,57 74,55
900 2,30 93,08

Giả sử  H của phản ứng (1) không đổi trong khoảng 8000C – 9000C.
Hằng số cân bằng của phản ứng (2) ở 9000C là 1,25.10-16 atm
Nhiệt tạo thành ở 9000C của CO2 là -390,7 kJ/mol.
Tính ∆H, ∆S ở 9000C đối với phản ứng (2).
3.2. Trong một bình kín dung tích không đổi có chứa 0,1 mol Fe3O4 và 0,4 mol H2. Nung nóng bình đến
1000K. Biết các phản ứng xảy ra trong bình như sau:
⎯⎯
→ 3FeO(r)
Fe3O4(r) + H2(k) ⎯
⎯ + H2O(k) (1) Kp (1) = 1,862

⎯⎯
→ Fe(r)
FeO(r) + H2(k) ⎯
⎯ + H2O(k) (2) Kp (2) = 0,615
Xác định thành phần của hệ tại thời điểm cân bằng.
Ý ĐÁP ÁN ĐIỂM
3.1 - Chấp nhận các khí là khí lý tưởng, áp suất cân bằng (atm) đối với phản ứng (1)
như sau:
Nhiệt độ (0C) pCO2 pCO
800 2,57 x 0,2545 2,57 x 0,7455
900 2,30 x 0,0692 2,30 x 0,9308
2
pCO
Hằng số cân bằng KP (1) = của phản ứng (1)
PCO2

(2,57.0,7455) 2
* Ở 1073K, KP (1) = = 5,6123 atm 0,25
2,57.0,2545

(2,30.0,9308) 2
* Ở 1173K, KP (1) = = 28,7962 atm
2,30.0,0692

 H tính theo phương trình: ln K P (T2 ) = H ( 1 −


1
)
0,25
K P (T1 ) R T1 T2

H
ln
28,7962
= (
1

1
)  H = 171,12 kJ/mol
5,6123 8,314 1073 1173

Vì  H không đổi trong khoảng 1073K – 1173K nên có thể xem giá trị  H tính
được này ứng với nhiệt độ 1173K đối với phản ứng (1).
0,25
- Để tính  H1173 đối với phản ứng (2) ta vận dụng định luật Hess:
⎯⎯
→ 2CO (1)
C( r ) + CO2 (k) ⎯
⎯  H1173 = 171,12 kJ/mol
⎯⎯
→ CO2 (k)
C( r ) + O2 (k) ⎯
⎯ (3)  H1173 = -390,7 kJ/mol
Đảo ngược (3) và tổ hợp với (1) ta được:
⎯⎯
→ 2CO(k) + O2 (k) (2)
2CO2 (k) ⎯
⎯  H1173 = 561,82 kJ/mol
0,25
Khi đó áp dụng:  G = -RTlnKP
  G = -8,314x1173ln(1,25.10-16) = 357112 J/mol
0,25
Áp dụng công thức:  G =  H - T  S   S = H − G
T

 S1173 = 561820 − 357112 = 174,5 J/K.mol


1173
3.2 Nhận xét: cả Kp (1) và Kp (2) đều có cùng biểu thức hằng số cân bằng nên đây
là các cân bằng nối tiếp.
Vì Kp (1) > Kp (2)  cân bằng (1) xảy ra trước cân bằng (2). 0,25
Khi phản ứng (1) xảy ra:
⎯⎯
→ 3FeO(r)
Fe3O4(r) + H2(k) ⎯
⎯ + H2O(k) Kp (1) = 1,862
0,1 mol 0,4 mol Ban đầu
0,1 - x 0,4 - x 3x x Cân bằng
p H2O
Kp (1) = =
x
= 1,862  x = 0,26 > 0,1
PH2 0,4 - x
0,25
Vậy (1) xảy ra hoàn toàn.
Sau phản ứng (1) thành phần hệ gồm: H2 : 0,3 mol
H2O: 0,1 mol
0,25
FeO: 0,3 mol.
Khi phản ứng (2) xảy ra:
⎯⎯
→ Fe(r) +
FeO(r) + H2(k) ⎯
⎯ H2O(k) (2) Kp (2) = 0,615
Ban đầu: 0,3 mol 0,3 mol 0,1 mol
Cân bằng: 0,3 - y 0,3 - y y 0,1 + y
p H2O
Kp (2) = =
0,1 + y
= 0,615  y = 0,052 mol < 0,3 mol
PH2 0,3 - y 0,25
Vậy cân bằng được thiết lập là cân bằng (2).
Tại thời điểm cân bằng: H2 : 0,248 mol
H2O: 0,152 mol
FeO: 0,248 mol. 0,25
Fe: : 0,052 mol.

Câu 4 (2,5 điểm) Hóa nguyên tố (Kim loại, phi kim nhóm IVA, VA). Phức chất.
4.1. Photpho là nguyên tố hoá học có vai trò quan trọng trong tự nhiên và đời sống của động thực vật, vì
thế những hiểu biết về hoá học của photpho mang ý nghĩa to lớn cả về lí thuyết và thực tế. Photpho tạo
thành các hợp chất PF3 và PF5 với flo.
a. Dựa vào thuyết lực đẩy của các cặp electron hoá trị (VSEPR), cho biết dạng hình học electron (bao gồm
cả phân bố không gian của các nguyên tử và các cặp electron không liên kết), hình học phân tử (phân bố
không gian của các nguyên tử) và trạng thái lai hoá của nguyên tử P trong các phân tử trên. Trả lời kèm
theo các hình vẽ tương ứng.
b. Hãy so sánh (có giải thích) độ dài các liên kết P-F trong PF5.
c. Trong hai florua nói trên, phân tử của hợp chất nào có cực, tại sao?
d. N và P cùng nằm trong cùng nhóm V(A). Nitơ có dễ dàng tạo thành hợp chất NF5 không, tại sao?
4.2. Hãy cho biết:
a. Tại sao SiO2 có nhiệt độ nóng chảy cao hơn CO2?
b. Tại sao photphin(PH3) có nhiệt độ sôi thấp hơn amoniac(NH3), nhưng Silan(SiH4) lại có nhiệt độ sôi cao
hơn metan(CH4)
c. Si có hòa tan trong dung dịch axit không? Nếu có hãy viết phương trình phản ứng?
4.3. Ru(SCN)2(CN)4]4– là ion phức của ruteni, được kí hiệu là P.

a. Viết công thức Lewis của phối tử thioxianat SCN–.


b. Cho biết dạng lai hóa của Ru trong P. Mô tả sự hình thành ion phức theo thuyết VB (Valence Bond). Giải
thích tại sao trong P, liên kết được hình thành giữa Ru và N của phối tử SCN– mà không phải là giữa Ru và
S. Cho biết phức có tính thuận từ hay nghịch từ, vì sao?
Ý ĐÁP ÁN ĐIỂM
4.1 a. Công thức VSEPR của PF3 là PF3L1, P ở trạng thái lai hoá sp3.
(1,0) Tính đến cả sắp xếp không gian của các cặp electron không liên kết thì PF3 có cấu trúc
tứ diện còn nếu dựa vào sự sắp xếp các nguyên tử thì PF3 có cấu trúc hình chóp tam 0,25
giác.

P P
F F
F F
F F
P trong PF5 ở trạng thái lai hoá sp d.
3

=>Công thức VSEPR của PF5 là PF5L0


Dựa vào sự sắp xếp các cặp electron cũng như dựa vào sự sắp xếp các nguyên tử trong
phân tử thì PF5 có cấu trúc lưỡng tháp tam giác.

F
F
P F
F
b. So sánh độ dài các liên kết trong phân tử PF5.
F của ba lực đẩy vuông góc.
- Mỗi liên kết trục chịu tác động
- Mỗi liên kết biên chỉ chịu tác động của hai lực đẩy vuông góc và hai lực đẩy 0,25
với góc liên kết 1200. Lực đẩy với góc liên kết 1200 yếu hơn nhiều so với lực đẩy các
liên kết vuông góc. Như vậy, các nguyên tử F trên các liên kết trục bị đẩy mạnh hơn
(3 lực đẩy của các liên kết vuông góc) so với các nguyên tử F tham gia các liên kết
biên. Độ dài của hai liên kết trục lớn hơn chút ít so với 3 liên kết biên. (Sự đo độ dài
liên kết bằng nhiễu xạ electron pha khí cho độ dài liên kết trục P-F bằng 158 pm còn
độ dài hai liên kết biên 153 pm).
c.Mặc dù các liên kết P-F là phân cực, nhưng phân tử PF5 hoàn toàn đối xứng nên các
lưỡng cực cục bộ bù trừ lẫn nhau làm cho phân tử trong toàn bộ là không phân cực.
Phân tử PF3 không hoàn toàn đối xứng nên có cực.
d. Nitơ (N) => Muốn tạo thành NF5, 1 electron 2s trong nguyên tử N phải chuyển sang 0,25
lớp electron có n = 3 để nguyên tử N có 5 electron độc thân. Sự chuyển như vậy đòi hỏi
năng lượng rất lớn. Ngoài ra, bán kính của nguyên tử N nhỏ gây trở ngại không gian cho
việc tạo thành phân tử NF5. Vì thế, cho đến nay, việc cô lập phân tử NF5 vẫn là khó khăn
và việc có hay không có NF5 vẫn còn có ý kiến khác nhau
0,25
4.1 a.CO2 có cấu trúc phân tử dạng đường thẳng(C lai hóa sp); mạng tinh thể CO2 là mạng
(1,0) tinh thể phân tử. 0,25
SiO2 có cấu trúc mạng tinh thể nguyên tử; nguyên tử Si ở trạng thái lai hóa sp3, Si ở
tâm và 4O ở 4 đỉnh.
Quá trình nóng chảy của CO2 không liên quan đến việc cắt đứt liên kết cộng hóa trị
còn qua strinhf nóng chảy của SiO2 liên quan đến việc cắt đứt liên kết cộng hóa trị 0,25
trong mạng tinh thể nguyên tử.
b.Giữa các phân tử NH3 có liên kết hidro nên nhiệt độ sôi của NH3 lớn hơn của PH3.
CH4 và SiH4 đều không có liên kết hidro, SiH4 có phân tử khối lớn hơn nên nhiệt độ
sôi cao hơn. 0,25
c. Có. 3Si + 4HNO3 + 18HF --> 3H2SiF6 + 4NO + 8H2O
0,25
4.3 a. Ru2+ có cấu hình electron [Kr]4d65s05p0, là ion trung tâm trong phức bát diện.
(0,5)
Vì CN– là phối tử trường mạnh nên ở phân lớp 4d6 của Ru2+ có sự ghép đôi tất cả các
electron, tạo ra 2 AO 4d trống. Do đó xảy ra sự lai hóa d2sp3 để tạo 6AO lai hóa hướng
tới 6 đỉnh của 1 hình bát diện. Các phối tử (L) sử dụng cặp electron tự do của nguyên
tử N gửi vào các obitan lai hóa đó để tạo các liên kết cho nhận giữa phối tử và ion
Ru2+. 0,25
2 3
d sp

[Ru(SCN)2(CN)4]4-
4d6 5s 5p
L ... L

So với S, N có độ âm điện lớn hơn và bán kính nguyên tử nhỏ hơn, do đó mật độ điện
tích âm trên nguyên tử N sẽ lớn hơn, ái lực phản ứng với ion dương Ru2+ lớn hơn, vì
vậy trong phức chất P, liên kết phức được hình thành giữa Ru và N mà không phải là 0,25
giữa Ru và S.
Phức P có tính nghịch từ vì trong ion phức không có electron độc thân.
Câu 5 (2,5 điểm) Đại cương hữu cơ.
5.1. So sánh và giải thích ngắn gọn các tính chất sau đây:
a.So sánh tính axit của H trong các phân tử sau:

b. So sánh nhiệt độ sôi của các chất sau:

5.2. Khi thay thế một nguyên từ H trong hợp chất A3 bằng một nguyên tử Br (hợp chất A4) thì có sự giảm
về momen lưỡng cực. Hãy giải thích sự thay đổi đó.

5.3. Cho biết chất nào có tính bazơ mạnh nhất trong số các chất sau đây. Giải thích lý do

5.4. Tốc độ thủy phân chất A1 tăng mạnh khi có mặt LiCl, còn A2 gần như không đổi. Hãy giải thích sự
khác biệt này.

5.5. Độ dài liên kết C-C trong cyclopropane được xác định là 151.3 pm, nhưng các kết quả nghiên cứu cho
thấy độ dài của phần lớn liên kết C-C trong cyclopropylmethyl carbocation thay đổi khá nhiều so với
cyclopropane. Hãy giải thích kết quả này.

Ý ĐÁP ÁN ĐIỂM
a.Tính axit của A>C>B. A tạo ra anion là hệ thơm, B tạo ra anion là hệ phản thơm, 0,25
5.1 C tạo ra anion hệ không thơm.
b. Nhiệt đội sôi D>F>E. D tạo liên kết H liên phân tử, F có phân tử khối lớn hơn E. 0,25
5.2. Cấu trúc trong hệ có tính thơm, chiều của momen lưỡng cực hướng về phía
nguyên tử oxy C=O. Chính vì vây việc gắn thêm brom sẽ làm giảm momen lưỡng 0,25
cực tổng của phân tử (Do liên kết C-Br ngược phía C-O làm giảm momen lưỡng
cực của phân tử).

0,25
5.3 Hợp chất có tính bazơ mạnh nhất là N,N-dimetyl-2,6-dimetylanilin. Ở đây 0,5
tương tác +C giữa cặp e của N với vòng benzen nhỏ nhất do tương tác đẩy giữa hai
nhóm phenyl với nhóm NMe2 khiến cho khả năng xen phủ của cặp electron trên N
với các obitan π của vòng thơm trở nên kém hơn hẳn so với ba chất còn lại.
5.4 Khi H2O tấn công vào axyl clorua sẽ tạo thành trung gian tứ diện có dạng:

0,25

Ở A1 sự tạo thành trung gian tứ diện không hề thuận lợi do trung gian tạo thành sẽ
tương tác mạnh với hai nhóm CH3 ở ortho. Thế nên ở A1 sẽ xảy ra sự phân li trực
tiếp tạo thành R-CO+. Việc thêm LiCl sẽ thúc đẩy mạnh hơn sự phân li điện tích,
dẫn đến sự thúc đẩy phản ứng.

0,25

Ở A2 không xảy ra quá trình này một phần là do nhóm NO2 làm giảm tính bền của
cabocation tạo thành.
5.5 - Do liên kết C-C trong vòng cyclopropane tựa như một liên kết  nên sẽ có tương
tác ổn định giữa C-C và AO p trống của carbocation. Điều này dẫn đến việc thu
ngắn lại liên kết C-C giữa vòng và carbocation, đồng thời kéo dài liên kết của hai 0,25
cạnh trong vòng cũng như làm giảm độ dài liên kết của cạnh còn lại (in đậm).

- Có thể thấy rõ điều này bằng các cấu trúc cộng hưởng như hình bên.

0,25

Câu 6 (2,5 điểm) Sơ đồ tổng hợp hữu cơ. Cơ chế phản ứng hóa hữu cơ.
6.1. Forskoline là một diterpene có trong tinh dầu húng chanh thường được sử dụng để chữa bệnh về hô
hấp. Dưới đây là một phần sơ đồ tổng hợp forskoline:
Biết E là but-2-inoic acid và I là vanadyl acetylacetonate, VO(acac)2. Xác định công thức cấu tạo các
chất chưa biết trong sơ đồ tổng hợp trên.
6.2. Hoàn thành cơ chế các phản ứng sau:
H
H+
HO OMe
MeOH
O
a/ O

b/
Ý ĐÁP ÁN ĐIỂM
6.1 0,15x10
= 1,5đ

H
6.2.a H -H+ O +H+/-H2O + MeOH/-H+
H O O
HO HO OH SP 0,5
OH
O H+ OH
6.2.b

0,5

Câu 7 (2,5 điểm) Xác định cấu trúc các chất hữu cơ (mô tả sơ đồ tổng hợp bằng lời dẫn)
7.1. Hợp chất C1 (C10H18O) phản ứng với CH3MgBr, tạo khí metan; phản ứng với PCC, tạo thành
axeton; phản ứng với KMnO4 loãng, lạnh tạo thành chất C10H20O3. Axetyl hóa C1 bằng CH3COCl, sau
đó ozon phân/ khử hóa, thu được C2 (C12H20O4). Oxi hóa C2 bằng nước brom thu được C3 (C12H20O3).
Chất C3 tham gia phản ứng chuyển vị Baeyer Villiger với m-CPBA (tỉ lệ mol 1:1) thu được nhiều đồng
phân trong đó có C4 (C12H20O6). Thủy phân C4 với H2SO4/H2O, thu được axit ađipic
HOOC(CH2)4COOH, butan-1,3-điol và axit axetic.
Xác định các chất C1, C2, C3 và C4.
7.2. Hợp chất A (C8H16O2) không tác dụng với H2/Ni đun nóng. Cho A tác dụng với HIO4, thu được A1
(C3H6O) có khả năng tham gia phản ứng iođofom và A2 (C5H8O). Đun nóng A có mặt H2SO4, thu được
chất B (C8H14O) chứa vòng 6 cạnh. Cho B phản ứng với 2,4-đinitrophenylhiđrazin, thu được C; cho B
phản ứng với H2/Ni đun nóng thu được chất D. Đun nóng D với H2SO4 đặc, thu được E (C8H14). Ozon
phân E, sau đó khử hóa ozonit với Zn/HCl hoặc oxi hóa với H2O2, đều thu được F (C8H14O2). F tham gia
phản ứng iođofom sau đó axit hóa, thu được G (C6H10O4). Xác định cấu tạo các chất A, A1, A2, B, C, D,
E, F và G. Đề xuất cơ chế từ A sang B
Ý ĐÁP ÁN ĐIỂM
7.1 Từ sản phẩm axit ađipic HOOC-(CH2)4-COOH, butanđiol-1,3 và axit axetic, theo
dữ kiện C1 phản ứng với PCC tạo thành xeton, suy ra đầu ancol bậc 2 của
butanđiol-1,3 có sẵn từ chất đầu C1. Vì vậy, cấu tạo của C4 hoàn toàn xác định. 0,25.4
Từ đó xác định được cấu tạo các chất C1, C2, C3. = 1,0

7.2 Xác định cấu tạo các chất A, A1, A2, B, C, D, E, F và G:


A (C8H16O2) không tác dụng với H2/Ni nên độ bất bão hòa bằng 1 và không chứa
vòng xiclopropan và xiclobutan, A tác dụng với HIO4, thu được A1 (C3H6O) và 0,25
A2 (C5H8O) nên A có cấu tạo điol có nhánh đimetyl. A có cấu tạo như sau:

1,0
Đề xuất cơ chế chuyển hóa từ A sang B

0,25
Câu 8 (2,5 điểm) Hóa học các hợp chất thiên nhiên (Cacbohidrat và các hợp chất hữu cơ chứa nito đơn
giản)
8.1. Chất G được sử dụng làm thuốc chống dị ứng. Từ p-chlobenzandehyd (chất A) tổng hợp chất G
theo sơ đồ sau

Xác định công thức cấu tạo của các chất B, C, D, E, F và G


Ý ĐÁP ÁN ĐIỂM
8.1

0,15x6=
0,9

8.2. Một cacbohiđrat A có công thức phân tử C12H20O11. A phản ứng được với thuốc thử phenylhiđrazin,
không làm mất màu dung dịch Br2/H2O, không tham gia phản ứng với thuốc thử Tollens.
Thủy phân A bằng dung dịch axit thu được hợp chất A1 và D-fructozơ. Metyl hóa A1 bằng MeOH
(xúc tác HCl khan) thu được hợp chất A2 (C7H12O6). Tiến hành oxi hóa A2 bằng dung dịch HIO4 rồi thủy
phân sản phẩm thu được hỗn hợp, trong đó có chứa các chất A3, A4, A5.

Đun nóng A với dung dịch NaOH thu được hỗn hợp sản phẩm, trong đó có B1 (C6H8O5). Oxi hóa
B1 bằng HIO4 và đun nóng nhẹ thu được hợp chất B2 (C5H8O4). Metyl hóa B2 bằng MeI/Ag2O thu được
hợp chất B3 (C7H14O5). Mặt khác, hiđro hóa B2 bằng H2/Ni, to thu được hợp chất B4 (C5H12O4).
Xác định và biểu diễn cấu trúc của A, A1, A2, B1, B2, B3, B4 ở dạng vòng Haworth (với hợp chất mạch
vòng) hoặc công thức Fisơ (với hợp chất mạch hở).

Ý ĐÁP ÁN ĐIỂM
8.2 A có công thức phân tử C12H20O11  độ không no k = 3.
A phản ứng được với thuốc thử phenylhiđrazin, không làm mất màu dung dịch
Br2/H2O, không tham gia phản ứng với thuốc thử Tollens  A chứa chức xeton.
Thủy phân A bằng dung dịch axit thu được hợp chất A1 và D-fructozơ  A là một
disaccarit hoặc gồm một monosaccarit là Fructozo và thành phần aglycol là A1.
Metyl hóa A1 bằng MeOH (xúc tác HCl khan) thu được hợp chất A2 (C7H12O6) 
A1 có 1 nhóm -OH hemiaxetal
Tiến hành oxi hóa A2 bằng dung dịch HIO4 rồi thủy phân sản phẩm thu được hỗn
hợp, trong đó có chứa các chất A3, A4, A5.
 công thức cấu tạo của A2:

8.0,2 =
1,6
Đun nóng A với dung dịch NaOH thu được hỗn hợp sản phẩm, trong đó có B1
(C6H8O5).
 A thực hiện phản ứng retroaldol tạo thành B1 và fructozo.

Oxi hóa B1 bằng HIO4 và đun nóng nhẹ thu được hợp chất B2 (C5H8O4).

Metyl hóa B2 bằng MeI/Ag2O thu được hợp chất B3 (C7H14O5).

Mặt khác, hiđro hóa B2 bằng H2/Ni, to thu được hợp chất B4 (C5H12O4).

Công thức Haworth của các chất chưa biết:


TRƯỜNG THPT CHUYÊN KÌ THI CHỌN HỌC SINH GIỎI
NGUYỄN TRÃI – HẢI DƯƠNG CÁC TRƯỜNG CHUYÊN VÙNG DUYÊN
HẢI VÀ ĐỒNG BẰNG BẮC BỘ
ĐỀ THI ĐỀ XUẤT
MÔN: HOÁ HỌC LỚP 11
Thời gian làm bài: 180 phút
(Đề thi có 04 trang, gồm 8 câu)
Câu 1: (2,5 điểm) Tốc độ phản ứng
⎯⎯ →
Tại 25oC phản ứng 2 N2O5 (k) ⎯ ⎯ 4 NO2 (k) + O2 (k) có hằng số tốc độ k = 1,8.10-5. s-1; biểu thức tính tốc
độ phản ứng v = k. C N2O5. Phản ứng trên xảy ra trong bình kín thể tích 20,0 lit không đổi. Ban đầu lượng N2O5
cho vừa đầy bình. Ở thời điểm khảo sát, áp suất riêng của N2O5 là 0,070 atm. Giả thiết các khí đều là khí lí
tưởng.
1. Tính tốc độ: a) tiêu thụ N2O5; b) hình thành NO2; O2.
2. Tính số phân tử N2O5 đã bị phân tích sau 30 giây.
⎯⎯ →
3. Nếu phản ứng trên có phương trình N2O5 (k) ⎯ ⎯ 1
2 NO2 (k) + 2O2 (k) thì trị số tốc độ phản ứng, hằng số
tốc độ phản ứng có thay đổi không? Giải thích.
Câu 2: (2,5 điểm) Cân bằng và phản ứng trong dung dịch. Pin điện – Điện phân
1. Dung dịch A gồm Ba(NO3)2 0,060 M và AgNO3 0,012 M.
a) Thêm từng giọt K2CrO4 vào dung dịch A cho đến dư. Có hiện tương gì xảy ra?
b) Thêm 50,0 ml K2CrO4 0,270 M vào 100,0 ml dung dịch A.
Tính nồng độ các ion trong hỗn hợp thu được.
Cho: BaCrO4↓ + H2O Ba2+ + HCrO4- + OH - ; K = 10-17,43
Ag2CrO4 + H2O 2Ag+ + HCrO4- + OH - ; K = 10-19,50 pKa của HCrO4- bằng 6,50.

2. Trình bày sơ đồ nhận biết và phương trình ion của các phản ứng đã xảy ra khi nhận biết các cation trong
dung dịch X gồm Ba2+, Fe2+, Pb2+, Cr3+, NO3-.

3. a. Hãy trình bày cách thiết lập sơ đồ pin sao cho khi pin hoạt động thì xảy ra phản ứng:
H3AsO4 + NH3 → H 2 AsO−4 + NH +4
b. Tính sức điện động của pin ở điều kiện tiêu chuẩn ( E pin ).
c. Biết CH3AsO4 = 0,025 M; C NH3 = 0,010 M.
i) Tính sức điện động của pin.
ii) Tính thế của từng điện cực khi hệ đạt trạng thái cân bằng.
Cho: pK ai(H3AsO4 ) = 2,13; 6,94; 11,50; pK a(NH+ ) = 9, 24 (pKa = - lgKa, với Ka là hằng số phân li axit). p H 2 = 1 atm;
4
RT
ở 25 oC: 2,303 = 0, 0592.
F

Câu 3: (2,5 điểm) Nhiệt động học và cân bằng hoá học
Cho các đại lượng nhiệt động sau:

H3PO4(dd) H2PO4-(dd) HPO42-(dd) PO43-(dd) H+ + OH- → H2O


Ho (kJ.mol-1) - 1288 - 1296 - 1292 - 1277 - 56
S (J.mol .K ) 158
o -1 -1 90 - 33 - 220 81
1. Tính G của phản ứng trung hoà từng nấc H3PO4 bằng OH-.
o

1
2. Tính hằng số phân ly axit nấc thứ nhất của H3PO4.
3. Trộn lẫn dung dịch H3PO4 0,10 M và NaOH 0,10 M, thu được 25,0 mL dung dịch hỗn hợp hai muối
NaH2PO4, Na2HPO4 và nhiệt lượng toả ra là 90,0 J. Tính thể tích hai dung dịch đã đem trộn lẫn.

Câu 4: (2,5 điểm) Hoá nguyên tố (kim loại, phi kim nhóm IVA, VA). Phức chất.
1. Một nguyên tố X có khả năng phản ứng với canxi cho chất Y. Mặt khác X tan được trong dung dịch
kiềm tạo ra một hợp chất A và khí B đều có chứa nguyên tố X. A phản ứng với clorua vôi thu được một kết
tủa C. Kết tủa này sẽ chuyển thành Y khi xử lý với nhôm ở nhiệt độ cao. Hòa tan chất Y trong dung dịch HCl
loãng thu được B. Biết rằng khi xử lý C với SiO2 và than cốc thu được X, còn trong trường hợp không có than
cốc thu được D. D tan được trong cả dung dịch axit loãng và kiềm loãng.
a. Lập luận xác định cấu trúc các chất chưa biết và viết các phương trình phản ứng xảy ra.
b. Đơn chất X tồn tại một dạng thù hình kém bền với không khí và dễ thăng hoa. Vẽ cấu trúc dạng thù
hình này và giải thích tại sao nó lại kém bền với không khí?
2.
a. Dựa trên cấu trúc của phức chất và sự sắp xếp electron của ion trung tâm, hãy dự đoán tính oxy hóa
khử của các ion phức bát diện sau đây: [Ti(aq)]3+;[Cr(aq)]2+.
b. Xếp các ion dưới đây thành 2 dãy: Dãy tạo nhiều họp chất có màu và dãy tạo thành nhiều họp̣ chất
không có màu. Giải thích: Cu2+ , Cu+, Cd2+, Rℎ4+, W6+, V2+.

Câu 5 (2,5 điểm) Đại cương hữu cơ


1. Các dẫn chất carbonyl cũng thể hiện tính acid Bronsted nhờ sự cho proton để tạo base liên hợp bền vững.
Cho các dẫn chất carbonyl sau:

a, Vẽ base liên hợp bền nhất của G1


b, Cho giá trị pKa (ứng với nhóm-CH-) của G2, G3 và G4 trong dung môi DMSO (xếp ngẫu nhiên) là 24,7;
26,4; 28,1. Gán giá trị pKa cho từng chất G2, G3 và G4.
c, Vẽ sản phẩm tạo thành khi cho G5 tác dụng với NaOH theo tỷ lệ 1:1.

2. (S)-BINOL và (?)-BINAP là các hợp chất hữu cơ phổ biến thường dùng để làm phối tử cho kim loại
chuyển tiếp trong các phản ứng tổng hợp bất đối xường. Cấu trúc của các chất này được biểu diễn bên dưới.

a, Hãy đưa ra ít nhất 2 yếu tố trong cấu trúc phân tử tạo nên thủ tính của (S)-BINOL?
b, Hãy xác định cấu hình lập thể tuyệt đối của (?)-BINAP?

3. So sánh độ dài liên kết C=O trong ba hợp chất A, B, C và giải thích:

2
Câu 6 (2,5 điểm) Sơ đồ tổng hợp hữu cơ. Cơ chế phản ứng hóa hữu cơ.
1. Trình bày cơ chế của phản ứng sau:
a,

b,

2. Phản ứng đa thành phần từ lâu đã được ứng dụng vào trong tổng hợp Hóa dược. Trong phản ứng đa thành
phần, các thành phần cấu tạo thành phân tử đích sẽ được cho vào nồi phản ứng cùng một lúc hay cho vào lần
lượt theo thứ tự. Một chất đối vận thụ thể prostaglandin chứa vòng azetidine, PF, được tổng hợp thông qua
bốn bước, trong đó bước tổng hợp P6 là một phản ứng bốn thành phần.

Vẽ công thức cấu tạo của các chất từ [P2] đến [P5], P7 và PF.
-Cho biết [P2] là một bicyclo, [P3] là một monocyclo, cả [P2] và[P3] đều là muối của lithium.
Câu 7 (2,5 điểm) Xác định cấu trúc các chất hữu cơ (mô tả sơ đồ tổng hợp bằng lời dẫn)
Hợp chất carbonyl có khả năng tạo thành lưỡng gốc tự do dưới tác dụng của ánh sáng tử ngoại với bước
sóng phù hợp. Lưỡng gốc tự do này ở trạng thái thuận từ (triplet) giống carbene và có khả năng phân mảnh
hay thực hiện quá trình chuyển nguyên tử hydrogen nội phân tử để tạo thành các sản phẩm khác nhau. Hợp
chất A1 (C7H14O) được xử lý bằng tia UV có khả năng thu được hỗn hợp các chất A2 (C7H14O), A3 (C3H6O)
và A4 (C4H8), trong đó A3 và A4 luôn tạo thành với hàm lượng bằng nhau. Biết rằng:
- A2 có vòng 4 cạnh.
- Khi thực hiện phản ứng bromine hóa A3 thì chỉ thu được một sản phẩm monobromine A5 (C3H5OBr) duy
nhất.
3
- A4 có tốc độ phản ứng với Br2 nhanh nhất trong số các đồng phân của nó.
a. Xác định cấu tạo của các chất từ A1 đến A5.
b. Đề nghị cơ chế chuyển hóa A1 thành các sản phẩm A2 đến A4.

Câu 8 (2,5 điểm) Hóa học các hợp chất thiên nhiên (Cacbohidrat và các hợp chất hữu cơ chứa nito
đơn giản)
Trong cơ thể người, tryptophan chuyển hóa thành melatonin, môt chất có vai trò điều
hòa hoạt động ngủ nghỉ và một số chất nội sinh khác theo sơ đồ sau:

Bảng bên dưới cho biết tên các nhóm enzyme có thể tương ứng với các kí hiệu từ E1 – E7 nhưng được sắp
xếp không theo thứ tự.

Cho biết tên các nhóm enzyme tương ứng với các kí hiệu từ E1 – E7 và hoàn thành sơ đồ phản ứng trên.

--------------------------HẾT --------------------------

Người soạn đề: Nguyễn Thị Mai Phương


SĐT: 0915151001

4
TRƯỜNG THPT CHUYÊN ĐỀ THI ĐỀ XUẤT DUYÊN HẢI BẮC BỘ
NGUYỄN TRÃI Môn: HÓA HỌC – LỚP 11
HƯỚNG DẪN CHẤM
-----***-----
Câu Hướng dẫn chấm Điểm
1.1 nN O Pi 0,07
pi V = ni RT→ C N2O5 = 2 5
= = = 2,8646.10-3. (mol.l-1)
V RT 0,082  298 0,5
V phản ứng = k C N2O5 = 1,8.10-5  2,8646.10-3 = 5,16. 10-8 mol. l-1.s-1.
Từ phương trình 2N2O5 (k) → 4NO2 (k) + O2 (k)
1 dCN O 1 dC dC
V phản ứng = −  = +  NO = + O
2 5 2 2

2 dt 4 dt dt
dCN O 0,5
Vtiêu thụ(N2O5) = − 2 5
= −2Vphản ứng= −25,16. 10 = −1,032.10 mol.l .s .
-8 -7 -1 -1
dt
Dấu - để chỉ “tiêu thụ N2O5 tức mất đi N2O5 hay giảm N2O5”
Vhình thành (NO2)= 4Vphản ứng = 4  5,16.10-8 = 2,064.10-7 mol.l-1.s-2. 0,5
Vhình thành(O2)= V phản ứng = 5,16.10-8 mol.l-1.s-2

1.2 Số phân tử N2O5 đã bị phân huỷ = Vtiêu thụ(N2O5) Vbình  t  N0(số avogadrro) 0,5
= 1,032.10-7 . 20,0 . 30,0 . 6,023.1023  3,7.1019 phân tử

1.3 Nếu phản ứng trên có phương trình: N2O5(k) → 2NO2(k) + 1/2 O2(k)thì tốc độ phản 0,5
ứng, cũng như hằng số tốc độ phản ứng đều không đổi (tại nhiệt độ T xác định), vì: - k
chỉ phụ thuộc nhiệt độ.
- Khi k = const; C N2O5 = const thì V = const
2.1 a) Hiện tượng: Có kết tủa BaCrO4 và Ag2CrO4. Xét thứ tự xuất hiện các kết tủa:
𝐾𝑠(𝐵𝑎𝐶𝑟𝑂4 )
Để bắt đầu có BaCrO4 ↓ : 𝐶𝐶𝑟𝑂42− > 𝐶 (1)
𝐵𝑎2+
𝐾𝑠(𝐴𝑔2 𝐶𝑟𝑂4 )
Để bắt đầu có Ag2CrO4 ↓ : 𝐶𝐶𝑟𝑂42− > (2)
𝐶𝐴𝑔+
Để tính tích số tan Ks cần tổ hợp cân bằng :
BaCrO4 ↓ ⇌ 𝐵𝑎2+ + 𝐶𝑟𝑂42− 𝐾𝑠1
+ −
𝐻2 𝑂 ⇌ 𝐻 + 𝑂𝐻 𝐾𝑤
2− + −
𝐶𝑟𝑂4 + 𝐻 ⇌ 𝐻𝐶𝑟𝑂4 𝐾𝑎−1
Được: 0,25
𝐵𝑎𝐶𝑟𝑂4 + 𝐻2 𝑂 ⇌ 𝐵𝑎2+ + 𝐻𝐶𝑟𝑂4− + 𝑂𝐻 − 𝐾1
Có: 𝐾1 = Ks1 . Kw . Ka-1
𝐾1 .𝐾𝑎 10−17,43 . 10−6,5
Suy ra: 𝐾𝑠1 = = = 10−9,93
𝐾𝑤 10−14
Ag2CrO4 ↓ ⇌ 2𝐴𝑔+ + 𝐶𝑟𝑂42− 𝐾𝑠2
𝐻2 𝑂 ⇌ 𝐻 + + 𝑂𝐻 − 𝐾𝑤
𝐶𝑟𝑂42− + 𝐻 + ⇌ 𝐻𝐶𝑟𝑂4− 𝐾𝑎−1
Được:
𝐴𝑔2 𝐶𝑟𝑂4 + 𝐻2 𝑂 ⇌ 2𝐴𝑔+ + 𝐻𝐶𝑟𝑂4− + 𝑂𝐻 − 𝐾2
−19,5
𝐾2 = 10

5
𝐾2 .𝐾𝑎 10−19,5 . 10−6,5
Suy ra: 𝐾𝑠2 = = = 10−12
𝐾𝑤 10−14
10−9,93
Từ (1): 𝐶𝐶𝑟𝑂42− (𝐵𝑎𝐶𝑟𝑂4) > = 1,96. 10−9 𝑀
0,06
10−9,93 0,25
Từ (2): 𝐶𝐶𝑟𝑂42− (𝐴𝑔2 𝐶𝑟𝑂4) > = 6,94. 10−9 𝑀
(0,012)2
𝐶𝐶𝑟𝑂42−(𝐵𝑎𝐶𝑟𝑂4) < 𝐶 (𝐴𝑔2 𝐶𝑟𝑂4 ) nhưng không nhiều, vì vậy sẽ có hiện tượng kết
𝐶𝑟𝑂42−
tủa vàng của BaCrO4 xuất hiện trước một ít, sau đó đến kết tủa vàng nâu (đỏ gạch của
𝐴𝑔2 𝐶𝑟𝑂4 và BaCrO4 vàng cùng xuất hiện

b. Sau khi thêm 𝐾2 𝐶𝑟𝑂4


0,27. 50
𝐶𝐶𝑟𝑂42− = = 0,09𝑀
150
0,012. 100
𝐶𝐴𝑔+ = = 0,008𝑀
150
0,06. 100
𝐶𝐵𝑎2+ = = 0,04𝑀
150
Các phản ứng:
2+ 2-
Ba + CrO4 BaCrO4 ↓
0,046 0,090
- 0,050
+
2 Ag + CrO42- Ag2CrO4 ↓
0,0080 0,050
- 0,046

Thành phần sau phản ứng :


BaCrO4 ↓ ; Ag2CrO4 ↓ ; CrO42-(0,046 M ).
Ag2CrO4 ↓ ⇌ 2𝐴𝑔+ + 𝐶𝑟𝑂42− 10−12

BaCrO4 ↓ ⇌ 𝐵𝑎2+ + 𝐶𝑟𝑂42− 10−9,93


Nồng độ CrO4 dư khá lớn, có thể coi nồng độ CrO42- do 2 kết tủa tan ra là không
2-

đáng kể.
CrO42- + H2O ⇌ HCrO4- + OH- 𝐾𝑏 = 10−7,5

C 0,046

[] (0,046 – x ) x x
0,25
𝑥2
= 10−7,5 → x = 3,8. 10−5 ≪ 0,046;
0,046 − 𝑥
[CrO42- ]= 0,046M
10−12
[Ag+ ] = √0,046 = 4,46. 10−6 𝑀 0,25
10−9,93
[Ba2+ ] = = 2,55. 10−9 𝑀
0,046
[Ba2+] và [Ag+] đều << [CrO42- ], chứng tỏ nồng độ CrO42- do 2 kết tủa tan ra là
không đáng kể.
Vậy trong dung dịch có:
[Ba2+] = 2,55.10-9M ; [Ag+] = 4,66.10-6M ;

6
[CrO42-] = 0,046M ; [OH-] = 3,8.10-5M ;[H+] = 2,63.10-10M;
[K+] = CK+ = 0,18M ; CNO3-= 0,088M.

2.

0,25

Ba2+ + HSO4- → BaSO4+ + H+


Pb2+ + HSO4- → PbSO4 + H+ 0,25
H + OH →
+ -
H2O
PbSO4↓ + 4 OH- → PbO22- + SO42-+ H2O
PbO22- + SO42- + 4H+ → PbSO4↓ + 2H2O
(PbO22- + 2H2S → PbS↓ đen + 2H2O)
Cr3+ + 3 OH - → Cr(OH)3↓
Cr(OH)3↓ + OH- → CrO2- + 2H2O
2CrO2- + 3 H2O2 + 2OH- 2CrO42- + 4H2O
Fe2+ + 2OH - → Fe(OH)2↓
2 Fe(OH)2 + H2O2 → Fe(OH)3↓
3. a. Phản ứng xảy ra trong pin được tổ hợp từ các cân bằng sau:
⎯⎯ → H AsO-4
H3AsO4 ⎯ ⎯ H+ + 2
⎯⎯ → NH +4
NH3 +H+ ⎯ ⎯

⎯⎯ → H 2 AsO-4 NH +4
H3AsO4 + NH3 ⎯ ⎯ + (*) K
Như vậy các cân bằng trên đều liên quan đến quá trình cho - nhận H+, do đó có thể
E + 0,25
chọn điện cực hiđro để thiết lập pin. Vì giá trị thế của điện cực hiđro ( 2H /H2 ) phụ
thuộc vào [H+]:
0,0592 [H + ]2
E 2H+ /H = lg
2 2 pH2
nên điện cực platin nhúng trong dung dịch H3AsO4(có [H+] lớn hơn) có thế dương
hơn, sẽ là catot. Ngược lại điện cực platin nhúng trong dung dịch NH3 sẽ là anot. Vậy
ta có sơ đồ pin:
(-) Pt(H2)│NH3(aq)║ H3AsO4(aq) │Pt (H2) (+)
pH2 pH2
= 1atm = 1atm
b. Quá trình oxi hóa xảy ra trên anot:

7
⎯⎯ →
H2 ⎯ ⎯ 2H++ 2e K=1
⎯⎯→ NH +4 (K -1
a )
2
2NH3 +H+ ⎯⎯ = (109,24)2
0
⎯⎯
→ NH +4
⎯
⎯ K1 =10-2.Ea /0,0592 (1)
2NH3 + H2 2 + 2e
9,24 . 2 . 0,0592
0
→ a=
E -2 = - 0,547 (V)
Quá trình khử xảy ra trên catot: 0,25
⎯⎯→ H AsO-4 (K a1 ) 2 = (10-2,13)2
2H3AsO4 ⎯⎯ H+ + 2
+ ⎯⎯
⎯ →

2H + 2e H2 K=1
0
⎯⎯
→ H AsO-4 K 2 =102.Ec /0,0592 (2)
2H3AsO4 + 2e ⎯
⎯ H2 + 2 2
-2,13 . 2 . 0,0592
0
→ Ec = 2 = - 0,126 (V)
E E0 E0
Vậy pin = c - a = 0,421 (V).
(Hoặc từ (*) ta có: K = Ka1.(Ka)-1 = 10E/0,0592→ E0pin = E = 0,421 (V))
c. Do sự phân li của nước trong dung dịch NH3 0,010 M và trong dung dịch H3AsO4
0,025 M không đáng kể, nên:
i) Tại dung dịch của nửa pin trái:
⎯⎯ → NH +4
NH3 +H2O ⎯ ⎯ + OH- Kb = 10-4,76
[ ] 0,010-x x x
+
→ [ NH 4 ] = [OH-] = x = 4,08.10-4 (M); [NH3] = 9,59.10-3 (M); [H+] = 2,45.10-11 (M)
0, 0592 [NH +4 ]2
lg
E 0
a
2 [NH3 ]2 .p H 2
Từ (1), ta có: Ea = +
2
0, 0592  4, 08.10−4 
lg 
p = 2  9,59.10−3 
Vì H 2 1atm nên: Ea = -0,547 +   = - 0,63 (V)
+
(Hoặc Ea = 0,0592.lg[H ])
Đối với H3AsO4, vì Ka1 Ka2 Ka3 nên tại dung dịch của nửa pin phải:

H3AsO4 → H+ + 2
H AsO-4 K =10-2,13
a1
[] 0,025-x x x 0,25
-
→ [ H 2 AsO4 ] = [H+] = x = 0,0104 (M); [H3AsO4] = 0,0146 (M)
0, 0592 [H3AsO 4 ]2
lg
E 0c 2 [H 2 AsO −4 ]2 .p H 2
Từ (2), ta có: Ec = +
2
0, 0592  0, 0146 
lg  
→ Ec = -0,126 + 2  0, 0104   - 0,12 (V)
(Hoặc Ec = 0,0592.lg[H+])
→ Epin = -0,12 + 0,63 = 0,51 (V)
ii) Khi hệ đạt trạng thái cân bằng thì thế của 2 điện cực bằng nhau: Ec = Ea

8
⎯⎯→ H AsO- +
H3AsO4 + NH3 2 4 + NH 4 K = 107,11
0,025 0,010
0,015 - 0,010 0,010

Hệ thu được gồm:


NH 4 0,010 M; H 2 AsO-4 0,010 M; H AsO 0,015 M. Do sự phân li
+
3 4
+
0,25
của
NH 4 và của nước không đáng kể, do đó pH của hệ được tính theo cân bằng:

H3AsO4 → H+ + 2
H AsO-4 K a1 =10-2,13
[ ] 0,015-x x 0,010+x
-
→ [H+] = x = 4,97.10-3 (M); [H3AsO4]  0,010 (M); [ H 2 AsO4 ]  0,015 (M).
0, 0592 [H3AsO 4 ]2 0, 0592  0, 01 
2
lg lg 
0 2 [H 2 AsO −4 ]2 .p H 2 
→ Ea = Ec = Ec + = -0,126 + 2  0, 015   -
0,136 (V)
(Hoặc Ea = Ec = 0,0592.lg[H+])

3 1.
Xét phản ứng: H+ + OH- → H2O. Ta có:
Ho = Ho(H2O) - Ho(H+) - Ho(OH-)
Ho = Ho(H2O) - Ho(OH-) = - 56 KJ.mol-1 (Vì Ho(H+) = 0)
So = So(H2O) - So(H+) - So(OH-)
So = So(H2O) - So(OH-) = 81 J.mol-1.K-1 (Vì So(H+) = 0)
* H3PO4 + OH- → H2PO4- + H2O (1)
H1o = Ho(H2PO4-) + [Ho(H2O) - Ho(OH-)] - Ho(H3PO4)
= - 1296 - 56 + 1288
= - 64 (kJ.mol-1)
S1 = So(H2PO4-) + [So(H2O) - So(OH-)] - So(H3PO4)
o

= 90 + 81 – 158
0,5
= 13 (J.mol-1.K-1)
G1o = H1o – T.S1o = - 64 – 298.0,013
G1o = - 67,9 (kJ.mol-1)
* H2PO4- + OH- → HPO42- + H2O (2)
Tương tự, ta được:
H2o = - 1292 - 56 + 1296 = - 52 (kJ.mol-1) 0,5
S2o = - 33 + 81 – 90 = - 42 (J.mol-1)
G2o = H2o – T.S2o = - 52 + 298.0,042
G2o = - 39,5 (kJ.mol-1)
* HPO42- + OH- → PO43- + H2O (3)
H3o = - 1277 – 56 + 1292 = - 41 (kJ.mol-)
S3o = - 220 + 81 + 33 = - 106 (J.mol-1.K-1) 0,5
G3o = H3o – T.S3o = - 41 + 298.0,106
G3o = - 9,4 (kJ.mol-1)
2.
H3PO4  H+ + H2PO4- Ka1
H+ + OH-  H2O Kw-1
H3PO4 + OH-  H2PO4- + H2O K = Ka1.Kw-1
Ta có:
9
G1o = - RTlnK 0,5
 K = exp(- G1o/RT) = exp(67900/(8,314.298) = 7,9.1011
Ka1 = K.Kw = 7,9.1011.10-14
Ka1 = 7,9.10-3
3.
Gọi x, y lần lượt là số mol NaH2PO4 và Na2HPO4 sinh ra.
H3PO4 + OH- → H2PO4- + H2O H1o = - 64 kJ.mol-1
x x x
H3PO4 + 2OH → HPO42- + 2H2O Ho = H1o + H2o = - 116 kJ.mol-1
-

y 2y y
Ta có:
64.x + 116. y = 0,09

x + y x + 2y
 0,1 + 0,1 = 0,025
 0,5
 x = y = 5.10-4
Vậy:
V(dung dịch H3PO4) = (x + y)/0,1 = 0,01 (L) = 10 (mL)
V(dung dịch NaOH) = (x + 2y)/0,1 = 0,015 (L) = 15 (mL)

4 1
a. Do X phản ứng được với Ca nên X phải là một phi kim. Trong dung dịch kiềm X hòa
tan sinh ra một muối tan và một khí. Nguyên tố X có mặt trong cả hai thành phần ấy.
Trong hợp chất khí tồn tại liên kết X – H. Như vậy chỉ có thể có ba khả năng là silan,
photphin và amoniac. X sinh ra khi cho than cốc tác dụng với muối C (có chứa X) và 0,25
SiO2 nên X chỉ có thể là photpho.

Các phản ứng xảy ra như sau: (Có thể viết dạng ion thu gọn)
P4 + 3NaOH + 3H2O → 3NaH2PO2 + PH3
P4 + 6Ca → 2Ca3P2
2NaH2PO2 + 4CaOCl2 → Ca3(PO4)2 + CaCl2 + 2NaCl + 4HCl
2Ca3(PO4)2 + 10C + 6SiO2 → 6CaSiO3 + 10CO + P4
3Ca3(PO4)2 + 16Al → 3Ca3P2 + 8Al2O3
Ca3P2 + 6HCl → 3CaCl2 + 2PH3
2Ca3(PO4)2 + 6SiO2 → 6CaSiO3 + P4O10
P4O10 + 6H2O → 4H3PO4 (trong dung dịch axit loãng)
1
P4O10 + 12NaOH → 4Na3PO4 + 6H2O
(có thể viết phản ứng tạo muối axit).
b. - Dạng thù hình kém bền với không khí của X là P4 (phôt pho trắng) dễ thăng hoa do
trong tinh thể các phân tử liên kết với nhau bằng liên kết Van der Walls giữa các phân
tử không cực.
- P4 phản ứng ngay với oxi khi tiếp xúc do liên kết trong phân tử rất kém bền, góc liên
kết PPP bị bẻ cong ép tử 900 về 600.

2.
a. Ti là nguyên tố phân nhóm IVB, đứng gần đầu dãy 3d nên số oxy hóa cao bền. Phức
bát diện [Ti(aq)]3+, ion trung tâm có cấu hình e là: 𝑑𝜀1 𝑑𝛾0 , do vậy ion Ti3+ dễ dàng 0,25
nhường 1e để tạo Ti4+ đạt cấu hình bền của khí hiếm. Ion Ti3+ có tính khử đặc trưng.

10
Với phức bát diện [Cr(aq)]2+: có ΔB<P, nên đây là phức spin cao, ion trung tâm có cấu
hình e là 𝑑𝜀3 𝑑𝛾1 . Do dư 1 electron so với cấu hình bán bão hòa 𝑑𝜀3 nên có xu hướng 0,25
mất 1e chuyển về phức [Cr(aq)]3+. Do vậy, ion Cr2+ cũng có tính khử đặc trưng.
b. Dãy ion tạo nhiều hợp chất có màu: Cu2+(3d9); Rh4+(4d5); V2+(3d3)
Dãy ion tạo nhiều hợp chất không có màu: Cu+(3d10); Cd2+(4d10); W6+(4f14)
Màu của một hợp chất (hay vật thể) là do một phần ánh sáng nhìn thấy bị chất (vật 0,25
thể) đó hấp phụ, phần còn lại bị phản chiếu hay truyền qua chất (vật thể) vào mắt ta
gây ra cảm giác màu.
Theo thuyết trường tinh thể, nguyên tử chất tạo phức bị giảm sự suy biến các phân 0,25
lớp d,f trong trường tinh thể của các phối tử. Sự giảm suy biến này tạo ra năng lượng
tách trường tinh thể. Năng lượng này thường nằm trong vùng bước sóng của ánh sáng
khả kiến. Do vậy, các ion có electron chưa bão hòa trên các phân lớp d, f sẽ dễ tạo
hợp chất có màu. 0,25
5.1 a,

0,25

G1a do có nhiều công thức cộng hưởng hơn


b,

0,25

c,

0,5
Nếu vẽ
G5c
cho
0,25

5.2 a,
11
1 Do tương tác của các C-H aryl làm cho 2 vòng thơm nằm trong hai mặt phẳng chếch
nhau trong không gian, tạo cấu trúc ba chiều (tương tự allene)

0,5

2 Năng lượng hoạt hóa của sự quay quanh trục liên kết C-C rất cao và không thể diễn
ra ở điều kiện nhiệt độ phòng, như vậy quá trình racemic hóa không thể diễn ra, tạo
thành hai đồng phân quang học (hai đối phân).
b,

0,5

5.3 0,5

Do sự cộng hưởng p- làm cho C=O mang một phần tính chất của liên kết đơn. Trong
đó N có sự cộng hưởng tốt hơn O, do độ âm điện của N bé hơn O, nên liên kết C=O của
B dài hơn C. Trong hợp chất A thì nguyên tử N ở đầu cầu nên không thể tham gia cộng
hưởng do cơ cấu cầu cứng nên không có sự đồng phẳng xen phủ tạo cộng hưởng p-
được, vì vậy C=O trong A hoàn toàn có tính chất của nối đôi riêng biệt nên có độ dài
ngắn nhất.

6.1 a. 0,5

12
b. 0,5

6.2 0,25x4
chất =
1,0
điểm

0,5

7 a,
2,0

b, 0,5

13
8

Mỗi
chất
đúng
0,2x7 =
1,8
điểm

đúng 1
ezyme
được
0,1x7 =
E1 Hydroxylase E2 Decarboxylase E3 N-acyltransferase 0,7
E4 O-metyltransferase E5 Monoamine oxidase (MAO) điểm
E6 Dehydrogenase E7 Redectase

14
TRƯỜNG THPT CHUYÊN LÊ QUÝ ĐÔN KỲ THI HỌC SINH GIỎI CÁC TRƯỜNG THPT CHUYÊN
TỈNH BÌNH ĐỊNH KHU VỰC DUYÊN HẢI VÀ ĐỒNG BẰNG BẮC BỘ
LẦN THỨ XIV, NĂM 2023
ĐÁP ÁN ĐỀ THI ĐỀ XUẤT
(Đáp án đề thi gồm 19 trang) ĐỀ THI MÔN: HÓA HỌC LỚP 11
Thời gian: 180 phút (Không kể thời gian phát đề)

Câu 1 (2,5 điểm) Tốc độ phản ứng.


Tiến hành nghiên cứu động học của phản ứng nhiệt phân trong pha khí của hai đồng
phân có công thức phân tử C4H6O4, mạch carbon không vòng, không chứa liên kết O–H.
Biết:
- Mỗi đồng phân đều bị phân cắt đồng li thành hai mảnh với cấu tạo giống hệt nhau,
đây là giai đoạn quyết định tốc độ phản ứng.
- Cả hai đồng phân đều tạo thành cùng sản phẩm nhiệt phân: một chất vô cơ và một
alkane với tỉ lệ mol 2:1.
Tiến hành khảo sát: loại bỏ hoàn toàn không khí khỏi một bình phản ứng với dung
tích 1L không đổi ở nhiệt độ thường (khoảng 20 °C) bằng khí argon cho đến khi đạt đến
áp suất cuối 5943,2 Pa. Đun nóng bình phản ứng đến 220 °C, sau đó đưa vào 288 mg của
một trong hai đồng phân (dạng hơi) và bắt đầu đếm thời gian, sau 3831 giây, áp suất bình
phản ứng đạt 30000 Pa. Đun nóng bình phản ứng lên 234 °C và bắt đầu đếm thời gian,
sau 1218 giây nữa thì áp suất bình phản ứng đạt đến 35852 Pa.
Trong thí nghiệm tương tự với đồng phân còn lại, các nhà nghiên cứu quan sát thấy
rằng ở 90 °C, một nửa chất phản ứng đã bị tiêu hao 3820 giây, trong đó ở 100 °C chỉ cần
1260 giây để tiêu hao lượng chất như trên.
a) Xác định cấu tạo của hai đồng phân.
b) Đề xuất và viết cơ chế gốc gồm 3 giai đoạn cho phản ứng nhiệt phân của mỗi đồng
phân.
c) Tính giá trị hằng số tốc độ tương ứng và giá trị năng lượng hoạt hóa của phản ứng nhiệt
phân mỗi đồng phân.
d) Gán cho mỗi đồng phân giá trị năng lượng tạo thành mol trong giản đồ dưới đây và vẽ
giản đồ năng lượng của phản ứng tương ứng với mỗi đồng phân.
e) Bằng các tính toán đơn giản, tính năng lượng tạo thành mol các mảnh phân tử cho mỗi
đồng phân.
Để đơn giản bài toán, xem H ≅ 𝑈. Các chất khí là khí lí tưởng.
Năng lượng trung bình Eb (kJ.mol-1) của liên kết: C–O = 358; C–C = 347; O–O = 146.
T = 273 + t°C ; R = 0,082 L.atm.mol-1.K-1 = 8,314 J.mol-1.K-1; 1 cal = 4,18 J.
Câu Hướng dẫn chấm Điểm
1 a) 0,25
(2,5đ)

dimethyl oxalate diacetyl peroxide


b) Cơ chế: 0,25

CH3-O-CO-CO-O-CH3 → 2 CH3-O-C•=O
CH3-O-C• = O → H3C• + CO2
2H3C• → CH3-CH3

Và CH3-CO-O-O-CO-CH3 → 2CH3-CO-O•
CH3-CO-O• → H3C• + CO2
2H3C• → CH3-CH3
c) Ở 20 °C, bình phản ứng có Ar ở 5943,2 Pa; đun nóng đến 220 °C thì 0,25
áp suất tăng:
PV P1 .V1 P2 .V2 P1 P2
PV = nRT => = const => = ; V = const => =
T T1 T2 T1 T2
T2 493K
 P2 = P1. => (PAr)493K = 5943,2 Pa. ≅ 104 Pa
T1 293K

Áp suất riêng phần của 288 mg C4H6O4 (2,441.10-3 mol) được nạp
vào bình phản ứng 1L ở 220 °C:
PV = nRT
nRT 2,441.10−3 mol.8,314 J.mol−1 K−1 .493K
 P= = ≅ 104 Pa
V 10−3 m3
Áp suất riêng phần (t = 0) của C4H6O4 ở 493K:
(𝑃𝐶4𝐻6 𝑂4 )𝑡=0;493𝐾 = 104 Pa
Áp suất bình phản ứng tổng khi bắt đầu đếm giờ ở 220 °C:
(𝑃𝑡ổ𝑛𝑔 )𝑡=0;493𝐾 = (PAr)493K + (𝑃𝐶4 𝐻6 𝑂4 )𝑡=0;493𝐾
= 104 + 104 = 2.104 Pa
Khi quá trình nhiệt phân diễn ra, áp suất tăng theo hệ số tỉ lượng:
C4H6O4 → C2H6 + 2CO2
Ban đầu (𝑃𝐶4𝐻6 𝑂4 )𝑡=0 0 0
(t=0)
Thời điểm (1-x).(𝑃𝐶4𝐻6 𝑂4 )𝑡=0 𝑥(𝑃𝐶4 𝐻6 𝑂4 )𝑡=0 2𝑥(𝑃𝐶4𝐻6 𝑂4 )𝑡=0
(t)
Do đó, áp suất tổng trong bình phản ứng ở thời điểm t là
(Ptổng)t = PAr + (1 + 2x). (𝑃𝐶4𝐻6 𝑂4 )𝑡=0
30000−20000
Sau 3831 giây: x = = 0,5
2.10000
Một nửa lượng chất phản ứng đã tiêu thụ, vậy (t1/2)493K = 3831 s
Theo đề bài đã cho ở trên, giai đoạn phân cắt đồng li đầu tiên là giai
đoạn quyết định, đây là phản ứng 1 chiều bậc 1, nên: t1/2 = ln2 / k1
 (k1)493K = ln2 / 3831 = 1,81.10-4 s-1
Sau t1/2 = 3831s, đun nóng đến 220°C (493K): áp suất tổng là 3000Pa,
trong đó lượng đồng phân C4H6O4 chưa phản ứng chiếm
(𝑃𝐶4𝐻6 𝑂4 )𝑡=0 / 2 = 5000 Pa.
Do nhiệt độ của bình phản ứng tăng tức thời lên 234 °C (507 K), nên
toàn bộ hỗn hợp khí bị giãn nở, do đó áp suất tổng sẽ đạt
𝑇3 507𝐾
𝑃3 = 𝑃2 . = 30000 Pa. = 30852 Pa
𝑇2 493𝐾

Trong đó lượng đồng phân C4H6O4 chưa phản ứng chiếm


507𝐾
5000 Pa. = 5142 Pa
493𝐾
Đây là các giá trị mới (Ptổng)t=0;507K; (𝑃𝐶4 𝐻6 𝑂4 )𝑡=0; 507𝐾 ở thời điểm bắt
đầu của lần đếm thời gian thứ hai.
∆𝑃 35852−30852
Thay x = = = 0,5
2.(𝑃𝐶4 𝐻6 𝑂4 )𝑡=0 2 .5000

Dẫn đến (t1/2)507K = 1218 s .


Từ đó: (k1)507K = ln2 /(t1/2)507K = 5,69.10-4 s-1
Tính Ea:
0,25
𝑘𝑇2 𝐸𝑎 1 1
𝑙𝑛 = (𝑇 − 𝑇 ) => Ea = 169,93 kJ.mol-1
𝑘𝑇1 𝑅 1 2

Tính tương tự đối với sự nhiệt phân đồng phân thứ hai, ta có:
0,5
(t1/2)363K = 3820 s-1 ; (k1’)363K = 1,81.10-4 s-1
(t1/2)373K = 1260 s-1 ; (k1’)373K = 5,5.10-4 s-1
 Ea’ = 125,1 kJ.mol-1
d) 0,25

Dimethyl oxalate diacetyl peroxide


Độ bền tương đối của phân tử các đồng phân này có thể được ước
tính bằng cách xét đến sự khác biệt về cấu tạo ở mức độ các liên kết
hóa học:
C-C C-O O-O Năng lượng liên kết
trung bình
dimethyl oxalate 1 4 0 EC-C + 4.EC-O = 1779
diacetyl peroxide 2 2 1 2EC-C + 2.EC-O + EO-O
= 1556

Từ kết quả ta thấy, dimethyl oxalate có năng lượng được giải phóng để
tạo thành các liên kết lớn hơn nên phân tử bền hơn. Chênh lệch về năng
lượng tạo thành giữa các đồng phân chỉ ra rằng dimethyl oxalate bền
hơn (với giá trị được tính rất gần với chênh lệch của các giá trị trong
giản đồ -535 – (-756) = 221 kJ.mol-1
0,25

e) Ta có: 0,5
E(CH3-O-C•=O) = (-756 + 170) / 2 = -293 kJ.mol-1
E(CH3-O-C-O•) = (-535 + 125) / 2 = -205 kJ.mol-1

Câu 2 (2,5 điểm) Cân bằng và phản ứng trong dung dịch. Pin điện - Điện phân.
2.1. Silver oxalate (Ag2C2O4) là muối ít tan trong nước. Ở 25 oC, độ tan của silver oxalate
trong pH = 7 là 2,06.10-4 mol/L. Một trong các yếu tố ảnh hưởng đến độ tan của muối là
pH, bởi anion oxalate (C2O42-) phản ứng với các ion H+. Tính độ tan của silver oxalate ở
25 oC trong nước được acid hóa với pH = 5.
Biết các hằng số acid của oxalic acid: Ka1 = 5,6.10-2 và Ka2 = 6,2.10-5.
2.2. Để chế tạo pin nhiên liệu với hoạt động của butane và oxygen, người ta đã khảo sát
một pin với cathode được nạp oxygen và anode được nạp butane. Nước tạo thành được
dẫn ra từ ngăn cathode. Khoảng giữa các điện cực được tách ra bởi một màng mà chỉ các
ion H+ lọt qua được, nên có thể dẫn được điện. Giả định rằng quá trình khảo sát ở nhiệt
độ chuẩn 298K và áp suất chuẩn 1 bar, tất cả các khí đều là khí lí tưởng, và nước được
tạo thành ở trạng thái lỏng.
a) Viết các phương trình bán phản ứng ở cathode và anode.
b) Tính suất điện động chuẩn (EMF) của pin nhiên liệu butane - oxygen.
c) Tính hiệu suất nhiệt động học lí tưởng của pin nhiên liệu butane. Hiệu suất nhiệt động
học lí tưởng là tỉ lệ giữa công suất tối đa và năng suất tỏa nhiệt của các pin nhiên liệu ở
nhiệt độ chuẩn 298 K. Bỏ qua sự phụ thuộc của enthalpy và entropy vào nhiệt độ trong
tất cả các tính toán.
Cho biết: Hằng số Faraday F = 96485
C4H10(g) O2(g) H2O(l) CO2(g) C(s) H2(g)
ΔfH° (kJ.mol−1) −126 −286 −393
S° (J.K−1.mol−1) 205 70 214 6 131
ΔfG° (kJ.mol−1) −17

Câu Hướng dẫn chấm Điểm


2.1 Ag2C2O4(s) ⇌ 2Ag+(aq) + C2O42-(aq) 0,5
(1,0 [] 2S S
đ) C2O42- + H+ ⇌ HC2O4- Ka2-1
HC2O4- + H+ ⇌ H2C2O4 Ka1-1
Ta có: C(C2O42-) = S = [C2O42-] + [ HC2O4- ] + [H2C2O4]
Ka1 .Ka2
 [C2O42-] = S. = S.α
[H ] +Ka1 .[H+ ]+Ka1 .Ka2
+ 2

Ở 25°C, pH = 7 => [H+] = 10-7 mol/L => α ≅ 1 => [C2O42-] ≅ S


 Ks = [Ag+]2.[C2O42-] = 4S3 = 4.(2,06.10-4)3 = 3,5.10-11
Ở 25°C, pH = 5 => [H+] = 10-5 mol/L => α ≅ 0,861
0,5
 Ks = [Ag+]2.[C2O42-] = 4S’3α = 4.S’3.0,861
 S’ = 2,166.10-4 mol/L.
2.2 a) Cathode: O2 + 4H+ + 4e → 2H2O 0,5
(1,5 Anode: C4H10 + 8H2O → 4 CO2 + 26H+ + 26e
đ) 0,5
b) Phản ứng trong pin là: 2C4H10 + 13O2 → 8 CO2 + 10H2O
Ở nhiệt độ 298K, ta có:
fG°(H2O(l)) = - 286 - 298.[70 – (131 + ½.205)].10-3
= -237,3 kJ.mol-1
fG°(CO2(g)) = -393 – 298.[214 – (6+205)].10-3 = -393,9 kJ.mol-1
fG°(C4H10(g)) = -17 kJ.mol-1 ; fG°(O2(g)) = 0
=> rG°(CO2(g)) = 8. fG°(CO2(g)) + 10. fG°(H2O(l))
– 2. fG°(C4H10(g)) – 13. fG°(O2(g))
= [8.(-393,9) + 10.(-237,3)] – [2.(-17) + 13.0]
= -5490 kJ.mol-1
∆𝑟 𝐺 ° −5490.103
𝐸° = = = 1,09V
𝑛.𝐹 52.96485
c) Hiệu suất nhiệt động lí tưởng là: 0,5
∆𝑟 𝐺 ° −5490.2
t = = = 0,954
∆𝑟 𝐻° [8.(−393)+10.(−286)]−[2.(−126)+13.0]

Câu 3 (2,5 điểm) Nhiệt động học và cân bằng hóa học.
3.1. A. Wexler (J.Res.Natl.Bur.Stand. A Phys.Chem., 1976, 80A, 775-785) đã thực hiện
các phép đo áp suất hơi nước ở các nhiệt độ khác nhau và các dữ kiện được cho trong
giản đồ bên dưới. Trong bình dung tích 100 L, một chất khí bền được bão hòa hơi nước
ở nhiệt độ 40 °C và áp suất tổng bằng 1000 mmHg. Xác định khối lượng nước được
ngưng tụ trong điều kiện đẳng áp nếu nhiệt độ của hỗn hợp khí giảm đi 15 °C.

Giản đồ áp suất hơi nước


3.2. Đốt cháy hoàn toàn 1 mol alkane trong khí oxygen tạo thành hỗn hợp sản phẩm chỉ
chứa các chất khí. Xác định công thức phân tử của alkane. Biết enthalpy phản ứng rH
(kJ.mol-1) ở 25 °C và 327 °C lần lượt là -1560,0; -1554,2. Và các giá trị nhiệt động của
các chất cho trong bảng sau:
Cấu tử CO2(g) H2O(g) O2(g) alkane(g)
𝑑𝐻 8,87 8,03 7,02 12,58
CP = (cal.K-1.mol-1)
𝑑𝑇
Câu Hướng dẫn chấm Điểm
3.1 Ta có: 0,5
(1,5 đ) - TT ban đầu (1), ở t = 40 °C: khí(1), H2O(1), 𝑝𝐻2 𝑂(1) = 7366 Pa
- TT cuối (2), ở t = 25 °C: khí(2), H2O(2), 𝑝𝐻2 𝑂(2) = 2995 Pa
𝑛𝐻2 𝑂 𝑛𝑘ℎí(𝑡ổ𝑛𝑔) .𝑝𝐻2 𝑂
Và: 𝑝𝐻2 𝑂 = . 𝑃 => 𝑛𝐻2 𝑂 =
𝑛𝑘ℎí(𝑡ổ𝑛𝑔) 𝑃

Sau khi nhiệt độ giảm và một phần hơi nước bị ngưng tụ sẽ còn lại lượng
chất khí là nkhí(2).
ntổng = nkhí(2) + 𝑛𝐻2 𝑂 => nkhí(2) = ntổng – 𝑛𝐻2 𝑂
Số mol nước ngưng tụ: 𝑛𝐻2 𝑂 = 𝑛𝐻2 𝑂(1) – 𝑛𝐻2 𝑂(2)
𝑛𝑘ℎí(𝑡ổ𝑛𝑔) .𝑝𝐻2 𝑂(1) 𝑝𝐻2 𝑂(2)
Ta có: 𝑛𝐻2 𝑂 = – (nkhí(tổng) – 𝑛𝐻2 𝑂 ).
𝑃 𝑃
𝑛𝑘ℎí(𝑡ổ𝑛𝑔) .𝑝𝐻2 𝑂(1) 𝑝𝑘ℎí(𝑡ổ𝑛𝑔) .𝑝𝐻2 𝑂(2) 𝑝𝐻2 𝑂(2)
𝑛𝐻2 𝑂 = – + 𝑛𝐻2 𝑂 .
𝑃 𝑃 𝑃
𝑝𝐻2 𝑂(2) 𝑛𝑘ℎí(𝑡ổ𝑛𝑔)
𝑛𝐻2 𝑂 – 𝑛𝐻2 𝑂 . = (𝑝𝐻2 𝑂(1) – 𝑝𝐻2 𝑂(2) )
𝑃 𝑃
𝑛𝑘ℎí(𝑡ổ𝑛𝑔)
𝑛𝑘ℎí(𝑡ổ𝑛𝑔) (𝑝𝐻2 𝑂(1) – 𝑝𝐻2 𝑂(2) ) 𝑝𝐻2 𝑂(1) – 𝑝𝐻2 𝑂(2)
𝑛𝐻2 𝑂 = 𝑃
𝑝𝐻 𝑂(2) = 𝑛𝑘ℎí(𝑡ổ𝑛𝑔) .
𝑃 1− 2 𝑃−𝑝𝐻2 𝑂(2)
𝑃
1000
( )𝑎𝑡𝑚.100𝐿 0,5
Ta được: n = 760
= 5,13 mol
0,082𝐿.𝑎𝑡𝑚/(𝑚𝑜𝑙.𝐾).313𝐾
7366−2995
 𝑛𝐻2 𝑂 = 5,13.1000.101325 = 0,17 mol
−2995
760
0,5
 𝑚𝐻2 𝑂 = 3,06 gam
3.2 ̅̅̅
Ta có: ∆𝑟 𝐻600𝐾 = ∆𝑟 𝐻298𝐾 + ∆𝐶 𝑃 ∆𝑇 1,0
(1,0 đ) ̅̅̅
 ∆𝐶
−1554,2 𝑘𝐽.𝑚𝑜𝑙 −1 +1560 𝑘𝐽.𝑚𝑜𝑙 −1
= 19,21 𝐽. 𝑚𝑜𝑙 −1 . 𝐾 −1
𝑃= 302𝐾

Từ phương trình đốt cháy alkane:


3𝑛+2
CnH2n+2 + 𝑂2 → nCO2 + (n+1) H2O
2
Alkane cần xác định là C2H6 (ethane).

Câu 4 (2,5 điểm) Hóa nguyên tố (Kim loại, phi kim nhóm IVA, VA). Phức chất.
4.1. A1 là một phức chất ngậm nước của đồng (copper). Khi cho dung dịch A1 tác dụng
với dung dịch barium chloride thu được kết tủa trắng A2 không tan được trong acid và
kiềm. Nung A2 ở nhiệt độ 700 °C với lượng dư carbon thu được A3. A3 tan trong
hydrochloric acid giải phóng khí A4 có mùi đặc trưng. Khí A4 tác dụng với dung dịch
A1 thu được kết tủa đen A5 không tan trong hydrochloric acid. Đun nóng dung dịch A1
với kiềm thì tạo được khí A6 và kết tủa màu đen A7. Biết rằng tỉ lệ khối lượng A5 và A7
thu được từ cùng một lượng chất A1 ban đầu là 1,2 : 1. A7 tan trong hydrochloric acid
tạo thành dung dịch chất A8. Khi cho dung dịch A8 tác dụng với dung dịch sodium
carbonate, thu được khí A9 và một kết tủa A10 có màu.
Nung A10 lại thu được khí A9 và chất rắn màu đen A7. Khi nung A1 ở 150 °C –
200 °C, thu được một chất bột màu trắng A11 có khối lượng bằng 0,65 lần khối lượng
A1. A11 dễ tan trong nước tạo thành dung dịch có màu và hỗn hợp sản phẩm dễ bay hơn
A12. A12 có tỉ khối hơi so với hydrogen là 8,6 và A12 được hấp thụ hoàn toàn bằng
sulfuric acid đặc.
a) Xác định công thức hóa học của các chất từ A1 đến A12. Giải thích ngắn gọn.
b) Viết các phương trình hóa học tạo thành các chất từ A2 đến A12 trong bài.
4.2. Từ kim loại M (có số hiệu nguyên tử ZM < 37), thực hiện các sơ đồ chuyển hóa sau:

M + nCO ⎯⎯⎯⎯ →A
0
150 ,15 atm

A + 4 KOH ⎯⎯ → B + C + 2 H 2O
0
t

a) Xác định chất B. Biết một hợp phần trong B có cấu trúc tứ diện, carbon chiếm 19,512%
khối lượng B.
b) Cho A phản ứng với dicyclopentadiene, đun nóng thu được D và hỗn hợp khí Y (CO,
H2) và có tỉ khối so với H2 là 85/7. Ở điều kiện thường, D là tinh thể màu đỏ tím đậm, dễ
dàng hòa tan trong các dung môi hữu cơ phân cực vừa phải như chloroform, pyridine, ít
tan trong các dung môi không cực (như benzene, CCl4) và không tan trong nước. Sản
phẩm của sự khử D bằng kim loại kiềm hoặc hydride được sử dụng rộng rãi vì khả năng
dễ alkyl hóa, acyl hóa của nó. Viết các đồng phân tương ứng của D.
Câu Hướng dẫn chấm Điểm
4.1 a) Xác định A1: 0,25
(2,0 đ) - A1 + NaOH tạo khí A6 => A1 là phức của ion Cu với NH3.
- A1 + BaCl2 => kết tủa trắng A2 không tan được trong acid và kiềm
=> A2 là BaSO4
Vậy, A1 là [Cu(NH3)4]SO4. xH2O
Xác định A12:
A12 có M = 17,2; A12 là hỗn hợp sản phẩm nên có thể chứa H2O và
một khí có Mkhí < 17. A12 được hấp thụ hoàn toàn bằng sulfuric acid
đặc => A12 chứa NH3 và H2O.
- Với MA12 = 17,2 => tỉ lệ số mol NH3 : H2O = 4 : 1 => phù hợp với
thành phần A1. Vậy với x = 1 thì A1: [Cu(NH3)4]SO4. H2O
Ta có tỉ lệ khối lượng A5 : A7 = 96 : 80 = 1,2 => A5: CuS ; A7: CuO
Vậy, các chất từ A2 đến A12: 0,75
A2: BaSO4 ; A3: BaS ; A4: H2S ; A5: CuS;
A6: NH3; A7: CuO; A8: CuCl2 ; A9: CO2;
A10: [Cu(OH)]2CO3; A11: CuSO4; A12: NH3 + H2O.
b) Các phương trình phản ứng tạo chất A2 đến A12: 1,0
[Cu(NH3)4]SO4 + BaCl2 → [Cu(NH3)4]Cl2 + BaSO4
BaSO4 + 4C → BaS + 4CO
BaS + 2HCl → BaCl2 + H2S
[Cu(NH3)4]SO4 + H2S (dư) → CuS + (NH4)2SO4 + (NH4)2S
𝑡°
[Cu(NH3)4]SO4 + 2NaOH → CuO + 4NH3 + Na2SO4 + H2O
CuO + 2HCl → CuCl2 + H2O
2CuCl2 + 2Na2CO3 + H2O → [Cu(OH)2]CO3 + CO2 + 4NaCl
𝑡°
[Cu(OH)2]CO3 → 2CuO + CO2 + H2O
𝑡°
[Cu(NH3)4]SO4.H2O → CuSO4 + 4NH3 + H2O
4.2 a) A có CTTQ: M(CO)n 0,25
(0,5 đ) Từ phương trình phản ứng: M(CO)n + 4KOH → B + C + 2H2O
=> C là muối K2CO3 => B: K2[M(CO)n-1] mà [M(CO)n-1]2- có cấu trúc
tứ diện => n-1 = 4 hay n = 5.
Trong B: %C = 19,512% => M = 56 (Fe)
Vậy B là K2[Fe(CO)4]
b) D là Fe2(CO)4(C5H5)2 0,25

Câu 5 (2,5 điểm) Đại cương hữu cơ.


5.1. Tại sao pKa của các proton Ha trong 1-acetylcyclohexene cao hơn pKa của các proton
H b?

5.2. Giải thích tại sao hơn chất A bền hơn hợp chất B nhiều?

5.3. Giải thích tại sao A kém tan trong nước hơn B, mặc dù cả hai hợp chất này có các
nhóm chức giống nhau.

5.4. Avobenzone và dioxybenzone là hai loại kem chống nắng thương mại. Sử dụng
những nguyên lí về tính tan, dự đoán loại kem chống nắng nào dễ bị rửa trôi khi người
dùng đi bơi. Giải thích lựa chọn của bạn.
5.5. Hợp chất X dưới đây có khả năng tác dụng với acid để giải phóng hydrogen. Hãy giải
thích điều này dựa trên cấu trúc của nó.

Câu Hướng dẫn Điểm


5.1 Sự tách loại Ha tạo thành 2 cấu trúc cộng hưởng mà điện tích âm 0,25
(0,5đ) không bao giờ ở trên O. Sự tách loại Hb tạo thành 3 cấu trúc cộng hưởng
mà một trong số đó có điện tích âm ở trên O, làm cho base liên hợp bền
hơn. Chính vì vậy Hb có tính acid mạnh hơn (pKa thấp hơn.)

0,25

5.2 Cấu trúc cộng hưởng thứ hai của A cho thấy nó có vòng liên hợp hoàn 0,25
(0,5đ) toàn với 6 π electron, vòng này có tính thơm và rất bền. Tương tự, với
cấu trúc B thì vòng liên hợp nhưng chỉ có 4 π electron (phản thơm),
chính vì vậy rất kém bền.

0,25

5.3 Trong A, các nhóm OH và CHO đủ gần nhau để tạo thành liên kết 0,25
(0,5đ) hydrogen nội phân tử. Trong B, hai nhóm này xa nhau. Do hai nhóm
chức phân cực tham gia tạo liên kết hydrogen nội phân tử nên A khó tạo
liên kết hydrogen với nước hơn B. Điều này dẫn đến độ tan trong nước
của A < B (có sẵn hai nhóm chức để tạo liên kết hydrogen với dung môi
H2O)
0,25

5.4 So sánh các nhóm chức trong 2 thành phần của kem chống nắng. 0,25
(0,5đ) Dioxybenzone có khả năng bị rửa bởi nước nhất nó chứa 2 nhóm
hydroxy và tan trong nước nhiều hơn.

0,25

5.5 Cấu dạng của X như sau: 0,25


(0,5đ)

Lúc này carbocation hình thành ở carbon trung tâm sẽ được bền hóa
mạnh do tương tác với ba cặp electron của ba nguyên tử N kế cận khiến
cho liên kết C - H trung tâm rất dễ phân ly nên có thể tương tác được 0,25
với acid để sinh ra H2.

Câu 6 (2,5 điểm) Sơ đồ tổng hợp hữu cơ. Cơ chế phản ứng hóa hữu cơ.
6.1. Cadien [C15H24] được tìm thấy trong tinh dầu một số loại thực vật, đề hydrogen hóa
Cadien bằng S tạo thành Cadalen [C15H18]. Hoàn thành sơ đồ tổng hợp Cadalen từ Cavone
theo dãy sau:

6.2. Giải thích cơ chế trong các phản ứng sau:


a)

b)

c)

d)

Câu Hướng dẫn Điểm


6.1
(1,0đ)

Mỗi chất 0,1 đ x 10 chất = 1,0 điểm


6.2 a) 0,5
(1,5đ)
b) 0,25

c) 0,25

d) 0,5

Câu 7 (2,5 điểm) Xác định cấu trúc các chất hữu cơ (mô tả sơ đồ tổng hợp bằng lời
dẫn)
7.1. Đun hồi lưu but-2-enoic acid với cyclopenta-1,3-dien rồi cho sản phẩm phản ứng với
O2 singlet/LDA thu được hợp chất A. Khử hóa A lần lượt bằng (EtO)3P và LiAlH4 thu
được vic-diol B. Hydrogen hóa B bằng H2, Pd/C rồi oxi hóa bằng NaIO4 thu được hợp
chất C. Ở điều kiện -78oC, C phản ứng với 2-(2-ethyl iodide)-1,3-dioxolan/NaHMDS rồi
xử lí hỗn hợp phản ứng bằng sulfuric acid loãng thu được hợp chất D. Tiếp tục cho D
phản ứng với ethyl-2-(dietoxyphosphoryl)propanoate/NaH, tách lấy đồng phân Z cho
phản ứng với CH2Br2/Zn/TiCl4 rồi cuối cùng khử hóa bằng LiAlH4 thu được β-santalol.
Xác định cấu trúc các chất A, B, C, D và β – santalol.
7.2. Hợp chất B là dẫn xuất của hệ vòng thiazole được sử dụng làm thuốc an thần. B có
thể được tổng hợp theo quy trình như sau: Bromine hóa B1 bằng dung dịch Br2 trong dung
môi CH3COOH thu được hợp chất B2. Cho B2 phản ứng với hexamethylenetetraamine rồi
thủy phân sản phẩm muối ammonium bậc IV bằng dung dịch HCl loãng cho hợp chất B3
(C8H8FNO). Đun nóng hỗn hợp B3 và 2,5-dimethoxytetrahydrofuran có mặt xúc tác acetic
acid cho sản phẩm B4 (C12H10FNO). Xử lý hợp chất B4 với Cu(SCN)2 thu được B5
(C13H9FN2OS). Đun nóng B5 với MeONa trong dung môi DMF tạo thành hợp chất B.
Xác định công thức cấu tạo các chất chưa biết trong quá trình tổng hợp trên. Cho biết:

Câu Hướng dẫn Điểm


7.1 0,25x3
(1,25
đ)

0,5

7.2.
(1,25
đ) 0,25x2

0,25x3
Câu 8 (2,5 điểm) Hóa học các hợp chất thiên nhiên (Cacbohidrat và các hợp chất hữu
cơ chứa nito đơn giản)
8.1. Tiến hành phân cắt trực tiếp một disaccharide A (C12H22O11) quan trọng trong công
nghiệp và trong đời sống bằng HIO4 thấy cần dùng 3 mol HIO4 sinh ra một sản phẩm B
và 1 mol HCOOH. Xử lý B bằng dung dịch bromine rồi sau đó thủy phân thu được hỗn
hợp ba chất gồm D-glyceric acid, glyoxylic acid, hydroxypyruvic acid với tỉ lệ số mol lần
lượt là 2 : 1 : 1. Biết rằng disaccharide A không có tính khử, và khi bị thủy phân bằng
enzym α-glucoxidase thu được hai đường mà khi tác dụng với PhNHNH2 cho cùng một
osazon như D-glucose. Hãy xác định công thức Haworth của A và B.
8.2. Nhóm Azanthracene là các hợp chất thiên nhiên thu hút được nhiều sự quan tâm của
các nhà khoa học trên tất cả các phương diện nhờ những đặc tính dược lí đặc biệt. Ngay
từ năm 1999, loại cây Hao-Laam trồng ở Thailand đã được ghi nhận là có chứa Markanin
A (I), có khả năng chống ung thư và sốt rét. Năm 2000, Kalasinamide (II) được phân lập
từ cây Polyalthia suberosa, được sử dụng là tiền chất của Markanin A. Quy trình tổng
hợp đơn giản các hợp chất này từ naphthoquinone-1,4 đã được các nhà khoa học Đức
công bố năm 2009 theo sơ đồ sau:

Hoàn thành sơ đồ tổng hợp Kalasinamide (II) và Markanin A (I) và xác định cấu
tạo các chất A - I. Biết rằng ammonium cerium (IV) nitrate - sử dụng trong giai đoạn cuối
của quy trình - là một chất oxi hóa mạnh.
Câu Hướng dẫn Điểm
8.1 0,25
(1,0đ)

0,25
0,25

0,25

Dựa vào sản phẩm cắt mạch disaccharide bằng HIO4 có thể thấy rằng
disaccharide này được tạo thành từ một đơn vị pyranose nối với một đơn
vị furanose qua liên kết α – glycozit giữa hai đầu C1. Chỉ có ba đường
tạo cùng một osazon với D-glucose là D-glucose, D-mannose và D-
fructose.
Do disaccharide này chiếm vai trò quan trọng trong cuộc sống cũng
như trong công nghiệp nên nó chỉ có thể là sucrose gồm một đơn vị
glucopyranose nối với một đơn vị fructofuranose.
8.2 0,5
(1,5đ)

0,25

0,25
0,25

0,25

--------------------
Giáo viên ra đề:
1. Đặng Họa My – 0982142309
2. Nguyễn Thị Kim Thu – 0972159219
TRƯỜNG THPT CHUYÊN KỲ THI HỌC SINH GIỎI CÁC TRƯỜNG THPT
LÊ QUÝ ĐÔN
CHUYÊN KHU VỰC DUYÊN HẢI VÀ ĐỒNG BẰNG
THÀNH PHỐ ĐÀ NẴNG
------------------- BẮC BỘ LẦN THỨ XIV
Môn thi: HÓA HỌC - LỚP 11
ĐÁP ÁN ĐỀ ĐỀ NGHỊ
NĂM HỌC 2022 - 2023

Thời gian: 180 phút (không kể thời gian giao đề)

Cho biết: Hằng số Plank h = 6,63.10-34 J.s; Hằng số Faraday F = 96485 C.mol-1; Tốc độ ánh sáng c =
3.108m.s-1; Số Avogadro NA = 6,02.1023mol-1; Khối lượng electron me = 9,1094.10-31kg; 0oC =273 K; R =
8,314 J.mol-1.K-1; O =16; H = 1; Ca = 40; P = 31; Cl = 35,5 .

Câu 1 (2,5 điểm) Tốc độ phản ứng.


1.1. Hydrocarbon 4 vòng (A) có nhiều tiềm năng trong công nghệ năng lượng Mặt trời. Dưới tác động
của bức xạ tử ngoại, chất B bị đồng phân hóa thành A. Phản ứng chuyển hóa ngược lại A → B là quá
trình tỏa nhiệt (∆Ho = –92,5 kJ/mol). Đây là một tính chất đầy hứa hẹn để chế tạo các thiết bị lưu trữ
năng lượng Mặt trời.
Xét một pin năng lượng Mặt trời với 10 mol B. Dưới tác động của ánh sáng, B chuyển thành A với độ
chuyển hóa 85 %, sau đó pin không bị chiếu xạ nữa.
a) Tính năng lượng cực đại nhận được từ mẫu pin này nếu hiệu suất của bộ thu là 65 %.
Vấn đề chính khi sử dụng chất vòng 4 cạnh này là việc lựa chọn các hệ xúc tác để làm tăng tốc độ phản
ứng tỏa nhiệt A → B tại nhiệt độ sử dụng pin. Khi không có xúc tác, phản ứng đồng phân hóa nhiệt của
A ở nhiệt độ phòng gần như không xảy ra mà chỉ bắt đầu ở các nhiệt độ cao hơn (chu kì bán chuyển hóa
ở 160 oC là 2,58 giờ; còn ở 200 oC là 5,14 phút).
b) Tính năng lượng hoạt hóa của phản ứng đồng phân hóa A → B nếu phản ứng này diễn ra theo quy
luật động học bậc nhất.
c) Sau bao lâu bộ thu có thể tiếp nhận 90 % năng lượng tích lũy trong hợp chất A ở 25 oC nếu không có
xúc tác.
1.2. Lớp ozone trong khí quyển Trái Đất là một bộ lọc tia tử ngoại tự nhiên, giúp hấp thụ bức xạ tử ngoại
nguy hiểm từ Mặt Trời trước khi chúng chạm tới bề mặt Trái Đất. Sự suy giảm lượng ozone trong khí
quyển có thể dẫn đến những hệ quả nguy hại tới sự sống trên Trái Đất.
Ozone chủ yếu được tìm thấy tại tầng bình lưu ở độ cao khoảng 15 đến
50 km với nồng độ cực đại ở độ cao khoảng 25 km.
Năm 1930, nhà hóa học người Anh Sydney Chapman đã đề xuất cơ chế
cho sự tạo thành ozone khí quyển, bao gồm bốn phản ứng (được gọi là
“chu trình Chapman” hay “chu trình ozone-oxygen”):

1. O2 + h ⎯⎯ → 2O
k
1
k1 = 3,0.10-12 s-1
2. O + O2 + M ⎯⎯ → O3 + M
k
2
k2 = 1,2.10-33 cm6.phân tử-2.s-1
3. O3 + h ⎯⎯ → O + O2
k
3
k3 = 5,5.10-4 s-1
4. O3 + O ⎯⎯ → 2O2
k
4
k4 = 6,9.10-16 cm3.phân tử-1.s-1

1
Sự hấp thụ bức xạ tử ngoại trong phản ứng 3 giải thích cho những đặc tính bảo vệ của lớp ozone. Phân
tử M có thể là bất kì phân tử nào được tìm thấy trong khí quyển (thường là N2). Kết quả của các phản
ứng này là một trạng thái bền được thiết lập, và nồng độ ozone trong khí quyển được duy trì gần như
không đổi. Lượng ozone trong tầng bình lưu biến đổi theo vĩ độ và mùa, nhưng trung bình không vượt
quá vài phần triệu (ppm). Mặc dù nồng độ này dường như rất thấp, nhưng nó hữu hiệu để ngăn chặn
khoảng 95 – 98% bức xạ tử ngoại của Mặt Trời.

hc
Dữ liệu tham khảo: Quang năng E = (h = 6,63.10-34J.s); Nồng độ [M] ở độ cao 25 km xấp xỉ bằng

1018 phân tử.cm-3; Tỉ lệ  O 3  được xác định theo thực nghiệm xấp xỉ bằng 10 ppm.
O 2 
a) Tính bước sóng cực đại của ánh sáng Mặt Trời có thể gây ra sự phân li của phân tử oxygen theo phản
ứng 1, biết năng lượng liên kết của phân tử oxygen là 498 kJ.mol-1.
b) Sử dụng cơ chế trên, hãy giải thích định tính tại sao nồng độ ozone biến đổi không đều theo độ cao
và đi qua một cực đại?
c) Giải thích tại sao phản ứng 2 cần sự hiện diện của một phân tử M?
d) Viết biểu thức tốc độ của mỗi phản ứng trong chu trình Chapman. Giả sử rằng nồng độ của O và O3
không đổi (có nghĩa là giả sử rằng tốc độ tạo thành các tiểu phân này cũng bằng tốc độ phân hủy của

chúng), hãy dẫn ra biểu thức cho tỉ lệ


O3  .
O 2 

e) Sử dụng biểu thức đã dẫn ra ở trên, hãy tính tỉ lệ


O3  ở độ cao 25 km tính từ bề mặt Trái Đất. So
O 2 
sánh giá trị tính được với giá trị thực nghiệm. Giải thích cho sự khác biệt giữa chúng.
1 Nội dung Điểm
1.1 a. E = 10  0,85  0,65  92,5 = 511,1 (kJ/mol) 2/8
b. ln 2 2/8
Đối với phản ứng bậc nhất: k = . Sử dụng phương trình Arrhenius:
t1/2

k  E  1 1 t 
ln  1  = a  −  = ln  1/2(2) 
 k 2  R  T2 T1   t1/2(1) 
 
Thế các giá trị t1/2(1) = 155 phút và t1/2(2) = 5,14 phút; T1 = 433 K và T2 = 473 K.
Ta có: Ea = 144,95 kJ/mol
c. ln 2 2/8
k 433 = = 4, 47.10−3 (phút-1)
t1/2(433)
Hằng số tốc độ phản ứng ở 298 K:

k  E  1 1 
ln  433  = a  −  = 17,81  k298 = 5,36.10 (phút )
-11 -1

 k 298  R  298 433 

2
C  ln10
ln  0  = kt  t = = 4, 296.1010 phút  81 735 năm
 C k
2. a. hc hc 6,63.10−34  3.108  6,02.1023 2/8
E= = = = 2, 40.10−7 m = 240nm
 E 498000
b. Theo cơ chế đã cho, ozone được tạo thành theo phản ứng 2 với v2 = k2[O][O2][M], 2/8
nghĩa là tỉ lệ thuận với các nồng độ [O], [O2] và [M]. Ở cao độ rất lớn, nồng độ
oxygen nguyên tử O khá lớn (do cường độ bức xạ tử ngoại lớn), nhưng nồng độ
O2 và M thì lại rất thấp. Mặt khác, ở cao độ thấp, trong khi nồng độ O2 và M cao
hơn, thì nồng độ oxygen nguyên tử lại rất thấp (do không đủ
bức xạ tử ngoại để tạo thành chúng). Điều này dẫn tới việc nồng độ
ozone biến đổi không đều theo độ cao và đi qua một cực đại.
c. Phân tử M hấp thụ năng lượng giải phóng trong phản ứng 2. Nếu không có phân 2/8
tử M thì phân tử ozone tạo thành sẽ ngay lập tức bị phân hủy trở lại.
d. Các biểu thức tốc độ phản ứng: r1 = k1[O2]; r2 = k2[O][O2][M]; r3 = k3[O3]; r4 = 6/8
k4[O3][O].
r2 = r3 + r4 hay k2[O][O2][M] = k3[O3] + k4[O3][O] (1)
2r1 + r3 = r2 + r4 hay 2k1[O2] + k3[O3] = k2[O][O2][M] + k4[O3][O] (2)
k1[O 2 ]
Cộng hai biểu thức này ta có: k1[O2] = k4[O3][O]  [O] = (3)
k 4 [O3 ]
Lấy biểu thức (2) trừ đi biểu thức (1), ta có: k2[O][O2][M] = k1[O2] + k3[O3]
k1 [O ] k [O ]
Theo đề bài ta có: = 5,5.10−9 và 3  10 −5 nên 1 2  5,5.10 −4
k3 [O 2 ] k 3[O3 ]
 k1[O2] k3[O3] và có thể bỏ qua k1[O2]. Vậy, biểu thức thu được:
k2[O][O2][M] = k3[O3] (4).
Thay (3) vào (4), ta được: k1k2[O2]2[M] = k3k4[O3]2
1

[O3 ]  k1k 2 [M]  2


 = 
[O2 ]  k 3k 4 
e. Thay các giá trị số vào, ta có: 2/8
1

[O3 ]  3,0.10−12  1, 2.10−33  1,0.1018  2 −5


= −4 −16  = 9,7.10
[O2 ]  5,5.10  6,9.10 
Giá trị nhận được lớn hơn khoảng 10 lần so với giá trị xác định bằng thực nghiệm
(~10 ppm). Khác biệt này được giải thích bởi thực tế rằng chu trình Chapman chỉ
xét đến những phân hủy “tự phát” của các phân tử ozone theo các phản ứng 3 và

3
4. Trong thực tế, các cơ chế phân hủy khác của ozone cũng diễn ra, ví dụ như khi
có mặt các gốc NO, OH, Cl.

Câu 2 (2,5 điểm) Cân bằng và phản ứng trong dung dịch. Pin điện - Điện phân.
1.1. Mạ bạc, theo đó bạc được phủ lên một bề mặt kim loại, được sử dụng để trang trí và tăng tính dẫn
điện. Trong quá trình mạ bạc, thường sử dụng dung dịch kiềm chứa Ag+ và CN-. Kim loại được mạ
đóng vai trò cathode, và các ion bạc được mạ lên bề mặt kim loại bởi phản ứng khử hoá kết tủa. Bạc
tinh khiết được sử dụng làm anode, từ đây các ion bạc được đưa vào các dung dịch bởi phản ứng oxi
hoá hoà tan.
Biết: Ag+ + e- Ag(s) có Eo = 0,799 V. Trong dung dịch, Ag+ tạo thành một số loại phức chất
với CN- và OH- với các hằng số tạo thành: Ag+ + CN- + OH- Ag(OH)(CN)- có KOHCN = 1013,20;
Ag(CN)i(i−1)− : lgi = 20,40; 21,40; 20,80.
Trừ khi có các ghi chú khác, sử dụng các dữ kiện sau [CN-] = [OH-] = 1,00 M; và nồng độ tổng của
các tiểu phân chứa Ag là 1,00 mM để trả lời các câu hỏi dưới đây. Sử dụng nhiệt độ tuyệt đối T =
298,2 K.
a) Viết ra 5 tiểu phân hoà tan chứa bạc trong dung dịch này theo trình tự nồng độ giảm dần. Tính nồng
độ của tiểu phân có nồng độ lớn nhất.
b) Viết bán phản ứng trong đó tiểu phân hoà tan chứa bạc có nồng độ cao nhất ở ý a) bị khử thành
Ag(s). Tính Eo của bán phản ứng.
c) Nếu [CN-] thay đổi khác 1,00 M thì trình tự tương đối của các tiểu phân ở ý a) cũng thay đổi. Hãy
tính nồng độ khoảng [CN-] sao cho trong khoảng đó thì trình tự tương đối đó thì trình tự tương đối
ở ý a) được giữ nguyên.
d) Tính thế cân bằng của cathode, giả sử rằng bề mặt cathode được bao phủ bởi Ag(s).
2.2. Thuốc APC là hỗn hợp của ba tác nhân dược
phẩm: aspirin (A), phenacetin (P) và caffeine (C).
Để phân tích hỗn hợp, tổ hợp các phương pháp
được sử dụng gồm: tách chiết, chuẩn độ và trắc
quang.
Viên thuốc 500 mg được nghiền thành bột, hoà tan vào 100 mL điclometan, rồi chuyển vào phễu
chiết. Aspirin (A) có thể được tách định lượng khỏi pha hữu cơ bởi 40 mL dung dịch xút 0,1 M.
[A]hc
Aspirin có hệ số phân bố K D = =105 và hằng số axit pKa = 3,5.
[A]n

a) Tại sao lại sử dụng dung dịch kiềm cho quá trình tách chiết?
b) Khi sử dụng môi trường trung tính (pH = 7) thì hiệu quả chiết là bao nhiêu?
c) Xác định giá trị cực tiểu pHmin để thu hồi được lớn hơn 99% aspirin trong quá trình chiết trên.
2 Nội dung Điểm
2.1. a) Xét 3 nồng độ cao nhất:

2 [CN − ]2 4*2/8
[Ag(CN)−2 ] = CAg =
2 [CN − ]2 + 3[CN − ]3 + 4 [CN − ]4 =8/8

1 1
= CAg = CAg = 0,07(mM)
  1 + 10 + 100,4
1,0
1 + 3 [CN − ] + 4 [CN − ]2
2 2

4
3[CN − ]3
[Ag(CN)32− ] = CAg
2 [CN − ]2 + 3[CN − ]3 + 4 [CN − ]4
3
[CN − ]
2 101,0
= CAg = CAg = 0,74(mM)
3 − 4 − 2 1 + 101,0
+ 10 0,4
1 + [CN ] + [CN ]
2 2

3− 4 [CN − ]4
[Ag(CN) ] = CAg
2 [CN − ]2 + 3[CN − ]3 + 4 [CN − ]4
4

4
[CN − ]2
2 100,4
= CAg = CAg = 0,19(mM)
3 − 4 − 2 1 + 101,0
+ 10 0,4
1 + [CN ] + [CN ]
2 2
[Ag+] và [Ag(OH)(CN)-] nhỏ hơn đáng kể so với nồng độ 3 tiểu phân trên và:

[Ag(OH)(CN) − ] [Ag(OH)(CN) − ]
K OHCN = 10 13,20
= =
[Ag + ][CN − ][OH − ] [Ag + ]
Do đó: [Ag(OH)(CN)-] [Ag+].
Vậy thứ tự nồng độ: Ag(CN)32− > Ag(CN)34− > Ag(CN)−2 > Ag(OH)(CN)- >

Ag+
Tiểu phân có nồng độ cao nhất: [Ag(CN)32− ] = 0,74(mM)

b) Ag(CN)32− + e- Ag (s) + 3CN- 1/8

RT
E o = 0,799 − ln 3 = −0, 467(V)
nF
c) Ở giới hạn trên của [CN-], [Ag(CN)2− ] = [Ag(CN)3− ]  3[CN-]3 = 4[CN-]4 2/8
3 4


CN −  = 3 = 100,60 = 4,0(M)
 4

Ở giới hạn dưới, [Ag(CN)−2 ] = [Ag(CN)34− ]  2[CN-]2 = 4[CN-]4 ( [Ag(CN)−2 ]

= [Ag(CN)32− ] thoả mãn ngay cả khi [CN-] thấp hơn.

2
CN −  = = 10−0,20 = 0,63(M)
4
Vậy khoảng [CN-] là 0,63 – 4,0 M.
d) Ag+ + e- Ag (s) 1/8

RT 1 RT 3[CN − ]3
E=E − o
ln =E −
o
ln
F [Ag + ] F [Ag(CN)32− ]

5
8,314  298, 2 1021,4 103
E = E = 0,799 − ln = −0,652(V)
96485 0,74 10−3
2.2. a) Các thành phần của thuốc đều tan tốt trong dung môi hữu cơ (CH2Cl2), với aspirin 2/8
có hệ số phân bộ KD = 105 là rất lớn. Khi sử dụng dung dịch kiềm thì aspirin có

phản ứng:
Sản phẩm phản ứng là muối tan tốt trong dung dịch nước.
b) Aspirin dạng axit ký hiệu HA, tỷ lệ chiết ký hiệu R (chiết khỏi pha hữu cơ), ta có: 4/8

n HA,n n HA,n [HA],n .Vn


R= = =
n n HA,n + n HA,hc [HA],n .Vn + [HA]hc .Vhc
([HA]n + [A − ]n )Vn
=
([HA]n + [A − ]n )Vn + [HA]hc .Vhc

1 [HA]hc .Vhc V [HA]hc


 = 1+ −
= 1 + hc 
R ([HA]n + [A ]n )Vn Vn [HA]n + [A − ]n

Vhc [HA]hc V 1 V h
= 1+  = 1 + hc  K D = 1 + hc  K D
Vn [HA] + [HA] K a Vn K
1+ a Vn h + Ka
n n
h h

1 100 10−7
Tại pH = 7, ta có: = 1+  10 −7
5
= 80,03
R 40 10 + 10−3,5
 R = 0,012495 (1,25%)
c) pH càng lớn thì hiệu quả chiết càng cao, khi R = 99%, ta có: 2/8

1 100 h
= 1+  105 −3,5
 h = 1, 278.10−11 (M)  pH = 10,89
0,99 40 h + 10
Vậy tại pH  10,89 thì hiệu quả chiết đạt lớn hơn 99%.

Câu 3 (2,5 điểm) Nhiệt động học và cân bằng hóa học.
3.1. Trộn cẩn thận 1 mol hydrogen và 0,5 mol oxygen ở áp suất 101,3 kPa và nhiệt độ 291 K trong một
cylinder (xi-lanh) làm bằng thép dày, có piston.
a) Xác định biến thiên enthalpy, entropy, năng lượng tự do Gibbs đi kèm với quá trình trộn khí trên.
b) Những giá trị nào (ΔH, ΔS, ΔG) sẽ thay đổi nếu quá trình được tiến hành ở nhiệt độ khác, ví dụ như
273?
Hỗn hợp tạo thành trong cylinder được nén nhanh (nhiệt trao đổi với môi trường không đáng kể) tới thể
tích 3,0 lít, sau đó được kích nổ.
c) Tính nhiệt độ của hỗn hợp ở thời điểm bắt đầu nổ (giả định rằng quá trình nén thuận nghịch).
d) Tại sao phản ứng tạo thành nước lại đi kèm với một vụ nổ?
6
3.2. Li2S6 là dạng polysulfua được nghiên cứu nhiều
nhất, phản ứng hình thành:
2Li+ + S8 + 2e → Li2S6 + 2S
Li2S6 tồn tại hai dạng cấu trúc: (I) và (II), sự phân
ly của Li2S6 trong dung môi điện phân DME (1,2-
dimethoxyethane) được mô tả ở sơ đồ bên dưới (gồm
cân bằng của các dạng Li 2S6 , LiS6− , S62− và LiS*3 trong
DME):

Năng lượng Gibbs phân ly (kJ.mol-1) của các quá trình


trong DME (25oC và 1 bar):

Gdo1 ( I ) Gdo1 ( II ) Gdo2 Gdro ( I ) Gdro ( II )


20,68 18,92 100,55 45,13 43,37
a) Tính tỷ lệ nồng độ Li2S6 tương ứng với hai dạng cấu trúc (I) và (II).
b) Tính hằng số phân ly biểu kiến của cân bằng: Li 2S6 Li + + LiS6−
− 2− *
c) Sắp xếp nồng độ cân bằng của các cấu tử Li 2S6 , LiS6 , S6 và LiS3 theo thứ tự giảm dần.

3 Nội dung Điểm


3.1 a. Hmix = 0 do các khí có thể xem là khí lí tưởng và không phản ứng với nhau. 4/8
Smix = −n  R(1 ln 1 + 2 ln 2 ) , trong đó n  là tổng số mol hỗn hợp khí lý
tưởng; i là phần mol của mỗi cấu tử. Vậy:

 2 2 1 1
Smix = −1,5  8,314   l n + ln  = 7,938 (J.K-1)
 3 3 3 3
G mix = −T.Smix = −291 7,938 = - 2309,95 (J)
b. Với khí lý tưởng Hmix và Smix không phụ thuộc vào nhiệt độ. 2/8
Gmix = -TSmix = -273  7,938 = -2167,07 (J)
Hỗn hợp tạo thành trong cylinder được nén nhanh (nhiệt trao đổi với môi
trường không đáng kể) tới thể tích 3,0 lít, sau đó được kích nổ.
c. Đây là quá trình đoạn nhiệt (không trao đổi nhiệt) thuận nghịch. 2/8
Cp
T1V1−1 = T2V2−1 , trong đó  =
Cv
Với các khí lưỡng nguyên tử: Cp = 7/2R, Cv = 5/2R,  = 1,4.
nRT1 1,5  8,314  291
V1 = = = 35,82(L)
p 101,3

7
T1V1−1 291 35,820,4
T2 = = = 784,7(K)
V2−1 3,00,4
d. Phản ứng diễn ra theo cơ chế dây chuyền phân nhánh, mỗi phản ứng tạo ra phân 2/8
tử H2O đều giải phóng năng lượng rất lớn khiến cho tốc độ phản ứng phân
nhánh khác diễn ra nhanh hơn. Tốc độ giải phóng năng lượng tăng theo cấp số
nhân và gây ra nổ.
3.2. a. Xét quá trình: (I) (II),  r G o = G d1
o
(I) − G d1
o
(II) = 1, 76 kJ.mol −1 2/8

[Li 2S6(II ) ] − 8,314


1,76.10 3

= K = e −r G =e = 0, 4915.
o
/RT 298
a) Ta có:
[Li 2S6(I ) ]

b. Xét các cân bằng: 4/8


3
[Li + ][LiS6− ] − 8,314
20,68.10
+ −
(I ) Li + LiS 6 K1 = [( I )] =e 298
= 2,371.10−4
3
[Li + ][LiS6− ] − 8,314
18,92.10

(II ) Li + + LiS6− K2 = [( II )] =e 298


= 4,825.10−4
+[( II )] [Li + ][LiS− ]
1
K1 + K12 = [Li+[(][LiS
I )]
− +
]
[( II )]
[Li + ][LiS− ]
= [([LiI )]+ ][LiS −  K = [( I )]+[( II )] =
]
6 K1K 2
K1 + K 2
6 6 6

[Li+ ][LiS6− ] [Li+ ][LiS− ]


Vậy, cân bằng: Li 2S6 Li + + LiS6− , K= [Li2S6 ] = [( I )]+[( II6)]

2,371.10−4 4,825.10−4
K= 2,371.10−4 + 4,825.10−4
= 1,59.10−4

c. Xét các cân bằng: 4/8


3
− 8,314
45,13.10

(I ) 2LiS3 K1 = e 298


= 1, 228.10−8
3
− 8,314
43,37.10

(II) 2LiS3 K2 = e 298


= 2, 498.10−8
−8 −8
 Li 2S6 2LiS3 , có K = 1,228.10
1,228.10 2,498.10
−8
+ 2,498.10−8
= 8, 233.10−9
3
− 100,55.10
Xét cân bằng: LiS6− Li + + S62− , có K  = e 8,314298
= 2,369.10−18

Viết lại: Li 2S6 Li + + LiS6− , K = 1,59.10−4

Vì K K K  [Li 2S6 ]  [LiS6− ]  [LiS3 ]  [S62− ].

Câu 4 (2,5 điểm) Hóa nguyên tố (Kim loại, phi kim nhóm IVA, VA). Phức chất.
4.1. Muối X1 chưa biết của kim loại X tan được trong nước, và khi điện phân dung dịch muối này có
thể thu được chất tẩy trắng chloride vôi (phản ứng 1). Muối Y1 chưa biết của kim loại Y tan được trong
nước, cho màu vàng trong thí nghiệm ngọn lửa, và dung dịch của muối này làm methyl da cam chuyển
màu vàng. Nhỏ giọt dung dịch nitric acid vào dung dịch Y1 cho đến khi methyl da cam chuyển thành
màu đỏ (phản ứng 2). Sau đó, thêm dung dịch silver nitrate vào dung dịch này, quan sát thấy có kết tủa
vàng (phản ứng 3). Nhỏ giọt 12,1 mL dung dịch X1 nồng độ 40,0 % (khối lượng riêng 1,396 g.cm-3) vào

8
dung dịch muối Y1 rồi khuấy mạnh cho đến khi tạo thành kết tủa trắng Z (phản ứng 4), hàm lượng X
trong đó là 37,98%. Lọc tách kết tủa Z rồi thêm lượng dư nitric acid và silver nitrate vào dịch lọc, khối
lượng kết tủa màu trắng tạo thành là 17,9 g. Nếu kết tủa trắng Z không được lọc tách nhanh, nó sẽ từ từ
chuyển thành các tinh thể M (phản ứng 5) – đóng vai trò quan trọng trong cơ thể người. Khi nung Z,
khối lượng giảm đi 1,90% (phản ứng 6).
a) Xác định các kim loại X, Y và các chất chưa biết X1, Y1, Z, M, biết rằng Z chứa ba anion, và M chỉ
chứa hai anion.
b) Viết phương trình các phản ứng 1 – 6.
4.2. Khi xử lí Co3O4 với hydrochloric acid đặc dư thì xuất hiện khí màu vàng lục và dung dịch chuyển
sang màu lam nhạt do sự tạo phức phối trí của ion B. Khi pha loãng dung dịch bằng nước, màu của dung
dịch chứa B thay đổi và phức phối trí của ion C được tạo thành. Quá trình này gắn liền với sự thay thế
ligand và sự thay đổi về phối trí của ion kim loại chưa biết. Phổ đồ quang phổ hấp thụ chất C cho thấy
vùng hấp thụ cực đại nằm vùng xanh lục (~510 nm).
Khi thêm vào dung dịch chứa C lượng dư ammonia ta thu được phức cation D có phân tử lượng bé hơn
6 g/mol so với phân tử lượng của phức C. Phức D được tạo thành với hiệu suất cao nhất khi có mặt muối
ammonium do khả năng xảy ra phản ứng phụ.
Khi đưa không khí đi qua dung dịch D chloride và ammonium chloride, dung dịch xuất hiện màu hồng
đỏ do sự hình thành của hợp chất phối trí E. Nếu phản ứng này được thực hiện khi có mặt carbon hoạt
tính thì sẽ tạo thành hợp chất phối trí F có màu vàng trong dung dịch. Phân tử lượng của E và F chênh
lệch 17 g/mol, D chloride và F chênh lệch 35,5 g/mol.
a) Xác định hợp chất B – F và viết các phương trình hoá học minh hoạ các phản ứng của các chuyển
hóa trên.
Biết rằng phức phối trí F có tính nghịch từ, và D chloride mang tính thuận từ với moment từ hiệu dụng
trên ion kim loại chuyển tiếp là eff = 3,87mB . Moment từ được tính bằng công thức eff = 2 S(S + 1)
, với S là spin tổng của ion.
b) Vẽ giản đồ năng lượng tách trường phối tử các orbital d của kim loại chuyển tiếp chưa biết trong hai
phức chất D và F.
Khi thêm từ từ potassium oxalate dư và hydrogen peroxide vào dung dịch chứa anion phức phối trí C,
màu sắc dung dịch chuyển sang xanh lục đậm. Khi làm lạnh dung dịch, ta thu được tinh thể xanh lục
đậm của hợp chất G chứa 7,56% khối lượng nước kết tinh.
c) Cho biết thành phần của G, vẽ tất cả đồng phân lập thể của ion phức phối trí thành phần của G.
4 Nội dung Điểm
4.1. Chloride vôi là CaOCl2, vậy X là Ca và X1 là CaCl2. Muối của kim loại Y tạo 6*1/8
ngọn lửa màu vàng, vậy Y là sodium. Các muối bạc màu vàng là phosphate và = 6/8
iodide, tuy nhiên, khi trộn lẫn Y1 với muối calcium thì tạo thành kết tủa, vậy
Y1 là sodium phosphate Na3PO4. Kết luận này cũng được chứng minh bởi sự
xuất hiện môi trường kiềm (dựa vào màu chỉ thị). Số mol CaCl2 được thêm vào

9
12,1 1,396  0, 4
để tạo thành Z: n CaCl2 = = 0,0624(mol)
40 + 35,5  2
 n Cl− = 0,1248(mol)

n AgCl = 0,1247(mol)
Vậy, gần như toàn bộ ion chloride đều ở trong dung dịch, có nghĩa rằng không
có ion chloride trong kết tủa.
40
Khối lượng đương lượng mol Z theo mỗi Ca: = 105,32(g / mol)
0,3798
Sodium phosphate bị thủy phân trong dung dịch nước:
PO34− + H2O HPO24− + OH−
 công thức Z gồm calcium và các anion phosphate, hydrophosphate và
hydroxide: Cak(PO4)l(HPO4)m(OH)n rồi thiết lập được hệ phương trình:

40k + 95l + 96m + 17n = 105,32k l = 10, 295m − 5, 255n


 
2k = 3l + 2m + n k = 16, 4425m − 7,3825n
Ta lập bảng biện luận như sau:
k l m n
9,0 5,0 1,0 1,0
15,34 2,0 1,0
18,12 10,1 2,0 2,0
41,8 25,5 3,0 1,0
10,7 4,8 2,0 3,0
Chọn nghiệm phù hợp: k = 9, l = 5, m = 1, n = 1.
CTHH Z: Ca9(PO4)5(HPO4)(OH)
b. Viết phương trình hoá học cho các phản ứng: 3*1/8
đ𝑖ệ𝑛 𝑝ℎâ𝑛 𝑑𝑑 𝑘ℎô𝑛𝑔 𝑚à𝑛𝑔 𝑛𝑔ă𝑛 =3/8
(1) CaCl2 + H2O → CaOCl2 + H2
(2) Na3PO4 + 2HNO3 → NaH2PO4 + 2NaNO3
Hoặc Na3PO4 + 3HNO3 → H3PO4 + 3NaNO3
(3) 3NaH2PO4 + 3AgNO3 → Ag3PO4 + 3NaNO3 + 2H3PO4
(4) 9CaCl2 + 6Na3PO4 + H2O → Ca9(PO4)5(HPO4)(OH) + 18NaCl
(5) 5Ca9(PO4)5(HPO4)(OH) + 6Na3PO4 + 4H2O
→ 9Ca5(PO4)3OH + 9Na2HPO4
(6) Ca9(PO4)5(HPO4)(OH) → 3Ca3(PO4)2 + H2O
4.2. a) Co3O4 + 14HCl → 3H2[CoCl4] + Cl2 + 4H2O

10
B: [CoCl4]2- 5*1/8
H2[CoCl4] + 6H2O → [Co(H2O)6]Cl2 + 2HCl = 5/8
Theo phổ đồ, vùng hấp thụ cực đại nằm ở vùng xanh lục (~510 nm), do đó màu
của dung dịch phức phối trí C quan sát được là đỏ.
C: [Co(H2O)6]2+
[Co(H2O)6]Cl2 + 6NH3.H2O → [Co(NH3)6]Cl2 + 12H2O
D: [Co(NH3)6]2+
4[Co(NH3)6]Cl2 + O2 + 4NH4Cl → 4[Co(NH3)5Cl]Cl2 + 8NH3 + 2H2O
E: [Co(NH3)5Cl]Cl2
Hoặc
4[Co(NH3)6]Cl2 + O2 + 4NH4Cl + 6H2O → 4[Co(NH3)5Cl]Cl2 + 8NH3.H2O
𝐶𝑎𝑟𝑏𝑜𝑛 ℎ𝑜ạ𝑡 𝑡í𝑛ℎ
4[Co(NH3)6]Cl2 + O2 + 4NH4Cl → 4[Co(NH3)6]Cl3 + 4NH3 +
2H2O
F: [Co(NH3)6]Cl3
𝐶𝑎𝑟𝑏𝑜𝑛 ℎ𝑜ạ𝑡 𝑡í𝑛ℎ
Hoặc 4[Co(NH3)6]Cl2 + O2 + 4NH4Cl + 2H2O → 4[Co(NH3)6]Cl3
+ 4NH3. H2O
b) Trong phức chất D, cobalt có trạng thái oxid hóa +2 và có 7 điện 1/8
tử ở các vân đạo d. Với giá trị eff = 3,87mB , S = 3/2. Do đó, 3 trên 7 điện tử

d chưa ghép cặp  có tính thuận từ.


Trong phức chất F, cobalt có trạng thái oxid hóa +3, do đó 6 điện tử
d đều ghép cặp.

2/8

c) Tìm lượng nước trong tinh thể: 3/8


Đặt công thức là K3[Co(C2O4)3].nH2O, ta có:
18n
%m H2O = = 0,0756  n = 2 .
440 + 18n

11
Vậy G là K3[Co(C2O4)3].2H2O
Ion phức [Co(C2O4)3]3- có hai đồng phân quang học – là đối quang của nhau:

Câu 5 (2,5 điểm) Đại cương hữu cơ.


5.1.
a) Xét các liên kết (bond) được kí hiệu A–D trong lactone (hình bên).
Hãy cho biết liên kết nào dài nhất, liên kết ngắn nhất. Giải thích ngắn
gọn.
b) Vẽ công thức cấu tạo của N,N-dimethylformamide và
N-methylacetamide. So sánh (có giải thích) nhiệt độ sôi của hai hợp
chất này.
5.2. So sánh pKa của các cặp chất sau. Giải thích.

i) ii)

5.3.
a) Hãy vẽ các cấu dạng ghế của 2-chlorotetrahydropyran và cho biết cấu dạng nào bền
nhất. Giải thích.
b) Theo thuyết FMO, hãy giải thích sự khác biệt tín hiệu trong phổ IR ứng với nhóm
carbonyl các hợp chất sau.

5 Nội dung Điểm


5.1 a) Liên kết dài nhất: C; Liên kết ngắn nhất: B 4/8

Liên kết C dài nhất vì là liên kết σ C sp3 – Csp3


Liên kết A ngắn hơn C vì σ C sp2 – Csp2 và có phần C=C do cộng hưởng.
Liên kết B ngắn nhất dó C=O và nguyên tử O có bán kính nhỏ hơn C
12
b)

2/8

Hai chất này là đồng phân của nhau, nhưng chỉ có N-methylacetamide có thể tạo
được liên kết Hydrogen nên có nhiệt độ sôi cao hơn.

2/8

5.2 Chất có tính acid mạnh hơn (pKa bé hơn)

i) pKa 2 < 1
Hiệu ứng cảm ứng của vòng thơm làm base liên hợp của 4 yếu hơn 3.
2/8
(Chú ý: Giải thích do cộng hưởng với vòng thơm là sai, vì cặp e của N không
vuông góc với mặt phẳng pi)

ii) pKa 4 < 3


O không làm bền hóa điện tích dương hiệu quả bằng N (do độ âm điện của O lơn 2/8
hơn)
5.3 a) Vẽ được 2 cấu dạng

2/8

Chỉ ra cấu dạng bền hơn và giải thích


Cấu dạng B (Cl ở vị trí trục) bền hơn vì có tương tác cặp electron của O với phản
liên kết σ*C-Cl (hiệu ứng anomeric)

2/8

b) Tín hiệu trên phổ IR của nhóm carbonyl có số sóng càng lớn là liên kết càng bền.
Vì F, Br có độ âm điện lớn (χF > χBr> χC), hút mạnh electron, làm giảm LUMO là 4/8
phản liên kết σ* C─C, tăng hiệu quả tương tác của nO → σ* C─C là bền hóa liên kết
C=O (tăng bậc liên kết)

13
Câu 6 (2,5 điểm) Sơ đồ tổng hợp hữu cơ. Cơ chế phản ứng hóa hữu cơ.
6.1. Đề xuất cơ chế cho các chuyển hóa sau
a)

b)

6.2. Năm 2021, Watanabe và các cộng sự đã thực hiện tổng hợp toàn phần Cannogenol (Glycosid
Tim). Không quan tâm lập thể, hãy xác định các chất A1 – A5.

Chú ý: BHT : butylated hydroxytoluene; reflux: đun hồi lưu.


14
6.3. Không quan tâm lập thể, hãy xác định các chất B1 – B8:

6 Nội dung Điểm

6.1. a) 3/8

b) 3/8

6.2
5*1/8
=5/8

15
6.3. B1 B2 B3 12/8

B4 B5 B6

B7 B8

Câu 7 (2,5 điểm) Xác định cấu trúc các chất hữu cơ (mô tả sơ đồ tổng hợp bằng lời dẫn)
7.1. Resiniferatoxin có nguồn gốc thiên nhiên, tiền chất của chất này được tổng hợp như sau:
Chất B1 (methyl (Z)-3-iodoprop-2-enoate) tác dụng với DIBAL trong DCM ở -78oC cho B2. Trộn
LHMDS với THF rồi làm lạnh đến 0oC. Sau đó cho 3-methyl cyclopentenone khuấy đều, làm lạnh đến
-78oC rồi trộn với B2. Tiếp tục cho TMSCl vào hỗn hợp thu được B3. Cho allylmagnesium bromide tác
dụng với B3 ở -78oC, sau đó cho từ từ HCl và Et3N được B4. Đưa hỗn hợp thu được lên 0oC, thêm
Pd(PPh3)4 khuấy đều rồi đun ở 75oC trong 3 tiếng thu được B5. Cho B5 tác dụng với TMSCl và imidazole
ở nhiệt độ phòng thu được B6. Tiếp tục cho silica gel (SiO2) vào thu được B7. Oxi hóa B7 bằng Dess-
Martin periodinane, NaHCO3 trong DCM thu được tiền chất Resiniferatoxin. Vẽ công thức cấu tạo của
các chất từ B2 đến B7.

7.2. Alstonlarsine A là hợp chất được tìm thấy trong rễ của cây hoa sữa. Quy trình tổng hợp tiền chất
của Alstonlarsine A được mô tả như sau [Yao, 2022]:
Dẫn xuất của indole X1 tác dụng với imidazole/TBSCl trong DCM ở 0oC được X2. Trộn X2 với Et3N
và tert-butyl hypochlorite trong THF ở -78oC; sau đó thêm Y (lithium enolate của dimethyl malonate)
được X3. Khi cho X3 tác dụng với halide Z, hệ xúc tác PdLn, KOAc trong DCM thu được X4. Hòa tan
16
X4 trong DMF rồi thêm LiCl và nước rồi đun ở 135oC cho X5. X5 tác dụng với Boc2O, Et3N và DMAP
thu được X6. Trộn X6 với TBAF trong THF ở 23oC thu được X7 (C24H31NO5). Xác định cấu tạo của
các chất từ X2 – X7.

7 Nội dung Điểm


7.1 2/8*6
=12/8

7.2
8/8

17
Câu 8 (2,5 điểm) Hóa học các hợp chất thiên nhiên (Cacbohidrat và các hợp chất hữu cơ chứa nito
đơn giản)
8.1. Furcatin (C19H26O10) là một saccharide không có tính khử có trong cây hoa kim ngân. Furcatin bị
thủy phân hoàn toàn bởi enzyme β-glicosidase thu được D-glucose; D-andopentose X và p-vinylphenol.
Khi methyl hóa hoàn toàn furcatin với methyl iodide có mặt Ag2O, rồi thủy phân với dung dịch HCl cho
thu được 2,3,4-tri-O-methyl-D-glucose, tri-O-methyl-D-andopentose và p-vinylphenol.
Furcatin tác dụng được với 4 đương lượng HIO4 thu được 1 đương lượng formaldehyde và 1 đương
lượng acid formic.
Furcatin cũng tác dụng được với 1 đương lượng acetone/ H+.
a) Lập luận xác định cấu trúc của furcatin và biểu diễn cấu dạng của nó.
b) Trong dung dịch có cân bằng giữa dạng mạch hở của X với 4 dạng vòng furanose. Giải thích và biểu
diễn cấu trúc 5 dạng của X trong dung dịch; gọi tên các dạng vòng của X.
8.2. Một loại thuốc chữa HIV được tổng hợp từ dẫn xuất indole E1 theo sơ đồ dưới đây. Xác định cấu
tạo của các chất từ E2 đến E5. Biết rằng E5 có công thức phân tử là C21H20ClNO2

8 Nội dung Điểm


8.1 a)
 − gli cos idase
- Furcatin (C19H26O10) ⎯⎯⎯⎯⎯ → D-glucose (C6H12O6) + D-andopentose (C5H10O5)
+ p-C2H3C6H4OH
→ tỉ lệ sản phẩm thủy phân là 1:1:1.

1/8

- Furcatin là đường không khử → (OH) anome của đơn vị D-glucose và X đều bị khóa
vào liên kết glicoside → dễ thấy trật tự liên kết trong furcatin:

2/8

- Furcatin ⎯⎯⎯
4HIO4
→ 1 HCHO + 1 HCOOH

18
Dễ thấy 2 HIO4 sử dụng để cắt liên kết C2-C3 và C3-C4 của glucose tạo 1 HCOOH → 2
HIO4 để phân cắt 2 liên kết C-C trong đơn vị X, tạo 1 HCHO → X có nhánh (CH2OH)
dạng 2/8

- Furcatin cũng tác dụng được với 1 đương lượng (CH3)2CO / H+ → dễ thấy đơn vị glucose
không có các nhóm (OH) cùng phía nên không tác dụng với (CH3)2CO → 2 nhóm OH ở
C2 và C3 trong đơn vị X cùng phía. X là đường dãy D- → cấu trúc của X phải là

1/8

 cấu dạng của furcatin:

2/8

b) Do C4 của X không bất đối → có 2 cách đóng vòng khác nhau; mỗi cách tạo ra 2 dạng , 1/8*4
: = 4/8

8.2
2/8*4
= 8/8

Giáo viên ra đề:

Phạm Thị Phương Dung - 0938473263


Đặng Công Nghĩa - 0905172198

19
HỘI CÁC TRƯỜNG CHUYÊN ĐỀ THI MÔN HÓA HỌC KHỐI 11
VÙNG DUYÊN HẢI VÀ ĐỒNG BẰNG BẮC BỘ NĂM 2023
TRƯỜNG THPT CHUYÊN LÊ HỒNG PHONG Thời gian làm bài 180 phút
TỈNH NAM ĐỊNH
(Đáp án có 08 trang, gồm 8 câu)
ĐÁP ÁN ĐỀ THI ĐỀ XUẤT

Câu Nội dung Điểm


Câu I 2,5
I.1
0,5
Propene
I.2

Phương pháp gần đúng trạng thái dừng áp dụng cho C* là sản phẩm trung gian kém bền, tồn 0,5
tại trong thời gian ngắn.


I.3 d[P]
a) Ở áp suất thấp:  k a [C][M]  Bậc phản ứng là bậc 2
dt
Bước quyết định tốc độ phản ứng là giai đoạn hoạt hóa cyclopropane (nồng độ khí va chạm 0,25
nhỏ nên sự ca chạm rất hiếm xảy ra và kích thích phân tử. Tuy nhiên, phân tử khi bị kích
hoạt thì sống đủ lâu để tạo thành sản phẩm)
d[P] k a k r
b) Ở áp suất cao:  [C]  Bậc phản ứng là bậc 1
dt kd
Bước quyết định tốc độ phản ứng là giai đoạn từ C*  P. Nồng độ khí va chạm cao dẫn tới 0,25
sự hình hành một cân bằng nhanh giữa C và C*, do đó tốc độ phản ứng chỉ phụ thuộc vào
tốc độ đồng phân hóa.

I.4 - Hơi cyclopropane-oxygen tạo thành một hỗn hợp dễ nổ có thể gây nguy hiểm nghiêm
trọng.
- Cyclopropane gây độc cho một số cơ quan nội tạng (gan, tim). 0,25
- Thuốc mê có khí không phải lúc nào cũng tốt và dễ dùng.

I.5 Giả sử phản ứng bậc 1.


t (phút) 0 10 20 40 80 120 ∞
I (P) (a.u) 0,37 23,24 38,21 62,98 87,26 95,88 100,0
100-I 99,63 76,76 61,79 37,02 12,74 4,12 0 0,5
ln(100-I) 4,601 4,341 4,124 3,611 2,545 1,416
Phương trình động học tích phân có dạng ln(100-I) = -kt + C
Dùng phương pháp hồi quy tuyến tính xác định được k = 0,0265 phút-1, với R2 = 0,999
I.6 Khả năng phản ứng được tính theo công thức:
0,5
p 86000
Tại áp suất 860 mbar: [M]    0, 0135M
RT 8,314  764
1 1
 w  2
 47%
kd
1  [M] 1  3,54  10
 0, 0135
kr 4, 20 104
p 2000
Tại áp suất 20 mbar: [M]    3,15 104 M
RT 8,314  764
1 1
w  2
 97%
1  d [M] 1  3,54 10 4  3,15 104
k
kr 4, 20 10
Như vậy ở áp suất thấp, phân tử bị hoạt hóa phản ứng thành công, nghĩa là khả năng phản
ứng cao hơn. Điều này phù hợp với cơ chế đưa ra tại áp suất khí va chạm thấp ở phần 3
 EA
I.7 k 1 1
k  A  e RT  EA = R  ln 1 / (  )
k 2 T2 T1 0,25
Thay k1 = 4,42.10 s , T1 = 764K; k2 = 1,30.10-3 s-1, T1 = 784K vào biểu thức trên
-4 -1

 EA = 269 kJ/mol.
Câu II 2,5
II.1 Cathode: Li+ + 6 C + e  LiC6
0,5
Anode: LiCoO2  Li+ + CoO2 + e
II.2 q 2600 103  3600
m(than chì) =  6 12   6 12  6,98 gam 0,25
F 96485
G 1920
II.3 
KIP = e RT
 e 8,314293  0, 455

LiPF6 (cặp ion) ⇌ Li+(solv) + PF6 (solv)

8-x x x
0,5
2
x 1
   x = 3,326 M
8  x 0, 455

 Eanode = Eo + 0,0592lg[Li+] = -3.04 + 0,0592 lg 3,326 = -3,01 V

II.4 E  E cathode  E anode = Eocathode  0,0592lg[Li  ]  (Eoanode  0,0592lg[Li  ])


0,25
E  Eocathode  Eanode
o
= 4,15 V

II.5 Li+(solv) + B- (solv) ⇌ LiB (cặp ion) KIP, B = 2

Li+(solv) + PF6 (solv) LiPF6 (cặp ion) KIP = 0,455

[LiB]=K IP , B  [Li  ]  [B ]


Ta có  
 [LiB]  [LiPF6 ]  [Li ](KIP , B [B ]  K IP [PF6 ]) 0,5
[LiPF6 ]=K IP  [Li ]  [PF6 ]

[LiB]  [LiPF6 ] 0,94


 [Li  ]   
 = 0,56 M
K IP , B  [B ]  K IP  [PF6 ] 2  0, 65  0, 455  0,8
II.6

0,5

Đại lượng thay đổi có thể đo được là từ tính


Co(III) nghịch từ S = 0
Co(IV) thuận từ S = ½

Câu
2,5
III
III.1 T = 0, U = 0, H = 0.
V2 20
w  nRT ln = 8,314  298  ln = -1717,3 J
V1 10
q = - w = 1717,3 J. 0,5
q 1717,3
S =   5,76 J/K;  G =  H - T  S = -1717,3 J
T 298
 S(tổng) =  S (hệ) +  S (môi trường) = 0
III.2 r Ho  5f Ho (N2 )  2f Ho (CO2 )  f Ho (H2O,g)  5f Ho (N 2O)  f H o (C2H 2 )
= 2x (-393,5) + (-241,8) – 5x81,50 – 226,7 = -1663 kJ/mol.
 r Ho 1663 103 0,25
T    T   = 6561K
Cp (H 2O, g)  2Cp (CO2 )  5Cp (N 2 ) 33,58  2  37,11  5  29,13
Nhiệt độ ngọn lửa là 6561 + 298 = 6859K
III.3 Cp  Cp (H2O,g)  2Cp (CO2 )  5Cp (N2 )  5Cp (N2O)  Cp (C2H2 )
= 21,72 J/K.mol
rS  So (H2O,g)  2So (CO2 )  5So (N2 )  5So (N2O)  So (C2H 2 )
o

= 272,3 J/K.mol
Tại 3500K
0,5
 r G   r H  3500   rS
3500
 r G  r Ho  Cp (3500  298)  3500(rSo  Cp ln )
298
3500
= -1663 + 0,02172 (3500-298) – 3500 (0,2723 + 0,02172 ln ) = -2733,8 kJ/mol
298
III.4 G 300700

K  e RT  e 8,314900  2,837x1017
pO2  p 2N2
16  p3O2
K 2
  2,837x1017  p O2 = 121 bar 0,25
p N2O 1
( 5 )2
10
Áp suất không khí = 5 p O2 = 605 bar.
III.5 R6 Ho  58,6 kJ/mol 0,25
III.6 Tại TTCB, thể tích hỗn hợp là 92/255,6 = 0,36 m3
PV 104  0,36
 Tổng số mol hỗn hợp tại cân bằng n    1, 453 mol
RT 8,314  298
N 2O4 2NO2
1-x 2x  1 + x = 1,453  x = 0,453 mol 0,25
PT n n 2NO2 0,1 0,9062 0,1
K p  Kn  ( )  K  ( ) ( ) = 0,1033
nT n N2O4 1, 453 0,547 1, 453
R6Go  RT ln K  8,314  298  ln 0,1033 = 5624 J/mol

III.7 K 298  R 6 H o 1 1
ln  (  )  K370 = 10,30
K 370 R 370 298
N 2O4 2NO2
1-x x
2
0,5
n NO2 PT 4x 2 0,1
K ( )  ( ) =10,30  x = 0,98.
n N2O4 n T 1 x 1 x
Vậy phần trăm phân hủy N2O4 là 98%

Câu
2,5
IV
IV.1 X A1 A2 A3 A4 A5 B1 B2 G
P H3PO2 H4P2O6 H4P2O6 H4P2O5 H4P2O7 NaH2PO2 Na2H2P2 PH3
O6 hoặc 0,5
Na4P2O6
b.

A1: A2: A3:


0,25

c.
B6 – Na5P3O8, B7 – Na5P3O9, B8 – Na5P3O10 0,25

d.

0,25

IV.2 a. [Cr(CO)6], [Fe(CO)5], [Ni(CO)4]

0,25

b. 34 electron 0,25
c.

0,25

1 và 2 là hai đồng phân đối quang.


d.
D - [Fe(CO)4H2], E - [Fe3(CO)12]

0,5

1. Fe(CO)5 + Ba(OH)2  [Fe(CO)4H2] + BaCO3


2. BaCO3 + 2 HCl  BaCl2 + CO2 + H2O
3. 3 [Fe(CO)4H2] + 3MnO2  [Fe3(CO)12] + 3 Mn(OH)2
Câu V 2,5
V.1 Tính acid của BF3 mạnh hơn so với B(OCH3)3.

Nguyên tử oxygen có độ âm điện nhỏ hơn so với fluoro nên cặp electron không liên kết
của oxygen dễ cho vào orbital trống của boron hơn. Do vậy, xác suất nguyên tử boron có
orbital trống trong BF3 nhiều hơn so với B(OCH3)3 (hay liên kết π B=F kém bền hơn liên
kết π B=O) nên BF3 dễ nhận cặp electron của nguyên tử khác hơn dẫn đến tính acid của
BF3 mạnh hơn.
0,5

V.2 Tính khử của hydride trong NaBH4 không đủ mạnh để khử imine.

Khi thêm BF3 vào hỗn hợp, BF3 phản ứng với imine kiểu acid base:

Tính nucleophile của nguyên tử carbon tăng lên nên nguyên tử hydride trong NaBH4 có thể 0,5
khử được.

V.3 Moment lưỡng cực của A lớn do tồn tại ở dạng ion lưỡng cực, khi đó mỗi vòng là một hệ
thơm bền vững hơn. 0,5
V.4 Nhóm C=O ở giữa có tính nucleophile mạnh (mạnh nhất trong 3 nhóm C=O) nên dễ phản
ứng với nước tạo thành nhóm gem-diol.

Sản phẩm monohydrate sinh ra có 2 nhóm -OH tạo liên kết hydrogen nội phân tử bền
vững.
0,5

V.5 Carbanion sinh ra từ B2 bền hơn so với carbanion sinh ra từ B1 do cặp electron không liên
kết của nguyên tử carbon trên B2 có thể cho vào orbital 3d trống của nguyên tử lưu huỳnh
giúp giải tỏa điện tích âm. Nguyên tử oxygen không có orbital 3d trống nên không có sự giải
tỏa điện tích như vậy.
0,5

Câu
2,5
VI
VI.1 Sơ đồ phản ứng:

1,0

VI.2 Cơ chế phản ứng:


1,5
Câu
2,5
VII
VII.1 Tổng hợp matrine

1,5

VII.2 Do các liên kết CO và CC trong anion tạo từ B đều bằng nhau. Do vậy anion có cấu trúc
như sau: 1,0
Công thức của B:

Tính acid của B mạnh hơn các acid hữu cơ thông thường do điện tích âm được giải tỏa
mạnh.

Câu
2,5
VIII
A có công thức phân tử C12H20O10, thủy phân A bằng dung dịch acid, đun nóng, thu được
duy nhất một monosaccharide là D-fructose suy ra A là disaccharide tạo thành từ D- 0,5
fructose.

A có thể bị thủy phân bởi enzyme α-glycosidase hoặc enzyme β-glycosidase suy A có cả 0,5
liên kết α-glycosidase và liên kết β-glycosidase.

Oxi hóa A bằng HIO4 dư thu được hợp chất A1. Thủy phân A1 trong dung dịch acid loãng
thu được hỗn hợp sản phẩm A2 trong đó có HOCH2-CHO và HOCH2-CO-CHO suy ra
fructose có cấu trúc vòng 6 cạnh. 1,0

Vậy công thức của A là

0,5

Lưu ý: Thí sinh làm cách khác nhưng đúng, vẫn cho điểm tối đa.
SỞ GD&ĐT TỈNH HOÀ BÌNH KỲ THI HỌC SINH GIỎI
TRƯỜNG THPT CHUYÊN CÁC TRƯỜNG KHU VỰC DUYÊN HẢI
HOÀNG VĂN THỤ VÀ ĐỒNG BẰNG BẮC BỘ
NĂM HỌC 2022 - 2023
ĐÁP ÁN ĐỀ THI MÔN HÓA HỌC LỚP 11
Thời gian làm bài 180 phút
(Đáp án gồm có 15 trang)

Câu 1(2,5 điểm). Tốc độ phản ứng.


A2(k) + 2 B(k) → 2 AB(k)
được xúc tác bởi chất C. Hằng số tốc độ tổng cộng tăng tuyến tính (bậc nhất) theo nồng độ
xúc tác. Các phép đo sau được thực hiện ở 400 K với [C] = 0,050 mol·L-1:
Thí nghiệm [A2] (mol·L- [B] (mol·L- Tốc độ đầu (mol·L-
1 1 1 -1
) ) ·s )
1 0,010 0,10 1,600×10-10
2 0,010 0,20 3,200×10-10
3 0,100 0,20 1,012×10-9
(a) Hãy xác định phương trình tốc độ của phản ứng.
(b) Hãy tính hằng số tốc độ tổng cộng ktc của phản ứng ở 400 K.
(c) Cơ chế phản ứng được đề xuất như sau:
k1
A2 (k) 2 A(g) cân bằng nhanh
k-1

k2
A(k) + B(k) + C(k) ABC(k) chậm
3 k
ABC(k) AB(k) + C(k)
Hãy chứng minh cơ chế đề xuất phù hợp với phương trình phản ứng tổng cộng.
(d) Chứng minh cơ chế đề xuất phù hợp với phương trình động học xác định được bằng thực
nghiệm.
(e) Hãy tính năng lượng phân ly liên kết của A2 từ các dữ kiện sau:
• Ở 400 K, khi [A2] = 1,0×10-1 mol·L-1 thì [A] = 4,0×10-3 mol·L-1.
• Khi thí nghiệm đầu tiên được thực hiện lặp lại ở 425 K thấy tốc độ phản ứng ban đầu
tăng gấp ba lần.
• Năng lượng hoạt hóa của giai đoạn chậm nhất là 45,0 kJ/ mol.
Hướng dẫn giải:
Câu 1 Hướng dẫn Điểm
1/2
1.a v = ktc[A2] [B] 0,25
-8 -1/2 -1
1.b ktc = 1,6.10 M .s ; 0,25
1.c k1
1 A2 (g) 2 A(g)
k-1
k2
2 A(g) + B(g) + C(g) ABC(g)
k3
2 ABC(g) AB(g) + C(g)
⎯⎯⎯⎯⎯⎯⎯⎯⎯⎯⎯⎯⎯⎯
0,5
A2(g) + 2 B(g) → 2 AB(g)
1.d k1[ A2 ] 0,25
k1[ A2 ] = k−1[ A]2 => [ A] =
k−1

Có: v = k2[ A][ B][C ] = k2 k1[ A2 ] [ B][C ] = k2 k1 [C ][ A2 ]1 / 2[ B] = koverall[ A2 ]1 / 2[ B]


k−1 k−1 0,25

k1
với k overall = k 2 [C ]
k −1

Nguyên tắc: ln K2 = − H  1 − 1  ; Biết giá trị hằng số cân bằng ở 2


1.e o

K1  R  T2 T1 

nhiệt độ khác nhau sẽ tìm được Ho;


k1
Tại 400K: A2 (g) 2 A(g)
k-1

k1 [ A]2 (4.10 −3 ) 2
K 400K = = = = 1,6.10 − 4
k −1 [ A2 ] 0,1

k1 0,25
Có: k overall = k 2 [C ] = 1,6.10 −8
k −1

=> k overall 1,6.10 −8


k 2, 400K = = = 2,53.10 −5
−4
k1 1,6.10 0,05
[C ]
k −1

k2, 425K E* 1 1 0,25


Có: ln =− ( − ):
k2, 400K R 425 400

k2, 425K 45.103 1 1


ln −5
= − ( − )
2,53.10 8,314 425 400
0,25
=> k2, 425K = 5,61.10-5;
Tại 425K; k'overall = koverall3 = 4,8.10-8;
k1
Có: k 'overall = k 2 [C ]
k −1
2 2
0,25
k  k'   4,8.10 −8 
=> K 425K = 1 =  overall  =   = 2,93.10 − 4
k −1  k 2 [C ]   5,61.10  0,05 
−5

K 425 H o  1 1 
Có: ln =−  − 
K 400 R  425 400 
2,93.10−4 H o  1 1 
=> ln −4
= −  − 
1,6.10 8,314  425 400 
=> Ho = 34,2 kJ/mol

Câu 2(2,5 điểm). Cân bằng và phản ứng trong dung dịch. Pin điện - Điện phân.
1(1,0 điểm). Trong dung dịch nước cân bằng giữa hai dạng tồn tại của acetic acid (AcOH)
diễn ra như sau:
[𝐴𝑐𝑂 − ][𝐻 + ]
AcOH AcO- + H+, có hằng số cân bằng Ka = [𝐴𝑐𝑂𝐻]
= 1,78.10-5
Với [i] là nồng độ cân bằng của các cấu tử I trong dung dịch.
a. Đặt pH = -log[H+] và pKa = -logKa. Hãy chứng minh phương trình Henderson-Hasselbalch
[𝐴𝑐𝑂 − ]
pH = pKa + log[𝐴𝑐𝑂𝐻]
b. Trong một quá trình phản ứng, ta cần giữ pH của hệ phản ứng là 5. Ta có thể pha chế dung
dịch đệm có pH = 5 từ acetic acid và sodium acetate. Tính thể tích acetic acid và khối lượng
sodium acetate cần để pha chế 250 ml dung dịch đệm có pH = 5 và tổng nồng độ cả hai tiểu
phân là 1M. Giả sử pH của dung dịch được tính theo phương trình Henderson-Hasselbalch ở
trên.
Cho biết: Khối lượng mol: M(AcOH) = 60,052 gam/mol; M(AcONa) = 82,034 gam/mol;
Khối lượng riêng: d(AcOH) = 1,05 gam/ml.
Trong công nghiệp, người ta cần xác định dấu hiệu để kết thúc một phản ứng hoá học. Đối
với phản ứng acid-base, người ta sử dụng giá trị pH của dung dịch đó. Do đó, trước khi tiến
hành phản ứng, người ta phải tính toán trước giá trị pH khi phản ứng đã xảy ra hoàn toàn.
Trong tiêu chuẩn công nghiệp phản ứng được xem là xảy ra hoàn toàn khi hiệu suất đạt 99%,
có nghĩa là nồng độ sản phẩm gấp 100 lần các chất. Người ta có thể tính toán pH của dung
dịch theo phương trình Henderson-Hasselbalch ở trên.
c. Trong quá trình điều chế sodium acetate AcONa, người ta cho AcOH phản ứng với NaOH
trong dung dịch theo phương trình:
AcOH + NaOH → AcONa + H2O
Hãy tính giá trị pH để kết thúc quá trình điều chế sodium acetate.
d. Ngược lại, trong quá trình điều chế acetic acid AcOH, người ta cho AcONa phản ứng với
HCl trong dung dịch theo phương trình:
AcONa + HCl → AcOH + NaCl
Hãy tính giá trị pH để kết thúc quá trình điều chế acetic acid.
2.(1,5 điểm) Pin sau được cấu tạo bởi điện cực hidro tiêu chuẩn (NHE) và điện cực kim loại
M nhúng trong dung dịch ion của nó (Mn+): NHE // Mn+ (x M) / M
Thực nghiệm cho thấy suất điện động của pin, Epin, phụ thuộc vào [Mn+] như trong bảng 1.
Nếu thêm vào pin trên NH3 thì pin tạo thành: NHE // Mn+ (0,001M), NH3 (y M) / M, lại có
suất điện động phụ thuộc vào [NH3]. Kết quả được biểu diễn trên đồ thị dưới. Biết Mn+ phản
ứng với NH3 theo phương trình:
[ M ( NH 3 ) np+ ]
Mn+ + p NH3 M(NH3)pn+ p =
[ M n + ][ NH 3 ] p
Hãy tính nồng độ cân bằng của các cấu tử trong 1 lit dung dịch có chứa 0,001 mol Mn+ và
1,05 mol NH3.
Bảng 1
x Epin, V
0,005 0,93
0,500 0,99

Mối quan hệ giữa Epin và log(y).


Hướng dẫn giải:
Câu 2 Hướng dẫn Điểm
1.a. [𝐴𝑐𝑂 − ][𝐻 + ] [𝐴𝑐𝑂𝐻]
Từ hằng số cân bằng Ka = [𝐴𝑐𝑂𝐻]
=> [𝐻 + ] = Ka [𝐴𝑐𝑂−]
[𝐴𝑐𝑂𝐻]
Logrit hoá 2 vế, ta được: log[𝐻 + ] = log(Ka [𝐴𝑐𝑂−] )
[𝐴𝑐𝑂 − ] [𝐴𝑐𝑂 − ] 0,25
=> -log[𝐻 + ] = -logKa + log[𝐴𝑐𝑂𝐻] => pH = pKa + log[𝐴𝑐𝑂𝐻]
1.b. [𝐴𝑐𝑂 − ]
Từ phương trình: pH = pKa + log[𝐴𝑐𝑂𝐻]
[𝐴𝑐𝑂 − ]
=> log[𝐴𝑐𝑂𝐻] = 𝑝𝐻 − 𝑝𝐾𝑎 = 5 – 4,75 = 0,25
[𝐴𝑐𝑂 − ]
=> [𝐴𝑐𝑂𝐻] = 100,25 (1)
Mặt khác, ta có: [𝐴𝑐𝑂 − ] + [𝐴𝑐𝑂𝐻] = 1 (2)
Giải hệ (1) và (2), ta được:
[𝐴𝑐𝑂− ] = 0,64 𝑀 𝑣à [𝐴𝑐𝑂𝐻] = 0,36 𝑀
=> 𝑛𝐴𝑐𝑂− = 0,64.0,25 = 0,16 mol => 𝑚𝐴𝑐𝑂𝑁𝑎 = 13,13 gam
0,25
𝑛𝐴𝑐𝑂𝐻 = 0,25.0,36 = 0,09 mol => 𝑉𝐴𝑐𝑂𝐻 = 5,19 ml
1.c. Trong phản ứng này, AcOH là chất tham gia, AcONa là sản phẩm nên:
[𝐴𝑐𝑂 − ]
= 102
[𝐴𝑐𝑂𝐻]
[𝐴𝑐𝑂− ]
=> 𝑝𝐻 = 𝑝𝐾𝑎 + 𝑙𝑜𝑔 = 4,75 + 2 = 6,75
[𝐴𝑐𝑂𝐻]
Vậy, khi pH = 6,75 thì ta có thể kết thúc quá trình điều chế sodium acetate.

0,25
1.d. Trong phản ứng này, AcONa là chất tham gia, AcOH là sản phẩm nên:
[𝐴𝑐𝑂𝐻]
= 10−2
[𝐴𝑐𝑂− ]
[𝐴𝑐𝑂𝐻]
=> 𝑝𝐻 = 𝑝𝐾𝑎 + 𝑙𝑜𝑔 = 4,75 − 2 = 2,75
[𝐴𝑐𝑂− ]
Vậy, khi pH = 2,75 thì ta có thể kết thúc quá trình điều chế acetic acid. 0,25
2. Vì nồng độ Mn+ tăng thì Epin tăng nên suy ra điện cực Mn+/M là điện cực
dương. Có:
0,0592
E pin = EM n+ / M − EH0 + / H = EM0 n+ / M + log[ M n + ]
2
n
0,0592
=> 0,93 = EM0 n+ / M + log 0,005 (1)
n
0,25
0,0592
0,99 = E 0
M n+ / M
+ log 0,500 (2)
n
(1), (2) => EM0 n+ / M = 1,00V ; n = 2; (Pd2+ ??? kiểm tra 4 nữa để confirm)
0,25
Khi có mặt NH3.
2+
0,0592 0,0592 [ M ( NH3 ) p ]
E pin = E0
M 2+ / M
+ log[ M 2 + ] = EM0 2+ / M + log
2 2  p [ NH3 ] p
Nếu C0NH3 >> 0,001 thì phương trình trên được viết lại thành:
0,0592 0,001
=> E pin = 1,00 + log
2  p [ NH3 ] p 0,25
Khi C0NH3 = 1,00 M thì Epin = 0,31 V
=>
0,0592 0,001 0,0592 0,001
0,31 = 1,00 + log = 1,00 + log
2  p  (1,00 − 0,001 p) p
2  p 1,00 p 0,25
=> p = 2,05 10 ; 20

Khi C0NH3 = 0,1 M thì Epin = 0,43 V 0,25


0,0592 0,001
=> 0,43 = 1,00 + log
2  p  (0,1 − 0,001 p) p
=> p=4
M2+ + 4 NH3 M(NH3)42+
0,001-x 1,05-4x x M
x 0,25
Có: 4 = = 2,05.1020
(0,001 − x)(1,05 − 4 x) 4
Giả sử x = 0,0001
0,001
=> [ M 2 + ] = (0,001 − x) = = 4,07.10− 24 M
2,05.1020 (1,05 − 0,004) 4
(Ktgt: thỏa mãn)
=> [NH3] = 1,045 M
[M(NH3)42+] = 0,001 M

Câu 3 (2,5 điểm) Nhiệt động học và cân bằng hóa học.
1. (1,5 điểm) Formaldehyde là một nguyên liệu thô quan trọng trong tổng hợp hữu cơ, được
sử dụng rộng rãi trong sản xuất nhựa, sợi tổng hợp, sơn và hoá chất vệ sinh nhà cửa, nội
thất,…Một phương pháp để sản xuất formaldehyde trong công nghiệp là oxi hoá methanol
bằng cách cho hơi methanol và một lượng không khí nhất định được dẫn qua lớp xúc tác Ag
ở nhiệt độ 550oC, khi đó methanol bị oxi hoá thành formaldehyde theo phương trình:
1
CH3OH(g) + 2 O2(g) → HCHO(g) + H2O(g) (*)
Phản ứng (*) có:
𝑜
- Biến thiên enthalpy chuẩn ở 298: ∆𝑟 𝐻298 = -149,43 (kJ/mol)
𝑜
- Biến thiên nhiệt dung mol đẳng áp chuẩn: ∆𝐶𝑝,𝑚 = 10,2 J/mol.K (Không phụ thuộc vào
nhiệt độ)
- Hằng số cân bằng ở 298 K: K298 = 3,95.1029.
a. Biến thiên enthalpy chuẩn của phản ứng phụ thuộc vào nhiệt độ theo định luật Kirchhoff
𝑇
∆𝑟 𝐻𝑇𝑜 = ∆𝑟 𝐻298
𝑜 𝑜
+ ∫298 ∆𝐶𝑝,𝑚 𝑑𝑇
Thiết lập biểu thức tính biến thiên enthalpy chuẩn của phản ứng theo nhiệt độ T.
b. Tính biến thiên enthalpy chuẩn của phản ứng sản xuất fomaldehyde ở 550oC.
c. Hằng số cân bằng của phản ứng phụ thuộc vào nhiệt độ theo phương trình van’t Hoff:
𝑑𝑙𝑛𝐾 ∆𝑟 𝐻𝑇𝑜
=
𝑑𝑇 𝑅𝑇 2
Từ định luật Kirchhoff và phương trình van’t Hoff , hãy chứng minh rằng: ln𝐾𝑇 =
1 𝑜 1 1 𝑜 𝑇 298
ln𝐾298 + 𝑅 [ ∆𝑟 𝐻298 (298 − 𝑇) + ∆𝐶𝑝,𝑚 (𝑙𝑛 298 + − 1)]
𝑇
Trong đó: K298 và KT lần lượt là hằng số cân bằng ở nhiệt độ 298 K và ở nhiệt độ T;
𝑜 𝑜
∆𝑟 𝐻298 là biến thiên enthalpy của phản ứng ở 298 K, ∆𝐶𝑝,𝑚 là biến thiên nhiệt dung mol đẳng
áp chuẩn của phản ứng.
d. Tính hằng số cân bằng của phản ứng sản xuất formaldehyde ở 550oC.
2(1 điểm). Một chất lỏng X có mùi dễ chịu chứa thành phần % theo khối lượng như sau:
52,2% C; 13,0% H, phần còn lại là oxi. Phổ khối của X không có peak trong vùng m/z > 90.
Chất X được điều chế cách đây hàng nghìn năm nhờ phản ứng oxi hóa - khử kị khí trong
dung dịch loãng.
(a) Hãy viết CTCT và gọi tên của X.
Áp suất hơi của chất lỏng X ở 60oC là p60 = 46,7 kPa và ở 70oC là p70 = 72,2 kPa. Nhiệt hóa
hơi của X là ΔHhh = 862 J.g-1.
(b) Hãy tính nhiệt độ sôi của X.
(c) Hãy tính entropy hóa hơi và hằng số nghiệm sôi của X.
Hòa tan 5,00 g andehit vanillic (4-hydroxi-3-metoxybenzencacbandehit) vào 100 g dung môi
X.
(q) Hãy tính áp suất hơi của dung dịch thu được ở 60°C?
Hướng dẫn giải:
Câu 3 Hướng dẫn Điểm
1.a. Áp dụng định luật Kirchhoff, ta có:
𝑇
∆𝑟 𝐻𝑇𝑜 = ∆𝑟 𝐻298
𝑜 𝑜
+ ∫298 ∆𝐶𝑝,𝑚 𝑑𝑇
𝑜 𝑜 0,25
= ∆𝑟 𝐻298 + ∆𝐶𝑝,𝑚 (𝑇 − 298)
1.b. Ở nhiệt độ 550oC ( tức là 823K), ta có:
∆𝑟 𝐻𝑇𝑜 = ∆𝑟 𝐻298
𝑜 𝑜
+ ∆𝐶𝑝,𝑚 (𝑇 − 298)
= -149,43 + 10,2.10-3(823-298) = -144,075 (kJ/mol) 0,25
1.c. Từ định luật Kirchhoff:
∆𝑟 𝐻𝑇𝑜 = = ∆𝑟 𝐻298
𝑜 𝑜
+ ∆𝐶𝑝,𝑚 (𝑇 − 298)
Thay vào phương trình van’t Hoff:
𝑜 + ∆𝐶 𝑜 (𝑇−298)
𝑑𝑙𝑛𝐾 ∆𝑟 𝐻𝑇𝑜 ∆𝑟 𝐻298 𝑝,𝑚
= =
𝑑𝑇 𝑅𝑇 2 𝑅𝑇 2
𝑜
1 ∆𝑟 𝐻298 𝑜 1 298
=𝑅[ + ∆𝐶𝑝,𝑚 (𝑇 − )]
𝑇2 𝑇2
𝑜
=> 𝑑𝑙𝑛𝐾 =
1
[
∆𝑟 𝐻298 𝑜
+ ∆𝐶𝑝,𝑚 (𝑇 −
1 298
)]𝑑𝑇 (**) 0,25
𝑅 𝑇2 𝑇2
Tích phân 2 về phương trình (**) cận từ 298 đến T, ta được:
𝑇 𝑜
1 𝑇 ∆𝑟 𝐻298 𝑜
1 298
∫ 𝑑𝑙𝑛𝐾 = ∫ [ + ∆𝐶𝑝,𝑚 ( − )]𝑑𝑇
298 𝑅 298 𝑇2 𝑇 𝑇2
𝑜 𝑇
1 ∆𝑟 𝐻298 𝑜 298 𝑇
𝑙𝑛𝐾𝑇 − 𝑙𝑛𝐾298 = [ (− ) + ∆𝐶𝑝,𝑚 (𝑙𝑛𝑇 + ) ] 0,25
𝑅 𝑇 298 𝑇 298
1 𝑜 1 𝑜 1 𝑇 298
=> ln𝐾𝑇 = ln𝐾298 + [ ∆𝑟 𝐻298 (298 − 𝑇) + ∆𝐶𝑝,𝑚 (𝑙𝑛 298 + − 1)]
𝑅 𝑇
(đpcm)
0,25
1.d. Ở nhiệt độ 550 C ( tức là 823K), ta có:
o

1 1 1 823
ln𝐾823 = ln(3,95.1029 ) + [ −149,43. 103 (298 − 823) + 10,2(𝑙𝑛 298 +
8,314
298 0,25
− 1)] = 30,14 => 𝐾823 = 1,23. 1013
823
2.a CH3-CH2-OH, etanol 0,25
2.b ΔHV,m = 862×46 = 39652 J/mol
Chất lỏng bay hơi khi PX = Pkq = 1 atm;
p(T2 ) HV 1 1 1 1 R p(T2 )
ln =  −   = −  ln
p(T1 ) R  T1 T2  T2 T1 HV p(T1 )
1
T1= 60°C: = 0.002840643  T2 = 352.033 K
T2
0,25
1
T1 = 70°C: = 0.002844445  T2 = 351.562 K
T2
Giá trị nhiệt độ sôi trung bình: 351,8 K = 78,7°C.
2.c HV,m 39652
S V,m = = = 113 J / K
T 351.8
TS2  R  M 351.82  8.314  46 0,25
K EB = = = 1.19 Kkg / mol
1000  HV 1000  39652

2.d n (etanol) = 100/46 = 2,1740 mol,


n (andehit vanillic) = 5/152 = 0,0329 mol,
n = 2,2069 mol
=> Xetanol = 0,9851 0,25
=> p (etanol) = P0×X = 46,0 kPa

Câu 4 (2,5 điểm) Hóa nguyên tố (Kim loại, phi kim nhóm IVA, VA). Phức chất.
1. Hợp chất X1 màu vàng, gồm hai nguyên tố, hòa tan được hoàn toàn trong axit nitric đặc khi
đun nóng, giải phóng ra một chất khí X2 có tỉ khối so với không khí bằng 1,586. Khi thêm
bari clorua dư vào dung dịch thu được ở trên, một chất rắn màu trắng X3 được tách ra có khối
lượng m3 (gam). Lọc kết tủa. Phần nước lọc cho phản ứng với một lượng dư dung dịch bạc
nitrat tạo thành kết tủa trắng X4, chúng cũng được tách ra bằng cách lọc. Thêm từng giọt
dung dịch natri hiđroxit vào phần nước lọc mới thu được đến khi dung dịch có môi trường
gần như trung tính (pH khoảng bằng 7). Tại thời điểm này thu được một chất bột màu vàng
X5 có khối lượng m5 gam (trong đó Ag chiếm 77,32 % về khối lượng). Khối lượng của X5
lớn hơn so với khối lượng của X3 trong phần kết tủa đầu tiên 2,4 lần.
(a) Xác định công thức hóa học của các chất từ X1 đến X5, Viết các PTPU.
(b) Vẽ cấu trúc của X1.
2. Xét phức chất sau: [Co(CN)6]3-
a. Vẽ giản đồ sự tách mức năng lượng trong phân tử phức theo thuyết trường tinh thể.
b. Tính năng lượng bền hóa trường tinh thể Ebh
Biết: Thông số tách năng lượng phức bát diện: o = 401 kJ/mol;
Năng lượng ghép đôi: P = 251 kJ/mol).
Hướng dẫn chấm:
Câu 4 Hướng dẫn Điểm
1 a)
X3 là BaSO4 vậy trong X1 có nguyên tố S.
Phần dung dịch phản ứng AgNO3 thu được kết tủa trắng là AgCl (X4) 0,25
Kết tủa X5 có Ag chiếm 77,32 % về khối lượng là X5 là Ag3PO4 vậy X1 có
nguyên tố P 0,25
ns m5 419 3
Trong X1 = x = vậy X1 là P4S3
nP m3 233 4
M X 2 = 1.586 × 29 = 46 vậy X2 là NO2. 0,25

Các PTPU
P4S3 + 38 HNO3 → 4 H3PO4 + 3 H2SO4 + 38 NO2 + 10 H2O
H2SO4 + BaCl2 → BaSO4 + 2HCl
HCl + AgNO3 → HNO3 + A gCl
0,5
BaCl2 + 2AgNO3 → Ba(NO3)2 + 2 AgCl
H+ + OH- → H2O
H3PO4 + 3 NaOH + 3 AgNO3 → Ag3PO4 + 3 NaNO3 + 3H2O
b. Cấu trúc P4S3 0,5

2 2.a. [Co(CN)6]3- 0,5


Co3+ : [Ar]3d6
Trong trường bát diện sự tách electron như sau:

b. (Năng lượng bền hóa phức chất phụ thuộc tổng năng lượng P +  và được cho 0,25
bởi công thức: Ebh = nt2 g .Et2 g + neg .Eg + x.P với x là số cặp elecctron ghép đôi
mới)
Ebh = -6.2/5 o + 2 P = -460 kJ/mol
Câu 5 (2,5 điểm) Đại cương hữu cơ.
1( 1,0 điểm): Cho 5 hợp chất dị vòng
NH N N
NH NH
N

N
A B C D E

Sắp xếp theo thứ tự tăng dần tính bazơ?


2( 1,5 điểm):
Axit domoic là một chất độc đối với hệ
COOH
thần kinh có công thức như sau:
a) Gọi tên các nhóm chức của (D).
b) Cho biết số đồng phân cấu hình
HOOC (D)
COOH
của (D). Xác định cấu hình cụ thể của D N
H
Hướng dẫn giải:
Câu 5 Hướng dẫn Điểm
1. Sắp xếp theo thứ tự tăng dần tính bazơ
NH N NH NH
N
N
N 0,5

Cặp e trên N Cả 2 N đều N sp2, không N sp2 chịu N sp3, vòng


đã tham gia chịu ảnh có hiệu ứng –I ảnh hưởng no đẩy e
0,5
vào hệ liên hưởng của của hiệu ứng
hợp hiệu ứng –I +C của NH
của nhau
(Nsp2)
2 Tương tự các bài tập trên, có thể xác định dễ dàng số đồng phân và danh pháp cấu
hình.
a) Các nhóm chức: Cacboxi, amin bậc 2, anken. 0,5
b) Cấu hình: 6 đồng phân cấu hình, từ trái sang phải: R, E, Z, S, S, S2 0,5

Câu 6 (2,5 điểm) Sơ đồ tổng hợp hữu cơ. Cơ chế phản ứng hóa hữu cơ.
1( 1,5 điểm). Đề xuất cơ chế của mỗi phản ứng sau
2( 1,0 điểm). Triquynacene được tổng hợp theo sơ đồ sau:

Hướng dẫn giải:


Câu 6 Hướng dẫn Điểm
1.a

0,25

0,25

0,25
0,25

0,25

0,25

2.
0,5

0,5

Câu 7 (2,5 điểm) Xác định cấu trúc các chất hữu cơ (mô tả sơ đồ tổng hợp bằng lời dẫn)
1(1,25 điểm). Hợp chất A(C10H12O3) là một hợp chất có tính quang hoạt, tan trong dung dịch
NaHCO3, và tham gia phản ứng idoform. Hợp chất B được tạo thành khi đun nóng A thì
không có tính quang hoạt và tan được trong dung dịch NaHCO3. Ozon phân B dưới sự hiện
diện của Zn/CH3COOH thì thu được hợp chất C và D. C có thể tham gia phản ứng idoform.
D khi đun nóng thì giả phóng khí CO2 và thu được E(C7H6O), E tham gia phản ứng tráng
gương. Xác định công thức cấu tạo các hợp chất A,B,C,D và E.
2(1,25 điểm). Xác định cấu trúc của các chất trong sơ đồ sau:

Hướng dẫn giải:


Câu 7 Hướng dẫn Điểm
1.

0,25x5

2. 0,25x5

Câu 8 (2,5 điểm) Hóa học các hợp chất thiên nhiên (Cacbohidrat và các hợp chất hữu cơ
chứa nito đơn giản)
Chất E (C17H29NO11) được tách ra từ một loài sao biển. Thủy phân E nhờ enzym β-
glicoziđaza, được chất G (C6H12O5, thuộc dãy D) và chất H. Cho E phản ứng với MeI/Ag2O
dư, sau đó sản phẩm được thủy phân trong môi trường axit, thu được axit (2S,3R)-3-hiđroxi-
1-metylpiroliđin-2-cacboxylic, chất F (C8H16O5) và chất I (C9H18O5). Khi oxi hóa F và I bởi
HNO3, thì từ F thu được hỗn hợp axit trong đó có axit axetic và axit (2S,3S)-2,3-
đimetoxibutanđioic (-T), còn từ I tạo thành axit axetic, axit (2R,3R)-2,3-đimetoxibutanđioic
(+T) và axit 2,3,4-trimetoxipentanđioic không quang hoạt (chất K). Đun nóng H, thu được
chất L (C10H14N2O4). Biết piroliđin là dị vòng no năm cạnh chứa 1 nguyên tử nitơ.
a) Vẽ công thức lập thể của chất K và chỉ rõ yếu tố đối xứng trong cấu trúc của K.
b) Vẽ công thức chiếu Fisơ của F và I, ghi rõ cấu hình tuyệt đối của các nguyên tử cacbon bất
đối.
c) Xác định cấu trúc của H, L và cho biết L có quang hoạt không?
d) Vẽ công thức phối cảnh của E.
Hướng dẫn giải:
Câu 8 Hướng dẫn Điểm
a.
0,25

0,25
Mặt phẳng đối xứng phân tử (tạo bởi C3, O và H) vuông góc với mặt phẳng giấy.
b. Khi bị oxi hóa bởi HNO3, từ I tạo thành axit axetic, axit (2R, 3R)-2,3-
đimetoxibutanđioic và axit 2,3,4-trimetoxipentanđioic không quang hoạt (chất K), 0,25
từ đó suy ra công thức chiếu Fisơ của I.

0,25

0,25
F và I đều tạo ra từ gốc G của E nên 4 cacbon bất đối ở F có cấu hình như ở I vì
vậy ta chỉ cần xác định vị trí của hai nhóm OH trong F. Do khi bị oxi hóa bởi
HNO3, từ F thu được hỗn hợp axit trong đó có axit axetic và axit (2S, 3S)-2,3-
đimetoxibutanđioic (-T) nên 1 nhóm OH phải gắn với C5 cạnh nhóm CH3 và 1
nhóm OH gắn với C2.

0,25

c. Thủy phân E (C17H29O11N) nhờ enzym β-glycozidaza, thu được chất G (C6H12O5)
và chất H. Đun nóng H,thu được chất L(C10H14N2O4). Do vậy E gồm 2 gốc G và
1gốc H có 5 cacbon. Cho E phản ứng với MeI/Ag2O dư rồi thủy phân sản phẩm 0,25
trong môi trường axit thu được axit (2S,3R)-3-hiđroxi-1-metylpiroliđin-2-
cacboxylic do đó H là axit (2S, 3R)- 3-hiđroxipiroliđin-2- cacboxylic.

L quang hoạt vì C2 và C2’ đều có cấu hình S và C3, C3’ đều có cấu hình R như ở
H. Nếu chúng có cấu hình ngược nhau thì phân tử có tâm đối xứng và sẽ không 0,25
quang hoạt.
d Thủy phân E nhờ enzym β-glicoziđaza thu được chất G chứng tỏ hai liên kết
glicozit đều ở dạng β. Cùng với dữ liệu về F và I ta có công thức lập thể phối cảnh 0,25
của E.

0,25

GV ra đề phần vô cơ: Võ Thị Phương Thảo - 0986772900


GV ra đề phần hữu cơ: Hoàng Ngân Khánh - 0915264152
HỘI CÁC TRƯỜNG CHUYÊN KỲ THI CHỌN HỌC SINH GIỎI THPT
VÙNG DUYÊN HẢI VÀ ĐBBB CHUYÊN - DUYÊN HẢI BẮC BỘ NĂM 2023
TRƯỜNG THPT CHUYÊN LÊ QUÝ ĐÔN MÔN: HÓA HỌC LỚP 11
TỈNH ĐIỆN BIÊN Thời gian làm bài: 180 phút

ĐỀ THI ĐỀ XUẤT

Câu 1 (2,5 điểm): Tốc độ phản ứng


1. Đối với phản ứng: A + B → C + D có biểu thức tốc độ phản ứng v = k. [A].[B]
a) Trộn 2 thể tích bằng nhau của dung dịch chất A và dung dịch chất B có cùng nồng độ
1,0 M:
- Nếu thực hiện phản ứng ở nhiệt độ 300 K thì sau 2 giờ nồng độ của C bằng 0,215 M.
Tính hằng số tốc độ của phản ứng.
- Nếu thực hiện phản ứng ở 370 K thì sau 1,33 giờ nồng độ của A giảm đi 2 lần. Tính
năng lượng hoạt hóa của phản ứng (theo kJ/mol).
b) Nếu trộn 1 thể tích dung dịch A với 3 thể tích dung dịch B đều cùng nồng độ 1,0 M, ở
nhiệt độ 300 K thì sau bao lâu A phản ứng hết 90%?
2. Cho cân bằng ở 250C: A  k
 B là phản ứng thuận nghịch bậc 1. Thành phần % của

1

k 2

hỗn hợp phản ứng được cho dưới đây:

Thời gian (giây) 0 45 90 270 ∞


%B 0 10,8 18,9 41,8 70

Hãy xác định giá trị k1, k2 của phản ứng. Tính hằng số cân bằng hằng, số tốc độ của phản
ứng.

Câu 1 Hướng dẫn chấm Điểm


1 Theo đề: v = k. [A].[B] nên phản ứng bậc 2.
1, 0
a) CA = CB = a =  0,5 M
2
Nồng độ đầu 2 chất phản ứng bằng nhau nên phương trình động học:
1 1 1
k= (  )
t ax a 0,25
1 1 1
Tại T1 = 300K: k1 = (  )  0,7544 (mol‒1.lít.giờ‒1)
2 0,5  0, 215 0,5
1 1 1
Tại T2 = 370K: k 2 = (  )  1,5037 (mol‒1.lít.giờ‒1) 0,25
1,33 0,5  0, 25 0,5
k E 1 1
Phương trình Arrhenius: ln 2  a (  )
k1 R T1 T2 0,25
1,5037 E 1 1
 ln  a (  )
0, 7544 8,314 300 370
 Ea = 9093,55 (J/mol)
b) Ở 300K, k = 0,7544 mol‒1.lít.giờ‒1 0,25
1, 0 1, 0
CA = a = 1.  0, 25 M; CB = b = 3.  0, 75 M,
4 4
theo đề: x = 90%. a = 0,225 M
Do nồng độ đầu hai chất khác nhau nên phương trình động học:
1 a.(b  x) 0,25
t .ln
k(b  a) b.(a  x)
1 0, 25  (0,75  0, 225)
= .ln  5,16 (giờ)
0,7544  (0,75  0, 25) 0,75  (0,25  0, 225)
0,25

2 Phương trình động học của phản ứng thuận nghịch bậc 1:
A 

k1
 B
k2

t=0: a 0
t: a–x x
t cân bằng: a – xcb xcb
1 x cb k1 [B]
Biểu thức: k1 + k 2 = .ln K cb = =
t x cb  x k2 [A]
Tại thời điểm cân bằng (∞): xcb = 70% 0,25
1 70
Tại t = 45 giây: x = 10,8%  k tong 1  .ln  3,72.103
45 70  10,8
1 70
Tại t = 90 giây: x = 18,9%  k tong 2  .ln  3,50.103
90 70  18,9
1 70
Tại t = 270 giây: x = 41,8%  k tong 3  .ln  3,37.103
270 70  41,8
0,25
Lấy trung bình:
k tong 1 + k tong 2 + k tong 3
k1 + k 2   3,53.103
3
k1 [B]cb 70
Mặt khác, hằng số cân bằng phản ứng: K cb = =   2,333
k2 [A]cb 30 0,25
‒3 ‒1
Do đó: k1 = 2,47. 10 s
k2 = 1,06. 10‒3 s‒1
0,25

Câu 2 (2,5 điểm): Cân bằng và phản ứng trong dung dịch. Pin điện – Điện phân

Một trong những thuốc thử đặc trưng để tìm ion Pb2+ (trong dung dịch) là Na2CrO4. Cho
biết, kết tủa PbCrO4 màu vàng, tan được trong dung dịch NaOH dư; trong khi đó, kết tủa PbS
màu đen, không tan được trong dung dịch NaOH.
Thêm từ từ 0,05 mol Pb(NO3)2 vào 1,0 L dung dịch X gồm 0,02 mol Na2S và 0,03 mol
Na2CrO4, thu được hỗn hợp Y gồm phần kết tủa và phần dung dịch (coi thể tích không thay đổi
khi thêm Pb(NO3)2 vào dung dịch X).
1. Tính pH của dung dịch X.
2. Bằng lập luận và đánh giá hợp lí, chứng tỏ rằng, pH phần dung dịch của Y xấp xỉ bằng 7,0.
3. Tính [Cr2O72 ] và [Pb2+] trong phần dung dịch của Y.
4. Trình bày cách thiết lập sơ đồ pin được ghép bởi điện cực chì (Pb) nhúng trong hỗn hợp
Y và điện cực hiđro tiêu chuẩn.
Cho biết:
0
pK a1(H2S) = 7,02; pK a2(H2S) = 12,90; pK  = 6,50; E Pb2+ /Pb = -0,126 V
a(HCrO4 )

pKs(PbS) = 26,60; pKs(PbCrO4 ) = 13,70; pKs(Pb(OH)2 ) = 14,90


2 CrO 24 + 2H+ Cr2O7
2
+ H2 O K = 3,13.1014

Pb2+ + H2O PbOH+ + H+ lg β1 = lg  β = -7,80
Pb(OH)

Pb2+ + 2H2O Pb(OH)2(dd) + 2H+ lg  β2 = lg  βPb(OH)2 = -17,20


 
Pb2+ + 3H2O Pb(OH)3 + 3H
+
lg β3 = lg  β = -28,00
Pb(OH)3

2,303RT
(với pKa = -lgKa; pKs = -lgKs; ở 25oC: = 0,0592 V)
F

Câu 2 Hướng dẫn chấm Điểm


1 Các quá trình xảy ra trong dung dịch X:
S2- + H2O HS- + OH- Kb1 = 10-1,1 (1)
HS- + H2O H2S + OH- Kb2 = 10-6,89 (2)
2 
CrO 4 + H2O HCrO 4 + OH- Kb = 10-7,50 (3)
-
Chú ý: Kb của (3) tính được từ pKa của HCrO4 và Kw(H2O). 0,25
H2O OH- + H+ Kw = 10-14 (4)
So sánh các cân bằng (1), (2), (3) và (4) ta có:
Kb1. CS 2 >> Kb2. CHS  > Kb. CCrO2 >> Kw nên pHX được tính theo (1):
4
0,25
2- - - -1,1
S + H2O HS + OH Kb1 = 10 (1)
[ ] 0,02 – x x x
 [OH-] = x = 0,0166 (M)  pH = 12,22.
2 Pb2+ + S2-  PbS 
0,05 0,02
0,03 -
Pb2+ + CrO 4  PbCrO4 
2

0,03 0,03
- -
Trong hỗn hợp Y, phần kết tủa gồm PbS và PbCrO4; phần dung dịch gồm Na+ và

NO3 . Từ cân bằng, ta có:
2 2
2 CrO 4 + 2H+ Cr2O7 + H2O K1 = 3,13.1014
+
2H2O 2H + 2OH- (KW)2 = 10-28
 2 CrO 24 + H2O Cr2O7
2
+ 2OH- K2 = 3,13.10-14
  
Vì KS(PbCrO4 ) >> KS(PbS) và vì β1 >> β2 >> β3 nên trong hỗn hợp Y chủ yếu xảy ra
các cân bằng:
2
PbCrO4 Pb2+ + CrO 4 KS = KS(PbCrO4 ) = 10-13,70 (5)
S S (với S là độ tan của PbCrO 4 trong
Y)

Pb2+ + H2O PbOH+ + H+ β1 = 10-7,80 (6)
2 
CrO 4 + H2O HCrO 4 + OH- Kb = 10-7,50
(7a)
2 2
2 CrO 4 + H2 O Cr2O7 + 2OH- K2 = 3,13.10-14
(7b)

H2O OH- + H+ Kw = 10-14 (8)


-14
Vì K2 = 3,13.10 nhỏ, nên chấp nhận bỏ qua quá trình (7b).
So sánh các cân bằng (6), (7a) và (8) ta thấy:
 0,5
β1 . S0 =  β1 K s = 10-14,65  Kb. S0 = Kb. K s = 10-14,35  KW = 10-14
(với S0 là độ tan của PbCrO4 trong nước không kể các quá trình (6), (7a) và (7b)), do
đó, có thể coi khả năng cho, nhận proton của các cấu tử trong hỗn hợp Y gần như
tương đương nhau, vì vậy pHY  7,0.
Chú ý:
Việc kiểm tra cho thấy ở pH = 7,0:
17,2  28
2 3
* *
[Pb(OH) 2(dd) ] 10 [Pb(OH)3 ] 10
2
  2
 14
1; 2
  3
 21
1
[Pb ] [H ] 10 [Pb ] [H ] 10

nghĩa là việc tính toán bỏ qua sự tạo phức Pb(OH)2(dd) và Pb(OH)3 là hợp lý.

3 Tính [Cr2O72-] và [Pb2+] trong phần dung dịch của Y:


Tại pH = 7,0 thì:
2 2 2
[ Cr2O7 ] = 3,13.1014.[ CrO 4 ]2.[H+]2 = 3,13.[ CrO 4 ]2
2  2
Mặt khác: [ CrO 4 ] + [ HCrO 4 ] + 2[ Cr2O7 ] = S
2 2
Giả sử [ Cr2O7 ] << [ CrO 4 ] thì:
2 1
[ CrO 4 ](1 + K a .[H+]) = S
S
 2
[ CrO 4 ] = 1 
(a)
1  K a .[H ]
Lại có: [Pb ] + [PbOH+] = S
2+

S
 2
[Pb ]   1
(b)
1  1.[H ]
*

Từ (a) và (b)
 S=  1 1 
K s .(1  1.[H ] ).(1  K a .[H ]) , thay các giá trị hằng số vào
*

thu được

S =1,74.10-7 (M)
Thay giá trị S vào (a) ta có:

[CrO42-] = 1,32.10-7 (M)  [Cr2O72-] = 3,13.[ CrO 4 ]2 = 3,13.(1,32.10-7)2 = 5,45.10-14


2

(M)
2
[Cr2O72-] << [ CrO 4 ] (thỏa mãn giả thiết đặt ra ban đầu!)
1,0
Từ (b), ta có:
[Pb2+] = 1,51.10-7 (M)

4 4. Vì trong hỗn hợp Y quá trình phân li của PbCrO4 là chủ yếu, nên:
0, 0592 2
2  E 2  E 2  lg[Pb ]  0,328 (V)  E  =
0 0
EPb = E
PbCrO4 /Pb, CrO4 Pb /Pb Pb /Pb 2H /H 2
2
0,00 (V)

do đó điện cực Pb là anot, điện cực hiđro tiêu chuẩn là catot. Vậy sơ đồ pin:
-
(-) Pb│PbS, PbCrO4 | Na+ 0,1 M, NO 3 0,1 M ║ H+ 1,0 M│ H2 (p = 1 bar) | Pt (+) 0,5

Câu 3 (2,5 điểm): Nhiệt động học và cân bằng hóa học
Cho cân bằng: 2SO2(k) + O2(k)  2SO3(k)
 (1)
Cân bằng (1) được nghiên cứu trong hai bình phản ứng, dưới áp suất được giữ không đổi
là 1,0 atm.
Các cân bằng được thực hiện từ các chất phản ứng SO2 và O2, theo các tỷ lệ hợp thức. Gọi
 là độ chuyển hóa của SO2, tức là tỷ số của lượng SO3 ở cân bằng với lượng SO2 ban đầu.
Bình thứ nhất ở 5500C,  = 0,80 và bình thứ hai ở 4200C,  = 0,97.
a) Hãy cho biết phản ứng (1) tỏa nhiệt hay thu nhiệt?
b) Xác định các hằng số cân bằng Kp của phản ứng (1) tại 5500C và 4200C. Từ đó suy ra
giá trị entanpi chuẩn ΔpưH0 và entropi chuẩn ΔpưS0 của phản ứng với giả thiết rằng các đại
lượng đó thay đổi không đáng kể trong khoảng nhiệt độ từ 4200C đến 7000C.
c) Xác định hằng số cân bằng Kp1 của phản ứng (1) ở 6500C.
Câu 3 Hướng dẫn chấm Điểm
1 a) Xét cân bằng: 2SO2(k) + O2(k) 2SO3 (k) (1)
Khi giảm nhiệt độ, hiệu suất phản ứng tăng lên nghĩa là cân bằng 0,5
chuyển dịch về phía thuận, vậy phản ứng đã cho là tỏa nhiệt độ.
b) Xét cân bằng: 2SO2(k) + O2(k) 2SO3(k) (1)
Ban đầu: 2a a
Cân bằng: 2a(1-) a(1-) 2a 0,25
Trong đó: 2a(1-) + a(1-) + 2a = 1 → a(3-) = 1atm
* Tính các hằng số cân bằng:
2
PSO
3  (2a)2 2 2 (3)
Kp   
2 .P
PSO [2a(1-)]2[a(1)] a(1)3 (1)3
2 O2
Tại nhiệt độ 5500C hay 823K,  = 0,80 → Kp(823K) = 176;
0,5
Tại nhiệt độ 4200C hay 693K,  = 0,97 → Kp(693K) = 7,074.104
* Tính ΔpưH0 và ΔpưS0
Từ công thức của định luật Van't Hoff, ta có:
0,25
K p (T2 )  pu H0  1 1 RT1T2 K p (T2 )
ln      H0  ln
K p (T1) R T  pu T2  T1 K p (T1)
 2 T1 
Thay số vào ta có: ΔpưH0 = -218,7 kJ/mol.
Ta có: ΔpưG0 = - RTlnKp(T) = ΔpưH0 - T.ΔpưS0 0,5
Với giá trị T ở 823K, ta có: -8,314.823.ln176 = - 218,7.103 - 823. ΔpưS0
ΔpưS0 = -222,75 J.K-1.mol-1
c) Tại nhiệt độ 6500C hay 923K 0,5
=> - 8,314.923 lnKp(923K) = -218,7.103 - 923.(-222,75)
→ Kp(923K) = 5,51.

Câu 4 (2,5 điểm): Hóa nguyên tố (Kim loại, phi kim nhóm IVA, VA). Phức chất
1. Kim loại M được dùng để sản xuất đuyra. M phản ứng với khí X, thu được hai chất rắn
là hợp chất A và đơn chất B. Chất M phản ứng với khí Y, thu được sản phẩm duy nhất là chất
D, trong đó M chiếm 72% về khối lượng. Nếu cho D tác dụng với nước, thu được hợp chất E ít
tan và hợp chất khí F có mùi đặc trưng. Khí F rất dễ tan trong nước (ở nhiệt độ 200C và áp suất
1 atm, 1 lít H2O hòa tan 700 lít khí F) và dung dịch thu được có phản ứng bazơ. Nếu nung nóng
hợp chất E ta được chất A và nước.
Cho biết công thức của các chất: M, X, Y, Z, A, B, D, E, F.
2. Một nguyên tố X có nhiều dạng thù hình, đa hóa trị, là nguyên tố thiết yếu cho cơ thể
sống, không bao giờ tồn tại ở trạng thái đơn chất trong tự nhiên. Cho 1,55 gam X màu trắng tan
hết trong axit HNO3 đặc nóng dư thu được 5,6 lít khí NO2 (đktc) là sản phẩm khử duy nhất và
dung dịch Y chỉ chứa axit. Cho sơ đồ phản ứng sau đây:
 ddBa ( OH )  H SO  ddCuSO 2 NaOH
Sơ đồ (1): X   A  
 B   D   E  F
0
2 2 4 4 600 C

 Ca ,t C  H 2O  ddAgNO3
Sơ đồ (2): X   G   L   M   D  Q
0 0 0
200 C 260 C

Biết A, B, D, E, F, G, L, M, Q đều là hợp chất của X có phân tử khối thỏa mãn:


MA + MG = 449; MB + ML = 100;
MF + MQ = 444; MD + MM = 180
Xác định nguyên tố X và các chất A, B, D, E, F, G, L, M, Q.
Câu 4 Hướng dẫn chấm Điểm
1 Khí F có mùi đặc trưng, dễ tan trong nước cho dung dịch có tính bazơ  F là
khí NH3  D có chứa nitơ
Đặt công thức của D là M3N2. 0,25
Theo đề bài % khối lượng của M trong D là 72%  MM = 12n  n=2, MM = 24
là hợp lí. M là kim loại Mg 0,25
- Khí Z là khí N2. Vậy D là Mg3N2  E là Mg(OH)2 và A là MgO  Y là O2 và
X là CO2  B là cacbon. 0,5
X: CO2 Y: O2 Z: N2 A: MgO
B: C D: Mg3N2 E: Mg(OH)2 F: NH3

- Xác định X: dựa vào bảo toàn e  MX = 31  X là P


- Xác định các chất trong sơ đồ
+ X + Ca  G  G là Ca3P2 0,25
MA + MG = 449  MA = 449-182 = 267
X + dung dịch Ba(OH)2 tạo thành A (muối)  A là Ba(H2PO2)2
G tác dụng với H2O  L  L là PH3
MB + ML = 100  MB = 100 – 34 = 66
Mà A + H2SO4 tạo B  B là H3PO2 0,5
B + CuSO4 có tính oxi hóa tạo D tác dụng được với NaOH  D là H3PO4  E
là Na2HPO4
MD + MM = 180  MM = 180-98 =82
Mà L + AgNO3 có tính oxi hóa tạo M, M mất nước tạo D  M là H3PO3 0,5
Nhiệt phân D mất nước tạo Q  Q là H4P2O7
MF + MQ = 444  MF = 444-178 = 266, nhiệt phân E tạo F  F là Na4P2O7 0,25

A B D E F G L M Q
Ba(H2PO2) H3PO2 H3PO4 Na2HPO4 Na4P2O7 Ca3P2 PH3 H3PO3 H4P2O7

Câu 5 (2,5 điểm): Đại cương hữu cơ


1. a) Vẽ dạng cấu trúc bền nhất của các chất dưới đây. Giải thích tại sao cấu trúc đó bền nhất.

b) Cho các chất dưới đây:


Chất nào trong mỗi cặp (C1, C2), (C3, C4), (C5, C6) có tính bazơ mạnh hơn? Giải thích.
2. So sánh nhiệt độ nóng chảy các chất sau:

3. So sánh pKa của các axit sau: HCOOH, C6H5COOH, CH3COOH, C6H5CH2COOH,
C6H5OH? Giải thích ngắn gọn?

Câ Hướng dẫn chấm Điể


u5 m
1
1. a) (0,75 điểm)
0,75

0,5
b) Tính bazơ: C1 < C2 (xét axit liên hợp), C3 < C4 (do nhóm Ph gây -C và -I yếu hơn
nhóm Ac), C5 > C6 (do nhóm CO2Me gây -I yếu hơn)

2 Inđol < Imiđazol < Purin 0,5


gần như ko còn liên kết H có một trung tâm N tạo liên kết H nhiều trung tâm N tạo liên kết H
liên kết H yếu, M nhỏ liên kết H bền hơn, M lớn

3 pKa: HCOOH < C6H5COOH < C6H5CH2COOH < CH3COOH < C6H5OH

0,75
Câu 6 (2,5 điểm): Sơ đồ tổng hợp hữu cơ. Cơ chế phản ứng hóa hữu cơ
1. Thực vật họ Cactaceae chứa ankaloit Mescalin. Con đường sinh tổng hợp Mescalin
theo sơ đồ sau

Trong đó quá trình metyl hóa được thực hiện bởi tác nhân sinh học SAM (S-
adenosylmethionin có vai trò giống như tác nhân hóa học MeI/Ag2O). Xác định cấu tạo của các
chất M, N, P, Q, S và mescalin.
2. Đề xuất cơ chế cho phản ứng sau của một β-keto ester.

Câu Hướng dẫn chấm Điểm


6
1 Cấu tạo các chất M, N, P, Q, S và mescalin

1,5
2

1,0

Câu 7 (2,5 điểm): Xác định cấu trúc các chất hữu cơ (mô tả sơ đồ tổng hợp bằng lời dẫn)
1. Hợp chất C1 (C10H18O) phản ứng với CH3MgBr, tạo khí metan; phản ứng với PCC, tạo
thành xeton; phản ứng với KMnO4 loãng, lạnh tạo thành chất C10H20O3. Axetyl hóa C1 bằng
CH3COCl, sau đó ozon phân/khử hóa, thu được C2 (C12H20O4). Oxi hóa C2 bằng nước brom, thu
được C3 (C12H20O5). Chất C3 tham gia chuyển vị Baeyer Villiger với m-CPBA (tỷ lệ mol 1:1) thu
được nhiều đồng phân trong đó có C4 (C12H20O6). Thủy phân C4 với H2SO4/H2O, thu được axit
ađipic HOOC[CH2]4COOH, butan-1,3-điol và axit axetic.
Xác định cấu tạo các chất C1, C2, C3 và C4.
2. Hai lacton thơm A, B có công thức phân tử [C10H10O4] đều tan trong dung dịch NaOH
loãng nhưng không tan trong dung dịch NaHCO3. Cả A và B cho phản ứng màu với dung dịch
FeCl3. Khi cho A phản ứng CH3I/K2CO3 tạo ra chất C[C11H12O4]. Biết C chứa ba nhóm metyl
không giống nhau, trong đó có một nhóm metyl liên kết vòng thơm. Xử lý C với BCl3 để tách
loại một nhóm metyl tạo ra D là một đồng phân mới của A. D có một nhóm hiđroxi tạo liên kết
hiđro nội phân tử.
Cho chất E (2-metyl-1,3-đihiđroxibenzen) phản ứng MeI/K2CO3 tạo F[C9H12O2], F được
khử bằng Li/NH3 lỏng có mặt 2 -metylpropan-2-ol cho một đien đối xứng và không liên hợp G.
Tiếp tục cho G phản ứng KNH2/NH3 lỏng tạo ra một sản phẩm H. Thực hiện phản ứng ozon
phân H rồi xử lý tiếp thu được nhiều sản phẩm trong đó có metyl 2-oxopropanoat. Thực hiện
phản ứng giữa H với đimetylbut-2-inđioat, đun nóng tạo K[C15H20O6], tiếp tục đun nóng K để
loại eten tạo được một este thơm L. Thủy phân L trong môi trường bazo rồi axit hóa dung dịch
tạo thành M[C11H12O6], đun nóng M trong chân không tạo ra N[C11H10O5]. Khử N bằng
NaBH4 trong DMF(N,N-đimetylformamit) tạo C và một lacton P đồng phân, P cũng có thể thu
được nhờ metyl hóa B. Hãy biện luận xác định cấu tạo các chất từ A đến P.
.
Câu Hướng dẫn chấm Điểm
7
1 Từ sản phẩm axit ađipic HOOC-(CH2)4-COOH, butanđiol-1,3 và axit axetic; theo
dữ kiện C1 phản ứng với PCC tạo thành xeton, suy ra đầu ancol bậc 2 của
butanđiol-1,3 có sẵn từ chất đầu C1. Vì vậy, cấu tạo của C4 hoàn toàn xác định.
Từ đó xác định được cấu tạo các chất C1, C2, C3.
O

O O O O
O O HO
COOH
1,0
COOH O CHO
O O
C4 C3 C2 C1

1,5

C loại một nhóm metyl tạo D, D có một nhóm hidroxi tạo liên kết hidro nội nên C có
nhóm cacbonyl cạnh nhóm -OMe. Do đó C là (II), P là (I), từ đó xác định cấu tạo B, D

Metyl hóa B được P nên có 2 cấu tạo B thỏa mãn là B1,B2 (0,125.2)

A là đồng phân của D nên A khác D ở vị trí nhóm hiđroxi và metoxi, cấu tạo phù hợp A(0,5):
Câu 8 (2,5 điểm): Hóa học các hợp chất thiên nhiên (Cacbohiđrat và các hợp chất hữu cơ
chứa nitơ đơn giản).
1. Trong mật mía có một chất đường không có tính khử là rafinozơ có công thức phân tử
C18H32O16. Khi thủy phân nó bằng axit thu được các sản phẩm D-fructozơ, D-galactozơ và
D-glucozơ. Nếu thủy phân bằng bằng men  - galactozida lại cho  -D-galactozơ và
saccarozơ. Nếu thủy phân bằng men invecta lại cho D-fructozơ và một đi saccarit.
Metyl hóa rafinozơ, sau đó đem thủy phân thu được các sản phẩm: 1,3,4,6-tetra-O- metyl-
D-fructozơ, 2,3,4,6-tetra-O-metyl-D-galactozơ và 2,3,4-tri-O-metyl-D-glucozơ
Viết công thức cấu tạo của rafinozơ.
2. Meloxicam là thuốc chữa bệnh viêm và thoái hóa khớp. Sơ đồ tổng hợp
meloxicam như sau:
O O
COOMe H
NH3 ClCH2 COOMe MeONa (CH3 )2 SO4 M5 C N
M1 M2 M3 N
M4
SO2 OH to (C H NO S) NaOH MeOH N
7 5 3 S S
O O Meloxicam
Xác định cấu tạo các chất từ M1 đến M5.

Câ Hướng dẫn chấm Điể


u8 m
1 Vì rafinozơ là đường không có tính khử nên không còn nhóm OH somiaxetat tự
do.
- Thủy phân bằng axit rafinozơ cho D- flucozơ, D-galactozơ và D-glucozơ
nên nó là đường ba và được tạo ra từ các mono saccarit trên
- Khi thủy phân bằng men cho  -D-galactozơ và saccarozơ nên  -D-
galactozơ ở đầu mạch
- Mặt khác khi thủy phân đường ba này lại cho D-fructozơ và một đi 1,25
saccarit, vì vậy D-fructozơ phải đứng ở một đầu mạch  D-glucozơ đứng
giữa mạch
Từ các sản phẩm sau khi sau khi metyl hóa rafinozơ suy ra:
+ Phân tử  -D –fructozơ có nhóm OH ở cacbon số 2 tham gia tạo liên kết
glucozit
+ Phân tử  -D-galactozơ có nhóm OH ở cacbon số 1 tham gia tạo liên kết
glucozit
+ Phân tử  - D-glucozơ có cacbon số 1 và số 6 tham gia liên kết glucozit
Từ trên suy ra công thức cấu tạo của rafinozơ như sau:
CH2OH

HO OH
H
1 6
OH H
H O CH2

H O H 1
H OH H H
OHCH2 O
OH H 2
OH O H HO

H OH
OH CH2OH
2 Xác định cấu tạo các chất M1, M2, M3, M4, M5. (Mỗi chất 0,25 điểm) 1,25
O O O O O
O O
S S COOMe COOMe COOMe N
NH N CH2COOMe + S
NH N N H2N
S S S
M1 O M2 O M3 O O M4 O O M4 O O M5
SỞ GIÁO DỤC VÀ ĐÀO TẠO KỲ THI HỌC SINH GIỎI CÁC TRƯỜNG THPT CHUYÊN
TỈNH BÌNH DƯƠNG KHU VỰC DUYÊN HẢI VÀ ĐỒNG BẰNG BẮC BỘ
TRƯỜNG THPT CHUYÊN LẦN THỨ X, NĂM 2023
HÙNG VƯƠNG
ĐỀ THI MÔN: HÓA HỌC - KHỐI 11
Thời gian: 180 phút.
ĐỀ THI ĐỀ XUẤT Đề thi gồm có: 08 câu, 09 trang.

HƯỚNG DẪN CHẤM


Câu 1: (2.5 điểm) Tốc độ phản ứng.
1.1 Phản ứng sau đây có thể được sử dụng để phân tích nồng độ ion iodide:
𝐼𝑂3− (𝑎𝑞) + 5𝐼 − (𝑎𝑞) ⟶ 3𝐼2 (𝑎𝑞) + 3𝐻2 𝑂(𝑙)
Khảo sát tốc độ phản ứng trên ở 25 oC và nhận được các kết quả sau đây:
Thí nghiệm [𝐼 − ], M [𝐼𝑂3− ], M [𝐻+ ], M 𝑣, mol. L−1 . s −1
1 0,010 0,100 0,010 0,60
2 0,040 0,100 0,010 2,40
3 0,010 0,300 0,010 5,40
4 0,010 0,10 0,020 2,40
a) Viết biểu thức tốc độ phản ứng và tính hằng số tốc độ 𝑘.
b) Năng lượng hoạt hóa của phản ứng trên là 84 kJ/mol ở 25 oC. Hằng số tốc độ phản ứng
tăng bao nhiêu khi năng lượng hoạt hóa giảm đi 10 kJ/mol.
1.2 Trong một số trường hợp, trạng thái dừng có thể được giả định như là một phép xấp xỉ
trong nghiên cứu động học. Điều kiện tiên quyết là trong hầu hết thời gian phản ứng, nồng độ
của tất cả các tiểu phân trung gian không thay đổi.
Phương pháp này có thể được sử dụng đặc biệt cho các phản ứng hóa sinh kiểu như sau:

Trong đó: S là cơ chất (ví dụ Penicillin).


E là enzyme (ví dụ 𝛽 − 𝐿𝑎𝑐𝑡𝑎𝑚𝑎𝑠𝑒).
ES là phức chất cơ chất – enzyme.
P là sản phẩm.
Gọi [𝐸]𝑡 là tổng nồng độ của E ([𝐸] + [𝐸𝑆]). Giả sử cân bằng được thiết lập rất nhanh, phản
ứng ngược của P thành ES có thể được bỏ qua, và nồng độ của S lớn hơn E rất nhiều.
Trang 1/22
a) Hãy dẫn ra phương trình Michaelis - Menten từ các dữ kiện đã biết:
𝑘2 [𝐸]𝑡 [𝑆]
𝑣=
𝐾𝑀 + [𝑆]
𝑘−1 +𝑘2
với hằng số Michaelis 𝐾𝑀 =
𝑘1

b) Thông thường, mối liên hệ giữa 𝑣 và [𝑆] được rút ra từ biểu thức Lineweaver – Burk:
1 1
= 𝑓( )
𝑣 [𝑆]
Tìm biểu thức Lineweaver – Burk.
c) Trong một thí nghiệm với nồng độ enzyme [𝐸]𝑡 = 1. 10−9 M, tốc độ đầu được xác
định như là một hàm số của nồng độ đồ S:
[𝑆]𝑜 . 10−6 (M) 3,0 5,0 10 20
𝑣𝑜 . 10−5 (M. min−1 ) 1,06 1,55 2,37 3,21
Tính hằng số Michael 𝐾𝑀 và hằng số tốc độ 𝑘𝑏 từ biểu thức Lineweaver – Burk.
d) Trong một phản ứng enzyme khác, xác định được hằng số Michaelis 𝐾𝑀 = 1,5. 10−5 M.
Tính tỉ lệ số phân tử enzyme đã liên kết với cơ chất, biết rằng nồng độ của cơ chất là
1,5. 10−5 M.

Ý Nội dung Điểm


1.1 a)
+ Ở thí nghiệm 1 và 2: Khi nồng độ ion 𝐼𝑂3− và 𝐻+ được giữ không đổi, nồng 0,1
độ ion 𝐼 − tăng 4 lần thì tốc độ phản ứng tăng 4 lần.
Như vậy phản ứng bậc 1 theo ion 𝐼 − .
+ Ở thí nghiệm 1 và 3: Khi nồng độ ion 𝐼 − và 𝐻+ được giữ không đổi, nồng độ
ion 𝐼𝑂3− tăng 3 lần thì tốc độ phản ứng tăng 9 lần. 0,1
Như vậy phản ứng bậc 2 theo ion 𝐼𝑂3− .
+ Ở thí nghiệm 1 và 4: Khi nồng độ ion 𝐼 − và 𝐼𝑂3− được giữ không đổi, nồng
độ ion 𝐻+ tăng 2 lần thì tốc độ phản ứng tăng 4 lần. 0,1

Như vậy phản ứng bậc 2 theo ion 𝐻 + .


Biểu thức tốc độ phản ứng: 𝑣 = 𝑘. [𝐼 − ]. [𝐼𝑂3− ]2 . [𝐻 + ]2
=> 𝑘. 0,01. 0,12 0,012 = 0,60 0,1

Trang 2/22
=> 𝑘 = 6. 107 (mol-4.L4.s-1) 0,1
b)
−𝐸𝐴1
Trước: 𝑘1 = 𝐴. 𝑒 𝑅𝑇

−𝐸𝐴2
Sau: 𝑘2 = 𝐴. 𝑒 𝑅𝑇

𝑘2 𝐸𝐴1 − 𝐸𝐴2 84000−74000


=> 𝑙𝑛 = = = 4,03 0,2
𝑘1 𝑅𝑇 8,314.298
𝑘2 0,1
Vậy = 56,6
𝑘1

1.2 a) Áp dụng nguyên lý nồng độ dừng:


𝑑[𝐸𝑆] 0,2
= 0 = 𝑘1 [𝐸][𝑆] − 𝑘−1 [𝐸𝑆] − 𝑘2 [𝐸𝑆]
𝑑𝑡
⇒ 𝑘1 ([𝐸]𝑡 − [𝐸𝑆]). [𝑆] = (𝑘−1 +𝑘2 )[𝐸𝑆]
⇒ 𝑘1 [𝐸]𝑡 [𝑆] = (𝑘−1 +𝑘2 )[𝐸𝑆]
⇒ 𝑘1 [𝐸]𝑡 [𝑆] = (𝑘1 [𝑆] + 𝑘−1 +𝑘2 )[𝐸𝑆]
𝑘1 [𝐸]𝑡 [𝑆]
⇒ [𝐸𝑆] =
𝑘1 [𝑆] + 𝑘−1 +𝑘2
[𝐸]𝑡 [𝑆]
⇒ [𝐸𝑆] =
𝑘−1 +𝑘2
+ [𝑆]
𝑘1
[𝐸]𝑡 [𝑆] 0,5
⇒ [𝐸𝑆] =
𝐾𝑀 + [𝑆]
𝑘−1 +𝑘2
với hằng số Michaelis 𝐾𝑀 =
𝑘1

𝑑[𝑃] 𝑘2 [𝐸]𝑡 [𝑆]


𝑣= = 𝑘2 [𝐸𝑆] =
𝑑𝑡 𝐾𝑀 + [𝑆] 0,1
b)
1 𝐾𝑀 + [𝑆] 𝐾𝑀 1 1
= = . +
𝑣 𝑘2 [𝐸]𝑡 [𝑆] 𝑘2 . [𝐸]𝑡 [𝑆] 𝑘2 . [𝐸]𝑡 0,2
c)
[𝑆]𝑜 . 10−6 (M) 3,0 5,0 10 20
𝑣𝑜 . 10−5 (M. min−1 ) 1,06 1,55 2,37 3,21
1 1
Tính hồi quy tuyến tính 𝑦 = 𝑎𝑥 + 𝑏 với 𝑥 = ; 𝑦= ta được:
[𝑆]𝑜 𝑣𝑜

Trang 3/22
1 1
= 1,993. 104 ⇒ = 1,993. 104 ⇒ 𝑘2 = 5,02. 104 min−1
𝑘2 [𝐸]𝑡 𝑘2 . 1. 10−9 0,2
𝐾𝑀 𝐾𝑀
= 0,223 => = 0,223 => 𝐾𝑀 = 1,12. 10−5
𝑘2 . [𝐸]𝑡 5,02. 104 . 1. 10−9
0,2
d) Tỉ lệ số phân tử enzyme đã liên kết với cơ chất:
[𝐸𝑆] [𝑆] 3. 10−5
= = = 0,67
[𝐸]𝑡 𝐾𝑀 + [𝑆] 1,5. 10−5 + 3. 10−5
0,3
Câu 2: (2.5 điểm) Cân bằng và phản ứng trong dung dịch, Pin điện – Điện phân.
2.1 Các đồng xu cổ được làm từ vàng Nordic, một hợp kim của đồng, kẽm, nhôm và thiếc.
Hàm lượng (phần trăm theo khối lượng) của hợp kim này có thể được xác định như sau:
Hòa tan đồng xu nặng 4,100 g vào dung dịch hydrochloric acid loãng. Quá trình này tạo ra
363,9 mL hydrogen (áp suất 1,013 bar, 298 K). Bả rắn không tan (chỉ gồm một kim loại màu
đỏ) được hòa tan hoàn toàn vào nitric acid đặc và được đem điện phân (sau quá trình tiền xử
lý phù hợp). Để kết tủa hoàn toàn trên cathode, cần tiến hành điện phân trong 16,71 phút với
dòng điện 13 A, hiệu suất dòng 85%.
Thiếc hòa tan trong hydrochloric acid có thể được kết tủa bởi N – benzoyl – N – phenylhydr-
oxylamine (C6H5CON(OH)C6H5) ở dạng phức chất (C13H11O2N)2SnCl2 (M = 616,11 g/mol).
Lọc kết tủa thu được 0,213 g phức chất.
a) Viết các phản ứng lần lượt diễn ra trong quá trình hòa tan các phần của đồng xu trong
hydrochloric acid.
b) Chỉ ra kim loại nào bị điện phân và tính hàm lượng của nó trong hợp kim.
c) Tính hàm lượng của thiếc và hai kim loại còn lại trong đồng xu.
d) Một đồng xu làm từ vàng thật không tan cả trong hydrochloric acid lẫn nitric acid. Một
giản đồ Latimer chưa hoàn chỉnh của vàng ở pH = 0 được đưa ra dưới đây:

Tính giá trị thế x còn thiếu.


e) Vàng không hòa tan được trong nitric acid, thay vào đó là trong nước cường toan (một
hỗn hợp theo tỉ lệ 3:1 của hydrochloric acid đặc và nitric acid đặc) được các nhà giả kim thuật
phát minh để hòa tan vàng. Phản ứng với nước cường toan tạo thành phức chất [𝐴𝑢𝐶𝑙4 ]− và
Trang 4/22
khí có màu nâu đỏ. Viết phương trình phản ứng hòa tan và tính hằng số tạo phức của [𝐴𝑢𝐶𝑙4 ]−
𝑜 𝑜
ở 298 K, biết 𝐸𝐴𝑢 3+ /𝐴𝑢 = 1,517 𝑉, 𝐸[𝐴𝑢𝐶𝑙 ]− /𝐴𝑢 = 1,002 𝑉.
4

2.2 Chứng minh rằng không thể hoà tan 0,1 mol CuS bằng 1,00 L dung dịch HCl 1,000 M
nhưng có thể hoà tan nó bằng 1,00 L dung dịch HNO 3 1,000 M. Cho rằng ion Cu2+ không tạo
phức bền với ion Cl- trong điều kiện trên, quá trình hoà tan này diễn ra ở 25 °C và không làm
thay đổi thể tích dung dịch.
Cho biết: pKa (HSO4-) = 1,99; *𝛽𝐹𝑒(𝑂𝐻)2+ = 10−2,17 ; pKs (Fe(OH)3) = 37,00; pKai (H2S) =
𝑜 𝑜
7,02; 12,9; *𝛽𝐶𝑢(𝑂𝐻)+ = 10−8 ; pKs (CuS) = 35,2; 𝐸𝑆/𝑆 2− = −0,48 𝑉; 𝐸𝑁𝑂 − /𝑁𝑂
3 (𝑎𝑞)
=

0,96 𝑉.
Độ tan của NO trong nước ở 25 °C là 2,530.10-2 M.
Độ tan của H2S trong nước ở 25 °C là 0,100 M.

Ý Nội dung Điểm


2.1 a)
2𝐴𝑙 + 6𝐻𝐶𝑙 ⟶ 2𝐴𝑙𝐶𝑙3 + 3𝐻2
𝑍𝑛 + 2𝐻𝐶𝑙 ⟶ 𝑍𝑛𝐶𝑙2 + 𝐻2 0,1x3
𝑆𝑛 + 2𝐻𝐶𝑙 ⟶ 𝑆𝑛𝐶𝑙2 + 𝐻2
b) Trong các kim loại, chỉ có đồng không phản ứng với hydrochloric acid. Nên 0,1
đồng là kim loại bị điện phân: 𝐶𝑢 ⟶ 𝐶𝑢2+ + 2𝑒 −
𝐼. 𝑡 13.16,71.60
𝑛𝐶𝑢 = = = 0,0675 𝑚𝑜𝑙
𝑛. 𝐹 2.96485
Do hiệu suất dòng là 85% nên ta có khối lượng Cu thực tế bị điện phân là: 0,2
𝑚𝐶𝑢 = 0,0675.63,55.0,85 = 3,646 𝑔
3,646
%𝐶𝑢 = . 100% = 88,93%
4,100
𝑚 0,213
c) 𝑛𝑝ℎứ𝑐 = = = 3,457. 10−4 𝑚𝑜𝑙 = 𝑛𝑆𝑛
𝑀 616,11
0,1
𝑚𝑆𝑛 = 𝑛𝑆𝑛 . 𝑀𝑆𝑛 = 3,457. 10−4 . 118,71 = 0,041 𝑔
0,041
%Sn = . 100% = 1,00%
4,100 0,2
𝑝.𝑉 1,013.105 .363,9.10−6 0,1
𝑛𝐻2 = = = 0,0149 𝑚𝑜𝑙
𝑅.𝑇 8,314.298

Gọi 𝑥, 𝑦 lần lượt là số mol của Zn và Al trong đồng xu

Trang 5/22
⇒ 65,41𝑥 + 26,98𝑦 = 4,100 − 3,646 − 0,041 = 0,413 (1)
3 3
𝑛𝐻2 = 𝑥 + 𝑦 + 3,457. 10−4 = 0,0149 => 𝑥 + 𝑦 = 0,01455 (2)
2 2
Từ (1) và (2) => 𝑥 = 3,190. 10−3 ; 𝑦 = 7,573. 10−3
0,1
3,190.10−3 .65,41
Vậy: %𝑍𝑛 = . 100% = 5,09%;
4,100

7,573. 10−3 . 26,98


%𝐴𝑙 = . 100% = 4,98%
4,100 0,2
3,1,517−1.1,830
d) 𝑥 = = 1,361 𝑉
2

e) 𝐴𝑢 + 3𝐻𝑁𝑂3 + 4𝐻𝐶𝑙 ⟶ 𝐻[𝐴𝑢𝐶𝑙4 ] + 3𝑁𝑂2 + 3𝐻2 𝑂


1,517.3 0,1
3+ −
𝐴𝑢 + 3𝑒 ⟶ 𝐴𝑢 𝐾1 = 10 0,0592

1,002.3
[𝐴𝑢𝐶𝑙4 ]− + 3𝑒 − ⟶ 𝐴𝑢 + 4𝐶𝑙 − 𝐾2 = 10 0,0592
(1,517−1,002).3
𝐴𝑢3+ + 4𝐶𝑙 − ⟶ [𝐴𝑢𝐶𝑙4 ]− 𝛽 = 𝐾1 . 𝐾2 −1 = 10 0,0592 =
0,2
1,25.1026
2.2 Khi hoà tan CuS trong HCl (không có phản ứng oxy hoá – khử hay tạo phức):
0,1
CuS + 2HCl ⇌ CuCl2 + H2S K1 = 10-35,2.107,02.1012,9 = 10-15,28

Giả sử có thể hoà tan được hoàn toàn 0,1 mol CuS bằng 1,00 L dung dịch HCl
1,000 M.
0,1
Sau phản ứng: C(H ) = 0,800 M, C(Cu ) = 0,100 M, C(H2S) = 0,100 M.
+ 2+

H2S ⇌ HS- + H+ Ka1 = 10-7,02


0,1
Cu2+ + H2O ⇌ Cu(OH)+ + H+ *𝛽 = 10−8

HS- ⇌ S2 + H+ Ka2 = 10-12,9

H2O ⇌ H+ + OH- Kw = 10-14

Do C(H+) ≫ √𝐾𝑎1 . 𝐶𝐻2 𝑆 ; √∗ 𝛽. 𝐶𝐶𝑢2+ ; √𝐾𝑎2 . 𝐶𝐻2 𝑆 ; √𝐾𝑤 nên pH quyết định
0,1
bởi [H+] = C(H+) = 0,800 M => pH = 0,10

0,1

Trang 6/22
Môi trường acid (pH = 0,10) có h ≫ *𝛽 => bỏ qua sự tạo thành phức hydroxo

của Cu2+.

[Cu2+] = 0,100 M.

10−7,02 .10−12,9
[S2-] = 𝛼𝑆 2− . 𝐶𝐻2 𝑆 = . 0,1 = 1,879. 10−21 M.
0,82 +10−7,02.0,8+10−7,02 .10−12,9

[Cu2+].[S2-] = 10−21,73 > Ks (CuS) 0,1

Như vậy, có kết tủa xuất hiện trở lại trong dung dịch nên không thể hoà tan 0,1

mol CuS bằng 1,00 L dung dịch HCl 1,000 M.

Khi hoà tan CuS bằng HNO3, có phản ứng oxy hoá - khử xảy ra:

3CuS + 8H+ + 2NO3- ⟶ 3Cu2+ + 3S + 2NO + 4H2O. 0,1

6.(0,96−(−0,48))
K2 = 10-35,2.10 0,0592 = 10110,75 ≫ 1.

Phản ứng xảy ra hoàn toàn, sau phản ứng:

C(Cu2+) = 0,100 M, C(H+) = 0,733 M, C(NO3-) = 0,933, C(NO) = 0,0253 M.

Do C(H+) ≫ √∗ 𝛽. 𝐶𝐶𝑢2+ nên pH quyết định bởi [H+] = C(H+) = 0,733 M. 0,1

pH = 0,13

[NO3-] = 0,933 M, [Cu2+] = 0,100 M, [NO] = 0,0253 M.


6.(0,96−(−0,48))
2- + -
3S + 8H + 2NO3 ⇌ 3S + 2NO + 4H2O K = 10 0,0592 = 10145,95

[𝑁𝑂] 3 0,0253
K = [𝑆 2−]3 => [𝑆 2− ] = √ 145,95 = 1,576.10−49 M.
.[𝐻 + ]8 .[𝑁𝑂3− ]2 10 .0,7338 .0,9332

[Cu2+].[S2-] = 0,1.1,576.10−49 = 1,576.10−50 < Ks (CuS). 0,1

Như vậy, không có kết tủa xuất hiện trở lại trong dung dịch, tức có thể hoà tan
được 0,1 mol CuS bằng 1,00 L dung dịch HCl 1,000 M.
Câu 3: (2.5 điểm) Nhiệt động học và cân bằng hóa học.

Trang 7/22
Enthalpy hóa hơi của một dung môi phụ thuộc vào nồng độ của các chất hòa tan trong đó. Các
phép đo không phải lúc nào cũng dễ dàng. Một phương pháp thú vị là dùng thiết bị bay hơi
cột bong bóng được mô tả như hình bên.
Bơm các bong bóng khí khô có đường kính 1 – 3 mm vào một
dung dịch muối từ bên dưới. Trong vài phần mười giây, bên
trong các bong bóng đã bão hòa hơi nước. Quá trình bay hơi này
loại bỏ nhiệt từ các bong bóng và nước, và nhiệt độ ở ở phần trên
của thiết bị thí nghiệm (𝑇𝑜 ) thấp hơn nhiệt độ phần dưới (𝑇𝑢 ).
Sau một thời gian, đạt được trạng thái bền, tại đó cả hai nhiệt độ
đều không thay đổi. Nhiệt được cung cấp bởi các bong bóng khô
sau đó chỉ dùng để làm bay hơi nước. Dung dịch không còn bị
mất nhiệt nữa. Do đó, có một cân bằng năng lượng:
Năng lượng các bong bóng mất đi = Enthalpy hóa hơi
=> 𝑉. (𝐶𝑝 Δ𝑇 + Δp) = 𝑉. 𝜌ℎơ𝑖 𝑛ướ𝑐 . Δℎℎ 𝐻
Trong đó: Δ𝑇 = 𝑇𝑢 – 𝑇𝑜 là chênh lệch nhiệt độ giữa phần dưới và phần trên,
𝐶𝑝 là nhiệt dung riêng của khí và bằng 1204 J.m-3.K-1,
Δp là chênh lệch áp suất giữa khí vào và khí ra ở áp suất khí quyển,
𝜌ℎơ𝑖 𝑛ướ𝑐 là khối lượng riêng của hơi nước và bằng 1,030 mol.m-3 ở 21,5 ∘ 𝐶.
Chia biểu thức cho V thì được năng lượng trên mỗi đơn vị thể tích.
Cho biết: 𝑇𝑜 = 21,5 ∘ 𝐶, 𝑇𝑢 = 54,0 ∘ 𝐶, áp suất đầu vào (𝑝𝑢 ) là 105453 Pa, áp suất đầu ra (𝑝𝑜 )
là 101325 Pa và tốc độ dòng khí là 0,18 L/s.
a) Hãy tính số lượng bong bóng khí được được bơm vào mỗi giây, giả sử bong bóng có
đường kính trung bình là 2 mm.
b) Tính giá trị Δℎℎ 𝐻 xác định được qua thí nghiệm này.
c) Tính áp suất hơi nước tinh khiết ở 𝑇𝑜 biết rằng enthalpy hóa hơi của nước tinh khiết là
2463,0 kJ/kg.
d) Các nhà nghiên cứu cũng đã xem xét để sử dụng thiết bị như vậy ở quy mô lớn hơn cho
việc làm mát bay hơi, như một loại điều hòa không khí. Tiến hành một thí nghiệm với tốc độ
dòng khí 150 m3/h, tính công suất làm mát của thiết bị.
2.2 Cho cân bằng: PCl5(k) ⇌ PCl3(k) + Cl2(k) KP = 1,85 atm ở 525 K
Thực hiện ba thí nghiệm:
Trang 8/22
+ Thí nghiệm 1: Cho 1 mol PCl5 vào bình chân không có dung tích không đổi. Lúc đạt cân
bằng ở 525 K, áp suất trong bình là 2 atm.
+ Thí nghiệm 2: Làm giống thí nghiệm 1 nhưng cho thêm vào bình 1 mol khí argon và vẫn
duy trì nhiệt độ là 525 K.
+ Thí nghiệm 3: Khi cân bằng ở thí nghiệm 2 được thiết lập nguời ta vẫn duy trì nhiệt độ của
bình là 525 K nhưng tăng dung tích của bình lên sao cho áp suất cân bằng là 2 atm.
Tính số mol PCl5 và Cl2 khi cân bằng trong mỗi thí nghiệm
Ý Nội dung Điểm
3.1 a) Từ hình ta nhận thấy các bong bóng khí có hình cầu với thể tích mỗi bóng
bóng là:
4 4 0,2
𝑉 = 𝜋𝑅3 = . 3,14. 13 = 4,1867 𝑚𝑚3
3 3
Tốc độ dòng khí là 0,18 L/s tức mỗi giây có 0,18 L bong bóng khí được bơm
0,2
vào, do đó số bong bóng được bơm vào mỗi giây là:
0,18. 106
= 42993
4,1867
𝐶𝑝 Δ𝑇+Δp 1204.(327−294,5)+(105453−101325)
b) Δℎℎ 𝐻 = = = 41998,058 J/mol ≈
𝜌ℎơ𝑖 𝑛ướ𝑐 1,030 0,25
42,0 kJ/mol
c) Ở điều kiện áp suất khí quyển (101325 Pa), nước tinh khiết hóa hơi ở 373 K
𝑝1 Δℎℎ 𝐻(𝐻2 𝑂) 1 1 101325 2453.18 1 1
𝑙𝑛 = .( − ) ⇒ 𝑙𝑛 = .( − ) 0,25
𝑝2 𝑅 𝑇2 𝑇1 𝑝2 8,314 294,5 373

=> 𝑝2 =2277,61 Pa
d) Năng lượng tiêu thụ trên mỗi đơn vị thể tích bong bóng mất đi:
𝐸 0,25
= 𝐶𝑝 Δ𝑇 + Δp = 1204. (327 − 294,5) + (105453 − 101325) = 43258
𝑉

J/m3
43258.150
Công suất làm mát: 𝑃 = = 1802,417 W 0,25
3600

3.2 + Thí nghiệm 1:


PCl5(k) ⇌ PCl3(k) + Cl2(k)
Cân bằng: 1–x x x (mol)
 𝑛𝑘ℎí = 1 + 𝑥 0,15

Trang 9/22
𝑥2 2 0,2
Có: 𝐾𝑝 = . = 1,85 => 𝑥 = 𝑛𝐶𝑙2 = 0,693 mol
1−𝑥 1+𝑥

=> 𝑛𝑃𝐶𝑙5 = 0,307 mol


0,4
+ Thí nghiệm 2: Thêm Ar vào ở T, V không đổi nên áp suất riêng phần của
từng chất và hằng số Kp không đổi. Cân bằng không chuyển dịch, kết qủa giống
thí nghiệm 1.
+ Thí nghiệm 3:
PCl5(k) ⇌ PCl3(k) + Cl2(k) + Ar
Cân bằng: 1–x x x 1 (mol)
0,15
 𝑛𝑘ℎí = 2 + 𝑥
𝑥2 2
Có: 𝐾𝑝 = . = 1,85 => 𝑥 = 𝑛𝐶𝑙2 = 0,770 mol 0,2
1−𝑥 2+𝑥

=> 𝑛𝑃𝐶𝑙5 = 0,23 mol


Câu 4: (2.5 điểm) Hóa nguyên tố (Kim loại, phi kim nhóm IVA, VA). Phức chất.
Các chất lưỡng nguyên tố X, Y, W - chứa nguyên tố Z có đơn chất là chất khí nhẹ nhất ở điều
kiện chuẩn. Hàm lượng của nguyên tố nặng hơn trong X, Y, W lần lượt là 82.35%, 78.28% và
91.18%. X, Y, W có công thức thực nghiệm tương tự nhau.
W và X không phản ứng trong điều kiện thường, nhưng khi X phản ứng với chất C thì thu
được muối của B - cũng có cation như trong muối của X1, được tạo thành từ phản ứng của X
với hydrochloric acid. Muối B là sản phẩm cộng của X, W, Y theo tỉ lệ mol 1:1:1. X và Y
phản ứng với nhau tạo thành hợp chất D chứa một liên kết cho-nhận (donor-acceptor).
Chlor hóa Y tạo thành khí Y1 chứa cùng số nguyên tử như W. Phản ứng của Y với sodium
hydride tạo thành muối Y2 chứa cùng anion như muối A. Trong trường hợp này, từ 131 mL
Y (ở 320 K và 1491,06 mm Hg) tạo ra 370 mg muối Y2. Muối này có thể được sử dụng để tạo
thành chất D màu trắng bởi phản ứng với X1 trong ether. Từ 1.31 g Y2 và một lượng đẳng
mol X1 thu được sodium chloride và đơn chất khí của Z nặng 0.069 g và D. Khi đun nóng, D
mất khoảng 20% khối lượng, tạo thành chất lưỡng nguyên tố F.
Phản ứng của X và Y tạo thành hợp chất ion A, chứa một cation phức đơn điện tích, có cấu
trúc tương tự như anion của muối B. A phản ứng với X1 trong ether, tương tự Y2, tạo thành
D và muối E - là chloride của cation chứa trong A. Hàm lượng hydrogen và chlorine trong E
lần lượt là 9.72% và 43.13%.

Trang 10/22
Chất W dễ bị oxi hóa bởi nhiều tác chất khác nhau. Khi W bị chlor hóa và fluor hóa, sẽ lần
lượt tạo thành các chloride W1 và fluoride W2 của cùng nguyên tố ở số oxi hóa cao hơn.
Trong đó, hàm lượng của nguyên tố này trong fluoride cao hơn 1.62 lần so với trong chloride.
W1 kết hợp với Y1 tạo thành muối H, với cation và anion có hình dạng giống nhau.
Một muối kì lạ có thể được tạo thành từ sản phẩm fluor hóa X trên xúc tác đồng, là khí X2
cũng chứa nhiều nguyên tử như trong W. Khi fluor hóa hỗn hợp X2 và W2 theo tỉ lệ 1:1, thu
được muối K chứa 80.85% fluorine. Cation của muối K có 6 mặt phẳng đối xứng và anion có
9 mặt phẳng đối xứng và có tâm đối xứng.
a) Xác định công thức các hợp chất X, Y, W, Z, X1, X2, Y1, Y2, W1, W2, A – K. Cho
biết tỉ khối hơi của X2 với không khí lớn hơn 2.
b) So sánh độ phân cực của các phân tử X, Y, W theo chiều tăng dần độ phân cực.
Ý Nội dung Điểm

a) Ở điều kiện chuẩn, đơn chất Z là chất khí nhẹ nhất là khí hydrogen
=> Z: H
Từ phần trăm nguyên tố nặng hơn H trong X, Y, W ta tìm được:
0,125x18
X: NH3 Y: B2H6 W: PH3
Muối B là sản phẩm cộng của X, W, Y theo tỉ lệ mol 1:1:1
=> B: NH4[PH2(BH3)2]
C: PH3-BH3
Y1: BCl3
1491,06.133,3224.131. 10−6
𝑛𝑌 = = 9,788. 10−3 𝑚𝑜𝑙
8,314.320
0,37
𝑀𝑌2 = = 37,8 𝑔/𝑚𝑜𝑙
9,788. 10−3
=> Y2: NaBH4
X1: NH4Cl
D: NH3-BH3 mất 20% khối lượng => F: BN
7,92 43,13 35,5.100
E có H:Cl = : = 8: 1 => 𝑀𝐸 = = 82,3 g/mol
1 35,5 43,13

=> E: [NH3BH2NH3]Cl và A: [NH3BH2NH3]BH4


W1: PCl5 và W2: PF5
W1 + Y1 => H: [PCl4][BCl4]
Trang 11/22
X2: NF3 + W2 => K: [NF4][PF6] (%F = 80,85%)
b) B2H6 < PH3 < NH3 (độ âm điện B < P < N) 0,25

Câu 5: (2.5 điểm) Đại cương hữu cơ.


5.1 Trong thời gian gần đây, các nhà hóa học hữu cơ ở Trung Quốc đã tiến hành nghiên cứu
và phát triển một loạt các tác nhân hữu cơ mới, bao gồm chuỗi CpFluors, được sử dụng để
fluor hóa các hợp chất hữu cơ. Trong danh sách các tác nhân fluor hóa này, CpFluor-2 đã được
chứng minh phản ứng với các chất alcohol một cách hiệu quả và đạt được hiệu suất cao cùng
độ chọn lọc tốt để tạo ra các sản phẩm fluor hóa:

a) Giải thích ngắn gọn tại sao liên kết C–F trong CpFluor-2 dễ bị phân cắt dị ly, từ đó đề
xuất cơ chế cho phản ứng fluor hóa 3-phenylpropanol trên.
Các nghiên cứu chỉ ra rằng có một sản phẩm phụ quan trọng trong phản ứng trên, có cấu trúc
như sau:

b) Đề xuất cơ chế phản ứng tạo thành sản phẩm phụ này
Lưu ý: Bạn có thể dùng kí hiệu Ar thay cho nhóm thế nhân thơm trong CpFluor-2 để đơn giản
hóa việc vẽ cấu trúc.

Trang 12/22
5.2 Chọn một hay nhiều hơn một trong những cấu hình sau đây để minh họa cho các khái
niệm sau:
a) Đồng phân cấu tạo (structural
isomer)
b) Đồng phân vị trí (positional isomer)
c) Đồng phân lập thể (stereoisomer)
d) Đồng phân hình học
e) Đồng phân quang học có đối quang
h) Hợp chất trùng với ảnh trong gương
(enantiomer)
(achiral compound)
f) Đồng phân quang học không đối
i) Đồng phân meso
quang (diastereomer)
j) Hỗn hợp racemic
g) Hợp chất không trùng với ảnh trong
gương (chiral compound)

Điể
Ý Nội dung
m
5. a) Vì khi phân cắt dị ly liên kết C–F tạo thành cation có tính thơm bền.
1
0,12

0,12

Trang 13/22
b)

0,25

5. a) A là đồng phân cấu tạo của tất cả các cấu hình khác
2 b) Các cặp đồng phân vị trí của nhau có thể là (B và C), (B và D), (C và D), (C
và E)
c) D và E là đồng phân lập thể của nhau
d) Các cặp đồng phân hình học của nhau có thể là (C và F), (D và E)
e) F và G là một cặp đồng phân quang học đối quang
Mỗi
f) Các cặp đồng phân quang học không đối quang có thể là (C và F), (D và E), câu
0,2đ
(C và G)
g) Hợp chất không trùng với ảnh trong gương là F và G
h) Hợp chất trùng với ảnh trong gương là A, B, D và E
i) Đồng phân meso là C
j) 50% của F và G là hỗn hợp racemic

Câu 6: (2.5 điểm) Sơ đồ tổng hợp hữu cơ. Cơ chế phản ứng hữu cơ.
6.1 Anammox là viết tắt của sự oxy hoá ammonium kị khí – ANAerobic AMMonium OXidation.
Mặc dù là một phần quan trọng của chu trình nitrogen, vi khuẩn chịu trách nhiệm cho quá trình
này chỉ mới được xác định gần đây, vào năm 1999. Trong màng tế bào của các vi khuẩn này,
đã tìm thấy một nhóm các lipid bất thường. Các lipid này chứa những vòng cyclobutane dung
hợp. Các phân tử như vậy được gọi là các ladderane do chúng có sự sắp xếp các vòng như cầu
thang (ladder). Để hiểu thêm về các lipid được tìm thấy trong các vi khuẩn này, một nhóm

Trang 14/22
nghiên cứu đã tổng hợp [5]-Ladderanoic acid – một thành phần then chốt của các lipid như
vậy. Quy trình tổng hợp được biểu diễn như sau:

Xác định công thức cấu tạo các chất từ A – K và [5]-Ladderanoic acid.
6.2 Xác định công thức cấu tạo của A, B, C trong chuỗi chuyển hóa sau:

Hãy dùng dấu * để ký hiệu cho 13C.


6.3 Swainsonine là một hợp chất alkaloid thuộc tự nhiên được tìm thấy trong một số loài thực
vật, đặc biệt là cây locoweeds (chi Astragalus). Nó được cô lập lần đầu từ cây Swainsona
canescens, nơi mà tên gọi của nó xuất phát. Swainsonine có tiềm năng trong lĩnh vực nghiên
cứu y học. Đặc tính ức chế enzyme alpha-mannosidase của nó đã làm nổi bật khả năng sử
dụng nó như một chất chống ung thư tiềm năng. Nghiên cứu cho thấy rằng swainsonine có thể
can thiệp vào quá trình xử lý glycoprotein trong tế bào ung thư, gây ra sự gián đoạn trong sự
phát triển và di căn của khối u.

Trang 15/22
Xác định công thức cấu trúc của A, B, [C1], [C2] và C trong chuỗi tổng hợp swainsonine trên.
Biết rằng ở giai đoạn đun hồi lưu, phản ứng đi qua các trung gian [C1], [C2] để hình thành
sản phẩm cuối là C.

*Mỗi chất đúng được 0,125đ

Ý Nội dung
6.1

Trang 16/22
6.2

6.3

Câu 7: (2.5 điểm) Xác định cấu trúc các chất hữu cơ (mô tả sơ đồ tổng hợp bằng lời dẫn).
7.1 Chất A được sử dụng trong bảo quản thực phẩm có mùi hăng. Trong môi trường acid, A
phản ứng với chất B (1) tạo thành nước và hợp chất C, có mùi thơm ngọt ngào dễ chịu. Hợp
chất B được tạo thành bởi sự lên men đường glucose (2) và A được tạo thành từ B khi có mặt
oxygen (3). Chất A tạo thành một nhóm các hợp chất hữu cơ, gồm cả B, các chất có mùi vị
trái cây chín mọng dễ chịu. Các đồng đẳng lớn hơn của B là D, E, F với mạch carbon thẳng,
lần lượt phản ứng với A tạo thành các chất G (mùi lê), H (mùi táo) và I (mùi cam). A phản
ứng với J (chất này được tạo thành bởi phản ứng khử benzoic acid) tạo thành K, được sử dụng
rộng rãi để làm mỹ phẩm và nước hoa nhờ mùi hương hoa nhài dễ chịu của nó.
Trang 17/22
a) Viết các phương trình phản ứng (1), (2), (3).
b) Vẽ cấu tạo các chất A – K, biết rằng B, D, E là các đồng đẳng liên tiếp và F chứa số
carbon gấp đôi E.
7.2. 2,4,5-trichloriphenoxyacetic acid (2,4,5-T) và 2,4-dichlorophenoxyacetic acid (2,4-D) là
hai chất diệt cỏ chính có trong chất độc màu da cam (Agent Orange) được quân đội Mỹ sử
dụng trong chiến tranh Việt Nam. Để tổng hợp chất diệt cỏ 2,4,5-T ta đun nóng A (C6H2Cl4)
với dung dịch NaOH đặc ở nhiệt độ và áp suất cao tạo thành B, sau đó cho B phản ứng với
chloroacetic acid trong môi trường kiềm NaOH ở 140 oC thu được C, acid hóa C ta được 2,4,5-
T.
a) Xác định công thức cấu tạo của A, B, C và 2,4,5-T.
b) Vì nhiệt độ của quy trình trên không được kiểm soát nghiêm ngặt, một sản phẩm phụ
không mong muốn là TCDD (C12H4Cl4O2) đã được tạo thành. Mặc dù chỉ xuất hiện trong
chất diệt cỏ với hàm lượng khoảng 13 ppm nhưng TCDD có độc tính rất cao, gây ra thảm
họa nhân đạo lâu dài cho nhiều thế hệ của người Việt Nam và cả quân đội Mỹ tham chiến bị
phơi nhiễm và sử dụng nguồn nước, thực phẩm có chứa chất độc màu da cam. Biết rằng
TCDD có 2 mặt phẳng đối xứng, hãy xác định công thức cấu trúc của sản phẩm phụ này.
Ý Nội dung Điểm
7. a)
1

0,25x3

b)

0,1
x11

Trang 18/22
7. a)
2
0,125x
4
b)

0,15

Câu 8: (2.5 điểm) Hóa học các hợp chất thiên nhiên (Carbohydrate và các hợp chất hữu
cơ chứa nitogen đơn giản).
8.1 Amino acid tự nhiên X là nguồn chính của nhóm amino để tổng hợp các amino acid khác
trong các cơ thể sống. Amino acid này được tạo thành, cùng với các chất khác, bởi phản ứng
của hợp chất A với ion 𝑁𝐻4+ , sau đó khử hóa xúc tác bởi một enzyme phù hợp (gọi là
transaminase). Trong phản ứng này, tác nhân khử là phân tử NAPDH, bị oxy hóa thành NADP+
(các phân tử kiểu này liên kết với enzyme và cần thiết cho hoạt tính của nó, chúng được gọi là
các coenzyme).

Trang 19/22
Hợp chất A là một trung gian trong chu trình citric acid (còn gọi là chu trình Krebs), ở dạng
phi-ion, được tạo thành bởi carbon (41,11% khối lượng), oxygen (54,75% khối lượng) và
hydrogen. Nó là đồng đẳng của một tiền chất amino acid quan trọng khác – hợp chất B3, cũng
tham gia vào chu trình Krebs, trong đó nó được tạo thành qua ba giai đoạn (dehydrogen hóa,
cộng nước, oxy hóa) từ succinic acid (butanedioic acid). Hợp chất B3 có thể được tạo thành
trong phòng thí nghiệm hóa học, ví dụ, bởi sự cộng nước acetylenedicarboxylic acid với xúc
tác phù hợp. Hợp chất B1 làm mất màu dung dịch nước bromine.

Hợp chất B3 phản ứng với amino acid X khi có mặt enzyme phù hợp, tạo thành hợp chất A và
amino acid Y.

Trong một phản ứng tương tự, hợp chất C tạo thành amino acid Z với thành phần nguyên tố
như sau (theo % khối lượng): 40,44% C; 35,91 % O; 15,72% N; 7,92 % H.

a) Xác định công thức các hợp chất A, B1, B2, B3, C, X, Y và Z (không cần xét đến cấu
hình tuyệt đối của các nguyên tử bất đối).
b) Biểu diễn cấu trúc của các đối quang D và L của amino acid X, Y, Z ở dạng công thức
chiếu Fischer.
c) Cho biết cơ chế khử hóa của NAPDH thành NAPD+, có thể dùng ký hiệu để đơn giản
hóa việc vẽ cấu trúc?
8.2 Amino acid là những hợp chất hữu cơ tạo thành các thành phần cơ bản của protein. Chúng
bao gồm một nhóm amino (-NH2), một nhóm carboxyl (-COOH) và một chuỗi phụ (còn được

Trang 20/22
gọi là nhóm R) gắn vào một nguyên tử carbon trung tâm. Có 20 amino acid chuẩn thường gặp
trong protein, mỗi amino acid đều có một chuỗi phụ độc đáo mang lại các đặc tính riêng.
a) Các amino acid sau có cấu hình R hay S?

b) Hpg, Dpg, và β-OH-Cl-Tyr là các amino acid phi-tiêu chuẩn. Hpg có cấu trúc tương tự
như Tyr, nhưng M(Tyr) - M(Hpg) = 14 g/mol. Dpg có cấu trúc tương tự Dpg với khối lượng
lớn hơn 16 g/mol. Tất cả các nhóm thế của Dpg trên vòng phenyl đều có khoảng cách với nhau
bằng nhau. Cấu trúc của β-OH-Cl-Tyr dựa trên cấu trúc Tyr, với nhóm hydroxyl gắn vào
nguyên tử β-carbon và vòng phenyl bị chlorine hóa ở vị trí ortho tương ứng với nhóm OH.
Xác định công thức cấu tạo của Hpg, Dpg và β-OH-Cl-Tyr.
Điể
Ý Nội dung
m
8. a) B1 làm mất màu dung dịch nước bromine nên B1 có nối đôi C=C. 0,12
1 5
x8

41,11 100−41,11−54,75 54,75


A có C:H:O = : : = 5: 6: 5 => A: (𝐶5 𝐻6 𝑂5 )𝑛
12 1 16

Mà A là đồng đẳng của một tiền chất amino acid quan trọng B3 và phản ứng A
với ion 𝑁𝐻4+ , sau đó khử hóa tạo thành amino acid X nên A: 𝐶5 𝐻6 𝑂5

40,44 7,92 15,72 35,91


Z có C:H:N:O = : : : = 3: 7: 1: 2 => Z: (𝐶3 𝐻7 𝑁𝑂2 )𝑛
12 1 14 16

Do Z là amino acid nên Z: 𝐶3 𝐻7 𝑁𝑂2

Trang 21/22
Từ đó có thể suy ra được C là:

b)

0,25

c)

0,25

8. a) Asn: R; Leu: R; Thr: 2S, 3R và Tyr: S 0,4


2 b)

Trang 22/22
0,2x
3

Chú ý:
- Học sinh giải theo các khác, đúng vẫn cho đủ điểm.
- Trong các tính toán, chấp nhận sai số không quá 1%.

Trang 23/22
THPT CHUYÊN KỲ THI HỌC SINH GIỎI CÁC TRƯỜNG THPT CHUYÊN
NGUYỄN TẤT THÀNH KHU VỰC DUYÊN HẢI VÀ ĐỒNG BẰNG BẮC BỘ
YÊN BÁI LẦN THỨ XIV, NĂM 2023

ĐÁP ÁN GIỚI THIỆU ĐỀ THI MÔN: HÓA HỌC-KHỐI 11


Thời gian: 180 phút

Câu 1. (2,5 điểm) Tốc độ phản ứng


Nhiên liệu hóa thạch chứa các hàm lượng lưu huỳnh khác nhau (0,05–6,0% đối với dầu thô,
0,5–3% đối với than và khoảng 10 ppm đối với khí tự nhiên, theo khối lượng), phải được loại bỏ
trước khi xử lý tiếp. Mặc dù đã loại bỏ lưu huỳnh khỏi nhiên liệu hóa thạch, nhưng lượng lưu huỳnh
còn lại (tối đa là 10 ppm lưu huỳnh trong xăng và dầu diesel theo tiêu chuẩn) vẫn gây ra vấn đề vì
sản phẩm đốt cháy của nó SO2 là chất gây ô nhiễm không khí chính.
Quá trình oxy hóa SO2 trong môi trường là một chủ đề được quan tâm vì nó là thủ phạm chính
gây mưa acid và acid hóa đại dương. Nghiên cứu đã chỉ ra rằng quá trình oxy hóa SO2 trong pha nước
là một con đường quan trọng để chuyển hóa hoàn toàn SO2.
Một nghiên cứu cơ chế đã được tiến hành đối với quá trình oxy hóa SO2 trong nước biển.
Đối với phản ứng 2 SO32− + O2 → 2SO42− phương trình tốc độ tổng thể có thể được viết là:
d  SO32− 
−  = k  SO32−  O2 
a b

dt
Với lượng oxy dư, phương trình tốc độ có thể được viết lại như sau:
d  SO32− 
= k '  SO32−  với k ' = k O2 
a

b

dt
Nồng độ sulfite được đo theo thời gian (t) và ba biểu đồ đã được vẽ, cụ thể là
1
 SO32−  − t ; − t ;ln  SO32−  − t
 SO32− 

1. Cho biết bậc phản ứng đối với ion sulfite là bao nhiêu?
2. Biết hằng số k ′ được đo ở các nồng độ oxy khác nhau, tóm tắt trong bảng bên dưới (không
hiển thị đơn vị):
[O2] 212,0 390,7 652,2 979,2
k’ 741,3 955,0 1230,3 1584,9
Tính bậc phản ứng (b) đối với O2.
3. Một cơ chế phản ứng gốc tự do dây chuyền đã được đề xuất về quá trình oxy hóa:
Khơi mào Fe3+ + SO32− ⎯⎯k1
→ Fe2+ + SO3•−
SO3•− + O2 ⎯⎯
k2
→ SO5•−
Phát triển mạch SO5•− + SO32− ⎯⎯
k3
→ SO52− + SO3•−
SO52− + SO32− ⎯⎯
k4
→ 2SO42−
Tắt mạch 1: 2 SO3•− ⎯⎯
k5
→ S 2O62−
Tắt mạch 2: SO3•− + SO5•− ⎯⎯
k6
→ S2O62− + O2
Bằng cách xấp xỉ trạng thái bền, hãy suy ra định luật tốc độ cho quá trình oxy hóa SO32- và
bậc phản ứng đối với O2 và SO32-với giả thiết rằng tắt mạch 1 là con đường chính để tắt mạch.
Câu 1 Lời giải Điểm
1 (
1) Trong số ba đường cong, chỉ có đường cong thứ 3 ln  SO32−  − t là tuyến tính. 0,5 )
Điều này có nghĩa là phản ứng là bậc nhất đối với SO32− .
Chú ý: Nếu đường cong  SO32−  − t là tuyến tính, thì phản ứng là bậc 0 đối với
1
SO32− . Nếu đường cong − t là tuyến tính, thì phản ứng là bậc hai đối với
 SO32− 

SO32−
2 Khi k’ = k[O2]b, lấy logarit tự nhiên hai vế sẽ cho: lnk′= lnk + bln[O2] 1,0
Bây giờ, có thể thêm hai hàng nữa vào bảng:
[O2] 212,0 390,7 652,2 979,2
k’ 741,3 955,0 1230,3 1584,9
ln[O2] 5,357 5,968 6,480 6,887
lnk’ 6,608 6,862 7,115 7,368
Tiến hành hồi quy tuyến tính, ta được b = 0,5
3 Phản ứng chung là: 2 SO32− + O2 → 2SO42− 1,0
Để xác định các chất trung gian trong cơ chế phản ứng, trước tiên chúng ta phải
xác định các chất phản ứng và sản phẩm.
Chất phản ứng là SO32 − ; O2
Chất sản phẩm: SO42− và sản phẩm phụ: S2O62−
Chất trung gian: SO3•− ; SO5•− ; SO52−
Áp dụng nguyên lý nồng độ ổn định cho các tiểu phân trung gian, ta được:
d  SO3•− 
= k1  Fe3+   SO32−  − k2  SO3•−  O2  + k3  SO5•−   SO32−  − 2k5  SO3•−   0
2

dt
(1)
d  SO5•− 
= k2  SO3•−  O2  − k3  SO5•−   SO32−   0 (2)
dt
d  SO52− 
= k3  SO5•−   SO32−  − k4  SO52−   SO32−   0 (3)
dt
Lấy (1) + (2) ta được:
2 k1  Fe3+   SO32− 
k1  Fe3+   SO32−  = 2k5  SO3•−  →  SO3•−  = (4)
2 k5

k1  Fe3+   SO32− 
k2 O2 
2k5 k2 O2  k1  Fe3+ 
Thay (4) vào (2):  SO  = •−
= (5)
k3  SO32−  2k5  SO32− 
5
k3

k O  k1  Fe 
3+
k
Thay (5) vào (3)  SO52−  = 3  SO5•−  = 2 2 (6)
k4 k4 2k5  SO32− 

d  SO42−  k4
Từ đó ta có: =  SO52−   SO32− 
dt 2

k4 k2 O2  k1  Fe 
3+

=  SO32− 
2 k4 2k5  SO3 
2−

k2 O2  k1  Fe   SO3 
3+ 2−

= (7)
2 2 k5
Phương trình (7) là định luật tốc độ của phản ứng tổng thể.
Định luật tốc độ này cho thấy rằng phản ứng là bậc nhất trong O2 và bậc ½ trong
 SO32− 

Câu 2. (2,5 điểm) Cân bằng và phản ứng trong dung dịch, pin điện, điện phân. Phức chất
Trộn 10,0 mL dung dịch CH3COOH 0,080M với 10,0 mL dung dịch Na3PO4 0,020M được
dung dịch A. Dung dịch B là (NH4)2S 0,050M.
a) Xác định pH của các dung dịch A và B.
b) Thiết lập pin điện được ghép bởi điện cực hiđro nhúng vào dung dịch A và điện cực hiđro nhúng
vào dung dịch B. Tính Epin.
c) Viết các quá trình xảy ra trên từng điện cực và phản ứng xảy ra trong pin khi pin hoạt động.
d) Chuẩn độ dung dịch A bằng dung dịch NaOH 0,03M bằng chỉ thị phenolphtalein (pH = 9). Tính
sai số của phép chuẩn độ.
Cho biết: p(H2) = 1 atm; NH4+ có pKa = 9,24; H2S có pKa1 = 7,02; pKa2 = 12,90; CH3COOH có pKa
= 4,76;
H3PO4 có pKa1 = 2,15; pKa2 = 7,21; pKa3 = 12,32.
Câu 2 Lời giải Điểm
a) Xét trong dung dịch A: 1,0
0,08 0,02
CCH COOH =
= 0,04M; CNa PO = = 0,01M
3
2 3 4
2
Na3PO4 → 3Na+ + PO43-
0,01
- 0,03 0,01
Ta có: Ka1 (H3PO4) > Ka (CH3COOH) > Ka2 (H3PO4) > Ka3 (H3PO4).
CH3COOH + PO43- ↔ CH3COO- + HPO42- K1 = 10-4,76.1013,32 = 107,56
0,04 0,01
0,03 - 0,01 0,01
CH3COOH + HPO4 2-
↔ CH3COO + H2PO4- K2 = 10-4,76.107,21 = 102,45
-

0,03 0,01 0,01


0,02 - 0,02 0,01
TPGH: CH3COOH 0,02M
CH3COO- 0,02M
H2PO4- 0,01M
+
Na 0,03M
=> dung dịch A là dung dịch đệm và pH của dung dịch được quyết định bởi hệ
đệm CH3COOH/CH3COO-
CCH COO-
3
=> pHA = pKa (CH3COOH) + lg = 4,76.
CCH COOH
3

Xét dung dịch B: (NH4)2S → 2NH4 + S2- +

0,05
- 0,1 0,05
+
Ta có: Ka1 (H2S) > Ka (NH4 ) > Ka2 (H2S).
NH4+ + S2- ↔ NH3 + HS- K3 = 10-9,24.1012,90 = 103,66 >> 1
0,10 0,05
0,05 - 0,05 0,05
TPGH của dung dịch B: +
NH4 0,05M
NH3 0,05M
HS- 0,05M
=> dung dịch B là dung dịch đệm và pH của dung dịch được quyết định bởi hệ
NH4+/NH3
CNH
pHB = pKa (NH4+) + lg 3
= 9,24.
CNH+
4

b) Điện cực làm việc với các dung dịch A và B là điện cực hiđro và [H+]A > 0,5
[H+]B
=> điện cực hiđro nhúng vào dung dịch B là cực âm;
=> điện cực hiđro nhúng vào dung dịch A là cực dương;
Cấu tạo pin: (-) H2 (p = 1 atm) | dung dịch B || dung dịch A| H2 (p = 1 atm)
(+)
[H+ ] ddA
Epin = E(+) - E(-) = 0,0592 lg = 0,265V
[H+ ] ddB
c) Phản ứng tại: 0,5
1
Cực âm: H2 + CH3COO- → CH3COOH + e
2
1
Cực dương: NH4+ + e → NH3 + H2
2
d) Tại pH = 9 0,5
[CH3COOH] [H+ ] 10- 9
= = - 4,76 = 10- 4,24 => [CH3COO-] >>
[CH3COO- ] Ka 10
[CH3COOH] => CH3COOH phản ứng hết.
[H 2PO-4 ] [H+ ] 10- 9
= = - 7,21 = 1,62.10- 2
[HPO24 ] Ka2 10

[HPO24- ] [H+ ] 10- 9


3-
= = - 12,32 = 103,32 => [HPO42-] >> [PO43-] => coi như
[PO4 ] Ka3 10
sự chuyển hóa của HPO42- sang PO43- là không đáng kể.
Gọi V (ml) là thể tích dung dịch NaOH 0,03M; Thể tích dung dịch thu được =
V + 20 (ml)
Tại điểm dừng chuẩn độ, thành phần của hệ gồm: Na+, H2PO4-, HPO42-,
CH3COO-, OH-
Áp dụng định luật bảo toàn điện tích:
[Na+] = [CH3COO-] + [H2PO4-] + 2[HPO42-] + [OH-]
+
0,03V + 0,03.20 20 20 [H ] + 2Ka2
= 0,04 + 0,01 . + + 10- 5 =>
V + 20 V + 20 V + 20 [H ] + Ka2
V = 19,907 mL
Các phản ứng chuẩn độ: H2PO4- + OH- → HPO42- + H2O

CH3COOH + OH- → CH3COO- + H2O


Ta có: VTĐ.0,03 = (0,01 + 0,02).20 => VTĐ = 20 mL
V - VTD
Sai số chuẩn độ, q = .100% = -0,467%.
VTD

Câu 3. (2,5 điểm) Nhiệt động học và cân bằng hóa học
Nước là hợp chất có vai trò rất quan trọng trong đời sống và sản xuất. Trong bài này ta sẽ khảo
sát quá trình hoá hơi của nước. Sự hoá hơi của nước tại áp suất bar có biến thiên enthalpy chuẩn
HV0 = 40, 7 ( kJ .mol −1 ) và biến thiên entropy chuẩn là SV0 = 109,1( J .mol −1.K −1 ) tại p0 = 1 bar.
1. Tính nhiệt độ sôi TS của nước tại áp suất 1 bar và áp suất hơi p của nước ở nhiệt độ 87°C.
2. Tính biến thiên năng lượng tự do Gibbs khi nước bay hơi ở 87°C có độ ẩm là 50%. Cho
biết: độ ẩm là tỉ số giữa áp suất riêng phần của hơi nước và áp suất hơi bão hoà của nước.
3. Một cylinder với một piston được mô tả như hình bên chứa 0,1 mol
argon và 1,0 mol nước (lỏng và hơi). Nhiệt độ của nước và argon luôn được
duy trì ở 87°C. Giả thiết rằng thể tích của chất lỏng không đáng kể so với chất
khí. Ban đầu, áp suất của hệ là 1,0 bar.
a) Tính áp suất riêng phần của argon và hơi nước (bar) trong bình.
b) Tính thể tích ban đầu của bình và số mol nước lỏng còn lại trong
cốc.
c) Sau đó piston được kéo nhanh đến thể tích 15,8 L. Tính áp suất riêng phần của argon và
hơi nước (bar) ngay khi thể tích vừa đạt 15,8 L.
d) Khi thể tích bình đạt 15,8 L, áp suất hơi nước tăng dần cho đến khí quá trình sôi của nước
dừng lại (hệ đạt trạng thái cân bằng). Xác định số mol nước lỏng còn lại trong cốc khi hệ đạt cân
bằng.
Câu 3 Lời giải Điểm
1 p = po = 1 bar (hệ đạt cân bằng) 1,0
HV0
GV = HV0 − T SV0 = 0 → T = = 373, 05 ( K ) (99,00C)
SV0
 p  H o  1 1 
ln  1  =  − 
 p2  R  Ts (2) Ts (1) 
 1  40700  1 1 
 ln   =  − 
 p2  8,314  360,15 373, 05 
 p2 = 0, 625 ( bar )
2 H2O(l) → H2O(g) 0,5
PH O( g )
KP = 2 = 0,5
P0
 G = − RT ln K = −8,314.360,15ln(0,5) = −2, 07 ( kJ .mol −1 )
3 a) Áp suất hơi bão hòa của H2O ở 87oC là 0,625 bar 0,25
→ PAr = 1 - 0,625 = 0,375 (bar)
n .RT 0,1.8,314.360,15
b) V = Ar = 5
= 7,985.10−3 (m3 ) = 7,985( L)
PAr 0,375.10 0,25
PH2O .V 0, 625.105.7,985.10−3
nH2O( g ) = = = 0,167(mol )
RT 8,314.360,15
nH2O(l ) = 1 − 0,167 = 0,833 ( mol )
c) Ngay khi thể tích vừa đạt 15,8(L) và do quá trình kéo piston diễn ra nhanh nên
đây là quá trình bất thuận nghịch 0,25
7,985
PH 2O = 0, 625. = 0,316 ( bar )
15,8
7,985
PAr = 0,375. = 0,189 ( bar )
15,8
0,25
d) Khi hệ đạt cân bằng thì PH2O = 0,625 ( bar )
PV 0, 625.105.15,8.10−3
nH2O( g ) = =  0,33 ( mol )
RT 8,314.360,15
nH2O(l ) = 1 − 0,33 = 0, 67 ( mol )

Câu 4. (2,5 điểm) Hóa nguyên tố (Kim loại, phi kim nhóm IVA, VA). Phức chất
Kim loại X đã được biết đến từ thế kỉ 18. Tuy nhiên, trước đó những thợ khai thác mỏ đã biết rõ về
quặng của X. X trông giống như đồng và được sử dụng trong việc sơn màu lục cho thủy tinh. X là
một kim loại khá không hoạt động. Để sử dụng nó trong sản xuất trước tiên phải chuyển hóa X bằng
các phản ứng với flo và các tác nhân oxi hóa mạnh khác.

Sơ đồ trên cho thấy sự biến đổi giữa các hợp chất của X với các mức độ oxi hóa khác nhau. Biết
rằng:
- Công thức phân tử I gồm 5 nguyên tử. Khi phân hủy I khối lượng giảm 14.1%;
- Phần trăm khối lượng các nguyên tố trong H: %(Xe) = 42.00 %; %(F) = 48.62 %; %(X) =
9,38 %
- Khi phân hủy D khối lượng giảm 67,88%;
- A là một hexahidrat với %(Cl) = 29,83 %;
- Trong E, G số phối trí của X là 4. Trong G: %(X) = 27,31 %;
- Trong phản ứng F → G, kali bị thiếu và amoniac lỏng là dung môi.
4.1. Xác định tất cả các chất chưa biết A - J và kim loại X, giải thích.
4.2. Viết phương trình của tất cả các phản ứng 1 - 10.

Câu Nội dung Điểm


4
4.1 А là hexahidrat mà theo đề bài:

(với n là số nguyên tử Cl).

Mặt khác:
⇒ 0,25
n 1 2 3 4
MX -24,69 58,71 142,11 225,51
X - Ni, Co - Ra
⇒ chọn n = 2
Khi X phản ứng với CO tạo ra tetracarbonyl niken - Ni(CO)4 hoặc octacarbonyl
dicobalt - Co2(CO)8; Co trong hợp chất với CO có số phối trí 5 không phải là 4, do
đó X = Ni. 0,25

⇒ А là NiCl2·6H2O.
Khi А tác dụng với kali oxalate, niken oxalate được hình thành, nó bị phân hủy để
tạo thành niken pyrophoric. 0,25
МD = 58,7/(1 − 0,6788) = 182,73 g⁄mol tương ứng với D là NiC2O4·2H2O.
Niken pyrophoric được sử dụng để điều chế niken tetracarbonyl ⇒ E là
Ni(CO)4).
Dưới nhiệt độ vừa phải, niken clorua hydrat bị khử nước tạo nickel clorua ⇒ B là
NiCl2.
Xác định thành phần của H theo % khối lượng của các nguyên tố trong H
M % %/M 0,25
Ni 58,693 9,38 0,1598 1
Xe 131,29 42,00 0,3199 2
F 19,00 48,62 2,5592 16 0,25
Vậy công thức H là Xe2F16Ni. Nếu cho rằng đây là hợp chất phức tạp chứa anion
[NiF6]2− thì thành phần cation là [Xe2F10]2+. Thông thường thành phần các ion chỉ
gồm một nguyên tử trung tâm, tức H gồm 2 cation [XeF5]+ ⇒ H là (XeF5)2[NiF6].
Asen (V) florua là axit Lewis sẽ thay thế niken (IV) florua trong H để tạo I chứa 5
nguyên tử, nên I là NiF4.
Khi phân hủy I sẽ mất khối lượng là 14,1%, tương ứng với việc loại bỏ một
nguyên tử flo và sự hình thành của niken trifluoride ⇒ J là NiF3. 0,25
NiF4 là một axit Lewis và có thể phản ứng với KF với tạo hợp chất phức ⇒ C là
K2[NiF6], cũng có thể tạo thành khi niken (II) clorua khan tác dụng với kali clorua
và flo.
Trong phản ứng của niken (II) clorua với kali xyanua tạo niken xyanua
⇒ F là Ni(CN)2.
Dưới tác dụng của kali ammoniac lỏng. MG = 58,70/0,2731 = 214,91 g/mol tương
ứng với công thức K2Ni(CN)3. Tuy nhiên theo đề bài, số phối trí của Nicken là 4
nên anion trong G là đime [Ni2(CN)6]4- ⇒ G là K4[Ni2(CN)6]

Vậy: A là NiCl2.6H2O ; B là NiCl2; C là K2[NiF6] ; D là


NiC2O4.2H2O
E là Ni(CO)4 ; F là Ni(CN)2 ; G là K4[Ni2(CN)6] ; I là NiF4 ;
H là (XeF5)2[NiF6] J là NiF3
4.2 1) (XeF5)2[NiF6] + 2AsF5 → 2XeF5[AsF6] + NiF4;
2) 2NiF4 → 2NiF3 + F2
3) NiF4 + 2KF → K2[NiF6]
4) NiCl2 + 2KCl + 3F2 → K2[NiF6] + 2Cl2 1,0
5) NiCl2 + 2KCN → Ni(CN)2↓ + 2KCl
6) 2Ni(CN)2 + 2KCN + 2K → K4[Ni2(CN)6]
7) NiCl2∙6H2O → NiCl2 + 6H2O
8) NiCl2 + K2C2O4 +2H2O → NiC2O4·2H2O↓ + 2KCl
9) NiC2O4 v Ni + 2CO2
10) Ni + 4CO → [Ni(CO)4]
Câu 5 (2,5 điểm) Đại cương hữu cơ.
b. Cho các hợp chất sau:

a. So sánh nhiệt độ sôi của các chất C1, C2, C3, C4 và giải thích.
b. So sánh tính base của các chất D1, D2, D3 và giải thích.
2. So sánh nhiệt hydrogen hóa (so sánh giá trị ΔH ohydrogen ) của các hydrocarbon sau và giải thích:

Biết rằng giá trị enthalpy chuẩn của phản ứng hydrogen hóa các hydrocarbon trên đều âm:

3. Cho các hợp chất sau:

a. Giải thích tại sao giá trị pKa của F1 lớn hơn pKa1 của F2.
b. Trong dung môi nước, hãy so sánh tính acid của hai đồng phân cis và trans-4-tert-butyl
cyclohexane carboxylic acid. Giải thích.
Câu 5 Lời giải Điểm
1.a.
Nhiệt độ sôi: C3 > C2 > C1 > C4. 0,25
C3 (có liên kết H liên phân tử bền hơn và nhiều hơn) > C2 (có liên kết H liên phân tử
kém bền hơn và ít hơn) > C1 (không có liên kết H liên phân tử, phân tử phân cực hơn
do nguyên tử oxygen có độ âm điện lớn hơn) > C4 (không có liên kết H liên phân tử,
phân tử phân cực hơn do nguyên tử nitrogen có độ âm điện nhỏ hơn)

0,25
1.b.
Tính base: D3 < D2 < D1. 0,25
D3 (phân tử mạch hở nên sự quay cấu dạng của gốc hydrocarbon cản trở sự solvat hóa
cation sinh ra) < D2 (nguyên tử nitrogen thứ hai có độ âm điện lớn, hút electron làm
giảm tính base của nguyên tử nitrogen còn lại so với D1) < D1. 0,25
Độ bền: E2 > E1 > E3  giá trị ΔH ohydrogen của E2 < E1 < E3.
0,25
E2 có sự biến dạng góc liên kết ít nhất: có một góc liên kết bị ép từ 120o thành 60o trong
2
khi E1 và E3 có 2 góc bị biến dạng như vậy.
E1 có 5 Hα so với E3 có 1 Hα nên E1 có hiệu ứng siêu liên hợp của Hα mạnh hơn giúp 0,25
làm bền hệ π.
3.a
Giá trị pKa của F1 lớn hơn pKa1 của F2 do F2 tạo được liên kết hydrogen nội phân tử
làm bền anion sinh ra nên tính acid của F2 mạnh hơn. 0,25

0,25
3.b
3 Tính acid của đồng phân trans mạnh hơn cis 0,25

Do anion sinh ra từ đồng phân trans có nhóm -COO- ở vị trí liên kết e ít bị cản trở
không gian khi solvat hóa trong dung dịch. Ở đồng phân cis, hai nguyên tử H ở vị trí
liên kết a đối diện sẽ cản trở sự solvat hóa nhóm -COO-.

0,25

Câu 6 (2,5 điểm) Sơ đồ tổng hợp hữu cơ. Cơ chế phản ứng hóa hữu cơ.
b. Viết cơ chế giải thích sự hình thành các sản phẩm sau:

2. Sơ đồ chuyển hóa
a. Một dẫn xuất furane C6 được tổng hợp theo sơ đồ sau:

b. (5S,9S)-5,9-dimethylpentadecane (D) là sex-pheromon, một chất dẫn dụ bướm đêm (Leucoptera


coffeella, gây tác hại đến cà phê). Chất D được tổng hợp từ isopulegol theo sơ đồ sau:
Câu Lời giải Điểm
a) 0,5

1 b)

0,5

b. Cấu tạo C1, C2, C3 (1/4); C4, C5, C6 (1/4). 0.5

b. Cấu trúc D1, D2, D3 (1/4); D4, D5, D6 (1/4); D7, D8, D9 (1/4); D10, D11 (1/4). 1,0

Câu 7 (2,5 điểm) Xác định cấu trúc các chất hữu cơ (mô tả sơ đồ tổng hợp bằng lời dẫn)
Phản ứng ozon- khử hóa hợp chất I (C3H6) thu được chất A và B. Phản ứng của A và B khi
có mặt của K2CO3 cho C. Chất C cũng phản ứng được với A khi có mặt của của Ca(OH)2 cho ancol
D và muối canxi E (C và D có cùng số nguyên tử cacbon). D phản ứng với PBr3 thu được chuyển
qua F rồi bị khử bởi Zn thành X (C5H8). X không làm mất màu dung dịch KMnO4 ở nhiệt độ thấp
(lạnh).
1. Viết công thức của các hợp chất từ A đến X, I.
2. Y là đồng phân của X và có thể tổng hợp từ xiclopentadien và dietyl este của axit
azodicarboxylic theo sơ đồ sau:

Viết công thức của các hợp chất từ G đến Y, biết rằng Y không phản ứng với KMnO4 ở 0 oC.

Câu Lời giải Điểm

0,2đ/c
1
hất

0,2đ/c
2
hất

Câu 8 (2,5 điểm) Hóa học các hợp chất thiên nhiên (Cacbohidrat và các hợp chất hữu cơ chứa
nito đơn giản)
1. Monosaccarit galactozơ có tên là (2R, 3S, 4S, 5R)-2,3,4,5,6-pentahidroxihexanal. Khi đun nóng
tới 165OC galactozơ bị tách nước sinh ra sản phẩm trong đó tạo thành một lượng nhỏ hợp chất hai
vòng B có tên là 1,6-anhidrogalactofuranozơ.
a. Đề nghị cấu trúc của B và giải thích sự tạo thành nó.
b. Từ galactozơ có thể nhận được các sản phẩm E (C5H10O5) và G (C5H8O7) theo sơ đồ phản ứng :

Hãy viết công thức cấu tạo của C, D, E, G.


c. Vẽ cấu dạng vòng 6 cạnh C1 và 1C của galactozơ và giải thích vì sao dạng vòng 6 cạnh của
galactozơ không tham gia phản ứng tách nước như trên.
2. Thủy phân hoàn toàn một hexapeptit M thu được Ala, Arg, Gly, Ile, Phe và Tyr. Còn khi thủy
phân không hoàn toàn thu được đipeptit E (chứa Phe, Arg) và peptit G (chứa Arg, Phe và Ile). Dùng
2,4-đinitroflobenzen xác định được amino axit đầu N của peptit M là Ala. Còn khi cắt mạch M bằng
tripsin thu được tripeptit A (chứa Ala, Arg và Tyr).
a. Xác định trật tự sắp xếp các amino axit trong M.
b. Cho biết aminoaxit có pHI cao nhất? Giải thích?
Cấu tạo chung của các amino axit là H2N-CHI-COOH với gốc R tương ứng như sau:

Câu 8 Nội dung Điểm


b. Cấu trúc của B và sự tạo thành:

0,5

b. C: CH2OH-(CHOH)4-COOH 0,5
D: CH2OH-(CHOH)4-COO- (hoặc muối canxi).
1 E: CH2OH-(CHOH)3-CHO
G: HOOC-(CHOH)3-CHO

c. 0,25
Cấu dạng bên trái bền hơn (các nhóm lớn lại nằm ở vị trí e) nhưng các nhóm –OH
số 1 và số 6 lại ở xa nhau nên không thể tách nước. 0,25
b. Hexapeptit M có đầu N là Ala.
Thủy phân M nhờ trypsin xác định được tripeptit có cấu tạo phù hợp là Ala – Tyr – 0,25
Arg.
Đipeptit E có cấu tạo phù hợp là Arg – Phe vì Arg thuộc tripeptit, có Arg ở đuôi C.
2
Từ cấu tạo của E → tripeptit G có cấu tạo phù hợp là Arg – Phe – Ile. 0,25
Còn duy nhất amino axit đuôi C là Gly.
Từ đó xác định được cấu tạo của M là Ala – Tyr – Arg – Phe – Ile – Gly. 0,25
b. pH I lớn nhất là của Arg, vì có nhóm guanidin có tính bazơ mạnh. 0,25

-------Hết-------
TRƯỜNG THPT CHUYÊN THÁI BÌNH ĐỀ THI ĐỀ XUẤT CHỌN HSG KHU VỰC ĐBDHBB
NĂM HỌC 2022 - 2023
Môn: Hóa học – Lớp 11
Thời gian làm bài 180 phút (không kể thời gian phát đề)

Câu 1 (2,5 điểm) Động học


Dưới đây là các giai đoạn phản ứng trong một quá trình este hóa có xúc tác là axit.
RCOOH + HA ⎯ ⎯⎯ k1
→ RCOOH ---HA
⎯ (1)
k2

RCOOH ---HA + R’OH ⎯⎯


k3
⎯ ⎯→ RCOOH ---HA --- R’OH (2)
k4

RCOOH ---HA --- R’OH ⎯⎯


k5
→ RCOOR’ + HA + H2O (3)

a. Viết phương trình phản ứng cho toàn bộ quá trình trên.
b. Các số liệu về phản ứng xúc tác axit được cho trong bảng dưới đây. Tìm biểu thức định luật tốc độ
phản ứng và giá trị hằng số tốc độ, thứ nguyên của nó. Cho hằng số cân bằng của phản ứng este hóa này
là đủ lớn.
[RCOOH]M [R’OH]M [HA]M Tốc độ đầu, M. phút−1
0,35 0,35 0,50 4,60
0,62 0,35 0,50 8,14
0,35 0,81 0,50 10,6
0,35 0,50 0,75 9,84
c. Cho biết giai đoạn quyết định tốc độ phản ứng dựa trên biểu thức định luật tốc độ phản ứng ở câu 3c,
giải thích cho câu trả lời của bạn.
d. Tốc độ đầu của phản ứng có thể được xác định bằng cách dập tắt phản ứng và xác định lượng R-OH
còn lại nhờ phản ứng của nó với Cr2O72−.
i. Viết phương trình phản ứng giữa Cr2O72− với R’-OH trong dung dịch axit. Cho rằng R’ là CH3-CH2-
và sản phẩm phản ứng là Cr3+ và CH3COOH.
ii. Mô tả sự thay đổi màu sắc trong thí nghiệm xác định lượng R-OH còn lại.
Câu 2 (2,5 điểm) Cân bằng trong dung dịch
Acqui chì được nhà hóa học Pháp Louis Gaston Plante phát minh vào năm 1859. Ở trạng thái nạp điện,
điện cực bên trái của acqui (với thế điện cực âm hơn) tạo thành từ chì kim loại, còn điện cực bên phải là
chì(IV) oxit. Chất điện li là dung dịch axit sulfuric nồng độ phần trăm C%  20 - 30%.
a) Xây dựng sơ đồ pin đơn giản nhất ứng với acqui chì.
b) Cho các thế điện cực chuẩn: E oPbSO ,SO2− /Pb = −0,359; E oPbO ,H+ ,SO2− /PbSO = 1, 682 và E oPb2+ /Pb = −0,126 V.
4 4 2 4 4

1. Viết phương trình phản ứng tổng cộng xảy ra trong pin.
2. Tính sức điện động chuẩn của pin điện hóa.
3. Xác định năng lượng Gibbs chuẩn của phản ứng tổng cộng.
4. Tính hằng số cân bằng của phản ứng tại 25oC.
c) Tại mặt phân cách “Pb|chất điện li”, xảy ra tương tác hóa học tạo ra chì(II) sulfat. Viết phương trình
phản ứng xảy ra.
d) Tại điện cực “PbO2|chất điện li”, xảy ra phản ứng hình thành chì(IV) sulfat là một chất kém bền, ít tan
và có thể phân hủy giải phóng oxi trong sự có mặt của chì(IV) oxit, sản phẩm phụ là chì(II) sulfat. Viết
phản ứng hình thành chì(IV) sulfat và phản ứng giải phóng oxi.
e) Trong một pin đã được nạp đầy ở 25oC, nồng độ cation chì(II) trong dung dịch là 9,3.10-7 M. Xác định
thế của điện cực Pb|H2SO4 ở trạng thái này.
f) Dung dịch tương ứng 376 g axit sulfuric trong 1000 g nước, với hệ số hoạt độ axit  H2SO4 = 0,165 và
hoạt độ của nước a H2O = 0, 7 mol.kg −1. Xác định sức điện động của acqui chì tại 25oC.
Câu 3: (2,5 điểm) Nhiệt động lực học - cân bằng hoá học
Tính chất nhiệt động của một số phân tử và ion ở trạng thái tiêu chuẩn tại 25oC như sau:
C3H8(k) O2(k) CO2(k) H2O(l) CO32 − (aq) OH-(aq)

1
H S0 (kJ/mol) -101,85 0 - 393,51 - 285,83 - 677,14 - 229,99
S0(J/molK) 269,91 205,138 213,74 69,91 - 56,9 - 10,75
Xét quá trình oxi hoá hoàn toàn 1 mol C3H8(k) với O2(k) tạo thành theo 2 cách :
a) Bất thuận nghịch
b) Thuận nghịch (trong 1 tế bào điện hoá)
1) Tính H0, U0, S0, G0 của phản ứng trong mỗi cách nói trên?
2) Tính nhiệt, công thể tích, công phi thể tích (tức là công hữu ích) mà hệ trao đổi với môi trường trong
mỗi cách?
3) Tính S của môi trường và S tổng cộng của vũ trụ khi tiến hành quá trình theo mỗi cách.
4) Một mô hình tế bào điện hoá khác làm việc dựa trên phản ứng oxi hoá C3H8(k) bởi O2(k) khi có mặt
dung dịch KOH 5M với điện cực Pt. Các loại phân tử và ion (trừ KOH) đều ở trạng thái tiêu chuẩn. Hãy
viết các nửa phản ứng ở catot và anot và phản ứng tổng cộng trong tế bào điện hoá. Nếu từ tế bào điện hoá
đó, ở 25oC, ta thu được dòng điện 100mA. Hãy tính công suất cực đại có thể đạt được.
Câu 4 (2,5 điểm) Phức chất và Hóa nguyên tố
1) Sử dụng thuyết trường tinh thể hãy xác định cấu hình electron của Fe2+ trong các phức spin cao và spin
thấp.
2) Tính năng lượng bền hóa trường tinh thể (CFSE, theo đơn vị kJ mol-1) của [Fe(CN)6]4− và [Fe(H2O)6]2+,
Cho biết rằng: Δo([Fe(CN)6]4−) = 32800 cm-1, Δo([Fe(H2O)6]2+) = 10400 cm-1, năng lượng ghép đôi của
Fe2+: P = 229,1 kJ mol-1.
3) Trong đa số trường hợp, dạng hình học của các phức bát
diện
không tương ứng với hình bát diện đối xứng lí tưởng, mà sẽ bị
biến dạng. Hiện tượng này được giải thích bởi hiệu ứng Jahn-
Teller, lí thuyết này phát biểu rằng phân tử với các orbital có
năng lượng bằng nhau có xu hướng bị biến dạng hình học và
do đó năng lượng của phân tử sẽ giảm khi cấu hình electron
thay đổi. Trong hình dưới đây, δ1 và δ2 là năng lượng tách mức
biến dạng trong các orbital của các nhóm eg và t2g.
a) Trong hình bên, hãy gán các obitan d của kim loại vào mỗi
giản đồ tương ứng. Sự rút ngắn hoặc kéo dài liên kết theo trục z trong
b) Bằng việc tính năng lượng bền hoá khi biến dạng (so với khi phức bát diện
chưa biến dạng), hãy cho biết các phức bát diện nào sau đây có
xu hướng biến dạng và cho biết sự rút ngắn hoặc kéo dài liên kết theo trục z xảy ra với các phức này:
i) [CrCl6]4- (spin cao). ii) [Mn(CN)6]4− (spin thấp). iii) [Mn(H2O)6]2+ (spin cao).
Câu 5 (2,5 điểm) Đại Cương
1) Phản ứng thủy phân ester CH3COOC2H5 trong kiềm là một phản ứng bậc hai với biểu thức vận tốc có
dạng v = k[ester][OH-].Có hai cơ chế thỏa mãn biểu thức vận tốc này. Hãy đề nghị hai cơ chế và cho
biết cơ chế nào ưu tiên hơn trong hai cơ chế đã đưa.
2) Hãy sắp xếp các cấu trúc sau đây theo thứ tự tăng dần tính bền liên kết amide C-N.

3) Tốc độ thủy phân dẫn xuất 7-norbornyl tosylate tăng lên khi có mặt nối đôi

2
a) Hãy giải thích ảnh hưởng của nối đôi đến sự tăng tốc dung môi phân
b) Khi đưa một nhóm p-methoxyphenyl vào vị trí 7 cũng làm tăng vận tốc phản ứng nhưng không
quá nhiều. Giải thích?
Câu 6 (2,5 điểm) Cơ chế phản ứng và Sơ đồ tổng hợp
1.Đề xuất cơ chế phản ứng cho các chuyển hoá:

a)

b)

c)

2. Hoàn chỉnh sơ đồ tổng hợp sau đây :

Câu 7 (2,5 điểm) Xác định cấu trúc


1) A (C9H16O5) là một hợp phần thu được khi thủy phân trioxacarcinose A, một hợp chất tự nhiên có khả
năng làm thay đổi cấu trúc DNA. Thủy phân A bằng acid loãng thu được B (C7H14O4), chất này phản
ứng được với bạc nitrat trong amoniac cho C có khả năng tạo kết tủa vàng với I2/NaOH và tạo thành
hợp chất D (C6H10O6). D bị phân cắt bằng HIO4 tạo thành oxalic acid và E (C4H6O3) không bền nhiệt
và bị tách tạo acetone khi đun nóng. Xác định cấu trúc các chất chưa biết. Biết rằng A không có khả
năng tham gia phản ứng tráng gương hay làm mất màu nước brom, và sự phân cắt C bằng HIO4 cũng
dẫn đến sự tạo thành E.
2) Xử lý geraniol (3,7-dimethylocta-6-enal) bằng [CH2=NMe2]+Cl- (muối Eschenmoser) trong sự có mặt
của Et3N thu được X1 (C11H18O). Chất này dưới ảnh hưởng của xúc tác Grubbs sẽ tạo thành X2
(C7H10O) cho phản ứng tráng gương. X2 phản ứng với dạng keto của 5-isopropyl-2-methylfuran-3-ol
khi có sự hỗ trợ của LDA/THF tạo thành X3 (C15H22O3). SmI2 phản ứng với X3 để tạo thành X4
(C15H24O3) không làm mất màu nước brom, không cho phản ứng tráng gương. Thủy phân X4 thu được
4,7,8-trihydroxy-7-isopropyl-1,4-dimethyloctahydroazulen-5-one. Hãy xác định cấu trúc các chất chưa
biết.
Câu 8 (2,5 điểm) Hợp chất thiên nhiên
1. Stagonolide D (C10H14O4) dễ dàng bị thủy phân trong môi trường acid tạo thành A (C10H18O6) có
tính acid, có khả năng làm mất màu 1 mol nước Br2. Phân cắt A bằng HIO4 thu được 1 mol acetic
acid, 1 mol formic acid và diacid B (C7H10O5) nhanh chóng bị đồng phân hóa thành monoacid C.
A tác dụng với OsO4 tạo thành D, chất này chịu tác dụng của HIO4 thu được succinic acid, acetic
acid và 4 mol formic acid. Xác định cấu trúc các chất chưa biết.

3
2. Coronamic acid (C6H11NO2) là một aminoacid trung gian trong quá trình sinh tổng hợp coronatine
là một chất độc được cô lập từ vi khuẩn. Hydrogen hóa coronamic acid thu được ba chất A1, A2,
A3 đều có cùng công thức C6H13NO2 đều có tính quang hoạt. Có thể tổng hợp coronamic acid
bằng cách cho N2CHCOOEt tác dụng với một chất B (C8H13NO3) rồi thủy phân sản phẩm thu
được trong acid. Thủy phân B trong môi trường acid tạo thành 2-oxopentanoic acid. Hãy xác định
cấu trúc các chất chưa biết.

Người ra đề:
Phạm Văn Tuân 0398483776

HƯỚNG DẪN GIẢI

Câu 1 (2,5 điểm) Động học


Dưới đây là các giai đoạn phản ứng trong một quá trình este hóa có xúc tác là axit.
RCOOH + HA ⎯ ⎯⎯ k1
→ RCOOH ---HA
⎯ (1)
k2

RCOOH ---HA + R’OH ⎯⎯


k3
⎯ ⎯→ RCOOH ---HA --- R’OH (2)
k4

RCOOH ---HA --- R’OH ⎯⎯


k5
→ RCOOR’ + HA + H2O (3)

a. Viết phương trình phản ứng cho toàn bộ quá trình trên.
b. Các số liệu về phản ứng xúc tác axit được cho trong bảng dưới đây. Tìm biểu thức định luật tốc độ
phản ứng và giá trị hằng số tốc độ, thứ nguyên của nó. Cho hằng số cân bằng của phản ứng este hóa này
là đủ lớn.
[RCOOH]M [R’OH]M [HA]M Tốc độ đầu, M. phút−1
0,35 0,35 0,50 4,60
0,62 0,35 0,50 8,14
0,35 0,81 0,50 10,6
0,35 0,50 0,75 9,84
c. Cho biết giai đoạn quyết định tốc độ phản ứng dựa trên biểu thức định luật tốc độ phản ứng ở câu 3c,
giải thích cho câu trả lời của bạn.
d. Tốc độ đầu của phản ứng có thể được xác định bằng cách dập tắt phản ứng và xác định lượng R-OH
còn lại nhờ phản ứng của nó với Cr2O72−.
i. Viết phương trình phản ứng giữa Cr2O72− với R’-OH trong dung dịch axit. Cho rằng R’ là CH3-CH2-
và sản phẩm phản ứng là Cr3+ và CH3COOH.
ii. Mô tả sự thay đổi màu sắc trong thí nghiệm xác định lượng R-OH còn lại.

Nội dung Điểm


a.
O O
HA
R C O H + R' O H R C O R' + H2O 0,5

b.Vì hằng số cân bằng của phản ứng lớn, trong giai đoạn đầu của phản ứng, tốc độ của
phản ứng nghịch là không đáng kể, có thể coi phản ứng là một chiều.
Theo định luật tác dụng khối lượng ta có: v = k. [RCOOH]a[R’OH]b[HA]c
Dựa vào kết quả thực nghiệm:
[RCOOH]M [R’OH]M [HA]M Tốc độ đầu, M. phút−1
4
0,35 0,35 0,50 4,60
0,62 0,35 0,50 8,14
0,35 0,81 0,50 10,6
0,35 0,50 0,75 9,84
Ta có:
v1 = k. (0,35)a(0,35)b(0,5)c = 4,60
v2 = k. (0,62)a(0,35)b(0,5)c = 8,14
v3 = k. (0,35)a(0,81)b(0,5)c = 10,6
v4 = k. (0,35)a(0,5)b(0,75)c = 9,84
Chia v2 cho v1:
v2 0, 62 a 8,14
=( ) =  1,77a = 1,75  a  1.
v1 0,35 4, 60
Chia v3 cho v1:
v3 0,81 a 10, 6
=( ) =  2,31b = 2,30  b = 1.
v1 0,35 4, 6
Chia v4 cho v1:
v4 0,5 0, 75 c 9,84
= .( ) =  c = 1.
v1 0,35 0,5 4, 6
Biểu thức định luật tốc độ phản ứng: v =k. [RCOOH]. [R’OH]. [HA].
Hằng số tốc độ phản ứng ở điều kiện nghiên cứu:
4, 6 0,5
k1 = = 75,1 M-2. min-1
0,35.0,35.0,5
8,14
k2 = = 75,02 M-2. min-1
0, 62.0,35.0,50
10, 6
k3 = = 74,78 M-2. min-1
0,35.0,81.0,50
9,84
k4 = = 74,97 M-2. min-1.
0,35.0,50.0, 75
_
75,1 + 75.02 + 74, 78 + 74,97
k= = 74,96 M-2. min-1.
4
0,5
c.
⎯⎯
k1
RCOOH + HA ⎯ → RCOOH ---HA
⎯ (1)
k2

RCOOH ---HA + R’OH ⎯⎯


k3
⎯ ⎯→ RCOOH ---HA --- R’OH (2)
k4

RCOOH ---HA --- R’OH ⎯⎯ k5


→ RCOOR’ + HA + H2O (3)
Để thuận tiện hơn ta kí hiệu các sản phẩm trung gian như sau:
RCOOH ---HA = C1
RCOOH ---HA --- R’OH = C2
Tốc độ phản ứng có thể được xác đinh qua biến thiên nồng độ của sản phẩm cuối cùng
RCOOR’:
d  RCOOR ' 
v= = k5[C2] (4)
dt
Ở trạng thái dừng, trong một khoảng thời gian nhất định, nồng độ của các sản phẩm trung
gian không thay đổi theo thời gian:

d  C2 
= k3[C1][R’OH] – k4[C2] – k5[C2] = 0 (5)
dt
Rút ra:

5
k3 .C1  .  R 'OH 
[C2] = (6)
k 4 + k5
d C1 
= k1[RCOOH][HA] – k-2[C1] –k3[C1][R’OH] + k4[C2] = 0 (7)
dt
Lấy (7) + (5) ta có:
k1[RCOOH][HA] – k2[C1] – k5[C2] = 0 (8)
Rút ra:
k .[RCOOH].[HA]-k 5 .[C2 ]
[C1] = 1 (9)
k2
Khi k5 << k1, (9) trở thành:
k .[RCOOH].[HA]
[C1] = 1 (10)
k2
Thay (10) vào (6), thu được:
k1.k 3[RCOOH].[HA].[R 'OH ]
[C2] = (11).
k 2 (k4 + k5 )
Thay (11) vào (4) ta có:
k .k [RCOOH].[HA].[R 'OH ]
v = k5 1 3 (12)
k 2 (k4 + k5 )
k1.k3 .k5
Đặt k= thì: v = k [RCOOH][HA][R’OH] (13)
k 2 ( k 4 + k5 )
(12) phù hợp với định luật tốc độ thực nghiệm, thu được với điều kiện
k5 << k1. Như vây, theo quan điểm động hóa học, cơ chế được đề nghị là có khả năng,
trong đó giai đoạn (3) là chậm, tức là tốc độ của phản ứng (3), phản ứng tách nước, quyết
định tốc độ phản ứng trong toàn bộ.
Có thể giải bài toán một cách khác: 0,5
Lần lượt đưa ra giả thiết về một giai đoạn nào là chậm và chứng minh rằng chỉ khi giai
đoạn (3) chậm còn các cân bằng (1) và (2) nhanh mới có thể rút ra được phương trình
động học thực nghiệm còn các giả thiết khác thì không.
Khi giai đoạn (3) quyết định tốc độ chung của phản ứng:
d  RCOOR ' 
v= = k5[C2] (4)
dt
Dựa vào cân bằng (2):
K2 = k3/k4 = [C2]/[C1].[R’OH] trong đó K2 là hằng số cân bằng của (2)
rút ra: [C2] = K2. [C1].[R’OH] (14)
Dựa vào cân bằng (1):
K1 = k1/k2 = [C1]/[RCOOH].[HA] trong đó K1 là hằng số cân bằng của (1)
rút ra: [C1] = K1.[RCOOH] .[HA] (15)
Thay (15) vào (14):
[C2] = K2. [C1].[R’OH] = K1. K2.].[RCOOH] .[HA] [R’OH] (16)
Thay (16) vào (4):
v = k5.K1. K2.[RCOOH] .[HA] [R’OH] = k.[RCOOH] .[HA] [R’OH] (17)
với k = k5.K1. K2
Từ đây có thể kết luận rằng cơ chế được đề nghị là có khả năng, vì từ cơ chế đó có
thể rút ra được biểu thức thực nghiệm của định luật tốc độ.
k1.k3 .k5
Điều kiện k = k5.K1. K2 cũng chính là điều kiện trong cách giải thứ
k 2 ( k 4 + k5 )
nhất, khi thêm giả thiết rằng giai đoạn (2) cũng đủ nhanh tức là k4>>k5.

d) i. 2Cr2O72− + 3CH3CH2OH + 16H+ → 4Cr3+ + 3CH3COOH + 11H2O 0,25


ii. Màu của dung dịch đổi từ da cam sang xanh lục. 0,25
6
Câu 2 (2,5 điểm) Cân bằng trong dung dịch
Acqui chì được nhà hóa học Pháp Louis Gaston Plante phát minh vào năm 1859. Ở trạng thái nạp điện,
điện cực bên trái của acqui (với thế điện cực âm hơn) tạo thành từ chì kim loại, còn điện cực bên phải là
chì(IV) oxit. Chất điện li là dung dịch axit sulfuric nồng độ phần trăm C%  20 - 30%.
a) Xây dựng sơ đồ pin đơn giản nhất ứng với acqui chì.
b) Cho các thế điện cực chuẩn: E oPbSO ,SO2− /Pb = −0,359; E oPbO ,H+ ,SO2− /PbSO = 1, 682 và E oPb2+ /Pb = −0,126 V.
4 4 2 4 4

1. Viết phương trình phản ứng tổng cộng xảy ra trong pin.
2. Tính sức điện động chuẩn của pin điện hóa.
3. Xác định năng lượng Gibbs chuẩn của phản ứng tổng cộng.
4. Tính hằng số cân bằng của phản ứng tại 25oC.
c) Tại mặt phân cách “Pb|chất điện li”, xảy ra tương tác hóa học tạo ra chì(II) sulfat. Viết phương trình
phản ứng xảy ra.
d) Tại điện cực “PbO2|chất điện li”, xảy ra phản ứng hình thành chì(IV) sulfat là một chất kém bền, ít tan
và có thể phân hủy giải phóng oxi trong sự có mặt của chì(IV) oxit, sản phẩm phụ là chì(II) sulfat. Viết
phản ứng hình thành chì(IV) sulfat và phản ứng giải phóng oxi.
e) Trong một pin đã được nạp đầy ở 25oC, nồng độ cation chì(II) trong dung dịch là 9,3.10-7 M. Xác định
thế của điện cực Pb|H2SO4 ở trạng thái này.
f) Dung dịch tương ứng 376 g axit sulfuric trong 1000 g nước, với hệ số hoạt độ axit  H2SO4 = 0,165 và
hoạt độ của nước a H2O = 0, 7 mol.kg −1. Xác định sức điện động của acqui chì tại 25oC.
Nội dung Điểm
a) Sơ đồ pin: -Pb, PbSO4|H2SO4| PbO2, Pb+ 0,5
b) 1) Pb + PbO2 + 2H2SO4 → 2PbSO4 + 2H2O
2) E opin = E oc − E oa = 1, 682 − (−0,359) = 2, 041 V.
3) G opin = −zFE opin = −2  96485  2, 041 = −393,852.103 J
3
−G o /RT − −393,852.10 0,5
4) K = e pin = e 8,314298 = 1, 092.1069
c) Pb + H2SO4 → PbSO4↓ + H2↑ 0,25
d) PbO2+ 2H2SO4 → Pb(SO4)2↓+ 2H2O
0,25
PbO2+ Pb(SO4)2 → 2PbSO4+ O2↑
e) E Pb /Pb = −0,126 + 0,0592
2+ 2 lg 9,3.10−7 = −0,305 V. 0,5
−1
f) Nồng độ molan của axit: c H2SO4 = 376
98 = 3,836 mol.kg

Vậy, hoạt độ của axit: a H2SO4 = 0,165  3,836 = 0, 6331 mol.kg −1


Từ phản ứng tổng cộng khi pin hoạt động, ta có:
a 2H2SO4
E pin = E opin + 0,0592 = 2, 041 + 0,0592
2 lg 0,7 2 = 2, 0384 V.
2
0,6331
2 lg a 2H2O 0,5

Câu 3: (2,5 điểm) Nhiệt động lực học - cân bằng hoá học
Tính chất nhiệt động của một số phân tử và ion ở trạng thái tiêu chuẩn tại 25oC như sau:
C3H8(k) O2(k) CO2(k) H2O(l) CO32 − (aq) OH-(aq)
H S0 (kJ/mol) -101,85 0 - 393,51 - 285,83 - 677,14 - 229,99
0
S (J/molK) 269,91 205,138 213,74 69,91 - 56,9 - 10,75
Xét quá trình oxi hoá hoàn toàn 1 mol C3H8(k) với O2(k) tạo thành theo 2 cách :
a) Bất thuận nghịch
b) Thuận nghịch (trong 1 tế bào điện hoá)
1) Tính H0, U0, S0, G0 của phản ứng trong mỗi cách nói trên?
2) Tính nhiệt, công thể tích, công phi thể tích (tức là công hữu ích) mà hệ trao đổi với môi trường trong
mỗi cách?
3) Tính S của môi trường và S tổng cộng của vũ trụ khi tiến hành quá trình theo mỗi cách.
4) Một mô hình tế bào điện hoá khác làm việc dựa trên phản ứng oxi hoá C3H8(k) bởi O2(k) khi có mặt
dung dịch KOH 5M với điện cực Pt. Các loại phân tử và ion (trừ KOH) đều ở trạng thái tiêu chuẩn. Hãy
7
viết các nửa phản ứng ở catot và anot và phản ứng tổng cộng trong tế bào điện hoá. Nếu từ tế bào điện hoá
đó, ở 25oC, ta thu được dòng điện 100mA. Hãy tính công suất cực đại có thể đạt được.

Nội dung Điểm


1. C3H8(k) + 5O2(k) → 3CO2(k) + 4H2O(l)
1) Do các hàm H, U, S, G là hàm trạng thái nên dù tiến hành theo cách nào thì các giá trị
U, H, S, G cũng như nhau với cùng trạng thái đầu và cuối. Vậy:
H pu
0
= 3 H S0,CO2 ( k ) + 4. H S0, H 2O (l ) - H S0,C3H8 ( k ) - 5. H S0,O2 ( k )
= -3. 393,51 - 285,83 .4 + 103,85 = -2220 (kJ)
H = 213,74. 3 + 4.69,91 - 269,91 - 5. 205,138 = -374,74 (J/K)
0
pu 0,5
G = H - T. S = -2220 + 298,15 .374,74.10
0
pu
0 0 -3
= -2108,27 (kJ)
U0 = H0 - (PV) = H0 - nRT = -2220 - (-3).8,314.298,15.10-3 = -2212,56(kJ)

2) a) Quá trình bất thuận nghịch:


- Nhiệt mà hệ trao đổi với môi trường là QBTN = H0 = -2220 (kJ)
2
- Wtt = -  P.dV = -P. V = -n(k) .RT
1 0,5
= 3. 8,3145.298,15 = 7436,9(J)
- W’ = 0
2 b) Quá trình thuận nghịch:
- QTN = T. S = 298,15 (-374,74) = - 111728,731(J)
- W’max = G = -2108,27(kJ) < 0 : Hệ sinh công 0,5
- Wtt = - n(k) .RT = 7436,9(J) > 0: hệ nhận công

3) a) Quá trình bất thuận nghịch:


Q Q H 0 2220.10 3
Smt = mt = - BTN = - =- = 7445, 916 (J/K)
T T T 298,15
 S vũ trụ = Smt + S hệ = 7445,916 - 374,74 = 7071,176(J/K)
b) Quá trình thuận nghịch: 0,5
Q Q 111728,731
Smt = mt = - TN = = 374,74( J / K )
T T 298,15
 S vũ trụ = Smt + S hệ = 0

4) Các nửa phản ứng:


Anot: C3H8 + 26OH- → 3 CO32 − + 17H2O + 20e
Catot: O2 + 2H2O + 4e → 4OH-
Phản ứng tổng cộng:
C3H8(k) + 5O2(k) + 6OH-(aq) → 3 CO32(−aq) + 7H2O(l)
 Sơ đồ pin:
(-) Pt, C3H8(1atm)/KOH(5M), K2CO3(1M)/ O2(1atm), Pt (+)
H pu0
= 3(-677,14) + 7.(-285,83) + 103,85 - 5.0 - 6(-229,99) = -2548,44(KJ)
S pu
0
= 3.(-56,9) + 7.69,91 - 269,91 - 5.205,138- 6(-10,74) = -912,43(KJ) 0,5
G = H = T. S
0
pu
0
pu
0
pu = -2548,44 + 298,15.912,43.10 -3
= - 2276,399(KJ)
G 0 2276399
 E pu
0
=- = = 1,18(V)
nF 20.96485

8
0,0592 [CO32− ]3 0,0592
→ E = E0 - lg − 6 5
= 1,18 - lg(5)-6 = 1,19(V)
20 [OH ] .PC3 H8 .PO2 20
→ P = E .I = 1,19 .0,1 = 0,119(W)

Câu 4 (2,5 điểm) Phức chất và Hóa nguyên tố


1) Sử dụng thuyết trường tinh thể hãy xác định cấu hình electron của Fe2+ trong các phức spin cao và spin
thấp.
2) Tính năng lượng bền hóa trường tinh thể (CFSE, theo đơn vị kJ mol-1) của [Fe(CN)6]4− và [Fe(H2O)6]2+,
Cho biết rằng: Δo([Fe(CN)6]4−) = 32800 cm-1, Δo([Fe(H2O)6]2+) = 10400 cm-1, năng lượng ghép đôi của
Fe2+: P = 229,1 kJ mol-1.
3) Trong đa số trường hợp, dạng hình học của các phức bát
diện
không tương ứng với hình bát diện đối xứng lí tưởng, mà sẽ bị
biến dạng. Hiện tượng này được giải thích bởi hiệu ứng Jahn-
Teller, lí thuyết này phát biểu rằng phân tử với các orbital có
năng lượng bằng nhau có xu hướng bị biến dạng hình học và
do đó năng lượng của phân tử sẽ giảm khi cấu hình electron
thay đổi. Trong hình dưới đây, δ1 và δ2 là năng lượng tách mức
biến dạng trong các orbital của các nhóm eg và t2g.
a) Trong hình bên, hãy gán các obitan d của kim loại vào mỗi
giản đồ tương ứng. Sự rút ngắn hoặc kéo dài liên kết theo trục z trong
b) Bằng việc tính năng lượng bền hoá khi biến dạng (so với khi phức bát diện
chưa biến dạng), hãy cho biết các phức bát diện nào sau đây có
xu hướng biến dạng và cho biết sự rút ngắn hoặc kéo dài liên kết theo trục z xảy ra với các phức này:
i) [CrCl6]4- (spin cao). ii) [Mn(CN)6]4− (spin thấp). iii) [Mn(H2O)6]2+ (spin cao).

Nội dung Điểm


1) Vẽ giản đồ tách mức năng lượng trong trường bát diện
Cấu hình Fe2+ trong phức
- spin cao: (t2g)4(eg)2. 0,25
- spin thấp: (t2g)6. 0,25

9
2)
*) Δo([Fe(CN)6]4−) = 32800 cm-1 = 6,0223.1023. 6,626.10-34.2,9979.1010. 32800=
392378,139 J/mol = 392,378 kJ.mol-1.
Δo([Fe(H2O)6]2+) = 10400 cm-1= 6,0223.1023. 6,626.10-34.2,9979.1010. 10400 =
124412,581 J.mol-1 = 124,413 kJ.mol-1.
0,25
*) Thấy Δo([Fe(CN)6]4−) > P nên phức Fe(CN)6]4− là phức spin thấp.
còn Δo([Fe(H2O)6]2+) < P nên [Fe(H2O)6]2+ là phức spin cao.
*) Năng lượng làm bền của
Fe(CN)6]4− là CFSE1 = 6.(-2/5. 392,378) + (3-1).229,1 = -483,5072 kJ.mol-1
0,25
[Fe(H2O)6]2+ là CFSE2 = 4.(-2/5.124,413) + 2.(3/5.124,413) + (1-1).229,1 = -49,7652
kJ.mol-1.
3).
a)

0,75

b)
Nếu phức biến dạng theo trục z kiểu rút ngắn liên kết thì theo vế trái của giản đồ trên; còn
nếu kéo dãn thì theo nửa phải của giản đồ. Nên
*) [CrCl6]4- là phức spin cao có cấu hình e kiểu (t2g)3(eg)1 có 0,25
Năng lượng làm bền khi biến dạng rút ngắn là: E1 = 1.(-21/3) + 2.(+1/3) + 1.(-2/2) = -2/2
Năng lượng làm bền khi biến dạng kéo dãn là: E2 = 2.(-1/3) + 1.(+2.1/3) + 1.(-2/2) = -2/2.
Phức chất [CrCl6]4- có xu hướng bị biến dạng. Cả hai xu hướng kéo dài và rút ngắn đều có
ưu thế như nhau.
*) [Mn(CN)6]4- là phức spin thấp có cấu hình e kiểu (t2g)5 có
Năng lượng làm bền khi biến dạng rút ngắn là: E1 = 2.(-21/3) + 3.(+1/3) = -1/3
Năng lượng làm bền khi biến dạng kéo dãn là: E2 = 4.(-1/3) + 1.(+2.1/3) = -22/2. 0,25
Phức chất [Mn(CN)6] có xu hướng bị biến dạng kéo dãn vì thuận lợi hơn về mặt năng
4-

lượng.
*) [Mn(H2O)6]2+ là phức spin cao có cấu hình e kiểu (t2g)3(eg)2 có
Năng lượng làm bền khi biến dạng rút ngắn là: E1 = 1.(-21/3) + 2.(+1/3) + 1.(-2/2) + 1.(2/2)
=0
Năng lượng làm bền khi biến dạng kéo dãn là: E2 = 2.(-1/3) + 1.(+2.1/3) + 1.(-2/2) + 1.(2/2)
= 0 Phức chất [Mn(H2O)6]2+ không có xu hướng bị biến dạng. Vì khi biến dạng năng 0,25
lượng không thấp hơn ban đầu.

10
Câu 5 (2,5 điểm) Đại Cương
4) Phản ứng thủy phân ester CH3COOC2H5 trong kiềm là một phản ứng bậc hai với biểu thức vận tốc có
dạng v = k[ester][OH-].Có hai cơ chế thỏa mãn biểu thức vận tốc này. Hãy đề nghị hai cơ chế và cho
biết cơ chế nào ưu tiên hơn trong hai cơ chế đã đưa.
5) Hãy sắp xếp các cấu trúc sau đây theo thứ tự tăng dần tính bền liên kết amide C-N.

6) Tốc độ thủy phân dẫn xuất 7-norbornyl tosylate tăng lên khi có mặt nối đôi

c) Hãy giải thích ảnh hưởng của nối đôi đến sự tăng tốc dung môi phân
d) Khi đưa một nhóm p-methoxyphenyl vào vị trí 7 cũng làm tăng vận tốc phản ứng nhưng không
quá nhiều. Giải thích?

Nội Dung Điểm


1 0,75

0,75

3a. 0,5

11
3b. 0,5

Câu 6 (2,5 điểm) Cơ chế phản ứng và Sơ đồ tổng hợp


1.Đề xuất cơ chế phản ứng cho các chuyển hoá:

d)

e)

f)

2. Hoàn chỉnh sơ đồ tổng hợp sau đây :

Câu Nội Dung Điểm

12
1a 0,5

1b 0,5

1c 0,5

2 1,0

13
14
Câu 7 (2,5 điểm) Xác định cấu trúc
3) A (C9H16O5) là một hợp phần thu được khi thủy phân trioxacarcinose A, một hợp chất tự nhiên có khả
năng làm thay đổi cấu trúc DNA. Thủy phân A bằng acid loãng thu được B (C7H14O4), chất này phản
ứng được với bạc nitrat trong amoniac cho C có khả năng tạo kết tủa vàng với I2/NaOH và tạo thành
hợp chất D (C6H10O6). D bị phân cắt bằng HIO4 tạo thành oxalic acid và E (C4H6O3) không bền nhiệt
và bị tách tạo acetone khi đun nóng. Xác định cấu trúc các chất chưa biết. Biết rằng A không có khả
năng tham gia phản ứng tráng gương hay làm mất màu nước brom, và sự phân cắt C bằng HIO4 cũng
dẫn đến sự tạo thành E.
4) Xử lý geraniol (3,7-dimethylocta-6-enal) bằng [CH2=NMe2]+Cl- (muối Eschenmoser) trong sự có mặt
của Et3N thu được X1 (C11H18O). Chất này dưới ảnh hưởng của xúc tác Grubbs sẽ tạo thành X2
(C7H10O) cho phản ứng tráng gương. X2 phản ứng với dạng keto của 5-isopropyl-2-methylfuran-3-ol
khi có sự hỗ trợ của LDA/THF tạo thành X3 (C15H22O3). SmI2 phản ứng với X3 để tạo thành X4
(C15H24O3) không làm mất màu nước brom, không cho phản ứng tráng gương. Thủy phân X4 thu được
4,7,8-trihydroxy-7-isopropyl-1,4-dimethyloctahydroazulen-5-one. Hãy xác định cấu trúc các chất chưa
biết.

Câu Nội Dung Điểm


7.1 1,5

7.2 1,0

15
Câu 8 (2,5 điểm) Hợp chất thiên nhiên
3. Stagonolide D (C10H14O4) dễ dàng bị thủy phân trong môi trường acid tạo thành A (C10H18O6) có
tính acid, có khả năng làm mất màu 1 mol nước Br2. Phân cắt A bằng HIO4 thu được 1 mol acetic
acid, 1 mol formic acid và diacid B (C7H10O5) nhanh chóng bị đồng phân hóa thành monoacid C.
A tác dụng với OsO4 tạo thành D, chất này chịu tác dụng của HIO4 thu được succinic acid, acetic
acid và 4 mol formic acid. Xác định cấu trúc các chất chưa biết.
4. Coronamic acid (C6H11NO2) là một aminoacid trung gian trong quá trình sinh tổng hợp coronatine
là một chất độc được cô lập từ vi khuẩn. Hydrogen hóa coronamic acid thu được ba chất A1, A2,
A3 đều có cùng công thức C6H13NO2 đều có tính quang hoạt. Có thể tổng hợp coronamic acid
bằng cách cho N2CHCOOEt tác dụng với một chất B (C8H13NO3) rồi thủy phân sản phẩm thu
được trong acid. Thủy phân B trong môi trường acid tạo thành 2-oxopentanoic acid. Hãy xác định
cấu trúc các chất chưa biết.

Câu Nội Dung Điểm


8.1 1,25

8.2 1,25

16
Người ra đề:
Phạm Văn Tuân 0398483776

17
HỘI CÁC TRƯỜNG THPT CHUYÊN KÌ THI HỌC SINH GIỎI NĂM 2023
VÙNG DUYÊN HẢI VÀ ĐỒNG BẰNG BẮC BỘ MÔN THI: HÓA HỌC LỚP 11
TRƯỜNG PT VÙNG CAO VIỆT BẮC Thời gian làm bài: 180 phút
---------------------
ĐỀ THI ĐỀ XUẤT

Câu 1. (2,5 điêm): Tốc độ phản ứng


1.1. Một chất A có thể đồng thời biến đổi thành chất B và chất C theo sơ đồ sau:
kAB, EAB, GAB
B
A kAC, EAC, G
AC
C
Với k, E, G lần lượt là hằng số tốc độ phản ứng, năng lượng hoạt động hóa và biến thiên năng lượng
Gibbs. Cho: EAB < EAC; kAB = kAC ở 298K; ΔGAB 0
(298)
= ΔGAC
0
(298)
và G của cả 2 phản ứng ít phụ thuộc
vào nhiệt độ.
Để tăng độ chọn lọc của sự tạo thành B, cần phải tăng hay giảm nhiệt độ, tại sao?
1
1.2. Cho phản ứng pha khí: N2O5 (h)→ 2NO2 (k)+ O2 (k) (1).
2
Thực nghiệm chứng tỏ rằng biểu thức định luật tốc độ của phản ứng trên có dạng v = k[N2O5] với
hằng số tốc độ k = 3,46.10-5 s-1 ở 25oC. Giả thiết phản ứng diễn ra trong bình kín ở 25oC, lúc đầu chỉ
chứa N2O5 với áp suất p(N2O5) = 0,100 atm.
a) Tốc độ đầu của phản ứng bằng bao nhiêu?
b) Tính thời gian cần thiết để áp suất tổng cộng trong bình phản ứng bằng 0,175 atm ở nhiệt độ không
đổi (25oC). Tính đạo hàm d[N2O5]/dt tại thời điểm đó.
c) Ở cùng nhiệt độ nói trên, sau bao nhiêu lâu thì khối lượng N2O5 trong bình chỉ còn lại 12,5% so với
lượng ban đầu?
d) Nếu phản ứng được viết ở dạng dưới đây, thì các giá trị tính được ở b) và c) thay đổi thế nào?
2N2O5 (h) → 4NO2 (k) + O2 (k) (2)
Gọi K(1), G (1); K(2), G (2) lần lượt là hằng số cân bằng và biến thiên năng lượng
O O

Gibbs của phản ứng (1) và (2). Ở cùng nhiệt độ và áp suất, hãy tìm biểu thức liên hệ GO(1) với
GO(2); K(1) với K(2).
1.3. Cho phản ứng pha khí : 2NO (k) + O2 (k) → 2NO2 (k) (3)
Phản ứng (3) tuân theo định luật tốc độ thực nghiệm v = k[NO] [O2].
2

Giả định rằng phản ứng không diễn ra theo một giai đoạn sơ cấp. Hãy đề nghị một cơ chế có khả
năng cho phản ứng (3) và chứng tỏ rằng cơ chế ấy phù hợp với thực nghiệm động học.

Câu 2. (2,5 điểm): Cân bằng và phản ứng trong dung dịch. Pin điện – Điện phân
1. Hãy trình bày cách thiết lập sơ đồ pin sao cho khi pin hoạt động thì xảy ra phản ứng:
H3AsO4 + NH3 → H 2 AsO−4 + NH +4
2. Tính sức điện động của pin ở điều kiện tiêu chuẩn ( E pin ).
3. Biết CH3AsO4 = 0,025 M; CNH3 = 0,010 M.
a) Tính sức điện động của pin.
b) Tính thế của từng điện cực khi hệ đạt trạng thái cân bằng.
Cho: pKai(H AsO ) = 2,13; 6,94; 11,50; pK a(NH + ) = 9, 24 (pKa = - lgKa, với Ka là hằng số phân li
3 4 4

axit).
RT
pH2 = 1 atm; ở 25 oC: 2,303 = 0, 0592.
F

Câu 3. (2,5 điểm): Nhiệt động và cân bằng hóa học


Tính chất nhiệt động của một số phân tử và ion ở trạng thái tiêu chuẩn tại 250C như sau:
C3H8 (k) O2(k) CO2(k) H2O (l) CO32- OH- (aq.)
(aq.)
H s (kJmol )
0 -1 - 103,85 0 -393,51 -285,83 - 677,14 - 229,99
0 -1 -1
S (J.K mol ) 269,91 205,138 213,74 69,91 - 56,9 - 10,75
Xét quá trình oxi hoá hoàn toàn 1 mol C3H8 (k) với O2 (k) tạo thành CO2 (k) và
H2O (l), phản ứng được tiến hành ở 250C, điều kiện tiêu chuẩn, theo 2 cách:
a) Bất thuận nghịch và b) Thuận nghịch (trong một tế bào điện hoá).
1. Tính H0, U0 , S0, G0 của phản ứng trong mỗi cách nói trên.
2. Tính nhiệt, công thể tích, công phi thể tích (tức là công hữu ích) mà hệ trao đổi với môi trường
trong mỗi cách.
3. Tính S của môi trường và S tổng cộng của vũ trụ khi tiến hành quá trình theo mỗi cách.
4. Một mô hình tế bào điện hóa khác làm việc dựa trên phản ứng oxi hoá C3H8 (k) bởi O2(k) khi có
mặt dung dịch KOH 5 M với điện cực Pt. Các loại phân tử và ion (trừ KOH) đều ở trạng thái tiêu
chuẩn. Hãy viết các nửa phản ứng ở catôt, ở anôt và phản ứng tổng cộng trong tế bào điện hoá. Nếu
từ tế bào điện hoá đó, ở 250C, ta thu được dòng điện 100 mA, hãy tính công suất cực đại có thể đạt
được.

Câu 4. (2,5 điểm): Hóa nguyên tố. Phức chất


Magie là một kim loại dễ cháy. Một khi đã được khơi mào thì rất khó có khả năng dập tắt do
nó có thể cháy được trong nước, khí cacbonđioxit và nitơ. Khi đun nóng magie kim loại trong khí
quyển N2 thì tạo thành một chất bột màu trắng hơi vàng A.Thuỷ phân A cho khí B không màu tan
trong nước tạo thành dung dịch bazơ. Phản ứng của B với dung dịch nước của anion hypoclorit tạo
thành ion clorua, nước và hợp chất C tan được trong nước. B phản ứng với hiđro peroxit cũng tạo
thành C và nước. Khi đun nóng khí không màu B với natri kim loại sẽ tạo thành hợp chất rắn D và
khí hydro. Phản ứng của D với đinitơ oxit tạo khí amoniac, natri hidroxit rắn và hợp chất rắn E. Khi
đun nóng E nó sẽ phân huỷ tạo natri kim loại và khí nitơ.
a,Viết các phản ứng tạo thành các chất A, B, C, D, và E.
b, Vẽ cấu trúc Lewis cho anion E. Chỉ ra cấu trúc cộng hưởng bền nhất.
c,Viết phương trình cho phản ứng của E với axit sunfuric để tạo ra axit hydrazoic (HN3) và natri
sunfat.

Câu 5. (2,5 điểm): Đại cương hóa hữu cơ


1. Sắp xếp (có giải thích) theo trình tự tăng dần tính axit của các chất trong từng dãy sau:
a) Axit: benzoic, phenyletanoic, 3-phenylpropanoic, xiclohexyletanoic,
1-metylxiclohexan-cacboxylic.
b) COOH COOH CH2COOH

; ; ;
N COOH N
(A) (B) (C) (D)
2. Sắp xếp (có giải thích) theo trình tự tăng dần nhiệt độ nóng chảy của các chất sau:
COOH COOH COOH

; ;
S
N
(A) (B) (C)

Câu 6. (2,5 điểm): Sơ đồ tổng hợp hóa hữu cơ. Cơ chế phản ứng hóa hữu cơ
Sildenafil (một loại thuốc tăng lực) được tổng hợp theo sơ đồ:
O O NH2NH2 1. Me2SO4, dd NaOH HNO3, H2SO4
OEt A B C
2. NaOH, H2O o
50 C, 2 h
O
o
SOCl2, DMF, toluen, 55 C, 6 h H2 (50 psi), 5% Pd/C
C o D 0 E
dd NH3, 20 C, 2 h EtOAc, 50 C, 4 h
OEt O
(N N)2CO
OH ClSO3H, SOCl2 CH3-N NH , H2O
G H I
to phßng, 18 h o
10 C, 2 h håi l- u, 2 h,- N NH

O CH3
N
OEt HN N
N NH
I E, EtOAc N
K L
to phßng, 70 h Pr

O2 S N N CH3
Sildenafil

1. Hãy hoàn chỉnh dãy phản ứng trên, biết rằng:


* Quá trình chuyển sang G có tạo thành axit sunfonic trung gian sau đó mới chuyển thành
sunfonyl clorua.
* N,N,-cacbonylđiimiđazol (CDI) là một loại tác nhân dùng để hoạt hoá axit cacboxylic cho
phản ứng thế nucleophin của nhóm cacbonyl.
2. Viết cơ chế phản ứng chuyển [I] thành K.

Câu 7. (2,5 điểm): Xác định cấu trúc các hợp chất hữu cơ
1. Hợp chất A (C10H18O) được phân lập từ một loại tinh dầu ở Việt Nam. A không làm mất màu nước
brom và dung dịch thuốc tím loãng, cũng không tác dụng với hiđro có xúc tác niken, nhưng lại tác
dụng với axit clohiđric đậm đặc sinh ra 1-clo-4-(1-clo-1-metyletyl)-1-metylxiclohexan. Hãy đề xuất
cấu trúc của A.
2. Hợp chất B (C10H20O2 ) có trong một loại tinh dầu ở Nam Mỹ. Từ B có thể tổng hợp được A bằng
cách đun nóng với axit.
a. Viết công thức cấu tạo và gọi tên B.
b. Dùng công thức cấu trúc, viết phương trình phản ứng và trình bày cơ chế đầy đủ của phản ứng tổng
hợp A.
3. Hợp chất B thường được điều chế từ C (2,6,6-trimetylbixiclo[3.1.1] hept-2-en) có trong dầu thông.
Dùng công thức cấu tạo, viết phương trình phản ứng và chỉ rõ các liên kết của C bị đứt ra.
4. Trong cây long não có hợp chất D tên là 1,7,7-trimetylbixiclo[2.2.1]heptan-2-on (hay là campho).
Viết sơ đồ các phản ứng tổng hợp D từ C và cho biết cơ chế của giai đoạn đầu.
5. Về cấu tạo hóa học, các hợp chất A, B, C và D ở trên có đặc điểm gì chung nhất? minh họa vắn tắt
đặc điểm đó trên các công thức cấu tạo của chúng.

Câu 8. (2,5 điểm): Hóa học hợp chất thiên nhiên


Apiin là một flavon-glycozit có trong cây cần tây, mùi tây. Thủy phân OH
apiin có xúc tác enzim -glycosidaza, thu được A (apigenin, công thức HO O
phân tử là C15H10O5), B (C6H12O6) và C (C5H10O5). Metyl hóa hoàn toàn 7
apiin bởi CH3I/Ag2O, sau đó thuỷ phân sản phẩm thì thu được D
5
(C17H14O5), E (C9H18O6) và F (C8H16O5). Oxi hóa E bằng CrO3/H2SO4, A (Apigenin)
OH O
thu được sản phẩm chính là axit (2S),(3S)-đimetoxisucxinic. Khi cắt mạch
Ruff C thì thu được G (C4H8O4).
Mặt khác, C chuyển hóa được theo sơ đồ dưới đây:
+ NaIO4 +
NaIO4
C MeOH/H C1 C2 C3 H C4 + C5
1. Xác định cấu trúc của B.
2. Vẽ công thức Havooc của các đồng phân có thể tồn tại của C khi ở dạng furanozơ.
3. Vẽ cấu trúc của C1, C2, C3, C4 và C5.
4. Vẽ cấu trúc của apiin, biết phần đisaccarit liên kết với nguyên tử cacbon ở vị trí số 7 của A.
Cho: E và F là các monosaccarit thuộc dãy D, có thể tồn tại ở dạng hỗn hợp các đồng phân anome;
Khi B ở dạng -piranozơ và C ở dạng -furanozơ thì đều phản ứng được với 1 đương lượng
(CH3)2CO/H2SO4; C có tính quang hoạt, còn G không có tính quang hoạt; C và G đều tham gia phản ứng
Tolenxơ.

Hết
HỘI CÁC TRƯỜNG THPT CHUYÊN KÌ THI HỌC SINH GIỎI NĂM 2023
VÙNG DUYÊN HẢI VÀ ĐỒNG BẰNG BẮC BỘ MÔN THI: HÓA HỌC LỚP 11
TRƯỜNG PT VÙNG CAO VIỆT BẮC Thời gian làm bài: 180 phút
---------------------
ĐÁP ÁN ĐỀ THI ĐỀ XUẤT

Câu 1: (2,5 điêm) Tốc độ phản ứng


1.1. Một chất A có thể đồng thời biến đổi thành chất B và chất C theo sơ đồ sau:
kAB, EAB, GAB
B
A kAC, EAC, G
AC
C
Với k, E, G lần lượt là hằng số tốc độ phản ứng, năng lượng hoạt động hóa và biến thiên năng lượng
Gibbs. Cho: EAB < EAC; kAB = kAC ở 298K; ΔGAB 0
(298)
= ΔGAC
0
(298)
và G của cả 2 phản ứng ít phụ thuộc
vào nhiệt độ.
Để tăng độ chọn lọc của sự tạo thành B, cần phải tăng hay giảm nhiệt độ, tại sao?
1
1.2. Cho phản ứng pha khí: N2O5 (h)→ 2NO2 (k)+ O2 (k) (1).
2
Thực nghiệm chứng tỏ rằng biểu thức định luật tốc độ của phản ứng trên có dạng v = k[N 2O5] với
hằng số tốc độ k = 3,46.10-5 s-1 ở 25oC. Giả thiết phản ứng diễn ra trong bình kín ở 25oC, lúc đầu chỉ
chứa N2O5 với áp suất p(N2O5) = 0,100 atm.
a) Tốc độ đầu của phản ứng bằng bao nhiêu?
b) Tính thời gian cần thiết để áp suất tổng cộng trong bình phản ứng bằng 0,175 atm ở nhiệt độ không
đổi (25oC). Tính đạo hàm d[N2O5]/dt tại thời điểm đó.
c) Ở cùng nhiệt độ nói trên, sau bao nhiêu lâu thì khối lượng N2O5 trong bình chỉ còn lại 12,5% so với
lượng ban đầu?
d) Nếu phản ứng được viết ở dạng dưới đây, thì các giá trị tính được ở b) và c) thay đổi thế nào?
2N2O5 (h) → 4NO2 (k) + O2 (k) (2)
Gọi K(1), G (1); K(2), G (2) lần lượt là hằng số cân bằng và biến thiên năng lượng
O O

Gibbs của phản ứng (1) và (2). Ở cùng nhiệt độ và áp suất, hãy tìm biểu thức liên hệ GO(1) với
GO(2); K(1) với K(2).
1.3. Cho phản ứng pha khí : 2NO (k) + O2 (k) → 2NO2 (k) (3)
Phản ứng (3) tuân theo định luật tốc độ thực nghiệm v = k[NO] [O2].
2

Giả định rằng phản ứng không diễn ra theo một giai đoạn sơ cấp. Hãy đề nghị một cơ chế có khả
năng cho phản ứng (3) và chứng tỏ rằng cơ chế ấy phù hợp với thực nghiệm động học.

Câu 1 Đáp án Điểm


− E AB / RT
k AB = Z AB .e (1)
1.1 k AC = Z AC .e − EAC / RT (2) 0,75
Chia vế (1) cho (2) rồi lấy logarit 2 vế nhận được:
k AB Z E − E AB
ln = ln AB + AC
k AC Z AC RT
Bởi ZAB và ZAC là hằng số, EAC - EAB > 0 → Khi T thì kAB/kAC.
Như vậy, để tăng độ chọn lọc B thì cần tăng nhiệt độ.

a) Số mol có trong bình N2O5:


n(N2O5) = pV/RT = 0,10.atm.V (L) /0,082L.atm.mol-1.K-1.298 K = 4,1.10-3.V
mol.
n( N 2O5 ) p ( N 2O5 ) 0,1 0,25
[N2O5] = = = (mol / L) = 4,1.10-3 mol/L
V RT 0, 082.298
v = 3,46.10-5 s-1. 4,1.10-3.mol/L = 1,42.10-7 mol.L-1.s-1

b)
N2O5 → 2NO2 + (1/2)O2
Po 0 0
Po -x 2x x/2
Ptổng = Po -x + 2x + x/2 = Po +(3/2)x = (7/4)Po→ x = Po/2 và Po - x = Po/2.
Ở cùng nhiệt độ, khi thể tích bình phản ứng không thay đổi, sự giảm áp suất
riêng phần tỉ lệ với sự giảm số mol. Trong phản ứng bậc 1, thời gian cần thiết
để nồng độ chất phản ứng giảm đi một nửa bằng:
t1/2 = ln2/k = 0,693/3,46.10-5 s-1 = 2.104 s
0,25
Trong phản ứng bậc 1, khi nồng độ giảm đi một nửa (so với ban đầu) thì tốc độ
phản ứng cũng giảm đi một nửa
1.2 d [N 2 O5 ]
v(t = t1/2) = − = (1/2)v(ban đầu)
dt
d[N 2 O5 ]
→ = - (1/2)v(ban đầu) = -7,1.10-8 mol.L-1.s-1
dt

c) Thời gian phản ứng bán phần của phản ứng bậc 1 không phụ thuộc vào nồng
độ đầu. Để khối lượng N2O5 còn lại 12,5% (1/8 nồng độ đầu) cần thời gian 3
lần thời gian phản ứng bán phần: 0,25
t = 3.2.104 s = 6.104 s

d [N 2 O5 ]
d) Vì tốc độ phân hủy N2O5, biểu thị bởi − không đổi nên các giá trị
dt
trên vẫn không đổi.
Học sinh có thể suy luận cách khác, nếu đúng vẫn được đủ điểm.
* So sánh 0,25
p*2NO2 p*1/2
O2 p*4NO2 p*O2
K(1)= ; K(2)= ; → K(1) = K(2)1/2.
p* N 2 O 5 p*2N 2O5
(p* là áp suất riêng phần ở trạng thái cân bằng)
GO(1) = -RTlnK(1) = -(1/2)RTlnK(2);
GO(2) = -RTlnK(2) → GO(1) = (1/2)GO(2)

Phản ứng có thể xảy ra theo cơ chế hai giai đoạn:


k1
2NO k−1
N2O2 (a) (nhanh)

N2O2 + O2 ⎯⎯ k2
→ 2NO2 (b) (chậm)
Cộng (a) với (b) sẽ thu được phản ứng tổng cộng (3).
Giai đoạn (b)chậm, quyết định tốc độ chung của phản ứng, nên:
v = k2[N2O2][ O2] (*)
Do giai đoạn (b) chậm và (a) nhanh nên có thể coi cân bằng (a) được thiết lập,
1.3 khi đó có: 0,75
[N2O2]/[NO]2 = k1/k-1 → [N2O2] = (k1/k-1)[NO]2 (2*)
Thay (2*) vào (*) thu được:
v = (k1/k-1)k2[NO]2[ O2] = k[NO]2[ O2] với k = (k1/k-1)k2.
Như vậy từ cơ chế giả định có thể rút ra được định luật tốc độ thực nghiệm. Cơ
chế là có khả năng.
Chú ý: Thí sinh có thể đưa ra cơ chế khác. Nếu chứng minh chặt chẽ rằng
cơ chế đó phù hợp với thực nghiệm thì cho đủ điểm.

Câu 2: (2,5 điểm) Cân bằng và phản ứng trong dung dịch. Pin điện – Điện phân
1. Hãy trình bày cách thiết lập sơ đồ pin sao cho khi pin hoạt động thì xảy ra phản ứng:
H3AsO4 + NH3 → H 2 AsO−4 + NH +4
2. Tính sức điện động của pin ở điều kiện tiêu chuẩn ( E pin ).
3. Biết CH3AsO4 = 0,025 M; CNH3 = 0,010 M.
a) Tính sức điện động của pin.
b) Tính thế của từng điện cực khi hệ đạt trạng thái cân bằng.
Cho: pKai(H3AsO4 ) = 2,13; 6,94; 11,50; pK a(NH + ) = 9, 24 (pKa = - lgKa, với Ka là hằng số phân li
4

axit).
RT
pH2 = 1 atm; ở 25 oC: 2,303 = 0, 0592.
F

Câu 2 Đáp án Điểm


1. Phản ứng xảy ra trong pin được tổ hợp từ các cân bằng sau:
H3AsO4 ⎯ ⎯⎯ → H+ + H 2 AsO-4

⎯⎯
→ NH +4
NH3 + H+ ⎯
⎯ 0,75

⎯⎯
→ H 2 AsO-4 + NH +4
H3AsO4 + NH3 ⎯
⎯ K (*)
Như vậy các cân bằng trên đều liên quan đến quá trình cho - nhận H+, do đó
có thể chọn điện cực hiđro để thiết lập pin. Vì giá trị thế của điện cực hiđro
( E 2H + /H ) phụ thuộc vào [H+]:
2

0,0592 [H + ]2
E 2H+ /H = lg
2 2 pH2
nên điện cực platin nhúng trong dung dịch H3AsO4 (có [H+] lớn hơn) có thế dương
hơn, sẽ là catot. Ngược lại điện cực platin nhúng trong dung dịch NH3 sẽ là anot.
Vậy ta có sơ đồ pin:
(-) Pt(H2) │ NH3(aq) ║ H3AsO4(aq) │ Pt (H2) (+)
p H 2 = 1atm p H 2 = 1atm

2. Quá trình oxi hóa xảy ra trên anot:


⎯⎯
H2 ⎯ → 2H+ + 2e
⎯ K=1
2 ⎯⎯
→ NH +4
NH3 + H+ ⎯
⎯ (K -1 2
a ) = (10
9,24 2
)
0
2 NH3 + H2 ⎯ ⎯⎯→ 2 NH +4 + 2e
⎯ K1 =10-2.Ea /0,0592 (1)
9,24 . 2 . 0,0592
→ E0a = = - 0,547 (V)
-2
Quá trình khử xảy ra trên catot:
2 H3AsO4 ⎯ ⎯⎯→ H+ + H 2 AsO-4
⎯ (K a1 ) 2 = (10-2,13)2 0,75
⎯⎯
→ H2
2H+ + 2e ⎯
⎯ K=1
0
2H3AsO4 + 2e ⎯ ⎯⎯⎯→ H2 + 2 H 2 AsO-4 K 2 =102.Ec /0,0592 (2)
-2,13 . 2 . 0,0592
→ E0c = = - 0,126 (V)
2
Vậy E pin = E0c - E0a = 0,421 (V).
(Hoặc từ (*) ta có: K = Ka1.(Ka)-1 = 10E/0,0592 → E0pin = E = 0,421 (V))

3. Do sự phân li của nước trong dung dịch NH3 0,010 M và trong dung dịch
H3AsO4 0,025 M không đáng kể, nên:
a) Tại dung dịch của nửa pin trái:
NH3 + H2O ⎯ ⎯⎯ → NH +4 + OH-
⎯ Kb = 10-4,76
[ ] 0,010-x x x
→ [ NH 4 ] = [OH ] = x = 4,08.10 (M);
+ - -4

[NH3] = 9,59.10-3 (M); [H+] = 2,45.10-11 (M)


1.0
0, 0592 [NH +4 ]2
Từ (1), ta có: Ea = E0a + lg
2 [NH3 ]2 .pH 2

2
0, 0592  4, 08.10−4 
= 1atm nên: Ea = -0,547 + lg 
 9,59.10−3 
Vì pH2 = - 0,63 (V)
2  
(Hoặc Ea = 0,0592.lg[H+])
Đối với H3AsO4, vì Ka1 Ka2 Ka3 nên tại dung dịch của nửa pin phải:
H3AsO4 H+ + H 2 AsO-4 Ka1 =10-2,13
[ ] 0,025-x x x
→ [ H 2 AsO4 ] = [H ] = x = 0,0104 (M); [H3AsO4] = 0,0146 (M)
- +

0, 0592 [H3AsO4 ]2
Từ (2), ta có: Ec = E0c + lg
2 [H 2 AsO4− ]2 .pH 2
2
0, 0592  0, 0146 
→ Ec = -0,126 + lg    - 0,12 (V)
2  0, 0104 
(Hoặc Ec = 0,0592.lg[H+])
→ Epin = - 0,12 + 0,63 = 0,51 (V)
b) Khi hệ đạt trạng thái cân bằng thì thế của 2 điện cực bằng nhau: Ec = Ea
H3AsO4 + NH3 ⎯⎯→ H 2 AsO-4 + NH +4 K = 107,11
0,025 0,010
0,015 - 0,010 0,010
+ -
Hệ thu được gồm: NH 4 0,010 M; H 2 AsO4 0,010 M; H3AsO4 0,015 M. Do
sự phân li của NH +4 và của nước không đáng kể, do đó pH của hệ được tính theo
cân bằng:
H3AsO4 H+ + H 2 AsO-4 K a1 =10-2,13
[ ] 0,015-x x 0,010+x
→ [H+] = x = 4,97.10 3 (M); [H3AsO4]  0,010 (M); [ H 2 AsO-4 ]  0,015
-

(M).
0, 0592 [H3AsO4 ]2
→ Ea = Ec = E0c + lg
2 [H 2 AsO4− ]2 .pH 2

2
0, 0592  0, 01 
= - 0,126 + lg    - 0,136 (V)
2  0, 015 
(Hoặc Ea = Ec = 0,0592.lg[H+])
Câu 3. (2,5 điểm) Nhiệt động và cân bằng hóa học
Tính chất nhiệt động của một số phân tử và ion ở trạng thái tiêu chuẩn tại 250C như sau:
C3H8 (k) O2(k) CO2(k) H2O (l) CO32- OH- (aq.)
(aq.)
H s (kJmol )
0 -1 - 103,85 0 -393,51 -285,83 - 677,14 - 229,99
0 -1 -1
S (J.K mol ) 269,91 205,138 213,74 69,91 - 56,9 - 10,75
Xét quá trình oxi hoá hoàn toàn 1 mol C3H8 (k) với O2 (k) tạo thành CO2 (k) và
H2O (l), phản ứng được tiến hành ở 250C, điều kiện tiêu chuẩn, theo 2 cách:
a) Bất thuận nghịch và b) Thuận nghịch (trong một tế bào điện hoá).
1. Tính H0, U0 , S0, G0 của phản ứng trong mỗi cách nói trên.
2. Tính nhiệt, công thể tích, công phi thể tích (tức là công hữu ích) mà hệ trao đổi với môi trường
trong mỗi cách.
3. Tính S của môi trường và S tổng cộng của vũ trụ khi tiến hành quá trình theo mỗi cách.
4. Một mô hình tế bào điện hóa khác làm việc dựa trên phản ứng oxi hoá C3H8 (k) bởi O2(k) khi có
mặt dung dịch KOH 5 M với điện cực Pt. Các loại phân tử và ion (trừ KOH) đều ở trạng thái tiêu
chuẩn. Hãy viết các nửa phản ứng ở catôt, ở anôt và phản ứng tổng cộng trong tế bào điện hoá. Nếu
từ tế bào điện hoá đó, ở 250C, ta thu được dòng điện 100 mA, hãy tính công suất cực đại có thể đạt
được.

Câu 3 Đáp án Điểm


Tính H , U , S , G của phản ứng
0 0 0 0

C3H8(k) + 5O2(k) -> 3CO2(k) + 4H2O(l)


H0 (pư) = - 2220,00 kjmol-1; S0 (pư) = - 374,74 JK-1mol-1;
U0 (pư) = H0 (pư) - (pV) = H0 - (n khí RT) =
- 2220,00 .103 jmol-1 - (-3mol . 8,3145 JK-1mol-1 . 298,15K )
= - 2220,00 . 103 jmol-1 + 7436,90 jmol-1 . U0 = - 2212,56. 103 jmol-1.
1
G0 = H0 - T S0 = [- 2220,00 . 103 - (298,15) . (-374,74) ]Jmol-1
G0 = - 2108,33 kjmol-1.
Vì H, U, S, G là các hàm trạng thái của hệ nên dù tiến hành theo cách thuận 0,5
nghịch hay bất thuận nghịch mà trạng thái đầu và trạng thái cuối của hai cách
giống nhau thì các đại lượng H, U, S, G cũng vẫn bằng nhau.

Tính nhiệt, công thể tích, công phi thể tích mà hệ trao đổi.
a) Quá trình bất thuận nghịch
- Nhiệt trao đổi của hệ q = H0
- Công thể tích Wtt = -pdV = -pV = - nkRT
nk = - 3 mol -> Wtt = -(3mol) . 8,3145 JK-1mol-1 . 298,15K = +7436,90 Jmol-1 0,5
>0 hệ nhận công.
2
- Công phi thể tích = 0
b) Quá trình thuận nghịch.
- Tổng năng lượng mà hệ trao đổi với môi trường là H0 trong đó nhiệt trao đổi
là :
TS = 298,15K . (-374,74) JK-1mol-1) = -111,29 KJmol-1.
- Công thể tích: Wtt = -nkRT = + 7436,90 Jmol-1 > 0 hệ nhận công
- Công phi thể tích cực đại: W' = G0 = - 2108,33 KJmol-1 <0 hệ sinh công 0,5
Tính S của môi trường và S tổng cộng.
a) Quá trình bất thuận nghịch Shệ = -374,74JK-1mol-1.
Smôi trường = qmt/T = - Hhệ /T = 2220.103jmol-1/298,15 = 7445,92 JK-1mol-1 0,25
Stổng cộng(vũ trụ) = Hhệ + Smôi trường =
= 7071,18 JK-1mol-1 > 0 => phản ứng tự phát.
3
b) Quá trình thuận nghịch
Shệ = -374,74JK-1mol-1.
Smôi trường = qmt/T = - qhệ /T = 0,25
= +111,7287 KJmol-1/298,15K = + 374,74 JK-1mol-1.
Stổng cộng(vũ trụ) = Hhệ + Smôi trường = 0.
Các nửa phản ứng
anốt: C3H8 + 26OH- - 20e -> 3CO32- + 17 H2O
catốt: O2+ 2H2O + 4e -> 4OH-

C3H8(k) + 5O2 (k) + 6OH- (aq) -> 3CO32- + 7H2O (l)


(L, anốt ) : Pt C3H8 | KOH , K2CO3 | O2 | Pt (catốt, R)
Công suất cực đại: P = IE I = 0,1A
E0 = - G0 / F
G0 (pư) = H0 (pư) - TS0 (pư)
H0 (pư) = [ 3 H0s (CO32-) + 7H0s (H2O)] - [H0s (C3H8) + 6H0s (OH-)]
4 H0 (pư) = -2548,44 KJ mol-1 0,5
S0 (pư) = [ 3 S0 (CO32-) + 7 S0(H2O)] - [ S0(C3H8) + 5 S0(O2) + 6S0(OH -)]
S0 (pư) = - 912,43 J K-1mol-1.
G0 (pư) = - 2548,44 .103 J mol-1 - 298,15 K . ( -912,43 J K-1mol-1)
G0 (pư) = - 2276399 J mol-1.
-1
2276399 J mol
E = - G / F = +
0 0
= 1,18 (V)
20 . 96485 Cmol -1
0,0592 0,0592
E = E0 + log[ OH − ]6 = 1,18 + log 5 6 = 1,19(V )
20 20
P = I.E = 0,1 A x 1,19 V = 0,119 W.
Câu 4. (2,5 điểm) Hóa nguyên tố. Phức chất
Magie là một kim loại dễ cháy. Một khi đã được khơi mào thì rất khó có khả năng dập tắt do
nó có thể cháy được trong nước, khí cacbonđioxit và nitơ. Khi đun nóng magie kim loại trong khí
quyển N2 thì tạo thành một chất bột màu trắng hơi vàng A.Thuỷ phân A cho khí B không màu tan
trong nước tạo thành dung dịch bazơ. Phản ứng của B với dung dịch nước của anion hypoclorit tạo
thành ion clorua, nước và hợp chất C tan được trong nước. B phản ứng với hiđro peroxit cũng tạo
thành C và nước. Khi đun nóng khí không màu B với natri kim loại sẽ tạo thành hợp chất rắn D và
khí hydro. Phản ứng của D với đinitơ oxit tạo khí amoniac, natri hidroxit rắn và hợp chất rắn E. Khi
đun nóng E nó sẽ phân huỷ tạo natri kim loại và khí nitơ.
a,Viết các phản ứng tạo thành các chất A, B, C, D, và E.
b, Vẽ cấu trúc Lewis cho anion E. Chỉ ra cấu trúc cộng hưởng bền nhất.
c,Viết phương trình cho phản ứng của E với axit sunfuric để tạo ra axit hydrazoic (HN3) và natri
sunfat.
Câu 4 Đáp án Ðiểm
a. A là Mg3N2 B là NH3 C là N2H4 D là NaNH2 E là NaN3 0,25
3Mg + N → Mg N
(r) 2(k) 3 2(r)
Mg N + 6H O → 3Mg(OH) + 2NH
3 2(r) 2 (l) 2(r) 3(k)
2NH (l) + OCl− → Cl− + H O + N H
3(k) (l) (l) 2 (l) 2 4(l)
2NH +H O →N H + 2H O 1,5
3(k) 2 2(l) 2 4(l) 2 (l)
2NH + Na → 2NaNH +H
3(k) (r) 2(r) 2(k)
2NaNH +N O → NH + NaOH + NaN
2(r) 2 (k) 3(k) (r) 3(r)
b, Các cấu trúc Lewis của ion azit của E là:
(-) (+) (-) (+) (2-) (2-) (+)
:N N N: :N N N: :N N N:
↔ ↔
:
:

:
:

0,5
(1) (2) (3)
Cấu trúc (1) là bền nhất
c. 2 NaN3+ H2SO4 → 2 HN3 + Na2SO4 0,25
Câu 5. (2,5 điểm) Đại cương hóa hữu cơ
1. Sắp xếp (có giải thích) theo trình tự tăng dần tính axit của các chất trong từng dãy sau:
a) Axit: benzoic, phenyletanoic, 3-phenylpropanoic, xiclohexyletanoic,
1-metylxiclohexan-cacboxylic.
b) COOH COOH CH2COOH

; ; ;
N COOH N
(A) (B) (C) (D)
2. Sắp xếp (có giải thích) theo trình tự tăng dần nhiệt độ nóng chảy của các chất sau:
COOH COOH COOH

; ;
S
N
(A) (B) (C)

Câu 5 Nội dung Điểm


a)
+I 2
H3C COOH CH2COOH CH2COOH COOH
+I 1 -I1CH2CH2COOH -I 2 -I 3
< < < < 0,75

+I 1
<
+I 2 -I 1 < -I 2 < -I 3
Các gốc hiđrocacbon có hiệu ứng +I lớn thì Ka giảm và -I lớn thì Ka
b)
1. CH2COOH COOH
-I 1 COOH
-I 2 -I 4
< C O <
< -C3 N -I 3
H O N -C4
(D) (C) (A) 0,75
(B)
Vì: - I1 < - I2 nên (C) có tính axit lớn hơn (D).
(A) và (B) có N nên tính axit lớn hơn (D) và (C)
(A) có liên kết hiđro nội phân tử làm giảm tính axit so với (B).

Tăng dần nhiệt độ nóng chảy của các chất:


COOH COOH COOH Vì:
M C < MA.
2. < < 1,0
S (B) có thêm liên kết hiđro
N
(C) (A) liên phân tử với N của
(B)
phân tử khác.
Câu 6. (2,5 điểm) Sơ đồ tổng hợp hóa hữu cơ. Cơ chế phản ứng hóa hữu cơ
Sildenafil (một loại thuốc tăng lực) được tổng hợp theo sơ đồ:
O O NH2NH2 1. Me2SO4, dd NaOH HNO3, H2SO4
OEt A B C
2. NaOH, H2O o
50 C, 2 h
O
o
SOCl2, DMF, toluen, 55 C, 6 h H2 (50 psi), 5% Pd/C
C o D 0 E
dd NH3, 20 C, 2 h EtOAc, 50 C, 4 h
OEt O
(N N)2CO
OH ClSO3H, SOCl2 CH3-N NH , H2O
G H I
to phßng, 18 h o
10 C, 2 h håi l- u, 2 h,- N NH

O CH3
N
OEt HN N
N NH
I E, EtOAc N
K L
to phßng, 70 h Pr

O2 S N N CH3
Sildenafil

1. Hãy hoàn chỉnh dãy phản ứng trên, biết rằng:


* Quá trình chuyển sang G có tạo thành axit sunfonic trung gian sau đó mới chuyển thành
sunfonyl clorua.
* N,N,-cacbonylđiimiđazol (CDI) là một loại tác nhân dùng để hoạt hoá axit cacboxylic cho
phản ứng thế nucleophin của nhóm cacbonyl.
2. Viết cơ chế phản ứng chuyển [I] thành K.

Câu 6 Đáp án Điểm


O O O
H CH3 CH3
O O NH2NH2
OEt EtO N 1. Me2SO4, dd NaOH HO N HNO3, H2SO4 HO N
N 2. NaOH, H2O N 50oC, 2 h N
O
O2N
A Pr B Pr C Pr
O O 1,0
CH3 CH3
o H2 N N H2 (50 psi), 5% Pd/C H2N N
SOCl2, DMF, toluen, 55 C, 6 h
C o
N 0
EtOAc, 50 C, 4 h
N
dd NH3, 20 C, 2 h
O2N H2N
D Pr E Pr
OEt O OEt O OEt O

OH ClSO3H, SOCl2 OH CH3-N NH , H2O OH


to phßng, 18 h o
10 C, 2 h

G SO2Cl H O2 S N N CH3
OEt O O
OEt O
(N N)2CO O N N N NH
H N N
håi l- u, 2 h , -N NH
Me
O2 S N N CH3
I K O2 S N N CH3 1,0

O CH3
OEt H2N N
O N
K E, EtOAc
to phßng, 70 h N
Pr
H

L O2 S N N CH3

2. Cơ chế từ [I] sang K


(- )
O
OEt O O OEt O
OEt O
C C
O N N O N N
N N N 0,5
HN N - H+ N + H+
+ CO2 + HN N
O2 S O2 S
N N CH3 N N CH3 O2 S N N CH3
Câu 7. (2,5 điểm): Xác định cấu trúc các hợp chất hữu cơ
1. Hợp chất A (C10H18O) được phân lập từ một loại tinh dầu ở Việt Nam. A không làm mất màu nước
brom và dung dịch thuốc tím loãng, cũng không tác dụng với hiđro có xúc tác niken, nhưng lại tác
dụng với axit clohiđric đậm đặc sinh ra 1-clo-4-(1-clo-1-metyletyl)-1-metylxiclohexan. Hãy đề xuất
cấu trúc của A.
2. Hợp chất B (C10H20O2 ) có trong một loại tinh dầu ở Nam Mỹ. Từ B có thể tổng hợp được A bằng
cách đun nóng với axit.
a. Viết công thức cấu tạo và gọi tên B.
b. Dùng công thức cấu trúc, viết phương trình phản ứng và trình bày cơ chế đầy đủ của phản ứng tổng
hợp A.
3. Hợp chất B thường được điều chế từ C (2,6,6-trimetylbixiclo[3.1.1] hept-2-en) có trong dầu thông.
Dùng công thức cấu tạo, viết phương trình phản ứng và chỉ rõ các liên kết của C bị đứt ra.
4. Trong cây long não có hợp chất D tên là 1,7,7-trimetylbixiclo[2.2.1]heptan-2-on (hay là campho).
Viết sơ đồ các phản ứng tổng hợp D từ C và cho biết cơ chế của giai đoạn đầu.
5. Về cấu tạo hóa học, các hợp chất A, B, C và D ở trên có đặc điểm gì chung nhất? minh họa vắn tắt
đặc điểm đó trên các công thức cấu tạo của chúng.

Câu 7 Đáp án Điểm


1. Xác định công tức cấu trúc của A(C10H18O)
A có độ chưa bão hoà là 2;
A không làm mất mầu dung dịch nước brom và dung dịch thuốc tím loãng
chứng tỏ trong A không có nối đôi hay nối ba;
A không tác dụng với hiđro trên chất xúc tác niken chứng tỏ trong A
không có nhóm chức cacbonyl;
A tác dụng với axit clohiđric đậm đặc sinh ra 1-clo-4-(1-clo-1-metyletyl)-
1-metylxiclohexan, trong A có vòng no và có liên kết ete. 0,5
Suy ra công thức cấu trúc của A
CH3 CH3 CH3
H3C O
O
O
O CH3

CH3

2. a. - H2O
B (C10H20O2) A (C10H18O)
Suy ra B là một điol có bộ khung cacbon như A

OH
+
H A 0,5
H2O

OH
B
Gọi tên B: 1-hiđroxi-4-(-1-hiđroxi-1-metyletyl)-1-metylxiclohexan
b. Dùng công thức cấu trúc, viết phương trình phản ứng và trình bày cơ chế đầy
đủ của phản ứng.
Cả 2 dạng trans và cis của B đều ở cấu dạng ghế bền vững, tuy vậy cấu
dạng ghế không thể tham gia đóng vòng mà phải đi qua dạng thuyền kém bền.
Dạng thuyền sẽ tham gia phản ứng SN1 nội phân tử.

(+)
OH

OH
OH OH OH H
+

H
+ O
OH
(+)
0,5
Cis-B H
+
A

(+)

OH
OH OH
OH OH + O
H
OH
(+)

Trans-B H
+
A

3. Liên kết của C bị đứt ở các đường chấm chấm:

OH
+
2 H2O H 0,25
+

OH

4. HO O
Cl
HCl H2O O

C D
H+ 0,5
Cl-

(+) (+)
chuyÓn vÞ
5. Đặc điểm chung nhất về cấu tạo hoá học: mỗi phân tử gồm 2 đơn vị isopren
(hoặc isopentan) nối với nhau. 0,25

Câu 8. (2,5 điểm): Hóa học hợp chất thiên nhiên


Apiin là một flavon-glycozit có trong cây cần tây, mùi tây. Thủy phân OH
apiin có xúc tác enzim -glycosidaza, thu được A (apigenin, công thức HO O
phân tử là C15H10O5), B (C6H12O6) và C (C5H10O5). Metyl hóa hoàn toàn 7
apiin bởi CH3I/Ag2O, sau đó thuỷ phân sản phẩm thì thu được D
5
(C17H14O5), E (C9H18O6) và F (C8H16O5). Oxi hóa E bằng CrO3/H2SO4, A (Apigenin)
OH O
thu được sản phẩm chính là axit (2S),(3S)-đimetoxisucxinic. Khi cắt mạch
Ruff C thì thu được G (C4H8O4).
Mặt khác, C chuyển hóa được theo sơ đồ dưới đây:
+ NaIO4 +
NaIO4
C MeOH/H C1 C2 C3 H C4 + C5
1. Xác định cấu trúc của B.
2. Vẽ công thức Havooc của các đồng phân có thể tồn tại của C khi ở dạng furanozơ.
3. Vẽ cấu trúc của C1, C2, C3, C4 và C5.
4. Vẽ cấu trúc của apiin, biết phần đisaccarit liên kết với nguyên tử cacbon ở vị trí số 7 của A.
Cho: E và F là các monosaccarit thuộc dãy D, có thể tồn tại ở dạng hỗn hợp các đồng phân anome;
Khi B ở dạng -piranozơ và C ở dạng -furanozơ thì đều phản ứng được với 1 đương lượng
(CH3)2CO/H2SO4; C có tính quang hoạt, còn G không có tính quang hoạt; C và G đều tham gia phản ứng
Tolenxơ.

Câu 8 Đáp án Điểm


1. Từ các dẫn xuất metyl, E và F, suy được thứ tự liên kết của các monosaccrit: C-
B-Apigenin.
Xác định B.
Kết hợp điều kiện B ở dạng -piranozơ phản ứng được với 1 đương lượng axeton
và sản phẩm oxi hóa E là axit (2S),(3S)-đimetoxisucxinic, suy ra B có thể là
D-glucozơ, D-sorbozơ,…

CH2OH 0,75
O
H OH
HO H
H OH
CH2OH
D-Sorbose
Khi B là D-glucozơ: Hai nhóm cacboxyl được tạo thành do oxi hóa nhóm OH ở C2 và
OH ở C5 của E. Do vậy, C liên kết với B qua vị trí 2.

MeO CHO
HO COOH
H OH
H O H H O H
MeO H MeO
s H
OH H OMe H H OMe H s OMe
HO OH MeO OH H OH
H OH H OH COOH
CH2OMe
-D-Glucopiranose E (-anome) Axit (2S),(3S)-§ imetoxisucxinic

2. Xác định đường C. Theo đầu bài C là monosaccarit dãy D, có tính quang hoạt,
khi cắt mạch Ruff cho G không quang hoạt, suy ra C là một D-andotetrozơ, dạng β
có nhánh CH2OH (apiozơ).

CHO
2 CHO
H OH Thoái phân Ruff
3 CH2OH
HO CH2OH HO

CH2OH CH2OH

(C) (G)

Nhóm CHO có thể nối với mỗi nhóm CH2OH, cho 2 dạng vòng furanozơ. Mỗi dạng
lại có 2 đồng phân: α- và β-anome. Như vậy, về mặt lí thuyết khi ở dạng furanozơ,
0,75
C có thể tồn tại 4 đồng phân như sau (Dãy D: OH ở C2 nằm bên phải ở công thức
Fisơ, và nằm dưới mặt phẳng vòng ở công thức Haworth):

O O OH O O OH

CH2OH CH2OH OH OH
OH OH
OH OH OH OH H2COH OH H2COH OH
-Anome -Anome -Anome -Anome
(1) (2) (3) (4)
Trong số 4 dạng trên chỉ có 2 dạng (1) và (2) có nhóm CH2OH ở phía trên của vòng
là đảm bảo dữ kiện của đề bài.

3. Công thức của C, C1, C2, C3, C4 và C5.


0,5
OH OCH3 O OCH3 OCH3
O O O
HOCH2-COOH
CH2OH + CH2OH NaIO4 +
H C4
MeOH/H NaIO4 O +
CHO COOH CHO
OH OH OH OH CH2OH OHC-CHO
C5
-Anome C1 C2 C3

4. Khi B là D-glucozơ thì công thức của Apiin là

OH OH
O
HO 2 O
O
HO 7
O O

CH2OH
2' OH O
OH OH Apiin

0,5
Khi B là D-sorbozơ thì “apiin” có công thức sau (mặc dù chưa tìm thấy chất này tồn
tại ở dạng glycozit trong thiên nhiên).
OH
O
HO 3 O O
HO 7
O O
CH2OH 5
CH2OH OH O
2'
OH OH
"Apiin"
ĐÁP ÁN GIỚI THIỆU KÌ THI CHỌN HỌC SINH GIỎI
THPT Chuyên Lam Sơn THPT CHUYÊN – DUYÊN HẢI BẮC BỘ NĂM
2023
MÔN: HOÁ HỌC - LỚP 11
Thời gian làm bài: 180 phút
Câu 1 (2,5 điểm) Tốc độ phản ứng.
1.1. Phản ứng thế phối tử trong phức chất [ML5]2− bằng tác nhân X trong môi trường acid
yếu xảy ra theo phương trình phản ứng:
3X+[ML5]2- →[ML3X2] +2L- (1)
Kí hiệu [ML5]2- là M và [ML3X2] là N
Quy luật động học của phản ứng được viết như sau: 𝑣 =𝑘 𝐶Ha 𝐶Mb 𝐶Xc , trong đó a,b,c lần lượt
là bậc riêng phần của phản ứng theo H+, M và X. Để nghiên cứu động học của phản ứng trên,
người ta thực hiện các thí nghiệm 1, 2 và 3. Các thí nghiệm này chỉ khác nhau ở nồng độ
đầu của các chất và pH, còn các điều kiện khác được giữ ổn định như nhau:
a. Thí nghiệm 1 và 2 được tiến hành đồng thời ở pH ổn định bằng 5,0; nồng độ đầu của M là 2,5
mmol L-1, nhưng nồng độ đầu của X (C0 (X)) khác nhau. Hãy xác định bậc riêng phần của phản
ứng đối với M và X dựa vào kết quả thực nghiệm được biểu diễn trong đồ thị ở hình dưới đây:

b. Trong thí nghiệm 3, nồng độ của các chất trong hỗn hợp phản ứng được xác định bằng phương
pháp đo độ hấp thụ quang. 𝐶0 là nồng độ M tại tời điểm 𝑡=0. Ở một bước sóng xác định: gọi 𝜀1 , 𝜀2
và 𝜀3 lần lượt là hệ số hấp thụ mol của X, M và N; A0, At, A∞ lần lượt là độ hấp thụ quang của
dung dịch tại thời điểm t=0, thời điểm t và thời điểm phản ứng kết thúc t∞. Độ hấp thụ quang của
dung dịch tuân theo biểu thức: 𝐴=∑𝑖 𝜀i .𝑙.𝐶i , trong đó 𝑙 là hằng số.
b1) Thiết lập biểu thức tính hằng số tốc độ theo A0, At, A∞, C0, t nếu X và M được lấy theo đúng
hệ số tỉ lượng trong phương trình phản ứng, bậc của phản ứng không thay đổi so với thí nghiệm ở ý
a.
b2) Trong điều kiện: 𝐶0 (M)=5,0 mmol L−1;𝐶0 (X)=15,0 mmol L-1, pH ổn định ở 5,5; kết quả
đo độ hấp thụ quang của dung dịch hỗn hợp phản ứng theo thời gian như sau:
t (phút) 0 5 10 15 20 ∞
Độ hấp thụ quang 0,870 0,796 0,738 0,693 0,655 0,33
Viết phương trình luật tốc độ đầy đủ cho phản ứng (1) và xác định hằng số tốc độ của phản ứng.
1.2. Sự phân hủy bằng xúc tác của chất A trên bề mặt xúc tác phù hợp, sinh ra các sản phẩm B, C, D
theo sơ đồ dưới đây, phù hợp với động học của phản ứng song song bậc nhất.
A ⎯⎯ k1
→ B ; A ⎯⎯k2
→ C ; A ⎯⎯k3
→D
Sau 10 giây tính từ khi bắt đầu phản ứng tại T K, nồng độ của các thành phần trong hỗn hợp là: CA
= 0,032 mol/lít; CB = 0,068 mol/lít; CC = 0,092 mol/lít; CD = 0,016 mol/lít
a. Nồng độ ban đầu C0 của A trong hệ bằng bao nhiêu?
b. Tính hằng số tốc độ k của quá trình A ⎯⎯ k
→ sản phẩm. Tính khoảng thời gian  1/2 trong đó nồng
độ của A giảm xuống còn c = c0/2?
c. Giá trị hằng số k1, k2 và k3 bằng bao nhiêu?
d. Giá trị nồng độ CB, CC, CD tại thời điểm t =  1/2 ?
Hướng dẫn

Câu Nội dung Điểm


1.1 a.
(1,5 đ) Trong thí nghiệm 1 và 2, H+ ổn định. Dựa vào đồ thị, ta thấy ln[M] trải dài từ
-24 đến -5, vậy [M] nằm trong khoảng từ 3,77.10-11 M < [M] < 6,378.10-3 M.
Khoảng giá trị này rất nhỏ so với nồng độ của X (0,5 M ở thí nghiệm 1 và
0,2 M ở thí nghiệm 2), do đó có thể xem thí nghiệm 1 và 2 là thí nghiệm giả
bậc theo M. Nghĩa là tốc độ phản ứng có thể đơn giản hóa thành: v = 0,25
k’⋅[M]b với k’ = k[H+]a [X]c
Ta thấy đồ thị liên hệ giữa ln[M] và t là đồ thị đường thẳng, là đặc trưng của
phản ứng bậc 1. Do đó, có thể kết luận phản ứng là bậc 1 theo M (b = 1).
Phương trình liên hệ giữa ln[M] và thời gian của phản ứng bậc 1 theo M ,
hằng số tốc độ k’là ln[M]=ln[M]0 −k't
Từ phương trình này, khi vẽ đồ thị liên hệ, hệ số góc sẽ là số đối của hằng số
tốc độ. Từ đồ thị, ta tìm được k’1 = 1,0691 min-1 (với [X]0 = 0,5 M); k’2 =
0,4278 min-1 (với [X]0 = 0,20 M)
k′1 (k[𝐻 + ]𝑎 [X] 𝑐 )𝑇𝑁1 k(10−5 )𝑎 ⋅ 0,5𝑐 0,5 1.0691
Lập tỉ lệ, ta có: = =k(10−5 )𝑎 ⋅ 0,2𝑐=( 0.2 )𝑧 =0,4278
k′2 (k[H+ ]𝑎 [X]𝑐 )𝑇𝑁2
Từ đây suy ra: n3 ≈ 1. 0,25
Vậy phản ứng là bậc 1 theo M và bậc 1 theo X
b.
b1) Giả sử phản ứng vẫn được thực hiện ở pH ổn định. Chấp nhận H+ và L-
không hấp thụ quang (hoặc hấp thụ không đáng kể) tại điều kiện đo.
Do X và M được lấy đúng theo tỉ lệ trên phương trình, nên [X] = 3[M]. Ta có
thể thu gọn dạng biểu thức tốc độ như sau:
v = k[𝐻 + ]𝑎 [M]. 3[M]
⇒ v = k’’[M]2 (với k’’ = 3k[𝐻 + ]𝑎
Vậy phương trình động học có thể viết là:
1 1 1 [𝑀]0 −[𝑀]
[𝑀]
- [𝑀] = k”t ⇒ k” = 𝑡 . [𝑀].[𝑀]0
(1)
0
0,25
Theo đề bài, ta có (Adm là độ hấp thụ quang của dung môi):
At = Adm + AX + AM + AN = Adm + ε1 𝑙 [X] + ε2 𝑙 [M] + ε3 𝑙 [N]
⇒ At = Adm + ε1 𝑙 3[M] + ε2 𝑙 [M] + ε3 𝑙 [N]
⇒ At = Adm + [M] ⋅ (3ε1 𝑙 + ε2 𝑙) + ε3 𝑙 [N] (2)
Bảo toàn nồng độ với 2 dạng phức, ta có: [M] + [N] = [M]0 = C0 (3)
Từ (2), (3), ta có:
At = Adm + [M] ⋅ (3ε1 𝑙 + ε2 𝑙) + ε3 𝑙 (C0 – [M])
⇒ At = Adm + ε3 𝑙 C0 + [M] ⋅ (3ε1 𝑙 + ε2 𝑙 – ε3 𝑙)
Đặt C1 = Adm + ε3 𝑙 C0 và C2 = (3ε1 𝑙 + ε2 𝑙 – ε3 𝑙);
Ta có: At = C1 + [M] ⋅ C2
Tại t = 0: A0 = C1 + [M]0 ⋅C2
Tại t = ∞ (tức M đã phản ứng hết): A∞ = C1 + 0⋅C2 = C1
→ A0 – At = C11 + [M]0 ⋅C2 – C1 − [M] ⋅ C = C2⋅ ([M]0 – [M]) (4)
→ At − A∞ = C1 + [M] ⋅ C2 – C1 = C2 ⋅ [M] (5)
Vậy nếu lập tỉ lệ giữa (4) và (5), ta có:
𝐴0 −𝐴𝑡 [𝑀]0 −[𝑀]
=
𝐴 −A∞
𝑡 [𝑀]
1 𝐴 −𝐴 1 𝐴 −𝐴 0,25
Từ (1) và (6), ta có: k” = . 𝐴 0−A∞𝑡 (7) ⇒ k = . 𝐴 0−A∞𝑡
𝑡.𝐶0 𝑡 3.[𝐻 + ]𝑎 .𝑡.𝐶0 𝑡
b2) Lập phương trình (7) để tính k theo từng số liệu trong bảng đề bài
cho (đổi C0 = 5.10−3 M), ta được kết quả:

t (min) 0 5 10 15 20 ∞
At 0,870 0,796 0,738 0,693 0,657 0,33
k” (M min )
-1 -1
6,353 6,431 6,475 6,500 0,25
Ta thấy các giá trị của k đều tương đương nhau => hợp lý.
Tính trung bình các giá trị của k’’, ta được k’’ (tb) = 6,44 M-1 ⋅min-1
Lập tỉ lệ k’’ với k’1 và k’2 ở phần 1, ta được:
k" (3k[H+ ]𝑎 )𝑇𝑁3 3.(10−5,5 )𝑎 6,44 M−1 ⋅min−1
= = (10−5)𝑎.0,5 =
k′1 (k[𝐻 + ]𝑎 [X])𝑇𝑁1 1,0691 min−1
Vậy a = 0. Phản ứng không phụ thuộc vào pH.
Làm tương tự với k’’ và k’2, ta cũng thu được kết quả tương tự.
Vậy phương trình định luật tốc độ đầy đủ của phản ứng là: v = k[M][X]
Tính hằng số tốc độ phản ứng:
• Từ k’1 = k[X] ⇒ k = 1,0691 min-1 / (0,5 M) = 2,1382 M-1 ⋅min-1
• Từ k’2 = k[X] ⇒ k = 0,4278 min-1 / (0,2 M) = 2,139 M-1 ⋅min-1
• Từ k’’ = 3k ⇒ k = 2,146 M-1 ⋅min-1 0,25
• Lấy trung bình, ta được k = 2,141 M-1 ⋅min-1
1.2 a. Ta có : C0 = CA + CB + CC + CD = 0,208 M 0,25
(1,0 đ) b. Sự thay đổi nồng độ của A theo thời gian t tuân theo định luật động học
C 1 C 1 0, 208
phản ứng bậc kt = ln 0  k = ln 0 = ln = 0,187( s −1 )
C t C 10 0, 032
ln 2 0, 693 0,25
 1/2 = = = 3, 75( s)
k 0,187
c. Ta có : k1 + k2 + k3 = k = 0,187 (s-1)(*)
mặt khác ta có : k1 :k2 :k3 = CC :CB :CD = 0,068 : 0,092 : 0,016
suy ra k1 = 0,072 (s-1).
k2 = 0,098 = 4,586.10-2 (s-1); k3 = 0017(s-1).
d. Tại thời điểm t =  1/2 ta có nồng độ của CA’ = C0/2 = 0,104 M 0,25

suy ra: C’B + C’C + C’D = 0,104 M (**)


Có: C’B: C’C:C’D = CB:CC:CD = 0,068 : 0,092 : 0,016 suy ra:
Suy ra CB = 0,04 M; CC = 0,055 M; CD = 9,3.10-3M 0,25

Câu 2 (2,5 điểm) Cân bằng và phản ứng trong dung dịch. Pin điện - Điện phân.
2.1. Cho dung dịch A gồm Fe2(SO4)3 0,020 M; FeSO4 0,0400 M và H2SO4 0,20 M.
a. Tính pH của dung dịch A.
b. Điều chỉnh pH của dung dịch đến 10,0, thu được dung dịch hỗn hợp B (coi thể tích dung dịch
không đổi sau khi điều chỉnh pH). Tính thế của dây Pt được nhúng trong dung dịch hỗn hợp B.
c. Người ta thêm lượng dư mạt sắt vào dung dịch hỗn hợp B, thu được dung dịch hỗn hợp C (coi
pH của dung dịch vẫn được giữ cố định bằng 10,0). Nêu hiện tượng và tính nồng độ cân bằng của
các ion Fe2+ và Fe3+ trong dung dịch hỗn hợp C sau khi trạng thái cân bằng được thiết lập.
d. Mặt khác, nếu thêm 0,240 gam NaOH vào 10,00 mL dung dich A thì thu được dung dịch hỗn
hợp D. Tính pH của dung dịch hỗn hợp D khi hệ cân bằng.
Cho biết: pKs(Fe(OH)3) = 37,0; pKs(Fe(OH)2) = 15,1; Eo(Fe3+/Fe2+) = 0,771 V; Eo(Fe2+/Fe) = –
0,440 V; pKw = 14,00; pKa(HSO4–) = 1,99; *β(FeOH2+)= 10-2,17 ; *β(FeOH+) = 10-5,92, M(NaOH)
= 40,0 g/mol.
2.2. Điện phân là một phương pháp điện hoá được ứng dụng trong nhiều lĩnh vực công nghiệp như
mạ điện, tinh chế kim loại, làm giàu kim loại, sản xuất hoá chất, …. Tiến hành điện phân dung dịch
KCl 0,20M ở pH = 7,0.
a) Tính hiệu điện thế lí thuyết tối thiểu cần áp vào hai điện cực để quá trình điện phân bắt đầu
xảy ra. Dự đoán sản phẩm ở anode và cathode khi áp hiệu điện thế lí thuyết tối thiểu.
b) Trên thực tế, để có quá trình điện phân bắt đầu xảy ra, người ta phải áp vào cathode một
thế nhỏ hơn (Ec(thực tế) = Ec(lí thuyết) – ŋc) và áp dụng vào anode một thế lớn hơn (Ea(thực tế) =
Ea(lí thuyết) + ŋa) so với thế lí thuyết. Tính hiệu điện thế thực tế tối thiểu cần áp vào hai điện
cực để quá trình bắt đầu điện phân xảy ra. Chỉ ra sản phẩm ở anode và canode khi áp hiệu
điện thế thực tế tối thiểu.
Cho biết: ở điều kiện điện phân trên:

H2(k) K(r) Cl2(k) O2(k)

ŋc (V) 0,60 0,00 - -

ŋa (V) - - 0,10 0,83

Eo(K+/K) = -2,94V, Eo(Cl2/Cl-) = 1,36V, Eo(O2, H+/H2O) = 1,23V, Eo(H+/H2) = 0,00V. Cho
áp suất của các khí sản phẩm thoát ra trên các điện cực đều 1 bar.
Hướng dẫn
Câu Nội dung Điểm
2.1 a) Ta có: C Fe3+ = 0,04 (M); C Fe2+=0,04 M; Co H+=0,2 M; Co SO42-=0,1 M; Co
o o

(1,5 đ) HSO4-= 0,2M


Xét các cân bằng sau: Fe3+ + H2O ⇌ Fe(OH)2+ + H+ *β1 = 10-2,17
(1)
Fe2+ + H2O ⇌ FeOH+ + H+ *β2 = 10–5,92
(2)
SO42- + H+ ⇌ HSO4- Ka = 10-1,99 0,25
(3)
H2O ⇌ H+ + OH- Kw = 10-14
(4)
Dễ thấy: CFe2+ .*𝛽2 << CFe3+ . *𝛽1 → Bỏ qua cân bằng (2)
Chọn mức không là Fe3+, SO42- , H2O, ta có:
[H+] = Co H++ [Fe(OH)2+] + [OH-] – ([HSO4-] - Co HSO4-)
∗β1 Kw h
h= C0H++ ∗β1+h . C0Fe3+ + - (Ka+h. (C0SO42-+ C0HSO4- ) – C0HSO4- )
h
Giải phương trình trên, ta được: h= 0,125 (M) → pH= 0,904 0,25
b) Giả sử không có Fe(OH)3 và Fe(OH)2 kết tủa, ta có:
h
[Fe3+] = ∗β1+h . CFe3+ = 5,92.10-10 M
h
[Fe2+] = ∗β2+h . CFe2+ = 3,33.10-6 M
=> [Fe3+] . [OH-]3 = 5,92.10-22> Ks(Fe(OH)3) -> Xuất hiện Fe(OH)3 kết tủa
[Fe2+] . [OH-]2 = 3,33.10-14 > Ks(Fe(OH)2) -> Xuất hiện Fe(OH)2 kết tủa
Tính E0 (Fe(OH)3/ Fe(OH)2)
Fe3+ + e ⇌ Fe2+ (K = 1013,024)
Fe2+ + 2OH- ⇌ Fe(OH)2 (Ks(Fe(OH)2)-1 =1015,1)
Fe(OH)3 ⇌ Fe3+ + 3OH- (Ks(Fe(OH)3) =10-37)
Fe(OH)3 + e ⇌ Fe(OH)2 + OH- (K’= 10-8,88)
=> E0 (Fe(OH)3 /Fe(OH)2) = -0,5255 (V)
0,0592 1
-> EB = E0 (Fe(OH)3 /Fe(OH)2) + . log([OH−] ) = -0,2887 (V)
1
0,25
c) Khi thêm mạt sắt dư thì kết tủa màu đỏ nâu biến mất do xảy ra
Fe(OH)3 + e ⇌ Fe(OH)2 + OH- (K’= 10-8,88)
Fe ⇌ Fe2+ + 2e K1= 102.0,44/0,0592
Fe2+ + 2OH- ⇌ Fe(OH)2 (Ks(Fe(OH)2)-1 =1015,1)
Tổ hợp 3 quá trình ta có: 2Fe(OH)3 + Fe ⇌ 3Fe(OH)2 K
suy ra K = 1012,21 >>1
Do K rất lớn, Fe dư nên xem như Fe(OH)3 phản ứng hết, ta có:
Ks(Fe(OH)2)
[Fe2+] = = 10-7,1 (M) 0,25
[OH−]^2
0,0592 [Fe3+]
Khi cân bằng thì E(Fe3+/Fe2+) = E (Fe2+/Fe)-> 0,771+ . log([Fe2+]) = –
1
0,0592
0,440 + .log([Fe2+])
2
Thay [Fe2+] = 10-7,1 , ta có: [Fe3+] = 7,83.10-32=10-31,11 M
Kiểm tra: [Fe3+].[OH-]3 = 7,83.10-44 < Ks(Fe(OH)3) -> Fe(OH)3 hết, giả sử 0,25
hợp lí
d) Ta có: C OH- = 0,6 M
CFe3+ = 0,04 M; CFe2+=0,04 M; CH+=0,2 M; CSO42-=0,1 M; CHSO4-=
0,2 M
-> Khi thêm 0,240 gam NaOH vào 10,00 mL dung dich A thì thu được dung
dịch hỗn hợp D có
Thành phần giới hạn gồm: Fe(OH)3, Fe(OH)2, SO42- 0,3M
Xét các cân bằng sau: Fe(OH)3⇌ Fe3+ + 3OH- (Ks1=10-37) (1)
2+ - -25,17
Fe(OH)3⇌ Fe(OH) + 2OH (K1 = KS1. *𝛽1.KW = 10 ) (2)
2+ - -15,1
Fe(OH)2⇌ Fe + 2OH (Ks2=10 ) (3)
+ -
Fe(OH)2⇌ Fe(OH) + OH (K2 = KS1. *𝛽1.KW = 10-7,02) (4)
2- - - -12,01
SO4 + H2O ⇌ HSO4 + OH (Kb = 10 ) (5)
+ -
H2O ⇌ H + OH (Kw = 10-14 ) (6)
4 3
Dễ thấy: √𝐾2 >> 3. √𝐾𝑠1; 2. √𝐾1; √𝐾𝑤; √𝐾𝑏. 𝐶(𝑆𝑂4)2 −
3
√𝐾2 > 2 √𝐾𝑠2 -> Xem như cân bằng (4) quyết định pH của hệ
Xét: Fe(OH)2⇌ Fe(OH)+ + OH- (K2 =10-7,02) (4)
-3,51
x x -> x = √𝐾2 = 10 -> pH = 10,49
Ks2
Kiểm tra: [Fe2+] = = 8,32.10-9(M) << [OH-] -> giả sử hợp lí -> 0,25
[OH−]^2
chấp nhận pH=10,49
2.2 a) + Ở cathode: K+ và H2O, những phản ứng khử có thể có:
(1,0 đ) K+ + e- ⇌ K(r) Eo(K+/K) = -2,94V
2H+(aq) + 2e- ⇌ H2(k) Eo(H+/H2) =
0,00V
E K+ /K = Eo K+ /K + 0,0592.lg[K+]

= -2,94V + 0,0592. lg(0,2) = -2,981V.


o 0,059 [H + ]2 0,25
E (H+/H2) = E (H+/H2) + .lg
2 p H2
0, 059 [10−7 ]2
= 0,00V + .lg = -0,4144V
2 1
E (H+/H2) > E (K+ /K) ⇒ sản phẩm ở cathode là H2.
(Nếu học sinh viết 2H2O(aq) + 2e- ⇌ H2(k) + 2OH-(aq) và tính
đúng vẫn được điểm. Sản phẩm ở cathode có thêm OH-)
+ Ở anode: Những phản ứng oxi hoá cơ thể có:
2Cl-(aq) ⇌ Cl2(k) + 2e- Eo(Cl2/Cl-) = 1,36V
2H2O(l) ⇌ O2(k) + 4H+(aq) + 4e- Eo(O2, H+/H2O) = 1,23V
0,059 p
E(Cl2/Cl-) = Eo(Cl2/Cl-) + .lg Cl2
2 [Cl- ]2
0,059 1
= 1,36V + .lg = 1,401V
2 [0,2]2
0,059
E(O2, H+/H2O) = Eo(O2, H+/H2O) + .lg(p O2 .[H + ]4 )
4
0,059
= 1,23 + .lg([10−7 ]4 )
4
= 0,816V
E(O2, H /H2O) < E(Cl2/Cl-) ⇒ H2O bị oxi hoá ⇒ sản phẩm ở anode là O2.
+

Hiệu điện thế thực tối thiểu:


U = Ea – Ec = E(O2, H+/H2O) - E (H+/H2) = 0,816V – (-0,4144V) = 1,23V. 0,25
b) + Ở cathode:
E K+ /K = Eo K+ /K + 0,0592.lg[K+]

= -2,94V + 0,0592V × 𝑙𝑔(0,2) = -2,981


o 0,059 [H + ]2
E (H+/H2) = E (H+/H2) + .lg
2 p H2
0, 059 [10−7 ]2
= 0,00V + .lg - 0,60V = -1,4144V.
2 1
E (H+/H2) > E (K+ /K) ⇒ sản phẩm ở cathode là H2
0,25
+ Ở anode:
0,059 p
E(Cl2/Cl-) = Eo(Cl2/Cl-) + .lg Cl2 + 0,1
2 [Cl- ]2
0,0592𝑉 1
= 1,36V + × 𝑙𝑔 [0,2]^2 = 1,501V
2
0,059
E(O2, H+/H2O) = Eo(O2, H+/H2O) + .lg(p O2 .[H + ]4 )
4
0,0592𝑉
= 1,23 + × lg([10^ − 7]4 × 1] + 0,83V
4
= 1,646V
E(Cl2/Cl-) < E(O2, H+/H2O) → Cl- bị oxi hoá → sản phẩm ở anode là Cl2
Hiệu điện thế thực tối thiểu:
U’ = Ea – Ec = E(Cl2/Cl-) - E (H+/H2) = 1,501V – (-1,4144V) = 2,915V. 0,25
Câu 3 (2,5 điểm) Nhiệt động học và cân bằng hóa học.
3.1. Một ống nghiệm chứa 100 mL chất lỏng benzen ở 200C, C6H6(l), được đưa vào bể nước đá (viên
đá cộng với nước) ở 0,20°C. Khi lấy ống nghiệm ra khỏi bể, nó chỉ bị đông một phần, còn lại 26,7
mL benzen ở dạng lỏng tiếp xúc với C6H6 đông đặc.
Bể cách thủy vẫn có đá trong nước và vẫn ở nhiệt độ 0,20°C.
a) Nhiệt độ của benzen trong ống sau khi lấy ra là bao nhiêu?
b) Sự thay đổi entropi đối với C6H6 sẽ được xác định như thế nào? Tính giá trị của sự thay đổi entropi
đối với C6H6 trong quá trình trên.
c) Liệu bể nước đá cũng có thay đổi entropi không? Nếu có, hãy tính giá trị ∆S đối với bể nước đá.
Cho các giá trị của benzene:
D = 0,876g/ml
Nhiệt nóng chảy Hfus = 10,6kJ/mol
Nhiệt độ đông đặc t = 5,6oC
Nhiệt dung: Cp(s) = 118 J/mol.K; Cp(liq) = 136 J/mol.K; Cp(g) = 82.4 J/mol.K
3.2. Khi nung nóng đến nhiệt độ cao PCl5 bị phân li theo phương trình
PCl5 (k) PCl3 (k) + Cl2(k)
a) Cho m gam PCl5 vào một bình dung tích V, đun nóng bình đến nhiệt độ T (K) để xảy ra phản ứng
phân li PCl5. Sau khi đạt tới cân bằng áp suất khí trong bình bằng P. Hãy thiết lập biểu thức của Kp
theo độ phân li α và áp suất P. Thiết lập biểu thức của Kc theo α, m, V.
b) Trong thí nghiệm 1 thực hiện ở nhiệt độ T1 người ta cho 83,4 gam PCl5 vào bình dung tích V1.
Sau khi đạt tới cân bằng đo được P1 = 2,700 atm. Hỗn hợp khí trong bình có tỉ khối so với hiđro
bằng 69,5. Tính α1 và Kp1.
c) Trong thí nghiệm 2 giữ nguyên lượng PCl5 và nhiệt độ như ở thí nghiệm 1 nhưng thay dung tích
V
là V2 thì đo được áp suất cân bằng là P2 = 0,500 atm. Tính tỉ số 2
V1

Câu Nội dung Điểm


3.1 a. Nhiệt độ benzene trong ống là 5,6oC vì benzene lỏng và benzene rắn đang 0,25
(1,0 đ) ở trạng thái cân bằng của quá trình chuyển pha.
b. Benzene có sự biến thiên entropi qua 2 quá trình
(1) Tất cả benzene lỏng từ 20oC xuống 5,6oC: S1
(2) 1 phần benzene lỏng chuyển thành benzene rắn: S2
Số mol benzene tổng: nbenzene = (100.0,876)/78= 1,123 mol 0,25
S1 = nCp(l).ln(T2/T1) = 1,123.136.ln(278,6/293) = − 7,7 J/K
Số mol benzene chuyển sang pha rắn: 0,733.1,123 = 0,823 mol C6H6
S2 = −Hfus/Tđđ = 0,823.10600/278,6 = −31,31 J/K
Biến thiên entropi của benzene trong quá trình trên là
S = S1 + S2 = − 39,0 J/K 0,25
c. Mặc dù bể nước đá T không thay đổi, cũng như vẫn còn cả hai pha của nước,
nhưng có một số nước đá đã tan chảy ở 0°C, hấp thụ nhiệt bị mất do làm lạnh
và đóng băng benzene. ∆S của nước là do sự chuyển pha này.
Qthu = −Qtỏa
 QC6H6,s + QC6H6,l = nH2O,s.Hfus,H2O
 0,823.(−10,6.1000) + 1,123.136.(5,6-20) = − 10923,08 J
Qtỏa,H2O = 10923,08J
0,25
SH2O = 10923,08/273 =40 J/K
3.2 a) Thiết lập biểu thức Kp,Kc
(1,5 đ) PCl5 (k) PCl3 (k) + Cl2(k)
Ban đầu a(mol)
Cân bằng a-x x x (mol)
Tổng số mol khi cân bằng: a + x = n
x
α= ; Khối lượng mol MPCl5 = 208,5 (g/mol), MPCl3 = 137,5 (g/mol), MÇl2
a
= 71 (g/mol)
m 0,25
= a mol PCl5 ban đầu
208,5
* Tính Kp:
Áp suất riêng phần lúc cân bằng của mỗi khí
−x PPCl3 = PCl2 =
x
PPCl5 = .P ; .P
+x +x
x
( . p)2
pCl2 . pPCl3  + x x2 ( + x)
Kp = = = . p2. ⎯⎯

pPCl5 −x ( + x ) 2
( − x ). p
( ). p
+x
x2
x2 x2  2 . p 2 =  .P
2
p
Kp = . = 2 . P ; Kp =
( + x) ( − x)  − x 2  2 x2 1− 2

2 2
* Tính Kc
a(1 −  ) a
 PCl5  = ;  PCl3  = Cl2  =
V V

Kc =
 PCl3 .Cl2  = (a )2 . V = a 2 = m 2
 PCl5  V 2 a (1 −  ) V (1 −  ) 208,5V (1 −  )
Hoặc Kp = Kc = ( RT )n Với nkhi = 1
pV pV
Kp=Kc(RT) PV=nRT = ( a + x) RT → RT = =
a + x a(1 −  )
pV 2 pV
Kp = Kc. → . p = Kc
a + x 1−  2
a+x
2 pV a 2 (1 + )
Thay x=aα → . p = Kc → Kc = .
1 − 2 a(1 + ) V 1 − 2
a 2 (1 + ) a 2 m 2
Kc =
. = =
V (1 − )(+ ) V (1 − ) 208,5V (1 − ) 0,25
83, 4
b) Thí nghiệm 1 nPCl5bandau = = 0, 40mol
203,5
̅ của hỗn hợp bằng 69,5 x 0,2 =139,0 (g/mol)
𝑀
0,25
Tổng số mol khí lúc cân bằng n1 = a( 1+ α1) = 83,4: 139,0 = 0,60 mol
n1 = a( 1+ α1) =0,4 ( 1+ α1)=0,6 → α1= 0,500
2 (0,5)2
* Tìm Kp tại nhiệt độ T1: Kp = .P = .2,7 = 0,9
1− 2 1 − 0,52 0,25
c)Thí nghiệm 2: - Giữ nguyên nhiệt độ →Kp không đổi
- Giữ nguyên số mol PCl5 ban đầu a=0,40 mol
- Áp suất ban đầu p2 = 0,5 atm.
22 22
Ta có . p2 = Kp = .0,5 = 0, 9 →  2 2 = 0, 64286 →  2 = 0,802
1 − 2 2
1 − 2 2

0,25
Tổng số mol khí lúc cân bằng: n2 = 0,4( 1+ α2) ≈ 0,721(mol)
n RT n RT
*Thể tích bình trong TN2 V2 = 2 so với V1 = 1
p2 p1
0,25
V2 n2 p1 0, 721.2, 7
= = = 6, 489 lần
V1 n1 p2 0, 6.0,5
Câu 4 (2,5 điểm) Hóa nguyên tố (Kim loại, phi kim nhóm IVA, VA). Phức chất.
4.1. Chất A1 là chất khí ở điều kiện thường, có màu đỏ nâu. Khi làm lạnh A1 màu đỏ nâu nhạt dần,
sau một thời gian thu được khí A2 không màu. Khi cho A2 phản ứng với Na ở điều kiện thích hợp
thu được khí A3. Cho A3 tác dụng với H2, xúc tác Pd/C trong môi trường HCl thu được hợp chất ion
A4. Trung hòa A4 bằng dung dịch EtONa, ta thu được A5. Khi cho A5 tác dụng với dung dịch NaClO
thu được khí A6 (có tỉ khối so với H2 là 22). Mặt khác, khi oxi hóa A1 trong điều kiện thích hợp, thu
được acid A7. Hỗn hợp của một thể tích A7 và 3 thể tích HCl đặc có khả năng hòa tan Au. Trong
phòng thí nghiệm, A8 có thể được điều chế bằng cách dùng P2O5 làm mất nước A7. Cho A8 tác dụng
với H2O2 khan ở khoảng -80 °C, thu được A9.
Cho biết: - A1, A2, A3, A6,A8 là các oxide của nitrogen
- A5 và A9 chứa 3 nguyên tố, cả hai đều chứa 1 nguyên tử nitrogen phân tử, %m N (A5)
= 42,42% và %mN (A9) = 17,72%.
a. Xác định công thức hóa học của các chất từ A1 đến A9. Viết PTHH của các phản ứng xảy ra.
b. Vẽ công thức cấu tạo của A2, A7, A8, A9.
4.2.
a. Vẽ các đồng phân có thể có cho các phức: [Ni(CN)2Br2]2- và [Co(H2O)2(ox)BrCl]−, với ox
là −O2CCO2−.
b. Trong dung dịch OH- 1,0M của [Co(NH3)5Cl]2+ tồn tại cân bằng:
[Co(NH3)5Cl]2+ + OH- [Co(NH3)4(NH2)Cl]+ + H2O.
Ở 25oC, tại thời điểm cân bằng xác định được rằng ít nhất 95% phức chất tồn tại ở dạng
axit [Co(NH3)5Cl]2+ Chứng minh [Co(NH3)5Cl]2+ là một axit rất yếu có Ka ≤ 5,26.10-16.
Hướng dẫn

Câu Nội dung Điểm


4.1 a.
(1,5 +) A1 và A2 là oxide của nitrogen, A1 ở điều kiện thường là chất khí có màu đỏ nâu.
đ) Khi làm lạnh A1 thu được chất khí A2 không màu → A1 là NO2 và A2 là N2O4
+) Ta thấy, A5 thu được bằng cách trung hòa A4 mà A4 là sản phẩm của phản ứng
hydro hóa oxide nitrogen A3. Mặt khác A5 chứa 3 nguyên tố → A5 chứa 3 nguyên tố
N,H,O
14.100 0,25
Tạ lại có: MA5 = .= 33(g/mol). Đặt A5 là NHaOb → MA5 = 14 + a +16b = 33
42,42
→ b=1 và a=3 → A5 có CTPT là NH3O hay NH2OH
+) A5 thu được khi trung hòa A4 → A4 là NH2OH.HCl hay [NH3OH]+[Cl]-
+) A3 là oxide của nitrogen, thu được khi cho A2 (N2O4) tác dụng với Na và khi hydro
hóa A3 trong môi trường HCl thì thu được A4 (NH2OH.HCl) → A3 là NO
+) Ta có:MA6 = 22.2 = 44 (g/mol) và A6 là oxide của nitrogen và từ phương pháp điều
chế A6 ->
0,25
A6 là N2O.
+) Khi oxi hóa A1(NO2) trong điều kiện thích hợp ta thu được acid A7. Hỗn hợp của
một thể tích A7 và 3 thể tích HCl đặc có khả năng hòa tan Au → A7 là HNO3
+) A8 là oxide của nitrogen và A8 có thể được điều chế bằng cách dùng P2O5 làm mất
nước A7 ->
A8 là N2O5
+) Cho A8 + H2O2 khan ở khoảng -80°C thì thu được A9 có chứa 3 nguyên tố → A9
gồm 3 nguyên tố là N, H, O
14.100 0,25
Mặt khác: MA9 = .=79 (g/mol). Đặt A9 là NHxOy
17,72
→ MA9 = 14 + x + 16y = 79 => x=1, y=4 => A9 là NHO4 hay HNO4
PTHH: 2NO2 ⇌ N2O4
N2O4 + Na → NaNO3 + NO
3NO + 3H2 + 2HCl --Pd/C→ NH2OH.HCl
NH2OH.HCl + EtONa → NH2OH + NaCl + EtOH
2NH2OH + 2NaClO → N2O + 2NaCl + 3H2O
4NO2 + O2 + 2H2O → 4HNO3
Au + 3HCl + HNO3 → AuCl3 + NO + 2H2O
2HNO3 + P2O5 → 2HPO3 + N2O5
0,25
N2O5 + H2O2 → HNO4 + HNO3
b.
A2 (N2O4) :

A7 (HNO3):

0,25

A8 (N2O5)

A9 (HNO4)

0,25

4.2 a. Vẽ các đồng phân có thể có cho các phức: [Ni(CN)2Br2]2- và [Co(H2O)2(ox)BrCl]−,
(1,0 với ox là −O2CCO2−.
đ)

0,25
0,25
Lưu ý: Các đồng phân này đều có đồng phân gương
b.
[Co(NH3)5Cl]2+ + H2O [Co(NH3)4(NH2)Cl]+ + H3O+
Kí hiệu [Co(NH3)5Cl]2+ là A; [Co(NH3)4(NH2)Cl]+ là B
0,25
[B].[H3O+ ] [B] Ka
Ta có Ka = (1) → = (2)
[A] [A] [H 3O + ]
[Co(NH3)5Cl]2+ + OH- [Co(NH3)4(NH2)Cl]+ + H2O (*)

0,25

Câu 5 (2,5 điểm) Đại cương hữu cơ.


5.1. Cho cấu tạo của hợp chất hữu cơ E

Hãy chỉ rõ trạng thái lai hóa của từng nguyên tử N ở cấu tạo E và ghi giá trị Pka (ở 25 oC): 1,8; 6,0;
9,2 vào từng trung tâm axit trong công thức tương ứng với E. Giải thích.
5.2. Cho dạng enol của các 1,3-dicarbonyl sau:

(1) (2) (3)


a. So sánh tính acid của chất 1 và chất 2.
b. Giải thích vì sao chất 3 cũng là ester nhưng có tính acid mạnh hơn chất 2 và mạnh hơn rất nhiều
sơ với chất 1 (100000 lần ) ?
5.3. Chuyển hóa sau đây tạo thành α-glucoside đi từ 2,3,4,6-tetra-O-benxyl-α-D-glucopyranosyl
bromide với alcohol , tetraethylammoium bromide và diisopropylethylamine trong CH2Cl2 . Giải
thích tính chọn lọc lập thể của phản ứng .( R=CH2Ph) . ( thí sinh phải mô tả bằng hình vẽ )
5.4. Hãy giải thích vì sao phản ứng dưới đây chỉ cần dùng 0,1 đương lượng DMAP hoặc DABCO
bằng phản ứng hóa học và trên phượng diện hóa lý hữu cơ (không xét đến cơ chế phản ứng ) ?

5.5. Vẽ giản đồ theo MO và chỉ rõ LUMO, HOMO trong phân tử buta-1,3-đien

Hướng dẫn
Câu Nội dung Điểm
5.1 sp 2 N COOH
sp 3
NH 2
N sp 2
H (E)
- Nguyên tử N nhóm NH ở trạng thái lai hóa sp2, cặp e chưa chia ở obitan p xen phủ
với 5 obitan p khác tạo thành hệ thơm được lợi về mặt năng lượng nhưng “mất” tính 0,25
bazơ.
- Nguyên tử N thứ hai ở trạng thái lai hóa sp2, cặp e chưa chia ở obitan sp2 không
tham gia vào hệ thơm nên còn tính bazơ.
- Nguyên tử N nhóm NH2 ở trạng thái lai hóa sp3.
6,0 COOH 1,8
H N
NH 3 9,2
N
H
- Nhóm NH3+ là axit liên hợp của nhóm H2Nsp3 , nhóm NH+ là axit liên hợp của nhóm
Nsp2.
- Bazơ càng mạnh thì axit liên hợp càng yếu, vì thế giá trị 9,2 là thuộc nhóm NH3+
0,25
còn giá trị 6,0 thì thuộc nhóm NH+.
5.2 * Tính acid của 2 lớn hơn 1 bởi vì :
- Nhóm carbonyl của ester (chất 1) nhận điện tử từ nhóm OEt nên làm tăng mật độ
điện âm trên oxy , khiến cho liên kết hydro nội phân tử giữa OH và C=O bền vững
hơn.
0,25

- Dạng enolate của chất 1 kém bền hơn 2 do nhóm carbonyl cuả ester không làm bền
hiệu quả được điện tích âm trên oxy như chất 2 do LUMO của nó bị chiếm dụng bởi
đôi điện từ trên nhóm OEt , trong khi chất 2 không có hiện tượng này ( hiệu ứng liên
hợp chéo)
* Do chất 3 không có khả năng nào để tạo thành liên kết hydro nội phân tử
- Oxy trong vòng lactone khó liên hợp hiệu quả vào nhóm C=O do làm tăng sức căng
vòng 5 cạnh . Điều này dẫn đến khả năng làm bền enolate của nhóm carbonyl được
củng cố mạnh (0.125đ )

- Do oxy trong vòng lactone không liên hợp tốt với nhóm carbonyl nên nó gây hiệu
ứng cảm –I là chủ yếu , làm tính acid của 3 nhỉnh hơn so với chất 2 0,25
5.3 Do nucleophile cộng vào Oxonium theo hướng trục , trong quá trình đi qua trạng thái
chuyển tiếp dạng ghế làm cho phản ứng chọn lọc về mặt lập thể trục.

0,25
0,25
5.4

- CO2 là acid sinh ra sau quá trình phản ứng nhưng lại bị thoát ra khỏi hệ , không tác
dụng được với các base khác như DMAP hay DABCO nên chúng không thể xem như 0,25
chất giúp dịch chuyển cân bằng
- DMAP hay DABCO có chức năng như các nucleophile để kích hoạt Boc2O hay
Me2CO3 , tăng tốc độ phản ứng ( không cần viết phương trình phản ứng )
- DMAP hay DABCO có chức năng deproton hóa , sau đó hoàn trả proton về cho các
ancol ở sản phẩm. Các base không tham gia vào phương trình tổng quát nên chỉ có
vai trò xúc tác 0,25
5.5

0,5

Câu 6 (2,5 điểm) Sơ đồ tổng hợp hữu cơ. Cơ chế phản ứng hóa hữu cơ.
6.1. Xác định cơ chế phản ứng

6.2. Xác định các chất trong các chuỗi sau


a.

b.

c.

Câu Nội dung Điểm


6.1 a.
(1,0
đ)

0,25

b.

0,25
c.

0,25
d.

6.2 a.
(1,5
đ)

0,25

0,25

b.
0,5
c.

0,25

0,25

Câu 7 (2,5 điểm) Xác định cấu trúc các chất hữu cơ (mô tả sơ đồ tổng hợp bằng lời dẫn)
7.1. Hợp chất A (C11H17NO3) không quang hoạt, không tan trong môi trường trung tính và kiềm
nhưng dễ tan trong HCl loãng. A có hai nguyên tử hidro linh động, A phản ứng với Ac2O tạo B
(C13H19NO4) trung tính. A phản ứng với MeI dư sau đó thêm AgOH, sản phẩm thu được C có công
thức là C14H25NO4. Đun nóng chất thu được Et3N và D (C11H14O3) trung tính. D phản ứng với O3
thu được HCHO và E. Andehit E thơm phản ứng với HI tạo sản phẩm chứa 3 nhóm -OH mà chúng
không tạo được liên kết hidro nội phân tử bền vững.
a) Xác định các chất chưa biết
b) Từ E và các hợp chất vô cơ, hãy điều chế chất A.
7.2. Hợp chất A được tổng hợp trực tiếp từ hợp chất cơ magie G với D.
Từ 4-metylpentan-1,4-điol tạo thành hợp chất đibrom B. Chất B tách HBr tạo thành B1, B2, B3, trong đó
B3 là sản phẩm chính. B3 tác dụng với Mg tạo ra G. Chất D được tạo thành từ phản ứng của isopren với
metyl vinyl xeton. Tìm công thức cấu tạo của A, B, B1, B2, B3, G và D.

Hướng dẫn
Câu Nội dung Điểm
7.1 E là andehit thơm, E phản ứng với HI thu được sản phẩm có 3 nhóm -OH không
(1,5 đ) tạo liên kết hidro nội phân tử bền nên:

0,25

E D
C nhiệt phân tạo D nên C có 2 công thức là:
0,25

0,25
1 2
A không quang hoạt nên C không quang hoạt nên công thức cấu tạo của C là 1
Vậy A và B là:

0,25

B A
0,5
b) Sơ đồ tổng hợp A từ E ( coi E là R-CHO)
𝑁𝑎𝐵𝐻4
RCHO → RCH2OH RCH2Br RCH2CN

A
7.2 Công thức cấu tạo của A, B, B1, B2, B3, G và D là
(1,0 đ)
0,5

0,5

Câu 8 (2,5 điểm) Hóa học các hợp chất thiên nhiên (Cacbohidrat và các hợp chất hữu cơ chứa
nito đơn giản)
Thủy phân hoàn toàn 1 mol chất A với xúc tác enzyme ß-glycosidase thu được 1 mol chất B
(C15H12O4), 1 mol chất C (C5H10O5) và 2 mol chất D (C6H12O6).
8.1. Trong môi trường trung tính hay acid, chất B không màu, chứa 11 carbon bất đối có cấu hình R,
nhưng khi bị kiềm hóa mãnh liệt, B mở vòng thành B1 có màu vàng. Methyl hóa hoàn toàn B1 bằng
CH3I/K2CO3 thu được B2 (C18H18O4). Cho B2 phản ứng với O3 ở -78 °C, sau đó khử hóa bằng Me2S
thì thu được B3 (C8H8O2) và B4 (C10H10O4). Đun nóng B3 trong dung dịch NaOH đậm đặc rồi acid
hóa, người ta thu được 2 sản phẩm, trong đó có acid p-methoxybenzoic. Tuy nhiên, thực hiện phản
ứng tương tự (đun nóng trong NaOH rồi acid hóa) với B4 thì chỉ thu được 1 sản phẩm B5 (C10H12O5)
duy nhất chứa 1 carbon bất đổi (tồn tại dưới dạng racemic). Mặt khác, oxy hóa nhẹ nhàng B4 rồi sinh
chuyển hóa sản phẩm thu được bằng enzyme decarboxylase với sự có mặt của NAD" thu được B6,
là một dẫn chất của acid p-methoxybenzoic chứa 2 nhóm methoxy. Xác định cấu trúc của các chất
B, B1 → B6 với lập thể phù hợp.
8.2. Khi cắt mạch Ruff C (oxy hóa bằng Br2/H2O rồi đun nóng sản phẩm với H2O2/Fe3+) thì thu được
C1 (C4H8O4). Trong chất A, C chỉ tồn tại ở dạng D-furanose, phản ứng được với tối đa 1 đương
lượng (CH3)CO/H2SO4 thu được C2; C có tính quang hoạt, còn C1 không có tính quang hoạt; C, C1
và C2 đều tham gia phản ứng Tollens. Biết trong dung dịch, chất D tồn tại chủ yếu ở dạng pyranose
với tất cả nhóm thế ở vị trí biên. Xác định cấu trúc của C tồn tại trong chất A, C1, D và C2 với lập
thể phù hợp.
Đun nóng chất A trong dung dịch kiềm mạnh rồi methyl hóa hoàn toàn bằng dimethylsulfate,
thủy phân hoàn toàn sản phẩm thu được bằng dung dịch acid thu được B7(C17H14O5), C3 (C7H14O5),
D1 (C9H18O6) và D2 (C10H20O6).
8.3. B7 (có màu vàng) là một dẫn xuất O-methyl của B1. Trong môi trường acid, B7 không cho phản
ứng gì. Oxy hóa mãnh liệt B7 có thể thu được B6 trong hỗn hợp sản phẩm. Ngoài ra C3 là dẫn xuất
O-methyl của C, nhưng không phản ứng được với NaIO4. Mặt khác D1 và D2 đều là dẫn xuất O-
methyl của D. D1 có khả năng phản ứng với phenylhydrazine để tạo osazone còn D2 thì chỉ tạo được
hydrazone.
Từ những dữ liệu trên ,xác định cấu trúc của A và B7, C3 với lập thể phù hợp. Biết khi thủy phân
một phần chất A , không phát hiện dấu vết của disaccharide nào có công thức phân tử C12H22O11 .
Hướng dẫn
Câu Nội dung Điểm
8.1
Methyl hóa hoàn toàn B1 bằng CH3I/ K2CO3, thu được B2 (C18H18O4)
=> B1 có 3 nhóm OH.
Chất B3 và B4 đều là aldehyde nên trong môi trường kiềm có thể thực hiện phản ứng
aldol hoặc Cannizzaro (nếu không có Hα).
- Đun nóng B3 trong dung dịch NaOH đậm đặc rồi acid hóa, người ta thu được 2 sản
phẩm, trong đó có acid p-methoxybenzoic

=> B3 là: 0,25


Thực hiện phản ứng tương tự với B4 thì chỉ thu được 1 sản phẩm B5 (C10H12O5) → Có
thể là phản ứng Cannizzaro nội phân tử.
- Oxy hóa nhẹ nhàng B4 → tạo thành acid, sinh chuyển hóa sản phẩm thu được bằng
enzyme decarboxylase với sự có mặt của NAD- thu được B6 → đây là quả trình chuyển
R-CO-CHO ) → R-COOH. Mặt khác, B6 là một dẫn chất của acid p-methoxybenzoic
chia 2 nhóm methoxy
→ B6 có thể là:

→ B4 có thể là: 0,25

→ B1 có thể là:

Trong đó chỉ có B1-A mới có khả năng đóng vòng trong môi trường acid-> chi có B1-
A thỏa mãn dữ liệu để bài 0,25
→ cấu tạo của các chất được tóm tắt như sau:
0,25

8.2
Khi cắt mạch Ruff C (quang hoạt) thì thu được C1 (không quang hoạt)
=> C và C1 cùng chứa nhánh - CH2OH. C tồn tại dạng D nên công thức Fischer của C
(D-apiose) và C1 lần lượt là:

0,25

- Xác định đúng C và C1 ở dạng mạch hở


Trong chất A, C chỉ tồn tại ở dạng D-furanose với carbon anomeric có cấu hình B (từ
dữ liệu thủy phân ở đầu bài), như vậy cấu trúc vòng của C có thể là:

0,25

_ C phản ứng được với tối đa 1 đương lượng (CH3)2CO/ H2SO4 sinh ra sản phẩm C2
tham gia phản ứng Tollens
=> Chỉ có C-A thỏa mãn dữ liệu đề bài. Cấu trúc của C2 có thể là:

Lưu ý: học sinh có thể kết luận C-A là đáp án mà không cần biện luận công thức của C-
B do C-B là đường L chứ không phải đường D.
Trong dung dịch, chất D tồn tại chủ yếu ở dạng D-pyranose với tất cả nhóm thể ở vị trí
biên. Xác định cấu trúc chất D với lập thể phủ hợp. Tổng: 0,1 đ
Chỉ có Beta-D-glucopyranose thỏa mãn dữ liệu để bài
=> Cấu tạo chất D:
0,25

8.3
- Chất B7 (C17H16O4 )chứa 2 nhóm methoxy => 1 nhóm OH phenol tạo liên kết
glycoside.
B7 không cho phản ứng gì → OH dùng để đóng vòng bị methyl hóa.
Oxy hóa mãnh liệt B7 có thể thu được B6 trong hỗn hợp sản phẩm
=> 2 nhóm methoxy nằm ở vòng chroman
=> Cấu tạo của B7 là:

- Chất C3 (C7H14O5) => C3 chứa 2 nhóm methoxy => có 2 nhóm OH tạo liên kết
glycoside.
Ngoại trừ OH hemiacetal, các khả năng bị methyl hóa của C3 là
0,25

- Vì OH hemiacetal không bị khóa nên vòng có khả năng chuyển thành mạch hở => C3-
A và C3-B đều có khả năng phản ứng với NaIO4, chỉ có cấu trúc C2-C thỏa mãn đã dữ
liệu đề bài.

- Chất D1 (C9H18O6) chia 3 nhóm methoxy và D2 (C10H20O6) chứa 4 nhóm metoxy.


D1 có khả năng phản ứng với phenylhydrazine để tạo osazone 0,25
=> OH ở carbon số 2 không bị O-Methyl hóa còn D2 thì thì chỉ tạo được hydrazone
=> toàn bộ OH ở carbon số 2, 3, 4, 6 đều bị O-Methyl hóa .
Như vậy cấu trúc của D1 và D2 lần lượt là:

+ Aglycol (genin) liên kết với phần glycoside ở vị trí OH phenol ngoài khung chroman.
+ Phần D-Apinso liên kết với 2 phần tử đường còn lại bằng liên kết 1,4 glycoside.
+ 1 phân tử glucose nằm ở cuối phần glycoside (do chỉ tạo 1 liên kết glycoside ở carbon
anomeric).
+ 1 phân tử glucose liên kết với 2 phiên tử đường còn lại bằng liên kết 1,2-glycoside.
+ Khi thủy phân 1 phần chất A, không phát hiện dấu vết của disaccharide nào có công
thức phân tử C12H22O11, => Trong A, 2 phân tử đường glucose không liên kết trực tiếp
với nhau.
Như vậy cấu tạo duy nhất của A là:

0,25

-------------- HẾT --------------

(Thí sinh không được sử dụng tài liệu. Cán bộ coi thi không giải thích gì thêm)

Giáo viên ra đề: Lê Văn Đạt – 0948708789


SỞ GIÁO DỤC VÀ ĐÀO TẠO SƠN LA ĐÁP ÁN ĐỀ THI CHỌN HSG KHU VỰC DUYÊN HẢI BẮC BỘ
TRƯỜNG THPT CHUYÊN NĂM HỌC 2022 – 2023
HƯỚNG DẪN CHẤM Môn: Hóa học – Lớp 11
Thời gian làm bài 180 phút (không kể thời gian phát đề)

Câu 1. (2,5 điểm) Tốc độ phản ứng


1
Cho phản ứng pha khí: N2O5 (k)→ 2NO2 (k)+
O2 (k) .
2
Thực nghiệm chứng tỏ rằng biểu thức định luật tốc độ của phản ứng trên có dạng
v = k[N2O5] với hằng số tốc độ k = 3,46.10-5 s-1 ở 25oC. Giả thiết phản ứng diễn ra trong bình kín ở 25oC, lúc
đầu chỉ chứa N2O5 với áp suất p(N2O5) = 0,100 atm.
a. Tốc độ đầu của phản ứng bằng bao nhiêu?
b. Tính thời gian cần thiết để áp suất tổng cộng trong bình phản ứng bằng 0,175 atm ở nhiệt độ không
đổi (25oC).
c. Phản ứng phân hủy của dinitơ pentoxit diễn ra theo cơ chế sau:
k1
(1) N2O5 NO2 + NO3
k -1
k2
(2) NO2 + NO3 NO2 + O2 + NO
k3
(3) NO + N2O5 3 NO2
Sử dụng nguyên lý trạng thái dừng đối với NO và NO3 hãy chứng minh cơ chế trên là phù hợp với luật tốc
độ của phản ứng.
d. Năng lượng hoạt động hóa của phản ứng ở 300K là EA = 103kJ. Ở nhiệt độ nào thì hằng số tốc độ
phản ứng tăng gấp đôi. Biết nồng độ ban đầu của các chất là như nhau, EA và A không đổi trong suốt bài
toán.
a. Số mol có trong bình N2O5: 0,50
-1 -1 -3
n(N2O5) = pV/RT = 0,10.atm.V (L) /0,082L.atm.mol .K .298 K = 4,1.10 .V mol.
n( N 2O5 ) p ( N 2O5 ) 0,1
[N2O5] = = = (mol / L) = 4,1.10-3 mol/L
V RT 0, 082.298
v = 3,46.10-5 s-1. 4,1.10-3.mol/L = 1,42.10-7 mol.L-1.s-1
b. N2O5 → 2NO2 + (1/2)O2 0,75
Po 0 0
Po -x 2x x/2
Ptổng = Po -x + 2x + x/2 = Po +(3/2)x = (7/4)Po→ x = Po/2 và Po - x = Po/2.
Ở cùng nhiệt độ, khi thể tích bình phản ứng không thay đổi, sự giảm áp suất riêng phần tỉ lệ với sự
giảm số mol. Trong phản ứng bậc 1, thời gian cần thiết để nồng độ chất phản ứng giảm đi một nửa
bằng: t1/2 = ln2/k = 0,693/3,46.10-5 s-1 = 2.104 s
dN2 O 5 
= k 1 N2 O 5  − k −1 NO 2 NO 3  + k 3 NON2 O 5 
0,75
c.v=− (1)
dt
dNO 3 
= k 1 N2 O 5  − k −1 NO 2 NO 3  − k 2 NO 2 NO 3  = 0 (2)
dt
dNO 
= k 2 NO 2 NO 3  − k 3 NO N 2 O 5  = 0 (3)
dt
k 1 N2 O 5 
từ (2): NO 3  =
(k −1 + k 2 )NO 2 
từ (3): NO = k 2 NO 2 NO 3  = k 2 NO 2 k 1 N2 O 5  = k 2k 1 Thay vào (1) dẫn đến:
k 3 N2 O 5  k 3 N2 O 5 (k −1 + k 2 )NO 2  k 3 (k −1 + k 2 )
dN2 O 5  k 1 N2 O 5 
= k 1 N2 O 5  − k −1 NO 2  + k 3 2 1 N2 O 5 
k k
v=−
dt (k −1 + k 2 )NO 2  k 3 k −1
v = kN 2 O 5  phản ứng bậc 1 với N2O5
d. Ta có: 0,50
k(T2 ) E A  1 1
ln =  −  = 0.693
k(T1 ) R  T1 T2 
vì: k(T2) = 2k(T1) .
Thay số vào và giải phương trình trên ta tính được: T2 = 305 K

Câu 2. (2,5 điểm) Điện, dung dịch


Một trong những thuốc thử đặc trưng để tìm ion Pb2+ (trong dung dịch) là Na2CrO4. Cho biết, kết tủa
PbCrO4 màu vàng, tan được trong dung dịch NaOH dư; trong khi đó, kết tủa PbS màu đen, không tan được
trong dung dịch NaOH. Thêm từ từ 0,05 mol Pb(NO3)2 vào 1,0 L dung dịch X gồm 0,02 mol Na2S và 0,03
mol Na2CrO4, thu được hỗn hợp Y gồm phần kết tủa và phần dung dịch (coi thể tích không thay đổi khi thêm
Pb(NO3)2 vào dung dịch X).
a. Tính pH của dung dịch X.
b. Bằng lập luận và đánh giá hợp lí, chứng tỏ rằng, pH phần dung dịch của Y xấp xỉ bằng 7,0.
2−
c. Tính [Cr2 O 7 ] và [Pb2+] trong phần dung dịch của Y.
d. Hãy thiết lập sơ đồ pin được ghép bởi điện cực chì (Pb) nhúng trong hỗn hợp Y và điện cực hiđro tiêu
chuẩn.
Cho biết:
0
pK a1(H2S) = 7,02; pK a2(H2S) = 12,90; pK − = 6,50; E Pb 2+ /Pb = -0,126 V
a(HCrO4 )

pKs(PbS) = 26,60; pKs(PbCrO4 ) = 13,70; pKs(Pb(OH)2 ) = 14,90


2− 2−
2 CrO 4 + 2H+  Cr2 O 7 + H2O K = 3,13.1014

Pb2+ + H2O  PbOH+ + H+ lg  β1 = lg β = -7,80
Pb(OH)+

Pb2+ + 2H2O  Pb(OH)2(dd) + 2H+ lg  β 2 = lg βPb(OH)2 = -17,20
− 
Pb2+ + 3H2O  Pb(OH) 3 + 3H+ lg  β3 = lg β = -28,00
Pb(OH)3−

2,303RT
(với pKa = -lgKa; pKs = -lgKs; ở 25oC: = 0,0592 V)
F
a Các quá trình xảy ra trong dung dịch X:
S2- + H2O  HS- + OH- Kb1 = 10-1,1 (1) 0,75
HS- + H2O  H2S + OH- Kb2 = 10-6,89 (2)
2− −
CrO 4 + H2O  HCrO 4 + OH- Kb = 10-7,50 (3)
Chú ý: Kb của (3) tính được từ pKa của HCrO4- và Kw(H2O).
H2O  OH- + H+ Kw = 10-14 (4)
So sánh các cân bằng (1), (2), (3) và (4) ta có:
Kb1>> Kb2 và Kb1. CS 2− >> Kb. CCrO2− >> Kw nên pHX được tính theo (1):
4

S2- + H2O  HS- + OH- Kb1 = 10-1,1 (1)


[ ] 0,02 – x x x
→ [OH-] = x = 0,0166 (M) → pH = 12,22.
b Pb2+ + S2- → PbS  K s1−1 = 1026,60 0,75
C0 0,05 0,02
C 0,03 -
Pb2+ + CrO 4
2−
→ PbCrO4  −1
K s2 = 1013,7
C0 0,03 0,03
C - -

TPGH của Y: phần kết tủa gồm PbS và PbCrO4; phần dung dịch gồm Na+ và NO 3 Từ các cân
bằng:
2− 2−
2 CrO 4 + 2H+  Cr2 O 7 + H2O K1 = 3,13.1014
2H2O  2H+ + 2OH- (KW)2 = 10-28
Tổ hợp ta được:
2− 2−
2 CrO 4 + H2O  Cr2 O 7 + 2OH- K2 = 3,13.10-14
Vì KS(PbCrO4 ) >> KS(PbS) và vì  β1 >>  β2 >>  β3 nên trong hỗn hợp Y chủ yếu xảy ra các cân
bằng:
2−
PbCrO4  Pb2+ + CrO 4 KS2 = 10-13,70 (5)
S S
(với S là độ tan của PbCrO4 trong Y)

Pb2+ + H2O  PbOH+ + H+ β1 = 10-7,80 (6)
2− −
CrO 4 + H2O  HCrO 4 + OH- Kb = 10-7,50 (7a)
2− 2−
+ H2O  Cr2 O 7 + 2OH-
2 CrO 4 K2 = 3,13.10-14 (7b)
H2O  OH- + H+ Kw = 10-14 (8)
-14
Vì K2 = 3,13.10 nhỏ, nên chấp nhận bỏ qua quá trình (7b).
So sánh các cân bằng (6), (7a) và (8) ta thấy:

β1 . S0 =  β1 Ks = 10-14,65  Kb. S0 = Kb.
Ks = 10-14,35  KW = 10-14
(với S0 là độ tan của PbCrO4 trong nước không kể các quá trình (6), (7a) và (7b)), do đó, có thể
coi khả năng cho, nhận proton của các cấu tử trong hỗn hợp Y gần như tương đương nhau, vì
vậy pHY  7,0.
Chú ý:
Việc kiểm tra cho thấy ở pH = 7,0:
−17,2 − −28
2 3
* *
[Pb(OH) 2(dd) ] 10 [Pb(OH)3 ] 10
2+
= + 2 = −14
= 1; 2+
= + 3 = −21 = 1
[Pb ] [H ] 10 [Pb ] [H ] 10

nghĩa là việc tính toán bỏ qua sự tạo phức Pb(OH)2(dd) và Pb(OH) 3 là hợp lý.
c Tính [Cr2O72-] và [Pb2+] trong phần dung dịch của Y: 0,5
Tại pH = 7,0 thì:
2− 2− 2−
[ Cr2 O 7 ] = 3,13.10-14.[ CrO 4 ]2.[OH-]-2 = 3,13.[ CrO 4 ]2
2− − 2−
Mặt khác: S = [ CrO 4 ] + [ HCrO 4 ] + 2[ Cr2 O 7 ]
2− 2−
Giả sử [ Cr2 O 7 ] << [ CrO 4 ] thì:
 h + Ka 
2−
[ CrO 4 ].   =S → 2−
[ CrO 4 ] =
Ka
.S (a)
 Ka  h + Ka
h.S
Lại có: [Pb2+] + [PbOH+] = S → [Pb 2+ ] = (b)
h + 1
*

Từ (a) và (b)
(h + K a ).(*  + h) (10−7 + 10−6,5 ).(10−7,8 + 10−7 )
 S= Ks . = 10−13,7.
h.K a 10−7.10−6,5
 S =1,744.10-7 (M)
Thay giá trị S vào (a) ta có:[CrO42-] = 1,32.10-7 (M)
→ [Cr2O72-] = 3,13.[ CrO 24− ]2 = 3,13.(1,32.10-7)2 = 5,45.10-14 (M)
2−
[Cr2O72-] << [ CrO 4 ] (thỏa mãn giả thiết đặt ra ban đầu)
Từ (b), ta có: [Pb2+] = 1,51.10-7 (M)
d Vì trong hỗn hợp Y quá trình phân li của PbCrO4 là chủ yếu, nên: 0,5
0,0592 2+
2− = E 2+ + lg[Pb ] = −0,328 (V)
0
EPb = E
PbCrO4 /Pb, CrO4 Pb /Pb
2
= −0,328 (V)  E
0
Ta có: E 2− + = 0,00 (V)
PbCrO 4 /Pb, CrO 4 2H /H 2

do đó điện cực Pb là anot, điện cực hiđro tiêu chuẩn là catot. Vậy sơ đồ pin:
-
(-) Pb│PbS, PbCrO4 , Na+ 0,1 M, NO 3 0,1 M ║ H+ 1,0 M│ H2 (p = 1 bar) | Pt (+)
Chú thích: Nếu học sinh ghi H2 (p = 1 atm) vẫn cho đủ điểm, có thể tính theo Pb2+/Pb.

Câu 3. (2,5 điểm)Nhiệt, cân bằng hóa học


PbCO3 và ZnO thường được sử dụng làm bột tạo màu trắng. H2S trong không khí có thể làm hư hại các bột
màu này do các phản ứng sau:
PbCO3 (r) + H2S (k) ⎯→ PbS (r) + CO2 (k) + H2O (h) (1)
ZnO (r) + H2S (k) ⎯→ ZnS (r) + H2O (h) (2)
a. Tính hằng số cân bằng của các phản ứng (1) và (2).
b. Cần khống chế nồng độ tối đa của H2S trong không khí bằng bao nhiêu g/m3 để các bột màu nói trên
không bị hư hại?
c. Trong 2 chất màu nói trên, chất nào ưu thế hơn khi môi trường có H2S, tại sao?
d. Bằng cách xử lí với dung dịch H2O2, có thể làm trắng lại các mảng bị đổi màu do sự hình thành PbS.
Viết phương trình của phản ứng xảy ra trong cách xử lí này.
e. Hãy chứng tỏ rằng, về mặt nhiệt động học, oxi của không khí có thể thay thế H2O2 trong phương pháp
xử lí trên.
g. Trong thực tế, ngay cả khi không khí chưa bị ô nhiễm nặng, chẳng hạn p(H2S) = 5,1.10-9 atm, mầu
trắng của PbCO3 để lâu trong không khí vẫn bị xám dần đi do sự hình thành PbS. Hiện tượng này có thể giải
thích như thế nào?
Để tính toán có thể sử dụng các dữ kiện và bảng sau: T= 298K; áp suất khí quyển p = 1,000 atm; % thể tích
của các khí và hơi trong không khí: N2 77,90; O2 20,70; CO2 0,026; H2O (h) 0,40; các khí khác: 1,03.
PbCO3(r) H2S(k) PbS(r) ZnO(r) ZnS(r) CO2(k) H2O(h) PbSO4(r) H2O2(l)
ΔfG°298
- 626,0 - 33,0 - 92,6 - 318,0 - 184,8 - 394,2 - 228,5 - 811,5 120,4
kJ/mol
Màu trắng đen trắng trắng trắng

Ý Đáp án Điểm
a Đối với phản ứng (1)
ΔG°(1) = (-92,6 – 394,2 – 228,5 + 626,0 + 33,0) kJ/mol = -56,3 kJ/mol
K(1) = e- ΔG°(1)/RT = e56300/8,314.298 = 7,4.109. 0,25
Đối với phản ứng (2)
ΔG°(2)=(-184,8 -228,5 + 318,0 + 33,0) kJ/mol = - 62,3 kJ/mol
- ΔG°(2)/RT 62300/8,314.298 10
K(2) = e =e = 8,3.10 0,25
b Đối với phản ứng (1)
2.6  10−4  4 10−3
ΔG(1)= -RTlnK(1)+ RT.ln
pH 2 S
Điều kiện để (1) ưu thế theo chiều thuận:
2.6  10−4  4 10−3
ΔG(1) =-RTlnK(1) + RT.ln <0 (a)
pH 2 S
2.6  10−4  4 10−3
→ pH2S> = 1,4.10-16 bar (b)
7, 4.109 0,25
Để bảo vệ được mầu trắng PbCO3 thì nồng độ H2S được phép trong không khí tối đa là:
34.(1,4.10-16.1000 L)/(0,082 L.bar.mol-1.K-1.298K) = 1,948.10-13 g/m3
Đối với phản ứng (2)
4 10−3
ΔG(2) = - RTlnK(2) + RT.ln
pH2 S
Điều kiện để (2) ưu thế theo chiều thuận:
4 10−3 0,25
ΔG(2) =- RTlnK(2) + RT.ln <0 (c)
pH2 S
4 10−3
→ PH2S> 10
= 4,8.10-14 bar
8,3.10
Để bảo vệ được mầu trắng ZnO thì nồng độ H2S được phép trong không khí tối đa là:
34.(4,8.10-14.1000 L)/(0,082 L.bar.mol-1.K-1.298K) = 6,68.10-11 g/m3
c ZnO ưu thế hơn vì:
- Phản ứng (1) Tự diễn biến ở những nồng độ H2S nhỏ hơn;
- Sản phẩm của (1) là PbS có mầu đen còn sản phẩm của (2) là ZnS vẫn còn là mầu trắng. 0,25
d PbS + 4H2O2⎯→ PbSO4 + H2O (3) 0,25
e PbS + 2 O2⎯→ PbSO4 (4)
ΔG° = -811,5 kJ/mol + 92.6 kJ/mol = - 718,9 kJ/mol
1
ΔG = - 718,9 kJ/mol + RT.ln = - 711,1 kJ/mol
0, 207 2
0,5
Phản ứng (4) có thể tự diển ra trong không khí ở nhiệt độ 298 K. Oxi của không khí có thể
tái tạo màu trắng bằng cách oxi hóa PbS ⎯→ PbSO4.

g Với p(H2S) = 5.1.10-9 bar thì


2.6  10−4  4 10−3
ΔG(1) = -56,3 kJ/mol + RT∙ln  -43 kJ/mol.
5,1.10−9
Trong không khí xảy ra đồng thời 2 quá trình: tạo ra và làm mất PbS.
k1
PbCO3 (r) + H2S (k) PbS + ...
O2
PbSO4
k2 0,5
Xét về phương diện nhiệt động học thì sự oxi hóa PbS bởi oxi không khí thuận lợi hơn rất
nhiều. Sự đổi màu của PbCO3có thể là do phản ứng oxi hóa PbS bởi oxi không khí bị cản
trở động học.

Câu 4. (2,0 điểm) Nhóm VA, IVA và kim loại nhóm IA, IIA, Al, Cr, Mn, Fe
1. Dựa vào cấu tạo phân tử, hãy giải thích:
a. Phân tử khí CO có năng lượng liên kết lớn (1070 kJ.mol–1), lớn hơn cả năng lượng liên kết ba trong
phân tử khí N2 (924 kJ.mol–1).
b. CO và N2 có tính chất vật lí tương đối giống nhau, nhưng có những tính chất hóa học khác nhau (CO
có tính khử mạnh hơn, có khả năng tạo phức cao hơn N2).
2. Thực nghiệm đã xác nhận tính dẫn điện tốt của bạc, đồng và vàng. Dựa vào cấu tạo nguyên tử, giải
thích kết quả đó. Thực tế, có thể dùng kim loại nhóm IA làm dây dẫn điện được không? Tại sao?
3. Nhiệt phân tinh thể không màu X ở 4500C thu được hỗn hợp Y gồm ba khí có tỉ khối so với hiđro là
40,6. Khi làm lạnh hỗn hợp Y đến 1500C thì được một chất lỏng và một hỗn hợp khí Z, có tỉ khối so với
hiđro là 20,7 và có thể tích nhỏ hơn 2,279 lần thể tích hỗn hợp Y đo ở 4500C. Hỗn hợp Z sau khi làm lạnh
đến 300C, được cho qua dung dịch kiềm dư thì chỉ còn lại một chất khí T không cháy, nhưng duy trì sự cháy.
Tỉ khối của T so với hiđro là 16 và T có thể tích nhỏ hơn 4,188 lần thể tích hỗn hợp Z ở 1500C. Viết các
phương trình phản ứng xảy ra và xác định công thức phân tử của X.
1 a. Mô tả cấu tạo phân tử CO và N2: 0,75

π π

σ σ

p p sp p
π π
Phân tử N2 Phân tử CO
Phân tử N2 có 1 liên kết  và 2 liên kết , đều được hình thành do sự xen phủ 2
obitan 2p của nguyên tử N.
Ở phân tử CO cũng có 1 liên kết  và 2 liên kết . Hai liên kết  được hình thành do
sự xen phủ 2 obitan 2p (trong đó có 1 liên kết  cho ngược từ O → C làm giảm mật độ
electron trên O). Liên kết  được hình thành do sự xen phủ obitan lai hóa sp của C với
obitan 2p của O. Đám mây xen phủ của các obitan sp – 2p lớn hơn so với mây xen phủ của
các obitan 2p-2p, nên liên kết  trong CO bền hơn liên kết  trong N2. Vì vậy năng lượng
liên kết trong phân tử CO lớn hơn năng lượng liên kết trong N2.
b. Phân tử CO, N2 là 2 phân tử đẳng electron, cấu trúc phân tử giống nhau (cùng có độ bội
liên kết bằng 3), khối lượng phân tử đều bằng 28, vì vậy chúng có tính chất vật lý giống 0,5
nhau (là chất khí không màu, không mùi, khó hóa lỏng, khó hóa rắn, ít tan trong nước).
Phân tử N2 có cặp electron chưa tham gia liên kết nằm trên obitan 2s, có mức năng
lượng thấp nên khá bền, ít tham gia vào quá trình tạo liên kết. Phân tử CO có cặp electron
chưa tham gia liên kết nằm trên obitan lai hóa sp của nguyên tử C, có năng lượng cao hơn
obitan 2s, đám mây xen phủ lại lớn nên thuận lợi cho quá trình hình thành liên kết, nguyên
tử C trong phân tử CO dễ nhường e thể hiện tính khử hoặc dễ hình thành liên kết cho nhận
khi tham gia tạo phức với các nguyên tố kim loại chuyển tiếp.

2 a. Ba nguyên tố Ag, Cu,Au ở nhóm IB trong bảng tuần hoàn, tiếp xúc với không khí ở điều 0,5
kiện thường đều trơ, vẫn tòn tại ở dạng nguyên chất. Cấu hình electron của mỗi nguyên tố
như sau:
Ag (Z=47) [Kr]4d105s1; Cu (Z=29) [Ar]3d104s1; Au (Z= 79) [Xe]5d106s1.
-Nguyên tử của mỗi nguyên tố đều có 1 electron ở vỏ hóa trị ns1 dễ dàng mất khi có diều
kiện thích hợp tạo ra ion có phân lớp bão hòa nd10 bền vững (trừ Cu có thể ở Cu+: 3d10
hoặc Cu2+: 3d9) nên chúng đều có tính dẫn điện tốt như thực nghiệm xác định.

3 Khí không cháy nhưng duy trì sự cháy có M = 32 là O2, ta có: 0,75
X → A + B + O2
→ Hỗn hợp (2) ở 150 C chứa B và O2, V2 = V(B) + V(O2); V(O2) ở 1500C lớn hơn ở 300C là:
0

423K/303K = 1,396 lần, vậy trong hỗn hợp (2) có:


VB + VO2 4,188 V 2
= =3 → B =
VO2 1,396 VO2 1
Vì khối lượng mol trung bình của hỗn hợp là 41,4
→ 2/3 M(B) + 1/3 M(O2) = 41,4 → M(B) = 46 → B là NO2 → X là nitrat.
Thể tích hỗn hợp (2) ở 4500C lớn hơn ở 1500C là 723K/423K = 1,709 lần.
VA + VNO2 + VO2 2, 279 4
Vậy ở 4500C: = = → V(A): V(NO2) : V(O2) = 1 : 2 : 1
VNO2 + VO2 1, 709 3
→ Suy ra khối lượng mol trung bình của hỗn hợp (1) là:
1/4M(A) + 1/4M(NO2) + 1/4M(O2) = 81,2
→ M(A) = 201 → A là Hg và X là Hg(NO3)2.
Phương trình phản ứng:
Hg(NO3)2 → Hg + 2NO2 + O2
2NO2 + 2NaOH → NaNO3 + NaNO2 + H2O

Câu 5. (2,5 điểm) Đại cương hữu cơ


1. Cho 3 dị vòng (hình bên). N
Hãy sắp xếp các dị vòng theo thứ tự tăng dần nhiệt độ sôi; tăng
dần tính bazơ. Giải thích. N N N
H H H
2. Cho các chất A, B có cấu tạo dưới đây:
A B C
(A) (B)
a. Gọi tên các chất A, B theo danh pháp hệ thống.
b. Giải thích tại sao A là chất lỏng phân cực còn B là chất lỏng không phân cực.
3. Vẽ các cấu dạng ghế của mỗi phân tử cis-1,3-dimetylxyclohexan (A) và trans-1,4-dimetylxyclohexan
(A) và dự đoán cấu dạng nào là bền hơn.
Tại sao chênh lệch năng lượng giữa hai cấu dạng ghế của (A) cao hơn (B)?

Ý ĐÁP ÁN Điểm


1. Nhiệt độ sôi: A < B < C
– A có vòng no, liên kết hiđro giữa nhóm –NH của dị vòng no nên rất yếu.
– B có vòng thơm, liên kết hiđro giữa nhóm –NH với dị vòng thơm chứa một
nguyên tử nitơ yếu hơn so với dị vòng thơm C có 2 nguyên tử N. 0,25
– C có vòng thơm, liên kết hiđro bền do dị vòng thơm có 2 nguyên tử N.
Tính bazơ: A > C > B
– A có tính bazơ mạnh nhất vì electron n ở N lai hóa sp3.
– C có tính bazơ trung bình vì electron n ở N lai hóa sp2.
– B không có tính bazơ không còn vì electron n ở N đã tham gia liên hợp 0,25
vòng thơm.
2. a. A: 5-(xicloprop-2-en-1-yliden)cyclopenta-1,3-dien 0,25
B: xiclopropylxiclopentan
b.
 B là một hydrocacbon thông thường có phân tử được tạo thành từ các liên kết C-
C, C-H là liên kết cộng hoá trị không phân cực. Do đó, phân tử B không phân cực. 0,25
 A cũng là hydrocacbon, nhưng A lại là chất phân cực bởi khác với B, dạng lưỡng
cực của A có tính thơm (vòng liên hợp kín, phẳng, số electron  liên hợp thoả mãn
quy tắc Huckel:  = 4n + 2) nên bền vững. Do đó, A tồn tại ở dạng lưỡng cực và làm
cho phân tử A phân cực. 0,25
+

6 2
3. Đối với (A) thì (I) bền hơn (II), đối với B thì (III) bền hơn (IV) 0,25
CH3 CH3
CH3
CH3
0,25
(I) (II)
CH3

CH3 CH3 0,25


CH3
(III) (IV)
(II) có tương tác trục-1,3 làm tăng năng lượng lên nhiều hơn so với (IV), nên chênh
lệch năng lượng giữa (I) và (II) cao hơn (III) và (IV).

Câu 6. (2,5 điểm) Sơ đồ phản ứng, cơ chế.


1. Hãy đề nghị cơ chế cho các phản ứng sau:

2. Viết công thức của các hợp chất trong các dãy phản ứng sau:
a.
b.

Hướng dẫn:

1 Mỗi gđ
0,1

2.a Mỗi
chất
0,125đ
2.b Mỗi
chất
0,125đ

Câu 7. (2,5 điểm) Tổng hợp và xác định cấu trúc chất hữu cơ
Cho 2-cabetoxixiclopentanon phản ứng với 1,3-đibrompropan khi có mặt NaH trong DMF. Sản phẩm
A nhận được được đun nóng với một đương lượng NaH trong hỗn hợp benzen - DMF cho phép thu được dẫn
xuất bixiclic B, C11H16O3.B chịu tác dụng của etanđithiol khi có mặt BF3 và Ni Raney trong metanol để hình
thành sản phẩm C. Xà phòng hoá C bằng NaOH, sau đó thuỷ phân rồi xử lí với thionyl clorua và cuối cùng
bằng NaN3 trong axeton. Đun hồi lưu hỗn hợp trên khi có mặt vết axit H+ sẽ thu được D, C8H15N. Bằng tác
dụng của fomanđehit trong axit fomic ở 1000C, D chuyển thànhE. Sau khi xử lí E bằng metyl iođua, sau đó
bằng Ag2O trong nước, đun sản phẩm thu được ở 2000C khi có mặt 1,3-điphenylisobenzofuran người ta sẽ
nhận được hai đồng phân C28H26O là (I) và (II) với hiệu suất thấp.
a. Hãy xác định công thức cấu trúc của các hợp chất trên.
b. Trình bày cơ chế của quá trình A → B và D → E.
Hướng dẫn:
a. Ta có thể tóm lược các quá trình phản ứng bằng sơ đồ sau, trong đó để ý rằng phản ứng tạo B là SN

Phản ứng tạo I và II là sản phẩm nhiệt phân Hopman sau đó đóng vòng Diels-Ander
b. Cơ chế
Từ A tạo thành B: SN

Từ D tạo thành E

HCHO H+
RNH2 RNHCH2OH H O RN = CH2 RNH CH3 + CO2
- 2 H C O H
O
Câu 8. (2,5 điểm) Hợp chất thiên nhiên
a. Rutinozơ là gốc đường của một số hợp chất có tác dụng làm bền thành mạch máu. Rutinozơ cho
phản ứng với thuốc thử Feling, khi bị thủy phân bởi  - glicozidaza cho andozơ A (C6H12O5) và D-andozơ B
(C6H12O6) theo tỉ lệ mol 1:1. Từ andozơ B tiến hành liên tiếp hai lần cắt mạch Ruff và sau đó oxi hóa với
axit nitric thu được axit meso-tactric. B dễ dàng cho dẫn xuất monoxetal với axeton trong axit. Hãy viết các
phản ứng để xác định B.
b. Andozơ B cho cùng sản phẩm osazon như một andohexozơ khác (kí hiệu là A1); A2 là đồng phân
đối quang của A1. Thực hiện chuyển hóa A2 theo sơ đồ sau thu được A.
CH3
H OH
H OH
HOCH2CH2OH H2/Ni Raney O2/Pt
A2 A3 OH H A5
xetal HO H Axit andonic
CH2OH
to Na-Hg/ PH 3-5
A6 A
Andolacton
(Lưu ý: phản ứng từ A4 đến A5 đặc trưng cho sự chuyển hóa ancol bậc 1 cuối mạch thành axit ).
Dùng CT chiếu Fisơ để biểu diễn cấu trúc của các chất A1, A2, A3, A5, A 6 và A. Biết rằng 1 mol A
phản ứng với 4 mol HIO4 cho 4 mol HCOOH và 1 mol CH3CHO.
Hướng dẫn:
a. B tiến hành liên tiếp 2 lần cắt mạch Ruff, sau đó oxi hóa thu được axit meso tatric  B có 2 nhóm
OH ở vị trí C số 4 và C số 5 cùng phía .
B + axeton  dẫn xuất monoaxetal nên nhóm -OH LK ở C số 2 và C số 3 khác phía và –OH ở C số
3 khác phía với OH LK với C số 4 và C số 5. Vậy B là :
CHO
H OH
OH H
H OH
H OH
CH2OH
Phản ứng Ruff 2 lần B :
Lần 1:
CHO COOH
CH O
H OH C= O
Br2/H2O Ca(OH)2 H2O2 HO -CO2 OH H
OH H H
H OH
H OH (CH3COO)3Fe H OH
H OH
H OH H OH
CH2OH
CH2OH CH2OH
Lần 2:
CHO
OH H CHO
Br2/H2O Ca(OH)2 H2O2 H OH
H OH
H OH (CH 3COO)3Fe H OH
-CO2
CH2OH CH2OH
b. B và A1 cho cùng 1 osazon khi tác dụng với phenylhidrazin nên B và A1 cùng vị trí C* số 3,4,5.
Công thức của A1 là : Đối quang của A1 là A2 có công thức như sau :
CHO CHO
HO H H OH
OH H H OH
H HO H
OH
H OH HO H
A1 CH2OH A2
CH2OH
Từ đó ta có sơ đồ sau:
CHO O O
H OH CH CH3
H OH HO-CH2CH2OH H OH H2/Ni Raney H OH
HO H H OH H OH O2/Pt
HO H HO H
HO H
HO H HO H
A2 CH2OH
A3 CH2OH CH2OH
CH3 CH3 CH3
H OH H OH H OH
Na-Hg
H OH H H OH
t0 PH: 3-5
HO H HO H OH H
O
HO H HO H OH H
COOH CHO A
O=C
SỞ GDĐT NINH BÌNH HDC ĐỀ THI ĐỀ XUẤT
TRƯỜNG THPT CHUYÊN KÌ THI CHỌN HỌC SINH GIỎI KHU VỰC
LƯƠNG VĂN TỤY DUYÊN HẢI NĂM 2023
******** MÔN: HÓA HỌC - KHỐI 11
Thời gian làm bài: 180 phút
( HDC này gồm 08 câu, 15 trang)
Câu 1 (2,5 điểm) Tốc độ phản ứng.
1.1. Nghiên cứu về động học phản ứng Michael của β-nitrostyrene với piperidine và trạng thái chuyển tiếp của
nó. Sơ đồ phản ứng như sau:

Phương trình tốc độ của phản ứng là:


k
v  1 .k2 .[   nitrostyrene].[ piperidine]2
k1
a) Xác định bậc toàn phần của phản ứng và bậc riêng phần đối với β-nitrostyrene và piperidine.
b) Biết rằng giai đoạn đầu của phản ứng là một cân bằng thuận nghịch, giai đoạn thứ 2 được xúc tác bởi phân
tử piperidine và là giai đoạn quyết định tốc độ phản ứng. Cả hai giai đoạn đều là phản ứng cơ bản. Thiết lập
phương trình tốc độ tạo thành sản phẩm. Tính hằng số tốc độ khả kiến.
1.2. Xét phản ứng sau: CH3X + Y → CH3Y + X (*)
Dữ liệu dưới đây cho hai thực nghiệm với phản ứng này tại 250C.
Thực nghiệm 1: [Y]0 = 3,0M Thực nghiệm 2: [Y]0 = 4,5M
[CH3X] (mol/l) t (giờ) [CH3X] (mol/l) t (giờ)
-3 -3
7,08×10 1,0 4,50×10 0
-3 -3
4,52×10 1,5 1,70×10 1,0
-3 -4
2,23×10 2,3 4,19×10 2,5
-4 -4
4,76×10 4,0 1,11×10 4,0
-5 -5
8,44×10 5,7 2,81×10 5,5
-5
2,75×10 7,0
Thực nghiệm cũng được tiến hành ở 850C, giá trị hằng số vận tốc xác định được tại nhiệt độ này là 7,88×108
(thời gian đơn vị là giờ), với [CH3X]0= 1,0×10-2M và [Y]0=3,0M.
a) Chứng minh phản ứng (*) có bậc 1 đối với CH3X. Xác định phương trình định luật vận tốc và giá trị k cho
phản ứng đó tại 250C.
b) Xác định chu kì bán hủy tại 850C.
c) Xác định năng lượng hoạt hóa Ea của phản ứng (*).
Câu 1 Nội dung Điểm
1.1 a) Bậc phản ứng theo β-nitrostyrene : 1
(1,25 Piperidine :2 0,25
điểm) Toàn phần :3
b) Tốc độ tạo thành sản phẩm có dạng: 0,25
v  k2 .[trunggian].[ piperidine] (1)
k1 [trunggian]

k1 [   nitrostyrene].[ piperidine] 0,25
Trong đó:
k1
 [trunggian]  .[   nitrostyrene].[ piperidine] 0,25
k1
Thế vào (1):
k
v  1 .k2 [   nitrostyrene].[ piperidine]2 0,25
k1
k
Hằng số tốc độ khả kiến: k  1 .k2
k1
1.2 a) Xét phương trình định luật vận tốc: v = k.[CH3X]n[Y]m (1)
(1,25 Trong cả hai thực nghiệm, Y được lấy dư nhiều so với X, nên [Y] coi như không
điểm) đổi nên phương trình (1) có thể viết
v = k’.[CH3X] (k’ = k[Y])
Xét thực nghiệm 1: 0,25
- Nếu phản ứng là bậc 1 đối với CH3X thì phương trình động học có dạng:
lnC = -kt + lnC0

Thực nghiệm 1: [Y]0=3,0M


t (giờ) [CH3X] (mol/l) ln[CH3X]
-3
1,0 7,08×10 -4,95
1,5 4,52×10-3 -5,40
-3
2,3 2,23×10 -6,11
-4
4,0 4,76×10 -7,65
5,7 8,44×10-5 -9,38
-5
7,0 2,75×10 -10,5
- Sử dụng máy tính để hồi quy tuyến tính ln[CH3X] theo t ta được:
ln[CH3X] = -0,93t – 3,99 với độ lệch r = -0,9998 (hợp lý)
Điều đó có nghĩa rằng phương trình động học bậc nhất đối với [CH3X] là phù 0,25
hợp thực nghiệm.
Xét thực nghiệm 2: 0,25
’’
Tương tự ta cũng có: ln[CH3X] = -0,92t – 5,44; k = 0,92 giờ .
-1

Thực nghiệm 2: [Y]0=4,5M


t (giờ) [CH3X] ln[CH3X]
(mol/l)
0 4,50×10-3 -5,40
1,0 1,70×10-3 -6,38
-4
2,5 4,19×10 -7,78
-4
4,0 1,11×10 -9,11
5,5 2,81×10-5 -10,48
Như vậy:
Thực nghiệm 1: ln[CH3X] = -0,93t – 3,99; k’ = 0,93 giờ-1.
Thực nghiệm 2: ln[CH3X] = -0,92t – 5,44; k’’= 0,92 giờ-1.
k ' 0,93 k .(0,30) m
Từ   1   m  0 , phản ứng là bậc không theo Y.
k '' 0,92 k .(0,45) m
Phương trình định luật vận tốc: v = k[CH3X] với k = 0,925 giờ -1 tại 250C.
b) t1/2 = ln2/k = 8,8. 10-10 (giờ -1) 0,25
c)

0,25
k  E  1 1 
ln  2   a   
 k1  R  T1 T2 
 7,88.108  Ea  1 1 
 ln     
 0,925  8,3145 J / K .mol  298 K 358K 
 Ea  3,04  105 J  3,04  102 kJ

Câu 2 (2,5 điểm) Cân bằng và phản ứng trong dung dịch. Pin điện - Điện phân.
2.1. Dung dịch X chứa Al3+ nồng độ 0,100M và Fe3+ 0,050M.
a) Có thể chuẩn độ riêng Fe3+ trong dung dịch X bằng EDTA được không khi pH trong quá trình chuẩn độ
được duy trì bằng 2. (Giả sử chuẩn độ riêng được khi hằng số bền điều kiện của phức hơn kém nhau >104 lần)?
b) Tính nồng độ cân bằng của các cấu tử khi đã thêm 25,0 mL dung dịch EDTA 0,050 M vào 25,0 mL dung
dịch X (pH của dung dịch luôn duy trì bằng 2).
Biết : EDTA có pK1 = 2,0; pK2 = 2,67; pK3 = 6,16; pK4 = 10,26;

EDTA: Ethylendiaminetetraacetic acid.


2.2. Giá trị Eo cho các bán phản ứng của các ion Fe3+ và ion Ce4+ như sau:
Fe3+ + e  Fe2+ Eo = +0,77V
Ce + e  Ce
4+ 3+
Eo = +1,61V
Hai chất chỉ thị sau được dùng để xác định điểm tương đương trong các phép chuẩn độ oxy hóa – khử:
Di-bolane (dip): Inox + 2e  Inkh ; Eodip = +0,76V
(Tím) (Không màu)
p-nitro-di-bolane (pn): Inox + 2e  Inkh ; Eodip = +1,01V
(Tím) (Không màu)
Cả hai chất chỉ thị trên đều đổi màu khi tỉ lệ nồng độ [Inox]/[Inkh] = 10. Bằng tính toán, hãy cho biết chất chỉ
thị nào ở trên thích hợp cho phép chuẩn độ Fe2+ bằng Ce4+?

Câu 2 Nội dung Điểm


2.1 Mô tả các cân bằng trong dung dịch.
(1.25 Tính hằng số bền điều kiện của các phức với EDTA
đ) 1 1
 Fe3  1
 0,597  Al3   0,995
1  *FeOH 2 .h 1  *AlOH 2 .h 1 0,25

K1.K 2 K3 K 4
 Y 4  
h 4  K1.h 3  K1.K 2 h 2  K1.K 2 K 3 h  K1.K 2 K 3 K 4
K1.K 2 K 3 K 4
= 4  3,671.10-14
h  K1.h  K1.K 2 h
3 2

'FeY  FeY .Fe .Y  2,759.1011 = 1011,44


0,25
'AlY  AlY .Al .Y  4,927.102 = 102,69.
Hằng số bền điều kiện của FeY lớn hơn của AlY 109 lần nên hoàn toàn có thể chuẩn độ riêng Fe3+
bằng EDTA.
- Khi thêm 25,0 mL dung dịch EDTA 0,05 M vào 25,0 mL dung dịch Al3+ 0,10 M và Fe3+ 0,05 0,25
M:
Vì CV = C01V0 nên chỉ chuẩn độ Fe3+
TpGH : FeY- 0,025 M, Al3+ 0,05 M
Trong dung dịch chỉ có phản ứng tạo phức cạnh tranh
Al3+ + FeY ⇄ AlY + Fe3+, 𝑲′ = 𝟏𝟎−𝟖,𝟕𝟓 0,25
𝟎, 𝟎𝟓 𝟎, 𝟎𝟐𝟓
𝟎, 𝟎𝟓 − 𝒙 𝟎, 𝟎𝟐𝟓 − 𝒙 𝒙 𝒙
 𝒙 = 𝟏, 𝟒𝟗 ⋅ 𝟏𝟎 M −𝟔 0,25
[Fe3+]' = 1,49⋅10-6 M  [Fe3+] = 8,9⋅10-7 M; [Al3+]' = 0,05 M  [Al3+] = 0,04975 M;
[Y4-]' = 6,092⋅10-8 M  [Y4-] = 2,236⋅10-21 M;
2.2. Với Di-bolane: 0,25
(1,25
Edip  Edip
o

0, 0592  In 
log ox  0, 76 
0, 0592
log10  0, 790(V )
điểm) 2  Inkh  2
Tại 0,79V, tính [Fe3+]/[Fe2+]
0, 0592  Fe3   Fe3 
E  EFe3 / Fe2 
o
log  0, 79  0, 77  0, 0592 log
1  Fe 2   Fe 2 
 Fe3 
  2,177
 Fe 2 
 Fe2  1 0,25
% [Fe ] còn lại =
2+
.100%  .100%  31, 48%
 Fe    Fe 
2 3
1  2,177
Vậy không dùng di-bolane làm chất chỉ thị được.
Với p-di-nitro-di-bolane:

E pn  E pn
o

0, 0592  In 
log ox  1, 01 
0, 0592
log10  1, 04(V )
2  Inkh  2
0,25
Tại 1,04V, tính [Fe3+]/[Fe2+]
0, 0592  Fe3   Fe3 
E  EFe3 / Fe2 
o
log  1, 04  0, 77  0, 0592 log
1  Fe2   Fe2 
0,25
 Fe3 
  3, 640.104
 Fe 2 
 Fe2  1
% [Fe ] còn lại =
2+
.100%  .100%  2, 75.103%
 Fe    Fe 
2 3
1  3, 640.10 4

Vậy với p-di-nitro-di-bolane thì % [Fe2+] còn lại là 2,75.10-3% nên p-di-nitro-di-bolane là 0,25
chất chỉ thị thích hợp.
(HS có thể làm theo cặp Ce)

Câu 3 (2,5 điểm) Nhiệt động học và cân bằng hóa học.
3.1. Ở điều kiện thường, Selen là chất rắn, phân tử gồm 8 nguyên tử selen. Selen bay hơi, tạo ra pha khí gồm
các dạng Sen cân bằng nhau (n = 2  8). Biết sinh nhiệt của Se8(k) là  f H Se0 8( k )  40,5 kcal.mol 1 . Biết hiệu
ứng nhiệt  r H 0 (kcal.mol-1) của các quá trình:
Phản ứng 3Se2(k)  Se6(k) 2Se4(k)  Se8(k) 2Se2(k)  Se4(k) Se6(k)  2Se3(k)
r H 0 -71,4 -35,5 -31,7 53,4
a) Xác định sinh nhiệt của Se6(k) và Se3(k) theo đơn vị kcal.mol-1. So sánh hai giá trị và giải thích.
b) Năng lượng trung bình mỗi liên kết trong phân tử Se6(k) là 49,4 kcal.mol-1. Xác định năng lượng liên kết
trong phân tử Se2(k)
c) Trong một thí nghiệm điều chế hơi selen, người ta nung selen thì thu được hỗn hợp các phân tử với áp suất
tương ứng như sau:
Phân tử Se8(k) Se7(k) Se6(k) Se5(k) Se4(k) Se3(k) Se2(k)
P(kPa) 12 10 9,8 8,7 6,1 2 1,5
Xác định số nguyên tử trung bình n trong phân tử khí Sen .
d) Giá trị n sẽ thay đổi như thế nào nếu:
i. Tăng áp suất
ii. Tăng nhiệt độ
Câu 3 Nội dung Điểm
3.1 a) 0,75 điểm 0,25
(1,5 Xét quá trình:
điểm) r H10  r H 20 r H 30  r H 40 4
Se8( r )   Se8( k )   2Se4( k )   4Se2( k )   Se6( k )
3
Như vậy: 4/3.  f H Se0 6( k ) =  r H10   r H 20   r H 30   r H 40
= 40,5 + 35,5 + 2.31,7 + 4/3.(-71,4) = 44,2 kcal.mol-1
Suy ra,  f H Se0 6( k ) = 33,15 kcal.mol-1
Xét quá trình:
r H 50 8 r H 60 8
Se8( r )   Se6( k )   Se3( k )
6 3
Như vậy: 8/3.  f H Se3( k ) =  r H 5   r H 6 = 8/6.33,15 + 8/6.53,4
0 0 0
0,25
Suy ra,  f H 0
Se3( k )
-1
= 43,275 kcal.mol .
Nhận xét:  f H Se0 3( k ) >  f H Se0 6( k ) do phân tử Se3(k) có dạng tam giác, các góc bị biến
dạng, chịu sức căng lớn nên phân tử kém bền 0,25
b) 0,25 điểm 0,25
Phân tử Se6(k) có cấu tạo lục giác đều nên quá trình 3Se2(k)  Se6(k) có:
r H o  3ESeSe ( Se2 )  6.ESeSe ( Se6 )  ESeSe( Se2 )  75kcal.mol 1
c) 0,25 điểm 0,25
Vì được đo trong cùng điều kiện nhiệt độ và áp suất nên tỉ lệ về áp suất cũng chính là tỉ
lệ về số mol. Vì vậy, phân tử khối trung bình của phân tử khí Selen Sen là
12.8.79  10.7.79  9,8.6.79  8, 7.5.79  6,1.4.79  2.3.79  1,5.2.79
M = 475,5
12  10  9,8  8, 7  6,1  2  1,5
g.mol-1.
Suy ra, n = 6,02
d) 0,25 điểm 0,25
Tăng áp suất thì cân bằng chuyển dịch theo chiều làm giảm số mol khí nên phân tử
selen có khối lượng phân tử lớn sẽ chiếm ưu thế, vì vậy, n sẽ tăng.
Tăng nhiệt độ, cân bằng chuyển dịch theo chiều phản ứng thu nhiệt, phá vỡ liên kết tạo
ra các phân tử selen có khối lượng phân tử nhỏ hơn, vì vậy, n giảm.

3.2. Khi đốt cháy 3,9 gam hơi benzen ở 250C, 1 atm với một lượng oxi dư toả ra 163400 J, sản phẩm gồm
CO2(k) và H2O(l).
a) Tính nhiệt toả ra khi đốt cháy 7,8 gam hơi benzen bằng oxi dư trong không khí ở 250C, sản phẩm gồm CO2(k)
và H2O(l).
b) Tính nhiệt độ của ngọn lửa 7,8 gam hơi benzen cháy ở áp suất 1 atm, 250C trong không khí (chứa 20% oxi
và 80% nitơ về thể tích). Biết tại nhiệt độ đó nước tạo ra ở thể khí.
Biết: C0P (CO2 khí) = 26,80 + 42,3.10-3T (J/mol.K);
C0P (N2 khí) = 27,10 + 6,00.10-3T (J/mol.K);
H 0 bay hơi của nước lỏng ở 373 K là 40,66 (kJ/mol);
C0P của nước lỏng là 75,3 (J/mol.K);
C0P của nước khí là 30,2 + 1,00.10-2T (J/mol.K).
3.2 Nội dung Điểm
(1,0
điểm)

a) Ở 298K: C6H6(h) + 7,5O2(k)  6CO2(k) + 3H2O(l)


- Nếu phản ứng này xảy ra ngoài không khí thì nhiệt toả ra:
163400 0,25
QP = H = = -3268000 J/mol = -3268 kJ/mol.
3,9
78
b) nhiệt độ ngọn lửa >373K (Nước tạo ra ở khí). Nhiệt toả ra do phản ứng đốt cháy 1
mol hơi benzen dùng để: 0,25
- Tăng nhiệt độ của 7,5 mol N2 (nếu có) và 6 mol CO2 từ nhiệt độ đầu đến nhiệt độ
ngọn lửa;
- Tăng nhiệt độ của 3 mol nước lỏng từ 298K đến nhiệt độ sôi;
- Chuyển pha 3 mol nước lỏng tại 373K;
- Tăng nhiệt độ của 3 mol nước hơi từ 373K lên nhiệt độ ngọn lửa.
Nếu đốt cháy benzen ngoài không khí thì:
15
CP của N2 và CO2 = 6CPCO + 4. CPN = 973,8 + 0,4338.T
2
2 2

T 373 T
 -H =  (973,8  0, 4338.T)dT   3C PH2O(l) dT  3H hoùa hôi H2O   3C PH2O(l) dT
298 298 373 0,25
T 373 T

 (973,8  0, 4338.T)dT   3.75,3dT  3.40, 66.1000   3.(30,2 + 10


2
= T)dT
298 298 373

 3268.1000 = 973,8.(T – 298) + 0,2169(T – 298 ) + 225,9.(373 – 298) + 121980 +


2 2

90,6(T – 373) + 0,015.(T2 – 3732) = 1064,4T + 0,2319T2 – 167889,0474


 1064,4T + 0,2319T2 – 3474412,223 = 0 0,25
 T = 2204,96 K
Câu 4 (2,5 điểm) Hóa nguyên tố (Kim loại, phi kim nhóm IVA, VA). Phức chất.
1. Amoniac lỏng được sử dụng rộng rãi làm dung môi cho các phản ứng mà gần như không thể thực hiện
được trong nước. Một số các chuyển hóa được cho trong sơ đồ sau. Hợp chất C có thể được tạo thành bằng
cách hòa tan oxit A trong amoniac lỏng. Phản ứng của C với KNO3 ở 100oC tạo thành hai hợp chất B và D.
Sản phẩm rắn duy nhất của phản ứng giữa C với F là muối E. Tương tác của C với PCl5 tạo thành ba chất rắn.
A, G, E, D là các hợp chất lưỡng nguyên tố. Biết % khối lượng oxi trong B là 28,57%.

a) Xác định các chất A-G, biết rằng tất cả các phản ứng được thực hiện trong amoniac lỏng. Chú ý rằng chỉ
các chất (A-G) là chất rắn ở 20oC.
b) Hòa tan G trong nước nóng thu được các muối X và Y có thành phần định tính giống nhau với tỉ lệ 2:1. Xác
định công thức các chất X, Y.
4.2.
a) Các peroxide của Cr có số oxi hóa cao thường kém bền và dễ phân hủy, nhưng Cr(O2)2[NH(C2H4NH2)2] thì
lại rất bền. Phức chất này vẫn giữ được những đặc trưng cấu trúc của peroxide. Xác định số oxi hóa của Cr, vẽ
cấu trúc của phức chất trên.
b) Phối tử L là một hợp chất hữu cơ được tổng hợp từ bippyridin và H2O2. L có khối lượng phân tử là 188.
Viết cấu tạo của L, biết bippyridin cũng có phản ứng tương tự pyridin:
C5H5N + [O]  C5H5NO
Phức chất của L với Fe có công thức là FeLm(ClO4)m.3H2O (A). Phức chất của L với Cr có công thức là
CrLxCly(ClO4)z.H2O (B). Hàm lượng nguyên tố của A và B được xác định như sau:
A: 5,47% Fe; 37,03% C; 3,09% H; 10,94% Cl; 8,64% N.
B: 8,44% Cr; 38,93% C; 2,92% H; 17,25% Cl; 9,08% N.
c) Xác định công thức phân tử, công thức cấu tạo của A, B. Biết giá trị momen từ của A là 6,13 và của B là
3,88.
Câu 4 Hướng dẫn chấm Điểm
4.1 a) A: K2O; B: KOH; C: KNH2; D: KN3; E: KCl; F: NH4Cl; G: P3N5. 0,75/8
(Phương trình phản ứng: không yêu cầu HS viết chất
(1,0điểm) K2O + NH3 → KNH2 + KOH
100o C
2KNH2 + KNO3   KN3 + 2KOH + H2O
KNH2 + NH4Cl → KCl + 2NH3
KNH2 + PCl5 → P3N5 + NH4Cl + KCl
KNH2 + N2O → KN3 + H2O
b) X: (NH4)2HPO4; Y: NH4H2PO4.
0,25
4.2
(1,5 điểm)
a. Cr có số oxi hóa +4

0,25

b. L được tổng hợp từ bippyridin và trong phản ứng bippyridin bị oxi hóa tương tự
pyridin.
Mbippyridin = 156; ML = 188 ⟹ ML - Mbippyridin = 32 ⟹ công thức phân tử của L là 0,25
C10H8N2O2.
Cấu tạo của bippyridin:

Cấu tạo của L là: (HS chỉ cần vẽ 1 trong các công thức dưới đây) 0,25

c) Phức chất của L với Fe có công thức là FeLm(ClO4)m.3H2O (A)


A: 5,47%Fe; 37,03%C; 3,09%H; 10,94%Cl; 8,64%N.
% Fe %C %Cl 5, 47 37, 03 10,94
: :  : :  0, 0977 : 3, 0858 : 0,3082 = 1: 31: 3
56 12 35,5 56 12 35,5
Thay vào ta có m  3 ⟹ công thức của A là FeC30H30Cl3N6O21 hay
[FeL3](ClO4)3.3H2O.
0,25
Tương tự cho phức B ta có công thức của phức B là: [CrL2 Cl2](ClO4).H2O (B).
Theo đề:
- Momen từ của A là 6,13 = n(n  2)  n  5 ⟹ cấu hình của Fe3+ tương ứng
0,25
với với cấu hình spin cao: t2g3eg2.
- Momen từ của B là 3,88 = n(n  2)  n  3 ⟹ cấu hình của Cr3+ tương ứng
với cấu hình: t2g3eg0.
Vậy cấu tạo của A, B là:

0,25

(A) (B)
Câu 5 (2,5 điểm) Đại cương hữu cơ
5.1. Khi thay thế một nguyên từ H trong hợp chất A3 bằng một nguyên tử Br (hợp chất A4) thì có sự giảm về
momen lưỡng cực. Hãy giải thích sự thay đổi đó.

5.2.
a) Giải thích tại sao: ở pH thấp hơn 2, hoặc cao hơn 12 các nhóm thế của axit cis-3-aminoxiclohexanoic đều
chiếm vị trí e còn ở pH 6 – 9 thì cả hai nhóm thế đều chiếm vị trí a.
b) So sánh tính axit hai đồng phân cis và trans của xiclohexa-1,3-diol và giải thích.
c)1,8-Bis=(dimetylamino) naphtalen (pKa = 12,8) là một bazơ hữu cơ rất mạnh
mặc dù nguyên tử N gắn trực tiếp vào vòng thơm. Tuy nhiên tính nucleophin của
nó rất yếu. Thực nghiệm cho thấy nó dễ dàng deproton hóa phenol, nhưng lại
không thể deproton hóa trinitrometan (là axit mạnh hơn phenol). Hãy giải thích
tính bazơ mạnh bất thường, tính nucleophin yếu của nó và cho biết lý do tại sao
nó không thể deproton hóa được trinitrometan.
Câu 5 Nội dung Điểm
5.1 Cấu trúc trong hệ có tính thơm, chiều của momen lưỡng cực hướng về phía nguyên 0,25đ
(0,5 tử oxy C=O. Chính vì vây việc gắn thêm brom sẽ làm giảm momen lưỡng cực tổng
điểm) của phân tử (Do liên kết C-Br ngược phía C-O làm giảm momen lưỡng cực của phân
tử).

0,25đ

5.2 a. Ở pH thấp hẳn hoặc cao hẳn, các nhóm thế phân bố ở e để tránh tương tác không 0,25đ
(2,0 gian, nhưng ở pH từ 6-9 thì nhóm amin ở dạng NH3+, nhóm axit ở dạng COO-. Điều
điểm) này dẫn đến việc các nhóm thế ở a sẽ bền hơn do tương tác hút tĩnh điện. 0,25đ

b. Đồng phân cis aa có liên kết hydro bền vững sẽ bền hóa anion RO- sinh ra nên có 0,25đ
tính axit cao hơn tất cả các cấu dạng còn lại

0,25đ

c.
- Các cặp electron của N không liên hợp được với vòng thơm do lực đẩy giữa chúng
0,25đ
làm mất tính song song với các e của vòng thơm nên tính bazơ mạnh bất thường.
- Dễ dàng deproton hóa phenol, nhưng lại không thể deproton hóa trinitrometan (là
axit mạnh hơn phenol):
Ban đầu các cặp e của N đẩy nhau mạnh tạo nên sức căng, cation sinh ra có liên kết 0,25đ
hydro nội phân tử triệt tiêu được sức căng này. Tuy nhiên do cấu trúc cồng kềnh nên
nó không thể deproton hóa được trinitrometan. 0,25đ

0,25đ
- Tính Nucleophin yếu do cặp e hướng vào phía trong, khó tiếp cận tác nhân
electrophin.

Câu 6 (2,5 điểm) Sơ đồ tổng hợp hữu cơ. Cơ chế phản ứng hóa hữu cơ.
Gephyrotoxin là một hợp chất tự nhiên có ở da của loài ếch nhiệt đới Dendrobates histrionicus ở Colombia.
Hợp chất này không độc và tác động tới hoạt động của hệ thần kinh. Gephyrotoxin được tổng hợp theo sơ đồ
phản ứng sau:

Hãy xác định cấu trúc các hợp chất từ A đến N trong sơ đồ tổng hợp Gephyrotoxin.
Câu 6 Nội dung Điểm
(2,5
điểm)
A: 0,1đ
B→N:
0,2đ/chất
Câu 7 (2,5 điểm) Xác định cấu trúc các chất hữu cơ (mô tả sơ đồ tổng hợp bằng lời dẫn)
Hidrocacbon A (C6H10) không có đồng phân lập thể, 1 mol A chỉ làm mất màu 1 mol KMnO4 (dung dịch nước)
hoặc 1 mol Br2 (trong dung dịch CCl4) ở nhiệt độ thường. A phản ứng với lượng dư H2/xúc tác Ni tạo thành
các hợp chất là đồng phân cấu tạo của nhau có cùng CTPT C6H14. Trong dung dịch axit H3PO4 50%, A chuyển
thành C (C6H12O) không làm mất màu dung dịch KMnO4 hoặc dung dịch Br2/CCl4 ở nhiệt độ thường. Chế hóa
C với CrO3/piridin thu được D.
7.1. Xử lý D với m-CPBA thu được 2 sản phẩm E1 và E2 là đồng phân cấu tạo của nhau có cùng CTPT
C6H10O2, trong đó E1 là sản phảm chính. Khử hóa E1 và E2 bằng LiAlH4 thu được F1 và F2 có cùng CTPT
C6H14O2; F1 hoặc F2 phản ứng với PCC hoặc C5H5N.SO3 thu được sản phẩm tương ứng X1 và X2, trong đó
X1 có phản ứng idofom. Xác định CTCT của các chất nêu trên.
7.2. Khi cho D tác dụng với hidroxiamin ở pH khoảng 5-6 thu được G. Xử lý G với PCl5 thu được H1 và H2
là đồng phân cấu tạo của nhau có cùng CTPT C6H11NO, trong đó H1 là sản phẩm chính. Cho H1 hoặc H2
phản ứng với LiAlH4, rồi đun nóng sảm phẩm thu được với xúc tác Se thu được sản phẩm tương ứng là I1 và
I2 có cùng công thức C6H7N không làm mất màu dung KMnO4 hoặc dung dịch Br2/CCl4 ở nhiệt độ thường.
Hãy xác đinh CTCT của các chất nêu trên biết rằng I1 phản ứng được với 4-nitrobenzandehit khi có mặt Ac2O
làm xúc tác cho E (C13H10N2O2).
Câu 7 Nội dung Điểm
(2,5
điểm)
7.1 Hidrocacbon A (C6H10, ∆= 2) không có đồng phân lập thể, 1 mol A chỉ làm mất 9 chất
(1,3 màu 1 mol KMnO4 (dung dịch) hoặc 1 mol Br2 (trong dung dịch CCl4) ở nhiệt
điểm) độ thường. A phản ứng với lượng dư H2/xúc tác Ni tạo thành các hợp chất là D: 0,1đ
đồng phân cấu tạo của nhau có cùng CTPT C6H14(∆= 0) => A có 1 vòng 4 cạnh
và có 1 liên kết đôi. Còn lại:
=> Các CTCT có thể có của A là: 0,15đ/chất

A1
A2 A3 A4

Trong dung dịch axit H3PO4 50%, A chuyển thành C (C6H12O) không làm mất
màu dung dịch KMnO4 hoặc dung dịch Br2/CCl4 ở nhiệt độ thường. Chế hóa C
với CrO3/piridin thu được D. Nên:

OH O
A: C: D:

O LiAlH4 OH PCC O
O
m-CPBA O OH
F1 O
E1 X1
OH O
D O
LiAlH4 PCC
O OH O
E2 F2
X2

7.2 6 chất
O N N
NH2OH PCl5 O
OH Cl CV NH +
(1,2 D
NH 0,2đ/chất
O
điểm) H1 H2

(G)
O

NH LiAlH4 NH Se N NO2 NO2


N
O Ac2O E
H1
H

O
LiAlH4 Se
NH
NH N
H2

Câu 8 (2,5 điểm) Hóa học các hợp chất thiên nhiên (Cacbohidrat và các hợp chất hữu cơ chứa nito đơn
giản)
8.1. Geniposit (hình dưới) là một hợp chất được tách ra từ quả dành dành. Thuỷ phân geniposit sinh
ra hai sản phẩm là genipin và D-glucozơ. Genipin tham gia phản ứng tạo màu với gelatin (đây là cơ
sở để phát hiện dấu vân tay trong kỹ thuật hình sự). Hãy viết sơ đồ phản ứng tạo genipin và phản ứng
của genipin với một aminoaxit để giải thích hiện tượng trên.
COOCH3

O
HOH2C
O
HO O
HO CH2OH
OH
8.2. Đisacarit X là một đường có tính khử. Thủy phân hoàn toàn 1 mol X cho 1 mol D-glucozơ và 1 mol D-
mannozơ. Metyl hóa hoàn toàn X bằng CH3I/Ag2O thu được hợp chất B không còn tính khử. Đun nóng B trong dung dịch
axit HCl loãng thu được C (2,3,6-tri-O-metyl của D-glucozơ) và D (2,3,4,6-tetra-O-metyl của D-mannozơ). Biết rằng X có
liên kết -1,4-glicozit. Xác định cấu trúc vòng Havooc, cấu dạng bền nhất và tên hệ thống của X.
Câu 8 Nội dung Điểm
(2,5 điểm)
8.1 Phản ứng thuỷ phân geniposit thu được genipin và D-glucozơ Viết đúng
(1,5 điểm) COOCH3 sản phẩm
COOCH3 OH 0,5 điểm
O
+ HO O
HOH2C OH
O O
HO
HO HOH2C OH
O OH

HO CH2OH
OH
Gelatin (có trong da) cấu tạo từ các polipeptit, lấy đại diện là một aminoaxit Viết đúng
như glyxin, ta có phương trình: sản phẩm
COOCH3 COOCH3 0,5 điểm
+ H2N-CH2-COOH
O N-CH2-COOH
Chỉ ra sản
HOH2C HOH2C
OH OH phẩm có
sản phẩm có màu để phát hiện dấu vân tay trong kỹ thuật hình sự. màu 0,5đ
8.2 X là đường có tính khử  X còn –OH hemiaxetal Cấu trúc
(1,0 điểm) Thủy phân hoàn toàn X cho 1 mol D-glucozơ và 1 mol D-mannozơ nên vòng: 0,5đ
X tạo ra từ 1 phân tử glucozơ và 1 phân tử mannozơ liên kết -1,4- glicozit Cấu dạng
1 mol X  1 mol D- glucozơ + 1 mol D-Mannozơ bền nhất:
0,25đ
Tên: 0,25đ

Thứ tự liên kết là -D- mannozơ và (,)-D-glucopiranozơ liên kết


-1,4-glicozit

4-O-(-D-Mannopiranozyl)-(,) -D-Glucopiranozơ

...........HẾT...........

Giáo viên ra đề và làm HDC

1. Trần Thị Liên, SĐT: 0913981122


2. Nguyễn Thị Thảo, SĐT: 036 2278702
SỞ GD&ĐT HƯNG YÊN KỲ THI CHỌN HỌC SINH GIỎI
TRƯỜNG THPT CHUYÊN HƯNG YÊN KHU VỰC DUYÊN HẢI & ĐỒNG BẰNG BẮC BỘ
LẦN THỨ XIV
ĐỀ ĐỀ XUẤT MÔN: HÓA HỌC - LỚP 11
Thời gian làm bài: 180 phút, không kể thời gian giao đề

Câu 1 (2,5 điểm). Tốc độ phản ứng.


Trong tầng bình lưu, nguyên tử Cl* có thể phân hủy tầng ozon theo phương trình sau:
Cl* + O3 ⎯⎯
ka
→ ClO* + O2 (a) với ka = 1,7.1010(M-1.s-1).exp(-260/T).
1. Hãy tính tốc độ phản ứng ở khoảng cách 20 km, khi đó nồng độ các chất :
[Cl]= 5.10-17M; [O3] = 8.10-9 M và T = 220K.
2. Ở độ cao 45 km, nồng độ thực tế các chất là [Cl]= 3.10-15M; [O3] = 8.10-11 M và T =
270K, hãy tính tốc độ phản ứng tại đó.
3.Tại 298K, hằng số tốc độ phản ứng (a) đo được là ka = 6,7.109M-1.s-1; thừa số
Arrhenius,
A = 6,7.1010M-1.s-1. Hãy tính năng lượng hoạt hóa của phản ứng ở 298K.
4. Phản ứng lưỡng nguyên tử pha khí với các sản phẩm khác nhau của gốc clomonoxit:
ClO* + ClO* ⎯
⎯→ Cl2 + O2 k1 = 2,9.106
ClO* + ClO* ⎯
⎯→ ClOO* + Cl* k
2 = 4,8.106.
ClO* + ClO* ⎯
⎯→ OClO* + Cl* k3 = 2,1.106.
Hãy xác định hiệu suất cho sản phẩm không chứa gốc tự do.
5. Biết phản ứng xảy ra ở phần (4), chất đầu là (ClO)2 phân hủy nhanh thành ClO*rồi sau
đó phân hủy theo 3 hướng sản phẩm trên. Biết thừa số Arrhenius và năng lượng hoạt hóa
của các phản ứng tại 298K là:
Phản ứng A (M-1.s-1) Ea (KJ/mol)
1 6,08.108 13,2
2 1,79.1010 20,4.
3 2,11.108 11,4.
Cho biết phản ứng nào có cho ∆H* lớn nhất và phản ứng nào cho ∆S* nhỏ nhất, tính các
k .T G *
giá trị đó. Cho biểu thức của Eyring: k = h exp(- RT ) và ∆U* = Ea – RT;
B

kB = 1,38.1023(J/K), h = 6,62.10-34(J.s).
Câu 2 (2,5 điểm). Cân bằng và phản ứng trong dung dịch. Pin điện
1. Nước ô nhiễm sắt thường có mùi “tanh” và không sử dụng được do ảnh hưởng tới sức
khỏe. Một mẫu nước giếng khoan (nước ngầm) ô nhiễm sắt ở dạng Fe2+ xác định được
nồng độ là 25 ppm.
a. Tính nồng độ Fe2+ theo đơn vị mol.L-1 . Biết rằng MFe = 55,85 g.mol-1 và 1 ppm = 1
mg.L-1.
b. Tính pH của mẫu nước ô nhiễm sắt. Coi các chất khác không ảnh hưởng tới pH của hệ.
c. Khi được hút lên và để tiếp xúc với không khí đủ lâu thì sắt(II) trong nước sẽ bị oxi
hóa hoàn toàn thành sắt(III). Khi đó một phần sắt(III) sẽ chuyển thành kết tủa Fe(OH) 3.
Hỏi có sử dụng mẫu nước sau khi cho tiếp xúc với không khí làm nước sinh hoạt được
hay không? Biết hàm lượng cho phép của ion sắt trong nước sinh hoạt là 0,3 mg.L -1 và
pH của nước không thay đổi.
Cho các giá trị nhiệt động ở 25oC:
Fe(OH)+ có -lgβ = 5,92; Fe(OH)2+ có -lgβ = 2,17; Fe(OH)3 có pKS = 37.
2. Để xác định độ tan của các muối khó tan người ta thường dùng kĩ thuật điện hóa
học. Một pin điện hóa gồm 2 điện cực được nối với nhau bằng cầu muối. Điện cực thứ
nhất gồm một thanh Zn nhúng trong dung dịch Zn(NO3)2 0,200 M, điện cực thứ hai gồm
một thanh Ag nhúng trong dung dịch AgNO3 0,100 M. Mỗi dung dịch có thể tích 1,00 L tại
250C.
a. Viết sơ đồ pin và phản ứng xảy ra trên từng điện cực, phản ứng xảy ra trong pin khi
pin hoạt động.
b. Tính sức điện động của pin.
c. Tính điện lượng phóng thích trong quá trình pin phóng điện (giả sử pin phóng điện
hoàn toàn và lượng Zn dư).
Câu 3 (2,5 điểm). Nhiệt động học và cân bằng hóa học.
1. Chu trình Brighton với 1 mol helium được biễu diễn ở hình sau:
Biết: Quá trình 1 ⟶ 2 và 3 ⟶ 4: đoạn nhiệt thuận nghịch.
Quá trình 2 ⟶ 3 và 4 ⟶ 1: đẳng áp thuận nghịch.
a) Hãy chọn ra chiều của chu trình (thuận hay ngược chiều kim đồng hồ) diễn ra với
động cơ nhiệt (sinh công).
b) Vẽ (định tính) chu trình trong hệ tọa độ T – S (S là entropy).
x
 
c) Hiệu suất của chu trình Brighton là η = 1 -  p1  . Xác định x biết p1 và p2 là áp
 p2 

suất khí ở điểm 1 và 2 tương ứng. Tính hiệu suất của chu trình trên.
d) Một chu trình tương tự được bắt đầu theo chiều ngược lại để nó hoạt động như
một máy lạnh. Hãy xác định hiệu suất máy lạnh trong trường hợp này.
2. a) Khi tăng áp suất 107 Pa thì nhiệt độ nóng chảy của nước đá (băng) giảm đi
0,744oC. Ở áp suất khí quyển, nước đá nóng chảy ở 0oC. Khối lượng riêng của nước đá là
0,9170 g.cm-3, nước lỏng là 0,9998 g.cm-3. Tính enthalpy nóng chảy (∆fusH) của nước đá.
Giả sử khối lượng riêng của nước lỏng và nước đá không thay đổi trong khoảng nhiệt độ
khảo sát.
Cho nguyên tử khối: H = 1,0079; O = 15,9994.
b) Ở nhiệt độ nào thì áp suất bão hòa của nước và acetic acid bằng nhau. Biết: ở áp
suất khí quyển nhiệt độ sôi của nước và acetic acid tương ứng là 100 oC và 118oC;
Enthalpy hóa hơi của nước và acetic acid trong khoảng nhiệt độ từ 50-100oC tương ứng là
42,0 kJ.mol-1 và 25,0 kJ.mol-1.
Câu 4 (2,5 điểm). Hóa nguyên tố (Kim loại, phi kim nhóm IVA, VA). Phức chất.
1. Chỉ thị axit–bazơ thường là các axit hoặc bazơ yếu. Để xác định hằng số phân li
axit của chỉ thị, ta dùng phương pháp đo quang. Chỉ thị HIn là một đơn axit yếu có pK a ≈
8, chỉ thị này có dạng HIn hấp thụ quang cực đại ở bước sóng 520 nm còn dạng ánh sáng
ở bước sóng này yếu hơn. Để xác định chính xác hằng số pK a của chất chỉ thị HIn, người
ta chuẩn bị 3 dung dịch chất chỉ thị có cùng nồng độ mol/l nhưng được điều chỉnh ở các
pH khác nhau. Tiến hành đo mật độ quang của 3 dung dịch đó tại bước sóng 520 nm, kết
quả được đưa ra ở bảng sau:
pH 1,0 7,4 13,0
Mật độ quang
0,90 0,64 0,10
A
Xác định hằng số phân li axit Ka của chất chỉ thị này.
2. Cho sơ đồ chuyển hóa sau:

Viết các phương trình phản ứng tương ứng với các chuyển hóa đó. Biết rằng mỗi mũi tên
ứng với một phản ứng hóa học.
Câu 5 (2,5 điểm). Đại cương hữu cơ.
Cho các công thức cấu tạo sau:
Me 1 2 4 5 Me Me OH Me O COOH
C(CH=CHF)2 3 N
Et O O Me
CH CH2F O 2N O O 2N OH NH2
O N
(A) (B) (C) (D) H (E)

1. Hãy vẽ công thức các đồng phân lập thể ứng với cấu tạo A.
2. Ứng với công thức cấu tạo B có bao nhiêu đồng phân lập thể, vì sao? Dùng các kí hiệu
thích hợp để chỉ rõ cấu hình của mỗi đồng phân đó.
3. Hãy viết cơ chế phản ứng để giải thích vì sao C và D khi tương tác với dung dịch
NaOH thì đều tạo thành natri 3-metyl-4-nitrobenzoat.
4. Hãy chỉ rõ trạng thái lai hóa của từng nguyên tử N ở cấu tạo E và ghi giá trị pKa (ở 25
o
C): 1,8; 6,0; 9,2 vào từng trung tâm axit trong công thức tương ứng với E, giải thích.
Câu 6 (2,5 điểm). Sơ đồ phản ứng và cơ chế phản ứng.
1. Viết cơ chế giải thích sự hình thành sản phẩm trong các phản ứng sau:
a.

b.

2. Viết công thức cấu tạo của các chất từ C đến C5 và hoàn thành sơ đồ chuyển hóa sau:

3. Veticadinol là một sesquiterpene tự nhiên nhóm candinane được phân lập từ tinh dầu
thực vật. Veticadinol được tổng hợp toàn phần bởi L.F. Tietze theo sơ đồ dưới đây:

Câu 7 (2,5 điểm). Xác định cấu trúc.


Hidrocacbon A (C6H10) không có đồng phân lập thể, 1 mol A chỉ làm mất màu 1 mol
KMnO4 (dung dịch nước) hoặc 1 mol Br2 (trong dung dịch CCl4) ở nhiệt độ thường. A
phản ứng với lượng dư H2/xúc tác Ni tạo thành các hợp chất là đồng phân cấu tạo của
nhau có cùng CTPT C6H14. Trong dung dịch axit H3PO4 50%, A chuyển thành C (C6H12O)
không làm mất màu dung dịch KMnO4 hoặc dung dịch Br2/CCl4 ở nhiệt độ thường. Chế
hóa C với CrO3/piridin thu được D.
a) Xử lý D với m-CPBA thu được 2 sản phẩm E1 và E2 là đồng phân cấu tạo của
nhau có cùng CTPT C6H10O2, trong đó E1 là sản phảm chính. Khử hóa E1 và E2 bằng
LiAlH4 thu được F1 và F2 có cùng CTPT C6H14O2; F1 hoặc F2 phản ứng với PCC hoặc
C5H5N.SO3 thu được sản phẩm tương ứng X1 và X2, trong đó X1 có phản ứng idofom.
Xác định CTCT của các chất nêu trên.
b) Khi cho D tác dụng với hidroxiamin ở pH khoảng 5-6 thu được G. Xử lý G với
PCl5 thu được H1 và H2 là đồng phân cấu tạo của nhau có cùng CTPT C6H11NO, trong
đó H1 là sản phẩm chính. Cho H1 hoặc H2 phản ứng với LiAlH4, rồi đun nóng sảm phẩm
thu được với xúc tác Se thu được sản phẩm tương ứng là I1 và I2 có cùng công thức
C6H7N không làm mất màu dung KMnO4 hoặc dung dịch Br2/CCl4 ở nhiệt độ thường.
Hãy xác đinh CTCT của các chất nêu trên biết rằng I1 phản ứng được với 4-
nitrobenzandehit khi có mặt Ac2O làm xúc tác cho K (C13H10N2O2).
Câu 8 (2,5 điểm). Hợp chất thiên nhiên.
1. Một cacbohiđrat A có công thức phân tử C12H20O11. A phản ứng được với thuốc
thử phenylhiđrazin, không làm mất màu dung dịch Br 2/H2O, không tham gia phản ứng
với thuốc thử Tollens. Thủy phân A bằng dung dịch axit thu được hợp chất A1 và D-
fructozơ. Metyl hóa A1 bằng MeOH (xúc tác HCl khan) thu được hợp chất A2 (C7H12O6).
Tiến hành oxi hóa A2 bằng dung dịch HIO4 rồi thủy phân sản phẩm thu được hỗn hợp,
trong đó có chứa các chất A3, A4, A5.

Đun nóng A với dung dịch NaOH thu được hỗn hợp sản phẩm, trong đó có B1 (C6H8O5).
Oxi hóa B1 bằng HIO4 và đun nóng nhẹ thu được hợp chất B2 (C5H8O4). Metyl hóa B2
bằng MeI/Ag2O thu được hợp chất B3 (C7H14O5). Mặt khác, hiđro hóa B2 bằng H2/Ni, to
thu được hợp chất B4 (C5H12O4).
a) Xác định và biểu diễn cấu trúc của A, A1, A2, B1, B2, B3, B4 ở dạng vòng Haworth (với
hợp chất mạch vòng) hoặc công thức Fisơ (với hợp chất mạch hở).
b) Đề xuất cơ chế quá trình chuyển hóa A thành B1 trong dung dịch NaOH.
2. Peptit E có khả năng ức chế một số chủng vi khuẩn. Để xác định cấu trúc bậc I
của E, có thể tiến hành thí nghiệm như sau: Peptit E không cho phản ứng với 2,4-
đinitroflobenzen và không bị thủy phân bởi enzym aminopeptiđaza. Thủy phân hoàn toàn
E thu được các amino axit như sau: Gly3, Pro2, Phe, Tyr. Khi thủy phân không hoàn toàn
E bằng enzym pepsin, thu được tetrapeptit E1 và tripeptit E2. Thành phần của E1 gồm
Phe, Gly2 và Pro, thành phần của E2 gồm Tyr, Gly và Pro. Mặt khác, thủy phân không
hoàn toàn E trong dung dịch axit loãng thu được hỗn hợp sản phẩm trong đó có mặt
tetrapeptit E3. Thành phần của E3 gồm Tyr và Gly3.
a) Xác định cấu trúc bậc I của E.
b) Biểu diễn cấu trúc của E ở dạng công thức Fisơ, biết rằng các aminoaxit tạo thành E
đều là đồng phân L-aminoaxit.
Cho biết công thức cấu tạo của một số aminoaxit như sau:

-------- HẾT --------


Thí sinh không được sử dụng tài liệu. Cán bộ coi thi không giải thích gì thêm.
SỞ GD&ĐT HƯNG YÊN KỲ THI CHỌN HỌC SINH GIỎI
TRƯỜNG THPT CHUYÊN HƯNG YÊN KHU VỰC DUYÊN HẢI & ĐỒNG BẰNG BẮC BỘ
LẦN THỨ XIV
ĐỀ ĐỀ XUẤT
MÔN: HÓA HỌC - LỚP 11

HƯỚNG DẪN CHẤM


Câu 1 (2,5 điểm). Tốc độ phản ứng.
Trong tầng bình lưu, nguyên tử Cl* có thể phân hủy tầng ozon theo phương trình sau:
Cl* + O3 ⎯⎯
ka
→ ClO* + O2 (a) với ka = 1,7.1010(M-1.s-1).exp(-260/T).
1. Hãy tính tốc độ phản ứng ở khoảng cách 20 km, khi đó nồng độ các chất :
[Cl]= 5.10-17M; [O3] = 8.10-9 M và T = 220K.
2. Ở độ cao 45 km, nồng độ thực tế các chất là [Cl]= 3.10-15M; [O3] = 8.10-11 M và T =
270K, hãy tính tốc độ phản ứng tại đó.
3.Tại 298K, hằng số tốc độ phản ứng (a) đo được là ka = 6,7.109M-1.s-1; thừa số
Arrhenius,
A = 6,7.1010M-1.s-1. Hãy tính năng lượng hoạt hóa của phản ứng ở 298K.
4. Phản ứng lưỡng nguyên tử pha khí với các sản phẩm khác nhau của gốc clomonoxit:
ClO* + ClO* ⎯
⎯→ Cl2 + O2 k1 = 2,9.106
ClO* + ClO* ⎯
⎯→ ClOO* + Cl* k
2 = 4,8.106.
ClO* + ClO* ⎯
⎯→ OClO* + Cl* k3 = 2,1.106.
Hãy xác định hiệu suất cho sản phẩm không chứa gốc tự do.
5. Biết phản ứng xảy ra ở phần (4), chất đầu là (ClO)2 phân hủy nhanh thành ClO*rồi sau
đó phân hủy theo 3 hướng sản phẩm trên. Biết thừa số Arrhenius và năng lượng hoạt hóa
của các phản ứng tại 298K là:
Phản ứng A (M-1.s-1) Ea (KJ/mol)
1 6,08.108 13,2
2 1,79.1010 20,4.
3 2,11.108 11,4.
Cho biết phản ứng nào có cho ∆H* lớn nhất và phản ứng nào cho ∆S* nhỏ nhất, tính các
k .T G *
giá trị đó. Cho biểu thức của Eyring: k = h exp(- RT ) và ∆U* = Ea – RT;
B

kB = 1,38.1023(J/K), h = 6,62.10-34(J.s).
Câu 1 Nội dung Điểm
1 Va = ka.[Cl*][O3] = 2,08.10-15 (M.s-1) 0,25
2 Vb = kb.[Cl*][O3] = 1,56.10-15 (M.s-1) 0,25
3 K = A.exp(-Ea/RT) => Ea = 5,70585 kJ/mol. 0,25
4 d [Cl2 ] d [O2 ]
= = k1[ClO*]2 ;
dt dt
d [ClOO*]
= k2 [ClO*]2 ;
dt
d [OClO*] 0,5
= k3 [ClO*]2 ;
dt
d [Cl*]
= (k2 + k 3 )[ClO*]2
dt
[Cl2 ] [O2 ] k1
 = = = 29,6%
 sp  sp 2(k1 + k2 + k3 ) 0,25

5 − Ea
Tính k = A. exp( ) ta được : k1 = 106,47 ; k2 = 106,677 ; k3 =
RT

106,32.
Theo phương trình Eyring :
k B .T −G *
k= exp( ) và từ ∆U* = Ea – RT ; ∆H* = ∆U* - ∆nRT
h RT

với 0,5

∆n = 1 ta suy ra : ∆H* = Ea -2RT.


Mà ∆G* = ∆H*-T∆S* = Ea-2RT- T∆S*
k B .T −E S * k B .T 2 S * −E
 k= .exp( a ).e 2 .exp = .e .exp .[exp( a )]
h RT R h R RT
k B .T 2 S *
 A= .e .exp
h R

a) ∆H* lớn nhất khi Ea lớn nhất đó là ở phản ứng (2):


0,5
∆H*= Ea – 2RT = 15,44 KJ. mol-1.
b) ∆S* lớn nhất khi A lớn nhất; mà A ở (3) lớn nhất, khi đó:
k B .T 2 S *
 A= .e .exp thay số vào ta tính được ∆S* = -102,2(J/mol)
h R

Câu 2 (2,5 điểm). Cân bằng và phản ứng trong dung dịch. Pin điện
1. Nước ô nhiễm sắt thường có mùi “tanh” và không sử dụng được do ảnh hưởng tới sức
khỏe. Một mẫu nước giếng khoan (nước ngầm) ô nhiễm sắt ở dạng Fe2+ xác định được
nồng độ là 25 ppm.
a. Tính nồng độ Fe2+ theo đơn vị mol.L-1 . Biết rằng MFe = 55,85 g.mol-1 và 1 ppm = 1
mg.L-1.
b. Tính pH của mẫu nước ô nhiễm sắt. Coi các chất khác không ảnh hưởng tới pH của hệ.
c. Khi được hút lên và để tiếp xúc với không khí đủ lâu thì sắt(II) trong nước sẽ bị oxi
hóa hoàn toàn thành sắt(III). Khi đó một phần sắt(III) sẽ chuyển thành kết tủa Fe(OH) 3.
Hỏi có sử dụng mẫu nước sau khi cho tiếp xúc với không khí làm nước sinh hoạt được
hay không? Biết hàm lượng cho phép của ion sắt trong nước sinh hoạt là 0,3 mg.L -1 và
pH của nước không thay đổi.
Cho các giá trị nhiệt động ở 25oC:
Fe(OH)+ có -lgβ = 5,92; Fe(OH)2+ có -lgβ = 2,17; Fe(OH)3 có pKS = 37.
2. Để xác định độ tan của các muối khó tan người ta thường dùng kĩ thuật điện hóa
học. Một pin điện hóa gồm 2 điện cực được nối với nhau bằng cầu muối. Điện cực thứ
nhất gồm một thanh Zn nhúng trong dung dịch Zn(NO3)2 0,200 M, điện cực thứ hai gồm
một thanh Ag nhúng trong dung dịch AgNO3 0,100 M. Mỗi dung dịch có thể tích 1,00 L tại
250C.
a. Viết sơ đồ pin và phản ứng xảy ra trên từng điện cực, phản ứng xảy ra trong pin khi
pin hoạt động.
b. Tính sức điện động của pin.
c. Tính điện lượng phóng thích trong quá trình pin phóng điện (giả sử pin phóng điện
hoàn toàn và lượng Zn dư).
Câu 2 Nội dung Điểm
1 1.a. 0,25
25
C = 25mg.L-1 = = 4,48.10−4 mol.L−1 .
1000x55,85

1.b.
Trong dung dịch có các cân bằng:
(1) Fe2+ + H2O Fe(OH)+ + H+ β = 10-5,92
(2) H2O OH- + H+ Kw = 10-14
Vì C.β >> Kw nên bỏ qua sự điện li của nước
Ta có: Fe2+ + H2O Fe(OH)+ + H+
β = 10-5,92
C 4,48.10-4
[] 4,48.10-4 - h h h

[H + ][Fe(OH)+ ] h2
= = −4
= 10−5,92 = h = 2,26.10−5
2+
[Fe ] 4,48.10 − h

 h = 2,26.10-5 M hay pH = 4,65 0,5


1.c.
Ở pH = 4,65. Gọi tổng nồng độ Fe(III) tan trong dung dịch là c
 [Fe3+] + [Fe(OH)2+] = c
KS KS
Ta có: [ Fe3+ ] = − 3
= 3
[ H + ]3
[ OH ] K w

[ Fe3+ ]
2+ K
[ Fe(OH) ] = +
= S3 [ H + ] 2
[H ] Kw

KS KS
c= 3
[ H + ]3 +  [ H + ] 2 = 3,4.10-7M = 0,019 mg/L < 0,30
K w
K w3

mg/L
0,5
 mẫu nước trên sử dụng được trong sinh hoạt.
2 0,0592
2.a. EZn 2+
/Zn
= E0Zn 2+
/Zn
+ .lg0,2 = -0,7807 V
2
0,25
0
EAg+ /Ag = E Ag+ /Ag
+ 0,0592.lg0,1 = 0,7408 V

- Nhận thấy: EAg +


/Ag
> EZn 2+
/Zn
 sơ đồ pin: (-) Zn(r) | Zn2+(aq) ||
0,25
Ag+(aq) | Ag(r) (+)
- Phản ứng tại các điện cực:
Điện cực (-): Zn(r) → Zn2+(aq) + 2e
0,25
Điện cực (+): Ag+(aq) + 1e → Ag(r)
- Phản ứng xảy ra trong pin: 2Ag+(aq) + Zn(r) → Zn2+(aq) + 2Ag(r)
2.b. Sức điện cực của pin: 0,25
Epin = EAg +
/Ag
- EZn2+ /Zn = 0,7408 - (-0,7807) = 1,5215 V

2.c. Khi pin đã phóng điện hoàn toàn thì Epin = 0 và phản ứng trong
0,0592
pin đạt đến trạng thái cân bằng  0 = 1,56 - lgK  K =
2

5,043.1052.
Vì K rất lớn nên phản ứng trong pin dịch chuyển hẳn theo chiều
thuận (về phía bên phải). Mặt khác Zn dư  có thể xem không còn 0,25

ion Ag+ trong dung dịch.


Lượng Ag+ và lượng electron đã vận chuyển:
nAg+ = 1.0,1 = 0,1 mol = ne

 Q = ne.F = 96485.0,1 = 9648,5 C


Câu 3 (2,5 điểm). Nhiệt động học và cân bằng hóa học.
1. Chu trình Brighton với 1 mol helium được biễu diễn ở hình sau:

Biết: Quá trình 1 ⟶ 2 và 3 ⟶ 4: đoạn nhiệt thuận nghịch.


Quá trình 2 ⟶ 3 và 4 ⟶ 1: đẳng áp thuận nghịch.
a) Hãy chọn ra chiều của chu trình (thuận hay ngược chiều kim đồng hồ) diễn ra với
động cơ nhiệt (sinh công).
b) Vẽ (định tính) chu trình trong hệ tọa độ T – S (S là entropy).
x
 
c) Hiệu suất của chu trình Brighton là η = 1 -  p1  . Xác định x biết p1 và p2 là áp
 p2 

suất khí ở điểm 1 và 2 tương ứng. Tính hiệu suất của chu trình trên.
d) Một chu trình tương tự được bắt đầu theo chiều ngược lại để nó hoạt động như
một máy lạnh. Hãy xác định hiệu suất máy lạnh trong trường hợp này.
2. a) Khi tăng áp suất 107 Pa thì nhiệt độ nóng chảy của nước đá (băng) giảm đi
0,744oC. Ở áp suất khí quyển, nước đá nóng chảy ở 0oC. Khối lượng riêng của nước đá là
0,9170 g.cm-3, nước lỏng là 0,9998 g.cm-3. Tính enthalpy nóng chảy (∆fusH) của nước đá.
Giả sử khối lượng riêng của nước lỏng và nước đá không thay đổi trong khoảng nhiệt độ
khảo sát.
Cho nguyên tử khối: H = 1,0079; O = 15,9994.
b) Ở nhiệt độ nào thì áp suất bão hòa của nước và acetic acid bằng nhau. Biết: ở áp
suất khí quyển nhiệt độ sôi của nước và acetic acid tương ứng là 100 oC và 118oC;
Enthalpy hóa hơi của nước và acetic acid trong khoảng nhiệt độ từ 50-100oC tương ứng là
42,0 kJ.mol-1 và 25,0 kJ.mol-1.

Câu 3 Nội dung Điểm


1. a) 1 ⟶ 2 ⟶ 3 ⟶ 4, theo chiều kim đồng hồ. 0,25
b)

0,25

c) Với 2 quá trình đẳng áp, ta có: p1 = p4; p2 = p3.


Hai quá trình đoạn nhiệt: p11− .T1 = p12− .T2 ; p13− .T3 = p14− .T4
Ta có: Qc = Cp(T4 – T1); Qh = Cp(T3 – T2).
Hiệu suất của chu trình:
1− 1−
 p3   p  
  .T3 −  2  .T2 −1
Qc T4 − T1 p  p1  p  
=1−  4 
0,5
 = 1− =1− = 1− 1 
Qh T3 − T2 T3 − T2  p2 

 −1 Cp 5
x= ; = =
 Cv 3
 5
V   24,36  3
p2 = p1  1  = 1.   = 4,41 (bar)
Mặt khác, ta có:  V2   10 
2
 1 5
 = 1−  = 0,448 hay 44,8%
Vậy, hiệu suất của máy nhiệt:  4,41 

d) Hiệu suất của máy lạnh:


Qc Qc T4 − T1
= = =
W Q h − Q c (T3 − T2 ) − (T4 − T1 )
1− 1−
 p3   p  
  .T3 −  2  .T2
=  p4   p1  0,5
1− 1−
p   p  
T3 − T2 −  3  .T3 +  2  .T2
 p4   p1 
1− 1−
 p2   p  
  .T3 −  2  .T2
=  p1   p1 
1− 1−
p   p  
T3 − T2 −  2  .T3 +  2  .T2
 p1   p1 
−1
 p2  
 
=  p1  = 1,234
−1
p  
1− 2 
 p1 

2. a) Áp dụng phương trình Clapeyron ta có:


1 1 0,5
M  −  .(P2 − P1 )
dP  fusH T V.(P2 − P1 )  s 
=  ln 2 = =  l
dT Tfus .V T1  fusH  fusH

 1 1  −6 7
18,016  − .10 .10
272,406  0,9998 0,9170 
 ln =   fusH = 5965,44 (Jmol −1 )
273,15  fusH

b) Áp dụng phương trình Clapeyron - Clausius ta có:


dlnP  vapH P  H 1 1 
= 2
 ln 2 = − vap  − 
dT RTfus P1 R  T2 T1 

Vì P đối với hai chất ở hai nhiệt độ là như nhau nên ta có: 0,5
P2 42000  1 1  25000  1 1 
ln =−  − =−  −   T2 = 349,5K
P1 8,314  T2 373,15  8,314  T2 391,15 

Câu 4 (2,5 điểm). Hóa nguyên tố (Kim loại, phi kim nhóm IVA, VA). Phức chất.
1. Chỉ thị axit–bazơ thường là các axit hoặc bazơ yếu. Để xác định hằng số phân li
axit của chỉ thị, ta dùng phương pháp đo quang. Chỉ thị HIn là một đơn axit yếu có pK a ≈
8, chỉ thị này có dạng HIn hấp thụ quang cực đại ở bước sóng 520 nm còn dạng ánh sáng
ở bước sóng này yếu hơn. Để xác định chính xác hằng số pKa của chất chỉ thị HIn, người
ta chuẩn bị 3 dung dịch chất chỉ thị có cùng nồng độ mol/l nhưng được điều chỉnh ở các
pH khác nhau. Tiến hành đo mật độ quang của 3 dung dịch đó tại bước sóng 520 nm, kết
quả được đưa ra ở bảng sau:
pH 1,0 7,4 13,0
Mật độ quang
0,90 0,64 0,10
A
Xác định hằng số phân li axit Ka của chất chỉ thị này.
2. Cho sơ đồ chuyển hóa sau:

Viết các phương trình phản ứng tương ứng với các chuyển hóa đó. Biết rằng mỗi mũi tên
ứng với một phản ứng hóa học.
Câu 4 Nội dung Điểm
1 Mật độ quang tỉ lệ với [HIn] trong dung dịch 1,0
A = a [HIn] + b = a.h/(h+Ka) + b
0,9 = a . 10-1 / (10-1 + Ka) + b
0,64 = a . 10-7,4 / (10-7,4 + Ka) + b
0,1 = a . 10-13 / (10-13 + Ka) + b
Ka = 1,917.10-8
K2MnO4 + 8HCl (đặc) ⎯⎯ → 2KCl + MnCl2 + 2Cl2 + 4H2O
o
2 (1) t
1,5
Mn(OH)2 + 2KNO3 + 2KOH ⎯⎯ → K2MnO4 + 2KNO2 + 2H2O
o
t
(2)
(3) → 2KNO3 + Mn(OH)2↓
Mn(NO3)2 + 2KOH (loãng) ⎯⎯
(4) Mn(NO3)2 + 3Cu + 8HCl ⎯⎯
→ MnCl2 + 3CuCl2 + 2NO +

4H2O
(10) 2KMnO4 ⎯⎯ → K2MnO4 + MnO2 + O2
o
t

(6) 2KMnO4 + 10FeSO4 + 8H2SO4 ⎯⎯


→ K2SO4 + 5Fe2(SO4)3 +

2MnSO4 + 8H2O
(7) Mn + H2SO4 (loãng) ⎯⎯
→ MnSO4 + H2

MnO2 + 2C ⎯⎯ → Mn + 2CO
o
t
(8)
MnO2 + 4HCl (đặc) ⎯⎯ → MnCl2 + Cl2 + 2H2O
o
t
(9)
(5) 2KMnO4 + 3C2H4 + 4H2O ⎯⎯
→ 3C2H4(OH)2 + 2MnO2 +

2KOH
→ K2SO4 + Mn(OH)2↓
(11) MnSO4 + 2KOH (loãng) ⎯⎯
(12) Mn(NO3)2 + C ⎯⎯ → 4CO + 2NO + Mn
o
t

Câu 5 (2,5 điểm). Đại cương hữu cơ.


Cho các công thức cấu tạo sau:
Me 1 2 4 5 Me Me OH Me O COOH
C(CH=CHF)2 3 N
Et O O Me
CH CH2F O 2N O O 2N OH NH2
O N
(A) (B) (C) (D) H (E)

1. Hãy vẽ công thức các đồng phân lập thể ứng với cấu tạo A.
2. Ứng với công thức cấu tạo B có bao nhiêu đồng phân lập thể, vì sao? Dùng các kí hiệu
thích hợp để chỉ rõ cấu hình của mỗi đồng phân đó.
3. Hãy viết cơ chế phản ứng để giải thích vì sao C và D khi tương tác với dung dịch
NaOH thì đều tạo thành natri 3-metyl-4-nitrobenzoat.
4. Hãy chỉ rõ trạng thái lai hóa của từng nguyên tử N ở cấu tạo E và ghi giá trị pKa (ở 25
o
C): 1,8; 6,0; 9,2 vào từng trung tâm axit trong công thức tương ứng với E, giải thích.
Câu 5 Nội dung Điểm
1 Hãy vẽ công thức các đồng phân lập thể ứng với cấu tạo A. 0,5
F F
F F F F
F
F F F

F (A1) (A2) (A3) F (A4)


2 Ứng với công thức cấu tạo B có bao nhiêu đồng phân lập thể, vì 0.75
sao? Dùng các kí hiệu thích hợp để chỉ rõ cấu hình của mỗi đồng
phân đó.
B có 3C bất đối, không có mặt phẳng và tâm đối xứng nên có 8 đồng
phân lập thể.
ví dụ: Cấu hình của B1 như chỉ ra trong bảng, viết gọn là (1R)-(2R)-
(4R).
Me Et B1 B2 B3 B4 B5 B6 B7 B8
1
O
2
3 O C1 R S S R S R R S
4
O
5 C2 R S R S S R S R
Me Me
C4 R S R S R S R S
3 Hãy viết cơ chế phản ứng để giải thích vì sao C và D khi tương tác 0.75
với dung dịch NaOH thì đều tạo thành natri 3-metyl-4-nitrobenzoat.
C , C1 , D và D1 là những đồng phân hỗ biến, xúc tác kiềm làm
thuận lợi cho sự hỗ biến đó:
HO O O O O O O O
OH-/- H2O H2O/- OH-
( C1 )

Me Me Me Me
NO2 NO2 NO2 NO2
( C)
( D1 ) ( D)
O O O O O O O OH
H2O/- OH-

Me Me Me Me
NO2 NO2 NO2 NO2

Xuất phát từ C , C1 , D hoặc D1 qua phản ứng chuyển vị benzylic


rồi tự mất nước đều chuyển thành hợp chất thơm bền vững, đều dẫn
đến cùng một sản phẩm, ví dụ:
O OH O
O O O O COOH HO COO COO-
OH
OH- H - H2O

Me Me Me Me
Me Me
NO2 NO2 NO2 NO 2 NO 2 NO 2
(D)
4 - Nguyên tử N nhóm NH ở trạng - Nhóm NH3+ là axit liên 0.5
thái lai hóa sp2, cặp e chưa chia ở hợp của nhóm H2Nsp3 ,
obitan p xen phủ với 5 obitan p nhóm NH+ là axit liên hợp
khác tạo thành hệ thơm được lợi của nhóm Nsp2.
về mặt năng lượng nhưng “mất” - Bazơ càng mạnh thì axit
tính bazơ. liên hợp càng yếu, vì thế giá
- Nguyên tử N thứ hai ở trạng thái trị 9,2 là thuộc nhóm NH3+
lai hóa sp2, cặp e chưa chia ở còn giá trị 6,0 thì thuộc
obitan sp2 không tham gia vào hệ nhóm NH+.
thơm nên còn tính bazơ.
- Nguyên tử N nhóm NH2 ở trạng
thái lai hóa sp3.

Câu 6 (2,5 điểm). Sơ đồ phản ứng và cơ chế phản ứng.


1. Viết cơ chế giải thích sự hình thành sản phẩm trong các phản ứng sau:
a.

b.

2. Viết công thức cấu tạo của các chất từ C đến C5 và hoàn thành sơ đồ chuyển hóa sau:
3. Veticadinol là một sesquiterpene tự nhiên nhóm candinane được phân lập từ tinh dầu
thực vật. Veticadinol được tổng hợp toàn phần bởi L.F. Tietze theo sơ đồ dưới đây:

1a 0,25

1b 0.25
2 1,0

3 Sơ đồ tổng hợp Veticadinol: 1,0

Câu 7 (2,5 điểm). Xác định cấu trúc.


Hidrocacbon A (C6H10) không có đồng phân lập thể, 1 mol A chỉ làm mất màu 1 mol
KMnO4 (dung dịch nước) hoặc 1 mol Br2 (trong dung dịch CCl4) ở nhiệt độ thường. A
phản ứng với lượng dư H2/xúc tác Ni tạo thành các hợp chất là đồng phân cấu tạo của
nhau có cùng CTPT C6H14. Trong dung dịch axit H3PO4 50%, A chuyển thành C (C6H12O)
không làm mất màu dung dịch KMnO4 hoặc dung dịch Br2/CCl4 ở nhiệt độ thường. Chế
hóa C với CrO3/piridin thu được D.
a) Xử lý D với m-CPBA thu được 2 sản phẩm E1 và E2 là đồng phân cấu tạo của
nhau có cùng CTPT C6H10O2, trong đó E1 là sản phảm chính. Khử hóa E1 và E2 bằng
LiAlH4 thu được F1 và F2 có cùng CTPT C6H14O2; F1 hoặc F2 phản ứng với PCC hoặc
C5H5N.SO3 thu được sản phẩm tương ứng X1 và X2, trong đó X1 có phản ứng idofom.
Xác định CTCT của các chất nêu trên.
b) Khi cho D tác dụng với hidroxiamin ở pH khoảng 5-6 thu được G. Xử lý G với
PCl5 thu được H1 và H2 là đồng phân cấu tạo của nhau có cùng CTPT C6H11NO, trong
đó H1 là sản phẩm chính. Cho H1 hoặc H2 phản ứng với LiAlH4, rồi đun nóng sảm phẩm
thu được với xúc tác Se thu được sản phẩm tương ứng là I1 và I2 có cùng công thức
C6H7N không làm mất màu dung KMnO4 hoặc dung dịch Br2/CCl4 ở nhiệt độ thường.
Hãy xác đinh CTCT của các chất nêu trên biết rằng I1 phản ứng được với 4-
nitrobenzandehit khi có mặt Ac2O làm xúc tác cho K (C13H10N2O2).

Câu 7 Nội dung Điểm


Hidrocacbon A (C6H10, ∆= 2) không có đồng phân lập thể, 1 mol A chỉ
làm mất màu 1 mol KMnO4 (dung dịch) hoặc 1 mol Br2 (trong dung
dịch CCl4) ở nhiệt độ thường. A phản ứng với lượng dư H 2/xúc tác Ni
tạo thành các hợp chất là đồng phân cấu tạo của nhau có cùng CTPT
C6H14(∆= 0) => A có 1 vòng 4 cạnh và có 1 liên kết đôi. 0,5đ
=> Các CTCT có thể có của A là: (0,5đ)

A1
a) A2 A3 A4

Trong dung dịch axit H3PO4 50%, A chuyển thành C (C6H12O) không
làm mất màu dung dịch KMnO4 hoặc dung dịch Br2/CCl4 ở nhiệt độ
0,5đ
thường. Chế hóa C với CrO3/piridin thu được D. Nên:

OH O
A: C: D:

18 chất trong sơ đồ a,b được 1,2 điểm 0,5đ


a)
O LiAlH4 OH PCC O
O
m-CPBA O OH
F1 O
E1 X1
OH O
D O
LiAlH4 PCC
O OH O
F2
b) E2
X2 0,5đ
b)

O N N
NH2OH PCl5 O
OH Cl CV NH +
NH
D O
H1 H2

0.5
H

NH
LiAlH4 NH Se N NO2 NO2
N
O Ac2O E
H1
H

O
LiAlH4 Se
NH
NH N
H2

Câu 8 (2,5 điểm). Hợp chất thiên nhiên.


1. Một cacbohiđrat A có công thức phân tử C12H20O11. A phản ứng được với thuốc
thử phenylhiđrazin, không làm mất màu dung dịch Br 2/H2O, không tham gia phản ứng
với thuốc thử Tollens. Thủy phân A bằng dung dịch axit thu được hợp chất A1 và D-
fructozơ. Metyl hóa A1 bằng MeOH (xúc tác HCl khan) thu được hợp chất A2 (C7H12O6).
Tiến hành oxi hóa A2 bằng dung dịch HIO4 rồi thủy phân sản phẩm thu được hỗn hợp,
trong đó có chứa các chất A3, A4, A5.

Đun nóng A với dung dịch NaOH thu được hỗn hợp sản phẩm, trong đó có B1 (C6H8O5).
Oxi hóa B1 bằng HIO4 và đun nóng nhẹ thu được hợp chất B2 (C5H8O4). Metyl hóa B2
bằng MeI/Ag2O thu được hợp chất B3 (C7H14O5). Mặt khác, hiđro hóa B2 bằng H2/Ni, to
thu được hợp chất B4 (C5H12O4).
a) Xác định và biểu diễn cấu trúc của A, A1, A2, B1, B2, B3, B4 ở dạng vòng Haworth (với
hợp chất mạch vòng) hoặc công thức Fisơ (với hợp chất mạch hở).
b) Đề xuất cơ chế quá trình chuyển hóa A thành B1 trong dung dịch NaOH.
2. Peptit E có khả năng ức chế một số chủng vi khuẩn. Để xác định cấu trúc bậc I
của E, có thể tiến hành thí nghiệm như sau: Peptit E không cho phản ứng với 2,4-
đinitroflobenzen và không bị thủy phân bởi enzym aminopeptiđaza. Thủy phân hoàn toàn
E thu được các amino axit như sau: Gly3, Pro2, Phe, Tyr. Khi thủy phân không hoàn toàn
E bằng enzym pepsin, thu được tetrapeptit E1 và tripeptit E2. Thành phần của E1 gồm
Phe, Gly2 và Pro, thành phần của E2 gồm Tyr, Gly và Pro. Mặt khác, thủy phân không
hoàn toàn E trong dung dịch axit loãng thu được hỗn hợp sản phẩm trong đó có mặt
tetrapeptit E3. Thành phần của E3 gồm Tyr và Gly3.
a) Xác định cấu trúc bậc I của E.
b) Biểu diễn cấu trúc của E ở dạng công thức Fisơ, biết rằng các aminoaxit tạo thành E
đều là đồng phân L-aminoaxit.
Cho biết công thức cấu tạo của một số aminoaxit như sau:

Ý Nội dung Điểm


1.a) A có công thức phân tử C12H20O11  độ không no k = 3. 1,0
A phản ứng được với thuốc thử phenylhiđrazin, không làm mất màu dung
dịch Br2/H2O, không tham gia phản ứng với thuốc thử Tollens  A chứa chức
xeton.
Thủy phân A bằng dung dịch axit thu được hợp chất A1 và D-fructozơ  A là
một disaccarit hoặc gồm một monosaccarit là Fructozo và thành phần aglycol
là A1.
Metyl hóa A1 bằng MeOH (xúc tác HCl khan) thu được hợp chất A2
(C7H12O6)  A1 có 1 nhóm -OH hemiaxetal
Tiến hành oxi hóa A2 bằng dung dịch HIO4 rồi thủy phân sản phẩm thu được
hỗn hợp, trong đó có chứa các chất A3, A4, A5.

 công thức cấu tạo của A2:

Đun nóng A với dung dịch NaOH thu được hỗn hợp sản phẩm, trong đó có B1
(C6H8O5).
 A thực hiện phản ứng retroaldol tạo thành B1 và fructozo.

Oxi hóa B1 bằng HIO4 và đun nóng nhẹ thu được hợp chất B2 (C5H8O4).

Metyl hóa B2 bằng MeI/Ag2O thu được hợp chất B3 (C7H14O5).

Mặt khác, hiđro hóa B2 bằng H2/Ni, to thu được hợp chất B4 (C5H12O4).
Công thức Haworth của các chất chưa biết:

1.b) Cơ chế chuyển hóa A → B1: 0,5

2.a) Peptit E không cho phản ứng với 2,4-đinitroflobenzen và không bị thủy phân 0,75
bởi enzym aminopeptiđaza
 không có nhóm amino đầu N.
Thủy phân hoàn toàn E thu được các amino axit như sau: Gly3, Pro2, Phe, Tyr.
 E là heptapeptit.
E không có đầu N mà các aminoaxit đều chỉ có 1 nhóm -COOH và 1 nhóm -
NH2
 E là heptapeptit mạch vòng.
E bằng enzym pepsin, thu được tetrapeptit E1 và tripeptit E2.
 Đuôi C của E1 và E2 là Tyr hoặc Phe.
E2 gồm Tyr, Gly và Pro.
 Đuôi C của E2 là Tyr.
Thành phần của E1 gồm Phe, Gly2 và Pro,
 Đuôi C của E1 là Phe.
Thành phần của E3 gồm Tyr và Gly3.
Có 4 trường hợp cho E3:
Tyr-Gly-Gly-Gly (1); Gly-Tyr-Gly-Gly (2); Gly-Gly-Tyr-Gly (3); Gly-Gly-
Gly-Tyr (4).
(1) và (4) loại vì khi thủy phân bằng enzim pepsin sẽ phải có một peptit chứa
3 Gly.
(3) loại vì khi thủy phân bằng enzim pepsin sẽ phải có một peptit chứa Gly-
Gly-Tyr (trong khí E2 chỉ có 1 Gly)
Vậy E3 có công thức: Gly-Tyr-Gly-Gly
Suy ra E2 có công thức: Pro-Gly-Tyr.
Vì E3 có cấu trúc Gly-Tyr-Gly-Gly nên E1 có cấu trúc Gly-Gly-Pro-Phe.
Công thức của peptit:

E1: Gly-Gly-Pro-Phe
E2: Pro-Gly-Tyr
E3: Gly-Tyr-Gly-Gly
2.b) 0,25
---------HẾT--------

Người ra đề
(ký và ghi rõ họ, tên)

SĐT: ……………….……………
TRƯỜNG THPT CHUYÊN ÐÁP ÁN ÐỀ THI ÐỀ XUẤT CHỌN HỌC SINH GIỎI
CAO BẰNG CÁC TRƯỜNG THPT CHUYÊN KHU VỰC
DUYÊN HẢI VÀ ÐỒNG BẰNG BẮC BỘ NĂM 2023

ÐỀ THI MÔN: HÓA HỌC LỚP 11


Thời gian: 180 phút (Không kể thời gian giao ðề)
(Ðáp án gồm 15 trang)

Câu 1. (2,5 điểm) Tốc độ phản ứng


1.1. Xét phản ứng tạo thành NO theo phương trình (1) như sau:
2NOCl (g) → 2NO (g) + Cl2 (g) (1)
Cho hằng số khí R = 8,314 J.mol-1.K-1 và các dữ kiện sau:
Nhiệt độ (K) Hằng số tốc độ (L.mol-1.s-1)
300 2,6.10-8
400 4,9.10-4
Phản ứng của NO với oxygen diễn ra theo phương trình (2) như sau
2NO (g) + O2 (g) → 2NO2 (g) (2)
Cơ chế đề xuất cho phản ứng (2) được cho như sau:
⎯⎯
NO (g) + O2 (g) ⎯
k1
→ NO3 (g) (3) Cân bằng nhanh

k−1

NO3 (g) + NO (g) ⎯⎯ k


→ 2NO2 (g) (4) Giai đoạn chậm
a) Tính năng lượng hoạt hóa của phản ứng (1).
b) Hãy viết phương trình tốc độ của phản ứng tạo thành NO2.
c) Thực nghiệm cho thấy phương trình tốc độ phản ứng (2) có dạng v=k.[NO]2.[O2].
Hãy cho biết cơ chế của phản ứng (2).
1.2. Sự hấp thụ thuốc vào một cơ thể sống thường tuân theo quy luật động học đơn
giản, mặc dù cơ chế thì rất phức tạp. Xét một loại thuốc được đưa vào bằng đường uống ở dạng
viên nang thông thường. Đặt [A]s là nồng độ thuốc trong dạ dày và giả sử rằng tốc độ đưa
thuốc vào mạch máu có bậc một theo [A]s. Giả sử tốc độ chuyển hóa hoặc loại bỏ thuốc khỏi
mạch máu tỉ lệ thuận với nồng độ trong máu, [A]b.
d [A]b
a) Hãy vẽ đồ thị [A]s theo thời gian và viết phương trình biểu diễn .
dt
b) Sau 1 giờ, 75% [A]s đã bị loại khỏi dạ dày. Hãy tính phần trăm [A]s ban đầu vẫn còn
lại trong dạ dày sau 2 giờ kể từ khi dùng thuốc.

Câu ý Nội Dung Điểm


1 1.1 a)
k2 4,9.10−4
R.ln(
) 8,314.ln( )
k2 Ea 1 1 k1 2, 6.10−8
ln( ) = ( − )  Ea = =
k1 R T1 T2 1 1 1 1
( − ) ( − )
T1 T2 300 400
 Ea = 98212 (J.mol −1 )
 Ea = 98, 212 (kJ.mol −1 )
b) Giai đoạn chậm quyết định tốc độ phản ứng. 0,5
Phương trình động học:
d [NO2 ]
v= = k3 .[NO3 ].[NO]
dt

1
k1 [NO3 ]
Từ cân bằng (3) ta có: K = =  [NO3 ] = K .[NO].[O2 ]
k−1 [NO].[O2 ] 0,5
d [NO2 ]
Ta có: v = = k3 .K .[NO]2 .[O2 ]
dt
0,5
c) k=k3.K và cơ chế đã cho phù hợp với tốc độ phản ứng xác định theo thực
nghiệm.

1.2 a) As ⎯⎯
k1
→ Ab ⎯⎯
→ c¸c s¶n phÈm (1)
d[ A]s
− = k1.[ A]s (2)  [ A]s = [ A]0.e− k1t
dt
Trong đó [A]0 là nồng độ thuốc trong dạ dày ở thời điểm ban đầu (t=0)

0,5

b) Ta có:
[ A]0 − [ A]s [ A] 1
= 0,75  s = 0,25 = ( )2
[ A]0 [ A]0 2
Do sau 1 giờ còn lại ¼ lượng ban đầu nên sau 2 giờ sẽ còn lại
1 1 0,5
( )2 = = 0,0625 hay 6,25% [ A]s.
4 16

Câu 2. (2,5 điểm) Cân bằng và phản ứng trong dung dịch, pin điện, điện phân.
2.1. a) Tính hằng số cân bằng của phản ứng:
Cr2O72- + H2O ƒ 2CrO42- + 2H+
b) Trộn 10,00 mL dung dịch K2Cr2O7 0,80M với 10,00 mL dung dịch A gồm BaCl2 0,08M
và SrCl2 0,08M thu được hỗn hợp B. Hãy cho biết hiện tượng xảy ra và thành phần hỗn hợp B.
Cho: CrO42- + H2O ƒ HCrO4- + OH- Kb = 10-7,5
Cr2O72- + H2O ƒ 2HCrO4- K = 10-1,64
pKw(H2O) = 14,0; pKs(BaCrO4) = 9,93; pKs(SrCrO4) = 4,65.
2.2. Cho hai pin điện hóa có sơ đồ:
Pin 1: Pt,H2 (1 atm)│HCl 10-3 M│Hg2Cl2,Hg
Pin 2: Pt,H2 (1 atm)│NaOH 10-3 M, NaCl 10-3 M│Hg2Cl2,Hg
Sức điện động của các pin tương ứng là E1 và E2. Biết EHg
o
2 Cl 2 /Hg
= 0,2682 V.
a) Viết phương trình hóa học của phản ứng xảy ra tại các điện cực và phản ứng tổng quát
khi các pin làm việc.
b) Tính E1 và thiết lập mối liên hệ giữa E2 và K H O ở 25oC.
2

c) Nối hai điện cực calomen của hai pin với nhau để tạo thành một pin kép. Ở 25oC, sức
điện động của pin này là 0,4726 V. Xác định K H O ở nhiệt độ này.
2

2
2,303RT
Cho ở 25C: = 0,0592
F

Câu ý Nội Dung Điểm


2 2.1 a)
Cr2O72- + H2O ƒ 2 HCrO-4 K1 = 10-1,64
2 HCrO-4 + OH- ƒ CrO2- 4 + H2O Kb-1 = 107,5
2 H2O ƒ H+ + OH- Kw = 10-14
0,25
Cr2O72- + H2O ƒ 2 CrO2-
4 + 2H
+
K2 = 10-14,64
b)
Cr2O72- + H2O ƒ 2 HCrO-4 K1 = 10-1,64 (1)
Cr2O72- + H2O ƒ 2 CrO2-
4 + 2H
+
K2 = 10 (2) -14,64

Sau khi trộn: C(Cr2O72-) 2+ 2+


= 0,4M; C(Sr ) = 0,04M; C(Ba ) = 0,04M.
Trong dung dịch, crom tồn tại dưới 3 dạng là Cr2O72-, HCrO4- và CrO42-.
Từ (1) và (2) ta thấy: K1 K2 → tính C'HCrO- theo (1):
4

Cr2O72- + H2O ƒ 2 HCrO-4 K1 = 10 -1,64

0,4 – x 2x → x = 0,045 → C'HCrO- = 0,09 M.


4
Kw
Thay vào cân bằng: HCrO-4 ƒ CrO2-
4 + H
+
Ka = = 10−6,5 (3)
Kb
0,09 – y y y
→ CCrO2- = y = 1,685.10 M  1,7.10-4 M.
' -4
0,25
4

→ CCrO
' ' -4
2- . CBa 2+ = 1,7.10 .0,04 = 6,8.10
-6
KS(BaCrO ) = 10-9,93
4 4

→ có kết tủa BaCrO4 vàng theo phản ứng (K = 10 ): 5,22

Cr2O72- + 2 Ba2+ + H2O ⎯⎯ → 2 BaCrO  + 2H+


4
(4)
0,4 0,04
0,38 - 0,04
"
Vì sau phản ứng (4), CCrO2-4 < 1,7.10-4 M, do đó:
−4 −6
C"CrO2- .CSr
'
2+  1.7.10 .0, 04 = 6,8.10  K S(SrCrO ) = 10−4,65 → không có kết 0,25
4 4

tủa SrCrO4 tách ra.


Vậy hỗn hợp B thu được gồm kết tủa BaCrO4; Cr2O72-dư; Sr2+; H+ tạo 0,25
thành (K+; Cl-)

2.2 a) Các phản ứng:


Pin 1: Tại anot (-): H2 → 2H+ + 2e
Tại catot (+): Hg2Cl2 + 2e → 2Hg + 2Cl-
Tổng quát: H2 + Hg2Cl2 → 2H+ + 2Cl- + 2Hg
Pin 2: Tại anot (-): H2 + 2OH- → 2H2O + 2e
Tại catot (+): Hg2Cl2 + 2e → 2Hg + 2Cl-
Tổng quát: Hg2Cl2 + 2OH- + H2 → 2Hg + 2Cl- + 2H2O 0,5

3
b) Sức điện động của pin 1 ở 25oC:
0, 0592 [H + ]2 [Cl- ]2 [H + ][Cl- ]
E1 = E1o − lg = E oHg 2Cl2 /Hg − E o2H+ /H − 0, 0592 lg
2 PH2 2
PH2
10−3.10−3
 E1 = 0, 2682 − 0, 0592 lg = 0, 6234V
1
Sức điện động của pin 2 ở 25oC:
0, 0592 [Cl- ]2 0, 0592 [Cl- ]2
E 2 = E o2 − lg = E o
− E o
− lg
[OH − ]2 PH2 [OH − ]2 PH2
Hg Cl /Hg H O/H
2 2 2 2 2
2
0, 0592 (10−3 ) 2
 E 2 = 0, 2682 − 0, 0592 lg K H2O − lg = 0, 2682 − 0, 0592 lg K H2O (V)
2 (10−3 ) 2 .1
0,5
c) Khi nối hai điện cực calomen của hai pin ta sẽ thu được pin kép là pin nồng
độ, khi đó điện cực hidro của pin 1 sẽ trở thành điện cực dương, điện cực
hidro của pin 2 là điện cực âm (vì nồng độ H+ trong pin 1 lớn hơn pin 2).
Sức điện động của pin kép:
E = (E hidro ) pin 1 − (E hidro ) pin 2 = −(E Hg 2Cl2 /Hg − E 2H+ /H ) + (E Hg 2Cl2 /Hg − E H 2O/H 2 ) = E 2 − E1
2

 E = 0, 2682 − 0, 0592 lg K H2O − 0, 6234 = −0,3552 − 0, 0592 lg K H2O

Vì E = 0, 4726  −0,3552 − 0, 0592 lg K H O = 0, 4726 2


0,4726 + 0,3552

 K H2O = 10 0,0592
 1, 04.10−14
0,5

Câu 3. (2,5 điểm) Nhiệt động học và cân bằng hóa học
3.1. Xét phản ứng: TiO2 (s) + 2C (graphite,s) + 2Cl2 (g) → 2CO (g) + TiCl4 (l).
Có ∆rHo (298K) = -80,01 kJ.mol-1. Cho biết các dữ kiện sau ở 25oC:
Chất TiO2 (s) Cl2 (g) C (graphite,s) CO (g) TiCl4 (l)
∆fH (kJ.mol )
o -1
-945 -110,5
-1 -1
CP,m (J.K .mol ) 55,06 33,91 8,53 29,12 145,2
a) Tính ∆rH ở 135,8 C.
o o

b) Tính ∆fHo của TiCl4 (l) ở 25oC.


Giả sử các giá trị nhiệt dung riêng không phụ thuộc vào nhiệt độ.
3.2. Hằng số cân bằng Kp của phản ứng 2I(k) ⇌ I2(k) phụ thuộc nhiệt độ từ 298 - 1000 K
7823 1
được cho ở biểu thức sau: lg K p = −3,815 + − lg T (đơn vị áp suất là bar).
T 2
a) Hãy đưa ra biểu thức cho thấy sự phụ thuộc ∆H vào nhiệt độ và từ đó tính giá trị này ở
300, 700 và 1000 K. Rút ra kết luận về sự phụ thuộc của enthalpy vào nhiệt độ.
b) Xác định chiều chuyển dịch cân bằng khi tăng nhiệt độ và tăng áp suất.
c) Tính nồng độ của iodine nguyên tử (nguyên tử/L) trong một bình kín thể tích 2L ở
300K nếu trong bình có sẵn iodine rắn (Áp suất cân bằng của iodine hơi so với iodine rắn trong
điều kiện đang xét là 4,04.10-4 bar).
d) Tính độ phân ly của iodine phân tử ở 1000K nếu áp suất chung của hệ là 2 bar và 100
bar.

4
Câu ý Nội Dung Điểm
3 3.1 a) Đổi nhiệt độ 135,8oC thành 408,8K
408,8K
 r H (408,8K ) =  r H (298K ) +  CP,mdT
0 0

298K

 r H 0 (408,8K ) =  r H 0 (298K ) + [ CP,m ( TiCl 4 , l ) + 2CP,m (CO, g) − CP,m ( Ti O2 , s)


−2CP,m (graphite, s) − 2CP,m (Cl2 , g)].(408,8 − 298) 0,5
= -80,01.103 + (145,2+2.29,12–55,06–2.8,53–2.33,91).(408,8-298)
=-72974,2 (J.mol-1) hay -72,974 (kJ.mol-1)

b) TiO2 (s) + 2C (graphite,s) + 2Cl2 (g) → 2CO (g) + TiCl4 (l).


 r H 0 = 2 r H 0 (CO, g) +  r H 0 ( TiCl4 , l ) −  r H 0 ( Ti O2 , s) − 2 r H 0 (graphite, s)
−2 r H 0 (Cl2 , g)]
  r H 0 ( TiCl4 , l ) =  r H 0 +  r H 0 (Ti O2 , s) + 2 r H 0 (graphite, s) 0,5

+2 r H 0 (Cl2 , g)] − 2 r H 0 (CO, g)


  r H 0 ( TiCl4 , l ) = −80,01 − 945 + 2.110,5 = -804,01 (kJ.mol −1)
3.2 d ln K p H 0  d ln K p  2
a) Ta có: = 2
 H 0 =   RT
dT RT  dT 
Do lnKp=2,303lgKp nên ta có:
d ln K p −2,303.7823 0,5
= −
dT T2 T
d ln K p  −2,303.7823 0,5 
H o = .RT2 =  −  .RT2
 T 
2
dT T
0,5
 −2,303.7823 0,5 
H o =  −  .8,314T
2

 
2
T T
 H 0 = −149788,09 − 4,157T (kJ)
Nhiệt độ (K) 300 700 1000
H (kJ)
0
-151,04 -152,7 -153,95
Khi tăng nhiệt độ thì nhiệt của phản ứng thu nhiệt tăng nhẹ.
0,5
d ln K p
b) Với phản ứng tỏa nhiệt thì biểu thức mang giá trị âm, tức là khi
dT
tăng nhiệt độ giá trị hằng số cân bằng sẽ giảm, tức là hiệu suất của phản ứng
cũng giảm. Theo nguyên lí chuyển dịch cân bằng Le Chartelier, khi tăng áp
suất cân bằng sẽ chuyển dịch theo chiều thuận.
c) Ta có:
7823
lgKp = -3,815+ - 0,5.lg300 = 21,02
300
PI
K p,300 = 22 = 1,05.1021
PI
PI2
PI = = 6,2.10−13
K p,
Biến đổ phương trình trạng thái khí lí tưởng về dạng:

5
N N A.P
= = 1,5.1013 (nguyên tử/m3) hay 1,5.1010 (nguyên tử/L)
V RT
Gọi  là độ phân ly của hơi iodine
⎯⎯
I2 (k) ⎯ → 2I(k)

1
(1-α) 2α
4 2
Ta có biểu thức: K = *
.P
(1−  )
p

1
Với K *p = , ở 1000K giá trị này là 0,0031.
Kp
 2 bar = 1,95%; 100 bar = 0,278%;
Khi tăng áp suất, cân bằng chuyển dịch theo chiều nghịch. 0,5

Câu 4. (2,5 điểm) Hóa nguyên tố (kim loại, phi kim nhóm IVA, VA). Phức chất
4.1. Bột tinh thể trắng của chất A dễ tan trong nước. Chất A được tạo thành bởi phản ứng
của sodium hydroxide với khí B không màu ở áp suất cao (phản ứng 1). Khi đun nóng chất A
với sodium hydroxide thì thu được chất C và khí không màu G, không tan trong nước (phản
ứng 2). Khi điện phân dung dịch chất A thu được các khí G và D (phản ứng 3). Khí D cũng có
thể được tạo thành bởi phản ứng của chất C với sulfuric acid (phản ứng 4). Xác định các chất
A, B, C, D, G và viết phương trình các phản ứng đã đề cập. Với phản ứng 3, hãy viết phương
trình các phản ứng cathode và anode.
4.2. Quá trình tổng hợp phức {Pt(CH3NH2)(NH3)[CH2COO]2} là thuốc chống ung thư
mới có hiệu quả cao lại ít độc và ít cho phản ứng phụ. Quá trình tổng hợp chất này như sau:
K2PtCl4 ⎯⎯ I
→ A (dung dịch nâu) ⎯⎯ II
→ B (tinh thể sáng) ⎯⎯
III
→ C (rắn đỏ nâu) ⎯⎯
IV
→ D (tinh
thể vàng kim) ⎯⎯ V
→ E (tinh thể vàng nhạt).
Phản ứng I thêm KI dư ở 700C; Phản ứng II cho A tác dụng với CH3NH2 theo tỉ lệ mol
A: CH3NH2 = 1:2; Phản ứng III cho thêm HClO4 và C2H5OH, phân tích phổ IR cho thấy trong
hợp chất C có hai loại liên kết Pt-I khác nhau và C có tâm đối xứng, biết MC = 1,88MB; Phản
ứng IV cho C tác dụng với lượng thích hợp dung dịch NH3 trong nước thu được chất D phân
cực; Phản ứng V cho D tác dụng với Ag2CO3 dư và malonic acid, thu được phức chất E. Cho
biết số phối trí của platium luôn không đổi trong quá trình tổng hợp và platium luôn giữ dạng
lai hóa dsp2 trong các phức.
a) Viết công thức cấu tạo các sản phẩm A, B, C, D, E.
b) Trong sản phẩm E thì không có chứa iodine. Như vậy tại sao lúc đầu phải chuyển
K2PtCl4 thành A.
c) Mục đích của việc sử dụng Ag2CO3 trong phản ứng cuối là gì?
Câu ý Nội Dung Điểm
4 4.1 Chất A được tạo thành bởi phản ứng của sodium hydroxide với khí B nên A
là muối sodium. Do đó, trong quá trình điện phân dung dịch A sẽ giải phóng
H2 (G) là chất khí không màu không tan trong nước. Khi đun nóng A với
sodium hydroxide cũng có H2 được giải phóng nên đây là phản ứng Dumas
với sodium formate.
A: HCOONa, B: CO, C: Na2CO3, D: CO2, G: H2
Các phương trình hóa học
(1) CO + NaOH → HCOONa

6
(2) HCOONa + NaOH → Na2CO3 + H2
(3) HCOONa + H2O ⎯⎯⎯⎯⎯⎯
§ iÖn ph©n dung dÞch
→ NaOH + H2 + CO2
Cathode: 2H2O + 2e →H2 + 2OH-
Anode: 2HCOO- →H2 + 2CO2 + 2e
(4) Na2CO3 + H2SO4 →Na2SO4 + H2O + CO2
1,0

4.2 a) Công thức cấu tạo các chất:

1,0

b) Để chắc chắn thu được sản phẩm cuối thì buộc B phải dạng cis nên
0,25
K2PtCl4 phải được chuyển thành A.
c) Ag2CO3 phản ứng vớ D theo phản ứng:
D + Ag2CO3 →DCO3 + 2AgI
0,25
Sau đó DCO3 phản ứng với malonic acid tạo thành E.

Câu 5. (2,5 điểm) Đại cương hữu cơ


5.1. Sắp xếp các chất sau theo chiều tăng tính axit và giải thích.

5.2. Cho 3 dị vòng ký hiệu là A, B, C. Hãy sắp xếp các dị vòng theo thứ tự tăng dần nhiệt
độ sôi; tăng dần tính bazơ của các nhóm –NH. Giải thích.

5.3. Hãy cho biết hợp chất dưới đây có bao nhiêu đồng phân lập thể và biểu diễn cấu
trúc không gian của các đồng phân lập thể đó.

7
Câu ý Nội Dung Điểm
5 5.1 Tính axit : Q < M < P < N
(Q) có tính axit yếu nhất do OH ở vị trí p có hiệu ứng + C mạnh
Tính axit của (P) khá lớn do OH ở vị trí m- có hiệu ứng chủ yếu là –I
Tính axit của (N) lớn nhất do HO có hiệu ứng ortho (Chủ yếu ở đây là ảnh hưởng
của lk hidro nội phân tử làm tăng tính axit ở nấc 1, ở nấc 2 tính axit lại giảm) 0,5
5.2 So sánh nhiệt độ sôi: A < B < C

Vòng no, liên kết Vòng thơm, liên Vòng thơm,


hiđro giữa nhóm – kết hiđro giữa liên kết
NH của dị vòng no nhóm –NH với dị hiđro bền.
nên rất yếu. vòng thơm chứa
một nguyên tử
nitơ yếu hơn so
với dị vòng thơm 0,5
C có 2 nguyên tử
N.

So sánh tính bazơ: A > C > B

A: Tính bazơ mạnh B: Tính bazơ không C: Tính bazơ trung


nhất vì electron n Nsp3 còn vì electron n đã bình vì electron n Nsp2 0,5
tham gia liên hợp vòng
thơm.
5.3 Hợp chất đã cho có 6 đồng phân lập thể. Trong các đồng phân này, các vòng 6
cạnh tồn tại ở dạng ghế.

8
1,0

Câu 6. (2,5 điểm) Sơ đồ tổng hợp hữu cơ. Cơ chế phản ứng
6.1. Herbicide oxyfluorfen có thể được điều chế bởi phản ứng giữa phenol và một aryl
fluoride. Hãy đề xuất cơ chế của phản ứng.

6.2. Testosterone là một trong những steroid giới tính quan trọng nhất. Khi testosterone
bị tách nước bởi xử lí với acid thì xảy ra sự chuyển vị, tạo thành sản phẩm như bên dưới. Hãy
đề xuất cơ chế để giải thích cho phản ứng này.

6.3. Xác định cấu trúc các chất chưa biết trong sơ đồ chuyển hóa sau:

9
Câu ý Nội Dung Điểm
6 6.1 Bước 1 là phản ứng cộng hợp nucleophile, tiếp theo là tách loại ion fluoride.
Nhóm nitro làm cho vòng suy giảm mật độ electron và dễ bị tác dụng bởi nhó
nucleophile RO-. Nó cúng giúp làm bền hóa phức mang điện âm.

0,5

6.2

0,5

6.3

10
1,5

Câu 7. (2,5 điểm) Xác định cấu trúc các chất hữu cơ (mô tả sơ đồ tổng hợp bằng lời dẫn)
7.1. Hợp chất C1 (C10H18O) phản ứng với CH3MgBr, tạo khí metan; phản ứng với PCC, tạo
thành xeton; phản ứng với KMnO4 loãng, lạnh tạo thành chất C10H20O3. Axetyl hóa C1 bằng
CH3COCl, sau đó ozon phân/khử hóa, thu được C2 (C12H20O4). Oxi hóa C2 bằng nước brom, thu
được C3 (C12H20O5). Chất C3 tham gia chuyển vị Baeyer Villiger với m-CPBA (tỷ lệ mol 1:1) thu
được nhiều đồng phân trong đó có C4 (C12H20O6). Thủy phân C4 với H2SO4/H2O, thu được axit
ađipic HOOC[CH2]4COOH, butan-1,3-điol và axit axetic.
Xác định cấu tạo các chất C1, C2, C3 và C4.
Ghi chú viết tắt: PCC: piriđini clocromat; m-CPBA: axit m-cloropebenzoic
7.2. Khi xử lí chất A (C13H18O2) bằng dung dịch HCl loãng, thu được chất B (C11H14O)
không quang hoạt. Khi B phản ứng với Br2/NaOH, sau đó axit hóa sản phẩm phản ứng, thu
được chất C. Khi đun nóng B với hiđrazin/KOH trong etylen glicol, thu được chất D. Đun B
với benzanđehit trong môi trường kiềm, thu được chất hữu cơ E (C18H18O) duy nhất. Khi bị oxi
hóa mạnh, các chất B, C, D và E đều cho axit phtalic (axit benzen-1,2-đicacboxylic). Xác định
công thức cấu tạo của các chất A, B, C, D và E.

Câu ý Nội Dung Điểm


7 7.1 Từ sản phẩm axit ađipic HOOC-(CH2)4-COOH, butanđiol-1,3 và axit axetic;
theo dữ kiện C1 phản ứng với PCC tạo thành xeton, suy ra đầu ancol bậc 2
của butanđiol-1,3 có sẵn từ chất đầu C1. Vì vậy, cấu tạo của C4 hoàn toàn
xác định. Từ đó xác định được cấu tạo các chất C1, C2, C3.

1,0

7.2 Khi bị oxi hóa mạnh, các hợp chất B, C, D và E đều cho axit phtalic chứng

11
tỏ các hợp chất này là dẫn xuất của benzen bị thế hai lần ở vị trí 1,2. B có độ
không no k = 5 và có phản ứng bromofom nên B có thể là:

0,5

Do B không quang hoạt nên loại B4. Đun B với benzanđehit trong môi
trường kiềm, thu được chất hữu cơ E (C18H18O) duy nhất nên loại B3 do B3
ngưng tụ với benzanđehit tạo ra 2 sản phẩm hữu cơ khác nhau.

0,5

Khi xử lí hợp chất A (C13H18O2) bằng dung dịch HCl loãng thu được hợp
chất B chính là phản ứng thủy phân xetal.

Vậy A là một trong hai công thức dưới đây: 0,5

Câu 8. (2,5 điểm) Hóa học các hợp chất thiên nhiên (Cacbohidrat và các hợp chất hữu cơ
chứa nito đơn giản)
8.1. Cho hợp chất HSCH2CH(NH2)COOH (xistein) có các pKa: 1,96; 8,18; 10,28. Các
chất tương đồng với nó là HOCH2CH(NH2)COOH (serin), HSeCH2CH(NH2)COOH
(selenoxistein), C3H7NO5S (axit xisteic).
a) Hãy xác định cấu hình R/S đối với serin và axit xisteic.

12
b) Hãy quy kết các giá trị pKa cho từng nhóm chức trong phân tử xistein. Viết công thức
của xistein khi ở pH = 1,5 và pH = 5,5.
c) Sắp xếp 4 amino axit trên theo thứ tự tăng dần giá trị pHI .
8.1. Cho hợp chất HSCH2CH(NH2)COOH (xistein) có các pKa: 1,96; 8,18; 10,28. Các
chất tương đồng với nó là HOCH2CH(NH2)COOH (serin), HSeCH2CH(NH2)COOH
(selenoxistein), C3H7NO5S (axit xisteic).
a) Hãy xác định cấu hình R/S đối với serin và axit xisteic.
b) Hãy quy kết các giá trị pKa cho từng nhóm chức trong phân tử xistein. Viết công thức
của xistein khi ở pH = 1,5 và pH = 5,5.
c) Sắp xếp 4 amino axit trên theo thứ tự tăng dần giá trị pHI .
8.2. Glicozit thiên nhiên X chứa hợp phần quercetin
(công thức hình bên ). Thủy phân X bằng enzim α-
glicoziđaza, thu được chất X1 (C6H12O5) và chất X2. Thủy
phân X2 bằng enzim β-glicoziđaza, thu được chất X3
(C6H12O6) và quercetin. Metyl hóa hoàn toàn X rồi thủy
phân sản phẩm có xúc tác axit, thu được chất X4
(C9H18O5), chất X5 (C9H18O6) và chất X6. Oxi hóa X4
bằng HNO3, thu được chủ yếu là axit (2R, 4R)-2,3,4-
trimetoxipentanđioic, axit meso-2,3-đimetoxibutanđioic và
axit axetic. Oxi hóa X5 bằng HNO3 tạo thành axit meso-
2,3,4-trimetoxipentanđioic. Cho biết, X1 có cấu hình L, X3
có cấu hình D và X6 tồn tại ở một cân bằng xeto-enol.
Xác định (có giải thích) cấu trúc của các chất X1 - X5 và X, trong đó các hợp phần
cacbohiđrat được biểu diễn dưới dạng công thức Haworth (Havooc).
Câu ý Nội Dung Điểm
8 8.1 a)

0,5

b) pKa(xistein) 1,96 (COOH); 8,18 (SH); 10,28 (NH3+).


1,96 + 8,18
pHI (xistein) = = 5,07
2
Ở pH = 1,5
HS-CH2-CH(NH3+)-COOH
Ở pH = 5,5 0,25
HS-CH2-CH(NH2)-COO-
a) Trình tự tăng dần pHI:
Axit xisteic< selenoxistein< xistein<serin 0,25

1.2 Thủy phân X2 bằng enzyme β-glucoziđaza, thu được chất X3 (C6H12O6) có cấu
hình D, do đó X3 bền nhất là β-D-glucopyranozơ. X5 (C9H18O6) có nguồn gốc
từ X3, oxi hóa bằng HNO3, X5 tạo thành axit 2,3,4-trimetoxipentanđioic do đó
X3, X5 là:

13
0,5
X4 có nguồn gốc từ X1, có cấu hình L, oxi hóa X4 bằng HNO3, thu được chủ
yếu axit (2R,4R)-2,3,4-trimetoxipentanđioic (T), axit meso-2,3-
đimetoxibutanđioic và axit axetic nên có thể xác định được lập thể các nhóm
OH và bộ khung cacbon, cấu trúc của X1 và X4 hoàn toàn xác định:

axit (2R,4R)-2,3,4-trimetoxipentandioic axit meso dimetoxibutandioic

X4 X1 L-Rhamnozo

Cấu trúc của X từ đường L-Rhamnozơ, một đường D-glucozơ và quercetin như
sau:

0,5

0,5

14
Chú ý: X1 thuộc dãy L nên nhóm –OH hemiaxetal có cấu hình α khi nằm
ngược chiều với nhóm –CH3.

--------------------- HẾT ---------------------

15
SỞ GIÁO DỤC VÀ ĐÀO TẠO HDC ĐỀ THI HSG CÁC TRƯỜNG THPT CHUYÊN
TRƯỜNG THPT CHUYÊN VĨNH PHÚC KHU VỰC DH&ĐBBB NĂM 2023
ĐỀ THI MÔN: HÓA HỌC 11
ĐỀ ĐỀ XUẤT Thời gian làm bài: 180 phút, không kể thời gian phát đề.
đ HDC gồm 18 trang

Lưu ý: Đề thi chấm điểm theo thang điểm 20.


Câu 1 (2,5 điểm). Tốc độ phản ứng
1. Với phản ứng: 2SO32- + O2 → 2SO32- (1)
Biểu thức tốc độ toàn phần có thể được biểu diễn như sau:
𝑑[𝑆𝑂32− ]
− = 𝑘. [𝑆𝑂32− ]𝑎 [𝑂2 ]𝑏
𝑑𝑡
Nếu oxi dư, Biểu thức tốc độ có thể được biểu diễn lại như sau:
𝑑[𝑆𝑂32− ]
− = 𝑘′. [𝑆𝑂32− ]𝑎
𝑑𝑡
Trong đó: k’ = k.[O2]b
Tiến hành đo nồng độ sunfit [SO32-] theo thời gian t và nhận được ba đồ thị:
[SO32-] -t 1/[SO32-] -t ln[SO32-] -t

(đồ thị 1) (đồ thị 2) (đồ thị 3)


a) Xác định bậc phản ứng theo sunfit.
Tiến hành đo k’ theo các nồng độ oxi khác nhau và thu được kết quả được tổng hợp ở bảng sau:
[O2] 212,0 390,7 652,2 979,2
k’ 741,3 955,0 1230,3 1584,9
b) Xác định bậc phản ứng (b) theo O2.
2.Trong dung dịch nước, este ethyl 2-aminoacetat có thể tồn tại ở hai dạng: Dạng trung hòa
NH2CH2COOCH2CH3 (E) và dạng proton hóa NH3+CH2COOCH2CH3 (HE+) nằm cân bằng với nhau
theo phương trình:

Trong môi trường kiềm, hai dạng này đều bị thủy phân theo 2 phản ứng sơ cấp sau:

Thực nghiệm cho biết phản ứng thủy phân ethyl 2- aminoacetate trong môi trường kiềm ở
pH không đổi có bậc 1 đối với este và bậc 1 đối với OH-.
a) Hãy thiết lập biểu thức tính hằng số tốc độ k toàn phần của phản ứng thủy phân este trong
dung dịch kiềm. Từ biểu thức nhận được, đề xuất phương pháp thực nghiệm để xác định k2 và k3.
b) Khi có mặt của một ion kim loại chuyển tiếp M2+ (đóng vai trò chất xúc tác) sẽ xuất hiện
thêm một phản ứng thủy phân (4), đây cũng là phản ứng sơ cấp:

Thiết lập biểu thức tính hằng số tốc độ k toàn phần của phản ứng thủy phân este trong
trường hợp này và đề xuất phương pháp thực nghiệm để xác định k4.

1
Hướng dẫn chấm
Câu 1 Nội dung Điểm
1 a) Trong ba đồ thị, chỉ có đường thứ ba, (ln[SO32-] – t), có dạng tuyến tính. Có 0,25
nghĩa phản ứng (1) có bậc một theo SO32-.
b) Biết k’ = k.[O2]b. Lấy logarit cả hai vế được: lnk’ = lnk + b.ln[O2] (*)
Đồ thị (*) là đường phụ thuộc của lnk’ vào ln[O2]. 0,25
∆(𝑙𝑛𝑘 ′ )
Giá trị b là hệ số góc của đồ thị được tính theo công thức: 𝑏 = ∆(ln[𝑂2 ])
Tính lnk’, ln[O2]. Lập được bảng:
[O2]. 212,0 390,7 652,2 979,2
k’ 741,3 955,0 1230,3 1584,9
ln[O2]. 5,357 5,968 6,480 6,887 0,25
lnk’ 6,608 6,862 7,115 7,368
Trọn hai cặp giá trị bất kì ở bảng trên từ đó tính được các giá trị b, sau khi tính
trung bình thu được 𝑏̅ = 0,5 0,25
2 a) *Từ thực nghiệm:

Trong đó [E]T là nồng độ este tại thời điểm t.


[E]T = [E] + [HE+]T, với [E] và [HE+]T là nồng độ E và HE+ tại thời điểm t.
Giả thiết (1) là nhanh, (2), (3) là các quá trình chậm:

Theo (1):

0,25

𝑘 𝐾[𝐸]𝑡
Vì 𝐾 = 𝑘 1 = 5,01.10-7 rất nhỏ → [E] << [E]T và [𝐻𝐸 + ] =
−1 [𝑂𝐻]−
Từ đó:

Vì phản ứng thực hiện ở pH = const (nên [OH-] cũng là hằng số), nên cơ chế đề
xuất phù hợp với thực nghiệm. Vậy:
0,25

* Dùng phương pháp đồ thị để xác định hằng số k2, k3.


Bằng thực nghiệm xác định k ở các giá trị pH khác nhau, vẽ sự phụ thuộc của k
vào 1/[OH-]. Điểm cắt trục tung tại hoành độ bằng 0 chính là k2 và hệ số góc là 0,25
K.k3. Biết K nên xác định được k3.
b) * Thiết lập biểu thức tính hằng số tốc độ k toàn phần

0,25

2
0,25

*Trong biểu thức (*), các giá trị K, k2, k3 đã biết nên k phụ thuộc bậc nhất vào
nồng độ M2+. Tiến hành vẽ đồ thị sự phụ thuộc của k vào [M2+] thu được đường
0,25
thẳng với hệ số góc là k4. Hoặc có thể giải phương trình (*) tìm k4 ở mỗi cặp điểm
(k, [M2+]), sau đó lấy trung bình cộng các giá trị k4 tìm được.

Câu 2 (2,5 điểm). Cân bằng và phản ứng trong dung dịch. Pin điện- điện phân
1. Cho pin điện hóa:
(-)Zn | Zn(NO3)2 0,2M || AgNO3 0,1 M | Ag (+)
Các dung dịch Zn(NO3)2 và AgNO3 trong pin đều có thể tích 1,00L và ở 25oC.
a) Viết phương trình phản ứng ở mỗi điện cực và phương trình phản ứng xảy ra trong pin khi pin
phóng điện. Tính sức điện động (sđđ) của pin.
b) Tính tổng lượng điện có thể giải phóng tới khi pin phóng điện hoàn toàn và hằng số cân bằng của
phản ứng xảy ra trong pin.
Trong một thí nghiệm khác, khi cho KCl(r) vào dung dịch AgNO3 của pin ban đầu xảy ra sự
kết tủa AgCl(r) và thay đổi sđđ. Sau khi thêm KCl(r), sđđ của pin bằng 1,04V và nồng độ ion K+ bằng
0,300M.
a) Tính nồng độ mol của ion Ag+ tại cân bằng.
b) Tính nồng độ mol của ion Cl- và tích số tan của AgCl.
Cho biết: Eo (Zn2+/Zn) = -0,76V; Eo(Ag+/Ag) = +0,80V
Trong các tính toán, dùng nồng độ thay cho hoạt độ.
2.Trộn hai thể tích bằng nhau của hai dung dịch NaNO2 0,1M và I2 5.10-4M tạo thành dung dịch A.
a) Sau đó thêm hết 1 mL dung dịch NaOH 0,02M cùng một vài giọt hồ tinh bột (coi thể tích hồ
tinh bột thêm vào không đáng kể) vào 1 mL dung dịch A thì hồ tinh bột có màu không? Giải
thích trên cơ sở tính toán cụ thể.
b) Nếu thêm tiếp 2 mL dung dịch HCl 0,1M vào dung dịch thu được ở a) thì có biến đổi gì nữa
không? Giải thích trên cơ sở tính toán cụ thể.
Cho biết Eo (I2/2I-) = 0,536V ; Eo (NO3-/NO2-, OH-) = 0,01V ; Eo (NO2-/NO, H+) = 0,99V,
Ka(HNO2) = 5,1.10-4
Hướng dẫn chấm
Câu 2 Nội dung Điểm
1 a) Trong pin: (-) Zn | Zn(NO3)2 0,2M || AgNO3 0,1 M | Ag (+)
Xảy ra các phản ứng:
Ở anot (-): Zn(r)  Zn2+(aq) + 2e
Ở catot (+): Ag+(aq)  Ag(r) + e
0,25
Phản ứng của pin: Zn(r) + 2Ag+(aq)  Zn2+(aq) + 2Ag (1)
Sức điện động của pin (Epin):
Eopin = Eo+ - Eo- = 0,80 – (-0,76) = 1,56V

3
2
0, 0592 aZn2 a Ag 0, 0592 aZn2
E pin  E pin
o
 lg 2  1,56  lg 2
n aAg  aZn 2 a Ag 
a - Hoạt độ; hoạt độ của chất rắn bằng 1; một cách gần đúng, hoạt độ được tính
0, 0592 0, 200 0,25
theo nồng độ: E pin  1,56  lg  1,52V
2 0,1002
Epin > 0 và ∆G = -nFE < 0; chứng tỏ phản ứng (1) tự xảy ra trong quá trình pin
phóng điện.
b) Tính điện lượng Q và hằng số cân bằng K:
Khi phóng điện hoàn toàn Epin = 0 và phản ứng đạt cân bằng
0, 0592 0,25
0  1,56  lg K  K  1052,7
2
K rất lớn, nên cân bằng của phản ứng (1) chuyển hẳn về bên phải, và do đó thực tế
không còn Ag+ trong dung dịch.
Số mol electron giải phóng ne = 0,1.1 = 0,1 (mol). Do đó lượng điện giải phóng
trong pin 0,25
Q = nF = 0,1.96500 = 9650,0 (Culong)
c)Tính nồng độ mol của ion Ag+ và Cl- tại cân bằng:
Điện cực bên trái không đổi, nghĩa là nồng độ Zn2+ duy trì tại 0,200M.
Gọi x là nồng độ mol của ion Ag+ sau khi thêm xong KCl. Ta có:
0, 0592 0, 200
1, 04  1,56  lg 2  x  7,354.1010 M 0,25
2 x
Nồng độ mol của ion Cl- tại cân bằng:
[Cl-] = nồng độ thêm – nồng độ giảm do AgCl kết tủa
= 0,300 – (0,1 – 7,354.10-10) = 0,200M 0,25
Tích số tan TAgCl = [Ag+][Cl-] = 7,354.10-10.0,2 = 1,47.10-10
2 a)Thêm 1mL dung dịch NaOH 2.10-2M vào dung dịch thì nồng độ các chất sau khi
trộn sẽ là: [NO2-] = 2,5.10-2M ; [I2] = 1,25.10-4M ; [OH-] = 10-2M
Ta có các bán phản ứng:
I2 + 2e = 2I- Eo = 0,536V
NO2- + 2OH- = NO3- + H2O + 2e Eo = -0,01V
Phản ứng tổng: NO2 + 2OH + I2 = NO3 + 2I + H2O
- - - -
Eo = 0,526V
Hằng số cân bằng của phản ứng trên được tính như sau:
nE o
lg K   17, 77  K  1017,77
0, 0592 0,25
Do hằng số cân bằng của phản ứng này rất lớn, mặt khác [I2] << [NO2-] và [OH-]
nên I2 phản ứng gần như hoàn toàn với NO2- để tạo thành I- và NO3-.
Gọi nồng độ của I2 lúc cân bằng là x (vô cùng nhỏ). Khi đó nồng độ của các chất
tại cân bằng được tính như sau:
NO2- + 2OH- + I2 → NO3- + 2I- + H2O
o -2 -2 -4
CM 2,5.10 10 1,25.10 0 0
Cân bằng 2,4875.10-2 9,75.10-3 x 1,25.10-4 2,5.10-4
Nồng độ I2 được tính dựa vào hằng số cân bằng
2
 NO3   I  
K  1017,77
 NO2  OH   I 2 
  2

Giải phương trình trên thu được x = 5,61.10-24M (vô cùng nhỏ).
4
Vậy giả thiết đặt ra là hợp lý. Chính vì lượng I2 còn lại vô cùng nhỏ nên dung dịch 0,25
thu được sẽ không có màu.

b) Thêm vào hỗn hợp trên 2 ml dung dịch HCl 0,1M


Nồng độ các chất trước thời điểm phản ứng:
[NO2-] = 1,244.10-2 M ; [OH-] = 4,875.10-3 M ; [NO3-] = 6,25.10-5 M ;
[I-] = 1,25.10-4 M và [H+] = 0,05 M.
Các phản ứng lần lượt như sau:
H+ + OH- = H2O
H+ + NO2- = HNO2
Lượng H+ dư = 0,05 – 4,875.10-3 – 1,244.10-2 = 3,269.10-2 M
Do dư H+ nên có thể xảy ra phản ứng sau:
2HNO2 + 2I- + 2H+ = I2 + 2NO + 2H2O
Phản ứng trên là tổ hợp của các phản ứng sau:
2 x HNO2 ⇌ H+ + NO2- K1 = (5,1.10-4)2 = 10-6,58
2NO2 + 4H+ + 2e = 2NO + 2H2O Eo = 0,99V  K2 = 1033,45
2I- = I2 + 2e Eo = -0,536V  K3 = 10-18,11
0,25
→ 2HNO2 + 2I + 2H = I2 + 2NO + 2H2O
- +
K = K1K2K3 = 108,75
K = 108,75>>, với [I-] << [HNO2] và [H+] nên có thể xem như phản ứng xảy ra
hoàn toàn.
→ [I2] = 1,25.10-4 / 2 = 6,25.10-5 ; [NO] = 1,25.10-4 M .
Gọi x là nồng độ của I- tại cân bằng (x vô cùng nhỏ).
NO2- + 2H+ + 2I- → NO + I2 + 2H2O
CMo 1,244.10-2 3,269.10-2 1,25.10-4 0 0
CB 1,232.10-2 3,257.10-2 x 1,25.10-4 6,25.10-5

Với K = 108,75 giải ra được x = 1,04.10-7 M (rất nhỏ, tức giả thiết đặt ra là đúng).
Nghĩa là lúc này toàn bộ I- đã được oxy hóa thành I2 (6,25.10-5M) nên dung dịch thu
được có màu xanh (và có khí nâu thoát ra do NO chuyển thành NO2 trong không 0,25
khí).

Câu 3 (2,5 điểm).


1.Khí ga X chỉ có thành phần propan và butan với tỉ lệ khối lượng 22: 29.
a) Đốt cháy hoàn toàn 1,0 kg khí ga X ở 250C và 1 bar. Tính nhiệt lượng tỏa ra. Giả thiết nước sinh
ra ở thể khí.
b) Một buồng đốt được nạp 2,0 mol X và 100,0 mol không khí. Đốt cháy hoàn toàn X ở 250C và 1
bar. Sau phản ứng cháy thu được hỗn hợp Y, áp suất của Y luôn được duy trì ở 1,0 bar.
i) Tính nhiệt độ của hỗn hợp Y. Cho rằng toàn bộ nhiệt của phản ứng cháy tỏa ra chỉ để làm nóng
hỗn hợp Y.
ii) Toàn bộ hỗn hợp Y được dẫn qua thiết bị trao đổi nhiệt chứa nước lỏng ở 100C. Nhiệt của hỗn
hợp Y làm tăng nhiệt độ của nước lên 400C.
ii.1) Tính khối lượng nước lỏng bị làm nóng, biết rằng sau khi đi qua thiết bị trao đổi nhiệt, nhiệt
độ của hỗn hợp Y là 1100C.
ii.2) Khi nhiệt độ của Y hạ xuống một nhiệt độ nào đó sẽ làm một phần hơi nước trong hỗn hợp
Y bị ngưng tụ. Tính nhiệt độ mà tại đó hơi nước trong hỗn hợp Y bắt đầu ngưng tụ
ii.3) Nếu nhiệt độ của Y sau khi đi qua thiết bị trao đổi nhiệt còn lại là 400C, áp suất 1 bar, thì
khối lượng nước trong hỗn hợp Y bị ngưng tụ và khối lượng nước lỏng trong thiết bị trao đổi nhiệt bị
làm nóng là bao nhiêu?

5
Giả thiết: Sự thất thoát nhiệt không đáng kể; Hỗn hợp Y chỉ trao đổi nhiệt với nước lỏng
trong thiết bị trao đổi nhiệt; Nước lỏng được làm nóng bay hơi không đáng kể; Oxi chiếm 20% thể
tích không khí, còn lại là nitơ; Các chất khí là khí lí tưởng; Giá trị nhiệt dung đẳng áp không phụ
thuộc vào nhiệt độ.
Cho biết:
3816,44
(23,1964− )
- Áp suất hơi bão hòa của nước: 𝑃𝐻𝑠𝑎𝑡
2𝑂
(𝑃𝑎) = 𝑒 𝑡(0 𝐶)+227,02 ;

0
- Áp suất hơi bão hòa của nước ở 400C là: (∆𝑣 𝐻313 )𝐻2 𝑂(𝑙) = 43,3𝑘𝐽/𝑚𝑜𝑙;
- Điều kiện chuẩn: 250C và 1,0 bar.
Chất Nhiệt tạo thành Nhiệt dung đẳng áp
fH2980 (Kj.mol-1) CP (J.K-1.mol-1)
C3H8(k) -104,7 -
C4H10(k) -125,6 -
O2(k) 0 29,4
N2(k) 0 29,1
CO2(k) -393,5 37,1
H2O(k) -241,8 37,5
H2O(l) -285,8 75,2
2.Axit Dicacboxylic được trộn với etanol theo tỉ lệ 1: x (x>1) với sự có mặt của chất
xúc tác. Hệ thống đạt đến trạng thái cân bằng. Các hằng số cân bằng cho sự hình thành
monoester từ axit và etanol và cho sự hình thành của đieste từ monoester và etanol là như
nhau: K1 = K2 = 20. Tìm x để năng suất tạo monoester là cực đại và tính giá trị năng xuất cực
đại đó.

Hướng dẫn chấm


Câu 1 Nội dung Điểm
1 a)Xét các phản ứng:
C3H8(k) + 5O2 → 3CO2 + 4H2O (1)
C4H10(k) + 6,5O2 → 4CO2 + 5H2O (2)
Ta có:
0 0 0 0
(∆𝐻298 )(1) = 3. (∆𝑓 𝐻298 )𝐶𝑂2(𝑘) + 4. (∆𝑓 𝐻298 )𝐻2 𝑂(𝑘) − . (∆𝑓 𝐻298 )𝐶3 𝐻8(𝑘)
= 3. (-393,5) + 4. (-241,8) - (- 104,7) = - 2043,0 (kJ.mol-1)
0 0 0 0
(∆𝐻298 )(2) = 4. (∆𝑓 𝐻298 )𝐶𝑂2(𝑘) + 5. (∆𝑓 𝐻298 )𝐻2 𝑂(𝑘) − (∆𝑓 𝐻298 )𝐶4 𝐻10(𝑘)
= 4. (-393,5) + 5. (-241,8) - (- 125,6) = - 2657,4 (kJ.mol-1)
Trong 1 kg X có:
1000
𝑛𝐶3 𝐻8 = 𝑛𝐶4 𝐻10 = (𝑚𝑜𝑙)
102
Do đó, nhiệt lượng tỏa ra khi đốt cháy 1 kg X trong điều kiện chuẩn là:
1000
Q = 102 (2043,0 + 2657,4) = 46082,4 (𝑘𝐽) 0,25
b)i)Giả sử nhiệt độ của hỗn hợp thu được sau phản ứng > 100 C, khi đó H2O ở thể
0

khí.
n (O2) = 20 mol, n (N2) = 80 mol.
Trong X tỉ lệ mol C3H8(k) : C4H10(k) = 1 : 1 nên ta có sơ đồ sau:

6
→ H = H1 + H2 = 0
0 0
→ (∆𝐻298 )(1) + (∆𝐻298 )(2) + (𝑇 − 298) [7(𝐶𝑝 )𝐶𝑂2(𝑘) + 9(𝐶𝑝 )𝐻2 𝑂(𝑘) +
8,5(𝐶𝑝 )𝑂2(𝑘) + 80(𝐶𝑝 )𝑁2(𝑘) ] = 0

0 0
(∆𝐻298 )(1) +(∆𝐻298 )(2)
→ 𝑇 = 298 −
(𝑇−298)[7(𝐶𝑝 )𝐶𝑂 + 9(𝐶𝑝 )𝐻2 𝑂(𝑘) + 8,5(𝐶𝑝 )𝑂 + 80(𝐶𝑝 )𝑁2 (𝑘) ]
2(𝑘) 2(𝑘)

0,25
(−2043,0 −2657,4).1000
→ 𝑇 = 298 − = 1778,4 (𝐾) = 1505,4(0 𝐶)
7.37,1+9.37,5 +8,5.29,4 +80.29,1
→ t > 1000 C. Giả sử đúng
b) ii.1) Gọi m là lượng nước lỏng trong thiết bị trao đổi nhiệt được làm nóng. Vì
chỉ có nước lỏng nhận nhiệt từ hỗn hợp Y nên:
𝑄ℎℎ𝑌 + 𝑄𝐻2 𝑂(𝑙) = 0
Ta có:
(1778,4 - 383) [7(𝐶𝑝 )𝐶𝑂2(𝑘) + 9(𝐶𝑝 )𝐻2 𝑂(𝑘) + 8,5(𝐶𝑝 )𝑂2(𝑘) + 80(𝐶𝑝 )𝑁2(𝑘) ] =
𝑚
(313 − 283) 18(𝐶𝑝 )𝐻2 𝑂(𝑙)
→ (1778,4 - 383)[7.37,1 + 9.37,5 + 8,5.29,4 + 80.29,1] = (313 −
𝑚
283) 18.75,2
→ m =35,350.103 (gam) = 35,350(kg).
0,25
ii.2) Áp suất riêng phần của hơi nước trong hỗn hợp Y là:
𝑛𝐻 𝑂(𝑘) 9
𝑃𝐻2 𝑂(𝑘) = 2 .𝑃 = . 105 = 8612,4 (𝑃𝑎).
𝑛ℎℎ𝑌 104,5
Hơi nước bắt đầu ngưng tụ khi 𝑃𝐻2 𝑂(𝑘) = 𝑃𝐻𝑠𝑎𝑡
2 𝑂(𝑘)
3816,44
Exp(23,1964 - 𝑡(0 𝐶)+ ) = 8612,4
227,02
3816,44
→ t = 23,1964 − − 227,02 = 42,97 (0 𝐶).
ln(8612,4)
Vậy hơi nước bắt đầu ngưng tụ ở 42,970C.
0,25
ii.3) Tại 400C, áp suất hơi bão hòa của hơi nước là:
3816,44
𝑃𝐻𝑠𝑎𝑡
2 𝑂(𝑘)
= Exp(23,1964 - 40 + 227,02) = 7359,06 (𝑃𝑎)
Mặt khác:
𝑛𝐻2 𝑂(𝑘)
𝑃𝐻2 𝑂(𝑘) = .𝑃
𝑛𝐶𝑂2 (𝑘) + 𝑛𝑂2 (𝑘) + 𝑛𝑁2(𝑘) + 𝑛𝐻2 𝑂(𝑘)
𝑃𝐻 𝑂(𝑘) .(𝑛𝐶𝑂2(𝑘) + 𝑛𝑂2 (𝑘) + 𝑛𝑁2 (𝑘) )
→ 𝑛𝐻2 𝑂(𝑘) = 2 𝑃− 𝑃 𝐻2 𝑂(𝑘)
7359,06.( 7+8,5 +80)
= = 7,586 (𝑚𝑜𝑙).
105 −7359,06
Vậy lượng hơi nước đã bị ngưng tụ là (9,000 – 7,586) = 1,414 (mol).
→ Khối lượng nước bị ngưng tụ là: 1,414.18 = 25,452 (gam).
Gọi m’ là khối lượng nước lỏng được làm nóng từ 100C lên 400C. Ta có: 0,25

7
(1778,4 - 313). [7(𝐶𝑝 )𝐶𝑂2(𝑘) + 9(𝐶𝑝 )𝐻2 𝑂(𝑘) + 8,5(𝐶𝑝 )𝑂2(𝑘) + 80(𝐶𝑝 )𝑁2(𝑘) ] +
0 𝑚′
1,414. . (∆𝑣 𝐻313 )𝐻2 𝑂(𝑙) = (313 − 283) 18 . (𝐶𝑝 )𝐻2 𝑂(𝑙)
→ m’ = 0,25
0
(1778,4 − 313).[7(𝐶𝑝 )𝐶𝑂 + 9(𝐶𝑝 )𝐻2 𝑂(𝑘) + 8,5(𝐶𝑝 )𝑂 + 80(𝐶𝑝 )𝑁2 (𝑘) ] + 1,414..(∆𝑣 𝐻313 )𝐻2 𝑂(𝑙)
2(𝑘) 2(𝑘)
18. (313−283).75,2
= 37,612. 103 (gam) = 37,612 (kg).
Tính hiệu suất:
𝐴 + 𝐸 ← ⃗⃗⃗ 𝑀 + 𝐻2 𝑂

𝑀 + ⃗⃗⃗
𝐸 ← 𝐷 + 𝐻2 𝑂

[𝑀]
Có hiệu suất tạo monoester là ℎ = [𝐴]
0
[A]0 = [M] + [A] + [D]
𝐾1 [𝐴][𝐸] 𝐾1 𝐾2 [𝐴][𝐸]2
Với: 𝑀 = ;𝐷=
𝐻2 𝑂 [𝐻2 𝑂]2
[𝑀] 1
→ h= [M] = [𝐻2 𝑂] [𝐸]
+ [A] + [D] + 1 + 𝐾2
𝐾1[𝐸] [𝐻2 𝑂] 0,25
[𝐻2 𝑂]
Đặt [𝐸]
=𝑎
1 𝑎 𝐾2
→h= 𝑎 𝐾2 Để h cựa đại thì (𝐾 + 1 + ) đạt giá trị nhỏ nhất.
+ 1 + 1 𝑎
𝐾1 𝑎
𝑎 𝐾2
→( 𝐾 + ) đạt giá trị nhỏ nhất
1 𝑎
𝑎 𝐾2 𝐾
Theo bất đẳng thức Cosi: 𝐾 + ≥ 2√𝐾2
1 𝑎 1

𝑎 𝐾2 𝐾
Vậy →( 𝐾 + )min = 2√𝐾2
1 𝑎 1
1 1
Với K1 = K2 = 20 → h= = 0,25
1 +
𝐾
2√ 2 3
𝐾1

Tính x:
𝑎 𝐾2 𝐾 𝐾
Ta có: ( 𝐾 + )min = 2√𝐾2 → 𝑎2 − 2𝑎𝐾1 √𝐾2 + 𝐾1. 𝐾2 = 0
1 𝑎 1 1

𝐾
→ (𝐾1 𝑎 − √𝐾2 )2 = 0 → a = √𝐾1 𝐾2
1

[𝐻2 𝑂] 𝐾1
Với [𝐸]
= √𝐾1 𝐾2 → [M]= [A]. √ và [A] = [D]
𝐾2
𝐾
Theo cân bằng có: [H2O]0 = [M] + 2[D] = [A].(2 +√𝐾1 )
2
2 1
[E] = [A].( + ) 0,25
√𝐾1 𝐾2 𝐾2
→ Nồng độ đầu của E và A lần lượt là:
2 1 𝐾
[E]0 = [E] + [M] + 2[D] = [A].( + ) + [A]. √𝐾1 + 2[A]
√𝐾1 𝐾2 𝐾2 2

8
𝐾
[A]0 = [M] + [A] + [D] = [A]. √𝐾1 + 2[A]
2
[𝐸0 ] 1 [𝐸0 ] 0,25
𝑥= = 1+ ; với: K1 = K2 = 20 → 𝑥= = 1,05
𝐴0 ] √𝐾1 𝐾2 𝐴0 ]
1
Vây: Hiệu suất lớn nhất khi 1: x = 1,05 và h= 3
Câu 4 (2,5 điểm).
1. Các dữ liệu về một số phức chất bát diện có chứa phối tử nitrit được cho trong bảng sau:

Phức chất %mM %mN %mC dN-O (Å) Góc ONO (o) Góc OMO (o)

[ML4(NO2)2] (1) 12,45 18,03 51,50 1,21 và 1,29 122 180

[MR2(NO2)]+ (2) 13,94 16,83 57,69 1,24 114,5 54

[MX4(NO2)2] (3) 26,61 38,53 - 1,24 115 -

Ở đây %m là khối lượng, d là độ dài liên kết; M là ion kim loại; L và R là các phối tử hữu
cơ thông dụng chỉ chứa C, H và N; phối tử X chỉ chứa N và H. Trong phối tử L, R và X, các
nguyên tử N đều tham gia phối trí.
1.Vẽ các kiểu liên kết có thể có của ion nitrit với nguyên tử kim loại trung tâm trong các phức chất
đơn nhân.
2. Xác định công thức phân tử và vẽ cấu trúc các phức chất (1), (2) và (3).
2.Cho sơ đồ phản ứng:
A + NH3(lỏng) → B + C
𝒕𝟎 𝑪
C + KNO3 → B + D + NH3
C + PCl5 → G(rắn) + E(rắn) + F(rắn)
C + F → E(rắn) + NH3
C + N2O → D + B + NH3
Biết các chất A, G, E, D đều chứa nguyên tử của hai nguyên tố trong phân tử. G chứa hai
nguyên tố thuộc cùng một nhóm và hai chu kì kế tiếp nhau. Chất A chứa kim loại hoạt động rất
mạnh. Xác định các chất có trong sơ đồ trên.
Hướng dẫn chấm
Câu 4 Nội dung Điểm
1 a) Các kiểu liên kết của ion nitrit trong các phức chất:

0,25

b)
* Phức chất 1
Trong phức chất (1), có độ dài liên kết N – O là 1,21 và 1,29 Å nên phối tử
nitrit liên kết với kim loại M theo kiểu (c) và là phối tử đơn càng. Mặt khác phức
chất (1) là phức bát diện, số phối trí của M là 6 nên phối tử L có dung lượng phối trí
1 (phối tử đơn càng) và mỗi phối tử L có một nguyên tử N. Do đó phức chất (1) có
6 nguyên tử N, chiếm 18,03% khối lượng.

9
14, 6
Suy ra khối lượng phân tử của (1) là: .100  466( g / mol )
18, 03
466.12, 45
Từ % khối lượng kim loại, ta có M   58( g / mol ) . Vậy kim loại là Ni.
100
Gọi công thức của phối tử L là CxHyN. Từ % khối lượng của C, ta có
466.51,5 0,25
x 5
100.4.12
Từ phân tử khối của phức (1) [Ni(C5HyN)4(NO2)2] = 466  y = 5.
Công thức của phức chất (1): [Ni(C5H5N)4(NO2)2], trong đó C5H5N: Pyridin (Py).
Trong phức chất (1) góc OMO là 180o nên phối tử ONO ở vị trí trans với
nhau, nên cấu trúc của phức chất (1) là:

0,25

*Phức chất (2):


Trong phức chất này độ dài liên kết N-O bằng nhau, góc OMO là 54o; góc
ONO là 114,5o do đó phối tử nitrit phối trí với kim loại theo kiểu (b) và là phối tử
hai càng.
M là Ni có số phối trí trong phức chất này là 6. R là phối tử hai càng nên R
có có 2 nguyên tử N. Tính toán tương tự như đối với phức (1), từ % khối lượng Ni
ta tính được phân tử khối của phức (2) là 416.
416.57, 69
Đặt R là CxHyN2, ta có x   10
100.2.12
Từ phân tử khối của [Ni(C10HyN2)2(NO2)]+ bằng 416  y = 8. Vậy R là
C10H8N2: bispyridin.

Phức chất (2) là [Ni(Bipy)2(NO2)]+


Cấu trúc của phức chất (2): 0,25

0,25

*Phức chất (3):


Ở phức chất (3), phối tử nitrit có độ dài liên kết N – O bằng nhau và là phối
tử đơn càng (do kim loại có số phối trí 6 và có 6 phối tử) nên phối tử nitrit liên kết
với M theo kiểu (a).

10
M là Ni, từ % khối lượng của Ni ta tính được phân tử khối của phức (3) là:
58.100
 218( g / mol )
26, 61
Từ phân tử khối của [NiX4(NO2)2] là 218  X = 17 là NH3. Vậy công thức
0,25
của phức (3) là [Ni(NH3)4(NO2)2].
Cấu trúc của phức chất:

0,25

2 - Oxit A tác dụng NH3, vì vậy NH3 tương tự nước tác dụng với A → H2O và muối
của NH2, H2O tác dụng oxit A tạo hidroxit tương ứng của A → do đó B, C có
cùng kim loại. Mặt khác B, D là sản phẩm khi C tác dụng KNO3 nên kim loại
trong A là kali (K).
→B là KOH và C là KNH2 → D là KN3. 0,75
- KNH2 tác dụng PCl5 tạo G chứa hai nguyên tố thuộc cùng 1 nhóm và hai chu kì
kế tiếp → G phải chứa P và N.
- KNH2 tác dụng PCl5, KNH2 có vai trò như một chất nucleophin tấn công PCl3 sẽ
đấy Cl và tạo muối KCl → E là KCl
- F là chất rắn không là hợp chất nhị phân nên F là muối, phân tử có N, H, Cl → F
là NH4Cl
- KNH2 + PCl5 → chất F là NH4Cl và chất E là KCl không có sự thay đổi số oxi
hóa nên P và N trong G vẫn có số oxi hóa là +5 và -3. → G là P3N5.
Kết quả :
A B C D E F G
K2O KOH KNH2 KN3 KCl NH4Cl P3N5
Câu 5 (2,5 điểm).
1. A, B, C, D là các đồng phân cấu dạng của [4,4,0] bixiclodecan-1,5-diol. Khi thực hiện cùng 1
chuyển hóa với TsCl, sau đó với Natri terbutylat, A và C đều cho trans xicloankenon còn B cho cis
xicloankenon, cuối cùng D không cho phản ứng. Lập luận để xác định cấu dạng các đồng phân A,
B, C, D.
2. Diclometan có cấu tạo như sau:

Giải thích vì sao không tồn tại phản ứng thế nucleophin của Diclometan.
3. Oxepin là chất không ổn định do đó nó nằm cân nhiệt với A

a. Vẽ cấu trúc (có lập thể) của A


b. Khi xử lí oxepin bằng tia cực tím, người ta không thấy sự xuất hiện của A mà thay vào đó thu
được B, xác định cấu trúc (có lập thể) của B
Hướng dẫn chấm
Câu 5 Nội dung Điểm
11
1 Phản ứng diễn ra với sự xen phủ HOMO của liên kết 2-3 vào LUMO của C-OTs ,
ứng với sự xen phủ đôi e oxi vào LUMO của liên kết 2-3 ( liên kết 2-3 ngược phía
so với liên kết C-OTs )

0,5

Do đó ứng với các đồng phân để thu được sản phẩm sẽ là:

0,5

Với A và C cho đồng phân trans còn B cho đồng phân cis.
ứng với đồng phân không sảy ra phản ứng ( D) sẽ là dạng không có sự xen phủ như
đã nói:

0,5
2 Vì có hiệu ứng anomeric. Đôi e tự do của Cl xen phủ vào LUMO ( σ*) của liên kết
C-Cl còn lại)

0,5

Các lk CCl đều đc ổn định . Do đó phản ứng thế Clo sảy ra khó khăn

3 a. đồng phân thu được trong điều kiện to dưới đây là cis

0,25

b. đồng phân thu được trong điều kiện hv dưới đây là cis

12
0,25
Câu 6 (2,5 điểm).
1.Vào những năm 60, một hợp chất hidrocacbon được tổng hợp. Hợp chất này được lấy theo tên của
vị giáo sư Barbara M. Ferrier. Quy trình tổng hợp của nó như sau:

LDA = LiN(i-Pr)2
TsNHNH2 = p-CH3C6H4SO2NHNH2
a.Xác định các chất chưa biết ở dãy trên
b.Cho biết phổ NMR-C13 của H có bao nhiêu tín hiệu tại nhiệt độ phòng 298K và 163K
2.Ondansetron là thuốc chống nôn phổ biến. Nó được biết đến là chất đối kháng thụ thể 5-HT3 có
chọn lọc cao. 5HT3 là chất được giải phóng ra khi tiến hành hóa trị hoặc xạ trị, chất này có ở ruột
non và gây phản xạ nôn. Dưới đây là quy trình tổng hợp nó:

Xác định cấu trúc các chất A, B, C


3.Đề xuất cơ chế hình thành sản phẩm cho các phản ứng dưới đây

Hướng dẫn chấm

Câu Nội dung Điểm


6
1 a)

0,5

13
0,25
b)
tồn tại quá trình tautome hóa như sau:

0,25

ở nhiệt độ phòng thì tồn tại sự đồng phân hóa nên tín hiệu 1-5 và 2-4 sẽ trùng nhau, nên
ta có 4 tín hiệu
Còn ở nhiệt độ 163K thì không đủ năng lượng để cho sự tautome hóa. Nên sẽ có 6 tín
hiệu.
2

0,5

3 a.

0,5

b.

14
0,5

Câu 7 (2,5 điểm).

Kim cương là một khoáng chất quý hiếm là một trong những dạng biến dị hướng của cacbon.
Các hidrocacbon A (C10H16) và B (C14H20) có bộ xương hidrocacbon giống kim cương. Có thể thu
được hợp chất A bằng cách đồng phân hóa C (C10H16) bằng cách đun nóng với nhôm bromua. Chất
C thu được bằng cách hidro hóa D (C10H12). Hợp chất D có các đặc tính thú vị, khi đun nóng nó sẽ
biến thành chất E, chất này khi ổn định lại chuyển đổi trở lại D. Phản ứng của E với etilen đun nóng
cho hidrocacbon F (C7H10). Khi chiếu xạ F dẫn đến hợp chất G (C14H20), đồng phân hóa G bằng tác
dụng của nhôm bromua thu được B.

1.Xác định các chất chưa biết ở trên và cho biết động lực của phản ứng đồng phân hóa ở bài trên là

2.Nêu sự khác biệt cơ bản giữa phản ứng tạo F → G và phản ứng D → E.

3.Metyl hóa A thu được A1 (không có cacbon bậc 4). Dưới tác dụng của AlBr3, A1 đồng phân hóa
thành A2. Tìm A1, A2 và đề suất cơ chế cho sự đồng phân hóa này.
Hướng dẫn chấm

Câu Nội dung Điểm


7
1 Dãy chuyển hóa như sau:

15
1,0

0,25

Động lực :
+ Cấu trúc ổn định hệ vòng của A và B
+ Sự hình thành A và B qua các cacbocation trung gian dưới sự lấy H- của acid
lewwis AlBr3 , và sự chuyển dịch dẫn đến các cacbocation bền là động lực
2 Sự khác biệt ở đây là sự restro D-A dưới tác dụng của nhiệt độ còn sự photodimerization
tạo G không sử dụng nhiệt được 0,25
3 A1 và A2 là:

0,5

Đề xuất cơ chế:

0,5

Câu 8 (2,5 điểm).


1.Xử lí glucose bằng vi sóng thu được A (C6H6O3). Trùng hợp sản phẩm khử của A là B với
hexametylen – 1,6-diisocyanat thu được C. Cho A tác dụng với anpha-naphthol trong môi trường
axit thu được D. Trong không khí D xuất hiện màu do hình thành E. Xác định các chất chưa biết.
2.Proline là aminoaxit rất đặc biệt trong 20 aminoaxit của cơ thể. Dưới đấu là các sơ đồ tổng hợp
nó. Hãy hoàn thành 2 dãy tồng hợp đó.

16
Pi là nhóm photphat
Hướng dẫn chấm
Câu Nội dung Điểm
8
1

1,0

2 Dãy 1

0,75

Dãy 2

0,75

--------------Hết---------------

17
18
SỞ GD – ĐT QUẢNG NAM KỲ THI HỌC SINH GIỎI CÁC TRƯỜNG THPT CHUYÊN
TRƯỜNG THPT CHUYÊN KHU VỰC DUYÊN HẢI VÀ ĐỒNG BẰNG BẮC BỘ
NGUYỄN BỈNH KHIÊM LẦN THỨ XIII, NĂM 2023
ĐỀ THI MÔN: HOÁ HỌC 11

HƯỚNG DẪN CHẤM Thời gian: 180 phút (không kể thời gian giao đề)

Câu 1: Tốc độ phản ứng


1.1. Xét phản ứng sau: CH3Br + OH- → CH3OH + Br- (1). Ở 25oC, thực hiện 2 thí nghiệm với
nồng độ OH- như sau:
Thí nghiệm 1: [OH-]0 = 3,0 M
t (h) 1,0 1,5 2,3 4,0 5,7 7,0
[CH3Br]
7,08.10-3 4,52.10-3 2,23.10-3 4,76.10-4 1,04.10-4 3,41.10-5
(M)
Thí nghiệm 2: [OH-]0 = 4,5 M
t (h) 0 0,70 1,7 2,6 3,6
[CH3Br]
4,50.10-3 1,70.10-3 4,19.10-4 1,11.10-4 2,81.10-5
(M)
Mặt khác, người ta cũng tiến hành thực hiện phản ứng (1) ở 85oC.
Khi [CH3Br]0 = 1,0.10-2 và [OH-]0 = 3,0 M, hằng số tốc độ phản ứng là 326 (đơn vị thời gian
trong hằng số tốc độ là giờ).
a. Xác định giá trị hằng số tốc độ của phản ứng ở 25oC và biểu thức tốc độ.
b. Xác định năng lượng hoạt hoá của phản ứng trên.
1.2. Trong các dung môi khác nước, phản ứng thế nucleophin thường diễn ra với sự tạo thành
một cặp ion. Một cơ chế bất thường đã được đề xuất cho phản ứng trong methanol: PtRBr+ +
I- → PtRI+ + Br- (R: phối tử bốn càng tris-(o-diphenylarsinophenyl)arsin) với sự tạo thành
không chỉ một mà là hai cặp ion:
⎯⎯
→ PtRBr+,I– (K)
(1) PtRBr+ + I– ⎯

PtRBr+,I– + I– → PtRI+,I– + Br– (kip)


PtRI+,I– → PtRI+ + I– (k1 rất nhanh)
(2) PtRBr+ + I– → PtRI+ + Br– (k0)
a. Đề xuất các công thức để tính phần mol PtRBr+ và PtRBr+, I– trong cân bằng (1) với I–
lấy dư ([I–] ≈ CI).

1
b. Sử dụng phương pháp gần đúng nồng độ dừng, hãy dẫn ra phương trình động học và biểu
thức hằng số tốc độ thực tế quan sát được (k’).
CÂU NỘI DUNG ĐIỂM
1.1 a. Trong 2 thí nghiệm, vì [OH-]0 >> [CH3Br] => nồng độ OH- thay đổi
không đáng kể.
(1,0 0,125
điểm) v = k.[OH ] .[CH3Br] = k’.[CH3Br] , với k’ = k.[OH ] = const
- m n n - m
0,125
TN1, xác định được n = 1 và k’(1) = 0,893 h -1
0,125
TN2, xác định được n = 1 và k’(2) = 1,42 h -1

Ta có: k’ = k.[OH-]m => ln k’ = ln k + m.ln [Y]


Từ đó thiết lập được hệ phương trình bậc nhất 2 ẩn là ln k và m
0,25
-1 -1
=> m = 1; k = 0,300 M .h .
0,125
Do đó biểu thức tốc độ là v = (0,300 M .h )[CH3X].[Y]
-1 -1

b. Theo phương trình Arrhenius, ta có:


k1
R.ln
k2
Ea = = 103 kJ/mol 0,25
1 1

T2 T1

1.2 ⎯⎯
→ PtRBr + , I−
a. PtRBr + + I− ⎯ [PtRBr + , I − ]
⎯ K=
[PtRBr + ][I − ]
(1,5 0,25
điểm) [I ] ≈ CI nên [PtRBr ,I ] = K[PtRBr ]CI (*)
- + − +

Theo cân bằng vật chất: Co = [PtRBr+,I–] + [PtRBr+] 0,125


Thế biểu thức (*) từ hằng số cân bằng,có:

Co = [PtRBr + ](1 + KCI ) 0,125

[PtRBr + ] 1
=> (PtRBr + ) = =
Co 1 + KCI

[PtRBr + , I− ] 1 KCI 0,25


(PtRBr + , I− ) = = 1− =
Co 1 + KCI 1 + KCI

b. Phản ứng diễn ra qua hai hướng:


+ Có sự ghép cặp ion vip = kip[PtRBr+,I-]CI
+ Không có sự ghép cặp ion vo = ko[PtRBr+]CI
+ −
0,25
Sử dụng phương pháp gần đúng nồng độ dừng d[PtRI , I ] = 0 , ta có:
dt

2
k1[PtRBr+,I-] = kip[PtRBr+,I-]CI
+
0,25
Tốc độ tổng v = d[PtRI ] = k o [PtRBr + ]+k1[PtRI+ , I− ]
dt

v = vo + vip = ko[PtRBr+]CI + kip[PtRBr+,I-]CI.


2
k o Co CI Kk ip Co CI k + Kk ipCI
v= + và k ' = o 0,25
1 + KCI 1 + KCI 1 + KCI

Câu 2: Cân bằng và phản ứng trong dung dịch. Pin điện - điện phân
Một mẫu dung dịch NaOH để lâu trong không khí bị hấp thụ CO2, tạo thành dung dịch X chứa
NaOH và Na2CO3. Tiến hành chuẩn độ 10,0 mL dung dịch X bằng dung dịch HCl 0,02M.
- Nếu sử dụng chỉ thị phenolphtalein (pT = 8,0) thì đến thời điểm mất mầu chị thị đã dùng
hết 12,5 mL dung dịch HCl. Dung dịch sau chuẩn độ được kí hiệu là dung dịch Y.
- Nếu sử dụng chỉ thị metyl da cam (pT = 4,0) thì đến thời điểm chỉ thị đổi màu từ vàng
sang đỏ đã dùng hết 15,0 mL dung dịch HCl.
a. Bỏ qua sai số chuẩn độ, lập luận để viết phương trình hóa học của phản ứng chuẩn độ
và tính nồng độ của NaOH và Na2CO3 trong X.
b. Thêm tiếp 33,50 mg Ba(NO3)2 rắn vào dung dịch Y, khuấy đều. Cho biết kết tủa BaCO3
có xuất hiện hay không?
c . Sục liên tục khí H2 (P = 1 atm) vào 50,0 mL dung dịch X đã được nhúng sẵn một thanh Pt
để tạo thành điện cực I. Ghép điện cực này với điện cực KCl (0,01M) |AgCl, Ag để tạo thành pin
điện ở 25oC.
Lập luận để viết sơ đồ pin điện ( Nếu HS không giải đc ý trên thì lấy nồng độ NaOH là
0,02M và Na2CO3 0,005M để giải ý này). Viết phương trình của phản ứng xảy ra tại các điện
cực và phản ứng tổng quát khi pin hoạt động.
Cho biết: H2CO3 có pKa1 = 6,35 và ; pKa2 = 10,33;
RT
ở 25o C: F lnX = 0,0592lgX ; E oAg +
/Ag
= 0, 799 V

Ba(OH)+ có log*β = 13,36; BaCO3 có pKs = 8,30; AgCl có pKs = 10.


Độ tan của CO2 trong điều kiện thí nghiệm trên là 0,03 M.
Coi quá trình thêm Ba(NO3)2 không làm thay đổi thể tích dung dịch Y.
Không có sự thay đổi thể tích gây ra bởi sự pha trộn dung dịch.
CÂU HƯỚNG DẪN ĐIỂM
(2,5 a. Gọi nồng độ của NaOH là x (M); Na2CO3 là y (M).
điểm)

3
* Khi mất màu phenolphtalein, pH = 8,0:
[CO32− ] K a 2

= = 10−2,33  [CO32-] << [HCO3-]
[HCO3 ] h

[HCO3− ] K a1
= = 101,65  [H2CO3] << [HCO3-]
[H 2CO3 ] h

 Thành phần chính của hệ ở pH = 8,0 là HCO3-. 0,125


Vậy phản ứng chuẩn độ là: OH + H → H2O - +
(1)
CO32- + H+ → HCO3- (2)
 x + y = 12,5.0,02/10 = 0,025 (I) 0,25
* Khi đổi màu metyl dacam, pH = 4,0:
[CO32− ] K a 2

= = 10−6,33  [CO32-] << [HCO3-]
[HCO3 ] h

[HCO3− ] K a1
= = 10−2,35  [HCO3-] << [H2CO3]
[H 2CO3 ] h
0,125
 Thành phần chính của hệ ở pH = 4,0 là H2CO3.
Vậy phản ứng chuẩn độ là: OH- + H+ → H2O (1)
CO32- + 2H+ → H2CO3 (2)
 x + 2y = 15.0,02/10 = 0,03 (II)
0,25
Giải hệ (I), (II)  x = 0,02M; y = 0,005M.
0,25
33,5
b. Nồng độ Ba2+ trong Y: CoBa = 2+ = 5, 70.10−3 M
261.22,5

Tại pH = 8,0 thì sự thủy phân của Ba2+ là không đáng kể.
Mặt khác, ta có:
−6,35−10,33
0, 005.10 K a1K a 2 0, 005.10 10
CCO2− = . 2 = . −16,68 = 2, 23.10−5 M
3
22,5 h + hK a1 + K a1K a 2 22,5 10 + 10−8,34−6,35 + 10−6,35−10,33 0,25

 CBa .CCO = 10−6,90  K s ( BaCO3 ) = 10−8,3


2+ 2−
3

0,25
 Có kết tủa BaCO3 xuất hiện.
c. Xét dung dịch X gồm NaOH Co1 = 0,02M và Na2CO3 Co2 = 0,005M.
Có các cân bằng:
⎯⎯
→ HCO3- + OH-
(1) CO32- + H2O ⎯
⎯ Kb1 = 10-3,67

4
⎯⎯
→ H2CO3 + OH-
(2) HCO3- + H2O ⎯
⎯ Kb2 = 10-7,65

⎯⎯
→ H+ + OH-
(3) H2O ⎯
⎯ Kw = 10-14

Do Kb1.Co2 >> Kb2.Co2 >> Kw nên pH tính theo cân bằng (1).
Xét cân bằng (1):

(1) CO32- + ⎯⎯

H2O ⎯
⎯ HCO3- + OH- Kb1 = 10-3,67

Co2 0,005 0,02


[] 0,005 – x x 0,02 + x
Giải x = 5,27.10-5M  [OH-] = 0,02M  pH = 12,30.
0,25
- Khi đó, ta có: E1 = -0,0592pH  - 0,7282 V
- Mặt khác, EAgCl/Ag = 0,799 + 0,0592lg(10-10/0,01) = 0,3254V
EAgCl/Ag > E1. Vậy sơ đồ pin là :
(-) Pt (H2, p = 1atm)| NaOH, Na2CO3 || KCl(0,01 M) | AgCl, Ag (+)
0,25
Các phản ứng:
Tại anot: H2 + 2OH- → 2H2O + 2e
0,125
Tại catot: AgCl + 1e → Ag + Cl -
0,125
Tổng quát: 2AgCl + H2 + 2OH → 2Ag + 2Cl + 2H2O
- -
0,25

Câu 3: Nhiệt động học và cân bằng hóa học


Metan được dùng làm nguyên liệu cho một số loại máy sưởi ấm khi trời lạnh. Tuy nhiên,
sử dụng máy sưởi này có nguy cơ ngộ độc khí CO khi hệ thống máy bị lỗi hoặc không được
điều chỉnh đúng. Nồng độ CO trong không khí bình thường cho phép là ≤ 1 ppm, ngưỡng cho
phép tối đa cho không khí trong nhà là 9 ppm.
Các phản ứng sau đây xảy ra trong máy sưởi khí metan:
(I) CH4(k) + 2 O2(k) → CO2(k) + 2 H2O(k)
3
(II) CH4(k) + O2(k) → CO(k) + 2 H2O(k)
2

Giả thiết không khí có thành phần gồm 80% N2 và 20% O2.
Cho các dữ kiện nhiệt động học ở 25 oC ở 1 atm như sau:
Chất CH4(k) O2(k) CO2(k) CO(k) H2O(k)
 f H 0 (kJ.mol−1 ) -74,9 0 -393,5 -110,5 -241,8

5
Sm 0 (J.K −1.mol−1 ) 186,2 205,0 213,6 197,6 188,7

a. Tính hằng số cân bằng của cả hai phản ứng ở T = 1500 K
b. Không khí được đưa vào lò đốt với lượng sao cho tỷ lệ số mol CH4/O2 = 1/2. Giả sử rằng
ở trạng thái cân bằng n(CH4) = 0 (T = 1500 K, p = 1 atm).
i) Tìm mối quan hệ giữa số mol oxi và cacbon monoxit ở trạng thái cân bằng.
ii) Giả sử số mol CO là rất nhỏ so với mol CO2. Tính thành phần phần trăm số mol CO ở
trạng thái cân bằng.
c. Khi không khí đưa vào lò đốt theo tỉ lệ mol CH4/O2 = 1/4. Tính lại phần trăm số mol CO ở
trạng thái cân bằng.
d. Quá trình hỗn hợp khí giảm từ 1500K đến nhiệt độ phòng, giả sử hơi nước bị ngưng tụ hết
và thành phần khí không thay đổi. Tính nồng độ CO (ppm) theo thể tích trong 2 trường hợp
b) và c).
CÂU NỘI DUNG ĐIỂM
(2,5 a. Phản ứng I
điểm) ΔHI0 = -393,5 + 2⋅(-241,8) - (-74,9) = –802,2 kJ . mol-1
ΔSI0 = 213,6 + 2.188,7 - 186,2 + 2⋅205,0 = –5,2 J.K-1 . mol-1
ΔGI0 = ΔH0 - T.ΔS0 = –802,2 - 1500.(-5,2.10-3) = –794,4 kJ . mol-1
−3
K I = e−G = e− ( −794,4)/(8,314.10 = 4,62.1027 0,25
0
/RT .1500)

Phản ứng II
ΔHII0 = (−110,5) + 2.(−241,8) − ( − 74,9) = − 519, 2 kJ.mol−1

3
ΔSII0 = 197, 6 + 2.188, 7 − 186, 2 − .205, 0 = 81,3 J.K −1.mol−1
2

ΔGII0 = ΔH0 - T.ΔS0 = –519,2 - 1500.(81,3.10-3) = - 641,15kJ.mol-1

K II = e−G
0
/RT
= e− ( −640,95)/(8,314.10
−3
.1500)
= 2,13.1022 0,25

b.
i) Vì KI >> KII và đều rất lớn nên có thể cho rằng phản ứng là hoàn toàn.
Do đó, khi n CH = 1 mol, n O = 2 mol thì ở trạng thái cân bằng:
4 2

n CH4 = 0, n H2O = 2n CH4 pu = 2mol, n N2 = 8 mol, n CO2 + n CO = n CH4 pu = 1 mol

n O2 = 2 − 2n CO2 − 1,5CO = 2 − 2(n CO2 + n CO ) + 0,5n CO = 2 − 2 + 0,5n CO = 0,5n CO

0,25

6
Vậy phương trình biểu diễn mối quan hệ giữa O2 và CO ở trạng thái cân
bằng: n O = 0,5n CO
2

ii)
- Trong phản ứng I) tổng số mol không thay đổi, trong phản ứng II, số mol
khí thay đổi từ 8,5 mol - 9 mol (tính cả số mol N2), nhưng do phần mol
CO rất nhỏ so với CO2 và KII << KI nên sự thay đổi này không đáng kể
=> có thể xem tổng số mol khí không thay đổi đáng kể (nsau ≈ ntrước).
- Tổng số mol khí = n CH + n O + n N = 1 + 2 + 8 = 11 mol
4 2 2 0,25
n CO2 1 − n CO 1
x CO2 = = 
n tong n tong 11

Mặt khác,
1/2
K II p CO .(p O2 ) x CO .(x O2 .p) 0,5 x CO .(0,5x CO .p) 0,5
= = = (vì x O2 = 0,5x CO )
KI p CO2 x CO2 x CO2

1/2
K 2
1/2
2,13.1022  2  1
= ( x CO ) = II .   .x CO2 = .  . = 5,84.10−7
3/2
27 
KI  p  4, 6.10  1  11 0,25
−5
= x CO = 7, 05.10

c.
- Tổng số mol khí = n CH + n O + n N = 1 + 4 + 16 = 21 mol
4 2 2

- Tại trạng thái cân bằng


n O2 = 4 − 2n CO2 − 1,5CO = 4 − 2(n CO2 + n CO ) + 0,5n CO = 4 − 2 + 0,5n CO = 2 + 0,5n CO = 20,25
n CO2 = 1 − n CO

1/2 1/2
K II x CO .(x O2 .p)  2
= = x CO   .21 = 421/2.x CO
KI x CO2  21 

K II 2, 09.1022 0,5
=> x CO = 42−1/2. = 42−1/2. = 7,145.10−7
KI 4, 6.1027

d. Do sự ngưng tụ của hơi nước, số mol trong pha khí thay đổi từ 11 thành
9 trong trường hợp b) và từ 21 thành 19 trong trường hợp c). Do đó phần
trăm số mol của CO thay đổi thành:
0,25
11
x CO (b) = 6,99.10−5. = 8, 617.10−5 = x CO (b) = 86,17 ppm
9
21
x CO (c) = 7, 01.10−7. = 7,897.10−7 = x CO (c) = 78,97 ppm 0,25
19

7
Câu 4: Hóa nguyên tố. Phức chất
4.1. Hợp chất X1 màu vàng, gồm 2 nguyên tố, hòa tan được hoàn toàn trong axit nitric đặc
khi đun nóng, giải phóng ra một chất khí có tỉ khối bằng 1,586 lần so với không khí. Khi thêm
bari clorua dư vào dung dịch thu được ở trên, một chất rắn màu trắng X2 được tách ra. Lọc
kết tủa. Phần nước lọc cho phản ứng với một lượng dư dung dịch bạc sunfat tạo thành một
kết tủa gồm hai chất rắn X2 và X3, chúng cũng được tách ra bằng cách lọc. Thêm từng giọt
dung dịch natri hiđroxit vào phần nước lọc mới thu được sau khi tách hai chất rắn trên đến
khi dung dịch có môi trường gần như trung tính (pH khoảng bằng 7). Tại thời điểm này một
chất bột màu vàng X4 (trong đó Ag chiếm 77,31 % về khối lượng) được tách ra từ dung dịch.
Khối lượng của X4 lớn hơn so với khối lượng của X2 trong phần kết tủa đầu tiên gần 2,4 lần.
a. Xác định công thức hóa học của các chất từ X1 đến X4.
b. Xác định công thức hóa học của chất khí thoát ra. Viết và cân bằng các phương trình hóa
học ở dạng ion hoặc dạng phân tử của tất cả các phản ứng xảy ra.
c. Trong một đơn vị cấu trúc của X1, cấu trúc tạo bởi các nguyên tử có tính đối xứng. Vẽ cấu
trúc của X1.
4.2. NH3 có khả năng phản ứng với nhiều ion kim loại chuyển tiếp. Alfred Werner (được giải
Nobel hóa học năm 1913) đã phân lập thành công một số phức chất giữa CoCl3 và NH3, trong
đó có phức chất bát diện với công thức phân tử là CoCl3.4NH3. Tùy thuộc vào điều kiện tổng
hợp, phức chất này có màu tím hoặc màu xanh. Khi cho lượng dư dung dịch AgNO3 tác dụng
với dung dịch chứa 1 mol phức chất này đều thu được 1 mol AgCl kết tủa. Hãy xác định các
công thức có thể có của phức chất nêu trên.

CÂU HƯỚNG DẪN CHẤM ĐIỂM


4.1 a. Xác định công thức các chất X1-X4:
(2,0 – Kết tủa X2, được tạo thành do thêm BaCl2 vào dung dịch trong môi 0,25
điểm) trường axit nên nhiều khả năng hợp lí là BaSO4.
– Kết tủa X3 được tạo thành do thêm Ag2SO4 nên nhiều khả năng là AgC1.
0,25
– Kết tủa vàng X4 được tạo thành bằng cách thêm dung dịch kiềm nên có
thể là HgO hoặc bạc photphat Ag3PO4. Tỉ lệ khối lượng X4 : X2 là 0,931
cho HgO : BaSO4 không hợp lí và 1,798 cho Ag3PO4 : BaSO4 cho ta giá
trị 2,4 khi được nhân với 4/3. Vậy X4 là Ag3PO4.
0,25
– Vì thế, tỉ lệ mol là 4Ag3PO4 : 3BaSO4 tương ứng với P : S=4 : 3, nói
cách khác công thức X1 là P4S3.
0,25
Vậy: X1 = P4S3; X2 = BaSO4; X3 = AgCl; X4 = Ag3PO4.
Khí giải phóng có khối lượng mol 1,586 x 29 = 46 (g/mol), vậy đó là NO2.
– Sự hòa tan X1: 0,75
P4S3 + 38HNO3 → 4H3PO4 + 3H2SO4 + 38NO2+ 10H2O

8
Sự tạo thành X2:
H2SO4 + BaCl2 → BaSO4 + 2HC1
– Sự tạo thành X2 và X3:
Ag2SO4 + 2HC1 → 2AgCl↓ + H2SO4
BaCl2 + Ag2SO4 → BaSO4↓ + 2AgC1↓
– Thêm NaOH và sự tạo thành X4:
HNO3 + NaOH → NaNO3 + H2O
H2SO4 + 2NaOH → Na2SO4 + 2H2O
2H3PO4 + 6NaOH + 3Ag2SO4 → 2Ag3PO4↓ + 3Na2SO4 + 6H2O
Cấu trúc P4S3:

0,25

4.2 Vì 1 mol phức CoCl3.4NH3 tác dụng dung dịch AgNO3 (dư) tạo 1 mol
AgCl → chỉ có 1 Cl- ở cầu ngoại trong phân tử phức: [Co(NH3)4Cl2]Cl. 0,125
(0,5 +
Do phức [Co(NH3)4Cl2] có cấu trúc bát diện nên có 2 đồng phân:
điểm)

0,25

Các đồng phân


0,125
Hai đồng phân này có màu sắc khác nhau (xanh và tím).

Câu 5. Đại cương hữu cơ


5.1. Hãy giải thích sự khác nhau trong hai phản ứng sau:

5.2. Khi đun (2S,3S)-2-brom-1,1,3-trimetylxiclohexan với MeONa trong MeOH thu được
hỗn hợp sản phẩm gồm 1 (C10H20O) và 2 (C9H16). Mặt khác khi đun (2S,3R)-2-brom-1,1,3-
trimetylxiclohexan với MeONa trong MeOH thu được chỉ 3 (C10H20O).
a) Viết cấu trúc của 1, 2, 3.
b) Đề nghị cơ chế phản ứng để giải thích các kết quả trên.
9
5.3. Sắp xếp các chất sau theo thứ tự tăng dần tính axit, giải thích

5.4. Nhiều hợp chất thiên nhiên chứa nhân tropon có hoạt tính sinh học phong
phú. Ví dụ hợp chất 4,5-benzotropon (A). Khử hoá (A) bằng LiAlH4 rồi
metyl hoá sản phẩm tạo thành bằng hệ NaH và MeI, thu được chất (B). Hợp
chất (B) phản ứng với MeMgI tạo thành hai đồng phân (D1) và (D2) (phân tử
khối bằng 156). Quá trình chuyển hóa này đi qua tiểu phân trung gian (C). Sau
đó, C tiếp tục phản ứng tạo thành (D1) và (D2).
Vẽ công thức cấu tạo của B, C và D1, D2. Giải thích sự hình thành C.
Hướng dẫn
CÂU ĐIỂM
5. 5.1. Ở trong hệ bicyclic việc chuyển trạng thái lai hóa ở carbon C=O từ sp2 0,25
(2,5 sang sp3 dẫn tới sự giải phóng phần nào sức căng góc ở vị trí này.
điểm)
5.2. MeO- là một tác nhân vừa có tính Nu vừa có tính bazơ nên có thể vừa tham 0,25
gia phản ứng SN2 hay E2. Trong cơ chế SN2 cấu hình tuyệt đối của nguyên tử
C sẽ bị thay đổi. Trong cơ chế E2, các nguyên tử H và Br phải nằm ở vị trí đối
trục.

0,25

5.3. Tính axit của một chất có thể xác định dựa trên độ bền của carbanion tương
ứng. Như vậy 6,6-dimethylfulvene, do sự phân cực của nó nên carbanion sinh
0,25
ra sẽ bền nhất, tức nguyên tử H của nhóm methyl thể hiện tính acid cao hơn
hẳn các hydrocarbon thơm còn lại (giải tỏa tương tự enolate).
Thứ tự độ bền carbanion (tương ứng với tính axit của hydrocarbon được xếp
theo chiều giảm dần từ trái sang phải sẽ như sau:

0,25

10
5.4. Công thức cấu tạo của B, C và D1, D2

(B) (C) (D1) (D2) 0,25x4

Cation C thỏa mãn điều kiện thơm 4.2 + 2 = 10 eπ theo Huckel. Trong quá trình
chuyển hoá B qua C, MeMgI đóng vai trò acid Lewis hỗ trợ tách nhóm MeO-
tạo thành tiểu phân C (10eπ) có tính thơm.

0,25

Câu 6. Sơ đồ tổng hợp, cơ chế.


6.1. Đề xuất cơ chế cho các phản ứng sau:

6.2. Hoàn thành sơ đồ chuyển hoá sau và vẽ công thức cấu tạo các chất.

11
Trình bày cơ chế tạo thành B1 và từ B4 đến B5

Hướng dẫn
CÂU ĐIỂM
6.1. a.
(1đ)

0,25

b.

0,25

c.

0,5

12
6.2 0,25
(1,5 đ)
b)

0,5

Cơ chế tạo thành B1

0,25
Cơ chế từ B4 đến B5

c)

0,5

13
Câu 7. Xác định cấu trúc hợp chất hữu cơ.
7.1. Khi oxi hoá các hidrocacbon X, Y, Z (đều có công thức phân tử là C5H8) bằng KMnO4
thu được các kết quả sau
Chất X tạo ra một axit 2 chức có chứa nguyên tử C bất đối.
Chất Y tạo ra một xeton 2 chức không chứa nguyên tử C bất đối.
Chất Z tạo ra một xeto-axit có chứa nguyên tử C bất đối.
Vẽ công thức cấu tạo của X, Y, Z
7.2. Hợp chất quang hoạt A có công thức phân tử C7H12O3 dễ dàng bị thủy phân trong axit
loãng cho B (C4H8O3). B tác dụng với lượng dư HIO4 thu được C (C3H6O2) và HCOOH. Đun
nóng A với HCHO trong sự có mặt của K2CO3/MeOH thì A sẽ chuyển thành D (C8H16O4)
không quang hoạt. Trong TsOH/EtOH thì D nằm cân bằng với đồng phân cấu tạo E có tính
quang hoạt. Thủy phân D trong môi trường axit loãng thu được F (C5H12O4). Khi bị oxid hóa
bằng lượng dư HIO4 thì 1 mol F sẽ tạo thành 1 mol G (C3H6O3) và 2 mol HCHO. Hãy xác
định cấu tạo các chất chưa biết.
7.3.
a) Hợp chất K (C17H18O9) là chất tự vệ của một số loài cây. Thủy phân chất K với enzym β-
glycosidase thu được chất H (C11H6O3) và D-glucose. Chất H có thể được tổng hợp theo sơ
đồ sau:

Biết rằng chất I (C6H6O2) có tính thơm, chứa 4 loại hydro trong phân tử, có phản ứng tạo
màu đặc trưng với dung dịch FeCl3 và không có phản ứng đặc trưng với phenylhydrazin.
Vẽ công thức cấu tạo của các chất từ I đến H và công thức cấu trúc của chất K.
b) Khi đun nóng trong nước thì chất K bị đồng phân hoá thành chất K’. Thủy phân chất K’
với enzym β-glycosidase thu được chất J(C11H8O4). Đun nóng chất J trong môi trường axit
thì thu được chất H.
Dùng công thức cấu tạo, vẽ sơ đồ giải thích quá trình chuyển từ chất K → K’→ J → H.
Hướng dẫn
CÂU NỘI DUNG ĐIỂM

14
7.1.

(Z)

(Y)
0,125x3

(X)
7.2 Dựa vào phân cắt chất B (C4H8O3) bằng HIO4 thu được được C (C3H6O2)
và HCOOH thấy trong B phải có nhóm -CH(OH)-CHO. Tức là B chỉ có
thể là 2,4-dihydroxybutanal và A là ketal tạo thành từ B và acetone
0,5

Như vậy các phản ứng xảy ra theo sơ đồ sau:

0,125x4

a. Do hợp chất I (C6H6O2) có tính thơm chứa 4 loại hydro trong phân tử,
có phản ứng tạo màu đặc trưng với dung dịch FeCl3 và không có phản ứng
7.3.
đặc trưng với phenylhydrazin, suy ra CTCT của I là

Từ cấu tạo của H, kết hợp với dữ kiện đầu bài, suy ra cấu trúc của K là
0,125x7

15
b. Quá trình chuyển từ K → K’→ J → H:

0,25

Câu 8.
L-hamamelose là một monosaccharide mạch nhánh. L-hamamelose có thể được tổng hợp từ
D-ribose theo sơ đồ như sau:

2) Các liên kết peptide có thể bị phân cắt chọn lọc khi có một số tác nhân đặc hiệu. Dưới đây
là các sơ đồ phân cắt liên kết peptide:

Vẽ công thức cấu tạo và đề nghị cơ chế tạo thành T1, biết T1 chứa ether 5 cạnh.

Cho biết T2 và T3 đều có cấu trúc dạng spiro. Vẽ công thức cấu tạo của T2, T3 và đề nghị
cơ chế phản ứng tạo thành T2.
Hướng dẫn
16
CÂU NỘI DUNG ĐIỂM
8.1. 1) Công thức các chất trong sơ đồ:
(1 đ)
0,125x8

8.2 2. T1
(1,5 đ)
0,25

Cơ chế tạo thành T1:

0,5

0,25

Cơ chế tạo thành T2:

0,5

17
HỘI CÁC TRƯỜNG CHUYÊN ĐÁP ÁN
VÙNG DUYÊN HẢI VÀ ĐỒNG BẰNG BẮC BỘ MÔN HÓA HỌC KHỐI 11
NĂM 2023
TRƯỜNG THPT CHUYÊN HÙNG VƯƠNG
Thời gian làm bài 180 phút
ĐỀ THI ĐỀ XUẤT
(Đáp án có 13 trang, gồm 10 câu)

Câu 1. (2,0 điểm) Tốc độ phản ứng

Phản ứng khử phức của Fe3+ ([FeL6]3+) thành phức của Fe2+ ([FeL6]2+) bằng axit ascobic (H2Asc)
xảy ra theo phản ứng: 2[FeL6]3+ + H2Asc → 2[FeL6]2+ + Asc + 2H+ (*)
Cơ chế của phản ứng được đề nghị như sau:
⎯⎯
H2Asc ⎯⎯
k1
→ H+ + HAsc- (1) và (-1)
k −1

[FeL6]3+ + HAsc- ⎯⎯ → [FeL6]2+ + HAsc•


k
2
(2)
⎯⎯→
HAsc• ⎯⎯
k3
H+ + Asc•- (3) và (-3)
k −3

[FeL6]3+ + Asc•- ⎯⎯ → [FeL6]2+ + Asc


k4
(4)
(1) và (-1) là các phản ứng nhanh so với các phản ứng còn lại.
1. Tìm biểu thức tính hằng số cân bằng K1 (của các phản ứng (1) và (-1)) theo k1 và k-1.
2. Chỉ ra các tiểu phân trung gian phản ứng trong cơ chế trên và cho biết có thể áp dụng nguyên lí nồng
độ dừng cho những tiểu phân nào? Tại sao?
3. Tốc độ phản ứng (*) được định nghĩa là tốc độ tạo thành sản phẩm Asc. Viết biểu thức định nghĩa
tốc độ của phản ứng. Tìm biểu thức của tốc độ phản ứng từ cơ chế và cho biết phản ứng có bậc động
học hay không?
4. Trong một số thí nghiệm người ta thấy tốc độ phản ứng có dạng: v = k.[FeL6 ]3+ .[HAsc] . Hãy cho
biết các điều kiện thí nghiệm để thu được biểu thức tốc độ như trên. Cho biết biểu thức của k và đơn vị
của k trong trường hợp này.

Hướng dẫn giải

Câu 1 Nội dung


(2,0
điểm) Điểm
+ −
1 Ta có: v1 = k1[H2 Asc] ; v −1 = k −1[H ][HAsc ]
Phản ứng đạt cân bằng khi: v1 = v −1  k 1[H2 Asc] = k −1[H+ ][Asc − ]
[H+ ][Asc − ] k 1
 K1 = =
[H2 Asc] k −1 (2/4 đ)
2 Các tiểu phân trung gian phản ứng: HAsc-, HAsc• và Asc•-
- Có thể áp dụng nguyên lí nồng độ dừng cho các tiểu phân HAsc• và Asc•- vì
chúng là các gốc tự do, có hoạt tính cao nên nồng độ của chúng rất nhỏ so với
các chất phản ứng và sản phẩm và gần như không đổi trong quá trình phản ứng.
- Không áp dụng được nguyên lí nồng độ dừng cho HAsc- vì (1) và (-1) là các
giai đoạn nhanh nên nó nằm cân bằng với H2Asc và do đó có nồng độ tương đối
lớn. (2/4 đ)
3 d[Asc]
- Theo định nghĩa: v =
dt
d[Asc]
- Từ cơ chế: v= = k 4 [FeL6 ]3+ [Asc•− ]
dt
(*) (2/4 đ)
1
Áp dụng nguyên lí nồng độ dừng:
d[HAsc• ]
= k 2 [FeL6 ]3+ [HAsc− ] − k 3[HAsc• ] + k −3[H + ][Asc•− ] = 0 (1)
dt
d[Asc•− ]
= k 3[HAsc• ] − k −3[H+ ][Asc•− ] − k 4[FeL6 ]3+ [Asc•− ] = 0 (2)
dt
Từ (1) và (2)  k 2[FeL6 ]3+ [HAsc − ]=k 4[FeL6 ]3+ [Asc•− ] = 0 (**)
Mặt khác: (1) và (-1) là các giai đoạn nhanh nên
k [H Asc]
 [HAsc− ] = 1 2 + (***)
k −1[H ]
k 1k 2 [FeL6 ]3+ [H2 Asc]
Thay (***) vào (**) rồi vào (*)  v =
k −1 [H+ ]
 Phản ứng không có bậc động học.
4 Để v có dạng v = k.[FeL6 ]3+ .[HAsc] thì [H+] phải không đổi trong quá trình
phản ứng. Vậy cần tiến hành trong điều kiện rất dư H+ tức là trong môi trường
axit đặc hoặc trong dung dịch đệm.
Nếu thực hiện trong điều kiện rất dư H+ sẽ kìm hãm phản ứng do làm chuyển
dịch cân bằng ((1) và (-1)) theo chiều nghịch. Còn thực hiện trong dung dịch
đệm có nhiều ưu điểm hơn.
Gọi [H+]o là nồng độ H+ trong dung dịch đệm, khi đó:
k 1k 2 k 1k 2
v= +
[FeL6 ]3+ [H2 Asc]  k =
k −1[H ]o k −1[H+ ]o
Phản ứng có bậc 2  đơn vị của k: m3.mol-1s-1 hoặc L.mol-1.s-1. (2/4 đ)

Câu 2. (2,0 điểm) Cân bằng và phản ứng trong dung dịch, pin điện, điện phân

1. Hấp thụ hoàn toàn 0,020 mol khí H2S vào nước cất, thu được 200,0 mL dung dịch A.
a) Trộn 20,0 mL dung dịch A với 20,0 mL dung dịch FeCl2 0,02 M, thu được 40,0 mL dung dịch B.
Có kết tủa xuất hiện từ dung dịch B hay không?
b) Tính giá trị pH của dung dịch B để có thể tách được ion Fe2+ hoàn toàn ra khỏi dung dịch dưới dạng
kết tủa, biết rằng ion Fe2+ được coi là tách hoàn toàn ra khỏi dung dịch khi tổng nồng độ còn lại của
sắt(II) trong dung dịch là 10–6 M.
c) Để điều chỉnh pH của dung dịch B đến khi kết tủa hoàn toàn ion Fe2+ (nồng độ còn lại của sắt(II)
trong dung dịch là 10–6 M) ta có thể dùng dung dịch đệm axetat. Tiến hành như sau: đầu tiên cho
CH3COOH đặc vào 40,0 mL dung dịch B đến nồng độ 0,10 M; sau đó cho từ từ CH3COONa vào dung
dịch thu được đến khi kết tủa hoàn toàn Fe2+ thì hết m (gam). Tính giá trị của m. Coi thể tích dung dịch
không đổi sau khi cho thêm đệm axetat.
Cho biết: pKS(FeS) = 17,2; pKa1(H2S) = 7,02; pKa2(H2S) = 12,90; pKa(CH3COOH) = 4,76;
*(FeOH ) = 10-5,92.
+

2. Ăcqui chì được nhà hoá học Pháp Louis Gaston Plante phát minh vào năm 1859. Ở trạng thái nạp
điện, điện cực bên trái của ăcqui (với thế điện cực âm hơn) tạo thành từ chì kim loại, còn điện cực bên
phải là chì (IV) oxit. Chất điện li là dung dịch H2SO4 có nồng độ phần trăm 20-30%.
a) Thiết lập sơ đồ pin đơn giản nhất ứng với acqui chì.
Cho các thế điện cực chuẩn:
EoPbSO ,SO2− /Pb = −0,359 ; EoPbO ,H+ ,SO2− /PbSO = 1,682 và E oPb2 + /Pb = −0,126 V.
4 4 2 4 4

b) Viết phương trình hoá học cho phản ứng tổng cộng xảy ra trong pin và tính suất điện động chuẩn
của pin điện hoá.
c) Tính năng lượng Gibbs chuẩn của phản ứng tổng cộng và hằng số cân bằng của phản ứng tại 25 oC.
d) Tại mặt phân cách “Pbchất điện li”, xảy ra tương tác hoá học tạo ra chì (II) sunfat. Viết phương
trình hoá học cho phản ứng xảy ra.
2
e) Tại điện cực “PbO2chất điện li”, xảy ra phản ứng hình thành chì (IV) sunfat là một chất kém bền, ít
tan và có thể phân huỷ giải phóng oxi trong sự có mặt của chì (IV) oxit, sản phẩm phụ là chì (II)
sunfat. Viết phương trình hoá học cho phản ứng hình thành chì (IV) sunfat và phản ứng giải phóng oxi.

Hướng dẫn giải

Câu 2 Nội dung


(2,0 điểm) Điểm
1.a Sau khi trộn:
C(Fe2+) = 0,01 M; C(H2S) = 0,05 M.
Nhận xét: Do Ka1(H2S) >> Ka2(H2S); C.Ka1(H2S)  C. *(FeOH+)
>> KW, do vậy có thể bỏ qua cân bằng phân li nấc 2 của H2S và cân bằng
phân li của H2O. Các cân bằng chính quyết định pH trong dung dịch là:
H2S ⇌ H+ + HS- Ka1 = 10-7,02
Fe2+ + H2O ⇌ H+ + FeOH+ * = 10-5,92
[H+] = [HS-] + [FeOH+]
 [H+] = C(H2S). Ka1/([H+] + Ka1) + C([Fe2+]). */([H+] + *)
Thay Ka1 = 10-7,02, * = 10-5,92 vào phương trình trên → [H+] =
1,292.10-4 M
 [Fe2+] = C([Fe2+]).[H+]/([H+] + *) = 9,905.10-3 M.
 [S2-] = C(H2S). Ka1 Ka2/([H+]2 + [H+]Ka1 + Ka1 Ka2) = 3,599.10-14
M;
 [Fe2+].[S2-] = 9,905.10-3. 3,599.10-14 = 10-15,45 > KS(FeS)
 Có kết tủa FeS xuất hiện. (1/4 đ)
1.b Tại thời điểm kết tủa hoàn toàn Fe2+ thì [Fe2+] + [FeOH+] = 10-6 M
*b h h
[Fe2+ ](1+ ) = 10-6 M ® [Fe2+ ] = 10-6. = 10-6.
h h +*b h +10-5,92

Mặt khác:
K a1.K a2 10-19,92
[S2- ] = (0,05- 0,01). = 0,04.
h 2 +K a1.h + K a1.K a2 h 2 +10-7,02.h +10-19,92

[Fe2+].[S2-] = Ks = 10-17,2.
10-19,92 h
Þ 0,04. -7,02
10-6. -5,92
=10-17,2
h +10
2
.h +10 -19,92
h +10
 h = 10-5,09 → pH = 5,09.
(1/4 đ)
1.c Phản ứng tạo kết tủa:
Fe2+ + H2S ⇌ 2H+ + FeS
Trước pư 0,01 0,05
Sau pư - 0,04 0,02
Gọi C là nồng độ CH3COONa thêm vào dung dịch để đến pH =
5,09.
[H+ ] = 0,02 + [HS- ] - ([CH 3COOH] - 0,10)
10-7,02 é 10-5,09 ù
Þ 10-5,09 = 0,02 + 0,04 -5,09 - ê(C + 0,1) - 0,10ú
10 + 10-7,02 êë 10-5,09 + 10-4,76 úû

 C = 0,278 M.
 m(CH3COONa) = 0,912 gam. (1/4 đ)
3
2.a Sơ đồ pin: (-) Pb, PbSO4H2SO4PbO2, Pb (+) (1/4 đ)
2.b Pb + PbO2 + 2H2SO4 → 2PbSO4 + 2H2O
Epino = Eco – Eao = 1,682 – (-0,359) = 2,041 V (1/4 đ)
2.c Gpino = -zFEpino = -2964852,041 = -393,852.103 J
− Go −393,852.103
K=e =e = 1,092.1069
pin
8,314 298
(1/4 đ)
RT

2.d Pb + H2SO4 → PbSO4 + H2 (1/4 đ)


2.e PbO2 + 2H2SO4 → Pb(SO4)2 + 2H2O
PbO2 + Pb(SO4)2 → 2PbSO4 + O2 (1/4 đ)

Câu 3. (2,0 điểm) Nhiệt động học và cân bằng hóa học

1. Cho các giá trị nhiệt động liên quan đến phản ứng trong bảng sau:
Ag2CO3 (s) Ag2O (s) CO2 (g)
−1
 H (298K)(kJ.mol ) -501,66
f
o -30,58 -393,51

S (298K)(J.mol−1.K −1 )
o 167,4 121,8 213,8
Cp (J.mol−1.K −1 ) 109,6 65,7 37,6
a) Ag2CO3 bị nhiệt phân theo phương trình sau: Ag2CO3 (s) → Ag2O (s) + CO2 (g). Tính hằng số cân
bằng K op (298 K) của phản ứng ở 298 K.
b) Làm khô Ag2CO3 (s) trong dòng khí nóng dưới áp suất 105 Pa và nhiệt độ không đổi 383 K. Tính áp
suất riêng phần tối thiểu của CO2 (g) cần có trong dòng khí để tránh sự phân huỷ Ag2CO3(s).
T2
Biết:  r Ho (T2 ) =  r Ho (T1 ) +  r Cop (T2 − T1 );  rSo (T2 ) =  rSo (T1 ) +  r Cop ln
T1
2.
a) Quá trình phân huỷ 1 mol PCl5 (g) trong một bình kín, có thể giãn nở dựa theo phương trình:
PCl5 (g) → PCl3 (g) + Cl2 (g).
Sau khi hệ đạt cân bằng, khối lượng riêng của hỗn hợp khí là 4,80 kg.m-3 tại 105 Pa và 440 K. Giả sử
các khí đều là khí lý tưởng, tính  r G o của phản ứng ở 440 K.
b) Nếu phản ứng phân huỷ PCl5 (g) được duy trì ở áp suất 105 Pa, và có chứa khí trơ Ar với áp suất
riêng phần là 0,5.105 Pa, tính phần trăm PCl5 (g) bị phân huỷ.
Hướng dẫn giải

Câu 3 Nội dung


(2,0
điểm) Điểm
1.a

(2/4 đ)
1.b

4
Áp suất tối thiểu của CO2 cần có là 1,58 kPa. (2/4 đ)
2.a

Khối lượng mol của PCl5 laf 208,5.10-3 kg/mol. Giả sử PCl5 là khí lý tưởng.

(2/4 đ)
2.b

y2 x2 0,187 2
= 2  = 2  = 0, 073
1 − y2 1 − x2 1 − 0,187 2
y = 0,26, phần trăm phân huỷ là 26%. (2/4 đ)

Câu 4. (2,0 điểm) Hóa nguyên tố (kim loại, phi kim nhóm VA, VIA)

Một nguyên tố X, nguyên tử có ba lớp electron (K, L, M) có các giá trị năng lượng ion hoá I (tính
theo kJ/mol) như sau:
I1 I2 I3 I4 I5 I6
1012 1903 2910 4956 6278 22230
1. Viết cấu hình electron của X.
2. Xác định công thức phân tử , viết tên gọi các chất trong sơ đồ và viết phương trình hóa học thực
hiện mỗi chuyển hóa theo sơ đồ sau:

(1) (2) (4)


(A) (B) (D) (E)
(10) (3) (5)
(6) (7) (12) (14) (H)
(9) (F) (G)
(11)
(8) (13) (15)
X (C)
Biết: Trong các hợp chất A, B, C, D, E, F, G, H thì X có mức oxi hóa cao nhất. B, D, E tạo dung dịch
làm đỏ quỳ tím. Dung dịch F, G phản ứng với axit mạnh và bazơ mạnh. Các chất F, G, H khi đốt cho
ngọn lửa màu tím.

5
Hướng dẫn giải
Câu 4 Nội dung
(2,0 điểm) Điểm
1 Ta thấy có bước nhảy đột ngột về năng lượng ion hóa sau khi tất cả các e
hóa trị đã bị tách ra. Ở đây sau I5 có sự tăng đột biến, như vậy X có 5 e hóa
trị, do đó thuộc nhóm VA, X có 3 lớp e nên thuộc chu kì 3. Vậy X là
photpho (P).
Cấu hình e của X: 1s22s22p63s23p3.
(3/4 đ)
Lập luận xác định được kết quả như sau:
A B C D
Ca3(PO4)2 H3PO4 P2O5 H4P2O7
Canxi photphat Axit photphoric Photpho(V) Axit
Axit oxit điphotphoric
orthophotphoric Anhidrit
photphoric
E F G H
HPO3 KH2PO4 K2HPO4 K3PO4
Axit Kali Kali Kali
metaphotphoric đihidrophotphat hidrophotphat photphat (2/4 đ)
Các PTHH:
(1) Ca3(PO4)2 + 3H2SO4 → 3CaSO4 + 2H3PO4
0
(2) 2H3PO4 ⎯⎯ t
→ H4P2O7 + H2O
2
(3) H4P2O7 + H2O → 2H3PO4
0
(4) H4P2O7 ⎯⎯ t
→ 2HPO3 + H2O
(5) 2HPO3 + H2O → H4P2O7
(6) Ca3(PO4)2 + 3SiO2 + 5C ⎯⎯⎯ → 3CaSiO3 + 2P + 5CO
0
1200 C

(7) P + 5HNO3 (đặc) → H3PO4 + 5NO2 + H2O


0
(8) 4 P + 5 O2 ⎯⎯
t
→ 2 P2O5
(9) P2O5 + 3H2O → 2H3PO4.
(10) H3PO4 + KOH → KH2PO4 + H2O
(11) P2O5 + 2KOH + H2O → 2KH2PO4
(12) KH2PO4 + KOH → K2HPO4 + H2O
(13) P2O5 + 4KOH → 2K2HPO4 + H2O
(14) K2HPO4 + KOH → K3PO4 + H2O
(15) P2O5 + 6KOH → 2K3PO4 + 3H2O (3/4 đ)

Câu 5. (2,0 điểm) Phức chất

1. Khi hoà tan CoCl2 trong dung dịch HCl đặc, nhận thấy sự thay đổi màu dung dịch theo nhiệt độ như
sau: [ Co(H2O)] 2+ + 4Cl− [CoCl4 ]2− + 6H2O
ñun noù
ng
laø
m laïnh

Màu hồng màu xanh


a) Phổ hấp thụ của dung dịch [Co(H2O)6]2+ và [CoCl4]2- được cho trong hình dưới đây. Phổ nào (I) hay
(II) là đặc trưng cho dung dịch ở nhiệt độ cao? Giải thích.

6
b) Hợp chất/ion phức có tính thuận từ khi chứa electron độc thân và moment từ được tính
 = n(n + 2) B với n là số electron độc thân. Tính giá trị  cho ion phức [Co(H2O)6]2+.
2. Sự chuyển màu “hồng (đôi khi là màu tím)  xanh” mô tả ở trên là do sự sắp xếp lại cầu phối trí
quanh ion Co2+: bát diện (oct)  tứ diện (tetr). Ví dụ đã được đề cập đến ở phần trên chính là do cân
2+ 2+
bằng: Co ( H2O )6  Co ( H2O )4  .
oct tetr

Thông thường, các hợp chất phối trí tứ diện không thông dụng bằng các hợp chất phối trí bát diện. Tuy
nhiên trong trường hợp của Co2+, lượng các phức tứ diện và phức bát diện gần như nhau. Để hiểu được
tính chất trên, ta xem xét các phức bát diện và phức tứ diện sau:
a) [Cr(H2O)6]3+ và [Cr(H2O)4]3+
b) [Co(H2O)6]2+ và [Co(H2O)4]2+
Vẽ các giản đồ mô tả năng lượng của các obitan 3d của các kim loại trong trường phối tử bát diện và
tứ diện, chỉ rõ thông số tách các obitan d (). Với mỗi ion kể trên, điền các electron ở phân lớp d ngoài
cùng của kim loại vào các giản đồ tương ứng. Tính năng lượng bền vững hóa trường tinh thể (CFSE)
của mỗi ion. So sánh và rút ra kết luận.

Hướng dẫn giải

Câu 5 Nội dung


(2,0 điểm) Điểm
1 a) Hình (II) vì màu sắc quan sát được khi đun nóng là màu xanh thì phổ
hấp thụ tương ứng có bước sóng cực đại lớn hơn phổ hấp thụ màu phụ của
màu nhìn thấy là màu hồng.
b)

(4/4 đ)
2 (4/4 đ)

7
Giả sử ∆t = 4/9 ∆o

Giá trị |CFSE(tetrahedron) – CFSE(octahedron)| nhỏ nhất với cấu hình d7


(ví dụ như Co2+). Thuyết trường tinh thể coi liên kết là liên kết ion. Điều
này đúng với trường hợp axit cứng (tâm ion) và bazơ cứng (phối tử) theo
thuyết HSAB. Trong trường hợp Co2+ (gần với axit mềm) yếu tố cộng hóa
trị trong liên kết của ion trung tâm và phối tử có khả năng cực hóa lớn là
nhân tố làm tăng độ bền phức tứ diện.

Câu 6. (2,0 điểm) Đại cương hữu cơ

1. Có ba hợp chất: A, B và C

a) Hãy so sánh và giải thích tính axit của A và B.


b) Hãy so sánh nhiệt độ sôi và độ tan trong dung môi không phân cực của B và C. Giải thích.
c) Cho biết số đồng phân lập thể có thể có của A, B và C.
2. Cho các chất:

- Giải thích tại sao mômen lưỡng cực của D (0,7 D), nhỏ hơn của C (1,7D)

Hướng dẫn giải

Câu 6 Nội dung Điểm


8
(2,0 điểm)
1.a Tính axit được đánh giá bởi sự dễ dàng phân li proton của nhóm –OH. Khả
năng này thuận lợi khi có các hiệu ứng kéo electron (–I hoặc –C) nằm kề
nhóm –OH. Ở A vừa có hiệu ứng liên hợp (–C) và hiệu ứng cảm ứng (–I) ; ở
B chỉ có hiệu ứng (–I).
→ Tính axit của (A) > (B).
(2/4 đ)
1.b Liên kết hydro làm tăng điểm sôi. Chất C có liên kết hydro nội phân tử, B
có liên kết hydro liên phân tử
→ nhiệt độ sôi của (C) bé hơn nhiệt độ sôi của (B). (C) có độ tan trong
dung môi không phân cực lớn hơn (B). (2/4 đ)
1.c A), (B) đều có 2 tâm bất đối, hai nhóm thế có thể nằm ở 2 phía
khác nhau của vòng cyclohexene và chúng có thể tồn tại 4 đồng phân lập
thể.
(C) có 4 tâm bất đối có 16 đồng phân.

(2/4 đ)
2 - Giải thích:
C có hiệu +I của 2 nhóm CH2, do vậy vectơ momen lưỡng cực được biểu
diễn như sau:

D có hiệu ứng liên hợp của O vào hệ electron  nên electron dồn ngược lại
vào hệ liên kết C-C:

Hiệu ứng –I của O trong phân tử D không lớn bằng C do nguyên tử Csp2 có
độ âm điện lớn hơn Csp3. (2/4 đ)

9
Câu 7. (2,0 điểm) Cơ chế phản ứng hóa hữu cơ

1. Hãy trình bày cơ chế phản ứng của các chuyển hóa sau:
a)
O
1. NaNH2
2
+
2. H3O
O
O
b)
COOCH3
CH2COOCH3 1.CH3ONa
+
COOCH3 2. H3O O
2. Người ta tổng hợp tổng hợp chất A từ chất B. Biết hai chất A,B có công thức cấu tạo
COCH3

HO

(A) (B)

Viết cơ chế phản ứng tổng hợp chất A từ chất B


Hướng dẫn giải
Câu Nội dung
7
(2,0
điểm) Điểm
1
OH O O
a) H O
NaNH2 O
NaNH2 H3O
+ O

O O O- O
O O

b) CH3ONa
-OCH
3
H H
H COOCH3 COOCH3
CHCOOCH3 COOCH3
H+
C-OCH3 COOCH3
OCH3
O- O
O O
O-
(4/4 đ)
2 Cơ chế phản ứng tổng hợp chất A từ chất B

HO
H+
- H2O
B
H2O+ HO COCH3

HOH

A (4/4 đ)

10
Câu 8. (2,0 điểm) Sơ đồ tổng hợp hữu cơ

Viết công thức cấu tạo của các chất từ C đến C5 và hoàn thành sơ đồ chuyển hóa sau:

(Biết C3 không làm mất màu KMnO4 loãng)

Hướng dẫn giải


Câu 8 Nội dung
(2,0 điểm) Điểm

(8/4 đ)

Câu 9. (2,0 điểm) Xác định cấu trúc các chất hữu cơ

Hợp chất A (C17H32O4) trơ khi đun nóng với dung dịch kiềm và không giải phóng CH4 khi tác dụng
với CH3MgBr. Đun nóng A với dung dịch H2SO4 loãng, thu được hợp chất B (C5H8O2) và hợp chất C
(C6H12O). Chất B phản ứng với hidroxylamin cho hợp chất D (C5H5N). Chất D không bị oxi hóa bởi
dung dịch KMnO4. Chất B cho phản ứng với H2 (xúc tác Ni, đun nóng) tạo ra hợp chất E. Chất E phản
ứng với PBr3 tạo ra hợp chất F (C5H10Br2). Chất D bị khử bởi Na/EtOH cho hợp chất I. Chất I phản
ứng với CH3MgBr giải phóng CH4. Khi đun nóng chất F với dung dịch NH3, sau đó cô cạn được một
chất rắn. Khi nung chất rắn này ở nhiệt độ cao thu được hợp chất I. Chất C phản ứng được với
hidroxylamin và có phản ứng halofom. Khi axit hóa dung dịch của phản ứng halofom tạo ra từ C, thu
được sản phẩm giống với sản phẩm thu được từ phản ứng giữa t-BuMgBr với CO2 rồi thủy phân.

11
Viết công thức cấu tạo các chất từ A đến I và giải thích vì sao khi đun nóng chất A trong dung dịch
H2SO4 thu được chất C.
Hướng dẫn giải

Câu 9 Nội dung


(2,0 điểm) Điểm
Công thức cấu tạo của các chất

(6/4 đ)
Giải thích sự tạo thành chất C từ chất A: phản ứng chuyển vị pinancol

(2/4 đ)

Câu 10. (2,0 điểm) Hóa học các hợp chất thiên nhiên

1. Cho sơ đồ chuyển hoá sau:


CH3
O C8H16O5 C
HO O 3I
/Ag 2
CH
HIO4/H2O CH2OH
OMe C6H12O5
H H OCH3
B 2 /N
i, t o
OH OH A C O
H3C H
CH2OH

a) Viết công thức cấu trúc của A (có biểu diễn cấu hình của từng nguyên tử cacbon bất đối). A thuộc
dãy nào (D hay L) và dạng α-glicozit hay β-glicozit?

b) Viết công thức cấu tạo của B và C biết rằng B có vòng 6 cạnh không chứa nhóm cacbonyl. Giải
thích sự tạo thành B.

c) Vì sao B không chứa nhóm cacbonyl mà vẫn bị khử bởi hiđro?

2. Hợp chất thiên nhiên A có CTPT C11H17O3N, hầu như không tan trong nước hoặc trong kiềm, dễ tan
trong hung dịch HCl loãng. A có hai nguyên tử H linh động. Khi cho A phản ứng anhidrit axetic thu
được chất B trung tính có CTPT C13H19ON. Cho A phản ứng với CH3I dư, sản phẩm sinh ra phản ứng
trực tiếp với AgOH tạo hợp chất C (C14H25O4N). Nhiệt phân C thu được (CH3)3N và hợp chất D
12
(C11H14O3) trung tính. Ozon phân D thu được fomandehit và một andehit thơm E có ba nhóm metoxi
trong phân tử. Cho E tác dụng với HI thu được andehit thơm chứa ba nhóm hidroxi mà ba nhóm
hidroxi này không tạo được liên kết hidro nội phân tử bền vững. Viết công thức cấu tạo các hợp chất
A, B, C, D, E.
Hướng dẫn giải
Câu Nội dung
10
(2,0
điểm) Điểm
1.a Cấu trúc của A là
Nó thuộc dẫy D, loại β -glicozit.
HO O OMe
CH3

OH OH (2/4
đ)
1.b . Sau khi phản ứng với HIO4, một nhóm anđehit bị hiđrat hoá thành gem điol, chất này
tạo vòng với nhóm anđehit thứ hai, tiếp theo là phản ứng metyl hoá.
HO O OMe O OMe O OMe O OMe
CH3 HIO4 HC CH3 HC CH3 CH3I HC CH3
H2O O Ag2O
CH HO O CH MeO O CH
OH OH HO OH (2/4
OH OMe
A B C đ)
1.c Dưới tác dụng của nhiệt, B mở vòng, bị đehiđrat hoá trở lại anđehit và do đó bị khử. (1/4
đ)
2

(3/4
đ)

13
KỲ THI CHỌN HỌC SINH GIỎI CÁC TRƯỜNG THPT CHUYÊN
KHU VỰC DUYÊN HẢI VÀ ĐỒNG BẰNG BẮC BỘ
LẦN THỨ XIV, NĂM 2023

ĐÁP ÁN MÔN: HÓA HỌC - LỚP 11

Câu 1 (2,5 điểm) Tốc độ phản ứng


Sự phân hủy H2O2 được thúc đẩy với sự có mặt của ion Fe2+. Cơ chế phản ứng được đề nghị như sau:
Fe2+ + H2O2 ⎯⎯→
k1
Fe3+ + .OH + OH- (1)
.
OH + H2O2 ⎯⎯→
k
HO2. + H2O
2
(2)
HO2. + H2O2 ⎯⎯→
k
H2O + O2 + .OH (3)
3

Fe2+ + .OH ⎯⎯→


k 4
Fe3+ + OH- (4)
a. Hãy cho biết phản ứng tổng quát trong môi trường axit. Có thể coi Fe2+ là chất xúc tác được không?
Tại sao?
b. Thiết lập biểu thức tốc độ phản ứng phân hủy H2O2. Phản ứng này có bậc động học hay không?
c. Giả sử tại thời điểm ban đầu nồng độ Fe2+, H2O2 tương ứng bằng 0,2 M và 0,15 M. Sau thời gian t,
nồng độ Fe2+ còn 0,12 M. Tính tốc độ phản ứng tại thời điểm ban đầu và tại thời điểm sau thời gian t
(ở nhiệt độ không đổi) theo hằng số tốc độ k1, k2, k3 và k4.
Câu 1 Đáp án Thang
điểm
a.
2Fe2+ + 3H2O2 ⎯ ⎯→ 2Fe3+ + O2 + 2H2O + 2OH-
Trong phản ứng tổng hợp trên vẫn xuất hiện sự tham gia của Fe2+ nên Fe2+ 0,25
không phải là chất xúc tác .
b. Áp dụng nguyên lý dừng:
d [ HO2. ]
= k 2 .[ .OH ] .[ H 2O2 ] − k3 .[ HO2. ].[ H 2O2 ] = 0  k 2 .[ .OH ] = k3 .[ HO2. ] (1)
dt 0,25
d [ .OH ]
= k1.[ Fe2+ ] .[ H 2O2 ] − k2 .[ .OH ].[ H 2O2 ] + k3 .[ HO2. ].[ H 2O2 ] − k4 .[ Fe2+ ].[ .OH ] = 0 0,25
dt
(2) 0,25
Thay (1) vào (2) ta được: k1.[H2O2] = k4.[ OH ] (3) .

Mặt khác , ta có:


d [ H 2O2 ]
- = k1.[ Fe 2+ ] .[ H 2O2 ] + k 2 .[ .OH ].[ H 2O2 ] + k3 .[ HO2. ].[ H 2O2 ] (4) 0,25
dt
d [ H 2O2 ] k
Thay (1), (3) vào (4): - = k1.[ Fe 2+ ] .[ H 2O2 ] + 2k 2 . 1 .[ H 2O2 ]2 0,25
dt k4
Mà theo PT (*):
1 d [ H 2O2 ] 1 k
vpứ = - − . = . (k1.[ Fe2+ ] .[ H 2O2 ] + 2k 2 . 1 .[ H 2O2 ]2 ) 0,25
3 dt 3 k4
(Biểu thức tốc độ của phản ứng)
Vậy phản ứng không có bậc động học. 0,25

c. Tốc độ pứ tại thời điểm ban đầu:


1 k k .k 0,25
vo = . (k1.0,2 .0,15 + 2k 2 . 1 .(0,15) 2 ) = 0,01.k1 + 0,015. 1 2
3 k4 k4
Tốc độ pứ tại thời điểm t:
1 k k .k 0,25
v= . (k1.0,12 .0,03 + 2k 2 . 1 .(0,03) 2 ) = 1,2.10 −3.k1 + 6.10 −4. 1 2
3 k4 k4

Câu 2 (2,5 điểm) Cân bằng và phản ứng trong dung dịch
2.1. Hóa học xanh (Green chemistry) luôn hướng tới các quá trình sản xuất sạch hơn, giảm thiểu ô
nhiễm môi trường, tách loại, thu hồi, tái sử dụng các chất thải. Dưới đây là một ví dụ:
Để tách loại các kim loại nặng Cr(VI), Ni(II) từ nước thải mạ điện, người ta tiến hành khử Cr(VI) về
Cr(III) bằng FeSO4 trong môi trường axit, sau đó dùng kiềm để kết tủa các hydroxide Cr(OH)3,
Ni(OH)2, Fe(OH)3 tại các pH thích hợp nhằm thu hồi, tái sử dụng lại hydroxide của các kim loại này.
Giả thiết nồng độ ban đầu của các ion Cr(VI) và Ni(II) trong nước thải đều bằng 10-3M; lượng FeSO4
lấy vừa đủ để khử Cr(VI) về Cr(III) (coi thể tích dung dịch nước thải không đổi). Hãy xác định các giá
trị pH sau đây đối với từng hydroxide kim loại:
- pHbđ của dung dịch khi bắt đầu xuất hiện kết tủa hydroxide kim loại.
- pHht của dung dịch khi kết tủa hoàn toàn hydroxide kim loại. (Các hydroxide kim loại được xem như
kết tủa hoàn toàn khi nồng độ ion kim loại còn lại trong dung dịch nhỏ hơn hoặc bằng 10-6M).
Cho tích số tan Ks của Fe(OH)3, Cr(OH)3, Ni(OH)2 lần lượt bằng 10-38, 10-30, 10-15.
2.2. Giải Nobel hoá học năm 2019 được trao cho ba nhà hoá học: John B. Goodenough (người Anh),
M. Stanley Whittingham (người Anh), và Akira Yoshino (người Nhật) vì sự nghiên cứu phát triển pin
sạc Liti ion.
Sức điện động chuẩn của pin là 3,70 V. Giả định rằng các bán phản ứng ở cathode và anode như sau:
Cathode: CoO2 + Li+ + e → LiCoO2
Anode: LiC6 → 6C + Li+ + e
a) Viết phản ứng tổng cộng xảy ra trong pin và tính năng lượng Gibbs chuẩn của phản ứng [kJ mol-1].
b) Tế bào điện hoá có điện cực được tạo ra từ LiCoO2 và graphit (C). Tính khối lượng anot khi pin
được sạc đầy và khi pin đã phóng điện hoàn toàn. Biết rằng ban đầu có 10,00 gam LiCoO2 và 10,00
gam graphite (C).
c) Tính năng lượng chuyển hoá cực đại tính trên 1 đơn vị khối lượng của pin (kJ kg-1). Giả sử lượng
chất làm anode và catode được lấy đúng tỉ lệ mol của phản ứng, tổng khối lượng của các điện cực
chiếm 50,0% khối lượng của pin. Từ kết quả tính được, hãy so sánh với mật độ năng lượng khoảng
200 kJ. kg-1 của pin lead-acid.

Câu 2 Đáp án Thang điểm


2.1 - Đối với Fe3+:
K s,Fe(OH)3
Để bắt đầu kết tủa Fe(OH)3 thì COH− = 3 = 1, 4938.10−12 M
CFe3+
0,25
pHbđ = 2,17
K s,Fe(OH)3
Khi Fe(OH)3 kết tủa hoàn toàn thì COH− = 3 = 2,154.10−11 M
10−6
pHht = 3,33 0,25
- Đối với Cr3+:
K s,Cr(OH)3
Để bắt đầu kết tủa Cr(OH)3 thì COH− = 3 = 10−9 M
CCr3+

pHbđ = 5,00 0,25

K s,Cr (OH)3
Khi Cr(OH)3 kết tủa hoàn toàn thì COH− = 3
−6
= 10−8 M
10
pHht = 6,00 0,25
- Đối với Ni2+:
K s,Ni(OH)2
Để bắt đầu kết tủa Ni(OH)2 thì COH− = = 10−6 M
C Ni2+

pHbđ = 8,00 0,25

K s,Ni(OH)2
Khi Ni(OH)2 kết tủa hoàn toàn thì COH− = −6
= 3,162.10−5 M
10
pHht = 9,50 0,25
2.2. a) CoO2 + LiC6 → LiCoO2 + 6C
a 0,25
ΔrGo = –nFEo = –1 × 96485 × 3,70 = –357 kJ mol-1
b Khi đã sạc đầy: Số mol LiCoO2 = 10,00/97,87 = 0,1022 mol.
Số mol C = 10,00/12,01 = 0,8326 mol > 0,1022 mol × 6 = 0,6132 mol.
Do vậy khối lượng anot khi pin được sạc đầy:
0,25
10,00 + 0,1022 × 6,94 = 10,71 g.
Khi pin phóng điện hoàn toàn thì khối lượng anot = 10,00 g. 0,25
c. 0,25
Mật độ năng lượng của pin liti ion
357kJ/(2.169,93.10-3kg) = 1050,4 kJ kg-1

Câu 3 (2,5 điểm) Nhiệt động và cân bằng hoá học


3.1. Cây xanh sử dụng ánh sáng mặt trời để chuyển hóa nước và khí carbonic trong không khí thành
glucose (C6H12O6) và oxygen.
a. Tại sao quá trình quang hợp ở cây xanh chỉ xảy ra được trong khoảng nhiệt độ tương đối hẹp? Tính
toán để chỉ ra rằng quá trình tổng hợp glucose của cây xanh không thể thiếu ánh sáng mặt trời.
b. Tính số photon tối thiểu để cây xanh tổng hợp nên một phân tử glucose ở 25 oC nếu xem cây xanh
hấp thụ ánh sáng có bước sóng 600 nm.
c. Hàng năm, quá trình quang hợp trên trái đất tiêu tốn hết khoảng 3,4 × 1018 kJ năng lượng ánh sáng
mặt trời. Ước lượng gần đúng khối lượng CO2 (theo đơn vị tấn) trong khí quyển được cây xanh loại bỏ
hàng năm.
Cho: Bảng số liệu nhiệt động ở áp suất tiêu chuẩn 1 bar, nhiệt độ 25oC:
Chất ΔsHo (kJ/mol) So (J/mol.K)
CO2 (g) -393,5 213,2
H2O (l) -285,8 69,9
C6H12O6 (s) -1273,3 212,1
O2 (g) 205,0
3.2. Cho cân bằng sau: CO (g) + 2H2 (g) CH3OH (g);

Cho rH0 = - 90,0 kJ.mol-1 (giả thiết là không đổi trong khoảng nhiệt độ tiến hành thí nghiệm).
KP (573K) = 2,5.10-3
a) Trong 1 bình kín, ban đầu lấy CO và H2 theo tỷ lệ mol 1 : 2 tại nhiệt độ 573K. Xác định áp suất
toàn phần của hệ để hiệu suất phản ứng đạt 70%.
b) Viết phương trình biểu diễn sự phụ thuộc giữa lnKP vào T.
c) Tại 200 bar, xác định nhiệt độ mà tại đó hiệu suất phản ứng đạt 70%.

Câu 3 Hướng dẫn chấm Điểm


Quá trình quang hợp ở cây xanh là một chuỗi các phản ứng hóa sinh phức tạp
3.1. mà hầu hết đều cần đến các enzyme làm xúc tác. Các enzyme thông thường chỉ 0,25
a hoạt động được trong một khoảng nhiệt độ hẹp nên quá trình quang hợp cũng
chỉ xảy ra trong một khoảng nhiệt độ nhất định.
6CO2 + 6H2O = C6H12O6 + 6O2
∆rHopư = 2802,5 kJ/mol
∆rSopư = -256,5 J/mol.K
Do quá trình quang hợp chỉ xảy ra trong một khoảng nhiệt độ hẹp nên ta có thể
coi giá trị ΔH và ΔS không phụ thuộc vào nhiệt độ. Vì ΔH dương và ΔS âm, 0,25
ΔG = ΔH - TΔS < 0 với mọi T, phản ứng không tự diễn biến ở mọi nhiệt độ,
tức là phản ứng này muốn xảy ra thì cần có năng lượng bên ngoài cung cấp, nói
cách khác cây xanh không thể thiếu ánh sáng mặt trời!

Ở 25oC: ∆rGopư = 2878,9 kJ/mol = 4,78 × 10-18 J/ phân tử glucozơ.


Năng lượng cung cấp bởi 1 photon: E = hv/λ = 3,31 × 10-19 J/photon 0,25
b
Số photon cần cho phản ứng tạo ra 1 phân tử glucozơ: 14,4 photon. Nói chung,
vì photon là lượng tử năng lượng nên số photon tối thiểu là 15 photon.
Số mol CO2 = 7,08 × 1015 mol hay khối lượng CO2 = 3,12 × 1017 g = 3,12 × 0,25
c
1011 tấn.
3.2 Xét cân bằng:
a. CO(k) + 2H2 (k)  CH3OH(k)
Ban đầu 1 2 0 ntổng = 3
Cân bằng 1-x 2 – 2x x ntổng = 3 – 2x
Hiệu suất đạt 70% -> x = 0,7
1− x
 PCO = . PT = 0,1875PT
3 − 2x
x 0,5
PH 2 = 2 PCO = 0,375PT ; PCH 3OH = . PT = 0, 4375 PT
3 − 2x

PCH3OH 0, 4375 1
KP = 2
= 2,5.10−3 = 2
. 2
PCO .P H2 0,1875.(0,375) PT
 PT = 81,468 (bar) 0,25

b K P (T ) H pu
0
1 1 
Ta có: ln =−  − 
K P (573K ) R  T 573 
10825 0,25
 lnKP(T) = − 24,88
T
c Tại PT = 200 bar và hiệu suất 70%
0, 4375 1
 KP(T) = 2
. 2
= 4,15.10−4
0,1875.(0,375) 200 0,5
10825
 lnKP(T) = -7,79 = − 24,88 => T = 633K
T
Câu 4 (2,5 điểm) Hoá nguyên tố. Phức chất
4.1. Chromium trioxide là chất oxihóa mạnh, nó oxi hóa được nhiều chất vô cơ và hữu cơ, bốc cháy
khi tiếp xúc với P, HI và C2H5OH. Tuy nhiên, CrO3 khô có thể kết hợp với khí HCl tạo nên chromyl
chloride là hợp chất có cấu tạo và tính chất tương tự sulfuryl halide. CrO3 tan dễ dàng trong nước tạo
thành dung dịch acid dung dịch có màu vàng chứa acid chromic (H2CrO4) và dung dịch đặc có màu
từ da cam đến đỏ chứa acid polychromic (dichromic, trichromic, tetrachromic).
Khi chế hóa dung dịch dichromate của kim loại kiềm với dung dịch ether của hỗn hợp H2O2 30%
và H2SO4 loãng người ta thu được dung dịch màu xanh chứa CrO5. Peroxide này chỉ tồn tại trong
dung dịch ether không tách ra được ở dạng tự do. CrO5 kém bền, phân hủy dễ dàng giải phóng
oxygen khi tác dụng với các dung dịch acid (H2SO4) hoặc kiềm( KOH).
Khi chế hóa cẩn thận CrO5 trong ether với H2O2 và dung dịch KOH người ta thu được tinh thể
hydrate màu xanh K2Cr2O12.2H2O. Những peroxychromate đó được coi là muối của acid H2Cr2O12
không tách ra được ở dạng tự do. K2Cr2O12 không bền phân hủy giải phóng oxygen trong dung dịch
nước, dung dịch kiềm và dung dịch acid.
Viết CTCT của CrO5, H2Cr2O7, H2Cr2O12.
4.2. Viết các đồng phân lập thể (có thể có) của các phức sau: [Pt(NH3)(NO2)py(NH2OH)]+(phức
vuông phẳng), [Co(NH3)2(H2O)2(NO2)2]+ , [Co(En)2ClNH3]2+ .

Câu Đáp án Thang


4 điểm
4.1.

0,25*3

4.2

0,50
0,75

0,50

Câu 5 (2,5 điểm) Đại cương hữu cơ


5.1. α-Arbutin là một thành phần trắng da vô cùng quen thuộc có mặt trong các loại mỹ phẩm có chức
năng làm sáng và nâng tone da như kem dưỡng da, chống nắng, đặc trị nám, thâm, tàn nhang.Nó ức
chế enzyme để sản sinh ra melanin trong tế bào. Trong tự nhiên, Alpha-Arbutin được tìm thấy ở cây
bearberry(cây dâu gấu), mầm lúa mì, lá và quả việt quất, nam việt quất,…

a) Dự đoán tính tan trong nước của α-arbutin. Giải thích.


b) Trong α-arbutin, tính acid của nhóm –OH nào mạnh nhất? Giải thích.
Đáp án:
Câu Nội dung Điểm
5.1. a α-Arbutin có thể tan trong nước 0,25
Giải thích: do α-arbutin phân tử phân cực, có các nhóm –OH có khả năng tạo 0,25
liên kết hydrogen với các phân tử nước 0,25
5.1. b Nhóm –OH gắn với vòng benzen có tính acid mạnh nhất 0,25
Giải thích: do hiệu ứng liên hợp làm bền dạng bazơ liên hợp. 0,25
0,25
5.2. Cho các ancol: p-CH3-C6H4-CH2OH, p-CH3O-C6H4-CH2OH, p-CN-C6H4-CH2OH và p-Cl-
C6H4-CH2OH. So sánh khả năng phản ứng của các ancol với HBr và giải thích.
Câu Nội dung Điểm
5.2. Phản ứng giữa các ancol đã cho với HBr là phản ứng thế theo cơ chế SN1. 0,25
Giai đoạn trung gian tạo cacbocation benzylic.
Nhóm –OCH3 đẩy electron (+C) làm bền hoá cacbocation này nên khả năng 0,25
phản ứng tăng. Nhóm CH3 có (+I) nên cũng làm bền hóa cacbocation này
nhưng kém hơn nhóm –OCH3 vì (+C) > (+I).
Các nhóm –Cl (-I > +C) và –CN (-C) hút electron làm cacbocation trở nên
kém bền do vậy khả năng phản ứng giảm, nhóm –CN hút electron mạnh hơn
nhóm –Cl. (0,25)
0,25
Vậy sắp xếp theo trật tự tăng dần khả năng phản ứng với HBr là:
p-CN-C6H4-CH2OH< p-Cl-C6H4-CH2OH < p-CH3-C6H4-CH2OH< p-CH3O-
C6H4-CH2OH.
0,25

Câu 6 (2,5 điểm). Sơ đồ tổng hợp hữu cơ. Cơ chế phản ứng hóa hữu cơ
6.1. Cho phản ứng sau:

a) Xác định cơ chế của phản ứng.


b) Viết cơ chế của phản ứng và xác định cấu trúc của sản phẩm.
Câu Nội dung Điểm
6.1 a) Cơ chế phản ứng: E2 0,25
b) Viết cơ chế phản ứng
0,25*3

6.2. Hãy xác định cấu trúc các chất M, N, L, O, P, Q.

Câu Nội dung Điểm


6.2 0,25*6

Câu 7:
Hợp chất X (C19H22O6) không tác dụng được với 2,4-đinitrophenylhiđrazin; không có phản ứng
iođofom. Khi tác dụng với HI dư, X cho 4 mol CH3I và chất Y (C15H14O5I2). Oxi hóa Y bằng dung
dịch KMnO4 đun nóng thu được axit 2,4,6-trihiđroxibenzoic, khí CO2 và axit 2,4-đihiđroxibenzoic có
số mol mỗi chất bằng số mol Y đã dùng. Khi cho Y tác dụng với dung dịch NaOH loãng sau đó cho
sản phẩm tác dụng với HIO4 thu được hợp chất Y1 và axit 2,4-đihiđroxibenzoic. Xác định cấu tạo của
X, Y, Y1.
Câu Ý Nội dung Điểm
X có ∆ = 9
X không phản ứng với 2,4-đinitrophenylhiđrazin nên X không có nhóm
cacbonyl
X → Y → (HO)3C6H2COOH + CO2 + (HO)2C6H3COOH → trong X có
2 vòng bz
Mà X có ∆ = 9 → có 1 liên kết đôi mạch ngoài
X + HI → 4CH3I → trong X có 4 nhóm OCH3
X + HI → Y mất đi 1 oxi chứng tỏ trong X có 1 nhóm OH ancol
Y + NaOH → để thủy phân halogen → điancol rồi oxi hóa tiếp bằng
HIO4 thu được 2,4-đihidroxibenzoic
Vậy điancol đó có dạng R-CH(OH)-CH(OH)-C6H3(OH)2
Vậy Y có dạng R-CH(I)-CH(I)-C6H3(OH)2
Vậy X chỉ có thể là
OCH3
OH
OCH3 OH OCH3 OCH3
H3CO O

H3CO O OCH3 hoặc OCH3

OH I OH

I
HO OH OH
Y là
OH

COOH

HO OH
Y1 là

Câu 8:
8.1. Camptothecin và các dẫn xuất của nó hoặc tương tự là những chất chống ung thư quan trọng. Cấu
trúc của Camptothecin như sau
O
N
N
(E)
O

O
OH

a/ Xác định loại nhóm chức, cấu hình tuyệt đối của C bất đối trong Camptothecin.
b/ So sánh tính bazơ của các nguyên tử N trong Camptothecin và giải thích?
8.2. Verapamil có tác dụng làm hạ huyết áp, chống loạn nhịp tim, tăng lưu lượng máu trên động mạch
vành. Verapamil có công thức cấu tạo như sau:
NC
N
(H3C)2HC
H3C

H3CO OCH3
OCH3 OCH3

CN
H3CO

Hãy viết sơ đồ điều chế Verapamil từ H3CO và các chất hữu cơ khác.

Câu Ý Nội dung Điểm


Các loại nhóm chức gồm: amin, xeton, ancol, este, amide.
a
Cấu hình tuyệt đối của C bất đối trong Camptothecin là R.
Tính bazơ
(1) O
N
N
(E)
(2) O
b
O
OH

VIII Thứ tự tính bazơ N(1) < N(2). Nguyên nhân là do cặp e trên nguyên tử N(1) tham
gia vào hệ liên hợp và bị hút e mạnh bới nhóm xeton, trong khi đó trên N(2) cặp e
không tham gia vào hệ liên hợp nên N(2) có tính bazơ mạnh hơn.
Sơ đồ điều chế Verapamil :

CH(CH3)2 CH(CH3)2
2 NC NC CH(CH3)2 NC OH NC Cl
EtONa Cl(CH2)3OH SOCl2
(CH3)2CHBr
OCH3
(A)
OCH3 OCH3 OCH3
OCH3 OCH3 OCH3 OCH3
CN CH3
NH2 1.C6H5CHO N
Pd/H2 2.CH3I H
3.H2O/H+ (B)
OCH3 OCH3 OCH3
OCH3 OCH3 OCH3

NC
N
(H3C)2HC
CH3
NaNH2
(A) + (B)
H3CO OCH3
OCH3 OCH3
HỘI CÁC TRƯỜNG THPT CHUYÊN ĐỀ THI CHỌN HỌC SINH GIỎI
KHU VỰC DUYÊN HẢI, ĐỒNG BẰNG BẮC BỘ MÔN THI: HÓA HỌC – KHỐI 11
TRƯỜNG THPT CHUYÊN BIÊN HÒA Ngày thi: 15-07-2023
TỈNH HÀ NAM Thời gian làm bài: 180 phút
(Đề này có 8 câu, gồm 04 trang)
Câu I (2,5 điểm). .
1) Khi cho Anilin tác dụng với dung dịch HNO2, HCl ở nhiệt độ thấp (-5oC), người ta thu được
muối benzene diazonium chloride được sử dụng nhiều trong tổng hợp hữu cơ. Muối này kém bền,
dễ bị phân hủy theo phản ứng:
C6H5N2Cl(aq) → C6H5Cl(l) + N2(g)
Thể tích khí N2 thu được từ 40,00 mL dung dịch C6H5N2Cl ở 50oC và 1 atm theo thời gian
được cho trong bảng sau:
Thời gian, s 6 9 14 22 30 
V(N2), mL 19,3 26,0 36,0 45,0 50,4 58,3
a) Xác định biểu thức động học dạng tích phân, dạng vi phân và giá trị hằng số tốc độ phản
ứng phân hủy muối benzene diazonium chloride.
b) Tính thể tích khí N2 thoát ra trong điều kiện thí nghiệm ở thời điểm 40 giây sau khi phản
ứng bắt đầu.
2) Phản ứng: 2Fe2+ + Tl3+ → 2Fe3+ + Tl+.
k[Fe2+ ]2 [Tl 3+ ]
có phương trình luật tốc độ phản ứng dạng: r = . Hãy dự đoán cơ chế của phản ứng.
[Fe2+ ] + k '[Fe3+ ]
Câu II. (2,5 điểm). Cân bằng và phản ứng trong dung dịch – điện phân
Dung dịch X gồm K2Cr2O7 0,010M; KMnO4 0,010M; Fe2(SO4)3 0,0050M và H2SO4 (pH của
dung dịch bằng 0). Thêm dung dịch KI vào dung dịch X cho đến nồng độ của KI là 0,50M, được
dung dịch Y (coi thể tích không thay đổi khi thêm KI vào dung dịch X).
1. Hãy mô tả các quá trình xảy ra và cho biết thành phần của dung dịch Y.
2. Tính thế của điện cực platin nhúng trong dung dịch Y.
3. Cho biết khả năng phản ứng của Cu2+ với I- (dư) ở điều kiện tiêu chuẩn. Giải thích.
4. Viết sơ đồ pin được ghép bởi điện cực platin nhúng trong dung dịch Y và điện cực platin nhúng
trong dung dịch gồm Cu2+, I- (cùng nồng độ 1 M) và chất rắn CuI. Viết phương trình hoá học của
các phản ứng xảy ra trên từng điện cực và xảy ra trong pin khi pin hoạt động.
0 0 0 0
Cho: E 2− = 1,330 V; E − = 1,510 V; E = 0,771 V; E − − = 0,5355 V
Cr2O7 /Cr 3+ MnO 4 /Mn
2+ 3+
Fe /Fe
2+
I3 /I

0 RT
E
Cu
2+
/Cu
+ = 0,153 V; pK s(CuI) = 12; ở 25 oC: 2,303 = 0,0592; Cr (z = 24).
F
Câu III (2,5 điểm).
1. Một động cơ nhiệt sử dụng 1 mol khí lí tưởng lưỡng nguyên
tử làm vật sinh công hoạt động theo chu trình thuận nghịch P
gồm 4 quá trình như hình vẽ. Trong đó AB là quá trình đẳng A
áp, BC là quá trình đoạn nhiệt, CD là quá trình đẳng nhiệt và B
P (atm)

DA là quá trình đẳng tích. Biết rằng hệ ban đầu ở trạng thái A
có áp suất 2 atm và nhiệt độ 600 K, tại trạng thái C hệ có nhiệt D
độ 500 K và thể tích gấp đôi thể tích ở trạng thái A. C
a) Tính biến thiên nội năng (theo kJ) kèm theo quá trình BC.
b) Xác định hiệu suất của động cơ thực hiện theo chu trình trên. V
2. Nitơ và oxi tạo được hợp chất khí không màu, không mùi NO.
Xác định năng lượng liên kết trong cation NO+ (kJ/mol) biết các dữ kiện sau:
- Sinh nhiệt của khí NO: f HoNO = 90, 25 kJ/ mol .
- Năng lượng liên kết trong NO, O2, N2 lần lượt là 629,8; 498,7 và 941,4 kJ/mol.
1
- Năng lượng ion hóa với các tiểu phân NO, O và N lần lượt là 893,9; 1313,9 và 1402,3 kJ/mol.
Câu IV (2,5 điểm). Kim loại Ni (M = 58,69) kết tinh ở mạng tinh thể lập phương tâm diện và có
khối lượng riêng 8,91 g/cm3.
1. Vẽ hình một ô mạng cơ sở. Có bao nhiêu nguyên tử kim loại Ni trong ô mạng cơ sở?
2. Tính độ dài cạnh ô mạng.
3. Góc nhiễu xạ cực đại bậc hai thu được khi chiếu tia X đến mặt phẳng [111] trong ô mạng cơ sở
của Ni là 20,38o. Xác định năng lượng tia X chiếu tới (eV).
Câu V (2,5 điểm). Đại cương
1.1. Sắp xếp các chất sau theo thứ tự giảm dần lực bazơ và giải thích.

1.2. Cho các chất sau đây:

So sánh lực axit giữa (4) và (5). Giải thích.


1.3. So sánh và giải thích nhiệt độ sôi của các hợp chất sau:

1.4. So sánh và giải thích nhiệt độ nóng chảy của các hợp chất (9), (10), (11), (12).

Câu VI: (2,5 điểm)


VI.1. Đề xuất cơ chế phản ứng sau:
1)

2)

VI.2. (1,5 điểm) Sơ đồ tổng hợp hữu cơ.

2
Dưới đây là một phần trong tổng hợp toàn phần của một số terpenoid, được một nhóm các nhà khoa
học Thuỵ Sĩ tiến hành vào năm 2006:

a) Xác định các chất A-Q, biết rằng Hantzsch Ester ở giai đoạn thứ ba khử sản phẩm ngưng tự
Knoevenagel.
b) Ở giai đoạn tạo thành chất O, một tác nhân khử khá đặc hiệu đã được sử dụng. Tại sao trong
trường hợp này không thể sử dụng các tác nhân khử borohydride cổ điển như NaBH4,
NaBH3CN, NaBH(OAc)3?
c) Đề xuất cơ chế chuyển D thành E.
Câu VII: ( 2,5 điểm) Xác định cấu trúc các chất hữu cơ (mô tả sơ đồ tổng hợp bằng lời)
7.1. Hợp chất F1 có công thức phân tử C10H18. F1 phản ứng với hidro có Pt làm xúc tác cho
hợp chất F2 có công thức phân tử C10H20. Trong quang phổ NMR, cả hai hợp chất F1 và F2 đều
không có hidro vinyl. F1 tác dụng với ozon sau đó cho tác dụng với bột Zn và H2O cho 2 hợp chất
F3 (C3H6O) và F4 (C7H12O). Cả hai hợp chất F3, F4 cũng không phản ứng với H2 có xúc tác nickel,
F3 có phản ứng iodofom, F4 không có phản ứng iodofom. Cả hai hợp chất (F4, F3) trong quang
phổ IR có dải hấp thu gần 1740 cm-1. Cả hai phản ứng với 2,4–dinitrophenylhyđrazon một cách
nhanh chóng. Xử lí hợp chất F3 với natriamit và CH3I dư cho hợp chất F6 (C9H18O). Dưới cùng
điều kiện hợp chất F4 chuyển hoá thành chất mới F5 có công thức phân tử C9H16O. Tìm cấu trúc từ
F1 đến F6.
7.2. Brom hóa 3-metylbutan-2-on cho hai sản phẩm đồng phân F8 và F9 (C5H9BrO) với tỉ lệ
số mol tương ứng là 95:5. Phổ 1H-MNR của F8 có các tín hiệu tại δ = 1,2 ppm (doublet, 6H); δ =
3,0 ppm (septep, 1H) và δ = 4,1ppm (singlet, 2H). Phổ 1H-MNR của F9 có hai pic đơn (singlet) tại
δ =1,9 ppm và δ = 2,5ppm với tỉ lệ điện tích tương ứng là 1:2. Lập luận để xác định công thức cấu
tạo hợp lí của hai hợp chất F8 và F9.
Câu VIII: (2,5 điểm) Hoá học các hợp chất thiên nhiên (Cacbohidrat và hợp chất hữu cơ chứa Nitơ
đơn giản).
1. Hợp chất D được dùng làm thuốc giảm đau và được tổng hợp theo sơ đồ sau:

Vẽ công thức cấu tạo của các chất từ A đến C. Cho biết PBr3 là tác nhân khử.
2. Hợp chất F có tác dụng ức chế sự phát triển của tế bào ung thư và được tổng hợp theo sơ
đồ dưới đây.

3
Vẽ công thức cấu tạo của các chất từ A đến E.
3. Vẽ công thức cấu tạo của các hợp chất từ A đến C trong sơ đồ tổng hợp thuốc zenarestat
(C) sau đây:

4. Hoàn thành sơ đồ tổng hợp Riboflavin (D) là thành phần cofactor trong các phân tử sinh
học sau đây:

--------Hết--------

4
HỘI CÁC TRƯỜNG THPT CHUYÊN ĐỀ THI CHỌN HỌC SINH GIỎI
KHU VỰC DUYÊN HẢI, MÔN THI: HÓA HỌC – KHỐI 11
ĐỒNG BẰNG BẮC BỘ Ngày thi: 15-07-2023
TRƯỜNG THPT CHUYÊN BIÊN HÒA, Thời gian làm bài: 180 phút
TỈNH HÀ NAM (Đề này có 8 câu, gồm 05 trang)
Câu I (2,5 điểm). Tốc độ phản ứng.
1) Khi cho Anilin tác dụng với dung dịch HNO2, HCl ở nhiệt độ thấp (-5oC), người ta thu được
muối benzene diazonium chloride được sử dụng nhiều trong tổng hợp hữu cơ. Muối này kém bền,
dễ bị phân hủy theo phản ứng:
C6H5N2Cl(aq) → C6H5Cl(l) + N2(g)
Thể tích khí N2 thu được từ 40,00 mL dung dịch C6H5N2Cl ở 50oC và 1 atm theo thời gian
được cho trong bảng sau:
Thời gian, s 6 9 14 22 30 
V(N2), mL 19,3 26,0 36,0 45,0 50,4 58,3
a) Xác định biểu thức động học dạng tích phân, dạng vi phân và giá trị hằng số tốc độ phản
ứng phân hủy muối benzene diazonium chloride.
b) Tính thể tích khí N2 thoát ra trong điều kiện thí nghiệm ở thời điểm 40 giây sau khi phản
ứng bắt đầu.
2) Phản ứng: 2Fe2+ + Tl3+ → 2Fe3+ + Tl+.
k[Fe2+ ]2 [Tl 3+ ]
có phương trình luật tốc độ phản ứng dạng: r = . Hãy dự đoán cơ chế của phản ứng.
[Fe2+ ] + k '[Fe3+ ]
Ý Nội dung Điểm
1 a) + Tính nồng độ ban đầu của C6H5N2Cl: Co = 0,055M
+ Nhận thấy, nồng độ ban đầu của C6H5N2Cl tỉ lệ với thể tích N2 thoát ra tại thời gian
vô cùng, nồng độ của C6H5N2Cl tỉ lệ với hiệu thể tích khí N2 thoát ra tại vô cùng và tại
thời điểm t.
+ Giả sử pư là bậc 1, ta có:
1 C0(C6 H5 N2Cl ) 1 V
k = ln = ln
t Ct (C6 H5 N2Cl ) t V − Vt
+ Thay các giá trị trong bảng, ta có
Thời gian, s 6 9 14 22 30 
V(N2), mL 19,3 26,0 36,0 45,0 50,4 58,3 0,5
k1 = 0,0670; k2 =0,0656 ; k3 = 0,0686; k4 = 0,0672 ; k5 = 0,0666 0,25
+ Các giá trị k xấp xỉ nhau nên giả sử đúng → Vậy pư là bậc 1, có
k1 + k2 + k3 + k4 + k5
k pu = = 0,0670 (s-1)
5 0,25
1 C0(C6 H5 N2Cl )
+ Phương trình dạng tích phân: k = ln 0,25
t Ct (C6 H5 N2Cl )
d[C6 H5 N2Cl ]
+ Phương trình dạng vi phân: v pu = − = k .[C6 H 5 N 2Cl ] 0,25
dt
b) Tại 40 giây kể từ khi bắt đầu phản ứng
1 V
k = ln → V40s = 54,3 (mL)
t V − Vt

5
2 Có thể có 2 trường hợp giới hạn:
k[Fe2+ ]2 [Tl 3+ ]
TH (1): [Fe ]<< k’[Fe ] → r =
2+ 3+
k '[Fe3+ ]
TH (2): [Fe2+]>> k’[Fe3+] → r = k[Fe2+ ][Tl3+ ]
Trong trường hợp (1), thành phần tổng cộng các chất có mặt trong giai đoạn sơ cấp
trung gian là (FeTl)4+. Trong trường hợp (2):(FeTl)5+ . Cơ chế có thể là:
k1 0,5
Fe2+ + Tl3+ k−1
Fe3+ + Tl2+
Fe + Tl ⎯⎯ k2
→ Fe3+ + Tl+
2+ 2+

Theo cơ chế: r = k2[Fe2+ ][Tl2+] (1)


d [Tl2+ ]
= k1[Fe2+ ][Tl3+ ]-k −1[Fe3+ ][Tl 2+ ]-k 2 [Fe 2+ ][Tl 2+ ] =0 (2)
dt
k1[Fe2+ ][Tl3+ ]
→[Tl2+] = (3)
k−1[Fe3+ ]+k 2 [Fe2+ ]
Thay (3) vào (1):
k1[Fe 2+ ]2 [Tl3+ ] 0,5
r=
k−1
[Fe3+ ]+[Fe 2+ ]
k2
Cơ chế được đề nghị là có khả năng.

Câu 2 (2,5 điểm). Cân bằng và phản ứng trong dung dịch – điện phân
Dung dịch X gồm K2Cr2O7 0,010M; KMnO4 0,010M; Fe2(SO4)3 0,0050M và H2SO4 (pH
của dung dịch bằng 0). Thêm dung dịch KI vào dung dịch X cho đến nồng độ của KI là 0,50M,
được dung dịch Y (coi thể tích không thay đổi khi thêm KI vào dung dịch X).
1. Hãy mô tả các quá trình xảy ra và cho biết thành phần của dung dịch Y.
2. Tính thế của điện cực platin nhúng trong dung dịch Y.
3. Cho biết khả năng phản ứng của Cu2+ với I- (dư) ở điều kiện tiêu chuẩn. Giải thích.
4. Viết sơ đồ pin được ghép bởi điện cực platin nhúng trong dung dịch Y và điện cực platin
nhúng trong dung dịch gồm Cu2+, I- (cùng nồng độ 1 M) và chất rắn CuI. Viết phương trình hoá
học của các phản ứng xảy ra trên từng điện cực và xảy ra trong pin khi pin hoạt động.
0 0 0 0
Cho: E 2− = 1,330 V; E − = 1,510 V; E = 0,771 V; E − − = 0,5355 V
Cr2O7 /Cr 3+ MnO 4 /Mn
2+ 3+
Fe /Fe
2+
I3 /I

0 RT
E
Cu
2+
/Cu
+ = 0,153 V; pK s(CuI) = 12; ở 25 oC: 2,303
= 0,0592; Cr (z = 24).
F
HƯỚNG DẪN CHẤM
Câu ý Nội dung Điể
m
2 a Do
0 0 0 0
E - 2+ = 1,51 V > E 2- 3+ = 1,33 V > E 3+ 2+ = 0,771V > E - - = 0,5355 V,
MnO 4 /Mn Cr2O7 /Cr Fe /Fe I3 /I

nên các quá trình xảy ra như sau:


- -
2 MnO 4 + 16 H+ + 15 I- → 2 Mn2+ + 5 I 3 + 8 H2O
0,01 0,5
- 0,425 0,01 0,025
2-
+ 14 H+ + 9 I- → 2 Cr3+ + 3 I 3 + 7 H2O
-
Cr2 O 7
0,01 0,425 0,025
6
- 0,335 0,02 0,055
-
2 Fe3+ + 3 I- → 2 Fe2+ + I 3
0,01 0,335 0,055
0,7
- 0,32 0,01 0,06
5
b Thành phần của dung dịch Y: I 3 0,060 M; I- 0,32 M; Mn2+ 0,01 M; Cr3+
-

0,02 M; Fe2+ 0,01 M.

-
I 3 + 2 e → 3 I-
0,0592 0,06
E - - = 0,5355 + .log 3
= 0,54 V.
I 3 /I
2 (0,32) 0,5

c 0 0
Do E - - = 0,5355 V > E Cu 2+ /Cu + = 0,153 V nên về nguyên tắc Cu2+ không
I3 /I

→ 2 Cu+ + I 3 hầu như


-
oxi hóa được I- và phản ứng: 2 Cu2+ + 3 I-
xảy ra theo chiều nghịch.
Nhưng nếu dư I- thì sẽ tạo kết tủa CuI. Khi đó
0 0 1
E 2+ =E 2+ + + 0,0592.log  0,863 V.
Cu /CuI Cu /Cu
K S(CuI) 0,7
5
Như vậy E Cu 2+ /CuI = 0,863 V > E - - = 0,5355 V → Cu2+ sẽ oxi hóa được
0 0
I3 /I
-
I do tạo thành CuI:
2 Cu2+ + 5 I- → 2 CuI  + I 3
-

d Vì E Cu 2+ /CuI = 0,863 V > E - - = 0,54 V → điện cực Pt nhúng trong dung


0
I /I 3

dịch Y là anot, điện cực Pt nhúng trong dung dịch gồm Cu2+, I- (cùng nồng
độ 1 M), có chứa kết tủa CuI là catot. Vậy sơ đồ pin như sau:
-
(-) Pt│ I 3 0,060 M; I- 0,32 M║CuI; Cu2+ 1 M; I- 1 M │Pt (+) 0,5
Trên catot: Cu2+ + I- + e → CuI 
-
Trên anot: 3 I- → I 3 + 2e
-
Phản ứng trong pin: 2 Cu2+ + 5 I- → 2 CuI  + I 3

Câu III (2,5 điểm).


1. Một động cơ nhiệt sử dụng 1 mol khí lí tưởng lưỡng nguyên tử làm vật sinh công hoạt động
theo chu trình thuận nghịch gồm 4 quá trình như hình vẽ. Trong đó AB là quá trình đẳng áp, BC
là quá trình đoạn nhiệt, CD là quá trình đẳng nhiệt và DA là quá trình đẳng tích. Biết rằng hệ ban
đầu ở trạng thái A có áp suất 2 atm và nhiệt độ 600 K, tại trạng thái C hệ có nhiệt độ 500 K và
thể tích gấp đôi thể tích ở trạng thái A.
P
A B
P (atm)

D
C

7
V
a) Tính biến thiên nội năng (theo kJ) kèm theo quá trình BC.
b) Xác định hiệu suất của động cơ thực hiện theo chu trình trên.
2. Nitơ và oxi tạo được hợp chất khí không màu, không mùi NO. Xác định năng lượng liên kết trong
cation NO+ (kJ/mol) biết các dữ kiện sau:
- Sinh nhiệt của khí NO: f HoNO = 90, 25 kJ/ mol .
- Năng lượng liên kết trong NO, O2, N2 lần lượt là 629,8; 498,7 và 941,4 kJ/mol.
- Năng lượng ion hóa với các tiểu phân NO, O và N lần lượt là 893,9; 1313,9 và 1402,3 kJ/mol.
Ý Nội dung trả lời Điểm
PB = 2 atm, VA = 24,6 l
nRTC 1.0, 082.500 5
PC = = = (atm)
VC 24, 6.2 6
AD pt Poission: TB .PB1− = TC .VC1− , trong đó γ = 7/5
→ TB = 642,1 K
0,5
5
U BC = 1. .8,314(500 − 642,1) = −2953,5(J)
2
1.
b) Theo đề tổng nhiệt lượng:
Q = QAB + QBC + QCD + QDA
7 1 5
= .8,314.(642,1 − 600) + 0 + 8,314.500.ln + 8,314. (600 − 500) = 422,155 J
2 2 2
Do ∆U = 0 nên A = -Q = -422,155 J
A 422,155 0,5
→ Hiệu suất  = = .100% = 12, 78%
Qthu 3303,5679
2. Do IN > IO nên Khi NO mất electron tạo NO+ sẽ mất e ở O. 0,55
Ta có sơ đồ sau:

0,5

0,5
→ 90,25 + 893,9 = 0,5.941,4 + 0,5.498,7 + 1313,9 - ENO+
→ ENO+ = 1049,8 kJ/mol
Câu IV (2,5 điểm). Kim loại Ni (M = 58,69) kết tinh ở mạng tinh thể lập phương tâm diện và có
khối lượng riêng 8,91 g/cm3.
1. Vẽ hình một ô mạng cơ sở. Có bao nhiêu nguyên tử kim loại Ni trong ô mạng cơ sở?
2. Tính độ dài cạnh ô mạng.
3. Góc nhiễu xạ cực đại bậc hai thu được khi chiếu tia X đến mặt phẳng [111] trong ô mạng cơ sở
của Ni là 20,38o. Xác định năng lượng tia X chiếu tới (eV).

8
Ý Nội dung trả lời Điểm
1. 0,5

0,25
4 nguyên tử
2. m 4.58,69 0,5
DNi = = 23 3
= 8,91  a = 3,524.10−8 (cm) = 352,4 ( pm)
V 6,02.10 a
3. AD định luật Bragg: 2 = 2d sin  = 2 a 0,25
sin 
h +k +l
2 2 2

a 352,4 0,5
 = sin  = sin 20,38 = 70,853 pm
h +k +l
2 2 2
3
Năng lượng tia X chiếu tới là :
hc 6,626.10−34.3.108 0,5
E= = = 2,805.10−15 ( J ) = 17512,65 eV
 70,853.10 −12

Câu V (2,5 điểm). Đại cương


1.1. Sắp xếp các chất sau theo thứ tự giảm dần lực bazơ và giải thích.

1.2. Cho các chất sau đây:

So sánh lực axit giữa (4) và (5). Giải thích.


1.3. So sánh và giải thích nhiệt độ sôi của các hợp chất sau:

1.4. So sánh và giải thích nhiệt độ nóng chảy của các hợp chất (9), (10), (11), (12).

9
Câu V Đáp án Điểm
1.1. - Thứ tự giảm dần lực bazơ: (2) > (1) > (3). 0,125
- Giải thích:
+ (2) có hiệu ứng +I gây ra bởi gốc hidrocacbon no (amin bậc III mạch vòng)
nên tính chất bazơ mạnh nhất. 0,125
+ Chất (1): Có hiệu ứng +I của gốc hidrocacbon no và hiệu ứng –I của vòng
benzen, nên lực bazơ của chất (1) yếu hơn lực bazơ của chất (2).
+ Chất (3): Có hiệu ứng +I của gốc hidrocacbon no và hiệu ứng –C giữa 0,125
nguyên tử N với vòng benzen, tạo thành hệ liên hợp p-π, làm mật độ electron
trên N giảm mạnh →Lực bazơ giảm mạnh. Chất (1) không có hiệu ứng – C 0,125
do cấu trúc không đồng phẳng nên lực bazơ của chất (1) mạnh hơn lực bazơ
của chất (3).
1.2. - Lực axit của (5) mạnh hơn lực axit của (4). 0,125
- Giải thích: Vì (5) có khả năng tách proton H+ tạo hệ có tính thơm.
0,25

0,125

(5)
1.3. - Thứ tự nhiệt độ sôi của các chất: to sôi (7) > to sôi (6) > to sôi (8). 0,25
- (8) không có liên kết hiđro, (6) có liên kết hiđro nội phân tử, 0,25
(7) có liên kết hiđro liên phân tử nên có nhiệt độ sôi cao nhất. 0,25
1.4. - Thứ tự nhiệt độ nóng chảy của các chất: tonc (12) > tonc (11) > tonc (10) > tonc 0, 25
(9).
- (12) ở dạng ion lưỡng cực nên có nhiệt độ nóng chảy lớn nhất. 0,25
0,125

0,125

- (10) và (11) đều có khối lượng lớn hơn và nhiều liên kết hidro liên phân
tử hơn (9) nên nhiệt độ nóng chảy của (10), (11) cao hơn (9).
- Nhóm NH2 ở (11) vừa làm tăng momen lưỡng cực, vừa tạo nhiều liên kết
hiđro liên phân tử hơn so với nhóm NO2 ở (10) vì vậy nhiệt độ nóng chảy
của (11) cao hơn của (10).
Câu VI: (2,5 điểm)
1. Đề xuất cơ chế phản ứng sau:
3)

4)

10
Câu VI Đáp án Điểm
1.

0,5

2 0.5

11
VI.2. (1,5 điểm) Sơ đồ tổng hợp hữu cơ.
Dưới đây là một phần trong tổng hợp toàn phần của một số terpenoid, được một nhóm các nhà khoa
học Thuỵ Sĩ tiến hành vào năm 2006:

d) Xác định các chất A-Q, biết rằng Hantzsch Ester ở giai đoạn thứ ba khử sản phẩm ngưng tự
Knoevenagel.
e) Ở giai đoạn tạo thành chất O, một tác nhân khử khá đặc hiệu đã được sử dụng. Tại sao trong
trường hợp này không thể sử dụng các tác nhân khử borohydride cổ điển như NaBH4,
NaBH3CN, NaBH(OAc)3?
f) Đề xuất cơ chế chuyển D thành E.
Hướng dẫn giải
Câu VI.2 Đáp án Điểm
a) 1,0

12
b) Tác nhân khử này cho phép khử chọn lọc nhóm chức aldehyde khi có mặt ketone. 0,25
NaBH4 thì sẽ khử cả hai nhóm còn cyanoborohydride và triacetocyboronhydride
thì không đủ hoạt tính.

c) 0,25

Câu VII: ( 2,5 điểm) Xác định cấu trúc các chất hữu cơ (mô tả sơ đồ tổng hợp bằng lời)
7.1. Hợp chất F1 có công thức phân tử C10H18. F1 phản ứng với hidro có Pt làm xúc tác cho
hợp chất F2 có công thức phân tử C10H20. Trong quang phổ NMR, cả hai hợp chất F1 và F2 đều
không có hidro vinyl. F1 tác dụng với ozon sau đó cho tác dụng với bột Zn và H2O cho 2 hợp chất
F3 (C3H6O) và F4 (C7H12O). Cả hai hợp chất F3, F4 cũng không phản ứng với H2 có xúc tác nickel,
F3 có phản ứng iodofom, F4 không có phản ứng iodofom. Cả hai hợp chất (F4, F3) trong quang
phổ IR có dải hấp thu gần 1740 cm-1. Cả hai phản ứng với 2,4–dinitrophenylhyđrazon một cách
nhanh chóng. Xử lí hợp chất F3 với natriamit và CH3I dư cho hợp chất F6 (C9H18O). Dưới cùng
điều kiện hợp chất F4 chuyển hoá thành chất mới F5 có công thức phân tử C9H16O. Tìm cấu trúc từ
F1 đến F6.
7.2. Brom hóa 3-metylbutan-2-on cho hai sản phẩm đồng phân F8 và F9 (C5H9BrO) với tỉ lệ
số mol tương ứng là 95:5. Phổ 1H-MNR của F8 có các tín hiệu tại δ = 1,2 ppm (doublet, 6H); δ =
3,0 ppm (septep, 1H) và δ = 4,1ppm (singlet, 2H). Phổ 1H-MNR của F9 có hai pic đơn (singlet) tại
δ =1,9 ppm và δ = 2,5ppm với tỉ lệ điện tích tương ứng là 1:2. Lập luận để xác định công thức cấu
tạo hợp lí của hai hợp chất F8 và F9.
Câu 7 ĐÁP ÁN ĐIỂM
7.1 Hợp chất F3 (C3H6O) có phản ứng iodofom vậy F3 là axeton:
0, 25

Hai hợp chất (F4, F3) trong quang phổ IR có dải hấp thu gần 1740 cm-1 =>
có nhóm xeton >C=O, vậy F4 có công thức:
0, 25

F1 tác dụng với ozon sau đó cho tác dụng với bột Zn và H2O cho 2 hợp
chất F3 và F4. Vậy F1 là: 0, 25

0,25
F1 phản ứng với hidro có Pt làm xúc tác cho hợp chất F2. Vậy F2 là:

0, 125
Xử lí hợp chất F3 với natriamit và CH3I dư cho hợp chất F6 (C9H18O).
Vậy F6 là:
13
0, 125

Vậy F5 là:

72 0,25

Câu VIII: (2,5 điểm) Hoá học các hợp chất thiên nhiên (Cacbohidrat và hợp chất hữu cơ chứa Nitơ
đơn giản).
5. Hợp chất D được dùng làm thuốc giảm đau và được tổng hợp theo sơ đồ sau:

Vẽ công thức cấu tạo của các chất từ A đến C. Cho biết PBr3 là tác nhân khử.
6. Hợp chất F có tác dụng ức chế sự phát triển của tế bào ung thư và được tổng hợp theo sơ đồ
dưới đây.

Vẽ công thức cấu tạo của các chất từ A đến E.


7. Vẽ công thức cấu tạo của các hợp chất từ A đến C trong sơ đồ tổng hợp thuốc zenarestat (C) sau
đây:

8. Hoàn thành sơ đồ tổng hợp Riboflavin (D) là thành phần cofactor trong các phân tử sinh học sau
đây:

14
Hướng dẫn giải:

Câu VIII Điểm


1 0,5

2 1,0

3 0,5

4 0,5

--------Hết--------

15
HỘI CÁC TRƯỜNG CHUYÊN HƯỚNG DẪN CHẤM
VÙNG DUYÊN HẢI VÀ ĐỒNG BẰNG BẮC BỘ ĐỀ THI CHỌN HỌC SINH GIỎI
MÔN HÓA HỌC - KHỐI 11
ĐỀ ĐỀ XUẤT Thời gian làm bài: 180 phút

Câu 1: (Tốc độ phản ứng)


1. Một trong các phản ứng pha khí được nghiên cứu kỹ là : H2(k) + I2(k) 2HI (k)
Khảo sát hằng số tốc độ của phản ứng ở các nhiệt độ khác nhau thu được kết quả sau
T(K) K1(l/mol.s) K -1 (l/mol.s)
400 8,37.10-12 3,25.10-14
500 2,48.10-7 1,95 .10-9
600 2,38.10-4 2,97.10-6
700 3,22.10-2 5,61.10-4
800 1,27 2,85.10-2
a. Cho biết phản ứng thu nhiệt hay tỏa nhiệt? Tính entanpi và entropi của phản ứng.
b. Tính độ phân ly của HI ở 600K.
2. Phản ứng clo hóa axit fomic trong pha khí xảy ra theo cơ chế dây chuyền sau:

- Sinh mạch: Cl2 2Cl.

- Phát triển mạch: Cl. + HCOOH HCl + .COOH

Cl2 + .COOH HCl + CO2 + Cl.

- Ngắt mạch: Cl. + thành bình Cl (thành bình)


Hãy chứng tỏ phương trình động học của phản ứng có dạng:

a) Ta có bảng sau:
T(K) 400 500 600 700 800
Kcb (K1/K-1) 257,54 127,18 80,13 57,40 44,56 0,25
Ta thấy giá trị Kcb giảm khi tăng nhiệt độ nên phản ứng tỏa nhiệt (ΔHo<0)
Từ phương trình Van’t Hoff ta tính được ΔHo = -11,7 kJ.mol-1.
𝛥𝐺 𝑜 = −𝑅𝑇𝑙𝑛𝐾𝑐𝑏 = −18,46 𝑘𝐽. 𝑚𝑜𝑙 −1 0,125

1
𝛥𝐻 𝑜 − 𝛥𝐺 𝑜 0,25
𝛥𝑆 𝑜 = = 16,9 𝐽. 𝐾 −1 . 𝑚𝑜𝑙 −1
𝑇

H2 + I2 → 2HI Kp = 80,13
Po 1 (atm)
Pcb 0,5α 0,5α 1-α (atm)
(1−α)2
 = 80,13
(0,5α)2 0,125
 𝛼 = 0,183
2. Áp dụng phương pháp nồng độ ổn định, ta có:
𝑑[𝐶𝑙. ] 0,25
= 2𝑣0 − 𝑣1 + 𝑣2 − 𝑣3 = 0
𝑑𝑡
𝑑[𝐶𝑂𝑂𝐻]
= 𝑣1 − 𝑣2 = 0
𝑑𝑡
Ta có: 2𝑣𝑜 = 𝑣3 0,25

 2𝑘0 [𝐶𝑙2 ] = 𝑘3 [𝐶𝑙. ]


2𝑘0
 [𝐶𝑙. ] = [𝐶𝑙2 ]
𝑘3

Ta có: 𝑣1 = 𝑣2
 𝑘1 [𝐶𝑙. ][𝐻𝐶𝑂𝑂𝐻] = 𝑘2 [𝐶𝑙2 ][𝐶𝑂𝑂𝐻]
2𝑘0 𝑘1
 [𝐶𝑙2 ][𝐻𝐶𝑂𝑂𝐻] = 𝑘2 [𝐶𝑙2 ][𝐶𝑂𝑂𝐻]
𝑘3
2𝑘0 𝑘1
 [𝐶𝑂𝑂𝐻] = [𝐻𝐶𝑂𝑂𝐻]
𝑘2 𝑘3
0,75
𝑑[𝐶𝑙2 ] 2𝑘0 𝑘1
Có − = −𝑘2 [𝐶𝑙2 ][𝐶𝑂𝑂𝐻] = − [𝐶𝑙2 ][𝐻𝐶𝑂𝑂𝐻]
𝑑𝑡 𝑘3
2𝑘0 𝑘1
= K[𝐶𝑙2 ][𝐻𝐶𝑂𝑂𝐻] 𝑣ớ𝑖 𝐾 = − 𝑘3

2
Câu 2: Cân bằng và phản ứng trong dung dịch – Pin điện
Dung dịch A gồm AgNO3 0,050 M và Pb(NO3)2 0,100 M.
1. Tính pH của dung dịch A.
2. Thêm 10,00 ml KI 0,250 M và HNO3 0,200 M vào 10,00 ml dung dịch A. Sau phản ứng người
ta nhúng một điện cực Ag vào dung dịch B vừa thu được và ghép thành pin (có cầu muối tiếp xúc
hai dung dịch) với một điện cực có Ag nhúng vào dung dịch X gåm AgNO3 0,010 M và KSCN
0,040 M.
a. Viết sơ đồ pin .
b. Tính sức điện động Epin tại 250C .
c. Viết phương trình phản ứng xảy ra khi pin hoạt động.
d. Tính hằng số cân bằng của phản ứng trên .
Cho biết: Ag+ + H2O AgOH + H+ (1) ; K1= 10 –11,70
Pb2+ + H2O PbOH+ + H+ (2) ; K2= 10 –7,80
pKs : AgI là 16,0; PbI2 là 7,86; AgSCN là 12,0 .

3. Epin sẽ thay đổi ra sao nếu:


a. Thêm một lượng nhỏ NaOH vào dung dịch B?
b. Thêm một lượng nhỏ Fe(NO3)3 vào dung dịch X?

1.

𝐴𝑔+ + 𝐻2 𝑂 ↔ 𝐴𝑔(𝑂𝐻) + 𝐻 + β = 10-11,70


*

𝑃𝑏 2+ + 𝐻2 𝑂 ↔ 𝑃𝑏(𝑂𝐻)+ + 𝐻 + *
β = 10-7,80

Do *β1 >> *β2 >> Kw


 pH tính theo CB (2)

𝑃𝑏 2+ + 𝐻2 𝑂 ↔ 𝑃𝑏(𝑂𝐻)+ + 𝐻 + *
β = 10-7,80
0,1 – x x x (M)
𝑥2 0,25
 = 10−7,8
0,1−𝑥

 x = 3,98.10-5 (M)
 pH = 4,4
𝐴𝑔+ : 0,025𝑀 ; 𝑃𝑏 2+ : 0,05𝑀
2. Điện cực A: TPBĐ: {
𝐼 − : 0,125𝑀 ; 𝐻 + 0,1𝑀
Ag+ + I- → AgI
Pb2+ + 2I- → PbI2
𝐴𝑔𝐼, 𝑃𝑏𝐼2
TPGH: {
𝑝𝐻 = 1,0

3
Tại pH = 1,0, coi nồng độ phức hidroxo là không đáng kể
Ta có kết tủa PbI2 tan là chủ yếu:
PbI2 ↔ Pb2+ + 2I- Ks = 10-7,86
y 2y (M)
 4y3 = 10-7,86
 y = 1,51.10-3 (M)
 [I-] = 3,022.10-3 (M)
10−16
0,25
 [Ag+] = [I− ]
= 3.31.10−14 (𝑀) (Hợp lí)

 𝐸𝐴𝑔+ /𝐴𝑔 = 0,799 + 0,0592 lg[𝐴𝑔+ ] = 0,001𝑉


Điện cực B: Ag+ + SCN- → AgSCN
Cân bằng hoà tan kết tủa:
AgSCN ↔ Ag+ + SCN- Ks = 10-12
Z 0,03 + z (M)
 z.(0,03 + z) = 10-12,0 0,25

 z = 3,33.10-11 (M)
 𝐸𝐴𝑔+ /𝐴𝑔 = 0,799 + 0,0592 lg[𝐴𝑔+ ] = 0,179𝑉
0,25
 Điện cực A là Anode, điện cực B là Cathode
0,125
Sơ đồ pin: 0,25
(-) Ag, AgI │H+ (0,1M) ║ SCN- (0,03M) │ Ag, AgSCN (+) 0,125
b) Epin = 0,179 - 0,001 = 0,178 V
c) 𝐴(−): 𝐴𝑔+ + 𝐼 − → 𝐴𝑔𝐼 + 1𝑒
𝐾(+): 𝐴𝑔𝑆𝐶𝑁 + 1𝑒 → 𝐴𝑔 + 𝑆𝐶𝑁 −
Phản ứng tổng: AgSCN + I- → AgI + SCN-
1016 0,375
d) 𝐾 = 1012 = 104,0
3.a)
TH1: Lượng NaOH rất ít, HNO3 dư → Epin không đổi.
TH2: Lượng NaOH vừa đủ trung hoà HNO3 → Tạo phức hidroxo →
[Pb2+] giảm → [I-] tăng → [Ag+] giảm → E(-) giảm → Epin tăng. 0,125
TH3: Lượng NaOH dư, có phản ứng:
2−
𝑃𝑏𝐼2 + 4𝑂𝐻 − → 𝑃𝑏(𝑂𝐻)4 + 2𝐼 −
 [Pb ] giảm → Epin tăng.
2+

b) Thêm Fe3+ → [SCN-] giảm → [Ag+] tăng → E(+) tăng → Epin tăng.

Câu 3. Nhiệt hoá học – Cân bằng hoá học

4
1. Để xác định phân tử CaCl có bền vững về mặt nhiệt động hay không, người ta dựa trên năng
lượng mạng lưới hoặc nhiệt hình thành ΔfHo. Dựa vào các dữ liệu sau, tính ΔfHo của CaCl theo chu
trình Born – Haber:
Nhiệt nóng chảy (ΔfusHo) của Ca: 9,3 kJ mol-1
Năng lượng ion hóa thứ nhất (I1) của Ca: 589,7 kJ mol-1
Năng lượng ion hóa thứ hai (I2) của Ca: 1145,0 kJ mol-1
Nhiệt hóa hơi (ΔvHo) của Ca: 150,0 kJ mol-1
Năng lượng phân cắt liên kết (D0) của Cl2: 240,0 kJ mol-1
Nhiệt hình thành (ΔfHo) của CaCl2: -796,0 kJ mol-1
Ái lực electron (EA) của Cl: -349,0 kJ mol-1
Năng lượng mạng lưới (ΔLHo) của CaCl: -751,9 kJ mol-1
2. Để biết liệu CaCl có tự oxi hóa – khử tạo thành Ca và CaCl2 không, người ta cần tính nhiệt của
phản ứng này (biến đổi entropy ΔS rất nhỏ, có thể bỏ qua). Sự tự oxi hoá - khử của CaCl có thể
xảy ra về mặt nhiệt động hay không? Dùng kết quả tính toán để chứng minh.
3. Cho cân bằng hóa học:

N2 (k) + 3H2 (k) → 2NH3 (k) ; = - 46 kJ.mol-1 .


Nếu xuất phát từ hỗn hợp chứa N2 và H2 theo tỉ lệ số mol đúng bằng hệ số tỉ lượng 1: 3 thì khi đạt
tới trạng thái cân bằng (450oC, 300 atm) NH3 chiếm 36% thể tích.
a) Tính hằng số cân bằng KP (ghi rõ đơn vị nếu có).
b) Giữ áp suất không đổi (300 atm), cần tiến hành ở nhiệt độ nào để khi đạt tới trạng thái cân bằng

NH3 chiếm 50% thể tích? Giả sử không thay đổi trong khoảng nhiệt độ nghiên cứu.

0,75

1
ΔfHoCaCl = 9,3 + 589,7 + 150 + 2 ∗ 240 − 349 − 751,9

= −231,9 𝑘𝐽. 𝑚𝑜𝑙 −1

5
 CaCl bền nhiệt động.
2. Do ΔSo rất nhỏ, coi ΔHo là yếu tố xét chiều phản ứng: 0,25
2CaCl → CaCl2 + Ca
ΔHo = -796,0 + 2.231,9 = -332,2 (kJ.mol-1)
Do ΔHo < 0, điều kiện đẳng nhiệt – đẳng ắp → CaCl bị phân hủy thành
CaCl2 và Ca
(Học sinh không nêu điều kiện đẳng nhiệt – đẳng áp chỉ được tối đa
0,125 điểm cho phần này)
3.a)
N2 + 3H2 → 2NH3 ΔHo = - 46 kJ/mol
no 1 3 (mol)
ncb 1-x 3-3x 2x (mol) 0,5
2𝑥
 = 0,36
4−2𝑥

 𝑥 = 0,5294 (𝑚𝑜𝑙)
2 𝛥𝑛
(𝑃𝑁𝐻3 ) 𝑃
Có 𝐾𝑝 = 3
𝑐𝑏
= 𝐾𝑛. ( 𝛴𝑛 )
𝑃𝑁2 .(𝑃𝐻2 )

(2𝑥)2 (4 − 2𝑥)2
= = 8,138.10−5
(1 − 𝑥)(3 − 3𝑥)3 . 3002
b)
2𝑥
Khi NH3 chiếm 50% thể tích, khi đó 4−2𝑥 = 0,5 → x = 0,667 mol

Khi đó Kp = 4,214.10-4
𝐾𝑝 𝛥𝐻 𝑜 1 1
𝐶ó ln 𝐾𝑝1 = − (𝑇 − 𝑇 )
2 𝑅 1 2

8,138.10−5 46.103 1 1
 ln 4,214.10−4 = . (723 − 𝑇 )
8,314 2

 T2 = 595,12K 0,5

6
Câu 4. (Hoá nguyên tố)
Nitrogen là một nguyên tố phi kim phố biến, nó tồn tại dưới dạng phân tử khí N2 chiếm đến
78% khí quyển Trái Đất. Ngoài ra, nó đóng vai trò quan trọng, có mặt trong mọi cơ thể sống. Hiện
nay, phương pháp cố định Nitrogen không khí phổ biến nhất là chu trình Haber-Bosch dựa trên
phản ứng giữa Nitrogen và Hydrogen để tạo nên Amonia.
Tuy nhiên ngày nay, người ta vẫn cố định Nitrogen thông qua muối M (Chứa 50% khối lượng Ca)
để làm trung gian cho sự tổng hợp các chất N, O, P và Q.

Từ các dữ kiện sau, xác định các chất K, L, M,…trong sơ đồ trên, viết phương trình hóa học cho
từng phản ứng:
- Các phản ứng tạo thành K, L, M, X, Q, R đều có sự tham gia của yếu tố nhiệt độ.
- Q là một trimer vòng của P và từng gây vụ bê bối lớn tại đất nước đông dân thứ hi thế giới
hiện tại.
- S là một muối trong đó cation và anion có cấu trúc hình học tương tự nhau.
Xác định các chất K,L,M,…: Xác định
K. CaO L. CaC2 M. CaCN2 các chất:
O. C4H4 P. H2N-CN N. C2H2 1,0
R. Q. S. (Mỗi chất
0,1đ x 8
Riêng chất
S được
0,2đ)
Viết PTHH:
CaCO3 → CaO + CO2 0,1đ x 10
CaO + 3C → CaC2 + CO
CaC2 + 2H2O → Ca(OH)2 + C2H2

7
2C2H2 → C4H4 (vinylacetylene)
CaC2 + N2 → CaCN2 + C
CaCN2 + 3H2O → CaCO3 + 2NH3
CaCN2 + H2O + CO2 → CaCO3 + H2N-CN

3 H2N-CN → (Melamine)

H2N-CN + NH3 →

+ HNO3 →

8
Câu 5. (Phức chất)
1. Sử dụng thuyết VB giải thích sự hình thành, từ tính và dạng hình học của các phức chất sau:
a) [Co(NH3)6]Cl2 b) K2[PtCl4]
2. Sử dụng thuyết trường tinh thể, giải thích sự hình thành, từ tính, momen từ và tính năng lượng
bền hoá tinh thể cho phức chất spin thấp [Mn(NH3)6]Cl3
3. Cho dòng khí X đi qua kim loại M thu được phức chất A1 chứa 28,57% kim loại M theo khối
lượng. Phức chất A1 bị quang phân tạo thành phức chất A2 chứa 30,77% kim loại M theo khối
lượng. Xác định A1 và A2, vẽ cấu trúc hình học của chúng.
1. 1. Mỗi chất
0,5
2. 0,5
3. Mỗi chất
a) Co2+: [Ar] 3d7
xác định
Lai hóa: 1AO4s + 3AO4p + 2AO4d → 6AOsp3d2
CTPT
0,125
Cấu tạo
0,125

Mỗi AOsp3d2 trống của Co3+ xen phủ trục với AO chứa cặp e tự do của N
tạo thành 6 liên kết 𝜎N→Co.
Lai hóa: sp3d2 – Dạng hình học: Bát diện – Thuận từ (Có e độc thân)

b) Pt2+: [Xe] 4f145d8


Do phân mức năng lượng 5d và 6s rất gần nhau nên xảy ra sự dồn e:

Lai hóa: 1AO5d + 1AO6s + 2AO6p → 4AOdsp2

9
Mỗi AOdsp2 trống của Pt2+ xen phủ trục với AO chứa cặp e tự do của Cl-
tạo thành 4 liên kết 𝜎Cl→Pt.
Lai hóa: dsp2 – Dạng hình học: Vuông phẳng – Nghịch từ (Không có e độc
thân)
2.
Mn2+: [Ar] 3d4
Phức spin thấp:

Từ tính: Thuận từ.


Momen từ: μ = √2(2 + 2) = 2,828 (𝐵𝑀)
2 8
CSFE = − 5 𝛥𝑜 ∗ 4 = − 5 𝛥𝑜

3. X: CO M: Fe A1: Fe(CO)5 A2: Fe2(CO)9


A1: A2:

10
Câu 6 (2,0 điểm): Đại cương hữu cơ
Meldrum’s acid (A) có pKa = 7.3 có tính acid cao bất thường so với các dẫn chất diester, ví dụ
như dimethyl malonate (B) (pKa=15,9), trong khi đó 5,5-dimethyl-1,3-cyclohexadione (C) và
pentane-2,4-dione (D) lại có tính acid không quá khác biệt (pKa=11.2 so với 13.43).

1. Nêu các lí do chung gây ra tính acid của các nguyên tử H trong các hợp chất trên.
2. Hãy giải thích lý do vì sao A và C lại có tính acid cao hơn so với các hợp chất mạch hở tương
ứng B và D.
3. Giải thích lí do vì sao Meldrum’s acid lại có tính acid cao bất thường.

Ý Hướng dẫn chấm Điểm


1. 2 lý do chung gây ra tính acid của các nguyên tử H kể trên: 0,25
- Độ bền của liên kết C–H: Các liên kết C–H trên bị làm kém bền do sự siêu liên hợp vào
hai nhóm C=O kề cận, cụ thể là tương tác σC-H → π*C=O.
- Độ bền của base liên hợp: Điện tích âm của các base liên hợp tương ứng được bền hóa 0,25
bởi hiệu ứng –C của hai nhóm C=O kề cận.
- Lưu ý: Không cho điểm nếu học sinh giải thích dựa vào độ phân cực của liên kết C–H.
2. Giải thích: 0,5
- Trong cấu dạng vòng 6 cạnh bền của A và C, MO σC-H (tương ứng với liên kết C–H nằm ở
vị trí trục giữa 2 nhóm C=O) có định hướng rất phù hợp để xen phủ cực đại với hai MO
π*C=O(tương ứng với 2 liên kết C=O kề cận). B và D là các hợp chất mạch hở, cấu dạng của
chúng có chuyển động quay linh hoạt hơn, do đó các tương tác σC-H → π*C=O không đạt
được hình học xen phủ tối ưu.

- Vẽ cấu dạng của A và C để minh họa cho sự xen phủ σC-H → π*C=O. Yêu cầu vẽ đúng hình 0,5
dạng của các MO, đặc biệt là π*(C=O) .

- Do đó, A và C đều có tính acid cao hơn so với các hợp chất mạch hở tương ứng là B và D.
- Học sinh có thể giải thích rằng base liên hợp của A và C thoáng lập thể hơn, do đó được
solvate

11
hóa tốt hơn B và D.

3. - Trong cấu dạng thuyền cứng nhắc, các tương tác nO → π*C=O và nO → σ*C-O đều không 0,25
thuận lợi, do đó tương tác σC-H → π*C=O trong A không bị ảnh hưởng như ở C. Học sinh có
thể vẽ hình minh họa phù hợp.

- Mặt khác, base liên hợp của A được bền hóa bởi tương tác no → σ*C-O (hiệu ứnganomeric). 0,25

12
Câu 7 (2,0 điểm): Cơ chế phản ứng hóa học hữu cơ
1.

2.

3.

4.

Ý Hướng dẫn chấm Điểm


1. 0,5

13
2. 0,5

14
3. 0,5

4. 0,5

15
Câu 8: Sơ đồ tổng hợp hữu cơ
1. Morphine là một loại thuốc phiện mạnh được tìm thấy tự nhiên trong cây thuốc phiện, một loại
nhữa màu nâu sẫm ở anh túc (Papaver somniferum). Nó chủ yếu được sử dụng như một loại
thuốc giảm đau. Nó tác động trực tiếp lên hệ thần kinh trung ương (CNS) để giảm đau và gây
thay đổi nhận thức cũng như phản ứng cảm xúc đối với cơn đau. Morphine gây nghiện và dễ bị
lạm dụng vì vậy đã có nhiều nhà nghiên cứu tổng hợp nó và sau đây là một trong số đó:

2. Tetrahydrocannabinol (THC) là thành phần hoạt động thần kinh chính của cần sa và là một
trong ít nhất 113 tổng số cannabinoid được xác định trên cây này. Giống như hầu hết các chất
chuyển hóa thứ cấp có hoạt tính dược lý của thực vật, THC là một chất béo có trong cần sa.Là
thành phần chính của cây cần sa để gây ra các hiệu ứng thần kinh. THC lần đầu tiên được phát
hiện và phân lập bởi nhà hóa học người Israel Raphael Mechoulam tại Israel vào năm 1964.
Người ta phát hiện ra rằng, khi hút thuốc, tetrahydrocannabinol được hấp thụ vào máu và di
chuyển đến não, tự gắn vào các thụ thể endocannabinoid tự nhiên nằm trong vỏ não, tiểu não và
hạch nền. Đây là những phần của não chịu trách nhiệm về suy nghĩ, trí nhớ, niềm vui, sự phối
hợp và chuyển động.
Dưới đây là 1 trong những cách tổng hợp THC người ta đã tìm thấy:

16
17
Ý Hướng dẫn chấm Điểm
1. Mỗi
chất
đúng
được
0,05
điểm.
Đúng
cả
dãy
được
thêm
0,25
điểm

18
2. Mỗi
chất
đúng
được
0,05
điểm.
Đúng
cả
dãy
được
thêm
0,25
điểm

19
Câu 9 (2,0 điểm): Xác định cấu trúc các chất hữu cơ
Khi brom với 2,3-dimethylbutane, một lượng lớn các bromua được hình thành. Hợp chất X chỉ
cho 1 tín hiệu trong dải 1H - NMR và 2 tín hiệu trong phổ 13C - NMR. Hợp chất này chứa 79,96%
brom theo khối lượng. Trong nghiên cứu phản ứng này, người ta phát hiện ra chất trung gian Y.
Chất Y đưa ra 3 tín hiệu trong phổ 13C – NMR và chứa 66,6% brom. Ngoài ra, người ta phát hiện
chất X được hình thành bằng cách brom hóa 3,3-dimethyl-2-butanol.
a. Xác định cấu trúc của các chất X và Y
b. Viết các cơ chế của các mô tả trên

Ý Hướng dẫn chấm Điểm


1. X: Y: 0,5

2. Cơ chế tạo X 0,75

0,75

20
Câu 10 (2,0 điểm): Hợp chất thiên nhiên
Amygladin (A), C20H27O11N là một disaccharide thiên nhiên không có tính khử và chỉ chứa liên
kết β-glycoside. Thủy phân A bằng dung dịch acid loãng thu được Glucose và hợp chất B
(C7H6O) làm mất màu nước brom. Khi đun A với dung dịch acid đặc thì thu được glucose và hợp
chất C (C8H8O3). Oxy hóa C bằng dung dịch KMnO4, sau đó acid hóa thì thu được D không làm
mất màu dung dịch nước brom. Cho A tác dụng với MeI dư trong môi trường kiềm, sau đó thủy
phân trong môi trường acid, rồi cho các sản phẩm tạo thành tác dụng với HIO4 dư thì thu được 1
số sản phẩm trong đó có 2,3,4,6-tetra-O-methyl-D-glucose, 2,3-dimethoxybutandial và
methoxyetanal.
Hãy xác định cấu tạo của Amygladin (A), B, C và D.

Ý Hướng dẫn chấm Điểm


1. A: B: C: D: 0,5
điểm/1
chất

21
HỘI CÁC TRƯỜNG CHUYÊN ĐÁP ÁN ĐỀ THI ĐỀ XUẤT
VÙNG DUYÊN HẢI VÀ ĐỒNG BẰNG BẮC BỘ MÔN HÓA HỌC KHỐI 11
TRƯỜNG THPT CHUYÊN BẮC NINH NĂM 2023
TỈNH BẮC NINH Thời gian làm bài 180 phút
ĐÁP ÁN ĐỀ THI ĐỀ XUẤT
Câu 1 (2,5 điểm): Tốc độ pư
1/ Hình dưới đây là đồ thị lnkobs với 1/T, trong đó kobs (được tính theo giờ-1) là hằng số tốc độ khả kiến của
phản ứng phân hủy bậc một cạnh tranh của hơi acetic acid thành methane (phản ứng 1) và thành methanol
(phản ứng 2); T là nhiệt độ Kenvin.

a/ Viết các phương trình phản ứng 1 và 2.


b/ Từ dữ kiện đồ thị, hãy tính các năng lượng hoạt hóa (Ea) và hệ số trước số mũ (A) cho cả hai phản ứng.
Cho biết rằng A1 > A2.
2/ Cho cơ chế của phản ứng: [Co(H2O)4Cl2]+(aq) + NH3(aq) → [Co(H2O)3(NH3)Cl2]+(aq) + H2O
như sau:
[Co( H 2O) 4 Cl2 ]+ ⎯⎯
k1
→[Co( H 2O)3 Cl2 ]+ + H 2O
[Co( H 2O)3 Cl2 ]+ + H 2O ⎯⎯
k−1
→[Co( H 2O) 4 Cl2 ]+
[Co( H 2O)3 Cl2 ]+ + NH 3 ⎯⎯
k2
→[Co( H 2O)3 Cl2 ( NH 3 )]+
a/ Sử dụng phương pháp nồng độ dừng cho [Co(H2O)3Cl2]+ và cơ chế phản ứng đã cho, hãy cho biết biểu
thức về tốc độ tạo thành sản phẩm [Co(H2O)3Cl2(NH3)]+.
b/ Giá trị của hằng số k1 của [Co(H2O)6]3+ lớn hay nhỏ hơn so với trường hợp của [Co(H2O)4Cl2]+? Giải
thích.
Hướng dẫn
1/ 1,5 điểm
a/ Phản ứng 1: CH3COOH → CH4 + CO2
Phản ứng 2: CH3COOH → CH3OH + CO
Ea 1
b/ Theo pt Arrhenius, ta có: ln k = ln A − . , do đó
R T
Ea
+ Độ dốc của đồ thị ứng với giá trị −
R
+ Kéo dài đồ thị đến điểm giao cắt với trục tung y ứng với giá trị ln A . Giá trị ln A càng lớn thì giá trị của
A càng lớn.
Theo đề bài, phản ứng 1 có giá trị của A1 lớn hơn phản ứng 2 là A2 nên đường ứng với độ dốc là – 17000
ứng với phản ứng 1 và đường ứng với độ dốc là -5300 ứng với phản ứng 2.
Từ điểm giao cắt giữ hai đồ thị, ta có thể tính được giá trị A1 và A2.
Ea1 1
+ ln A1 − . = ln A1 − 17000.6, 25.10−4 = 5, 4 → A1 = 9.106 (giờ-1)
R T
Ea1
+ − = −17000 → Ea1 = 141 kJ/mol
R

1
Ea 2 1
+ ln A2 − . = ln A2 − 5300.6, 25.10−4 = 5, 4 → A2 = 6.103 (giờ-1)
R T
Ea 2
+ − = −5300 → Ea 2 = 44 kJ/mol
R
2/ 1 điểm
a/ Áp dụng nguyên lí nồng độ dừng cho tiểu phân [Co(H2O)3Cl2]+, ta có:
d [Co( H 2O)3 Cl2 ]+
=0
dt
→ k1. [Co(H2O)4Cl2]+ - k-1. [Co(H2O)3Cl2]+.[H2O] – k2. [Co(H2O)3Cl2]+.[NH3] = 0
k1[Co( H 2O)4 Cl2 ]+
→ [Co( H 2O)3 Cl2 ]+ =
k−1[ H 2O] + k2 [ NH 3 ]
Tốc độ tạo thành sản phẩm [Co(H2O)3Cl2(NH3)]+ là v2, ta có
k1k2 [ NH 3 ].[Co( H 2O)4 Cl2 ]+
v2 = k2 .[Co( H 2O)3 Cl2 ]+ .[ NH 3 ] =
k−1[ H 2O] + k2 [ NH 3 ]
Nếu HS bỏ qua nồng độ của nước có lập luận, chỉ cho 1 nửa số điểm, còn không lập luận thì không cho
điểm.
b/ Tách nước ra khỏi ion 3+ thì có lực hút tĩnh điện giữa đầu âm của H2O với ion 3+. Tương tác này mạnh
hơn khi tách nước ra khỏi ion 1+. Vì vậy, với ion 3+, cần nhiều năng lượng hơn để vượt qua lực hút này. Do
đó, hằng số k1 giảm đi đáng kể.
Câu 2 (2,5 điểm): Cân bằng và pư trong dung dịch. Pin điện – Điện phân
1/ Nhiễm độc chì luôn luôn đáng lo ngại. Trong cơ thể con người, mức độ độc hại của chì có thể được giảm
bớt bằng cách sử dụng phối tử EDTA4- để tạo phức [Pb(EDTA)]2- rất bền (hằng số bền β(Pb) = 1018,0 và
được thận bài tiết. Phối tử EDTA4- được cung cấp bằng cách tiêm truyền dd Na2[Ca(EDTA)]. Biết phức
[Ca(EDTA)]2- tương đối kém bền (hằng số bền β(Ca) = 1010,7), sự trao đổi canxi với chì chủ yếu diễn ra
trong mạch máu.
a) Hàm lượng chì trong máu của một bệnh nhân là 0,828 μg/mL. Tính nồng độ chì theo μmol/L trong máu
của bệnh nhân này..
b) Trong một thí nghiệm, người ta điều chế một dung dịch hỗn hợp từ Ca(NO3)2.4H2O và Na2[Ca(EDTA)].
Trong dung dịch thu được, nồng độ của Ca(NO3)2 là 2,5 mM và của Na2[Ca(EDTA)] là 1,0 mM. Thêm
Pb(NO3)2 rắn vào để đạt được nồng độ chì tương ứng với nồng độ chì trong máu của bệnh nhân nêu trên.
Không xét tính chất axit bazơ của các tiểu phân có liên quan và thể tích dung dịch coi như không đổi, tính tỉ
lệ nồng độ [[Pb(EDTA)]2-]/[Pb2+] trong dung dịch thu được tại thời điểm cân bằng.
c) Sự giảm nồng độ của phức [Pb(EDTA)]2- trong máu thông qua quá trình bài tiết tuân theo quy luật động
học bậc nhất. Sau 2 giờ, nồng độ của phức [Pb(EDTA)]2- trong máu của hầu hết bệnh nhân giảm 60%. Tính
“chu kỳ bán hủy sinh học” của phức [Pb(EDTA)]2-.
2/ Các pin lithium có thể sạc lại là một giải pháp thay thế khác của pin nhiên liệu. Pin lithium-ion thường sử
dụng graphite cho một trong các điện cực, trong đó các cụm (cluster) lithium đan xen giữa các lớp graphite.
Một điện cực khác được làm từ LiCoO2, các bán phản ứng trong pin như sau:
(C ) n + Li + + e− → Li(C ) n E 0 = −3,05V
CoO2 + Li + + e− → LiCoO2 E 0 = +0,19V
a) Hãy viết phản ứng tổng cộng xảy ra khi pin hoạt động.
b) Giả sử rằng 1 đơn vị C6, 1 đơn vị CoO2 và 1 nguyên tử Li là đủ để tạo thành một pin hoạt động có khả
năng chuyển 1 electron giữa các điện cực. Sử dụng các giá trị suất điện động (EMF) chuẩn tương ứng, tính:
i) Dung lượng pin riêng (theo mAh g-1) của pin lithium ion trên.
ii) Mật độ năng lượng (theo kWh kg-1) đối với pin lithium ion trên.
iii) Cho biết để sử dụng pin này trong chiếc VF8 (động cơ điện 310 kW hoạt động với tỉ lệ 15 % công suất
cực đại) để đi quãng đường đường từ Hà Nội đến Hải Phòng rồi quay về trên đường 5B (300 km), với vận
tốc trung bình 100 km giờ-1, hiệu suất động cơ điện 95 %, mới tiêu tốn 60% dung lượng pin, thì khối lượng
của pin (bỏ qua vật liệu phụ gia) là bao nhiêu?
Hướng dẫn chấm
1/ 1,25 điểm
2
0,828.10−6 / 207
a/ 0,25 điểm CPb2+ = −3
= 4.10−6 (M ) = 4(  M )
10

b/ 0,75 điểm
Xét phản ứng: [Ca(EDTA)]2-(aq) + Pb2+(aq) [Pb(EDTA)]2-(aq) + Ca2+(aq) (1)
[ Pb( EDTA) 2− ] [Ca ( EDTA) 2− ] [Ca 2+ ].[ EDTA4− ]
Ta có:  ( Pb) =   −1
; (Ca ) = → (Ca ) =
[ Pb 2+ ].[ EDTA4− ] [Ca 2+ ].[ EDTA4− ] [Ca ( EDTA) 2− ]
[ Pb( EDTA) 2− ] [Ca 2+ ].[ EDTA4− ] [ Pb( EDTA) 2− ] [Ca 2+ ]
→  ( Pb). (Ca) =
−1
. = . =K
[ Pb 2+ ].[ EDTA4− ] [Ca ( EDTA) 2− ] [ Pb 2+ ] [Ca ( EDTA) 2− ]
→ K =  ( Pb). −1 (Ca) = 107,3
Do khả năng tạo phức mạnh và ion Ca2+ dư so với tổng lượng EDTA nên tất cả các phối tử coi như sẽ liên
kết hết với chì hoặc canxi trong phức chất.
Xét cân bằng:
[Ca(EDTA)]2-(aq) + Pb2+(aq) [Pb(EDTA)]2-(aq) + Ca2+(aq) (1)
Co(M): 10-3 4.10-6 2,5.10-3
[ ] (M) (10-3 – 4.10-6 + x) x (4.10-6 – x) (2,5.10-3 + 4.10-6 – x)
Từ giá trị của K và giả sử x << 4.10 , tính được x = 5.04.10 (thỏa mãn giả sử)
-6 -13

[ Pb( EDTA) 2− ] 4.10−6


Vậy, 2+
= −13
= 7,944.106
[ Pb ] 5,04.10
c/ 0,25 điểm
Áp dụng phương trình động học bậc nhất, tính được k = 0,458(giờ-1) → t1/2 = 1,51 (giờ)
2/ 1,25 điểm
a. 0,25 điểm
Li(Cn) + CoO2 → LiCoO2 + (C)n Eo = 3,24 V
b. 1,0 điểm
i/ 0,25 điểm
3, 24.1.96485
Dung lượng pin riêng = = 567,813 A.s.g-1 = 157,748 mA.h.g-1
3, 24.169,9
ii. 0,25 điểm
3, 24(V ).96485(C )
Mật độ năng lượng = = 0,5111 . 10-3kWh.g-1 = 0,5111 kWh.kg-1
169,9( g / mol ).3600( s).1000
iii. 0,5 điểm
Thời gian xe di chuyển là 3h. Công điện đã được xe sử dụng cho việc di chuyển là:
100.46,5.3.3600.1000 m
A= = 0, 6.3, 24.96485. pin ⇒ mpin = 476303,47 g = 476,303 kg
95 169,9
Vậy nếu bỏ qua các vật liệu và phụ gia khác khi tạo pin thì khối lượng pin là 476,303 kg

Câu 3 (2,5 điểm): Nhiệt động học và cân bằng hóa học
Nước là một trong những hợp chất quan trọng nhất trên Trái Đất. Nước được coi là dấu hiệu của sự
sống do tham gia vào hầu hết tất cả các quá
trình hóa lí của sự sống. Điều này là do cấu tạo
và các tính chất hóa lí đặc biệt của nước.
Tùy thuộc vào điều kiện nhiệt độ và áp
suất, một chất có thể tồn tại ở thể khí, lỏng
hoặc rắn hoặc đồng thời các thể này. Có thể
tóm tắt đặc điểm này của một chất bằng một
giản đồ gọi là giản đồ pha, là giản đồ minh hoạ
mối quan hệ giữa thể tồn tại của chất với nhiệt
độ và áp suất. Đường phân cách giữa các pha
3
(thể) xác định điều kiện để các pha liền kề đường đó cùng tồn tại cân bằng. Hình dưới đây là một ví dụ về
giản đồ pha của nước.
1. Quan sát giản đồ pha của nước và đánh dấu “×” vào ô ứng với những phát biểu đúng.
Đường A–B, A–C và A–D ứng với điều kiện tồn tại cân bằng giữa các pha.
Ở điều kiện 0oC, 760 mmHg, nước tồn tại đồng thời cả pha rắn và pha lỏng nằm cân bằng.
Ở điều kiện –0,5oC, 760 mmHg, nước tồn tại đồng thời cả pha rắn và pha hơi nằm cân bằng.
Ở điều kiện 150oC, 760 mmHg, nước tồn tại đồng thời cả pha lỏng và pha hơi
Tồn tại một điểm (về nhiệt độ áp suất) mà tại đó cả ba pha rắn, lỏng, hơi của nước nằm cân bằng.
Khi tăng áp suất, nhiệt độ nóng chảy của nước đá sẽ giảm.
Khi tăng áp suất, nhiệt độ hóa hơi của nước lỏng sẽ giảm.
2. Dựa vào cấu trúc của nước ở thể lỏng và thể rắn, trả lời các câu hỏi sau:
a) Vì sao ở cùng điều kiện nhiệt độ và áp suất khối lượng riêng của nước lỏng luôn lớn hơn khối lượng riêng
của nước rắn? Ví dụ: ở 0oC và 1 atm khối lượng riêng của nước ở thể rắn: 0,915 g cm–3; khối lượng riêng
của nước ở thể lỏng: 0,998 g cm–3.
b) Vì sao nước là một dung môi tốt cho nhiều chất? Vì sao tính chất này lại quan trọng đối với sự sống.
3. Phương trình Clausius - Clapeyron cho cân bằng của nước ở thể rắn và nước ở thể lỏng:
𝑑𝑃 ∆𝐻
H2O (r) ⇌ H2O (l) ; 𝑑𝑇 = 𝑇 𝑛𝑐 . Biết rằng ở áp suất khí quyển nước đá nóng chảy ở 0oC. Cho biết nhiệt
𝑛𝑐 𝑛𝑐∆𝑉
nóng chảy của nước đá ∆Hnc = 6,01 kJ.mol-1, không phụ thuộc vào nhiệt độ. Giả sử rằng ở điều kiện này
khối lượng riêng của nước ở thể lỏng và thể rắn lần lượt là 0,998 và 0,915 g.cm-3.
a) Hãy tính nhiệt độ nóng chảy của nước ngay dưới khối băng dày 5000 m. Nêu ý nghĩa của giá trị tính
được.
b) Trong môn trượt băng nghệ thuật sở dĩ vận động viên có thể trượt trên mặt băng và không bị ngã là do có
một lớp nước mỏng dưới lưỡi dao của giầy trượt băng. Trong một sân băng, mặt băng luôn được giữ ở nhiệt
độ - 0,5oC tại áp suất khí quyển, một vận động viên trượt băng nghệ thuật nặng 61,5 kg trượt trên một chiếc
giầy trượt băng, trong đó, lưỡi dao của giầy tiếp xúc với mặt băng dài 6,0 cm và rộng 1,0 mm. Tại phần tiếp
xúc này, mặt băng chịu thêm một áp lực gây ra bởi trọng lượng của vận động viên.
i) Lập luận kèm theo tính toán để giải thích lý do vận động viên này có thể trượt trên mặt băng và không bị
ngã.
ii) Hãy tính sự thay đổi năng lượng tự do Gibbs (J.mol–1) cho 1 mol nước lỏng; cho 1 mol băng (nước đá) do
áp lực gây ra dưới lưỡi dao của giầy trượt. Kết quả ở có phù hợp với kết quả của ý i) hay không? Giải thích.
HDC
1/ 0,5 điểm
 Đường A–B, A–C và A–D ứng với điều kiện tồn tại cân bằng giữa các pha.
 Ở điều kiện 0oC, 760 mmHg, nước tồn tại đồng thời cả pha rắn và pha lỏng nằm cân bằng.
Ở điều kiện –0,5oC, 760 mmHg, nước tồn tại đồng thời cả pha rắn và pha hơi nằm cân bằng.
Ở điều kiện 150oC, 760 mmHg, nước tồn tại đồng thời cả pha lỏng và pha hơi.
 Tồn tại một điểm (về nhiệt độ áp suất) mà tại đó cả ba pha rắn, lỏng, hơi của nước nằm cân bằng.
 Khi tăng áp suất, nhiệt độ nóng chảy của nước đá sẽ giảm.
Khi tăng áp suất, nhiệt độ hóa hơi của nước lỏng sẽ giảm.
2.a/ 0,25 điểm
Nước đá có cấu trúc tinh thể tứ diện. Một phân tử nước nằm ở tâm của tứ diện liên kết với 4 phân tử nước
khác ở 4 đỉnh bằng liên kết hydrogen, do đó, tinh thể nước đá có cấu tạo rỗng. Khi chuyển nước đá từ thể rắn
sang thể lỏng, cần cung cấp năng lượng (là quá trình thu nhiệt) làm phá vỡ 1 phần liên kết hydrogen, làm sập
khung tinh thể của nước đá, dẫn đến cac phân tử nước ở gần nhau hơn → khối lượng riêng tăng.
2.b/ 0,25 điểm

4
Phân tử nước có cực và có thể tạo liên kết hydrogen với nhiều chất nên có khả năng hòa tan nhiều chất. Các
quá trình hóa lí của sự sống hầu như xảy ra trong dung dịch nên sự hòa tan các chất có ý nghĩa quan trọng
đối với sự sống.
3a. 0,5 điểm
Tính áp suất ở độ sâu 5000 m được tạo bởi khối băng:
F
P= = h g = 5000 (m)  0,915  103 (kg m −3 )  9,8(m s −2 ) = 448,35  105 Pa = 448,35 bar.
A
dP H fus T V ( P2 − P1 ) M ( P2 − P1 )  1 1 
=  ln 2 = =  − 
dTfus Tfus V T1 H fus H fus  l s 
T M  P  1 1 
ln 2 =  − =
T1 H fus   (l )  ( s ) 
18 10−3 kg mol−1 ( 448,35 105 Pa − 1, 013 105 Pa )  1 1  3 −1
= −1
 − 3 
m kg = −0, 01217
6, 0110 J mol
3
 0,998 10 0,915 10 
3

T2
= e −0,01217 = 0,987  T2 = 269, 7 K.
T1
Tfus (5000 m) = -3,3 C.
Ý nghĩa: ở sâu dưới những khối băng dày, do áp suất lớn, nước sẽ nóng chảy ở nhiệt độ âm hơn so với ở
trên bề mặt, nơi mà áp suất chỉ 1,013 bar.
Chú ý: nếu thí sinh tính theo công thức
P H fus
=
T T V
Mà không có lý luận đúng tại sao có thể dùng được công thức này thì không cho điểm.
b.
i/ 0,5 điểm
Tại điểm giày trượt tiếp xúc với sân băng, mặt sân chịu thêm một áp lực gây bởi trọng lượng của vận động
viên
F = 61,5 kg . 9,81 m.s-2 = 6, 033.102 kg. m.s-2
Diện tích tiếp xúc của giày trượt với sân băng : A = 6, 0.10-2 m.10-3 m = 6, 0.10-3 m2
Áp suất tác dụng thêm lên sân băng do trọng lực của người trượt:
𝐹 6,033.102 𝑘𝑔.𝑚.𝑠−2
P= 𝐴 = = 1,0.107 Pa
6,0.10−5 𝑚2
Dưới áp suất 107 Pa = 7,5 x 104 mmHg ở − 0,5 0C, từ giản đồ pha sẽ thấy nước ở thể lỏng. Như vậy, vận
động viên sẽ trượt trên một lớp nước lỏng mỏng.
− Nước bị chảy dưới lưỡi dao tạo thành một rãnh nhỏ, tránh cho vận động viên bị ngã.
ii/ 0,5 diểm
dG = -SdT + VdP ⟹ dGT = VdP
Áp suất tác dụng thêm lên sân băng do trọng lực của người trượt P = 1,0.107 Pa.
18,0 𝑔.𝑚𝑜𝑙 −1
V(mol) (s) = 0,917 𝑔𝑐𝑚−3 = 19,629 cm3mol-1 = 1,963. 10-5 m3mol-1
dGmol= VmoldP ⟹ ∆Gmol(s) = 1,963.10-5 m3mol-1.1,0.107 kgm-1s-2
∆Gmol(s) = 196,3.J.mol-1.
18,0 𝑔.𝑚𝑜𝑙 −1
V(mol) (l) = = 18,018 cm3mol-1 = 1,802. 10-5 m3mol-1
0,999 𝑔𝑐𝑚−3
dGmol= VmoldP ⟹ ∆Gmol(l) = 1,802.10-5 m3mol-1.1,0.107 kgm-1s-2
∆Gmol(l) = 180,2.J.mol-1.
Vậy ∆Gmol(s) = 196,3.J.mol-1.
∆Gmol(l) = 180,2.J.mol-1.
Lập luận này có phù hợp. Khi chịu thêm trọng lực của vận động viên, ∆𝐺 giảm từ 196,3 J (thể rắn) xuống
180,2 J (thể lỏng) nên xảy ra quá trình chuyển pha của nước từ thể rắn sang thể lỏng. Khi trượt qua, vì nhiệt
độ sân băng luôn được giữ không đổi ở -0,5oC nên nước lỏng lại chuyển thành nước đá.

5
Câu 4 (2,5 điểm): Hóa nguyên tố (KL, PK nhóm IV, VA). Phức chất
Cho sơ đồ biến hóa sau:

Biết: A2 nghịch từ, trong đó Cr chiếm 27,66 %, O chiếm 42,55% và C chiếm 25,53% theo khối
lượng. Từ dung dịch A6 tách được muối khan A6*. Khi hòa tan 0,131 g A6* trong H2SO4 loãng, dung dịch
thu được phản ứng vừa đủ với 10,0 mL dung dịch FeSO4 nồng độ 0,40 M. Chất lỏng A8 bốc khói mạnh
trong không khí ẩm. Khi hòa tan cẩn thận 0,372 g A8 vào 160 mL dung dịch Ba(OH)2 0,05 M không thấy có
khí thoát ra, thu được 0,607 g kết tủa màu vàng và dung dịch không màu X. Để trung hòa X cần 64,0 mL
dung dịch HCl 0,10 M.
Xác định các chất từ A1 đến A8 và viết các phương trình phản ứng trong sơ đồ biến hóa.
Hướng dẫn
Cr + HCl(loãng) → CrCl2 + H2
→A1 là CrCl2
Cho A1 tác dụng với CH3COONa thu được A2 là 1 kết tủa đỏ có CTTQ: CrxCyOzHt, %H = 4,26%
27, 66 25,53 42,55 4, 26
Ta có: x : y : z : t = : : : = 1: 4 : 5 : 8 → CTĐGN của A2 là CrC4O5H8
52 12 16 1
Mặt khác, A2 nghịch từ, là phức được tạo bởi các phối tử CH3COO- và H2O; Cr2+có cấu hình electron là
[Ar]3d4 → A2 có công thức là Cr2C8O10H16 hay Cr2(CH3COO)4(H2O)2
+ A2 tác dụng với O2, H2SO4 loãng tạo ra dung dịch A3 có màu xanh lam → A3 là Cr2(SO4)3 (O2 trong môi
trường axit oxi hóa Cr2+ thành Cr3+)
+ A3 tác dụng với dung dịch KOH dư thu được dung dịch A4 màu lục nhạt → A4 là K3[Cr(OH)6]
+ A4 tác dụng với H2O2 là chất oxi hóa. Sẽ oxi hóa Cr(III) trong môi trường kiềm thành Cr(VI) → dung dịch
A5 màu vàng chanh là K2CrO4
+ A5 trong dung dịch H2SO4 đặc tạo thành A6 → A6: K2CrxOy. A6 tiếp tục chuyển thành A7 là chất rắn màu
đỏ thẫm khi H2SO4 đặc dư → A7 là CrO3
+ Muối A6* oxi hóa FeSO4 trong môi trường axit
Theo bảo toàn electron, n(Cr(VI).3 = n(Fe(II)).1 → n(Cr(VI)) = 4.10-3/3 (mol) → M(A6*) = 123,75x
Vậy, ta có: 52x + 16y = 123,75 – 78 và bt điện tích: 2 + 6x = 2y. Giải hệ được x = 4; y = 13
→A6* là K2Cr4O13
+ Ba(OH)2 pư với A8 = 8.10-3 – 6,4.10-3/2 = 4,8.10-3 mol, BaCrO4 (kt vàng) = 2,4.10-3 mol
→ dd X chứa BaCl2 = 2,4.10-3 mol.
Vậy A8 chứa: Cr: 2,4.10-3 mol; Cl: 4,8.10-3 mol → O: 4,8.10-3 mol
→Cr : Cl : O = 1 : 2 : 2 → A8 là CrCl2O2.
+ Các phương trình phản ứng:
(1)/ Cr + 2HCl → CrCl2 + H2
(2)/ CrCl2 + 2CH3COONa + 2H2O → [Cr2(OOC – CH3)2(OH2)2] + 2NaCl
(3)/ [Cr2(OOC – CH3)2(OH2)2] + O2 + 3H2SO4 (loãng) → 2CH3COOH + Cr2(SO4)3 + 4H2O
(4)/ Cr2(SO4)3 + 12KOH → 3K2SO4 + 2K3[Cr(OH)6]
(5)/ 2K3[Cr(OH)6] + 3H2O2 → 2K2CrO4 + 2KOH + 6H2O
(6)/ 4K2CrO4 + 3H2SO4(đặc) → K2Cr4O13 + 3K2SO4 + 3H2O
(7)/ K2Cr4O13 + 6H2O2 + 10KOH + H2O → 4K3[Cr(OH)6] + 6O2
(8)/ K2Cr4O13 + H2SO4 (đặc dư) → K2SO4 + 4CrO3 + H2O(H2SO4đ)
(9)/ CrO3 + 2HCl → CrCl2O2 + H2O

Câu 5. (2,5 điểm)Cơ sở hữu cơ

1. So sánh và giải thích độ bền hai đồng phân hình học của
6
a) 2,2,5,5-tetramethyl-3-hexene
b) cyclooctene
c) Cyclodecene có thể tồn tại ở dạng cis và trans, nhưng cyclohexene thì không.
Hướng dẫn
a) Trong cả hai trường hợp, đồng phân trans đều bền hơn đồng phân cis. Chênh lệch về độ bền
đồng phân cis-trans trở nên rõ ràng hơn nhiều trong hợp chất tetramethyl. Hãy thử biểu diễn cấu
tạo của cis-2,2,5,5-tetramethyl-3-hexene, bạn sẽ nhận thấy sức căng không gian lớn gây ra bởi các
nhóm methyl.

b. Tồn tại sức căng xoắn làm cho phân tử trans kém bền hơn cis

b) Với cyclohexen vòng 6 cạnh quá nhỏ, gây ra sức căng vòng lớn nên trans kém bền hơn cis

Cyclodecene là vòng đủ lớn, đủ linh hoạt để đáp ứng cấu hình đồng phân cis và trans

2. Cho biết 4-methylcyclohexylideneacetic acid có quang hoạt không? Vẽ và giải thích

Hướng dẫn
Nguyên nhân gây ra tính quang hoạt của 4-methylcyclo hexylideneacetic acid là không có

7
mặt phẳng đối xứng và không thể chồng khít với ảnh gương của mình. Cũng tương tự như trường
hợp allene, hai nhóm ở một đầu của phân tử nằm trên mặt phẳng vuông góc với mặt phẳng chứa
hai nhóm của đầu kia.

3. Cho các chất dưới đây:

Chất nào trong mỗi cặp (B1, B2), (B3, B4) và (B5, B6) có giá trị momentlưỡng cực lớn hơn? Giải thích.

Hướng dẫn

B2 có moment lưỡng cực lớn hơn B1: do B2 có cộng hưởng kéo dài liên quan đến NH2 và NO2.

B4 có moment lưỡng cực lớn hơn B3: Do B4 có thể chuyển thành hệ thơm,điện tích (–) ở vòng 5
cạnh và (+) ở vòng 7 cạnh.

B5 có momen lưỡng cực lớn hơn B6 do B5 có khả năng tạo liên kết H nội phân tử khi hai nhóm OH
ở vị trí syn. B 6 không tạo được liên kết H nội phân tử nên 2 nhóm OMe ở vị trí anti

4.Cho các giá trị pKa: 2, 5 , 11, 13, 18, 20 , 25. Ghép pKa phù hợp cho mỗi hợp chất sau, hợp chất
nào đã cho sẽ bị proton hóa 99% trong dung dich EtONa/EtOH ?

8
Hướng dẫn
- Sắp xếp pKa

-Để poroton hóa 99%, phải có pKa thấp hơn/bằng 13, nên a, c,d,e sẽ thỏa mãn.

5. Xác định và giải thích dạng bền của các cân bằng cấu dạng sau đây
a.

b.

Hướng dẫn

9
a.

Đối với dạng 1

Ở dạng này không có sự tương tác xen phủ nào giữa các orbital.
Đối với dạng 2:

- Không có tương tác 1,3-diaxial nào.


- Có 4 tương tác nN → σ*C-N làm bền phân tử.
→ Dạng 2 bền hơn và chiếm ưu thế trong cân bằng
b.

Dạng 1 có tương tác xen phủ nN → σ*C-Ph

Dạng 2 có tương tác xen phủ nN → σ*C-Cl

Mà E σ*C-Cl thấp hơn Eσ*C-Ph do độ âm điện Cl lớn hơn nhóm phenyl. Do đó


tương tác xen phủ nN → σ*C-Cl hiệu quả hơn.
→ Dạng 2 bền hơn và chiếm ưu thế trong cân bằng
Câu 6(2,5 điểm) cơ chế xác định chất
6.1.Viết cơ chế
1.
10
2.

3.

4.

Hướng dẫn
1.

2.

11
3.

4.

12
2.2. Đề xuất cấu trúc các hợp chất từ E-H. hợp chất E (C5H8) quang hoạt, khi khử chọn lọc liên kết đôi trong
E bằng xúc tác thích hợp tạo ra F (C5H10) không quang hoạt (duy nhất). Hợp chất G có công thức C6H10 và
quang hoạt, G không có liên kết ba. Khử hóa G thu được H (C6H14) không quang hoạt (duy nhất).
Hướng dẫn

HC C CH

E F G H

2.3. Hợp chất A, C8H10O3 có tính quang hoạt và chỉ chứa vòng năm cạnh. Cho A tác dụng với m-
CPBA thu được A1. Đun nóng A1 với dd NaOH loãng, dư, sau khi axit hóa thì thu được B. Cho B
tác dụng với dd HIO4 (H5IO6) thu được C và D đều không quang hoạt và có cùng CTPT là C4H6O3.
Cả C và D đều tác dụng với dd NaHCO3 giải phóng CO2, nhưng chỉ C tác dụng được với thuốc thử
Tollens. Cho D tác dụng với I2/NaOH, axit hóa sản phẩm tạo thành rồi đun nóng thu được axit
axetic. Hãy xác định công thức cấu tạo có thể có của các chất từ A đến D.

Gợi ý:

Câu 7.(3,0 điểm) Sơ đồ


7.1. Dưới đây là công thức của (+)-Grandisol (K)- một chất dụ dẫn của loài bọ cái sống trên cây bông.
13
Hỗn hợp raxemic Grandisol được điều chế theo sơ đồ sau, sử dụng 5-hepten nitrin (A) làm chất
đầu:

Xác định cấu tạo các chất chưa biết (không cần biểu diễn hóa lập thể), cơ chế phản ứng của các
chuyển hóa từ C→ D.

Hướng dẫn

Khử C=O, Wolft-Kischner

Cơ chế phản ứng từ C tạo D

14
7.2. Cedrene là một thành phần trong tinh dầu của cây tuyết tùng châu Mỹ. Cedrene tự nhiên, phân lập bằng
cách chưng cất phân đoạn dầu gỗ cây tuyết tùng. Một trong những phương pháp tổng hợp α-cedrene tinh
khiết do Breitholle và Fallis thực hiện được trình bày trong sơ đồ sau:

(p-TsCl: p-CH3C6H4SO2Cl; HMPA: Hexamethylphosphoramide (dung môi phản ứng); py: pyridine).
Xác định công thức cấu tạo các hợp chất A, B, C, D và E (không cần hóa lập thể)
Hướng dẫn

7.3. Phản ứng chuyển vị Vinylcyclopropane-thành-cyclopentene (VCR) và những phản ứng cùng kiểu của
các hợp chất dị tố tương ứng được xem là phương pháp quan trọng để tổng hợp các hợp chất đồng và dị
vòng 5 cạnh. Các chuyển vị này có thể được thực hiện trong điều kiện nhiệt, quang hóa hoặc hoạt hóa xúc
tác. Các phản ứng nhiệt và quang hóa xảy ra qua sự hình thành gốc đôi bền nhất.

15
a.Xác định công thức cấu tạo của sản phẩm tạo thành từ hai phản ứng trên, biết rằng: cả hai phản ứng đều
thuộc loại VCR; phân tử Y có trục đối xứng.

b.VCR là giai đoạn đầu tiên trong tổng hợp toàn phần terperne hydrocarbon (±)-zizaene (Z):

b1.Tính độ bất bão hòa của zizaene (Z)?


b2. Zizaene có quang hoạt không? Hãy vẽ đối quang của nó?
b3. Xác định công thức cấu trúc các hợp chất từ A - L(có chú ý hóa lập thể)
Hướng dẫn
a)

b1.Zizaene là hợp chất 3 vòng (tricyclic) và có chứa 1 liên kết C=C. Do vậy, độ bất bão hòa bằng 4

b2. Zizaene có quang hoạt

16
b3. Hoàn thành sơ đồ:

Câu 8 (2,0 điểm)


8.1.Các hợp chất mô phỏng đường (glycomimetics) là một trong số nhiều cách chuyển hóa hoạt
tính sinh học của các hợp chất đường để nghiên cứu điều trị y học. Việc tổng hợp các hợp chất mô
phỏng đường chứa hệ thống cyclobutane không chỉ góp phần nghiên cứu cấu trúc mà cũng có thể
tạo ra các phản ứng mới hữu dụng trong tổng hợp hữu cơ.
Năm 2009, một hợp chất vừa có bộ khung vòng oxetane vừa liên kết với một base nitrogen là
albucidin đã được cô lập. Các khảo sát hoạt tính sinh học cho thấy hợp chất này có khả năng diệt cỏ
dại rất tốt. Quá trình tổng hợp bắt đầu từ xylose được thể hiện ở sơ đồ sau:

Xác định cấu trúc các chất chưa biết, biết phức fac-[Ir(Mppy)3] với ppy là tên viết tắt của p-
tolylpyridine còn M biểu thị cho IridiuM, để cắt liên kết C-I.

Hướng dẫn

17
Quá trình tổng hợp bắt đầu bằng việc oxy hóa lacton hóa phá vỡ hệ thống vòng xylose, sau
đó hai nhóm OH tự do được bảo vệ bằng benzaldehyde. Nhóm OH còn lại sau đó được chuyển
thành dẫn xuất mesylate vốn là một nhóm đi ra tốt, tạo thuận lợi cho sự mở vòng lacton đóng vòng 4
cạnh. Nhóm COONa sau đó tham gia phản ứng Kochi decarboxylation (một biến thể của phản ứng
Hunsdiecker) để tạo thành dẫn xuất chloride tương ứng. Deproton hóa adenine dưới tác dụng của
NaH rồi base sinh ra thế Cl để tạo thành khung albucidin. Chuyển hóa nhóm chức để loại bỏ nhóm
OH bậc hai bằng phản ứng quang hóa xúc tác với ánh sáng khả kiến (visible light photocatalyst)
theo cơ chế gốc tự do sẽ thu được sản phẩm mong muốn.

8.2. 1,4-Dideoxy-1,4-imino-D-arabinitol [(2R, 3R, 4R)-2-hydroxymethyl pyrrolidine-3,4-diol, DAB1] đã


được tìm thấy ở các loại dương xỉ Arachniodes standishii và Angylocalyx boutiqueanus. Hợp chất
DAB1 là một chất ức chế mạnh enzyme glucosidase (ức chế 50% ở nồng độ1,8 x 10-7 M). DAB1 còn
ức chế quá trình thủy phân sinigrin và progoitrin (là các hợp chất nhóm glucosinolate) từ rệp
Brevicoryne Brassicae của bắp cải. DAB1 được tổng hợp từ D-glucose theo sơ đồ dưới đây:

Hãy xác định công thức cấu trúc của các hợp chất trong sơ đồ.
18
Hướng dẫn

19
Trường THPT chuyên Chu Văn An – KỲ THI CHỌN HỌC SINH GIỎI CÁC TRƯỜNG THPT CHUYÊN
Bình Định
KHU VỰC DUYÊN HẢI VÀ ĐỒNG BẰNG BẮC BỘ
HƯỚNG DẪN CHẤM LẦN THỨ XIV, NĂM 2023
(Hướng dẫn chấm gồm 18 trang) HƯỚNG DẪN CHẤM MÔN: HÓA HỌC - LỚP 11
Thời gian: 180 phút (Không kể thời gian giao đề)
Câu 1 (2,5 điểm) Tốc độ phản ứng.
Để phân hủy hiđro peoxit (H2O2) với chất xúc tác là ion iođua trong dung dịch có môi trường trung tính,
người ta trộn dung dịch H2O2 3% (chấp nhận tương đương với 30 gam H2O2 trong 1lít dung dịch) và dung dịch KI
0,1 M với nước theo tỉ lệ khác nhau về thể tích để tiến hành thí nghiệm xác định thể tích oxi ( VO2 ) thoát ra.
Thí nghiệm VH2O2 (ml) VKI (ml) VH2O (ml) υ O2 (ml/phút)
ở 298 K và 1 atm
1 25 50 75 4,4
2 50 50 50 8,5
3 100 50 0 17,5
4 50 25 75 4,25
5 50 100 0 16,5
1.1. Xác định bậc phản ứng phân huỷ đối với H2O2 và đối với chất xúc tác I .
-

1.2. Viết phương trình hoá học và biểu thức tính tốc độ phản ứng.
1.3. Tính nồng độ mol của H2O2 khi bắt đầu thí nghiệm 4 và sau 4 phút.
1.4. Cơ chế phản ứng được xem là một chuỗi hai phản ứng sau:
k
H2O2 + I- ⎯⎯
1 → H O + IO-
2 (1)
k
IO- + H2O2 ⎯⎯ 2 → O + I- + H O
2 2 (2)
Hãy cho biết hai phản ứng này xảy ra với tốc độ như nhau hay khác nhau? Phản ứng nào quyết định tốc độ
phản ứng giải phóng oxi? Giải thích.
Hướng dẫn chấm
Ý Nội dung Điểm
1.1 0,75
Từ phương trình phản ứng: 2 H2O2 → 2 H2O + O2
ta có: thể tích oxi thoát ra trong 1 đơn vị thời gian tỉ lệ thuận với tốc độ phản ứng
Theo các thí nghiệm 1, 2, 3 khi tăng gấp đôi thể tích dung dịch H2O2 và giữ nguyên thể tích
của dung dịch KI thì tốc độ phản ứng tăng gấp đôi, điều đó có nghĩa là tốc độ phản ứng tỉ lệ
thuận với nồng độ của H2O2 → phản ứng là bậc 1 đối với H2O2.
Tương tự, từ các thí nghiệm 2, 4, 5 ta thấy tốc độ phản ứng tỉ lệ thuận với nồng độ của I- →
phản ứng là bậc 1 đối với I-.

1.2 0,25
Phương trình phản ứng: 2 H2O2 → 2 H2O + O2
Biểu thức của định luật tốc độ phản ứng: V = k . C H 2O2 .C I-

1.3 0,75
Khi pha loãng 3 lần thì nồng độ của H2O2 (C0) ở thí nghiệm 4 giảm 3 lần:
10
→ C0 = 10 gam H2O2/1 lit. Hay C0 = = 0,294 M.
34
Vì phản ứng xảy ra chậm nên có thể coi như tốc độ phản ứng (thể tích oxi thoát ra) không
thay đổi trong khoảng thời gian ngắn (4 phút).
Sau 4 phút sẽ thoát ra: 4,25 . 4 = 17 (ml) oxi, khi đó:
P.V 1 . 17.10-3
n O2 = = = 0,695.10-3 (mol)
R.T 0,082 . 298
Lúc đầu có: n H2O2 = 0,294 . 0,15 = 44,1.10-3 (mol).
Sau 4 phút, số mol H2O2 chỉ còn: 44,1.10-3 – 2 . 0,695.10-3 = 42,71.10-3 (mol).

Trang 1/15
0, 04271
Vậy sau 4 phút: C H 2 O 2 = = 0,285 (M).
0,15

1.4 Phản ứng: 0,75


2 H 2O2 ⎯⎯
→ 2 H 2 O + O2
I-
(*)
1 d  H 2O2 
v=−
2 dt
Cơ chế:
H 2 O 2 + I- ⎯⎯
k1
→ H 2 O + IO- (1)
IO- + H 2 O2 ⎯⎯
k2
→ H 2 O + I- + O2 (2)
Xét 3 trường hợp:
1/ Nếu phản ứng (1) chậm và quyết định tốc độ thì tốc độ của phản ứng tổng hợp (*)
bằng tốc độ của phản ứng (1):
1 d  H 2O2 
v=− = k1[H 2 O2 ][I- ]
2 dt
Cơ chế phù hợp với định luật tốc độ.
2/ Nếu phản ứng (2) chậm thì:
1 d  H 2O2 
v=− = k2 [H 2O2 ][IO- ] (a)
2 dt
Chấp nhận nồng độ của IO- là ổn định ta có:
d [IO- ] k
= k1[H 2O2 ][I- ] − k2 [IO- ][H 2O 2 ] = 0 → [IO - ] = 1 [I - ] (b)
dt k2
Thay (b) vào (a) ta được:
1 d  H 2O2 
v=− = k1[H 2 O2 ][I- ]
2 dt
Cơ chế phù hợp với định luật tốc độ.
3/ Nếu hai phản ứng có tốc độ xấp xỉ nhau thì:
= ( k1[H 2 O2 ][I- ] + k2 [H 2 O2 ][IO- ])
1 d [H 2 O 2 ] 1
v=−
2 dt 2
Chấp nhận nồng độ của IO- là ổn định, rồi tính [IO-] như ở trường hợp 2 và thay vào biểu
thức trên ta được:
1 d  H 2O2 
v=− = k1[H 2 O2 ][I- ]
2 dt
Cơ chế phù hợp với định luật tốc độ. Trong 3 trường hợp, trường hợp đầu hợp lí hơn
cả vì ở đây không cần chấp nhận điều kiện gì; mặt khác ở trường hợp 2, nếu đã giả thiết phản
ứng (2) là chậm thì việc chấp nhận nồng độ của IO- ổn định là không hợp lí.
Câu 2 (2,5 điểm) Cân bằng và phản ứng trong dung dịch. Pin điện - Điện phân.
Một trong những thuốc thử đặc trưng để tìm ion Pb2+ (trong dung dịch) là Na2CrO4. Cho biết, kết tủa PbCrO4
màu vàng, tan được trong dung dịch NaOH dư; trong khi đó, kết tủa PbS màu đen, không tan được trong dung dịch
NaOH.
Thêm từ từ 0,05 mol Pb(NO3)2 vào 1,0 lít dung dịch X gồm 0,02 mol Na2S và 0,03 mol Na2CrO4, thu được
hỗn hợp Y gồm phần kết tủa và phần dung dịch (coi thể tích không thay đổi khi thêm Pb(NO3)2 vào dung dịch X).
2.1. Tính pH của dung dịch X.
2.2. Bằng lập luận và đánh giá hợp lí, chứng tỏ rằng, pH phần dung dịch của Y xấp xỉ bằng 7,0.
2−
2.3. Tính [Cr2 O 7 ] và [Pb2+] trong phần dung dịch của Y.
2.4. Trình bày cách thiết lập sơ đồ pin được ghép bởi điện cực chì (Pb) nhúng trong hỗn hợp Y và điện cực
hiđro tiêu chuẩn.
Cho biết:
0
pK a1(H S) = 7,02; pK a2(H S) = 12,90; pK
2 2 − = 6,50; E 2+ = -0,126 V
Pb /Pb
a(HCrO4 )

Trang 2/15
pKs(PbS) = 26,60; pKs(PbCrO4 ) = 13,70; pKs(Pb(OH)2 ) = 14,90
2− 2−
2 CrO 4 + 2H+ Cr2 O 7 + H2O K = 3,13.1014
 
Pb2+ + H2O PbOH+ + H+ lg β1 = lg β = -7,80
Pb(OH)+
 
Pb2+ + 2H2O Pb(OH)2(dd) + 2H+ lg β 2 = lg βPb(OH) = -17,20
2
−  
Pb2+ + 3H2O Pb(OH) + 3H+ lg β3 = lg β = -28,00
3 Pb(OH)3−

2,303RT
(với pKa = -lgKa; pKs = -lgKs; ở 25oC: = 0,0592 V)
F
Hướng dẫn chấm
Ý Nội dung Điểm
2.1 Các quá trình xảy ra trong dung dịch X: 0,75
S2- + H2O HS- + OH- Kb1 = 10 -1,1
(1)
-
HS + H2O H2S + OH- Kb2 = 10-6,89 (2)
2− −
CrO 4 + H2O HCrO 4 + OH-
Kb = 10-7,50 (3)
Chú ý: Kb của (3) tính được từ pKa của HCrO4 và Kw(H2O).
-

H2O OH- + H+ Kw = 10-14 (4)


So sánh các cân bằng (1), (2), (3) và (4) ta có:
Kb1. CS 2− >> Kb2. CHS − > Kb. CCrO2− >> Kw nên pHX được tính theo (1):
4

S2- + H2O HS- + OH- Kb1 = 10-1,1 (1)


[ ] 0,02 – x x x
→ [OH-] = x = 0,0166 (M) → pH = 12,22.
2.2 Pb2+ + S2- → PbS  0,75
0,05 0,02 0,02
Pb2+ + CrO 4
2−
→ PbCrO4 
0,03 0,03 0,03

Trong hỗn hợp Y, phần kết tủa gồm PbS và PbCrO4; phần dung dịch gồm Na+ và NO 3 . Từ
cân bằng, ta có:
2− 2−
2 CrO 4 + 2H+ Cr2 O 7 + H2O K1 = 3,13.1014
2H2O 2H+ + 2OH- (KW)2 = 10-28
→ 2 CrO 24− + H2O 2−
Cr2 O 7 + 2OH- K2 = 3,13.10-14
Vì KS(PbCrO4 ) >> KS(PbS) và vì  β1 >>  β2 >>  β3 nên trong hỗn hợp Y chủ yếu xảy ra các
cân bằng:
2−
PbCrO4 Pb2+ + CrO 4 KS = KS(PbCrO4 ) = 10-13,70 (5)
S S (với S là độ tan của PbCrO4 trong Y)

Pb 2+
+ H2O PbOH +
+ H +
β1 = 10-7,80 (6)
2− −
CrO 4 + H2O HCrO 4 + OH- Kb = 10-7,50 (7a)
2− 2−
2 CrO + H2O
4
Cr2 O + 2OH- K2 = 3,13.10-14
7 (7b)
H2O OH- + H+ Kw = 10-14 (8)
-14
Vì K2 = 3,13.10 nhỏ, nên chấp nhận bỏ qua quá trình (7b).
So sánh các cân bằng (6), (7a) và (8) ta thấy:

β1 . S0 =  β1 Ks = 10-14,65  Kb. S0 = Kb. K s = 10-14,35  KW = 10-14
(với S0 là độ tan của PbCrO4 trong nước không kể các quá trình (6), (7a) và (7b)), do đó, có
thể coi khả năng cho, nhận proton của các cấu tử trong hỗn hợp Y gần như tương đương nhau,

Trang 3/15
vì vậy pHY  7,0.
Chú ý:
Việc kiểm tra cho thấy ở pH = 7,0:
−17,2 − −28
2 3
* *
[Pb(OH) 2(dd) ] 10 [Pb(OH) 3 ] 10
2+
= + 2
= −14
1; 2+
= + 3
= −21
1
[Pb ] [H ] 10 [Pb ] [H ] 10

nghĩa là việc tính toán bỏ qua sự tạo phức Pb(OH)2(dd) và Pb(OH) 3 là hợp lý.
2.3 Tính [Cr2O72-] và [Pb2+] trong phần dung dịch của Y: 0,5
Tại pH = 7,0 thì:
2− 2− 2−
[ Cr2 O 7 ] = 3,13.1014.[ CrO 4 ]2.[H+]2 = 3,13.[ CrO 4 ]2
2− − 2−
Mặt khác: [ CrO 4 ] + [ HCrO 4 ] + 2[ Cr2 O 7 ] = S
2− 2−
Giả sử [ Cr2 O 7 ] << [ CrO 4 ] thì:
2− −1
[ CrO 4 ](1 + K a .[H+]) = S
S
→ [ CrO 4 ] =
2−
−1 +
(a)
1 + K a .[H ]
Lại có: [Pb2+] + [PbOH+] = S
S
→ [Pb ] =
2+
+ −1
(b)
1 + 1.[H ]
*

Từ (a) và (b)
→ S = Ks .(1 + * 1.[H + ] −1).(1 + K a−1.[H + ]) , thay các giá trị hằng số vào thu
được: S =1,74.10-7 (M)
Thay giá trị S vào (a) ta có:[CrO42-] = 1,32.10-7 (M) → [Cr2O72-] = 3,13.[ CrO 4 ]2 =
2−

3,13.(1,32.10-7)2 = 5,45.10-14 (M)


2−
[Cr2O72-] << [ CrO 4 ] (thỏa mãn giả thiết đặt ra ban đầu)
Từ (b), ta có: [Pb2+] = 1,51.10-7 (M)
2.4 Vì trong hỗn hợp Y quá trình phân li của PbCrO4 là chủ yếu, nên: 0,5
0,0592 2+
=E =E + lg[Pb ] = −0,328 (V)  E
0 0
EPb = E 2− 2+ 2+ +
=
PbCrO4 /Pb, CrO 4 Pb /Pb Pb /Pb 2H /H 2
2
0,00 (V)do ,đó điện cực Pb là anot, điện cực hiđro tiêu chuẩn là catot. Vậy sơ đồ pin:
-
(-) Pb│PbS, PbCrO4 | Na+ 0,1 M, NO 3 0,1 M ║ H+ 1,0 M│ H2 (p = 1 bar) | Pt (+)
Chú thích: Nếu học sinh ghi H2 (p = 1 atm) vẫn cho đủ điểm.

Câu 3 (2,5 điểm) Nhiệt động học và cân bằng hóa học.
3.1. Cho một khối kim loại X nặng 2,0 kg ở 0oC vào một bình có chứa sẵn 1,0 mol hơi nước ở 100oC và 1
atm thấy có 86% lượng hơi nước đã ngưng tụ. Giả sử trong điều kiện khảo sát, sự trao đổi nhiệt chỉ xảy ra giữa X
và nước, áp suất trong bình không đổi và quá trình ngưng tụ nước diễn ra ở 100oC.
a) Tính nhiệt độ cuối của hệ X-nước và nhiệt lượng mà X đã trao đổi.
b) Tính biến thiên entropi của X, của nước và của hệ X-nước.
Biết: X không chuyển pha: (𝐶𝑝 )𝑋 = 0,385 J.K-1.g-1; (𝐶𝑝 )𝐻 𝑂(𝑙) = 75,3 J.K-1.mol-1;
2
(𝐶𝑝 )𝐻2 𝑂(𝑘) = 33,6 J.K-1.mol-1; (∆𝐻ℎℎ;100𝑜 𝐶 )𝐻2 𝑂(𝑙) = 40,656 kJ.mol-1.
⎯⎯
→ 2NO2(k)
3.2. Xét cân bằng: N2O4(k) ⎯

a) Một lượng khí N2O4 được đặt trong một xilanh ở nhiệt độ T1 = 250C. Sau khi cân bằng được thiết lập, áp
suất chung của hệ là 1,5 atm và có 16% số mol của N2O4 bị phân hủy thành NO2. Tính áp suất ban đầu của N2O4 và
Kp của phản ứng phân hủy ở 250C.

Trang 4/15
b) Nếu nhiệt độ vẫn giữ ở 200C nhưng tăng dần thể tích của xilanh cho tới khi áp suất chung của hệ bằng 1
atm. Tính áp suất cân bằng của NO2 và N2O4 trong điều kiện này.
c) Tính phần trăm số mol của N2O4 đã bị phân hủy ở trạng thái cân bằng mới (áp suất chung là 1atm,
0
25 C).
d) Cho sinh nhiệt tiêu chuẩn ở 298K của N2O4(k) bằng 9,2 kJ/mol và của NO2(k) bằng 33,2 kJ/mol. Tính
S0 phản ứng và nhiệt độ T2 để phản ứng phân hủy N2O4 có hằng số cân bằng Kp bằng 1. Coi H0pư và S0pư
không phụ thuộc nhiệt độ.
Câu Đáp án Điểm
3.1

1,0

Trang 5/15
0,5

3.2
a) a) N2O4(k) 2NO2(k) α = 0,16
Ban đầu: P0 0 (atm)
Cân bằng: 0,84P0 0,32P0
1,5
 P chung = 1,16.P0 = 1,5 atm  P0 = = 1,293 atm
1,16 0,25
Ở cùng nhiệt độ và thể tích, tỉ lệ số mol các khí bằng tỉ lệ áp suất nên:
nNO2 0,32 P0
PNO2 = .Pc = .1,5 = 0, 414 atm
nc 1,16.P0
nN2O4 0,84 P0
PNO2 = .Pc = .1,5 = 1, 086 atm
nc 1,16.P0
2
PNO (0, 414) 2
KP = 2
= = 0,158
PN 2O4 1, 086
b)
Coi như thực hiện phản ứng ở thể tích mà áp suất cân bằng là 1atm
N2O4(k) 2NO2(k)
Ban đầu: P0 0 (atm)
Cân bằng: 1-x x 0,25
2
PNO x2
KP = 2
= = 0,158  x2 + 0,158x – 0,158 = 0
PN2O4 1− x
 x = 0,326
Vậy: PNO = 0,326 atm ; PN O
2 2 4
= 0, 674 atm
c) 1 0,326
- Áp suất N2O4 đã bị phân hủy bằng PNO2 = = 0,163 atm.
2 2
- Áp suất ban đầu của N2O4 bằng áp suất cân bằng của nó cộng với áp suất đã
phân hủy: 0,25
P0 = 0,674 + 0,163 = 0,837 atm
Vậy tỉ lệ phân hủy của N2O4 là:

Trang 6/15
0,163
' = .100% = 19, 47%
0,837

d) Ta có: H 0 pu = 2H NO
0
− H N0 O = 2.33, 2 − 9, 2 = 57, 2kJ
2 2 4

G = − RT ln K P = −8,314.298ln(0,158) = 4571,5 J
0
298, pu

H − G 298, pu
0 0
0,25
S = = 176, 6 J/K
0 298, pu
Ta có: pu
298
G = − RT ln1 = 0
0
- Khi hằng số Kp = 1 => T , pu

G = H − T .S = 0
0 0 0
T T T

Vì H0pứ và S0pứ không phụ thuộc vào nhiệt độ nên:


H 0
T= = 323,9K  324K
S 0

Câu 4 (2,5 điểm) Hoá nguyên tố (Kim loại, phi kim nhóm IVA, VA). Phức chất.
4.1. Các kim loại nhóm IA như Li, Na, ... có hoạt tính hóa học cao. Một số hợp chất của chúng có ứng dụng
rộng rãi trong nhiều lĩnh vực.
a) Viết các phương trình hóa học xảy ra khi đốt cháy từng kim loại Li, Na trong không khí.
b) Hiện nay, một số hợp chất chứa kim loại kiềm là các vật liệu tiềm năng cho pin nhiên liệu bởi khả năng lưu trữ
hiđro cao, Li3N là một trong những hợp chất như vậy. Một phân tử Li3N có khả năng phản ứng với hai phân tử H2
qua hai phản ứng theo sơ đồ sau:
Li3N + H2 → A + B
A + H2 → C + B
Hoàn thành các phương trình hóa học và cho biết công thức phân tử các hợp chất A và B.
4.2. Một loại quặng chỉ chứa MnO2 và tạp chất trơ. Cân chính xác 0,5000 gam quặng trên rồi cho vào bình cầu
có nhánh. Thêm từ từ vào bình này khoảng 50 mL dung dịch HCl đặc. Đun nóng đến khi mẫu quặng tan hết, chỉ còn
lại tạp chất trơ. Hấp thụ hoàn toàn khí Cl2 thoát ra bằng lượng dư dung dịch KI, thu được dung dịch X. Chuyển toàn
bộ X vào bình định mức 250 mL, thêm nước cất đến vạch mức, lắc đều. Chuẩn độ 25,00 mL dung dịch này bằng dung
dịch chuẩn Na2S2O3 0,05 M (chỉ thị hồ tinh bột) thì hết 22,50 mL.
a) Viết các phương trình hóa học xảy ra.
b) Tính hàm lượng % theo khối lượng của MnO2 trong quặng trên.
4.3. Có 6 lọ hóa chất bị mất nhãn, mỗi lọ đựng một dung dịch muối nitrat của một kim loại: Ba(NO3)2, Al(NO3)3, Pb(NO3)2,
Zn(NO3)2, AgNO3, Cd(NO3)2. Để nhận biết từng dung dịch muối, chỉ được dùng 3 dung dịch thuốc thử. Hãy cho biết tên
của 3 dung dịch thuốc thử đó và trình bày cách tiến hành thí nghiệm để nhận biết mỗi dung dịch muối đựng trong mỗi lọ
và viết phương trình hóa học (dạng phương trình ion, nếu có) để minh họa.

Hướng dẫn chấm


Ý Nội dung Điểm

Trang 7/15
a) Các phương trình phản ứng xảy ra khi đốt cháy Li và Na trong không khí: 0,5
- Phản ứng của Li và Na với O2 :
4Li + O2 ⎯⎯ → 2Li2O
o
t

2Na + O2 ⎯⎯ → Na2O2
o
t

Na + O2 ⎯⎯ → NaO2
o
t

- Phản ứng của Li và Na với N2:


6Li + N2 ⎯⎯ → 2Li3N
o
t

6Na + N2 ⎯⎯ → 2Na3N
o
t

Chú ý: học sinh chỉ cần viết phản ứng của Li và Na với O2 và N2 cũng cho tối đa điểm.
Bên cạnh các phản ứng chủ yếu trên, trên thực tế còn xảy ra một số phản ứng sau:
- Phản ứng của Li, Na, Li2O, Na2O2, NaO2 với H2O.
- Phản ứng của LiOH, NaOH, Na2O2, NaO2 với CO2.
b) Hoàn thành các phản ứng trong sơ đồ: 0,25
Li3N + H2 ⎯⎯
→ Li2NH + LiH
to

(A) (B)
Li2NH + H2 ⎯⎯ → LiNH2 + LiH
o
t

(A) (C) (B)

a) Khử MnO2 bằng lượng dư dung dịch HCl nóng: 0,5


MnO2 + 4HCl → MnCl2 + Cl2 + 2H2O
Toàn bộ lượng Cl2 thoát ra được hấp thụ vào dung dịch KI dư :
Cl2 + 3KI → KI3 + 2KCl
Chuẩn độ lượng KI3 bằng dung dịch chuẩn Na2S2O3 :
KI3 + 2Na2S2O3 → Na2S4O6 + 2NaI + KI

b) Hàm lượng phần trăm về khối lượng MnO2 trong quặng 0,5
Từ các phản ứng trên ta có:
1
n MnO = nCl2 = n I = nNa S O
2 2
2 223
Số mol Na2S2O3 tiêu tốn để chuẩn độ 25,00 mL dung dịch X:

22,50.0, 05
nNa2 S2O3 = = 1,125.10−3 (mol )
1000

Số mol I2 (dạng I3-) có trong 250,0 mL dung dịch X:

1,125.10−3.10
nI2 = = 5, 625.10−3 (mol )
2

Số mol MnO2 = Số mol I2 (theo phương trình phản ứng) = 5,625.10-3 (mol)

% Khối lượng MnO2:


5,625.10−3.(55 + 16.2)
% mMnO = = 97,88%
2
0,5000

Dùng dung dịch axit clohiđric, dung dịch natri hiđroxit, dung dịch amoniac làm thuốc thử. 0,75
Tiến hành thí nghiệm để nhận biết mỗi dung dịch muối:
Đánh số thứ tự cho mỗi lọ hóa chất bị mất nhãn, ví dụ: Ba(NO3)2 (1), Al(NO3)3 (2), Pb(NO3)2 (3),
Zn(NO3)2 (4), AgNO3 (5), Cd(NO3)2 (6).
Thí nghiệm 1:
Mỗi dung dịch muối được dùng ống hút nhỏ giọt (công tơ hút) riêng biệt để lấy ra một
lượng nhỏ (khoảng 2 ml) dung dịch vào mỗi ống nghiệm đã được đánh số tương ứng. Dùng
công tơ hút lấy dung dịch HCl rồi nhỏ vào mỗi dung dịch muối trong ống nghiệm, có hai dung
Trang 8/15
dịch xuất hiện kết tủa, đó là các dung dịch Pb(NO3)2, AgNO3 do tạo thành các kết tủa trắng PbCl2
và AgCl.
Thí nghiệm 2:
Tách bỏ phần dung dịch, lấy các kết tủa PbCl2, AgCl rồi dùng công tơ hút nhỏ dung dịch
NH3 vào mỗi kết tủa, kết tủa nào tan thì đó là AgCl, do tạo ra [Ag(NH3)2]Cl, còn kết tủa PbCl2
không tan trong dung dịch NH3. Suy ra lọ (5) đựng dung dịch AgNO3, lọ (3) đựng dung dịch
Pb(NO3)2.
Các phương trình hóa học xảy ra:
Pb2+ + 2 Cl- → PbCl2↓ (1)
Ag+ + Cl- → AgCl↓ (2)
AgCl + 2 NH3 → [Ag(NH3)2]Cl (3)
Còn lại 4 dung dịch Al(NO3)3, Ba(NO3)2, Zn(NO3)2, Cd(NO3)2 không có phản ứng với
dung dịch HCl (chấp nhận bỏ qua các quá trình tạo phức cloro của Cd2+). Nhận biết mỗi dung dịch
muối này:
Thí nghiệm 3:
Cách làm tương tự như thí nghiệm 1 nhưng thay dung dịch HCl bằng dung dịch NaOH.
Nhỏ từ từ NaOH cho đến dư vào mỗi dung dịch muối trong ống nghiệm, dung dịch Ba(NO3)2
không có phản ứng với dung dịch NaOH, còn ba dung dịch Al(NO3)3, Zn(NO3)2 và Cd(NO3)2 tác
dụng với NaOH đều sinh ra các kết tủa trắng, nhưng sau đó kết tủa Cd(OH)2 không tan, còn
Al(OH)3 và Zn(OH)2 tan trong NaOH dư. Nhận ra được lọ (1) đựng dung dịch Ba(NO3)2; lọ (6)
đựng dung dịch Cd(NO3)2.
Các phương trình hóa học xảy ra:
Al3+ + 3 OH- → Al(OH)3↓ (4)
Al(OH)3 + OH- → [Al(OH)4]- (5)
Zn2+ + 2 OH- → Zn(OH)2↓ (6)
Zn(OH)2 + 2 OH → [Zn(OH)4]
- 2-
(7)
Cd2+ + 2 OH- → Cd(OH)2↓ (8)
Còn lại 2 dung dịch Al(NO3)3, Zn(NO3)2. Nhận biết mỗi dung dịch muối này:
Thí nghiệm 4:
Cách làm tương tự như thí nghiệm 1 nhưng thay dung dịch HCl bằng dung dịch NH3.
Nhỏ từ từ dung dịch NH3 cho đến dư vào từng dung dịch Al(NO3)3, Zn(NO3)2 đựng trong 2 ống
nghiệm, dung dịch muối nào tạo ra kết tủa không tan là dung dịch Al(NO3)3 (2), còn dung dịch
nào tạo thành kết tủa, sau đó kết tủa tan thì đó là dung dịch Zn(NO3)2 (4).
Các phương trình hóa học xảy ra:
Al3+ + 3 NH3 + 3H2O → Al(OH)3↓ + 3 NH4+ (9)
Zn2+ + 2 NH3 + 2H2O → Zn(OH)2↓ + 2 NH4+ (10)
Zn(OH)2 + 4 NH3 → [Zn(NH3)4]2+ + 2 OH- (11)
Câu 5 (2,5 điểm) Đại cương hữu cơ.
5.1. Nghiên cứu cấu trúc hợp chất (2R,3S)-2,3-dichloro-1,4-dioxane dưới đây bằng phương pháp nhiễu xạ tia
X, người ta thấy độ dài của liên kết C-Cl trục (1.819 Å) lớn hơn của liên kết C-Cl biên (1.781 Å). Đồng thời, độ dài
của liên kết C-O của nguyên tử C mang liên kết C-Cl trục (1.394 Å) lại ngắn hơn liên kết C-O của nguyên tử C
mang liên kết C-Cl biên (1.425 Å). Giải thích các giá trị thực nghiệm này.

5.2. Hãy cho biết hợp chất nào dễ tham gia phản ứng SN1 nhất trong số ba hợp chất dưới đây?

.
5.3. Hãy giải thích sự biến đổi lực bazơ của các hợp chất dưới đây:
Trang 9/15
pKa 10,58 7,79 5,06
5.4. a) Cho biết sản phẩm của các phản ứng sau đây:
(i) (ii)

(iii) (iv)

b) Sản phẩm ở ý 1.2_(iv) là calicene. Momen lưỡng cực của calicene theo tính toán lý thuyết là 4,66 D; biết
rằng giá trị moment lưỡng cực của formaldehyde là 2,33 D, của acetone là 2,88 D. Giải thích độ lớn bất thường của
momen lưỡng cực của calicene.
c) Hãy viết phương trình phản ứng của calicene với:
c1) HBr;
c2) i. NaCN, ii. H3O+.
Hướng dẫn chấm
Ý Nội dung Điểm
5.1 Mật độ electron dịch chuyển vào obitan phản liên kết bằng hiệu ứng siêu liên hợp sẽ làm yếu 0,75
(và làm tăng độ dài) của liên kết tương ứng.
- Đối với liên kết trục C-Cl, obitan phản liên kết của liên kết C-Cl (σ*C-Cl) có sự xem phủ với
obitan không liên kết (nO) của nguyên tử oxy.
- Đối với liên kết biên C-Cl, σ*C-Cl có sự xem phủ của cặp e-n với obitan liên kết của liên kết
C-C (σC-C).

- Tương tác nO → σ*C-Cl mạnh hơn σC-C → σ*C-Cl (xem giản đồ năng lượng bên phải hình), mật
độ electron dịch chuyển vào σ*C-Cl của liên kết C-Cl trục nhiều hơn, do đó, liên kết C-Cl trục
có độ dài lớn hơn liên kết C-Cl biên.
- Cũng do sự tương tác nO → σ*C-Cl mạnh hơn σC-C → σ*C-Cl nên liên kết C-O của nguyên tử
C mang liên kết C-Cl trục (1.394 Å) có tính chất của liên kết đôi nhiều hơn và do đó ngắn
hơn liên kết C-O của nguyên tử C mang liên kết C-Cl biên (1.425 Å).
5.2 4-Chlorocyclobut-2-enone: 0,25

5.3 0,25

pKa = 10,58
Nguyên tử nitơ ở trạng thái lai hóa sp3. Mật độ e trên nguyên tử nitơ được tăng cường bởi ba
nhóm ankyl đẩy e. Mặt khác, do có cấu trúc vòng nên các nhóm ankyl không gây hiệu ứng
không gian đối với nguyên tử nitơ.

Trang 10/15
0,25

pKa = 7,79
Nguyên tử nitơ lai hóa sp3 (lưu ý: nguyên tử nitơ trong hợp chất này không thể lai hóa sp2
do nằm ở đỉnh của 2 vòng no. Mật độ e trên nguyên tử nitơ giảm do hiệu ứng cảm ứng hút e
từ các nguyên tử Csp2 vòng benzene.
0,25

pKa = 5,06
Nguyên tử nitơ ở trạng thái lai hóa sp2, có độ âm điện lớn hơn nitơ sp3. Mặt khác, mật độ e
trên nguyên tử nitơ giảm mạnh do hiệu ứng liên hợp âm (-C) của vòng benzene.
a) Cho biết sản phẩm của các phản ứng sau đây:
5.4 0,25
(i) (ii)

(iii) (iv)

b) Calicene tồn tại ở một dạng công thức cộng hưởng với hai vòng thơm mang điện tích.
Như vậy, calicene giống như một hợp chất lưỡng cực nên có momen lưỡng cực lớn.
0,25

c) Căn cứ vào sự phân cực trong phân tử calicene, có thể dự đoán sản phẩm trong hai phản
ứng là:
0,25

Câu 6 (2,5 điểm) Sơ đồ tổng hợp hữu cơ. Cơ chế phản ứng hóa hữu cơ.
6.1. Viết cơ chế giải thích sự tạo thành các sản phẩm ở mỗi phản ứng sau:

a.

b.

c.

Trang 11/15
d.

6.2. Hoàn thành sơ đồ tổng hợp các hợp chất trong các dãy phản ứng sau:
a.

b.

c.

Hướng dẫn chấm


Ý Nội dung Điể
m
6.1 0,25

a. 0,25
b.

Trang 12/15
0,25

0,25

c.

d.

6.2 a. 0,25

Trang 13/15
b. 0,25

0,5

c.

Câu 7 (2,5 điểm) Xác định cấu trúc các chất hữu cơ (mô tả sơ đồ tổng hợp bằng lời dẫn)
7.1. Xử lý một hỗn hợp gồm 2-nitrotoluen và isopropyl nitrit với MeONa/MeOH ở nhiệt độ thường, sau đó
đun nóng với hỗn hợp thu được với một lượng dư dung dịch HCl đặc thì thu được hợp chất A (C7H5NO2) từ hỗn
hợp phản ứng. Khi xử lý 2-nitrotoluen với CrO3 trong dung AcOH/Ac2O có mặt H2SO4 đặc ở 5 oC thu được hợp
chất B (C11H14NO6); Thủy phân B trong dung dịch HCl 10%/EtOH cũng tạo thành A nêu trên.Sục khí Cl2 vào dung
dịch 2-nitrotoluen trong CCl4 và đun hồi lưu hỗn hợp phản ứng người ta thu được C (C7H6NO2Cl); xử lý C với
dung dịch KOH/EtOH thu được hợp chất D (C14H10N2O4). Ozon hóa D rồi chế hóa ozonit tạo thành với (CH3)2S
cũng thu được A nêu trên. Hợp chất A còn được tạo thành trực tiếp từ C khi cho C tác dụng với DMSO/NaHCO3.
Hãy xác định công thức cấu tạo của các hợp chất từ A đến D nêu trên.
7.2. Cho Tropon (Tên hệ thống: xiclohepta-2,4,6-trienon) tác dụng với Cl2/CCl4 với tỉ lệ 1 : 1 thu được hợp chất
A (C7H6Cl2O). Trong môi trường phân cực, A chuyển thành hợp chất dạng muối hyđroclorua B có cùng công thức
phân tử. Xử lý B với dung dịch NaHCO3 thu được 2-clorotropon.
a. Hãy xác định cấu tạo của A, B và giải thích sự hình thành của chúng.
b. Khi chiếu sáng tropon người ta thu được một lượng nhỏ chất lỏng C (C6H6) và một chất khí C với số mol
bằng nhau. Hãy xác định cấu tạo và giải thích sự hình thành C và D.

Trang 14/15
7.3. Khi cho xiclohepta-2,4,6-trien tác dụng với PCl5 người ta thu được hợp chất E (C7H7Cl). Xử lý E với dung
dịch NaOH thì thu được hợp chất F (C17H14O). Trong môi trường axit HCl 20%, chất F dễ dàng chuyển hóa thành
tropon và xiclohepta-2,4,6-trien. Hãy xác định cấu tạo của E và F.
7.4. Tropolon (Tên hệ thống: 2-hyđroxyxiclohepta-2,4,6-trienon) là một dẫn xuất của tropon. Hợp chất này được
tổng hợp dễ dàng qua hai giai đoạn: Đầu tiên, phản ứng của xiclopenta-1,3-đien với 2,2-đicloaxetyl clorua
(Cl2CHCOCl) có mặt Et3N trong n-hexan ở 0 – 5 oC tạo ra sản phẩm trung gian G (C7H6Cl2O). Tiếp đó, xử lý G
với CH3COONa trong môi trường CH3COOH/H2O thì thu được tropolon. Xác định cấu tạo của G và giải thích sự
hình thành G và tropolon bằng cơ chế phản ứng.
Hướng dẫn chấm
Ý Nội dung Điểm
7.1 0,5

7.2 a. Giải thích sự hình thành 2-clotropon: 0,5

b. Giải thích sự hình thành benzen (C ) và khí CO (D): 0,5

7.3 0,5

Trang 15/15
7.4 0,5

Câu 8 (2,5điểm) Hóa học các hợp chất thiên nhiên (Cacbohidrat các hợp chất hữu cơ chứa nito đơn giản)
8.1. Porantherine là một alkaloid nhóm Euphorbaceae được phân lập từ cây bụi thân gỗ poranthera corymbosa
Brogn. Porantherine gây ngộ độc cho gia súc và được phát hiện lần lần đầu ở New South Wales và Queensland,
Australia. Theo công bố của Stevens, các liên kết C-C và C-N trong phân tử porantherine được tổng hợp đi qua
enol (hoặc enolate) và ion iminium (hoặc imine) dựa vào hiệu ứng siêu liên hợp của liên kết C-C đang hình thành
với cặp e-n trên nguyên tử nitơ. Hãy hoàn thành sơ đồ phản ứng tổng hợp porantherine dưới đây:

8.2.Pyrroloquinoline quinone ( PQQ ), còn được gọi là methoxatin , là một cofactor thực hiện vai trò trong quá
trình oxy hóa khử, tương tự NADH hay NADPH. Nó được tìm thấy trong đất và thực phẩm như quả kiwi, cũng như
sữa mẹ. Dưới đây là dãy tổng hợp methoxatin

. 8.3.Pelargonidin là một anthocyanidin , một loại sắc tố thực vật tạo ra màu cam đặc trưng, được sử dụng trong

Trang 16/15
thực phẩm và thuốc nhuộm công nghiệp. Pelargonidin có thể được tìm thấy trong các loại quả mọng như quả mâm
xôi chín và dâu tây. Dưới đây là 2 sơ đồ tổng hợp hóa học Pelargonidin

Hướng dẫn chấm


Ý Nội dung Điể
m
8. 1
1

Trang 17/15
8. 1
2

8. 0,5
3

------------- HẾT --------------


Giáo viên ra đề: Nguyễn Thị Xuân Vinh SĐT: 0977405123

Trang 18/15
KỲ THI CHỌN HỌC SINH GIỎI CÁC TRƯỜNG THPT CHUYÊN
KHU VỰC DUYÊN HẢI VÀ ĐỒNG BẰNG BẮC BỘ
LẦN THỨ XIV, NĂM 2023

ĐỀ THI MÔN: HÓA HỌC - LỚP 11


ĐỀ ĐỀ XUẤT Thời gian: 180 phút (Không kể thời gian giao đề)
Ngày thi: 15/7/2023
(Đề thi gồm 06 trang)
Câu 1. (2,0 điểm)
Phản ứng A: S2O82-(dd) + 3I-(dd) → 2SO42-(dd) + I3-(dd)
I3- hình thành phản ứng tức thời với axit ascorbic
Phản ứng B: I3-(dd) + C6H8O6(dd) → 3I-(dd) + C6H6O6(dd) + 2H+(dd)
Khi toàn bộ lượng axit ascorbic đã phản ứng hết, sẽ xảy ra phản ứng:
I3-(dd) + tinh bột → Phức màu xanh
Thời gian từ lúc trộn các chất phản ứng đến lúc xuất hiện màu xanh phụ thuộc vào lượng I3- sinh ra
trong phản ứng A. Ở 25oC, người ta trộn 5,0 mL dd C6H8O6 0,02 M, 5,0 mL dd hồ tinh bột với 25 mL
dd (NH4)2 S2O8 và 25 mL dd KI với các nồng độ khác nhau và thu được các giá trị của thời gian bắt
đầu xuất hiện màu xanh cho trong bảng dưới đây:
TN số [(NH4)2 S2O8]0 (mol/L) [KI]0 (mol/L) t(s)
1 0,200 0,200 20,5
2 0,100 0,200 41,0
3 0,050 0,200 82,0
4 0,200 0,100 41,0

Xem phản ứng B là tức thời thì thời gian xuất hiện phức màu xanh sẽ càng nhỏ nếu tốc độ phản ứng A
càng lớn. Vì thế tốc độ phản ứng tỉ lệ nghịch với t:
a) Tìm biểu thức của định luật tốc độ của phản ứng A.
b) Tính hằng số tốc độ của phản ứng A.
c) Người ta đề nghị cơ chế sau đây cho phản ứng A:
S2O82-(dd) + I-(dd) ⎯⎯ k
1
→ IS2O83-(dd)
IS2O83-(dd) ⎯⎯ k
2
→ 2SO42-(dd) + I+(dd)
I+(dd) + I-(dd) ⎯⎯k3
→ I2(dd)
I2(dd) + I-(dd) ⎯⎯→ I3-(dd)
k4

Hãy rút ra định luật tốc độ từ cơ chế trên khi chấp nhận sự gần đúng về trạng thái dừng đối với các
sản phẩm trung gian và nhận xét về sự phù hợp với thực nghiệm của cơ chế đó

Câu 2: (2,5 điểm) (Cân bằng và phản ứng trong dung dịch. Pin điện-Điện phân)
1. Pin sạc axit chì là một trong những loại pin phổ biến nhất được sử dụng trong xe hơi ở đầu thế kỷ
21. Nó có một số đặc điểm vượt trội và nó có thể được tái chế gần như hoàn toàn. Trong suốt quá trình
pin phóng điện thì điện cực chì và chì (IV) oxit chuyển thành điện cực chì sunfat. Axit sunfuric được
sử dụng như là chất điện phân.
a) Viết các quá trình hóa học xảy ra ở mỗi điện cực, phản ứng chung xảy ra khi pin phóng điện
và sơ đồ pin.
Cho: Eo 2+
Pb / Pb
= −0,126V; EoPbO / Pb2+
= 1, 455V; pKa(HSO− ) = 2,00; pKs(PbSO4 ) = 7,66;
2 4

Trang 1 / 23
RT
tại 25oC: 2,303 = 0,0592
F
o o
b) Tính E PbSO /Pb ; E PbO /PbSO và suất điện động của pin khi CH SO = 1,8M .
4 2 4 2 4

2. Điện phân dung dịch NaCl với Catot là hỗn hống Hg dòng chảy đều và dùng cực Titan bọc Ruteni
và Rođi làm Anot. Khoảng cách giữa Anot và Catot chỉ vài mm.
a. Viết phương trình phản ứng xảy ra tại điện cực khi mới bắt đầu điện phân pH = 7. Tính các giá trị
thế điện cực và thế phân giải.
b. Sau một thời gian, pH tăng lên đến giá trị pH = 11. Giải thích tại sao. Viết các phương trình xảy ra
tại pH đó. Tính thế điện cực và thế phân giải.
Cho biết : E oNa + / Na = −2, 71V ; E o2H O+ / H = 0, 00V ; E oO2 / H2O = 1, 23V . Với dung dịch NaCl 25% và
3 2

0,2% Na trong hỗn hống Na/Hg: E o


Na + / Na (Hg)
= −1, 78V . E oCl / Cl−
= 1,34V cho dung dịch NaCl 25% theo
2

khối lượng; H2 = 1,3V trên Hg; O2 = 0,8V trên Ru/Rd.

Câu 3: (2,5 điểm) Nhiệt động học và cân bằng hóa học
1. Tính ∆G0298 của phản ứng?
Pb2+ (dd) + HCO3- (dd) PbCO3 + H+(dd)
Cho biết ở 250C: T(PbCO3) = 7,4.10-14; pKa2(H2CO3) = 10,33.
2. Hai chất PbCO3 và ZnO dễ tác dụng với khí H2S có trong không khí theo các phản ứng sau đây?
PbCO3 (r) + H2S (k) ⎯→ PbS (r) + CO2 (k) + H2O (h) (1)
ZnO (r) + H2S (k) ⎯→ ZnS (r) + H2O (h) (2)
a) Tính hằng số cân bằng của các phản ứng (1) và (2).
b) Cần khống chế nồng độ tối đa của H2S trong không khí bằng bao nhiêu g/m3 để các bột màu nói
trên không bị hư hại?
c) Trong 2 chất màu nói trên, chất nào ưu thế hơn khi môi trường có H2S, tại sao?
d) Bằng cách xử lí với dung dịch H2O2, có thể làm trắng lại các mảng bị đổi màu do sự hình thành
PbS. Viết phương trình của phản ứng xảy ra trong cách xử lí này.
e) Hãy chứng tỏ rằng, về mặt nhiệt động học, oxi của không khí có thể thay thế H2O2 trong phương
pháp xử lí trên.
g) Trong thực tế, ngay cả khi không khí chưa bị ô nhiễm nặng, chẳng hạn p(H2S) = 5,1.10-9 atm, mầu
trắng của PbCO3 để lâu trong không khí vẫn bị xám dần đi do sự hình thành PbS. Hiện tượng này có
thể giải thích như thế nào?
Để tính toán có thể sử dụng các dữ kiện và bảng sau: T= 298K; áp suất khí quyển p = 1,000 atm;
% thể tích của các khí và hơi trong không khí: N2 77,90; O2 20,70; CO2 0,026; H2O (h) 0,40; các khí khác: 1,03.

PbCO3(r) H2S(k) PbS(r) ZnO(r) ZnS(r) CO2(k) H2O(h) PbSO4(r) H2O2(l)


ΔfG°298
- 626,0 - 33,0 - 92,6 - 318,0 - 184,8 - 394,2 - 228,5 - 811,5 120,4
kJ/mol
Màu trắng đen trắng trắng trắng

Trang 2 / 23
Câu 4: (2,5 điểm) Hóa nguyên tố
Một số muối trung hòa chứa Na và N (hoặc P) với phần trăm khối lượng các nguyên tố được cho trong
bảng dưới đây. Trạng thái oxi hóa của N và P nằm trên cùng một hàng là như nhau.
Muối chứa N Muối chứa P
Muối % Na %N Muối % Na %P
A 46.9 9.5 E 42.1 18.9
B 27.1 16.5 F 22.5 30.4
C 33.3 20.3 G 36.5 24.6
D 43.4 26.4 H 26.1 35.2
D và H là muối của axit tương ứng I và J. Cả hai axit này đều là chất rắn màu trắng, dễ bị phân hủy
bởi nhiệt. I phân hủy nhiệt theo một giai đoạn trong khi đó J qua hai giai đoạn như sau

1. Xác định công thức phân tử từ A – H và vẽ công thức cấu tạo của chúng.
2. Xác định công thức phân tử của I, J, K, L, M, Q và viết phương trình phân hủy nhiệt của I và J.
3. Vẽ công thức cấu tạo của K. Đề nghị các chất khác cho sản phẩm phân hủy giống sản phẩm phân
hủy của I và viết phương trình phản ứng tương ứng.

Câu 5. (2,5 điểm) Đại cương hữu cơ


1.
a) So sánh tính acid của hiđro linh động trong các hợp chất sau

b) So sánh độ bền của các hợp chất A1, A2, A3, A4, A5 (sắp xếp theo giá trị │ΔHcháy│tăng dần) và giải
thích ngắn gọn.

2.
a) N2F2 có thể tồn tại hai đồng phân cis hoặc trans:

Đồng phân nào bền hơn, giải thích sự lựa chọn của bạn.
b) Một hợp chất khác là hydrazine cũng chứa liên kết N-N. Hydrazine tồn tại chủ yếu 2 cấu dạng là
anti và gauche. Hãy vẽ 2 cấu dạng gauche và anti. Cấu dạng nào có năng lượng thấp hơn, giải thích?
3. Phản ứng thế nucleophile giữa một alcohol với một nucleophile chứa nitrogen thông thường khó xảy
ra. Tuy nhiên, 5-methyldibenzosulberenol (1) có khả năng thực hiện được phản ứng thế hoặc tách với
các nucleophile có chứa nitrogen (A – D) trong cùng một điều kiện ở nhiệt độ phòng.

Trang 3 / 23
a) Hãy so sánh tính bazơ của các chất sau: PhCH2NH2 (A), NH2OH (B), NH2NH2 (C) và CO(NH2)2
(D). Không cần giải thích.
b) Vì sao chất 1 có khả năng cho phản ứng rất nhanh trong môi trường acid ở nhiệt độ phòng?
c) Trong các chất A, B, C và D, chỉ có hai chất có khả năng phản ứng thế với chất 1 (theo điều kiện ở
đầu bài) để tạo thành chất 2, trong khi hai chất còn lại thì chỉ cho duy nhất sản phẩm tách 3. Hãy xác
định hai chất cho phản ứng thế và giải thích vì sao hai chất còn lại không thể tham gia phản ứng thế.
d) Khi thực hiện phản ứng từ 1 sang 2 với tác nhân nucleophile phù hợp, nếu sử dụng acid
trifluoroacetic (CF3COOH) và acid acetic (CH3COOH) thay cho acid dichloroacetic, không có acid
nào cho sản phẩm thế mong muốn. Hãy giải thích hiện tượng trên.

Câu 6. (2,5 điểm) Sơ đồ tổng hợp hữu cơ. Cơ chế phản ứng hóa hữu cơ.
1. Hãy đề nghị cơ chế giải thích sự hình thành sản phẩm sau:

b)

2. Lithospermic acid được Johnson và cộng sự phân lập từ một loại thảo dược Trung Hoa với
nhiều đặc tính sinh học quan trọng. Sơ đồ dưới đây biểu diễn một phần chuỗi tổng hợp toàn phần
Lithospermic acid.

Trang 4 / 23
a. Vẽ cấu trúc của chất số 2, 3, 4, 5 và 7 biết chất số 7 có phân tử khối M = 30 gam/mol.

b. Chọn đáp án đúng cho điều kiện của chuyển hóa số 8:


A. Ce(NH4)2(NO3)6
B. CrO3.(Pyridine)2
C. H2CrO4/H2O
D. CH3COCH3, Al[OiPr]3
c. Vẽ cấu trúc của chất số 10.

Câu 7. (2,5 điểm) Xác định cấu trúc các chất hữu cơ.
1. Cho hợp chất A (C6H8O2, chỉ có một loại hiđro) tác dụng với một đương lượng chất B (C2H6O2, có
2 loại hiđro với tỉ lệ 2:1) trong môi trường acid thì thu được chất C (C8H12O3). Xử lý C với một đương
lượng hợp chất Grignard D (RMgBr) thì thu được hợp chất E (C10H16O3, phổ 1H-NMR của E được
trình bày bên dưới). Đun nóng E trong hỗn hợp acetone/nước với xúc tác acid trong 2 tiếng thì thu
được sản phẩm F (C8H12O2).
1
H NMR (500 MHz, CDCl3) δ (E) 5.97 (dd, J = 17.4, 10.8 Hz, 1H), 5.28 (dd, J = 17.3, 1.4 Hz, 1H),
5.04 (dd, J = 10.7, 1.4 Hz, 1H), 3.99 – 3.88 (m, 4H), 1.95 (td, J = 12.6, 4.2 Hz, 2H), 1.77 (td, J = 13.0,
4.0 Hz, 2H), 1.69 – 1.62 (m, 2H), 1.62 – 1.54 (m, 2H).
a) Xác định công thức cấu tạo của các chất từ A đến F.
b) Một nhóm nghiên cứu ở trong trường ĐH X không có sẵn máy cộng hưởng từ hạt nhân (NMR) mà
chỉ có máy phổ hồng ngoại (IR). Bằng thiết bị nghiên cứu sẵn có, làm thế nào để biết phản ứng tạo
thành chất E đã xảy ra hoàn toàn (không còn nguyên liệu C trong hỗn hợp phản ứng)?
c) Cho biết số lượng tín hiệu proton mà chất F cho trên phổ 1H-NMR? Biểu diễn cấu trúc phân tử để
chỉ rõ các tín hiệu proton và giải thích.

2. Gemifloxacin (C18H20N5O4F) là một loại thuốc kháng sinh để chữa bệnh viêm
phế quản và được tổng hợp theo các giai đoạn sau:
- Giai đoạn 1: Thủy phân X trong dung dịch H2SO4, thu được 47. Cho 47 phản
ứng với SOCl2 và sau đó với NaCH(COOEt)2, thu được 48. Đun 48 trong dung
dịch TsOH ở 140oC, thu được 49. Xử lí 49 với HC(OEt)3 trong Ac2O, tạo ra 50.
Cho 50 phản ứng với xiclopropyl amin trong EtOH, thu được 51
(C14H13N2O3Cl2F). Thực hiện phản ứng đóng vòng nội phân tử với 51 trong t-BuOK và đioxan, thu
được 52 (C12H8N2O3ClF).
- Giai đoạn 2: Cho etyl 2-aminoetanoat phản ứng với acrilonitrin có mặt KOH trong nước, thu được 53
(C7H12N2O2). Bảo vệ nhóm amin bằng cách cho 53 tác dụng với Boc2O trong CHCl3, thu được 54. Khi
có mặt EtONa trong EtOH, 54 sẽ ngưng tụ nội phân tử để tạo ra 55. Khử 55 bằng lượng dư LiAlH4,
tạo ra 56. Tiếp tục bảo vệ nhóm amin mới tạo thành bằng cách cho 56 tác dụng với Boc2O trong hỗn
hợp đioxan và nước, thu được 57. Oxi hóa 57 bằng PCC rồi cho tác dụng tiếp với MeONH2.HCl có
mặt NaHCO3 trong EtOH và THF, thu được 58. Xử lí 58 với AcCl trong MeOH để gỡ bỏ các nhóm
bảo vệ, thu được 59 (C6H13N3O).
- Giai đoạn 3: Cho 52 phản ứng với 59 trong CH3CN và có mặt DBU, sẽ thu được gemifloxacin.

Trang 5 / 23
Biết trong phân tử gemifloxacin có một nhóm amino và một nhóm cacboxyl. Vẽ công thức cấu tạo của
các chất từ 47 đến 59 và gemifloxacin.
Câu 8. (2,5 điểm) Hóa học các hợp chất thiên nhiên
1. D-andotetrozơ A khi phản ứng với nitric acid cho hợp chất không hoạt động quang học. Cũng
andotetrozơ này khi phản ứng với HCN, tiếp theo với dung dịch nước Ba(OH)2 cho hai axit andonic
epime B và C. Các andonic acid này nằm trong cân bằng với các -andolacton D và E tương ứng của
chúng. Xử lý hỗn hợp này với Na - Hg và nước ở pH 3-5 thu được các chất F và G tương ứng. Oxi hóa
F bằng nitric acid thu được andaric acid không hoạt động quang học H, trong khi thực hiên phản ứng
này với E thu được andaric acid hoạt động quang học I. Cho biết cấu trúc các chất từ A đến I.
2. Tiến hành cắt mạch một nonapeptit bằng enzyme dipeptidylamine peptidase (cắt tuần tự 2
aminoaxit tính từ đầu N) thu được hỗn hợp các dipeptit Gly – Phe; Leu – Arg; Tyr – Gly;
Lys – Tyr và Pro. Còn nếu cắt mạch nonapeptit bằng enzyme dipeptidylcarboxipeptidase (cắt tuần tự 2
aminoaxit tính từ đầu C) được hỗn hợp các dipeptit: Phe – Leu; Gly – Gly; Arg – Lys; Tyr – Pro và
Tyr.
a. Xác định trật tự các aminoaxit trong nonapeptit trên.
b. Trong số các dipeptit tạo thành thì dipeptit nào không quang hoạt? Dipeptit nào có điểm đẳng điện
lớn hơn 7? Và dipeptit nào khó bị thủy phân cho ra aminoaxit nhất?
c. Một decapeptit có hoạt tính tương tự aminoaxit đang xét có thể được tổng hợp bằng cách ghép thêm
Lys vào sau Pro. Hãy đề nghị phương pháp tổng hợp peptit này từ nonapeptit ban đầu. Yêu cầu không
quá 5 giai đoạn.
d. Tiến hành cắt mạch decapeptit mới tổng hợp bằng dipeptidylamine peptidase và dipeptidylcarboxi
peptidase sẽ thu được tổng cộng bao nhiêu dipeptit?

-------------- HẾT --------------

(Thí sinh không được sử dụng tài liệu. Cán bộ coi thi không giải thích gì thêm)

Họ và tên thí sinh: .................................................................. Số báo danh: .................................

Trang 6 / 23
KỲ THI CHỌN HỌC SINH GIỎI CÁC TRƯỜNG THPT CHUYÊN
KHU VỰC DUYÊN HẢI VÀ ĐỒNG BẰNG BẮC BỘ
LẦN THỨ XIV, NĂM 2023

(HDC gồm 17 trang) HƯỚNG DẪN CHẤM ĐỀ ĐỀ XUẤT MÔN: HÓA HỌC - LỚP 11

Câu 1: (2,5 điểm) (Động hóa học)


Phản ứng A: S2O82-(dd) + 3I-(dd) → 2SO42-(dd) + I3-(dd)
I3 hình thành phản ứng tức thời với axit ascorbic
-

Phản ứng B: I3-(dd) + C6H8O6(dd) → 3I-(dd) + C6H6O6(dd) + 2H+(dd)


Khi toàn bộ lượng axit ascorbic đã phản ứng hết, sẽ xảy ra phản ứng:
I3-(dd) + tinh bột → Phức màu xanh
Thời gian từ lúc trộn các chất phản ứng đến lúc xuất hiện màu xanh phụ thuộc vào lượng I3- sinh ra
trong phản ứng A. Ở 25oC, người ta trộn 5,0 mL dd C6H8O6 0,02 M, 5,0 mL dd hồ tinh bột với 25 mL
dd (NH4)2 S2O8 và 25 mL dd KI với các nồng độ khác nhau và thu được các giá trị của thời gian bắt
đầu xuất hiện màu xanh cho trong bảng dưới đây:
TN số [(NH4)2 S2O8]0 (mol/L) [KI]0 (mol/L) t(s)
1 0,200 0,200 20,5
2 0,100 0,200 41,0
3 0,050 0,200 82,0
4 0,200 0,100 41,0

Xem phản ứng B là tức thời thì thời gian xuất hiện phức màu xanh sẽ càng nhỏ nếu tốc độ phản ứng A
càng lớn. Vì thế tốc độ phản ứng tỉ lệ nghịch với t:
a) Tìm biểu thức của định luật tốc độ của phản ứng A.
b) Tính hằng số tốc độ của phản ứng A.
c) Người ta đề nghị cơ chế sau đây cho phản ứng A:
S2O82-(dd) + I-(dd) ⎯⎯ k
1
→ IS2O83-(dd)
IS2O83-(dd) ⎯⎯ k
2
→ 2SO42-(dd) + I+(dd)
I+(dd) + I-(dd) ⎯⎯k3
→ I2(dd)
I2(dd) + I-(dd) ⎯⎯→ I3-(dd)
k4

Hãy rút ra định luật tốc độ từ cơ chế trên khi chấp nhận sự gần đúng về trạng thái dừng đối với các
sản phẩm trung gian và nhận xét về sự phù hợp với thực nghiệm của cơ chế đó

Câu ý Nội dung điểm


1 a a) 0,75
v = k[(NH4)2 S2O8]a[KI]b = α/t
1
v1 t1 t2 41, 0 k .(0, 200) a (0, 200)b
= = = = →a=1
v2 1 t1 20,5 k .(0,100)a (0, 200)b
t2

Trang 7 / 23
v4 t 20,5 k .(0, 200) (0,100)b
== 1 = = →b=1
v1 t4 41, 0 k .(0, 200) (0, 200)b
Biểu thức của định luật tốc độ: v = k[(NH4)2 S2O8][KI]

b Tính tốc độ đầu của phản ứng A 0,75


Lượng I3- sinh ra cho tới lúc bắt đầu xuất hiện màu xanh chính là lượng I3-
phản ứng hết với axit ascorbic trong phản ứng B.
n(I3-) = n(C6H8O6) = 5.10-3x0,02 mol = 0,1 mmol
Lượng I- tiêu thụ cho phản ứng A:
0,1mmolx3 = 0,3 mmol
1 d[I- ] 1 [I- ] 0,1.1000
v=- = = = 8,1.10-5 M.s-1
3 dt 3 t 60.20,5
8,1.10-5 M.s-1 = k(0,2x25/60) (0,2x25/60) = k.6,94.10-3 M2
→ k= 8,1. 10-5 M.s-1 / 6,94.10-3 M2 = 1,17.10-2 M-1.s-1

c d. v = tốc độ hình thành sản phẩm I3- = k4[I2][ I-] (1)


d [I 2 ]
= k3[I+][ I-] – k4[I2][ I-] = 0 (2)
dt
d [I + ]
= k2[IS2O83-] - k3[I+][ I-] = 0 (3) 0,5
dt
d [IS2 O83- ]
= k1[S2O82-][ I-] - k2[IS2O83-] = 0 (4)
dt
(2) + (3) → [I2] = k2[IS2O83-]/ k4[ I-] (5)
Từ (4) → [IS2O8 ] = k1[S2O8 ][ I-]/ k2
3- 2-
(6)
Thay (6) vào (5) → [I2] = k1[S2O8 ]/ k4 2-
(7)
Thay (7) vào (1) → v = k4[I2][ I-] = k1[S2O8 ][ I-] (8)
2-

Cơ chế đề nghị phù hợp với định luật động học thực nghiệm
0,5
k = A.e-Ea/RT → k và A có cùng đơn vị
∆G0 = -RTlnK → K =e-∆G0/RT

Trang 8 / 23
Câu 2: (2,5 điểm) (Cân bằng và phản ứng trong dung dịch. Pin điện-Điện phân)
1. Pin sạc axit chì là một trong những loại pin phổ biến nhất được sử dụng trong xe hơi ở đầu thế kỷ
21. Nó có một số đặc điểm vượt trội và nó có thể được tái chế gần như hoàn toàn. Trong suốt quá trình
pin phóng điện thì điện cực chì và chì (IV) oxit chuyển thành điện cực chì sunfat. Axit sunfuric được
sử dụng như là chất điện phân.
a) Viết các quá trình hóa học xảy ra ở mỗi điện cực, phản ứng chung xảy ra khi pin phóng điện
và sơ đồ pin.
Cho: Eo 2+
Pb / Pb
= −0,126V; EoPbO / Pb2+
= 1, 455V; pKa(HSO− ) = 2,00; pKs(PbSO4 ) = 7,66;
2 4

RT
tại 25oC: 2,303 = 0,0592
F
o o
b) Tính E PbSO /Pb ; E PbO /PbSO và suất điện động của pin khi CH SO = 1,8M .
4 2 4 2 4

2. Điện phân dung dịch NaCl với Catot là hỗn hống Hg dòng chảy đều và dùng cực Titan bọc Ruteni
và Rođi làm Anot. Khoảng cách giữa Anot và Catot chỉ vài mm.
a. Viết phương trình phản ứng xảy ra tại điện cực khi mới bắt đầu điện phân pH = 7. Tính các giá trị
thế điện cực và thế phân giải.
b. Sau một thời gian, pH tăng lên đến giá trị pH = 11. Giải thích tại sao. Viết các phương trình xảy ra
tại pH đó. Tính thế điện cực và thế phân giải.
Cho biết : E oNa + / Na = −2, 71V ; E o2H O+ / H = 0, 00V ; E oO2 / H2O = 1, 23V . Với dung dịch NaCl 25% và
3 2

0,2% Na trong hỗn hống Na/Hg: E o


Na + / Na (Hg)
= −1, 78V . E oCl / Cl−
= 1,34V cho dung dịch NaCl 25% theo
2

khối lượng; H2 = 1,3V trên Hg; O2 = 0,8V trên Ru/Rd.

HDG
câu ý Nội dung điểm
2 1 a)
2(1,455)
Catot: PbO2 + 4H+ + 2e → Pb2+ + 2H2O K1 = 10 0,0592
HSO4– → SO42− + H+ K2 = 10–2
Pb2+ + SO42– → PbSO4 K3 = 107,66
Quá trình khử tại catot:
PbO2 + HSO4– + 3H+ + 2e → PbSO4 + 2H2O Kcatot = K1.K2.K3 (*)
−2( −0,126)
Anot: Pb → Pb2+ + 2e K ’= 10 0,0592
1

HSO4 SO42– +H +
K2’ = 10–2
Pb2+ + SO42– → PbSO4 K3’ = 107,66
Quá trình oxh tại anot:
Pb + HSO4– → PbSO4 + H+ + 2e Kanot = K1’.K2’.K3’ (**)
Phản ứng chung khi pin phóng điện:
PbO2 + Pb + 2HSO4– + 2H+ → 2PbSO4 + 2H2O (***)
Sơ đồ pin:
(–) Pb│PbSO4, H+, HSO4–│PbO2 (Pb) (+) 0,75

Trang 9 / 23
b)
2E oPbO2 /PbSO4 2(1,455)
(*): 10 0,0592 = Kcatot = 10 0,0592 10−2107,66
o
 E PbO /PbSO = 1,62 V
2 4
Theo (**):
−2E oPbSO4/Pb −2( −0,126)
10 0,0592 = Kanot = 10 0,0592 10–2107,66
o
 E PbSO /Pb = – 0,29 V
4
Theo (***):
Epin = E(c) – E(a)
o o 0, 0592 − 2 + 2
= E PbO /PbSO – E PbSO /Pb + lg[HSO 4 ] [H ]
2 4 4 2

Trong đó [HSO4 ], [H ] được tính từ cân bằng sau:
+

− 2−
HSO 4 H+ + SO4 Ka = 10−2
[ ] 1,8 – x 1,8 + x x 0,75
2– –3
[SO4 ] = x = 9,89×10 M
 [H+] = 1,81 M; [HSO4–] = 1,79 M
0, 0592 2 2
Epin = 1,62 + 0,29 + lg(1, 79) (1,81) = 1,94 V
2
2 a. Trong dung dịch NaCl có: NaCl ⎯⎯
→ Na++Cl-;
⎯⎯
→ H3O + + OH-
2H2O ⎯

Khi điện phân có thể có các quá trình sau xảy ra:
Catode: Na+ + Hg + e → Na(Hg) 1 (1)
+ -
H2 O → H + OH 2
+
2 H + 2e → H2 1
-
2H2O + 2e → H2 + OH (2)
Anode: 2 H2 O → O2 + 4H+ + 4e (3)
2 Cl- → Cl2 + 2e (4)
E Na + / Na (Hg) = −1, 78V , E 2H O+ / H = 0, 00V + 0, 0592 lg10−7 = −0, 413V
3 2

E '
2H3O+ / H 2
=E o
2H3O+ / H 2
+ H2 = −1, 713 . Do E '2H O+ / H  E oNa + / Na (Hg) nên khi mới bắt
3 2

đầu điện phân, ở Katode quá trình (2) sẽ xảy ra, có H2 thoát ở Anode
Ở Anode: Từ (3) ta có:
+
EO2 / H2O = E o
O2 / H2O + 0,0592lg[H3O ] = 0,817V ; E '
O2 / H2O = EO2 / H2O + O2 = 1,617V 0,5
Bởi vì: E Cl / 2Cl−
 E 'O2 / H2O nên ở Anode xảy ra quá trình (4) và có Cl2 bay ra
2

Phương trình điện phân: 2Cl- + 2H2O → H2


b. Sau một thời gian, do [OH-] tăng nên pH cũng tăng. Khi pH = 11, phản ứng điện
phân xảy ra như sau: Tại Catode: [H+] =10-11.
E 2H O+ / H = −0, 649V ; E '2H O+ / H = −1,949V  E oNa + / Na nên ở Anode có quá trình (1)
3 2 3 2

xảy ra
Trang 10 / 23
+
Tại Anode: EO2 / H2O = EO2 / H2O + 0,0592lg[H3O ] = 0,581V ; EO2 / H2O = 1,381V
o '

Do E Cl / 2Cl−
 E 'O2 / H2O nên ở Anode vẫn có Cl2 bay ra
2

Phương trình điện phân: 2Na+ + 2Cl- + 2Hg → Cl2 + 2Na(Hg)


Thế phân giải: V = E − E = 3,12V 0,5
' '
A K

Câu 3: (2,5 điểm) Nhiệt động học và cân bằng hóa học
1. Tính ∆G0298 của phản ứng?
Pb2+ (dd) + HCO3- (dd) PbCO3 + H+(dd)
Cho biết ở 250C: T(PbCO3) = 7,4.10-14; pKa2(H2CO3) = 10,33.
2. Hai chất PbCO3 và ZnO dễ tác dụng với khí H2S có trong không khí theo các phản ứng sau đây?
PbCO3 (r) + H2S (k) ⎯→ PbS (r) + CO2 (k) + H2O (h) (1)
ZnO (r) + H2S (k) ⎯→ ZnS (r) + H2O (h) (2)
a) Tính hằng số cân bằng của các phản ứng (1) và (2).
b) Cần khống chế nồng độ tối đa của H2S trong không khí bằng bao nhiêu g/m3 để các bột màu nói
trên không bị hư hại?
c) Trong 2 chất màu nói trên, chất nào ưu thế hơn khi môi trường có H2S, tại sao?
d) Bằng cách xử lí với dung dịch H2O2, có thể làm trắng lại các mảng bị đổi màu do sự hình thành
PbS. Viết phương trình của phản ứng xảy ra trong cách xử lí này.
e) Hãy chứng tỏ rằng, về mặt nhiệt động học, oxi của không khí có thể thay thế H2O2 trong phương
pháp xử lí trên.
g) Trong thực tế, ngay cả khi không khí chưa bị ô nhiễm nặng, chẳng hạn p(H2S) = 5,1.10-9 atm, mầu
trắng của PbCO3 để lâu trong không khí vẫn bị xám dần đi do sự hình thành PbS. Hiện tượng này có
thể giải thích như thế nào?
Để tính toán có thể sử dụng các dữ kiện và bảng sau: T= 298K; áp suất khí quyển p = 1,000 atm;
% thể tích của các khí và hơi trong không khí: N2 77,90; O2 20,70; CO2 0,026; H2O (h) 0,40; các khí khác: 1,03.
PbCO3(r) H2S(k) PbS(r) ZnO(r) ZnS(r) CO2(k) H2O(h) PbSO4(r) H2O2(l)
ΔfG°298
- 626,0 - 33,0 - 92,6 - 318,0 - 184,8 - 394,2 - 228,5 - 811,5 120,4
kJ/mol
Màu trắng đen trắng trắng trắng

HDG
câu ý Nội dung điểm
3 1 [H ] +
[H ][CO ] +
K 2- 0,5
1. K= 2+ -
= 2+ - 2-
= a2 = 632
3

[Pb ][HCO3 ] [Pb ][HCO3 ][CO3 ] T


Suy ra ∆G = -RTlnK = -15977,6 J

Trang 11 / 23
2 2.
a) Đối với phản ứng (1)
ΔG°(1) = (-92.6 – 394.2 – 228.5 + 626.0 + 33.0) kJ/mol = -56,3 kJ/mol
K(1) = e- ΔG°(1)/RT = e56300/8,314.298 = 7,4.109.
Đối với phản ứng (2)
ΔG°(2)=(-184.8 -228.5 + 318.0 + 33.0) kJ/mol = - 62,3 kJ/mol
- ΔG°(2)/RT 62300/8,314.298 10
K(2) = e =e = 8,3.10 0,5
b) Đối với phản ứng (1)
2.6  10−4  4 10−3
ΔG(1)= -RTlnK(1)+ RT.ln
pH 2 S
Điều kiện để (1) ưu thế theo chiều thuận:
2.6  10−4  4 10−3
ΔG(1) =-RTlnK(1) + RT.ln <0 (a)
pH 2 S
2.6  10−4  4 10−3
→ pH2S > 9
= 1,4.10-16 bar (b)
7, 4.10
0,5
Để bảo vệ được mầu trắng PbCO3 thì nồng độ H2S được phép trong không khí tối
đa là:
34.(1,4.10-16.1000 L)/(0,082 L.bar.mol-1.K-1.298K) = 1,9.10-13 g/m3
Đối với phản ứng (2)
4 10−3
ΔG(2) = - RTlnK(2) + RT.ln
pH 2 S
Điều kiện để (2) ưu thế theo chiều thuận:
4 10−3
ΔG(2) = - RTlnK(2) + RT.ln <0 (c)
pH 2 S
4 10−3
→ pH2S > 10
= 4,8.10-14 bar
8,3.10
Để bảo vệ được mầu trắng ZnO thì nồng độ H2S được phép trong không khí tối đa
là: 34.(4,8.10-14.1000 L)/(0,082 L.bar.mol-1.K-1.298K) =
6,7.10-11 g/m3
c) ZnO ưu thế hơn vì: 0,25
- Phản ứng (1) Tự diễn biến ở những nồng độ H2S nhỏ hơn;
- Sản phẩm của (1) là PbS có mầu đen còn sản phẩm của (2) là ZnS vẫn còn là mầu
trắng.
d) PbS + 4H2O2 ⎯→ PbSO4 + H2O (3)
e) PbS + 2 O2 ⎯→ PbSO4 (4) 0,25
ΔG° = -811.5 kJ/mol + 92.6 kJ/mol = - 718.9 kJ/mol
1
ΔG = - 718.9 kJ/mol + RT.ln = - 711,1 kJ/mol
0.207 2
Phản ứng (4) có thể tự diển ra trong không khí ở nhiệt độ 298 K. Oxi của không khí
có thể tái tạo màu trắng bằng cách oxi hóa PbS ⎯→ PbSO4.
g) Với p(H2S) = 5.1.10-9 bar thì
2.6  10−4  4 10−3
ΔG(1) = -56,3 kJ/mol + RT∙ln  -43 kJ/mol.
5,1.10−9

Trang 12 / 23
Trong không khí xảy ra đồng thời 2 quá trình: tạo ra và làm mất PbS. 0,25
k1
PbCO3 (r) + H2S (k) PbS + ...
O2
PbSO4
k2
Xét về phương diện nhiệt động học thì sự oxi hóa PbS bởi oxi không khí thuận lợi
hơn rất nhiều. Sự đổi màu của PbCO3 có thể là do phản ứng oxi hóa PbS bởi oxi
không khí bị cản trở động học.
0,25

Câu 4: (2,5 điểm) Hóa nguyên tố


Một số muối trung hòa chứa Na và N (hoặc P) với phần trăm khối lượng các nguyên tố được cho trong
bảng dưới đây. Trạng thái oxi hóa của N và P nằm trên cùng một hàng là như nhau.
Muối chứa N Muối chứa P
Muối % Na %N Muối % Na %P
A 46.9 9.5 E 42.1 18.9
B 27.1 16.5 F 22.5 30.4
C 33.3 20.3 G 36.5 24.6
D 43.4 26.4 H 26.1 35.2
D và H là muối của axit tương ứng I và J. Cả hai axit này đều là chất rắn màu trắng, dễ bị phân hủy
bởi nhiệt. I phân hủy nhiệt theo một giai đoạn trong khi đó J qua hai giai đoạn như sau

1. Xác định công thức phân tử từ A – H và vẽ công thức cấu tạo của chúng.
2. Xác định công thức phân tử của I, J, K, L, M, Q và viết phương trình phân hủy nhiệt của I và J.
3. Vẽ công thức cấu tạo của K. Đề nghị các chất khác cho sản phẩm phân hủy giống sản phẩm phân
hủy của I và viết phương trình phản ứng tương ứng.

Câu 4:
Ta thấy tổng phần trăm khối lượng của các nguyên tố trong các muối A-H
nhỏ hơn 100%. Do đó nguyên tố còn lại là oxi. Từ phần trăm khối lượng của
các nguyên tố đã biết dễ dàng tính được %O. Vậy công thức từng muối
tương ứng:
A: Na : N : O = 46.9/23 : 9.5/14 : 43.6/16
= 2.039 : 0.678 : 2.725 = 3:1:4 → A = Na3NO4
B: Na : N : O = 27.1/23 : 16.5/14 : 56.4/16
= 1.178 : 1.178 : 3.525 = 1:1:3 → B = NaNO3 0,5
1
С: Na : N : O = 33.3/23 : 20.3/14 : 46.4/16
= 1.448 : 1.45 : 2.9 = 1:1:2 → C = NaNO2
D: Na : N : O = 43.4/23 : 26.4/14 : 30.2/16
= 1.887 : 1.886 : 1.888 = 1:1:1
→ D = (NaNO)n tương ứng với Na2N2O2 0,25
E: Na : P : O = 42.1/23 : 18.9/31 : 39/16
= 1.83 : 0.61 : 2.44 = 3:1:4 → E = Na3PO4
F: Na : P : O = 22.5/23 : 30.4/31 : 47.1/16
Trang 13 / 23
= 0.978 : 0.98 : 2.944 = 1:1:3 → F = NaPO3 0,25
G: Na : P : O = 36.5/23 : 24.6/31 : 38.9/16
= 1.587 : 0.794 : 2.431 = 1.999:1:3.062
Nếu G là Na2PO3 ứng với trạng thái oxi hóa của P là +4, chứ không phải +3.
Do đó có thể dự đóan G chứa hidro, và công thức tương ứng là
G = Na2HPO3.
H: Na : P : O = 26.1/23 : 35.2/31 : 38.7/16
= 1.135 : 1.135 : 2.419 = 1:1:2, tương tự như vậy H sẽ có công thức H =
NaH2PO2.
Chú ý hóa trị tối đa của N là 4 và P là 5 nên ta có thể vẽ công thức cấu tạo
của các anion được tạo ra từ các muối tương ứng như sau: 0,25

0,5

Các axit tương ứng của 2 muối Na2N2O2 và NaH2PO2 là H2N2O2 và H3PO2
Sự phân hủy của các muối:
2 0,5
Do đó: I = H2N2O2, J = H3PO2, K = N2O, L = H3PO3,
M = H3PO4, Q = PH3
Công thức cấu tạo của N2O

3 0,25
Các chất phân hủy cho N2O và H2O là:

Câu 5. (2,5 điểm) Đại cương hữu cơ


1.
a) So sánh tính acid của hiđro linh động trong các hợp chất sau

b) So sánh độ bền của các hợp chất A1, A2, A3, A4, A5 (sắp xếp theo giá trị │ΔHcháy│tăng dần) và giải
thích ngắn gọn.

2.
E. a) N2F2 có thể tồn tại hai đồng phân cis hoặc trans:

Trang 14 / 23
F.
Đồng phân nào bền hơn, giải thích sự lựa chọn của bạn.
b) Một hợp chất khác là hydrazine cũng chứa liên kết N-N. Hydrazine tồn tại chủ yếu 2 cấu dạng là
anti và gauche. Hãy vẽ 2 cấu dạng gauche và anti. Cấu dạng nào có năng lượng thấp hơn, giải thích?
3. Phản ứng thế nucleophile giữa một alcohol với một nucleophile chứa nitrogen thông thường khó xảy
ra. Tuy nhiên, 5-methyldibenzosulberenol (1) có khả năng thực hiện được phản ứng thế hoặc tách với
các nucleophile có chứa nitrogen (A – D) trong cùng một điều kiện ở nhiệt độ phòng.

G. a) Hãy so sánh tính bazơ của các chất sau: PhCH2NH2 (A), NH2OH (B), NH2NH2 (C) và
CO(NH2)2 (D). Không cần giải thích.
H. b) Vì sao chất 1 có khả năng cho phản ứng rất nhanh trong môi trường acid ở nhiệt độ
phòng?
I. c) Trong các chất A, B, C và D, chỉ có hai chất có khả năng phản ứng thế với chất 1 (theo
điều kiện ở đầu bài) để tạo thành chất 2, trong khi hai chất còn lại thì chỉ cho duy nhất sản
phẩm tách 3. Hãy xác định hai chất cho phản ứng thế và giải thích vì sao hai chất còn lại
không thể tham gia phản ứng thế.
J. d) Khi thực hiện phản ứng từ 1 sang 2 với tác nhân nucleophile phù hợp, nếu sử dụng acid
trifluoroacetic (CF3COOH) và acid acetic (CH3COOH) thay cho acid dichloroacetic, không
có acid nào cho sản phẩm thế mong muốn. Hãy giải thích hiện tượng trên.
K.
L. HƯỚNG DẪN CHẤM
Câu ý Nội dung Điểm
1 1 a) 0,5

Trang 15 / 23
Cacbanion của (B) chỉ có 1 nhóm cacbonyl nên mật độ điện tích âm được giải tỏa
ít nhất làm cho cacbanion kém bền nhất.
Hiệu ứng +C của nhóm N(CH3)2 > OCH3 làm cho mật độ electron của O trong
nhóm C=O bên phải của (A) giàu hơn (D) làm giảm hiệu ứng +C của cacbanion
vào C=O bên phải dẫn đến cacbanion (A) kém bền hơn so với (D).
 Tính axit của hiđro linh động: (C) > (D) > (A) > (B)
b)
Ta có: ΔHcháy = ∑Elk (chất đầu) - ∑Elk (sản phẩm). Sản phẩm thu được giống nhau, do
vậy chất đầu càng kém bền thì ∑Elk (chất đầu) càng nhỏ dẫn đến ΔHcháy càng âm, khi
đó │ΔHcháy│càng lớn. Như vậy hợp chất càng kém bền thì │ΔHcháy│càng lớn.
So sánh độ bền: A2 > A1 > A3 > A4 > A5  │ΔHcháy│: A2 < A1 < A3 < A4 < A5. 0,25
Nguyên nhân:

sức căng Bayer (sức căng góc) nhỏ, sức căng Pitzer (liên kết xen kẽ) nhỏ

sức căng Bayer (sức căng góc) nhỏ, sức căng Pitzer (liên kết xen kẽ) nhỏ, những
các sức căng này lớn hơn so với A2

0,25
sức căng Bayer lớn, sức căng Pitzer (liên kết che khuất) lớn. Hai nguyên tử H ở
dạng cis gần nhau nên lực đẩy nhỏ hơn so với A4.

sức căng Bayer lớn, sức căng Pitzer (liên kết che khuất) lớn. Nguyên tử H ở gần
nhóm Me nên lực đẩy lớn hơn so với A3.

sức căng Bayer rất lớn, sức căng Pitzer (liên kết che khuất hoàn toàn) rất lớn.
2 a)
M. Đồng phân cis bền hơn do có hiệu ứng siêu liên hợp bù trừ cho tương tác
không gian: 0,25

b) 0,25

N.
Cấu dạng gache bền hơn do có tương tác giữa nN với σ*N-H hiệu quả hơn σN-H

Trang 16 / 23
với σ*N-H

3 O. A > C > B > D P. 0


,
2
5
Q. Trong môi trường acid, chất 1 có khả năng tạo thành carbocation thơm S. 0
(14 electron π) bền vững nên sự ra đi của OH xảy ra rất nhanh. ,
2
5

R.

T. Hai nucleophile có khả năng tạo sản phẩm 2 là: B và C . V. 0


U. Chất A có tính base quá mạnh nên trong môi trường acid bị proton hóa ,
gần như hoàn toàn. Chất D có tính nucleophile quá yếu do cặp electron 2
trên nitrogen liên hợp vào nhóm C=Onên sự tách proton diễn ra nhanh 5
hơn. Chỉ có chất B và C có tính nucleophile mạnh và tính base vừa
phải để phản ứng với carbocation thơm.
W. TFA có tính acid quá mạnh nên nucleophile bị proton hóa gần như X. 0
hoàn toàn, không thể tham gia phản ứng. Acid acetic quá yếu nên ,
không thể proton hóa được chất 1 để tạo thành carbocation . 2
5

Câu 6. (2,5 điểm) Sơ đồ tổng hợp hữu cơ. Cơ chế phản ứng hóa hữu cơ.
1. Hãy đề nghị cơ chế giải thích sự hình thành sản phẩm sau:

b)

Trang 17 / 23
2. Lithospermic acid được Johnson và cộng sự phân lập từ một loại thảo dược Trung Hoa với
nhiều đặc tính sinh học quan trọng. Sơ đồ dưới đây biểu diễn một phần chuỗi tổng hợp toàn phần
Lithospermic acid.

Y. a. Vẽ cấu trúc của chất số 2, 3, 4, 5 và 7 biết chất số 7 có phân tử khối M = 30 gam/mol.

Z. b. Chọn đáp án đúng cho điều kiện của chuyển hóa số 8:


AA. Ce(NH4)2(NO3)6
BB. CrO3.(Pyridine)2
CC. H2CrO4/H2O
DD. CH3COCH3, Al[OiPr]3
EE. c. Vẽ cấu trúc của chất số 10.
FF.
GG. HƯỚNG DẪN CHẤM
Trang 18 / 23
Câu ý Nội dung Điểm
6 1 a)

0,5

b)
0,5

R-O-CS2Me xanthate=> tách Ei tạo anken hoặc chất tạp gốc


Khơi mào

Phát triển mạch:

2 a)

1,0

b) H2CrO4/H2O 0,25

Trang 19 / 23
c) 0,25

Câu 7. (2,5 điểm) Xác định cấu trúc các chất hữu cơ.
1. Cho hợp chất A (C6H8O2, chỉ có một loại hiđro) tác dụng với một đương lượng chất B (C2H6O2, có
2 loại hiđro với tỉ lệ 2:1) trong môi trường acid thì thu được chất C (C8H12O3). Xử lý C với một đương
lượng hợp chất Grignard D (RMgBr) thì thu được hợp chất E (C10H16O3, phổ 1H-NMR của E được
trình bày bên dưới). Đun nóng E trong hỗn hợp acetone/nước với xúc tác acid trong 2 tiếng thì thu
được sản phẩm F (C8H12O2).
1
H NMR (500 MHz, CDCl3) δ (E) 5.97 (dd, J = 17.4, 10.8 Hz, 1H), 5.28 (dd, J = 17.3, 1.4 Hz, 1H),
5.04 (dd, J = 10.7, 1.4 Hz, 1H), 3.99 – 3.88 (m, 4H), 1.95 (td, J = 12.6, 4.2 Hz, 2H), 1.77 (td, J = 13.0,
4.0 Hz, 2H), 1.69 – 1.62 (m, 2H), 1.62 – 1.54 (m, 2H).
a) Xác định công thức cấu tạo của các chất từ A đến F.
b) Một nhóm nghiên cứu ở trong trường ĐH X không có sẵn máy cộng hưởng từ hạt nhân (NMR) mà
chỉ có máy phổ hồng ngoại (IR). Bằng thiết bị nghiên cứu sẵn có, làm thế nào để biết phản ứng tạo
thành chất E đã xảy ra hoàn toàn (không còn nguyên liệu C trong hỗn hợp phản ứng)?
c) Cho biết số lượng tín hiệu proton mà chất F cho trên phổ 1H-NMR? Biểu diễn cấu trúc phân tử để
chỉ rõ các tín hiệu proton và giải thích.

2. Gemifloxacin (C18H20N5O4F) là một loại thuốc kháng sinh để chữa bệnh viêm
phế quản và được tổng hợp theo các giai đoạn sau:
HH. - Giai đoạn 1: Thủy phân X trong dung dịch H2SO4, thu được 47.
Cho 47 phản ứng với SOCl2 và sau đó với NaCH(COOEt)2, thu được
48. Đun 48 trong dung dịch TsOH ở 140oC, thu được 49. Xử lí 49 với
HC(OEt)3 trong Ac2O, tạo ra 50. Cho 50 phản ứng với xiclopropyl
amin trong EtOH, thu được 51 (C14H13N2O3Cl2F). Thực hiện phản ứng đóng vòng nội phân
tử với 51 trong t-BuOK và đioxan, thu được 52 (C12H8N2O3ClF).
II. - Giai đoạn 2: Cho etyl 2-aminoetanoat phản ứng với acrilonitrin có mặt KOH trong nước,
thu được 53 (C7H12N2O2). Bảo vệ nhóm amin bằng cách cho 53 tác dụng với Boc2O trong
CHCl3, thu được 54. Khi có mặt EtONa trong EtOH, 54 sẽ ngưng tụ nội phân tử để tạo ra
55. Khử 55 bằng lượng dư LiAlH4, tạo ra 56. Tiếp tục bảo vệ nhóm amin mới tạo thành
bằng cách cho 56 tác dụng với Boc2O trong hỗn hợp đioxan và nước, thu được 57. Oxi hóa
57 bằng PCC rồi cho tác dụng tiếp với MeONH2.HCl có mặt NaHCO3 trong EtOH và THF,
thu được 58. Xử lí 58 với AcCl trong MeOH để gỡ bỏ các nhóm bảo vệ, thu được 59
(C6H13N3O).
JJ. - Giai đoạn 3: Cho 52 phản ứng với 59 trong CH3CN và có mặt DBU, sẽ thu được
gemifloxacin.
Biết trong phân tử gemifloxacin có một nhóm amino và một nhóm cacboxyl. Vẽ công thức cấu tạo của
các chất từ 47 đến 59 và gemifloxacin.
KK.
LL.
MM.
NN.
Trang 20 / 23
OO. HƯỚNG DẪN CHẤM
Câu 7 Nội dung Điểm
1

0,5

PP.Sử dụng phổ hồng ngoại, ta quan sát thấy: 0,25


- Trên hợp chất C có tín hiệu mạnh ở vùng 1700 – 1650 cm-1 của nhóm carbonyl.
- Trên hợp chất E có tín hiệu mạnh ở vùng khoảng 1600 cm-1 của olefin hoặc tín
hiệu tù và rộng từ 3500 – 3200 cm-1 của nhóm OH.
Nếu trên phổ hồng ngoại không còn quan sát thấy tín hiệu của nhóm carbonyl nữa
mà xuất hiện tín hiệu của alkene hay OH thì chứng tỏ phản ứng đã xảy ra hoàn
toàn.
QQ. Chất F có 8 tín hiệu proton. (Cho nếu học sinh trả lời 6 – 7 tín hiệu 0,125
proton)

RR. Giải thích: trong cấu dạng bền nhất của F, các hydrogen ở vị trí trục sẽ cho
tín hiệu khác các hydrogen ở vị trí biên.
2 1,625

Bài 8. (2,5 điểm) Hóa học các hợp chất thiên nhiên
Trang 21 / 23
1. D-andotetrozơ A khi phản ứng với nitric acid cho hợp chất không hoạt động quang học. Cũng
andotetrozơ này khi phản ứng với HCN, tiếp theo với dung dịch nước Ba(OH)2 cho hai axit andonic
epime B và C. Các andonic acid này nằm trong cân bằng với các -andolacton D và E tương ứng của
chúng. Xử lý hỗn hợp này với Na - Hg và nước ở pH 3-5 thu được các chất F và G tương ứng. Oxi hóa
F bằng nitric acid thu được andaric acid không hoạt động quang học H, trong khi thực hiên phản ứng
này với E thu được andaric acid hoạt động quang học I. Cho biết cấu trúc các chất từ A đến I.
3. Tiến hành cắt mạch một nonapeptit bằng enzyme dipeptidylamine peptidase (cắt tuần tự 2
aminoaxit tính từ đầu N) thu được hỗn hợp các dipeptit Gly – Phe; Leu – Arg; Tyr – Gly;
Lys – Tyr và Pro. Còn nếu cắt mạch nonapeptit bằng enzyme dipeptidylcarboxipeptidase (cắt tuần tự 2
aminoaxit tính từ đầu C) được hỗn hợp các dipeptit: Phe – Leu; Gly – Gly; Arg – Lys; Tyr – Pro và
Tyr.
a. Xác định trật tự các aminoaxit trong nonapeptit trên.
b. Trong số các dipeptit tạo thành thì dipeptit nào không quang hoạt? Dipeptit nào có điểm đẳng điện
lớn hơn 7? Và dipeptit nào khó bị thủy phân cho ra aminoaxit nhất?
c. Một decapeptit có hoạt tính tương tự aminoaxit đang xét có thể được tổng hợp bằng cách ghép thêm
Lys vào sau Pro. Hãy đề nghị phương pháp tổng hợp peptit này từ nonapeptit ban đầu. Yêu cầu không
quá 5 giai đoạn.
d. Tiến hành cắt mạch decapeptit mới tổng hợp bằng dipeptidylamine peptidase và dipeptidylcarboxi
peptidase sẽ thu được tổng cộng bao nhiêu dipeptit?
SS. HƯỚNG DẪN CHẤM
Câu ý Nội dung Điểm
8 1 Chỉ D-andotetrozơ dạng erythro mới cho sản phẩm không hoạt động quang
học khi bị oxi hóa bởi axit nitric :
CHO COOH 0,25
H OH + HNO 3 H OH
H OH H OH
CH 2OH COOH
(A)
Xử lí A bằng HCN, tiếp theo với dung dịch nước Ba(OH)2 cho hai axit
andonic epime B và C :
COOH COOH
CHO H OH HO H
H OH 1.HCN H OH H OH
+
H OH 2. Ba(OH) 2 H OH H OH 0,25
CH 2OH CH 2OH CH 2OH
(A) (B) (C)
Các -lacton D và E tương ứng là :
COOH CO COOH CO
H OH H OH HO H HO H
O O
H OH H OH H OH H OH
H OH H H OH H
CH 2OH CH 2OH CH 2OH CH 2OH 0,25
(B) (D) (C) (E)
Xử lý hỗn hợp này với Na - Hg và nước ở pH 3-5 thu được các chất F và G.
Oxi hóa bằng axit nitric thu được axit andaric không hoạt động quang học H
và hoạt động quang học I :
CHO COOH CHO COOH
Na(Hg) H OH H OH Na(Hg) HO H HNO 3 HO H
HNO 3
(D) H OH H OH (E) H OH H OH
pH=3-5 pH=3-5
H OH H OH H OH H OH 0,5
CH 2OH COOH CH 2OH COOH
(F) (H) (G) (I)

Trang 22 / 23
2 a. Trật tự sắp xếp peptit: Tyr-Gly-Gly-Phe-Leu-Arg-Lys-Tyr-Pro 0,25

TT. b. 0,5
- Gly-Gly sẽ là aminoaxit không quang hoạt
- Leu-Arg; Lys-Tyr và Arg-Lys sẽ có pHI > 7 do có nhiều nhóm NH2 hơn
COOH
- Tyr-Pro sẽ khó bị cắt về aminoaxit nhất, do nhóm amit trong dipeptit này
có bậc ba khó bị thủy phân hơn.
c. 0,25
Có thể tổng hợp theo cách sau đây:

UU.
d. 0,25
Cắt từ đầu N hay từ phía đầu C đều thu được 5 dipeptit đồng nhất bao gồm: Tyr-
Gly; Gly-Phe; Leu-Arg; Lys-Tyr và Pro-Lys.

Trang 23 / 23
KỲ THI CHỌN HỌC SINH GIỎI CÁC TRƯỜNG THPT CHUYÊN
KHU VỰC DUYÊN HẢI VÀ ĐỒNG BẰNG BẮC BỘ
LẦN THỨ XIV, NĂM 2023

ĐÁP ÁN ĐỀ ĐỀ XUẤT MÔN: HÓA HỌC - LỚP 11


(Đáp án gồm 15 trang)

Câu 1: Tốc độ phản ứng.


1. Cho phản ứng thuận nghịch bậc 1 – 1 giữa A1 và A2 như sau:
⎯⎯⎯ →
k1
A1(dd) ⎯⎯
k−1
⎯ A2(dd) có k1 = 0,4 s−1 và hằng số cân bằng K = 8.
Ban đầu, nồng độ của A2 bằng 0. Hãy xác định thời gian phản ứng để nồng độ A2 bằng nồng độ A1.
2. Chuyển hoá không thuận nghịch của chất B thành chất B1 trong dung dịch (chỉ chứa các bước nối tiếp)
được nghiên cứu bằng phổ UV–vis ở bước sóng λ1 = 320 nm. Đầu tiên, một dung dịch chất B có nồng độ
0,00131 M được chuẩn bị. Sự phụ thuộc của độ hấp phụ (A) của dung dịch vào thời gian (t) được đưa ra trong
bảng dưới:
t, phút 0 2 4 6 10 20 50 100 ∞
A 0,982 2,073 2,538 2,727 2,883 2,713 2,369 1,919 0,681
a) Số bước chuyển hoá liên tiếp ít nhất có thể có trong chuyển hoá từ B sang B1 là bao nhiêu?
b) Tính độ hấp thụ riêng (ghi rõ đơn vị) của chất B ở bước sóng đã nhắc tới.
c) Một chuỗi dung dịch chuẩn của sản phẩm cuối (B1) được nghiên cứu bằng phổ UV–vis ở bước sóng 320
nm trong một cuvette với độ dày 1 cm. Tính độ hấp thụ riêng của B1 sử dụng các số liệu bên dưới:
C(B1), M 0,00100 0,00200 0,00300 0,00500
A 0,56 1,08 1,60 2,64
d) Viết sơ đồ chuyển hoá B thành B1 ứng với số bước đã đề xuất ở ý a (tất cả các giai đoạn của chuyển hoá
đều là phản ứng bậc 1), sử dụng kết quả của ý c.
e) Tính độ hấp thụ quang riêng của ở bước sóng 320 nm của các trung gian đã đề xuất.
f) Ước lượng tất cả các hằng số tốc độ trong sơ đồ đã đề xuất.
g) Chất đầu và chất cuối có độ hấp thụ riêng bằng nhau ở bước sóng λ2 = 710 nm, và không có chất trung gian
nào hấp thụ ánh sáng. Biểu diễn sự hấp thụ giữa độ hấp thụ quang của dung dịch được đo ở bước sóng này
vào thời gian (có chứa điểm t = ∞).
ĐÁP ÁN
1. Đặt a = [A1]0, b = [A2]0, x(t) = [A1]t. Ta có:
k 0, 4
k−1 = 1 = = 0, 05s −1
K 8
a
Tại thời điểm t: x( t ) =
2
k1 b + x x k1a
K= = = = x =
k−1 a − x a − x k1 + k−1
x 2k
ln  =
1 1 1 = 1 0.8
t= ln −1
ln = t = 1.84s
k1 + k−1 x − x k + k k −k 0, 45s 0.35
 1 −1 1 −1
2. a) Do sự phụ thuộc của mật độ quang vào thời gian không ứng với sự giảm nồng độ chất đầu, nên chắc
chắn phải có ít nhất 2 bước liên tiếp.

Trang 1 / 15
b) Ở thời điểm t = 0, dung dịch chỉ có chất B nên độ hấp thụ quang riêng εB = 1500 M–1.cm–1.
c) εB1 = 520 M–1.cm–1.
d) Tính toán nồng độ B1 tại thời điểm cuối: [B1]t=∞ = 0,00262 M, gấp đôi nồng độ chất B ban đầu, nên
một trong hai giai đoạn sinh ra hai phân tử, do đó sơ đồ có thể được biểu diễn như sau:
k1 k2 k1 k2
B→ B’ → 2B1 (sơ đồ 1) hoặc B → 2B’ → 2B1 (sơ đồ 2)
e) Có thể thấy được độ hấp thụ quang đạt cực đại ở thời gian tương đối nhanh so với cả quá trình, do đó
có thể giả sử rằng nồng độ B’ tại thời điểm cực đại (t = 10 phút) là bằng với nồng độ chất B ban đầu, ứng với:
ε B’ = 2.883 / 0.00131 / 0.5 = 4400 M –1 .cm –1 (sơ đồ 1) hoặc
ε B’ = 2.883 / 0.00262 / 0.5 = 2200 M –1 .cm –1 (sơ đồ 2).
f) Trong sơ đồ đề xuất, chỉ có một chất trung gian, do đó cần xác định 2 hằng số tốc độ k1 và k2. Khi sản
phẩm cuối chưa được hình thành nhiều, độ hấp thụ quang được biểu diễn qua phương trình sau:
A = C 0 ( B )  exp ( – k1t )   B’ + C 0 ( B )  1 – exp ( – k1t )    B
Trong sơ đồ B → 2B’, thì hợp phần thứ 2 của biểu thức sẽ được nhân đôi. Ở thời gian t = 2 phút và
εB’ = 4400 M–1.cm–1, k1 = 0,43 phút–1. Giá trị này là như nhau đối với cả 2 sơ đồ.
Để tính k2, ta cần sử dụng thành phần của hỗn hợp tại thời điểm t lớn (giả sử B đã phản ứng hết), và tỉ lệ
C(B’, t1)/C(B’, t2) = exp[k2(t2 – t1)]. Bằng cách sử dụng hai điểm thời gian lớn nhất (50 và 100 phút), thu được
k2 = 0,0062 phút–1.
g) Mối quan hệ giữa độ hấp thụ quang của dung dịch theo log(t):

Câu 2: Cân bằng và phản ứng trong dung dịch. Pin điện – Điện phân
1. Dung dịch C gồm MgCl2 0,10 M; AlCl3 0,10 M; Zn(CH3CO2) 0,10 M. Thêm dần dần dung dịch NH3 vào
dung dịch C tới khi C(NH3) = 0,10, thu được dung dịch C1 (thể tích dung dịch không đổi sau khi cho thêm
NH3).
a) Tính pH của dung dịch C.
b) Tính pH của dung dịch C1, cho biết hiện tượng xảy ra trong dung dịch C1? Giải thích bằng tính toán.
Cho biết: pKa(CH3CO2H) = 4,76; pKa(NH4+) = 9,24; *β(AlOH2+) = 10–4,30; *β(MgOH+) = 10–12,80; *β(ZnOH+)
= 10–8,96; β(Zn(NH3)42+) = 108,89; pKs(Al(OH)3) = 32,40; pKs(Zn(OH)2) = 15,52; pKs(Mg(OH)2) = 9,20; pKw =
14,00

Trang 2 / 15
2. Pin sau được cấu tạo từ điện cực hydrogen tiêu chuẩn (SHE) và điện cực kim loại M nhúng trong dung dịch
ion của nó (Mn+): SHE | | Mn+ (x M) | M
Thực nghiệm cho thấy suất điện động của pin, Epin, phụ thuộc vào [Mn+] như sau:
x, M Epin, V
0,005 0,932
0,500 0,991
Nếu thêm vào pin này NH3 thì pin tạo thành: SHE | | M (0,001 M), NH3 (y M) | M, có suất điện động phụ
n+

thuộc vào [NH3]. Kết quả được biểu diễn trên đồ thị ở hình 1. Biết rằng Mn+ phản ứng với NH3 theo cân bằng:
 M ( NH 3 ) np+ 
M n+ ⎯⎯
→ M ( NH 3 ) p
+ p NH 3 ⎯

n+
 p = ( n+
 M   NH 3 
p

Epin/ V

Hình 1: Mối liên hệ giữa Epin và log(y). Với C(NH3) = 1,0 M thì Epin = 0,310 V; với C(NH3) = 0,10 M thì
Epin = 0,430 V.
a) Xác định các chỉ số n và p.
b) Tính nồng độ cân bằng của các cấu tử có trong 1 L dung dịch chứa 0,001 mol Mn+ và 1,05 mol NH3.
Cho biết: pKa(NH4+) = 9,24. Bỏ qua quá trình tạo phức hydroxo của Mn+.
ĐÁP ÁN

1. Để Al(OH)3 kết tủa: [Al ].[OH ] = Ks(Al(OH)3), ứng với:
3+ 3

h K
C 0 ( Al 3+ ). * .( w )3 = 10−32,4 = h = 2.771.10−4 M ; ứng với pH = 3,56
1 + h h,
Tương tự như vậy, Mg(OH)2 bắt đầu kết tủa khi:
0 2+ h Kw 2 −9,20 −10
C ( Mg ). * .( ) = 10 = h = 1.259.10 M
3 + h h ; ứng với pH = 9,90

Và Zn(OH)2 bắt đầu kết tủa khi: ứng với pH = 6,74


0 2+ h Kw 2 −15.52 −7
C ( Zn ). * .( ) = 10 = h = 1.820.10 M
2 + h h ; ứng với pH = 6,74

a) Các cân bằng trong dung dịch:


(1) Al3+ + H2O ⇌ AlOH2+ + H+ *β1 = 10–4,30
(2) Zn 2+ + H2O ⇌ ZnOH+ + H+ *β2 = 10–8,96
(3) Mg 2+ + H2O ⇌ MgOH+ + H+ *β3 = 10–12,80

(4) CH3CO2 + H2O ⇌ CH3CO2H + OH– Kb = 10–9,24
(5) H2O ⇌ H+ + OH– Kw = 10–14,00

Trang 3 / 15
Do *β1.C0(Al3+) >> *β2.C0(Zn2+) >> *β3.C0(Mg2+) > Kw; C0(CH3CO2–).Kb >> Kw, ta có thể bỏ qua ảnh
hưởng của cân bằng (2), (3) và (5) khi tính toán pH.
Có biểu thức điều kiện proton (h = [H+]):
*
0 3+ 1 0 − h
h = C ( Al ). * − C ( CH 3 CO2 ).
1 + h h + Ka

Giải phương trình thu được h = 1,18.10–5 M, ứng với pH = 4,93. Ở pH này xảy ra sự kết tủa Al3+.
Các cân bằng trong dung dịch:
(1) Al3+ + H2O ⇌ AlOH2+ + H+ *β1 = 10–4,30
(2) Al3+ + 3H2O ⇌ Al(OH)3 + 3H+ K = 10–9,7
(2) Zn2+ + H2O ⇌ ZnOH+ + H+ *β2 = 10–8,96
(3) Mg2+ + H2O ⇌ MgOH+ + H+ *β3 = 10–12,80
(4) CH3CO2– + H2O ⇌ CH3CO2H + OH– Kb = 10–9,24
(5) H2O ⇌ H+ + OH– Kw = 10–14,00
Do Al3+ còn dư, ta có thể giả sử bỏ qua được cân bằng phân ly acid (2), (3). Có biểu thức điều kiện
proton:
K s .h2 .* 1 0 − h K s . h3
h = − C ( CH 3 CO2 ). + 3.(0, 1 − )
K w3 h + Ka K w3
Giải phương trình thu được h = 1,446.10–4 M; ứng với pH = 3,84. Giả sử bỏ qua các cân bằng phân ly
acid còn lại là hợp lý.
b) – Để Al(OH)3 kết tủa: [Al3+].[OH–]3 = Ks(Al(OH)3), ứng với:
h K
C 0 ( Al 3+ ). * .( w )3 = 10−32,4 = h = 2.771.10−4 M
1 + h h,
Tương tự như vậy, Mg(OH)2 bắt đầu kết tủa khi:
0 2+ h Kw 2 −9,20 −10
C ( Mg ). * .( ) = 10 = h = 1.259.10 M
3 + h h

Và Zn(OH)2 bắt đầu kết tủa khi:


0 2+ h Kw 2 −15.52 −7
C ( Zn ). * .( ) = 10 = h = 1.820.10 M
2 + h h

– Tại thời điểm chưa có kết tủa: với C0(NH3) = 0,10 M, phản ứng sau xảy ra hoàn toàn:
NH3 + Al3+ + H2O → AlOH2+ + NH4+ K = 104,94 >> 2
thành phần giới hạn của dung dịch bao gồm: NH4+, Zn2+, AlOH2+, Mg2+ 0,10 M; CH3CO2– 0,20 M.
Các cân bằng trong dung dịch:
(1) Zn2+ + H2O ⇌ ZnOH+ + H+ *β2 = 10–8,96
(2) Mg2+ + H2O ⇌ MgOH+ + H+ *β3 = 10–12,80
(3) NH4+ ⇌ NH3 + H+ Ka = 10–9,24
(4) CH3CO2– + H2O ⇌ CH3CO2H + OH– Kb1 = 10–9,24
(5) AlOH2+ ⇌ Al3+ + OH– Kb2 = 10–9,70
(6) H2O ⇌ H+ + OH– Kw = 10–14,00
Do *β1.C0(Zn2+) >> *β2.C0(Mg2+) > Kw, C0(CH3CO2–).Kb1 ≈ C0(AlOH2+) >> Kw, nên ta có thể bỏ qua
ảnh hưởng của cân bằng (2) và (6) khi tính toán pH.

Trang 4 / 15
Có biểu thức điều kiện proton:
0 2+
*
2 0 + Ka 0 − h 0 2+ h
h = C ( Zn ). + C ( NH 4 ). − C ( CH 3 CO2 ). − C ( AlOH ).
*
2 + h h + Ka h + K a1 h + Ka 2
Giải phương trình thu được h = 7,157.10–6 M; ứng với pH = 5,14. Ở pH này, chỉ diễn ra sự kết tủa
Al(OH)3. Do đó khi thêm NH3 vào dung dịch thì chỉ có sự kết tủa NH3 xảy ra theo phản ứng:
Al3+ + 3NH3 + 3H2O → Al(OH)3 + 3NH4+ K = 1018,12
Do đó, NH3 phản ứng hết và vẫn còn dư Al3+. Các cân bằng phân ly trong dung dịch lúc này:
(1) Al3+ + H2O ⇌ AlOH2+ + H+ *β1 = 10–4,30
(2) Zn2+ + H2O ⇌ ZnOH+ + H+ *β2 = 10–8,96
(3) Mg2+ + H2O ⇌ MgOH+ + H+ *β3 = 10–12,80
(4) NH4+ ⇌ NH3 + H+ Ka = 10–9,24
(5) CH3CO2– + H2O ⇌ CH3CO2H + OH– Kb = 10–9,24
(6) Al(OH)3 ⇌ Al3+ + 3OH– Ks = 10–32,40
(7) H2O ⇌ H+ + OH– Kw = 10–14,00
Do Al3+ còn dư, ta có thể giả sử bỏ qua được cân bằng phân ly acid của (2), (3), (4). Có biểu thức điều
kiện proton:
K s .h2 .* 1 0 − h K s . h3
h = − C ( CH 3 CO2 ). + 3.(0, 1 − )
K w3 h + Ka K w3
Giải phương trình thu được h = 1,308.10–4 M; ứng với pH = 3,88. Giả sử bỏ qua các cân bằng phân ly
acid phụ là hợp lý.
Hiện tượng xảy ra: dung dịch xuất hiện nhiều kết tủa Al(OH)3 trắng keo hơn.
2. a) – Xác định n: Có: Epin = Ec – Ea. Nồng độ Mn+ tăng thì Epin tăng, do đó điện cực M là điện cực dương.
Từ đây suy ra được: Epin = E0pin + 0,0592.log[Mn+]/n.
Thiết lập phương trình tuyến tính, thu được n = 2; E0pin = 1,00.
– Xác định p: Do nồng độ NH3 là rất lớn so với M2+, ta có thể coi coi [M(NH3)p2+] = 0,001 M và NH3 không
phân ly base đáng kể so với chính nó.
Từ đây suy ra được:
Epin = E0pin + 0,0592.log(1/βp)/2 + 0,0592.log([M(NH3)p2+]/[NH3]p])/2 = const – 0,0592.p.log[NH3]/2
Thiết lập phương trình tuyến tính, thu được p = 4.
b) Từ chênh lệch giữa 2 giá trị Epin khi đã thêm NH3, tính được E0pin + 0,0592.log(1/β4)/2 = 0,31. Từ đây
có thể tính được β4 = 1023,31.
Hằng số tạo phức β4 = 1023,31 rất lớn, do đó tính được [M(NH3)42+] = 0,001 M.
Với định luật tác dụng khối lượng, tính được [NH4+] = [OH–] = (Kb.C(NH3))1/2 = 4,271.10–3 M.
Từ đó tính được [NH3] = 1,00 M; [M2+] = 4,898.10–27 M; [H+] = 2,341.10–12 M.

Câu 3: Nhiệt động học và cân bằng hóa học.


Đối với phản ứng hoá học tổng hợp NH3 ở 450oC và 300 atm, ở trạng thái cân bằng người ta thu được số mol
của N2, H2, NH3 lần lượt là 10 mol, 30 mol, 12 mol. Ở áp suất cố định 300 atm người ta tăng nhiệt độ lên đến
455oC. Biết ∆rHo = -46 kJ.mol-1. Giả sử hiệu ứng nhiệt của phản ứng không phụ thuộc vào nhiệt độ.
1. Xác định thành phần của hệ ở trạng thái cân bằng tại nhiệt độ 455oC.
2. Cân bằng dịch chuyển theo chiều nào? Có phù hợp với nguyên lí dịch chuyển Le Chatelier không?
3. Xác định chiều chuyển dịch cân bằng của phản ứng khi:

Trang 5 / 15
a) Thêm vào bình khí N2.
b) Thêm vào bình khí He và giữ nguyên thể tích của bình.
c) Tăng áp suất của bình phản ứng bằng cách nén hỗn hợp.
ĐÁP ÁN
1. Áp suất của NH3, N2, H2 lần lượt là: 69,23 atm; 57,69 atm; 173,1 atm.
→ Ở 450oC: Kp1 = 1,6.10-5
→ Ở 455oC: Kp2 = 1,52.10-5
Gọi x là áp suất do phần N2 phản ứng gây ra.
Lập phương trình và giải hệ ta được x = -0,615 atm → cân bằng dịch chuyển theo chiều tạo ra thêm N2 hay là
chiều nghịch → áp suất riêng phần của N2 ở trạng thái cân bằng mới là: 58,305 atm.
→ Gọi a là số mol của N2 được tạo ra thêm ta có:
Tổng số mol của N2 và hệ lúc cân bằng là: 10 + a và 52 + a.
→ Từ tỉ lệ về số mol và áp suất của N2 trong hệ ta có: a = 0,16 mol
Vậy: Số mol của N2, H2, NH3 ở trạng thái cân bằng tại nhiệt độ lần lượt là: 10,16 mol; 30,48 mol; 11,68 mol.
2. Kết quả phù hợp với nguyên lí chuyển dịch cân bằng (khi tăng nhiệt độ cân bằng chuyển dịch theo chiều
của phản ứng thu nhiệt tức là chiều nghịch – chiều thuận toả nhiệt → chiều nghịch thu nhiệt).
3. a) Thêm vào khí N2 thì nồng độ khí N2 tăng → cân bằng chuyển dịch theo hướng làm giảm nồng độ N2 →
cân bằng chuyển dịch theo chiều thuận.
b) Thêm vào bình khí He là khí trơ nên không tham gia vào phản ứng với H 2, N2, NH3; thể tích của bình giữ
không đổi nên áp suất riêng phần của từng khí không đổi → cân bằng không chuyển dịch.
c) Tăng áp suất của bình bằng cách nén hỗn hợp → cân bằng chuyển dịch theo chiều giảm áp suất tức là chiều
làm giảm số mol khí → cân bằng chuyển dịch theo chiều thuận.

Câu 4: Hóa nguyên tố (Kim loại, phi kim nhóm IVA, VA). Phức chất.
1. Xét sơ đồ chuyển hoá của kim loại D thành các hợp chất của nó dưới đây:

Biết rằng tỉ lệ phần trăm khối lượng trong D1 và D4 là 1,511. Xác định kim loại D công thức của các hợp chất
từ D1 đến D8, viết phương trình phản ứng xảy ra.
2. Oxygen được dẫn qua một dung dịch cobalt(II) chloride có chứa acid hydrochloric và ethylenediamine.
Dung dịch này sau đó được làm lạnh, dẫn tới sự kết tinh của tinh thể hình kim E1 (%mCo = 15,47%). Đun tinh
thể này trong bình hút ẩm thu được E2 cùng với sự giảm khối lượng chất rắn là 9,44%. Từ dung dịch đầu có
thể phân lập được một lượng nhỏ tinh thể E’ (%mCo = 15,45%), dung dịch chất này có môi trường acid yếu.
Xử lý E’ trong kiềm trong một thời gian dài thì thu được E2.
a) Xác định công thức cấu tạo của E1, E’ và E2. Viết phương trình cho phản ứng tạo thành E1.

Trang 6 / 15
b) Xác định năng lượng CSFE (năng lượng làm bền trường tinh thể) trong cation E1 và cation tứ diện E3
chứa ít một phân tử phối tử hơn E1. Từ đó rút ra kết luận về tính ổn định nhiệt động và động học của cả
hai phức chất.
Cho biết:
Năng lượng ghép cặp, P (cm−1): Co3+: 21000; Co2+: 22500;
Hệ số f (hệ số đánh giá độ mạnh của trường mà phối tử tạo ra, hệ số f của H2O là 1,00) của en: 1,28;
Hệ số g (∆o ): Co3+: 18200; Co2+: 9000.
c) Sử dụng dữ liệu có sẵn, chứng minh rằng phức cobalt (+2) trong dung dịch nước có spin cao.
d) Bên dưới là phổ hấp phụ màu của E1. Dự đoán màu của chất này.

Cho biết: Màu sắc của ánh sáng tương ứng với mỗi bước sóng trong vùng khả kiến: tím: 400 – 450 nm;
lam: 430 – 480 nm; lục: 480 – 560 nm; vàng: 560 – 590 nm; cam: 590 – 630 nm; đỏ: 630 – 750 nm.
ĐÁP ÁN
1. N2O4(l) là tác nhân nitrate hoá muối chloride, do đó D là kim loại có 2 số oxi hoá, trong đó sự oxi hoá
bằng HNO3 cho sản phẩm có số oxi hoá thấp. Có thể giả sử D có 2 số oxi hoá là +2 và +4, ứng với D1 là
D(NO3)2 và D4 là D(NO3)4. Từ tỉ lệ phần trăm D trong 2 chất, thu được:
M ( D ) + 62.4
= 1.511 ⇒ M(D) = 118,66 (Sn)
M ( D ) + 62.2

Từ đó suy ra được công thức các chất:


D D1 D2 D3 D4
Sn Sn(NO3)2 SnO2 SnCl4 Sn(NO3)4
D5 D6 D7 D8
SnCl2 H2[SnCl6] Na2[Sn(OH)6] Sn(OH)4
Phương trình phản ứng:
3Sn + 8HNO3 → 3Sn(NO3)2 + 4H2O + 2NO
Sn(NO3)2 → SnO2 + 2NO2
Sn + 2Cl2 → SnCl4
SnCl4 + 4N2O4 → Sn(NO3)4 + 4NOCl
Sn(NO3)4 → SnO2 + 4NO2 + O2
Sn + 2HCl → SnCl2 + H2
2SnCl2 + 8HCl + O2 → 2H2[SnCl6] + 2H2O
SnO2 + 6HCl → H2[SnCl6] + 2H2O
SnO2 + 2NaOH + 2H2O → Na2[Sn(OH)6]
Na2[Sn(OH)6] + 2CO2 → 2NaHCO3 + Sn(OH)4
2. a) Giả sử E1 là phức chất đơn nhân, có nghĩa là M(E1) = 381,5 g.mol-1. Đun nóng E1 tạo thành E2 kèm
theo sự giảm khối lượng là 9,44%, ứng với sự giảm 36 g.mol-1 = 2 phân tử H2O. Do đó có thể kết luận:
M(D1) = M(Co) + 3M(Cl) + 3M(en) + 2M(H2O), E1 là [Co(en)3]Cl3.2H2O, D2 là [Co(en)3]Cl3.

Trang 7 / 15
Tính được M(E’) = 382 g.mol-1 = M(E2) + M(HCl), do đó D’ là dạng proton hoá 1 lần của E2. Do đó
phức E’ là [Co(en)2(H2NCH2CH2NH3)Cl]Cl3.
Phương trình phản ứng: 4CoCl2 + O2 + 4HCl + 12en + 6H2O → 4[Co(en)3]Cl3.2H2O
b) en có hệ số f lớn hơn 1 nên có thể coi f là phối tử trường mạnh, từ đó cho biết cấu hình electron trong
trường bát diện của Co3+, ion kim loại trung tâm trong E1: (t2g)6 (eg)0.
Từ đó tính được CSFE của E1: CSFE = -6.0,4.∆o + 3P = 19320 cm-1
Đối với cation tứ diện E3, [Co(en)2]3+, cấu hình electron là (eg)4 (t2g)2.
Từ đó tính được CSFE của E3: CSFE = (-4.0,6 + 2.0,6) ∆o + 2P = 20160 cm-1
c) Trong dung dịch, cấu hình electron của phức Co(+2) spin thấp là: (t2g)6 (eg)1
Tính được CSFE = (-6.0,4 + 0,6).∆o + 3P = 34740 cm-1
Đối với phức Co(+2) spin cao: (t2g)5 (eg)2
Tính được CSFE = (-5.0,4 + 0,6.2).∆o + 2P = 30440 cm-1
CSFE của phức spin cao âm hơn phức spin thấp nên phức spin cao trong dung dịch bền hơn.
d) E1 hấp phụ ánh sáng lam – lục, do đó sẽ có màu cam.

Câu 5: Đại cương hữu cơ.


1. Gán các giá trị pKa đo được trong dung môi DMSO: 9,3; 13,0; 15,6; 18,5; 21,2 (không theo thứ tự) cho các
hợp chất muối phosphonium F1 – F5 ở hình bên dưới.

2. Giải thích tại sao tốc độ của phản ứng (1) nhanh gấp 8000 phản ứng (2):

3. Dưới đây là một phản ứng cộng nucleophile – acyl hoá một ketene sinh ra sản phẩm F6 và F7:

Tỉ lệ sản phẩm F6 : F7 = 3 : 1. Tại sao F6 lại sinh ra nhiều hơn?


4. Các hợp chất F8 và F9 có thể chuyển hoá qua lại với nhau trong một thời gian dài ở 60oC theo cân bằng ở
hình bên:

F8 F9
a) Đề xuất cơ chế cho chuyển hoá này, biết rằng chuyển hoá không thông qua cơ chế liên phân tử.

Trang 8 / 15
b) Tại cân bằng, tỉ lệ [F9] : [F8] > 20 : 1. Giải thích.
ĐÁP ÁN
1. Các giá trị pKa ứng với từng chất:
Chất F1 F2 F3 F4 F5
pKa 9,3 21,2 13,0 18,5 15,6
2. Trong 2 trạng thái chuyển tiếp C hoá trị V của phản ứng SN2 của 2 phản ứng, trạng thái chuyển tiếp của
phản ứng (1) có sự liên hợp với vòng phenyl, trong khi phản ứng (2) không có nên phản ứng (1) thuận lợi về
mặt động học hơn rất nhiều so với phản ứng (2), làm cho tốc độ phản ứng (1) nhanh hơn rất nhiều so với phản
ứng (2).
3. – Ở chất phản ứng ketene ban đầu, xuất hiện tương tác đẩy giữa nhóm phenyl và nhóm mesityl, thêm vào
đó còn xuất hiện tương tác đẩy giữa nhóm C=O và nhóm mesitylene, do đó nhóm mesitylene phải quay ra
khỏi mặt phẳng liên hợp.
– Do tấn công từ góc Burgi–Dunitz (tác nhân nucleophile và nhóm C=O tạo thành một góc 107o) vào orbital
π* của nhóm C=O, MesMgBr, một tác nhân nucleophile cồng kềnh sẽ ưu tiên tấn công ketene vào bên nào ít
bị cản trở không gian hơn, vốn là bên có nhóm Mes do nhóm Mes đã bị lệch ra khỏi mặt phẳng liên hợp. Sau
khi tiến hành acyl hoá các enolate tạo thành, sản phẩm F6 sẽ được tạo thành nhiều hơn do ứng với sản phẩm
cộng Nu cùng nhóm Mes.
4. a) Cơ chế đề xuất:

b) Sự xem phủ của cặp electron không liên kết của oxygen vào orbital σ* của liên kết C4-C3 là một tương tác
nội phân tử làm bền hoá phân tử:

Câu 6: Sơ đồ tổng hợp hữu cơ. Cơ chế phản ứng hóa hữu cơ.
1. Đề xuất cơ chế cho các phản ứng dưới đây:

Trang 9 / 15
a) b)

c)

2. Melodinine E là một alkaloid, được tìm thấy trong loài Melodinus cochinchinensis. Hợp chất này được tổng
hợp vào năm 2019 theo sơ đồ sau:

Xác định công thức cấu tạo của các chất từ G1 đến G11.
3. Các carbenoid và vòng nhỏ được sử dụng rộng rãi dưới dạng các trung gian quý giá trong tổng hợp hữu
cơ. Một ví dụ của việc ứng dụng lớp chất này vào việc tổng hợp một alkaloid tự nhiên được chiết xuất từ loài
nấm thuộc chi Asperigillus được đưa ra dưới đây:

Trang 10 / 15
Vẽ công thức cấu tạo của các chất từ G12 đến G14 và công thức cấu trúc của các chất từ G15 đến G21.
ĐÁP ÁN
1. Cơ chế đề xuất:
a)

b)

Trang 11 / 15
c)

2. Cấu tạo các chất:

Trang 12 / 15
3. Cấu tạo và cấu trúc các chất:

Câu 7: Xác định cấu trúc các chất hữu cơ (mô tả sơ đồ tổng hợp bằng lời dẫn).
1. Cho phloroglucinol (1,3,5-trihydroxylbenzene) phản ứng với chloroacetonitrile (ClCH 2CN) khi có mặt
ZnCl2 thu được chất H1 (C8H8NO3Cl). Sục khí HCl vào dung dịch chứa chất H1 rồi đun hồi lưu thu được chất
H2. Đun hỗn hợp gồm H2, methyl iodide và K2CO3 trong DMF ở 80oC. Chiết sản phẩm bằng ethyl acetate
rồi chưng cất loại dung môi thu được chất H3. Vừa khuấy vừa đun nhẹ ở 60oC trong 30 phút hỗn hợp gồm H3
và 3,4-dimethoxybenzaldehyde trong methanol có cho thêm KOH thu được chất H4 (C19H18O6).
a) Vẽ công thức cấu tạo các chất từ H1 đến H4. Đề xuất cơ chế phản ứng tạo thành H1 và H2.
b) Xử lý H4 bằng H2O2 thu được chất H5 (C19H18O7). Khử chất H5 bằng H2/Pd-C thu được chất H6
(C19H20O7). Xử lý chất H6 bằng K2CO3 thu được flavonoid H7. Vẽ công thức cấu tạo các chất từ H5 đến H7.
2. Cho aniline và ethyl acetoacetate vào bình phản ứng, thêm vài giọt HCl đặc rồi lắc mạnh 15 phút. Đặt bình
phản ứng vào bình hút ẩm trong vòng 2 – 3 ngày nhận được hợp chất I1. Đun nóng I1 trong dung môi thích
hợp ở nhiệt độ cao thu được hợp chất I2. Đun nóng I2 trong POCl3 ở 70oC, sau đó trung hoà bằng NaOH thu
được hợp chất I3. Cho I3 phản ứng với NaN3 với sự có mặt của một vài tinh thể KI trong dung môi DMF khan
ở 70oC thu được hợp chất I4. Ở một thí nghiệm khác, làm lạnh hỗn hợp chứa -D-glucopyranose pentaacetate
với propacgyl alcohol (HC≡CCH2OH) trong dung môi DCM về 0oC rồi nhỏ từ từ BF3.Et2O vào hỗn hợp phản

Trang 13 / 15
ứng. Sau khi phản ứng kết thức thu được hợp chất I5. Thực hiện phản ứng “Click” chứa I4 và I5 với sự có
mặt của Cu(I) sẽ thu được hợp chất I6. Vẽ công thức cấu tạo các chất từ I1 đến I6.
ĐÁP ÁN
1. a) Cấu tạo các chất:

b) Cơ chế đề xuất:

c) Cấu tạo các chất:

2. Cấu tạo các chất:

Câu 8: Hóa học các hợp chất thiên nhiên (Carbohydrate và các hợp chất hữu cơ chứa nitrogen đơn giản).
1. Cho α-D-glucopyranose phản ứng với hỗn hợp Ac2O/HBr thu được chất K1. Chất này tiếp tục phản ứng với
triethylamine cho chất K2. K2 dưới ảnh hưởng của dung dịch nước Ba(OH)2 thu được chất K3 (C6H10O5).
Chất K3 không có khả năng phản ứng với thuốc thử Fehling và không tham gia vào quá trình lên men rượu. 1

Trang 14 / 15
mol K2 phản ứng được với 2 mol HIO4 cho ra K4 và 1 mol HCO2H. Oxi hóa K4 bằng Br2 đầu tiên cho ra sản
phẩm K5, oxi hóa tiếp chất này thu được hỗn hợp đẳng số mol của H2C2O4 và acid CHOH(CO2H)2. Hãy xác
định cấu trúc các chất từ K1 đến K5.
2. Nấu ăn ở nhà lẫn trong công nghiệp có thể tạo thành các chất có khả năng tổn hại con người. Những chất
này có thể được tạo thành ở nhiệt độ cao, môi trường pH khắc nghiệt và từ các hợp chất không có mặt trong
thiên nhiên (ví dụ như các chất bảo quản).
Một α-amino acid L1 không có mặt trong tự nhiên chứa 40,44% carbon; 7,92% hydrogen; 15,72% nitrogen
và 35,92% oxygen về khối lượng được tạo thành từ amino acid L2 có trong tự nhiên, ví dụ từ quá trình hạt
đậu nành chuyển hoá thành sản phẩm thuỷ phân của nó.
a) Vẽ công thức cấu trúc của L1 và L2, nếu như biết rằng phản ứng tạo thành L1 chỉ sử dụng L2.
L1 là một chất phản ứng trong sự tổng hợp sinh hoá của dipeptide L3 (chứa 26,1% C; 52,2% H và 13,0% O
về số mol). L3 được tìm trong thành tế bào peptidoglycan dưới vai trò là một cặn cuối, và vô cùng quan trọng
đối với sự bền của vi khuẩn. Thuỷ phân L3 bằng kiềm trong môi trường khắc nghiệt dẫn tới sự hình thành sản
phẩm duy nhất là một hợp chất không hoạt động quang học, L4.
b) Vẽ công thức cấu trúc của L3 và L4.
ĐÁP ÁN
1. Dựa vào công thức phân tử chất K3 có thể thấy rằng nó là sản phẩm dehydrate hóa của glucose. Rất có khả
năng đây là sự tạo thành một liên kết ether từ hai nhóm alcohol. Việc chất K3 không có khả năng lên men
cũng như cho kết quả âm tính với thuốc thử Fehling cho thấy nhóm hydroxyl ở K4 đã tham gia phản ứng loại
nước này. Các dữ kiện oxy hóa tiếp theo bằng HIO4 và dung dịch Br2 cho thấy rằng chất K4 là 1,2,3-triol, tức
nhóm OH tham gia phản ứng ete hóa với OH ở C1 là OH ở C6. Vậy K3 chính là 1,6-anhydro-β-D-pyranozơ.
Như vậy sơ đồ phản ứng sẽ như sau:

2. a) Trong amino acid L1: C : H : N : O = 3 : 7 : 1 : 2. L1 là đồng phân alanine không có trong tự nhiên,
D-alanine, còn L2 là L-alanine.
b) Từ % số mol, tính được trong L3: C : H : N : O = 6 : 12 : 2 : 3, ứng với sự ngưng tụ của 2 phân tử Ala, do
đó dipeptide L3 bao gồm 2 mắt xích Ala.
L4 không hoạt động quang học, do đó L4 là hỗn hợp racemix của Ala, ứng với dipeptide L3 được hình thành
từ L-alanine và D-alanine, do đó L3 có thể có 2 cấu tạo:

Trang 15 / 15
HỘI CÁC TRƯỜNG THPT CHUYÊN KỲ THI CHỌN HỌC SINH GIỎI
VÙNG DH&ĐB BẮC BỘ LẦN THỨ XIV, NĂM 2023
HƯỚNG DẪN CHẤM MÔN: HÓA HỌC - LỚP 11

HƯỚNG DẪN CHẤM

(Hướng dẫn chấm gồm 16 trang)


Câu 1 (2,5 điểm).
1.1. Chuyên Nguyễn Tất Thành – Yên Bái + Chuyên Vĩnh Phúc.
Với phản ứng: 2 + O2 → 2 (1)
[ ]
Biểu thức tốc độ toàn phần có thể được biểu diễn như sau:
[ ]
Nếu oxi dư, biểu thức tốc độ có thể được biểu diễn lại như sau:
Trong đó: k’ = k.[O2]b
Tiến hành đo nồng độ sunfit [ ] theo thời gian t và nhận được ba đồ thị sau:
[ ] -t 1/[ ] -t ln[ ] -t

(Đồ thị 1) (Đồ thị 2) (Đồ thị 3)


a) Xác định bậc phản ứng theo sunfit.
Tiến hành đo k’ theo các nồng độ oxi khác nhau và thu được kết quả như sau:
[O2] 212,0 390,7 652,2 979,2
k’ 741,3 955,0 1230,3 1584,9
b) Xác định bậc riêng phần theo O2.
Hướng dẫn chấm
Câu 1 Nội dung Điểm
2-
1.1 a) Trong ba đồ thị, chỉ có đường thứ ba, (ln[SO3 ] – t), có dạng tuyến tính. Có 0,25
2-
nghĩa phản ứng (1) có bậc một theo SO3 .
b) Biết k’ = k.[O2]b. Lấy logarit cả hai vế được: lnk’ = lnk + b.ln[O2] (*)
Đồ thị (*) là đường phụ thuộc của lnk’ vào ln[O2]. 0,25
Giá trị b là hệ số góc của đồ thị được tính theo công thức:
Tính lnk’, ln[O2]. Lập được bảng:
[O2]. 212,0 390,7 652,2 979,2
k’ 741,3 955,0 1230,3 1584,9
ln[O2]. 5,357 5,968 6,480 6,887 0,25
lnk’ 6,608 6,862 7,115 7,368
Chọn hai cặp giá trị bất kì ở bảng trên từ đó tính được các giá trị b, sau khi tính
trung bình thu được ̅ = 0,5 0,25

Hướng dẫn chấm môn Hóa học 11 –HSG DH & ĐB BB Lần XIV năm 2023 Trang 1/16
1.2. Chuyên Biên Hòa – Hà Nam
Phản ứng: 2Fe2+ + Tl3+ → 2Fe3+ + Tl+.
k[Fe2 ]2 [Tl 3 ]
có phương trình định luật tốc độ phản ứng dạng: r = . Hãy dự đoán cơ chế của phản ứng.
[Fe2 ]  k '[Fe3 ]
Hướng dẫn chấm
1.2 Có thể có 2 trường hợp giới hạn:
k[Fe2 ]2 [Tl 3 ]
TH (1): [Fe2+]<< k’[Fe3+] → r =
k '[Fe3 ]
TH (2): [Fe2+]>> k’[Fe3+] → r = k[Fe2+ ][Tl3+ ]
Trong trường hợp (1), thành phần tổng cộng các chất có mặt trong giai đoạn sơ cấp
trung gian là (FeTl)4+. Trong trường hợp (2):(FeTl)5+ . Cơ chế có thể là:
k1
0,25
Fe2+ + Tl3+ k1
Fe3+ + Tl2+
Fe2+ + Tl2+  k2
 Fe3+ + Tl+
Theo cơ chế: r = k2[Fe2+ ][Tl2+] (1)
2
d [Tl ]
 k1[Fe2 ][Tl3 ]-k 1[Fe3 ][Tl2 ]-k 2[Fe 2 ][Tl 2 ] =0 (2)
dt
k1[Fe2 ][Tl3 ]
→[Tl2+] = (3)
k1[Fe3 ]+k 2 [Fe2 ]
k1[Fe 2 ]2 [Tl3 ]
Thay (3) vào (1): r = 0,25
k1 3 2
[Fe ]+[Fe ]
k2
Cơ chế được đề nghị là có khả năng.
1.3. Chuyên Lê Quý Đôn - Điện Biên
Đối với phản ứng: A + B → C + D có biểu thức tốc độ phản ứng v = k. [A].[B]
Trộn 2 thể tích bằng nhau của dung dịch chất A và dung dịch chất B có cùng nồng độ 1,0 M:
- Nếu thực hiện phản ứng ở nhiệt độ 300 K thì sau 2 giờ nồng độ của C bằng 0,215 M. Tính hằng số tốc độ
của phản ứng.
- Nếu thực hiện phản ứng ở 370 K thì sau 1,33 giờ nồng độ của A giảm đi 2 lần. Tính năng lượng hoạt hóa
của phản ứng (theo kJ/mol). Cho R = 8,314 J/mol.K
Hướng dẫn chấm
1.3 Theo đề: v = k. [A].[B] nên phản ứng bậc 2.
1, 0
a) CA = CB = a =  0,5 M
2
Nồng độ đầu 2 chất phản ứng bằng nhau nên phương trình động học:
1 1 1
k= (  )
t ax a 0,25
1 1 1
Tại T1 = 300K: k1 = (  )  0,7544 (mol‒1.lít.giờ‒1) 0,25
2 0,5  0, 215 0,5
1 1 1 0,25
Tại T2 = 370K: k 2 = (  )  1,5037 (mol‒1.lít.giờ‒1)
1,33 0,5  0, 25 0,5
k2 Ea 1 1
Phương trình Arrhenius: ln  (  )
k1 R T1 T2
1,5037 E 1 1
 ln  a (  ) 0,25
0,7544 8,314 300 370
 Ea = 9093,55 (J/mol)

Hướng dẫn chấm môn Hóa học 11 –HSG DH & ĐB BB Lần XIV năm 2023 Trang 2/16
Câu 2 (2,5 điểm). Cân bằng và phản ứng trong dung dịch. Pin điện – Điện phân.
2.1. Chuyên Hoàng Lê Kha –Tây Ninh
Hóa học xanh (Green chemistry) luôn hướng tới các quá trình sản xuất sạch hơn, giảm thiểu ô nhiễm môi
trường, tách loại, thu hồi, tái sử dụng các chất thải. Dưới đây là một ví dụ:
Để tách loại các kim loại nặng Cr(VI), Ni(II) từ nước thải mạ điện, người ta tiến hành khử Cr(VI) về Cr(III)
bằng FeSO4 trong môi trường axit, sau đó dùng kiềm để kết tủa các hiđroxit Cr(OH)3, Ni(OH)2, Fe(OH)3 tại
các pH thích hợp nhằm thu hồi, tái sử dụng lại hiđroxit của các kim loại này.
Giả thiết nồng độ ban đầu của các ion Cr(VI) và Ni(II) trong nước thải đều bằng 10-3M; lượng FeSO4 lấy
vừa đủ để khử Cr(VI) về Cr(III) (coi thể tích dung dịch nước thải không đổi). Hãy xác định các giá trị pH
sau đây đối với từng hiđroxit kim loại:
- pHbđ của dung dịch khi bắt đầu xuất hiện kết tủa hiđroxit kim loại.
- pHht của dung dịch khi kết tủa hoàn toàn hiđroxit kim loại. (Các hiđroxit kim loại được xem như kết tủa
hoàn toàn khi nồng độ ion kim loại còn lại trong dung dịch nhỏ hơn hoặc bằng 10-6M).
Cho tích số tan Ks của Fe(OH)3, Cr(OH)3, Ni(OH)2 lần lượt bằng 10-38, 10-30, 10-15. KW = 10-14
Hướng dẫn chấm
Câu 2 Đáp án Thang điểm
3+
2.1 - Đối với Fe :
K s,Fe(OH)3
Để bắt đầu kết tủa Fe(OH)3 thì COH  3  1, 4938.1012 M
CFe3
0,25
pHbđ = 2,17
K s,Fe(OH)3
Khi Fe(OH)3 kết tủa hoàn toàn thì COH  3
6
 2,154.1011 M
10
pHht = 3,33 0,25
- Đối với Cr3+:
K s,Cr(OH)3
Để bắt đầu kết tủa Cr(OH)3 thì COH  3  109 M
CCr3
0,25
pHbđ = 5,00
K s,Cr(OH)3
Khi Cr(OH)3 kết tủa hoàn toàn thì COH  3
6
 108 M
10
pHht = 6,00 0,25
- Đối với Ni2+:
K s,Ni(OH)2
Để bắt đầu kết tủa Ni(OH)2 thì COH   106 M
C Ni2
pHbđ = 8,00 0,25
K s,Ni(OH)2
Khi Ni(OH)2 kết tủa hoàn toàn thì COH   3,162.105 M
106
pHht = 9,50 0,25

Hướng dẫn chấm môn Hóa học 11 –HSG DH & ĐB BB Lần XIV năm 2023 Trang 3/16
2.2. Chuyên Hùng Vương – Phú Thọ
Ăcqui chì được nhà hoá học Pháp Louis Gaston Plante phát minh vào năm 1859. Ở trạng thái hoạt động,
điện cực bên trái của ăcqui (với thế điện cực âm hơn) tạo thành từ chì kim loại, còn điện cực bên phải là chì
(IV) oxit. Chất điện li là dung dịch H2SO4 có nồng độ phần trăm 20-30%.
a) Thiết lập sơ đồ pin đơn giản nhất ứng với acqui chì.
Cho các thế điện cực chuẩn: EoPbSO 2  0,359V ; EoPbO 
,SO24  /PbSO4
 1,682V và E oPb2  /Pb  0,126 V.
4 ,SO4 /Pb 2 ,H

b) Viết phương trình hoá học cho phản ứng tổng cộng xảy ra trong pin và tính suất điện động chuẩn của pin
điện hoá.
c) Tính năng lượng Gibbs chuẩn của phản ứng tổng cộng và hằng số cân bằng của phản ứng tại 25 oC.
d) Tại mặt phân cách “Pbchất điện li”, xảy ra tương tác hoá học tạo ra chì (II) sunfat. Viết phương trình
hoá học cho phản ứng xảy ra.
Hướng dẫn chấm
2.2. a Sơ đồ pin: (-) Pb, PbSO4H2SO4PbO2, Pb (+) (1/4 đ)
b Pb + PbO2 + 2H2SO4  2PbSO4 + 2H2O
Epino = Eco – Eao = 1,682 – (-0,359) = 2,041 V (1/4 đ)
c Gpin = -zFEpin = -2964852,041 = -393,852.10 J
o o 3

 G o 393,852.103
Ke e  1,092.1069
pin
8,314 298
(1/4 đ)
RT

d Pb + H2SO4  PbSO4 + H2 (1/4 đ)

Hướng dẫn chấm môn Hóa học 11 –HSG DH & ĐB BB Lần XIV năm 2023 Trang 4/16
Câu 3 (2,5 điểm). Nhiệt động học và cân bằng hóa học
3.1. Chuyên Cao Bằng
Xét phản ứng: TiO2 (s) + 2C (graphite,s) + 2Cl2 (g) → 2CO (g) + TiCl4 (l).
Có ∆rHo (298K) = -80,01 kJ.mol-1. Cho biết các dữ kiện sau ở 25oC:
Chất TiO2 (s) Cl2 (g) C (graphite,s) CO (g) TiCl4 (l)
o -1
∆fH (kJ.mol ) -945 0 0 -110,5 ?
-1 -1
CP,m (J.K .mol ) 55,06 33,91 8,53 29,12 145,2
o o
a) Tính ∆rH ở 135,8 C.
b) Tính ∆fHo của TiCl4 (l) ở 25oC.
Giả sử các giá trị nhiệt dung riêng không phụ thuộc vào nhiệt độ.
Hướng dẫn chấm
Câu Ý Nội Dung Điểm
o
3 3.1 a) Đổi nhiệt độ 135,8 C thành 408,8K
408,8 K
 r H (408,8K )   r H (298K )   CP,m dT
0 0

298 K

 r H 0 (408,8K )   r H 0 (298K )  [CP,m ( TiCl4 , l)  2CP,m (CO, g )  CP,m ( TiO 2 , s)


2CP,m (graphite, s)  2CP,m (Cl2 , g )].(408,8  298) 0,5
= -80,01.103 + (145,2+2.29,12–55,06–2.8,53–2.33,91).(408,8-298)
=-72974,2 (J.mol-1) hay -72,974 (kJ.mol-1)
b) TiO2 (s) + 2C (graphite,s) + 2Cl2 (g) → 2CO (g) + TiCl4 (l).
 r H 0  2 r H 0 (CO, g )   r H 0 ( TiCl4 , l)   r H 0 ( TiO2 , s)  2 r H 0 (graphite, s)
2 r H 0 (Cl2 , g )]
  r H 0 ( TiCl4 , l)   r H 0   r H 0 ( TiO2 , s)  2 r H 0 (graphite, s)
0,5
2 r H 0 (Cl2 , g )]  2 r H 0 (CO, g )
 r H 0 (TiCl4 , l)  80,01  945  2.110,5 = -804,01 (kJ.mol1 )

3.2. Chuyên Lê Quý Đôn – Đà Nẵng


3.2. Li2S6 là dạng polysulfua được nghiên cứu nhiều
nhất, phản ứng hình thành chất này như sau:
2Li+ + S8 + 2e  Li2S6 + 2S
Li2S6 tồn tại hai dạng cấu trúc: (I) và (II), sự phân ly
của Li2S6 trong dung môi điện phân DME (1,2-
dimethoxyethane) được mô tả ở sơ đồ bên (gồm cân
bằng của các dạng Li 2S6 , LiS6 , S62 và LiS*3
trong DME)
Năng lượng Gibbs phân ly (kJ.mol-1) của các quá
trình trong DME (25oC và 1 bar):
Gdo1 ( I ) Gdo1 ( II ) Gdo 2 Gdro ( I ) Gdro ( II )
20,68 18,92 100,55 45,13 43,37
a) Tính tỷ lệ nồng độ Li2S6 tương ứng với hai dạng
cấu trúc (I) và (II).
b) Tính hằng số phân ly biểu kiến của cân bằng:
Li 2S6 Li   LiS6
c) Sắp xếp nồng độ cân bằng của các cấu tử Li 2S6 , LiS6 , S62 và LiS*3 theo thứ tự giảm dần.

Hướng dẫn chấm môn Hóa học 11 –HSG DH & ĐB BB Lần XIV năm 2023 Trang 5/16
Hướng dẫn chấm
3.2. a. Xét quá trình: (I) (II),  r G o  G d1
o
(I)  G d1
o
(II)  1, 76 kJ.mol 1 0,25
[Li 2S6(II ) ]  8,314
1,76.10 3

 K  e r G e  0, 4915.
o
/RT 298 0,25
a) Ta có:
[Li 2S6(I ) ]
b. Xét các cân bằng:
3
   8,314
20,68.10

(I ) Li   LiS6 K1  [Li [(][LiS


I )]
6]
e 298
 2,371.104
0,25
3
[Li  ][LiS6 ]  8,314
18,92.10
  4
(II) Li  LiS 6 K2  [( II )] e 298
 4,825.10
 
[( II )]
1
K1  K12  [Li[(][LiS
I )]
 
]
[( II )]
[Li  ][LiS ]
 [([LiI )] ][LiS   K  [( I )][( II )]  K  K
]
[Li ][LiS6 ] K1K 2
1 2
6 6 6

   
Vậy, cân bằng: Li 2S6 Li  LiS6 ,

K  [Li[Li][LiS
2S6 ]
6]
 [([LiI )]][LiS 6]
[( II )]
4 4 0,25
2,371.10 4,825.10
 K  2,371.104
 4,825.104
 1,59.104
c. Xét các cân bằng:
3
 8,314
45,13.10

(I ) 2LiS3 K1  e 298


 1, 228.108
3
0,25
 8,314
43,37.10

(II) 2LiS3 K2  e 298


 2, 498.10 8

8 8
 Li 2S6 2LiS3 , có K  1,228.10
1,228.10  2,498.10
8
 2,498.108
 8, 233.109
3
 100,55.10
Xét cân bằng: LiS6 Li   S62 , có K  e 8,314298
 2,369.1018
Viết lại: Li 2S6 Li   LiS6 , K  1,59.104 0,25
Vì K K K [Li 2S6 ]  [LiS ]  [LiS3 ]  [S ].

6
2
6

Hướng dẫn chấm môn Hóa học 11 –HSG DH & ĐB BB Lần XIV năm 2023 Trang 6/16
Câu 4 (2,5 điểm). Hóa nguyên tố. Phức chất
4.1. Chuyên Nguyễn Trãi – Hải Dương
Một nguyên tố X có khả năng phản ứng với canxi cho chất Y. Mặt khác X tan được trong dung dịch kiềm
tạo ra một hợp chất A và khí B đều có chứa nguyên tố X. A phản ứng với clorua vôi thu được một kết tủa C.
Kết tủa này sẽ chuyển thành Y khi xử lý với nhôm ở nhiệt độ cao. Hòa tan chất Y trong dung dịch HCl
loãng thu được B. Biết rằng khi xử lý C với SiO2 và than cốc thu được X, còn trong trường hợp không có
than cốc thu được D. D tan được trong cả dung dịch axit loãng và kiềm loãng.
a) Lập luận xác định cấu trúc các chất chưa biết và viết các phương trình phản ứng xảy ra.
b) Đơn chất X tồn tại một dạng thù hình kém bền với không khí và dễ thăng hoa. Vẽ cấu trúc dạng thù hình
này và giải thích tại sao nó lại kém bền với không khí?
Hướng dẫn chấm
4.1 a. Do X phản ứng được với Ca nên X phải là một phi kim. Trong dung dịch kiềm X
hòa tan sinh ra một muối tan và một khí. Nguyên tố X có mặt trong cả hai thành phần
ấy. Trong hợp chất khí tồn tại liên kết X – H. Như vậy chỉ có thể có ba khả năng là
silan, photphin và amoniac. X sinh ra khi cho than cốc tác dụng với muối C (có chứa
X) và SiO2 nên X chỉ có thể là photpho. 0,2
Các phản ứng xảy ra như sau: (Có thể viết dạng ion thu gọn)
P4 + 3NaOH + 3H2O  3NaH2PO2 + PH3
P4 + 6Ca  2Ca3P2
2NaH2PO2 + 4CaOCl2  Ca3(PO4)2 + CaCl2 + 2NaCl + 4HCl
2Ca3(PO4)2 + 10C + 6SiO2  6CaSiO3 + 10CO + P4
3Ca3(PO4)2 + 16Al  3Ca3P2 + 8Al2O3
9
Ca3P2 + 6HCl  3CaCl2 + 2PH3
pư.0,1
2Ca3(PO4)2 + 6SiO2  6CaSiO3 + P4O10
= 0,9
P4O10 + 6H2O  4H3PO4 (trong dung dịch axit loãng) điểm
P4O10 + 12NaOH  4Na3PO4 + 6H2O
(có thể viết phản ứng tạo muối axit).
b.

- Dạng thù hình kém bền với không khí của X là P4 (photpho trắng) dễ thăng hoa do 0,2
trong tinh thể các phân tử liên kết với nhau bằng liên kết Van der Walls giữa các phân
tử không cực.
- P4 phản ứng ngay với oxi khi tiếp xúc do liên kết trong phân tử rất kém bền, góc 0,2
liên kết PPP bị bẻ cong ép tử 900 về 600.
4.2. Chuyên Thái Bình
4.2. Trong đa số trường hợp, dạng hình học của các
phức bát diện không tương ứng với hình bát diện
đối xứng lí tưởng, mà sẽ bị biến dạng. Hiện tượng
này được giải thích bởi hiệu ứng Jahn-Teller, lí
thuyết này phát biểu rằng phân tử với các orbital có
năng lượng bằng nhau có xu hướng bị biến dạng
hình học và do đó năng lượng của phân tử sẽ giảm
khi cấu hình electron thay đổi. Trong hình bên, δ1
và δ2 là năng lượng tách mức biến dạng trong các
orbital của các nhóm t2g và t2g.
a) Trong hình bên, hãy gán các obitan d của kim
loại vào mỗi
Sự rút ngắn hoặc kéo dài liên kết theo trục z trong phức bát
Hướng dẫn chấm môn Hóa học 11 –HSG DH & ĐB BB Lần XIV năm 2023 Trang 7/16
giản đồ tương ứng.
b) Bằng việc tính năng lượng bền hoá khi biến dạng (so với khi chưa biến dạng), hãy cho biết các phức bát
diện nào sau đây có xu hướng biến dạng và cho biết sự rút ngắn hoặc kéo dài liên kết theo trục z xảy ra với
các phức này:
i) [CrCl6]4- (spin cao). ii) [Mn(CN)6]4− (spin thấp). iii) [Mn(H2O)6]2+ (spin cao).

Hướng dẫn chấm


4.2.
a)

0,25

b)
Nếu phức biến dạng theo trục z kiểu rút ngắn liên kết thì theo vế trái của giản đồ trên; còn nếu
kéo dãn thì theo nửa phải của giản đồ. Nên
*) [CrCl6]4- là phức spin cao có cấu hình e kiểu (t2g)3(eg)1 có
Năng lượng làm bền khi biến dạng rút ngắn là: E1 = 1.(-2 1 /3) + 2.(+ 1 /3) + 1.(-  2/2) = -  2/2 0,25
Năng lượng làm bền khi biến dạng kéo dãn là: E2 = 2.(- 1 /3) + 1.(+2.  1/3) + 1.(-  2/2) = -  /2.
Phức chất [CrCl6]4- có xu hướng bị biến dạng. Cả hai xu hướng kéo dài và rút ngắn đều có ưu
thế như nhau.
*) [Mn(CN)6]4- là phức spin thấp có cấu hình e kiểu (t2g)5 có
Năng lượng làm bền khi biến dạng rút ngắn là: E1 = 2.(-21/3) + 3.(+1/3) = -1/3
0,25
Năng lượng làm bền khi biến dạng kéo dãn là: E2 = 4.(-  1/3) + 1.(+2.  1/3) = -2  2/3.
Phức chất [Mn(CN)6]4-có xu hướng bị biến dạng kéo dãn vì thuận lợi hơn về mặt năng lượng.
*) [Mn(H2O)6]2+ là phức spin cao có cấu hình e kiểu (t2g)3(eg)2 có
Năng lượng làm bền khi biến dạng rút ngắn là:
E1 = 1.(-2  1/3) + 2.(+  1/3) + 1.(-  2/2) + 1.(  2/2) = 0
Năng lượng làm bền khi biến dạng kéo dãn là: 0,25
E2 = 2.(-  1/3) + 1.(+2.  1/3) + 1.(-  2/2) + 1.(  2/2) = 0 Phức chất [Mn(H2O)6]2+ không có xu
hướng bị biến dạng. Vì khi biến dạng năng lượng không thấp hơn ban đầu.

Hướng dẫn chấm môn Hóa học 11 –HSG DH & ĐB BB Lần XIV năm 2023 Trang 8/16
Câu 5 (2,5 điểm). Đại cương hóa hữu cơ
5.1. Chuyên Amsterdam – Hà Nội
Gán các giá trị pKa đo được trong dung môi DMSO: 9,3; 13,0; 15,6; 18,5; 21,2 (không theo thứ tự) cho các
hợp chất muối phosphonium F1 – F5 ở hình bên dưới.

5.2. Chuyên Amsterdam – Hà Nội


Giải thích tại sao tốc độ của phản ứng (1) nhanh gấp 8000 lần phản ứng (2)?

;
Hướng dẫn chấm
5.1 Các giá trị pKa ứng với từng chất: 0,5
Chất F1 F2 F3 F4 F5
pKa 9,3 21,2 13,0 18,5 15,6
5.2 Trong 2 trạng thái chuyển tiếp C hoá trị V của phản ứng SN2 của 2 phản ứng, trạng thái 0,5
chuyển tiếp của phản ứng (1) có sự liên hợp với vòng phenyl, trong khi phản ứng (2) không
có nên phản ứng (1) thuận lợi về mặt động học hơn rất nhiều so với phản ứng (2), làm cho tốc
độ phản ứng (1) nhanh hơn rất nhiều so với phản ứng (2).
5.3. Chuyên Lam Sơn Thanh Hóa
Cho cấu tạo của hợp chất hữu cơ E như hình bên.
Hãy chỉ rõ trạng thái lai hóa của từng nguyên tử N ở cấu tạo E và ghi giá trị
pKa (ở 25oC): 1,8; 6,0; 9,2 vào từng trung tâm axit trong công thức tương
ứng với E. Giải thích.

5.4. Chuyên Lam Sơn Thanh Hóa


Một số hợp chất 1,3-đicacbonyl có dạng enol như
hình bên:
a) So sánh tính axit của chất (1) và chất (2). (1) (2) (3)
b) Giải thích vì sao chất (3) cũng là este nhưng có
tính axit mạnh hơn chất (2) và mạnh hơn rất nhiều
(100000 lần) so với chất (1)?
Hướng dẫn chấm
Nội dung Điểm
5.3 sp 2 COOH
N
sp 3
NH 2
N sp 2
H (E)
- Nguyên tử N nhóm NH ở trạng thái lai hóa sp2, cặp e chưa chia ở obitan p xen phủ
với 5 obitan p khác tạo thành hệ thơm được lợi về mặt năng lượng nhưng “mất” tính 0,25
bazơ.
- Nguyên tử N thứ hai ở trạng thái lai hóa sp2, cặp e chưa chia ở obitan sp2 không tham
gia vào hệ thơm nên còn tính bazơ.
- Nguyên tử N nhóm NH2 ở trạng thái lai hóa sp3.

Hướng dẫn chấm môn Hóa học 11 –HSG DH & ĐB BB Lần XIV năm 2023 Trang 9/16
6,0 COOH 1,8
H N
NH 3 9,2
N
H 0,25
+ 3 +
- Nhóm NH3 là axit liên hợp của nhóm H2Nsp , nhóm NH là axit liên hợp của nhóm
Nsp2.
- Bazơ càng mạnh thì axit liên hợp càng yếu, vì thế giá trị 9,2 là thuộc nhóm NH3+ còn
giá trị 6,0 thì thuộc nhóm NH+.
5.4 a) Tính acid của 2 lớn hơn 1 bởi vì :
- Nhóm carbonyl của ester (chất 1) nhận điện tử từ nhóm OEt nên làm tăng mật độ
điện âm trên oxy , khiến cho liên kết hydro nội phân tử giữa OH và C=O bền vững
hơn.

- Dạng enolate của chất 1 kém bền hơn 2 do nhóm carbonyl cuả ester không làm bền 0,25
hiệu quả được điện tích âm trên oxy như chất 2 do LUMO của nó bị chiếm dụng bởi
đôi điện từ trên nhóm OEt , trong khi chất 2 không có hiện tượng này ( hiệu ứng liên
hợp chéo)
b) Do chất 3 không có khả năng nào để tạo thành liên kết hydro nội phân tử
- Oxy trong vòng lactone khó liên hợp hiệu quả vào nhóm C=O do làm tăng sức căng
vòng 5 cạnh . Điều này dẫn đến khả năng làm bền enolate của nhóm carbonyl được
củng cố mạnh (0.125đ )

- Do oxy trong vòng lactone không liên hợp tốt với nhóm carbonyl nên nó gây hiệu 0,25
ứng cảm –I là chủ yếu , làm tính acid của 3 nhỉnh hơn so với chất 2
5.5. Chuyên Lam Sơn Thanh Hóa
Vẽ giản đồ theo MO-  và chỉ rõ LUMO, HOMO trong phân tử buta-1,3-đien

0,5

Hướng dẫn chấm môn Hóa học 11 –HSG DH & ĐB BB Lần XIV năm 2023 Trang 10/16
Câu 6 (2,5 điểm). Sơ đồ tổng hợp hữu cơ. Cơ chế phản ứng hữu cơ.
6.1. Chuyên Bắc Ninh
Công thức của (+)-Grandisol (K) một chất dụ dẫn của loài bọ cái, sống trên cây bông
được cho như hình bên.
Hỗn hợp raxemic Grandisol được điều chế từ hept-5-ennitrin (A) làm chất đầu theo sơ
đồ sau: (K)

Xác định công thức cấu tạo các chất chưa biết (không cần biểu diễn hóa lập thể), viết cơ chế phản ứng của
chuyển hóa từ C→ D.
Hướng dẫn chấm
6.1 Nội dung Điểm
1 điểm/
10 chất
(mỗi
chất 0,1
điểm)

Khử C=O, Wolft-Kischner

Cơ chế phản ứng từ C tạo D 0,5

Hướng dẫn chấm môn Hóa học 11 –HSG DH & ĐB BB Lần XIV năm 2023 Trang 11/16
6.2. Chuyên Nguyễn Bỉnh Khiêm – Quảng Nam
Đề xuất cơ chế cho các phản ứng sau:

Hướng dẫn chấm


6.2. a.

0,25

b.

0,25

c.

0,5

Hướng dẫn chấm môn Hóa học 11 –HSG DH & ĐB BB Lần XIV năm 2023 Trang 12/16
Câu 7 (2,5 điểm). Xác định cấu trúc hợp chất hữu cơ (lời dẫn)
7.1. Chuyên Lương Văn Tụy – Ninh Bình. Chuyên Hưng Yên
Hiđrocacbon A (C6H10) không có đồng phân lập thể, 1 mol A chỉ làm mất màu 1 mol KMnO4 (trong nước)
hoặc 1 mol Br2 (trong CCl4) ở nhiệt độ thường. A phản ứng với lượng dư H2/xúc tác Ni tạo thành các hợp
chất là đồng phân cấu tạo của nhau có cùng công thức phân tử C6H14. Trong dung dịch axit H3PO4 50%, A
chuyển thành C (C6H12O) không làm mất màu dung dịch KMnO4 hoặc dung dịch Br2/CCl4 ở nhiệt độ
thường. Chế hóa C với CrO3/piriđin thu được D. Xử lý D với m-CPBA thu được 2 sản phẩm E1 và E2 là
đồng phân cấu tạo của nhau có cùng công thức phân tử C6H10O2, trong đó E1 là sản phẩm chính. Khử hóa
E1 và E2 bằng LiAlH4 thu được F1 và F2 có cùng công thức phân tử C6H14O2; F1 hoặc F2 phản ứng với
PCC hoặc C5H5N.SO3 thu được sản phẩm tương ứng X1 và X2, trong đó X1 có phản ứng iđofom. Xác định
công thức cấu tạo của các chất nêu trên.

Hướng dẫn chấm


Câu 7 Nội dung Điểm
7.1 Hidrocacbon A (C6H10, ∆= 2) không có đồng phân lập thể, 1 mol A chỉ làm mất
màu 1 mol KMnO4 (dung dịch) hoặc 1 mol Br2 (trong dung dịch CCl4) ở nhiệt độ Lập
thường. A phản ứng với lượng dư H2/xúc tác Ni tạo thành các hợp chất là đồng phân luận
cấu tạo của nhau có cùng CTPT C6H14(∆= 0) => A có 1 vòng 4 cạnh và có 1 liên kết 0,1
đôi. điểm
=> Các CTCT có thể có của A là:

A1
A2 A3 A4
Trong dung dịch axit H3PO4 50%, A chuyển thành C (C6H12O) không làm mất màu
dung dịch KMnO4 hoặc dung dịch Br2/CCl4 ở nhiệt độ thường. Chế hóa C với
CrO3/piridin thu được D. Nên:

O 9 chất:
OH
A: C: D: 0,9
điểm

O LiAlH4 OH PCC O
O
m-CPBA O OH
F1 O
E1 X1
OH O
D O
LiAlH4 PCC
O OH O
E2 F2
X2

Hướng dẫn chấm môn Hóa học 11 –HSG DH & ĐB BB Lần XIV năm 2023 Trang 13/16
7.2. Chuyên Chu Văn An - Hà Nội. Vùng Cao Việt Bắc
Hợp chất X (C10H18O) được phân lập từ một loại tinh dầu ở Việt Nam. X không làm mất màu nước brom và
dung dịch thuốc tím loãng, cũng không tác dụng với hiđro khi có xúc tác niken, nhưng lại tác dụng với axit
clohiđric đậm đặc sinh ra 1-clo-4-(1-clo-1-metyletyl)-1-metylxiclohexan.
a) Hãy đề xuất cấu trúc của X.
b) Hợp chất Y (C10H20O2) có trong một loại tinh dầu ở Nam Mỹ. Từ Y có thể tổng hợp được X bằng cách
đun nóng với axit.
- Viết công thức cấu tạo và gọi tên Y.
- Dùng công thức cấu trúc, viết sơ đồ phản ứng và trình bày cơ chế đầy đủ của phản ứng tổng hợp X từ Y.
Hướng dẫn chấm
Xác định cấu trúc của X(C10H18O):   2
Câu - X không làm mất mầu dung dịch nước brom và dung dịch thuốc tím loãng chứng tỏ trong
7.2 X không có nối đôi hay nối ba.
- X không tác dụng với hiđro trên chất xúc tác niken chứng tỏ trong X không có nhóm 0,25
chức cacbonyl.
- X tác dụng với axit clohiđric đậm đặc sinh ra 1-clo-4-(1-clo-1-metyletyl)-1-
metylxiclohexan, trong X có vòng no và có liên kết ete.
=> Suy ra cấu trúc của X:

0,25

a. - H O2 -H O
B+)B
(C(C
10H
HO
1020 202O
) 2) 2 A (CH10HO)
A (C10 18 18
O)

Suy ra Y là một điol có bộ khung cacbon như X


OH
H
+ 0,25
H2O XA

OH
BY

Gọi tên Y: 1-hiđroxi-4-(-1-hiđroxi-1-metyletyl)-1-metylxiclohexan 0,25


4-(1-hiđroxi-1-metyletyl)-1-methylxiclohexan-1-ol
+)
(+)
OH

OH
OH OH OH H
+

+ O
H
OH
0,25
(+)

Cis-B
Cis-Y H
+
A
X

(+)

OH
OH OH
OH OH + O
H
OH 0,25
(+)

Trans-B
Trans-Y H
+
XA

Hướng dẫn chấm môn Hóa học 11 –HSG DH & ĐB BB Lần XIV năm 2023 Trang 14/16
Câu 8 (2,5 điểm). Hóa học các hợp chất thiên nhiên
8.1. Chuyên Bắc Giang
D-anđotetrozơ A khi phản ứng với axit nitric cho hợp chất không hoạt động quang học. Khi A phản ứng với
HCN, tiếp theo với dung dịch nước Ba(OH)2 cho hai axit anđonic epime B và C. Các axit anđonic này nằm
trong cân bằng với các -anđolacton D và E tương ứng của chúng. Xử lý hỗn hợp này với Na - Hg và nước
ở pH 3-5 thu được các chất F và G tương ứng. Oxi hóa F bằng axit nitric thu được axit anđaric không hoạt
động quang học H, trong khi thực hiên phản ứng này với G thu được axit anđaric hoạt động quang học I.
Cho biết cấu trúc các chất từ A đến I.
Hướng dẫn chấm
Câu ý Nội dung Điểm
8 1 Chỉ D-andotetrozơ dạng erythro mới cho sản phẩm không hoạt động quang học
khi bị oxi hóa bởi axit nitric :
CHO COOH 0,25
H OH + HNO 3 H OH
H OH H OH
CH2OH COOH
(A)
Xử lí A bằng HCN, tiếp theo với dung dịch nước Ba(OH)2 cho hai axit andonic
epime B và C :
COOH COOH
CHO H OH HO H
H OH 1.HCN H OH H OH
H OH 2. Ba(OH)2 H OH
+ 0,25
H OH
CH2OH CH2OH CH2OH
(A) (B) (C)
Các -lacton D và E tương ứng là :
COOH CO COOH CO
H OH H OH HO H HO H
O O
H OH H OH H OH H OH
H OH H H OH H 0, 25
CH2OH CH2OH CH2OH CH2OH
(B) (D) (C) (E)

Xử lý hỗn hợp này với Na - Hg và nước ở pH 3-5 thu được các chất F và G. Oxi
hóa bằng axit nitric thu được axit andaric không hoạt động quang học H và hoạt
động quang học I :
CHO COOH CHO COOH
Na(Hg) H OH H OH Na(Hg) HO H HNO3 HO H
HNO3
(D) H OH H OH (E) H OH H OH 0,5
pH=3-5 pH=3-5
H OH H OH H OH H OH
CH2OH COOH CH2OH COOH
(F) (H) (G) (I)

Hướng dẫn chấm môn Hóa học 11 –HSG DH & ĐB BB Lần XIV năm 2023 Trang 15/16
8.2. Chuyên Trần Phú – Hải Phòng
Xác định các chất hữu cơ A, B, C, D, E, F, G, H, I, J trong sơ đồ chuyển hóa sau:
Biết rằng:
- [H] = khử hóa; [O] = oxi hóa; t = đun nóng
- C cũng là một amino axit có trong tự nhiên;
- D, E, F, J có hệ 3 vòng ngưng tụ;
- D và J là các đồng phân cấu tạo.

Hướng dẫn chấm


Không yêu cầu lập thể
8.2 1,25

………………………HẾT………………………

Hướng dẫn chấm môn Hóa học 11 –HSG DH & ĐB BB Lần XIV năm 2023 Trang 16/16

You might also like